You are on page 1of 383

z

B
E a

c
I

Y C X b A
s-b s-c s- a
Coordinate
Geometry Booster
with Problems & Solutions
for

JEE
Main and Advanced

CG_Prelims.indd 1 2/7/2017 2:33:46 PM


About the Author
REJAUL MAKSHUD (RM)
Post Graduated from Calcutta University in PURE MATHEMATICS.
Presently, he trains IIT Aspirants at RACE IIT Academy, Jamshedpur.

CG_Prelims.indd 2 2/7/2017 2:33:46 PM


Coordinate
Geometry Booster
with Problems & Solutions
for

JEE
Main and Advanced

Rejaul Makshud
M. Sc. (Calcutta University, Kolkata)

McGraw Hill Education (India) Private Limited


CHENNAI

McGraw Hill Education Offices


Chennai New York St Louis San Francisco Auckland Bogotá Caracas
Kuala Lumpur Lisbon London Madrid Mexico City Milan Montreal
San Juan Santiago Singapore Sydney Tokyo Toronto

CG_Prelims.indd 3 2/7/2017 2:33:48 PM


McGraw Hill Education (India) Private Limited

Published by McGraw Hill Education (India) Private Limited


444/1, Sri Ekambara Naicker Industrial Estate, Alapakkam, Porur, Chennai - 600 116

Coordinate Geometry Booster

Copyright © 2017, McGraw Hill Education (India) Private Limited.

No part of this publication may be reproduced or distributed in any form or by any means, electronic, mechanical,
photocopying, recording, or otherwise or stored in a database or retrieval system without the prior written permission
of the publishers. The program listings (if any) may be entered, stored and executed in a computer system, but they
may not be reproduced for publication.

This edition can be exported from India only by the publishers,


McGraw Hill Education (India) Private Limited

ISBN (13): 978-93-5260-509-5


ISBN (10): 93-5260-509-8

Information contained in this work has been obtained by McGraw Hill Education (India), from sources believed to be
reliable. However, neither McGraw Hill Education (India) nor its authors guarantee the accuracy or completeness
of any information published herein, and neither McGraw Hill Education (India) nor its authors shall be responsible
for any errors, omissions, or damages arising out of use of this information. This work is published with the
understanding that McGraw Hill Education (India) and its authors are supplying information but are not attempting
to render engineering or other professional services. If such services are required, the assistance of an appropriate
professional should be sought.

Typeset at The Composers, 260, C.A. Apt., Paschim Vihar, New Delhi 110 063 and text and cover printed at

Cover Designer: Creative Designer

visit us at: www.mheducation.co.in

CG_Prelims.indd 4 2/7/2017 2:33:48 PM


Dedicated to
My Parents

CG_Prelims.indd 5 2/7/2017 2:33:48 PM


CG_Prelims.indd 6 2/7/2017 2:33:48 PM
Preface

COORDINATE GEOMETRY BOOSTER with Problems & Solutions for JEE Main and Advanced is meant for aspirants
preparing for the entrance examination of different technical institutions, especially NIT/IIT/BITSAT/IISc. In writing this
book, I have drawn heavily from my long teaching experience at National Level Institutes. After many years of teaching, I have
realised the need of designing a book that will help the readers to build their base, improve their level of mathematical concepts
and enjoy the subject.
This book is designed keeping in view the new pattern of questions asked in JEE Main and Advanced Exams. It has six
chapters. Each chapter has the concept booster followed by a large number of exercises with the exact solutions to the problems
as given below:
Level - I : Problems based on Fundamentals
Level - II : Mixed Problems (Objective Type Questions)
Level - III : Problems for JEE Advanced Exam
Level - IV : Tougher Problems for JEE-Advanced Exam
(0.......9) : Integer Type Questions
Passages : Comprehensive Link Passages
Matching : Matrix Match
Previous years’ papers : Questions asked in previous years’ JEE-Advanced Exams

Remember friends, no problem in mathematics is difficult. Once you understand the concept, they will become easy. So
please don’t jump to exercise problems before you go through the Concept Booster and the objectives. Once you are confident
in the theory part, attempt the exercises. The exercise problems are arranged in a manner that they gradually require advanced
thinking.
I hope this book will help you to build your base, enjoy the subject and improve your confidence to tackle any type of
problem easily and skilfully.
My special thanks goes to Mr. M.P. Singh (IISc. Bangalore), Mr. Manoj Kumar (IIT, Delhi), Mr. Nazre Hussain (B. Tech.),
Dr. Syed Kashan Ali (MBBS) and Mr. Shahid Iqbal, who have helped, inspired and motivated me to accomplish this task. As
a matter of fact, teaching being the best learning process, I must thank all my students who inspired me most for writing this
book.
I would like to convey my affectionate thanks to my wife, who helped me immensely and my children who bore with
patience my neglect during the period I remained devoted to this book.
I also convey my sincere thanks to Mr Biswajit Das of McGraw Hill Education for publishing this book in such a beautiful
format.

CG_Prelims.indd 7 2/7/2017 2:33:48 PM


viii Preface

I owe a special debt of gratitude to my father and elder brother, who taught me the first lessons of Mathematics and to all my
learned teachers— Mr. Swapan Halder, Mr. Jadunandan Mishra, Mr. Mahadev Roy and Mr. Dilip Bhattacharya, who instilled
the value of quality teaching in me.
I have tried my best to keep this book error-free. I shall be grateful to the readers for their constructive suggestions toward
the improvement of the book.
Rejaul Makshud
M. Sc. (Calcutta University, Kolkata)

CG_Prelims.indd 8 2/7/2017 2:33:48 PM


Contents

Preface vii
Chapter 1 Straight Lines 1.1–1.74
Rectangular Cartesian Co-ordinates 1.1
Introduction 1.1
Co-ordinate Axes and Rectangular Axes 1.1
Cartesian Co-ordinates 1.1
Polar Co-ordinates 1.1
Relation between the Cartesian and Polar Co-ordinates 1.1
Distance between two Points (Cartesian Form) 1.2
Section Formulae 1.2
Area of a Triangle 1.4
Locus and its Equation 1.4
Geometrical Transformations 1.5
Rotation of Co-ordinate Axes 1.5
Straight Line 1.6
Introduction 1.6
Definitions 1.6
Angle of Inclination of a Line 1.6
Forms of a Straight Line 1.7
Reduction of General Equation into Standard Form 1.8
Position of Two Points with Respect to a Given Line 1.8
Equation of a Line Parallel to a Given Line 1.8
Equation of a Line Perpendicular to a Given Line 1.8
Distance from a Point to a Line 1.8
Point of Intersection of two Lines 1.9
Equation of Straight Lines Passing through a Given Point and Making a
Given Angle with a Given Line 1.10
A Line is Equally Inclined with Two Lines 1.10
Equation of Bisectors 1.10
Foot of the Perpendicular Drawn From the Point (x1, y1) to the Line ax + by + c = 0 1.10
Image of a Point (x1, y1) with Respect to a Line Mirror ax + by + c = 0 1.10
Reflection of Light 1.11
Exercises 1.12
Answers 1.28
Hints and Solutions 1.31

CG_Prelims.indd 9 2/7/2017 2:33:49 PM


x Contents

Chapter 2 Pair of Straight Lines 2.1–2.20


Introduction 2.1
Homogeneous Equation 2.1
Pair of Straight Lines through the Origin 2.1
Angle Between the Lines Represented by ax2 + 2hxy + by2 = 0 2.2
Bisectors of Angles between the Lines Represented by ax2 + 2hxy + by2 = 0 2.2
General Equation of 2nd Degree is ax2 + 2hxy + by2 + 2gx + 2fy + c = 0 2.2
Exercises 2.3
Answers 2.8
Hints And Solutions 2.9
Chapter 3 Circle 3.1–3.83
Introduction 3.1
Mathematical Definitions 3.1
Standard Equation of a Circle 3.2
General Equation of a Circle 3.2
Nature of the Circle 3.2
Concentric Circles 3.2
Concyclic Points 3.2
Parametric Equation of a Circle 3.2
Diametric form of a Circle 3.3
Equation of a Circle Passing through Three Non-collinear Points 3.3
Cyclic Quadrilateral 3.3
Condition for Concyclic Points 3.3
Intercepts Made on the Axes by a Circle 3.3
Different Forms of a Circle 3.3
Position of a Point with Respect to a Circle 3.4
Shortest and Largest Distance of a Circle from a Point 3.4
Intersection of a Line and a Circle 3.4
Length of Intercept Cut off from a Line by a Circle 3.5
Tangent and Secant 3.5
Director Circle 3.7
Normal 3.7
Chord of Contact 3.8
Chord Bisected at a Given Point 3.8
Diameter of a Circle 3.8
Pole and Polar 3.9
Common Chord of Two Circles 3.9
Intersection between Two Circles 3.9
Angle of Intersection of Two Circles 3.11
Radical Axis 3.11
Properties of the Radical Axis 3.11
Radical Centre 3.13
Family of Circles 3.13
Exercises 3.14
Answers 3.35
Hints and Solutions 3.37
Chapter 4 Parabola 4.1–4.63
Introduction 4.1
Basic definitions 4.1
Conic Section 4.2
Recognition of Conics 4.2
Equation of Conic Section 4.2
Parabola 4.3

CG_Prelims.indd 10 2/7/2017 2:33:49 PM


Contents xi

Mathematical Definitions 4.3


Standard Equation of a Parabola 4.3
Important Terms Related to Parabola 4.4
General Equation of a Parabola 4.5
Equation of a Parabola When the Vertex is (h, k) and Axis is Parallel to x-axis 4.5
Equation of a Parabola when the Vertex is (h, k) and Axis is Parallel to y-axis 4.5
Focal Chord 4.5
Position of a Point Relative to a Parabola 4.6
Intersection of a Line and a Parabola 4.6
Tangent 4.6
Normal 4.7
Chord of Contact 4.8
Chord Bisected at a Given Point 4.8
Diameter 4.8
Reflection Property of a Parabola 4.9
Exercises 4.9
Answers 4.21
Hints and Solutions 4.23
Chapter 5 Ellipse 5.1–5.68
Introduction 5.1
Mathematical Definitions 5.1
Equation of an Ellipse 5.2
Properties of an Ellipse 5.2
Parametric Equation of an Ellipse 5.3
Important Properties Related to Chord and Focal Chord 5.4
Position of a Point with Respect to an Ellipse 5.5
Intersection of a Line and an Ellipse 5.6
The Length of the Chord Intercepted by the Ellipse on the Line y = mx + c. 5.6
Various Forms of Tangents 5.6
Director Circle 5.6
Pair of Tangents 5.6
Various Forms of Normals 5.7
Number of Normals are Drawn to an Ellipse From a Point to its Plane 5.7
Chord of Contact 5.8
Chord Bisected at a Given Point 5.8
Pole and Polar 5.8
Diameter 5.8
Conjugate Diameters 5.8
Reflection Property of an Ellipse 5.10
Exercises 5.10
Answers 5.24
Hints and Solutions 5.27
Chapter 6 Hyperbola 6.1–6.59
Introduction 6.1
Mathematical Definitions 6.1
Standard Equation of a Hyperbola 6.2
x2 y 2
Definition and basic terminology of the hyperbola 2 - 2 = 1 6.2
a b
Position of a Point with Respect to a Hyperbola 6.5
Intersection of a Line and a Hyperbola 6.5
Different Forms of Tangents 6.6
Different Forms of Normals 6.7
Chord of Contact 6.7

CG_Prelims.indd 11 2/7/2017 2:33:49 PM


xii Contents

Equation of the Chord Bisected at a Point 6.7


Pole and Polar 6.7
Diameter 6.8
Conjugate Diameters 6.8
Asymptotes 6.8
Rectangular Hyperbola 6.9
Rectangular Hyperbola xy = c2 6.9
Reflection Property of a Hyperbola 6.10
Exercises 6.10
Answers 6.23
Hints and Solutions 6.24

CG_Prelims.indd 12 2/7/2017 2:33:49 PM


C H A P T E R

1 Straight Lines

R ECTANGULAR C ARTESIAN C O - ORDINATES

CONCEPT BOOSTER 3. CARTESIAN CO-ORDINATES


This system of representing a point in 2-dimensions is called
1. INTRODUCTION cartesian system. We normally denote PN by x and PM by y.
Thus an ordered pair of two real numbers describes the carte-
A french mathematician and great philosopher Rane Descartes sian co-ordinates of P.
(1596–1665) introduced an idea to study geometry with the The reference lines XOX¢ and YOY¢ are respectively, called
help of algebra. x- and y-axis. These lines divide the plane into four equal
It is called co-ordinate Geometry or analytical geometry. parts and each part is called quadrant.
Here we use points, lines and curves by the different forms of
algebraic equations. 4. POLAR CO-ORDINATES
2. CO-ORDINATE AXES AND RECTANGULAR AXES Y

The position of a point in a plane with reference to two in- P


tersecting lines called the co-ordinate axes and their point of
intersection is called the origin. If these two axes cut at right
r
angles, they are called rectangular axes, else they are called
oblique axes. q
O M X
Y

II(Second) I(First)
(–, +) (+, +)
The position of a point in a plane can also be described by
other co-ordinate system, called polar co-ordinate system. In
X¢ X this case, we consider OX as initial line, O as origin. If P be
O
any point on the plane such that OP = r and –POX = q, then
(r, q) is called the polar co-ordinate of the point P, where
III (Third) IV (Fourth) r > 0 and 0 £ q < 2p.
(–, –) (+, –)
5. RELATION BETWEEN THE CARTESIAN
AND POLAR CO-ORDINATES
Let P be any point in the plane. Draw perpendiculars from
P parallel to reference lines X¢OX and YOY¢, respectively. Let P(x, y) be the cartesian co-ordinates with respect to OX
The lengths PN and PM are called the co-ordinates of the and OY and P(r, q) be the polars co-ordinates with respect to
point P. the pole O and the initial line OX.

CG_01.indd 1 2/6/2017 3:57:32 PM


1.2 Coordinate Geometry Booster

Y
(ii) an equilateral triangle, show that all sides are
P equal
(iii) a scalene triangle, show that all sides are un-
equal.
r (iv) a right-angled triangle, say DABC, show that AB2
q + BC2 = AC2
X
O M
2. When 4 points are given and in order to prove
(i) a square, show that all sides and diagonals are
equal.
(ii) a rhombus, show that all sides are equal but di-
Now from the figure, x = r cos q and y = r sin q agonals are not equal.
(iii) a rectangle, show that opposite sides and diago-
Ê yˆ
Thus, r = x 2 + y 2 and q = tan -1 Á ˜ . nals are equal.
Ë x¯
(iv) a parallelogram, show that opposite sides and
Therefore, (x, y) fi (r cos q, r sin q) diagonals are equal.
Ê Ê yˆˆ
and (r , q ) fi Á x 2 + y 2 , tan -1 Á ˜ ˜ .
Ë Ë x¯¯
7. SECTION FORMULAE
6. DISTANCE BETWEEN TWO POINTS (CARTESIAN FORM) (i) Internal Section formula
m n
Y Q(x2, y2) P(x1, y1) R(x, y) Q(x2, y2)

If a point R(x, y) divides a line segment joining the


points P(x1, y1) and Q(x2, y2) internally in the ratio
(x1, y1) m : n, then
P R mx + nx1 my2 + ny1
x= 2 ,y=
m+n m+n
O X
A(x1, O) B(x2, O) (ii) External section formula
m
The distance between any two points P(x1, y1) and Q(x2, y2) is n
P(x1, y1) Q(x2, y2) R(x, y)
PQ = ( x2 - x1 ) 2 + ( y2 - y1 ) 2
If a point R(x, y) divides a line segment joining the
Note: If the three points P, Q, R are collinear, then points P(x1, y1) and Q(x2, y2) externally in the ratio
|PQ| ± |QR| = |PR|. m : n, then
mx - nx1 my2 - ny1
x= 2 ,y=
m-n m-n
Polar Form Y (iii) Tri-section formula
Let P(r1, q1) and Q(r2, q2) Q(r2, q2)
(x¢, y¢)
2 S 1
be any two points in polar r2
form and q be the angle be- q2 r 1 P(r1, q1) P 1 R 2 Q
tween then. q1 (x1, y1) (x, y) (x2, y2)
X¢ X
Then O Let the points P and Q be (x1,y1) and (x2,y2) respectively.
r 2 + r22 - PQ 2 Then the co-ordinates of the point R is
cos q = 1
2r1r2 Ê x2 + 2x1 y2 + 2y1 ˆ

ÁË ,
fi PQ = r + r – 2r1r2 cosq
2 2 2 3 3 ˜¯
1 2
and the co-ordinates of the point S is
fi PQ = r12 + r22 - 2r1r2 cos q Ê 2x2 + x1 2y2 + y1 ˆ
ÁË , .
3 3 ˜¯
Notes:
(iv) Mid-point formula
1. When 3 points are given and in order to prove
If a point R(x, y) divides a line segment joining the
(i) an isosceles triangle, show that any two sides are
points P(x1, y1) and Q(x2, y2) internally in the same
equal.
ratio, then

CG_01.indd 2 2/6/2017 3:57:34 PM


Straight Lines 1.3

(iv) If DABC is an equilateral triangle,


x +x y + y2
x= 1 2,y= 1 in-centre = centroid.
2 2 (v) Ex-centre: The point of intersection of the external bi-
(v) If the line joining the points (x1, y1) and (x2, y2) is sectors of the angles of a triangle is called its ex-centre.
divided by the
y1
(a) x-axis,then its ratio is -
y2 B r
x1 I1
(b) y-axis , then its ratio is - . a r r
x2
a
(vi) If a line ax + by + c = 0 divides the line joining the C
A
points P(x1, y1) and Q(x2, y2) at R, then the ratio
The circle opposite to the vertex A is called the escribed
PR
= - ax1 + by1 + c . circle opposite A or the circle excribed to the side BC.
RQ ax2 + by2 + c If I1 is the point of intersection of internal bisector of
–BAC and external bisector of –ABC and –ACB, then
Ê - ax1 + bx2 + cx3 - ay1 + by2 + cy3 ˆ
Important Points of a Triangle I1 = Á ,
Ë -a + b + c - a + b + c ˜¯
(i) Centroid: The point of intersection of the medians of a
Ê ax - bx2 + cx3 ay1 - by2 + cy3 ˆ
triangle is called centroid. I2 = Á 1 ,
Ë a—b+c a — b + c ¯˜
A(x1, y1)
Ê ax + bx2 - cx3 ay1 + by2 - cy3 ˆ
and I3 = Á 1 ,
2 Ë a+b-c a + b - c ˜¯
F E
G respectively.
1 (vi) Circumcentre: The point of intersection of the per-
pendicular bisectors of a triangle is called its circum-
B(x2, y2) D C(x3, y3) centre.
If the vertices of DABC be A(x1, y1), B(x2, y2) and C(x3, A
y3), the co-ordinates of its centroid is
Ê x1 + x2 + x3 y1 + y2 + y3 ˆ
ÁË , ˜¯ . F E
3 3 O

(ii) If (a1, b1), (a2, b2) and (a3, b3) be the mid-points of
the sides of a triangle, its centrod is also given as C
B D
Ê a1 + a2 + a3 b1 + b2 + b3 ˆ
ÁË , ˜¯ .
3 3 If the vertices of DABC be A(x1, y1), B(x2, y2) and C(x3,
(iii) Incentre: The point of intersection of an angle bisec- y3), then the co-ordinates of its circumcentre is
tors of a triangle is called its incentre. Ê x1sin 2A + x2 sin 2B + x3sin 2C
C(x3, y3)
ÁË sin 2A + sin 2B + sin 2C ,

E y1sin 2A + y2 sin 2B + y3sin 2C ˆ


sin 2A + sin 2B + sin 2C ˜¯
F
b 1 a

Notes:
1. If O(x, y) be the circumcentre of DABC, its co-ordi-
A(x1, y1) D c B(x2, y2) nates is determined by the relation
OA = OB = OC.
If the vertices of DABC be A(x1, y1), B(x2, y2) 2. In case of a right-angled triangle, mid-point of the
and C(x3, y3), the co-ordinates of its incentre is hypotenuse is the circumcentre.
Ê ax1 + bx2 + cx3 ay1 + by2 + cy3 ˆ
ÁË a + b + c , a + b + c ˜¯ , (vii) Orthocentre: The point of intersection of the altitudes
of a triangle is called its orthocentre.
where a = BC, b = CA, c = AB.

CG_01.indd 3 2/6/2017 3:57:35 PM


1.4 Coordinate Geometry Booster

A
x1 y1 1
1
or x2 y2 1
2
x3 y3 1
F E
O
1 x1 y1 1 x2 y2 1 x3 y3
or + +
2 x2 y2 2 x3 y3 2 x1 y1
B D C
Notes:
If the vertices of DABC be A(x1, y1), B(x2, y2) and 1. The area of a polygon, with verti-
C(x3, y3), the co-ordinates of its orthocentre is ces (x1, y1), (x2, y2), (x3, y3), …, (xn, yn) is
Ê x1tan A + x2 tan B + x3 tan C
1 x1 y1 1 x2 y2 1 xn yn
ÁË tan A + tan B + tan C , + + + .
2 x2 y2 2 x3 y3 2 x1 y1
y1tan A + y2 tan B + y3 tan C ˆ
2. If the vertices of a triangle be A(r1, q1), B(r2q2) and
tan A + tan B + tan C ˜¯ (r3, q3), its area is
Notes: 1
1. In case of a right-angled triangle, orthocentre is the [r r sin (a1 – q2) + r2r3sin (q2 – q3) + r3r1sin (q3 – q1)]
2 12
right-angled vertex.
2. In an isosceles triangle, G, O, I and C lie on the same 3. If ai x + bi y + Ci = 0, i = 1, 2, 3 be the sides of a tri-
line and in an equilateral triangle, all these four points angle, its area is
2
coincide. a1 b1 c1
3. The orthocentre of DABC are (a, b), (b, a) and (a, a) 1
a2 b2 c2 ,
is (a, a). 2C1C2 C3
4. The orthocentre, the centroid and the circumcentre a3 b3 c3
are always collinear and centroid divides the line where C1, C2 and C3 are the cofactors of c1, c2 and c3
joining ortho-centre and the circumcentre in the ratio a1 b1 c1
2 : 1.
in the determinant a2 b2 c2 .
a3 b3 c3
4. If A(x1, y1), B(x2, y2) and C(x3, y3) be the vertices of a
Orthocentre
Centroid triangle, its area is given by
Circumcentre 1 x1 - x3 x2 - x3
2 y1 - y3 y2 - y3
Euler line
5. If A(x1, y1), B(x2, y2) and C(x3, y3) be three collinear,
5. In case of an obtuse-angled triangle, the circumcentre points then
and orthocentre both lie outside of the triangle. x1 - x3 x2 - x3
=0
y1 - y3 y2 - y3

8. AREA OF A TRIANGLE or
x1(y2 – y3) + x2(y3 – y1) + x3(y1 – y2) = 0
A(x1, y1)

C(x3, x3) 9. LOCUS AND ITS EQUATION


B(x2, y2) The locus of a moving point is the path of a point which satis-
X fies some geometrical conditions.
O M L N
If a point moves in a plane in such a way
that its distance from a fixed point is always d
the same. Then the locus of the movable point
P
If A(x1, y1), B(x2, y2) and C(x3, y3) be the vertices of DABC, its is called a circle.
area is given by If a points moves in a plane in such a way
1 that its distances from two fixed points are always the same,
[ x1 ( y2 - y3 ) + x2 ( y3 - y1 ) + x3 (y1 - y2 )]. the locus of the point is a perpendicular bisector.
2

CG_01.indd 4 2/6/2017 3:57:36 PM


Straight Lines 1.5

C The coordinates of a point in a plane, when the origin is


E shifted from origin to a new point (h, k), the new axes remain
parallel to the original axes.
F
Y Y¢
G P(x¢, y¢)
P(x, y)
H

A B O¢ M¢
D k
h L X
O M
Equation of a Locus
If the co-ordinates of every point on the locus satisfy the Let O be the origin and, OX and OY the original axes.
equation as well as if the co-ordinates of any point satisfy the Let the origin O be shifted to a new point O¢, whose co-
equation, that point must lie on the locus. ordinates are (h, k). Through O¢ draw O¢X¢ and O¢Y¢ parallel
to OX and OY, respectively. O¢ is the new origin and, O¢X¢,
Rule to Find Out the Locus of a Moving Point and O¢Y¢ are the new axes.
(i) If x and y co-ordinates of the moving point are given in Let P be any point in this plane, whose co-ordinates ac-
terms of a third variable, say t, eliminating t between x cording to the original and new axes be (x, y) and (x¢, y¢)
and y and then get the required locus. respectively.
(ii) Sometimes the co-ordinates of the moving point is tak- Now we have to establish a relation between the new co-
en as (x, y). In this case, the relation in x and y can be ordinates and the original co-ordinates.
directly obtained by eliminating the variable. From P, draw a perpendicular PM to OX which intersects
(iii) If some geometrical conditions are given and we have O¢X¢ at M¢.
to find the locus, we take a variable point (a, b) and Again from O¢, draw a perpendicular O¢L to OX.
write the given conditions in terms of a and b. Elimi- Then OL = h, O¢L = k ;
nating the variables and the relation between a and b OM = x, PM = y;
is obtained. Finally replacing a by x and b by y the
O¢M¢ = x¢, PM¢ = y¢
required locus is obtained.
Thus, x = OM = OL + LM
10. GEOMETRICAL TRANSFORMATIONS = OL + O¢M
When talking about geometric transformations, we have to = h + x¢ = x¢ + h
be very careful about the object being transformed. We have and y = PM = PM¢ + M¢M
two alternatives, either the geometric objects are transformed = PM¢ + M¢M
or the co-ordinate system is transformed. These two are very
= y¢ + k
closely related, but the formulae that carry out the job are dif-
ferent. We only discuss transforming geometric objects here. Hence, we obtained the relation between the new and the
We shall start with the traditional Euclidean transforma- original co-ordinates as
tions that do not change length and angle measures. Ï x = x¢ + h Ï x¢ = x - h
Ì and Ì
Ó y = y¢ + k Ó y¢ = y - k
Euclidean Transformations
The Euclidean transformations are the most commonly used
transformations. An Euclidean transformation is either a 11. ROTATION OF CO-ORDINATE AXES
translation, a rotation, or a reflection. We shall discuss trans- Y
lations and rotations only. Y¢
(i) If the origin be shifted to a new point, say (h, k) and X¢
the new axes remain parallel to the original axes, the R(0, y) P(x, y)
transformation is called translation of axes. P¢(x, y¢)
(ii) If the axes are rotated through an angle q and the origin Q¢(x¢, 0)
remain fixed, the transformation is called rotation of
q
axes. R¢
X
(iii) If the origin be shifted to a new point, say (h, k) and the Q(x, 0)
axes also be rotated through an angle q, the transforma-
tion is termed as translation and rotation of axes.

CG_01.indd 5 2/6/2017 3:57:37 PM


1.6 Coordinate Geometry Booster

Let OX and OY be the original axes and OX¢ and OY¢ be the Then x = x ¢cos q – y ¢sin q
…(i)
new axes obtained after rotating OX and OY through an angle y = x ¢sin q – y ¢cos q
q in the anti-clockwise direction. Let P be any point in the and x ¢ = x cos q – y sin q
plane having co-ordinates (x, y) with respect to axes OX and …(ii)
y ¢ = x sin q – y cos q
OY and (x¢, y¢) with respect to axes OX¢ and OY¢.

S TRAIGHT L INE
1. INTRODUCTION The angle of inclination of the
line lies in between 0° and 180°.
The notion of a line or a straight line was introduced by ancient
mathematicians to represent straight objects with negligible Slope or Gradient of q
width and depth. Lines are an idealisation of such objects. a Lines
Euclid described a line as ‘breadthless length’, and in-
If the inclination of a line be q,
troduced several postulates as basic unprovable properties
then tan q where 0 < q <180° and
from which he constructed the geometry, which is now called
p
Euclidean geometry to avoid confusion with other geometries q π is called the slope or gradient of the line. It is usually
which have been introduced since the end of nineteenth cen- 2
tury (such as non-Euclidean geometry, projective geometry, denoted by m.
and affine geometry, etc.). (i) If a line is parallel to x-axis, its slope is zero.
A line segment is a part of a line that is bounded by two (ii) If a line is perpendicular to x-axis, the slope is not de-
distinct end-points and contains every point on the line be- fined.
tween its end-points. Depending on how the line segment is (iii) If a line is equally inclined with the axes, the slope is
defined, either of the two end-points may or may not be a part ±1.
of the line segment. Two or more line segments may have (iv) If P(x1, y1) and Y
Q(x2, y2)
some of the same relation as lines, such as being parallel, in- Q(x2,y2) are two
tersecting, or skew. points on a line L, the
slope of the line L is P(x1, y1) q
2. DEFINITIONS equal to
y - y1 X¢ X
Definition 1 m= 2 . O M N
It is the locus of a point which moves in a plane in a constant x2 - x1
direction. Y C
(v) If three points A, B, C are
Definition 2 B
collinear, then A
Every first-degree equation in x and y represents a straight
line. m(AB) = m(BC) X
O
Definition 3 = m(CA).
A straight line is also defined as the curve such that the line
segment joining any two points on it lies wholly on it. (vi) If two lines, having slopes m1 and m2 and the angle be-
Definition 4 tween them be q, then
In 3D geometry, the point of intersection of two planes be a m - m2
tan q = 1 .
line. 1 + m1m2
Notes: (vii) If two lines, having slopes m1 and m2, are parallel, then
1. The equation of a straight line is the relation between m 1 = m2
x and y, which is satisfied by the co-ordinates of each (viii) If two lines, having slopes m1 and m2, are perpendicu-
and every point on the line. lar, then m1m2 = –1
2. A straight line consists of only two arbitrary constants. (ix) If m is a slope of a line, the slope of a line perpendicular
-1
to it is .
m
3. ANGLE OF INCLINATION OF A LINE (x) The equation of x-axis is y = 0.
The angle of inclination of a line is the measure of the angle (xi) The equation of y-axis is x = 0.
between the x-axis and the line measured in the anti-clock- (xii) The equation of a line parallel to x-axis is y = constant
wise direction. = b (say)

CG_01.indd 6 2/6/2017 3:57:37 PM


Straight Lines 1.7

(xiii) The equation of a line parallel to y-axis is x = constant The equation of a line passing through (x1, y1) and (x2, y2) is
= a (say). Ê y - y2 ˆ
y - y1 = Á 2 ¥ ( x - x1 )
Ë x2 - x1 ˜¯
4. FORMS OF A STRAIGHT LINE
or
(i) Slope-intercept Y Ê y - y2 ˆ
y - y2 = Á 2 ¥ ( x - x2 )
Form P(x, y) Ë x2 - x1 ˜¯
The equation of a
x y 1
straight line whose Q q
slope is m and cuts an or x1 y1 1 = 0
q c}
intercept c on the y-axis X x2 y2 1
R O M
is
y = mx + c. x - x1 y - y1
or =0
x1 - x2 y1 - y2
Notes:
1. The general equation of a straight line is
(iv) Intercept Form
y = mx + c.
2. The equation of non-vertical lines in a plane is Y
y = mx + c B(0, b)
3. The general equation of any line passing through the
origin is
y = mx.

X
O A(a, 0)
(ii) Point-Slope Form
Y The equation of a straight line which cuts off intercepts a and
, y) b on x- and y-axes, respectively is
P(x
x y
+ = 1.
y 1) a b
1,
Q(x q
Notes:
1. The intercepts a and b may be positive or negative.
X 2. The intercept cut on negative side of x and y axes are
O L M
taken as negative.

Y
(v) Normal Form
The equation of a line passing through the point (x1, y1) and
The equation of a straight B(0, b)
having slope m is
line upon which the length
y – y1 = m(x – x1).
of a perpendicular from the P(x, y)
origin is p and this normal
Note: The equation y – y1 = m(x – x1) is also known as one- p
makes an angle a with the
point form of a line. X¢ X
positive direction of x-axis is O A(a, 0)
x cos a + y sin a = p.
(iii) Two Point Form Y¢

P(x, y)
Note: Here, p is always taken is positive and a is measured
Y y 2) from the positive direction of x-axis in anti-clockwise di-
x 2, rection such that 0 £ a < 2p.
R(
, y 1)
Q(x 1
(vi) Distance Form or Parametric Form or
O L N M
X Symmetric Form
The equation of a line passing through the point (x1, y1) and
making angle q with the positive direction of x-axis is

CG_01.indd 7 2/6/2017 3:57:39 PM


1.8 Coordinate Geometry Booster

x - x1 y - y1 Ê -A ˆ Ê -B ˆ C
= =r, fi x- y=
cos q sin q Á 2 2˜ Á 2 2˜
Ë A +B ¯ Ë A +B ¯ ( A + B2 )
2

where r is the distance of the point (x, y) from the point which is the normal form of the line Ax + By + C = 0.
(x1, y1).
Y y)
P(
x, 6. POSITION OF TWO POINTS WITH RESPECT
TO A GIVEN LINE
y 1) q
x 1,
Q( The points P(x1, y1) and Q(x2, y2) lie on the same or on
the opposite sides of the line ax + by + c = 0 according as
ax1 + by1 + c
O
X > 0 or < 0.
ax2 + by2 + c
Y
Q
Notes: P
1. If Q(x1, y1) be a point on a line AB which makes an P
Q
angle q with the positive direction of x-axis, there P Q
will be two points on the line AB at a distance r from
Q(x1, y1) and their co-ordinates will be (x1 ± r cosq, y1
X
± r sinq). O
2. If a point, say P, lies above Q(x1, y1) on the line AB,
we consider r is positive.
Thus, the co-ordinates of P will be Notes:
(x1 + r cosq, y1 + r sinq). 1. The side of the line, where origin lies, is known as
3. If a point, say R, lies below Q(x1, y1) on the line AB, origin side.
we consider r is negative. 2. A point (a, b) will lie on the origin side of the line
Thus, the co-ordinates of R will be ax + by + c = 0, if ab + bb + c and c have the same
(x1 – r cosq, y1 – r sinq). sign.
3. A point (a, b) will lie on the non-origin side of the
line ax + by + c = 0, if aa + bb + c and c have the
5. REDUCTION OF GENERAL EQUATION INTO opposite signs.
STANDARD FORM
Let Ax + By + C = 0 be the general equation of a straight line, 7. EQUATION OF A LINE PARALLEL TO A GIVEN LINE
where A2 + B2 π 0.
(i) Reduction into slope-intercept form The equation of any line parallel to a given line
The given equation is Ax + By + C = 0. ax + by + c = 0 is ax + by + k = 0
fi By = –Ax – C
Note: The equation of a line parallel to ax + by + c = 0 and
Ê Aˆ Ê Cˆ
fi y = -Á ˜ x - Á ˜ passing through (x1, y1) is
Ë B¯ Ë B¯
a(x – x1) + b(y – y1) = 0
which is in the form of y = mx + c.
(ii) Reduction into intercept form
The given equation is Ax + By + C = 0. 8. EQUATION OF A LINE PERPENDICULAR TO A GIVEN LINE
fi Ax + By = –C
Ax By The equation of a line perpendicular to a given line
fi + =1 ax + by + c = 0 is bx – ay + k = 0.
-C -C
x y Note: The equation of a line perpendicular to ax + by + c =
fi + =1
Ê C ˆ Ê Cˆ 0 and passing through (x1, y1) is
ÁË - ˜ Á - ˜
A¯ Ë B ¯ b(x – x1) – a(y – y1) = 0.
x y
which is in the form of + = 1 .
a b
(iii) Reduction into normal form 9. DISTANCE FROM A POINT TO A LINE
The given equation is Ax + By + C = 0. The length of the perpendicular from a point (x1, y1) to the
fi –Ax – By = C line ax + by + c = 0 is given by

CG_01.indd 8 2/6/2017 3:57:39 PM


Straight Lines 1.9

Y P(x
ax1 + by1 + c 1,
D C
. y1 )
a +b2 2 b)
, –c/ p1
B(0
Area of DPAB q
M A
1 M
B
= ¥ AB ¥ PM X q
2 O A(–c/a, 0)
p2
We know that F
x1 y1 1 The distance between two parallel sides a1x + b1 y + c1 = 0,
1 c a1x + b1 y + d1 = 0 is given by
D= - 0 1
2 a (c - d )
p1 = 1 1
c a12 + b12
0 - 1
b The distance between two parallel sides a2x + b2 y + c2 = 0,
1 Ê cx cÊ cˆˆ a2x + b2 y + d2 = 0 is given by
= Á 1 + Á y1 + ˜ ˜
2Ë b aË b¯¯ (c - d 2 )
p2 = 2 .
1 Ê cx1 cy1 c 2 ˆ a22 + b22
= + + ˜
2 ÁË b a ab ¯ b1 b2
-
c a1 a2 a b -ab
= (ax1 + by1 + c) Now, tan q = = 2 1 1 2
2ab b1 b2 a1a2 + b1b2
1+ ◊
Also, PM = a 2 + b 2 a1 a2
1 c a1b2 - a2 b1
Thus, ¥ PM ¥ AB = (ax1 + by1 + c) fi sin q =
2 2ab a1 + b12 a22 + b22
2

1 c2 c2 c p ¥ p2
fi ¥ PM ¥ + = (ax1 + by1 + c) Thus, the area of a parallelogram = 1
2 a 2 b 2 2ab sin q
c c (c1 - d1 )(c2 - d 2 )
fi PM ¥ a 2 + b2 = (ax1 + by1 + c) =
2ab 2ab (a1b2 - a2 b1 )

fi PM =
(ax1 + by1 + c ) . . Rule 4. If p1 = p2, the parallelogram becomes a rhombus
and its area is given by
a 2 + b2 (c1 - d1 ) 2
.
Rule 1. The length of the perpendicular from the origin to Ê a12 + b12 ˆ
|(a1b2 - a2 b1 )| Á 2 ˜
the line ax + by + c = 0 is Ë a2 + a22 ¯
c Rule 5. The area of a parallelogram whose sides are y = mx
.
a +b2 2 + a, y = mx + b, y = nx + c and y = nx + d is given by
(a - b)(c - d )
Rule 2. The distance between two parallel lines ax + by + .
c1 = 0 and ax + by + c2 = 0 is given by ( m - n)

c1 - c2
. 10. POINT OF INTERSECTION OF TWO LINES
a 2 + b2
Let a1x + b1y + c1 = 0 and a2x + b2y + c2 = 0 be two lines.
P ax + by + c1 = 0 (i) Simply solve the given equations and get the point of
intersection P.
(ii) Concurrent lines: If three or more lines meet at a
point, we say that these lines are concurrent lines and
Q ax + by + c2 = 0
the meeting point is known as point of concurrency.
Rule 3. The area of a parallelogram whose sides are a1x + The three lines a1x + b1 y + c1 = 0, i = 1, 2, 3 are said
b1 y + c1 = 0, a1x + b1 y + d1 = 0, a2 x + b2 y + c2 = 0 and a2x + to be concurrent if
b2 y + d2 = 0 is given by a1 b1 c1
p1 ¥ p2 (c1 - d1 )(c2 - d 2 ) a2 b2 c2 = 0
= .
sin q (a1b2 - a2 b1 ) a3 b3 c3

CG_01.indd 9 2/6/2017 3:57:40 PM


1.10 Coordinate Geometry Booster

(iii) Family of lines: Any line passing through the point of


Y C
intersection of the lines L1: a1x + b1 y + c1 = 0 and L2:
B
a2x + b2 y + c2 = 0 can be defined as

N
P
(a1x + b1 y + c1) + l(a2x + b2 y + c2) = 0, where l ΠR.
M
A D
X
O

Notes:
L2 = 0 L1 = 0 1. When we shall find the equation of bisectors, first we
make c1 and c2 positive.
11. EQUATION OF STRAIGHT LINES PASSING THROUGH A 2. Two bisectors are perpendicular to each other.
3. The positive bisector, the equation of the bisector
GIVEN POINT AND MAKING A GIVEN ANGLE WITH A contains the origin.
GIVEN LINE 4. The negative bisector, the equation of the bisector
The equation of the straight lines which pass through a given does not contain the origin.
point (x1, y1) and make an angle a with the given straight line 5. If a1a2 + b1b2 > 0, then the negative bisector is the
y = mx + c are acute-angle bisector and the positive bisector is the
y – y1 = tan (q ± a)(x – x1), obtuse-angle bisector.
6. If a1a2 + b1b2 < 0, then the positive bisector is the
where m = tan q. acute-angle bisector and the negative bisector is the
Y obtuse-angle bisector.
F D 7. If a1a2 + b1b2 > 0, the origin lies in obtuse angle and
P(x1, y1) if a1a2 + b1b2 < 0, the origin lies in acute angle.
180° – q B
8. The equation of the bisector of the angle between the
a q two lines containing the point (a, b) will be positive
a R bisector according as (a1a + b1b + c1) and (a2a + b2b
q E + c2) are of the same sign and will be negative bisec-
q Q tor if (a1a + b1b + c1) and (a2a + b2b + c2) are of the
X
A
O
C
opposite signs.

12. A LINE IS EQUALLY INCLINED WITH TWO LINES 14. FOOT OF THE PERPENDICULAR DRAWN FROM THE
L2 POINT (x1, y1) TO THE LINE ax + by + c = 0
L P(x1, y1)

L1
q
q

O A M ax + by + c = 0 B

If two lines y = m1x + c1 and y = m2x + c2 be equally inclined The foot of the perpendicular from the point (x1, y1) to the line
to a line y = mx + c, then ax+ by + c = 0 is given by
x2 - x1 y2 - y1 (ax + by + c)
Ê m1 - m ˆ Ê m2 - m ˆ = = - 1 2 12 .
ÁË 1 + mm ˜¯ = - ÁË 1 + mm ˜¯ . a b (a + b )
1 2

15. IMAGE OF A POINT (x1, y1) WITH RESPECT TO A LINE


13. EQUATION OF BISECTORS
MIRROR ax + by + c = 0
The equation of the bisectors of the angles between the
The image of a point (x1, y1) with respect to a line mirror
lines a1x + b1 y + c1 = 0 and a2x + b2 y + c2 = 0 is given by
ax + by + c = 0 is given by
Ê a1 x + b1 y + c1 ˆ Ê a x + b2 y + c2 ˆ x2 - x1 y2 - y1 Ê ax + by + c ˆ
=± 2 . = = - 2 Á 1 2 12 ˜
Á ˜ Á ˜
Ë a12 + b12 ¯ Ë a22 + b22 ¯ a b Ë a +b ¯

CG_01.indd 10 2/6/2017 3:57:42 PM


Straight Lines 1.11

P(x1, y1) Y
Rule 6 The image of a point P
y=x
P(a, b) with respect to the line M
y = x is Q(b, a).
Q
A M ax + by + c = 0 B X¢ O X
Note: The image of a line
ax + by + c = 0 about the
line y = x is ay + bx + c = 0.

Q(x2, y2)
Y Rule 7 The image of a point P(a, b) with respect to the line y
Rule 1 The image of a point P = mx, where m = tan q, is given by
P(a, b) with respect to x-axis
is given by Q(a, – b). Q(a cos 2q + b sin 2q, a sin 2q – b cos 2q).
X¢ X Y
O
Note: The image of the line
P
ax + by + c = 0 about x-axis
M
is given by ax – by + c = 0. Q
Y¢ q Q
X¢ x
Y O
Rule 2 The image of a point
mx
P(a, b) with respect to y-axis Q P y=
is given by Q(–a, b).
X¢ X Y¢
Note: The image of the line O
ax + by + c = 0 about y-axis Note: The image of a line ax + by + c = 0 with respect to the
is given by –ax + by + c = 0. line y = mx, where m = tan q, is given by

a(a cos 2q + b sin 2q) + b(a sin 2q – b cos 2q) + c = 0.
Rule 3 The image of a point Y
P(a, b) with respect to the P
origin is given by Q(–a, –b). 16. REFLECTION OF LIGHT
Note: The image of the X¢ O X When you play billiards, you will notice that when a ball
line ax + by + c = 0 with bounces from a surface, the angle of rebound is equal to the
respect to origin is given angle of incidence. This observation is also true with light.
Q
by –ax – by + c = 0. When an incident light ray strikes a smooth surface (like a

plane mirror) at an angle, the angle formed by the incident
Rule 4 The image of a point
ray measured from the normal is equal to the angle formed
P(a, b) with respect to the
Y by the reflected ray.
line x = a is given by Q(2a
M
– a, b). P Q
Law of Reflection
Note: The image of the X¢ X The angle of incidence is equal to the angle of reflection. The
line ax + by + c = 0 with reflected and incident rays lie in a plane that is normal to the
respect to the line x = l is x=a reflecting surface.
given by a(2l – x) + by +
c = 0. Y¢
Angle of Angle of
Y incidence equals reflection
Rule 5 The image of a point P
P(a, b) with respect to the line
M
y = b is given by Q(a, 2b – b). y=b
Q Incident ray Reflected ray
Note: The image of the X¢ X
O
line ax + by + c = 0 about
the line y = m is ax + b(2m
Plane mirror
– y) + c = 0. Y¢

CG_01.indd 11 2/6/2017 3:57:45 PM


1.12 Coordinate Geometry Booster

E XERCISES

LEVEL I 22 The vertices of a DABC are A(0, 0), B(0, 2) and C(2, 0).
Find the distance between the circum-centre and ortho-
(Problems based on Fundamentals) centre.
ABC OF COORDINATES
AREA OF A TRIANGLE
1. Find the polar co-ordinates of the points whose carte-
sian co-ordinates are (3, –4) and (–3, 4). 23. Find the area of a triangle whose vertices are (3, –4),
2. Transform the equation r2 = a2 into cartesian form. (7, 5) (–1, 10).
3. Transform the equation r = 2a cos q into cartesian form. 24. Find the area of a triangle whose vertices are (t, t + 2),
(t + 3, t) and (t + 2, t + 2).
x2 y 2
4. Transform the equation 2 + 2 = 1 into polar form. 25. If A( x, y), B(1, 2) and C( 2, 1) be the vertices of a
a b triangle of area 6 sq.u., prove that x + y +9 = 0.
5. Transform the equation 2x2 + 3xy + 2y2 = 1 into polar 26. Find the area of a quadrilateral whose vertices are
form. (1, 1), (7, –3), (12, 2) and (7, 21).
27. Find the area of a pentagon whose vertices are (4, 3),
DISTANCE BETWEEN TWO POINTS
(–5, 6) (0, 07) (3, –6) and (–7, –2).
6. Find the distance between the points (a cos a, a sina) 28. Prove that the points (a, b + c), (b, c + a) and (c, a + b)
and (a cos b, a sinb), where a > 0. are collinear.
7. Find the distance between the points 29. Let the co-ordinates of A, B, C and D are (6, 3), (–3, 5),
Ê pˆ Ê 5p ˆ DDBC 1
ÁË 3, ˜¯ and ÁË 7, ˜. (4, –2) and (x, 3x), respectively and = , find x.
4 4¯ DABC 2
8. If the point (x, y) be equidistant from the points (6, –1) 30. Find the area of a triangle formed by the lines
and (2, 3) such that Ax + By + C = 0, find the value of 7x – 2y + 10 = 0, 7x + 2y – 10 = 0 and 9x + y + 2 = 0.
A + B + C + 10.
9. The vertices of a triangle ABC are A(–2, 3), B( 2, –1) LOCUS OF A POINT
and C( 4, 0). Find cos A.
31. If the co-ordinates of a variable point P be (a cos q,
10. Prove that the points (–4, –1), (–2, –4), (4, 0) and (2, 3)
a sinq), where q is a parameter, find the locus of P.
are the vertices of a rectangle.
32. If the co-ordinates of a variable point P be (at2, 2at),
11 Two vertices of an equilateral triangle are (3, 4) and
find the locus of P.
(–2, 3). Find the co-ordinates of the third vertex.
33. Find the locus of a movable point P, where its distance
from the origin is 3 time its distance from y-axis.
SECTION FORMULAE
34. If a point moves in a plane in such a way that its dis-
12. Find the point, which divides the line joining the points tance from the point (a, 0) to its distance is equal to its
(2, 3) and (5, –3) in the ratio 1 : 2. distance from y-axis.
13. In what ratio does y-axis divide the line segment join- Ê 1 1ˆ
35. If the co-ordinates of a variable point P be Á t + , t - ˜ ,
ing A(–3, 5) and B(7, 2). Ë t t¯
14. Find the ratio in which the join of the points (1, 2) and where t is a parameter, find the locus of P.
(–2, 3) is divided by the line 3x + 4y = 7. 36. Find the locus of a movable point P, for which the sum
15. Find the co-ordinates of the points which trisect the of its distance from (0, 3) and (0, –3) is 8.
line segment joining (1, –2) and (–3, 4). 37. A stick of length l slides with its ends on two mutually
16. The co-ordinates of the mid-points of the sides of a tri- perpendicular lines. Find the locus of the mid-point of
angle are (1, 1), (3, 2) and (4, 1). Find the co-ordinates the stick.
of its vertices. 38. If O be the origin and A be a point on the line y2 = 8x.
17. The co-ordinates of three consecutive vertices of a paral- Find the locus of the mid-point of OA.
lelogram are (1, 3), (–1, 2) and (2, 5). Find its fourth vertex. 39. If P be the mid-point of the straight line joining the
18. Find the centroid of DABC, whose vertices are A(2, 4), points A(1, 2) and Q where Q is a variable point on the
B(6, 4), C(2, 0). curve x2 + y2 + x + y = 0. Find the locus of P.
19. Two vertices of a triangle are (–1, 4) and (5, 2). If its 40. A circle has the centre (2, 2) and always touches x and
centroid is (0, –3), find its third vertex. y axes. If it always touches a line AB (where A and B lie
20. Find the in-centre of DABC, whose vertices are A(1, 2), on positive x and y axes), find the locus of the circum-
B(2, 3) and C(3, 4). centre of the DOAB, where O is the origin.
21. If a triangle has its orthocentre at (1, 1) and circum- 41. Find the locus of a movable point P, for which the dif-
centre at (3/2, 3/4), find its centroid. ference of its distances from (2, 0) and (–2, 0) is 6.

CG_01.indd 12 2/6/2017 3:57:45 PM


Straight Lines 1.13

42. If A(1, 1) and B(–2, 3) are two fixed points, find the 3. Prove that the line joining the points (2,–3) and (–5, 1)
locus of a point P so that the area of DPAB is 9 sq.u. is parallel to the line joining (7,–1) and (0,3).
43. Find the locus of a point whose co-ordinates are given 4. Prove that the points (a, b + c), (b, c + a) and (c, a + b)
by x = t2 + t, y = 2t + 1, where t is parameter. are collinear.
x y 5. Find the angle between the lines joining the points
44. If a variable line + = 1 intersects the axes at A and
a b (0, 0), (2, 2) and (2, –2) (3, 5).
B, respectively such that a2 + b2 = 4 and O be the origin, 6. If the angle between two lines is 45° and the slope of
find the locus of the circumcentre of DOAB. one of them is 1/2, find the slope of the other line.
45. A variable line cuts the x and y axes at A and B respec- 7. Find the equation of a line passing through the point
tively where OA = a, OB = b (O the origin) such that (2, –3) and is parallel to x-axis.
a2 + b2 = 27, find the locus of the centroid of DOAB. 8. Find the equation of a line passing through the point
TRANSFORMATION OF COORDINATES (3, 4) and is perpendicular to y-axis.
9. Find the equation of a line which is equidistant from
46. Find the equation of the curve 2x2 + y2 – 3x + 5y – 8 =
the lines x = 6 and x = 10.
0, when the origin is shifted to the point (–1, 2) without
changing the direction of the axes. VARIOUS FORMS OF A STRAIGHT LINE
47. The equation of a curve referred to the new axes retain-
ing their directions and origin is (4, 5) is x2 + y2 = 36. 10. Find the equation of a straight line which cuts off an
Find the equation referred to the original axes. intercept of 7 units on y-axis and has the slope 3.
48. At what point, the origin be shifted if the co-ordinates 11. Find the equation of a straight line which makes an
of a point (–1, 8) become (–7, 3) ? angle of 135° with the positive direction of x-axis and
49. If the axes are turned through 45°, find the transformed cuts an intercept of 5 units on the positive direction of
form of the equation 3x2 + 3y2 + 2xy = 2. y-axis.
50. Transform to parallel axes through the point (1, –2), the 12. Find the equation of a straight line which makes an
equation y2 – 4x + 4y + 8 = 0. Ê 3ˆ
angle of tan -1 Á ˜ with the positive direction of x-axis
51. If a point P(1, 2) is translated itself 2 units along the Ë 5¯
positive direction of x-axis and then it is rotated about and cuts an intercept of 6 units in the negative direction
the origin in anti-clockwise sense through an angle of of y-axis.
90°, find the new position of P. 13. Find the equation of a straight line which cuts off an
p
52. If the axes are rotated through , the equation intercept of 4 units from y-axis and are equally inclined
x2 + y2 = a2 is transformed to lxy +4 a2 = 0, find the with the axes.
value of l = 10. 14. Find the equations of bisectors of the angle between the
p
53. Transform to axes inclined at to the original axes, co-ordinate axes.
3 15. Find the equation of a line passing though the point
the equation x 2 + 2 3 y - y 2 = 2a 2 . (2, 3) and making an angle of 120° with the positive
54. What does the equation 2x2 +2xy + 3y2 – 18x – 22y + direction of x-axis.
50 = 0 become when referred to new rectangular axes 16. Find the equation of the right bisectors of the line join-
p
through the point (2, 3), the new set making with ing the points (1, 2) and (5, 7).
the old? 4 17. Find the equation of a line which passes through the
55. If a point P(2, 3) is rotated through an angle of 90° point (1, 2) and makes an angle q with the positive
about the origin in anti-clockwise sense, say at Q, find 3
direction of x-axis, where cos q = - .
the co-ordinates of Q. 5
56 If a point P(3, 4) is rotated through an angle of 60° 18. A line passes through the point A(2, 0) which makes an
about the point Q(2, 0) in anti-clockwise sense, say at angle of 30° with the positive direction of x-axis and is
R, find the co-ordinates of R. rotated about A in clockwise direction through an angle
of 15°. Find the equation of the straight line in the new
STRAIGHT LINE position.
19. Find the equation of a line passing through the points

LEVEL I (1, 2) and ( 3, 4).


20. The vertices of a triangle are A(10, 4), B(–4, 9) and
(Problems Based on Fundamentals) C(–2, –1). Find the equation of its altitude through A.
21. The vertices of a triangle are A(1, 2), B(2, 3) and
ABC OF STRAIGHT LINE
C(5, 4). Find the equation of its median through A.
1. Find the slope of the line PQ, where P(2, 4) and Q(3, 10). 22. Find the equation of the internal bisector of –BAC
2. Find the value of l, if 2 is slope of the line through (2, 5) of the DABC, whose vertices are A(5, 2), B(2, 3) and
and (l, 7). C(6, 5).

CG_01.indd 13 2/6/2017 3:57:45 PM


1.14 Coordinate Geometry Booster

23. A square is inscribed in a DABC, whose co-ordinates 2


are A(0, 0), B(2, 1) and C(3, 0). If two of its vertices lie section with the line x + y = 4 may be at a distance
3
on the side AC, find the vertices of the square. from the point (1, 2).
24. The line joining the points A(2, 0) and B(3, 1) is rotated 42. The centre of a square is at the origin and one vertex is
about A in the anti-clockwise direction through an angle P(2, 1). Find the co-ordinates of other vertices of the
of 15°. Find the equation of a line in the new position. square.
25. Find the equation of a line which passes through the 43. The extremities of a diagonal of a square are (1, 1) and
point (3, 4) and makes equal intercepts on the axes. (–2, –1). Find the other two vertices.
26. Find the equation of a line which passes through the 3p
44. A line through (2, 3) makes an angle with the
point (2, 3) and whose x-intercept is twice of y-intercept. 4
27. Find the equation of a line passes through the point negative direction of x-axis. Find the length of the line
(2, 3) so that the segment of the line intercepted be- segment cut off between (2, 3) and the line x + y = 7.
tween the axes is bisected at this point. 45. If the straight line drawn through the point P( 3, 2)
28. Find the equation of a line passing though the point p
and making an angle of with the x-axis meets the
(3, –4) and cutting off intercepts equal but of opposite 6
signs from the two axes. line 3 x - 4y + 8 = 0 at Q. Find the length of PQ.
29. Find the equation of a line passing through the point 46. Find the distance of the point (2, 3) from the line
(3, 2) and cuts off intercepts a and b on x- and y-axes 2x – 3y + 9 = 0 measured along the line 2x – 2y + 5 = 0.
such that a – b = 2. 47. Find the distance of the point (3, 5) from the line
30. Find the area of a triangle formed by lines x y = 0 and 2x + 3y = 14 measured parallel to the line x – 2y = 1
2x + 3y = 6. 48. Find the distance of the point (2, 5) from the line 3x + y
31. Find the equation of a straight line passing through the + 4 = 0 measured parallel to the line 3x – 4y + 8 = 0.
point (3, 4) so that the segment of the line intercepted 49. A line is drawn through A(4, –1) parallel to the line
between the axes is divided by the point in the ratio 2 : 3. 3x – 4y + 1 = 0. Find the co-ordinates of the two points
32. Find the equation of the straight line which passes on this line which are at a distance of 5 units from A.
through the origin and trisect the intercept of the line
REDUCTION OF A STRAIGHT LINE
3x + 4y = 12.
33. Find the area of a triangle formed by the lines xy – x – y 50. Reduce x + 3 y + 4 = 0 into the
+ 1 = 0 and 3x + 4y = 12. (i) slope intercept form and also find its slope and
34. A straight line cuts off intercepts from the axes of co- y-intercept.
ordinates, the sum of the reciprocals of which is a con- (ii) intercept form and also find the lengths of x and y
stant. Show that it always passes through a fixed point. intercepts.
35. The length of the perpendicular from the origin to a line (iii) normal form and also find the values of p and a.
is 5 and the line makes angle of 60° with the positive
direction of y-axis. Find the equation of the line. PARALLEL/PERPENDICULAR FORM OF A STRAIGHT LINE
36. Find the equation of the straight line upon which the 51. Find the location of the points (2, 2) and (3, 5) with
length of the perpendicular from the origin is 2 and the respect to the line
slope of the perpendicular is 5/12. (i) 2x + 3y + 4 = 0 (ii) 3x – 2y + 2 = 0
(iii) x + y – 7 = 0
DISTANCE FORM OF A STRAIGHT LINE 52. Determine whether the point (2, –7) lies on the origin
37. A line passing through the point (3, 2) and making an side of the line 2x + y + 2 = 0 or not.
angle q with the positive direction of x-axis such that 53. Write the parallel line to each of the following lines.
tan q = 3/4. Find the co-ordinates of the point on the (i) 3x – 4y + 10 = 0 (ii) 2x + 5y + 10 = 0
line that are 5 units away from the given point. (iii) –x + y – 2012 = 0 (iv) y = x
38. Find the co-ordinates of the points at a distance 4 2 (v) x = 5 (vi) y = 2013
units from the point (–2, 3) in the direction making an 54. Find the equation of a line parallel to 3x + 4y + 5 = 0
angle of 45° with the positive direction of x-axis. and passes through (2, 3).
55. Find the equation of a line parallel to 3x – 4y + 6 = 0
39. A point P(3, 4) moves in a plane in the direction of
Ÿ Ÿ and passing through the mid-point of of the line joining
i + j . Find the new position of P. the points (2, 3) and (4, –1).
40. A line joining two points A(2, 0) and B(3, 1) is rotated 56. Find the equation of a line passing through (2, 1) and
about A in anti-clockwise direction through an angle of parallel to the line joining the points (2, 3) and (3, –1).
15°. Find the new position of B. 57. Write the perpendicular line to each of the following
41. Find the direction in which a straight line must be lines.
drawn through the point (1, 2) so that its point of inter- (i) 5x – 3y + 2010 = 0 (ii) 3x + 5y + 2011 = 0

CG_01.indd 14 2/6/2017 3:57:46 PM


Straight Lines 1.15

(iii) –x + y – 2012 = 0 (iv) y = 2011x 78. If 4a2 + 9b2 – c2 + 12ab = 0, the family of the straight
(v) x = 2014 (vi) y = 2013 lines ax + by + c = 0 is either concurrent at (m, n) or
58. Find the equation of a line perpendicular to 2x + 3y – ( p, q). Find the value of m + n + p + q + 10.
2012 = 0 and passing through (3, 4). 79. The family of lines x(a + 2b) + y(a – 3b) = a – b passes
59. Find the equation of a line perpendicular to 2x – 3y – 5 through a fixed point for all values of a and b. Find the
= 0 and cutting an intercept 1 on the x-axis. co-ordinates of the fixed point.
60. Find the equation of the right bisectors of the line 80. Find the equation of a line passing through the point
joining the points (1, 2) and (3, 5). of intersection of 2x + 3y + 1 = 0, 3x – 5y – 5 = 0 and
61. Find the equation of the altitude through A of DABC, equally inclined to the axes.
whose vertices are A(1, 2), B(4, 5) and C(3, 8). 81. Find the slope of the lines which make an angle of 45°
62. Find the equation of a line passing through the point of with the line 3x – y + 5 = 0.
intersection of the lines 2x + y = 8 and x – y = 10 and is
perpendicular to 3x + 4y + 2012 = 0. EQUATION OF STRAIGHT LINES PASSING THROUGH A GIVEN
POINT AND MAKING A GIVEN ANGLE WITH A GIVEN LINE
63. Find the distance of the point (4, 5) from the straight
line 3x – 5y + 7 = 0. 82. Find the equations of the lines through the
64. The equation of the base of an equilateral triangle be The line makes an angle 45° with the line x – 2y = 3.
x + y = 2 and one vertex is (2, –1). Find the length of the 83. A vertex of an equilateral triangle is (2, 3) and the equa-
sides of the triangle. tion of the opposite side x + y = 2. Find the equation of
65. If a and b be the intercepts of a straight line on the x and the other sides of the triangle.
y axes, respectively and p be the length of the perpen- 84. A line 4x + y = 1 through the point A(2, –7) meets the
1 1 1 line BC, whose equation is 3x – 4y + 1 = 0 at the point
dicular from the origin, prove that 2 + 2 = 2 . B. Find the equation of the line AC so that AB = AC.
a b p
66. Find the distance between the lines 3x – 4y = 5 and 85. Find the equations of straight lines passing through
6x – 8y + 11 = 0. (–2, –7) and having an intercept of length 3 between
67. Let L has intercepts a and b on the co-ordinate axes. the straight lines 4x + 3y = 12 and 4x + 3y = 3.
When the axes are rotated through an angle, keeping 86. Find the equations of the lines passing through the
the origin fixed, the same line L has intercepts p and q, point (2, 3) and equally inclined to the lines 3x – 4y = 7
1 1 1 1 and 12x – 5y + 6 = 0.
prove that 2 + 2 = 2 + 2 . 87. Two straight lines 3x + 4y = 5 and 4x – 3y = 15 intersect
a b p q at A. Points B and C are chosen on these lines such that
ARIA OF A PARALLELOGRAM AB = AC. Determine the possible equations of the line
BC passing through the point (1, 2).
68. If the area of a parallelogram formed by the lines x + 3y 88. Two equal sides of an isosceles triangle have the equations
= a, 3x – 2y + 3a = 0, x + 3y + 4a = 0 and 3x – 2y + 7a 7x – y + 3 = 0 and x + y = 3 and its third side passes through
= 0 is ma2, find the value of 11m + 30. the point (1, –10). Find the equation of the third side.
69. Prove that the four lines ax ± by ± c = 0 enclose a rhom-
2c 2 EQUATIONS OF BISECTORS
bus, whose area is .
|ab| 89. Find the equations of the bisectors of the angle between
the straight lines 3x – 4y + 7 = 0 and 12x + 5y – 2 = 0.
FAMILY OF STRAIGHT LINES 90. Find the equation of the bisectors bisecting the angle
70. Find the point of intersection of the lines x – y + 4 = 0 containing the origin of the straight lines 4x + 3y = 6
and 2x + y = 10. and 5x + 12y + 9 = 0.
71. Prove that the three lines 2x – 3y + 5 = 0, 3x + 4y = 7 91. Find the bisector of the angle between the lines 2x + y
and 9x – 5y + 8 = 0 are concurrent. = 6 and 2x – 4y + 7 = 0, which contains the point (1, 2).
72. Find the equation of a line which is passing through 92. Find the equation of the bisector of the obtuse angle
the point of intersection of the lines x + 3y – 8 = 0, between the lines 3x – 4y + 7 = 0 and 12x + 5y = 2.
2x + 3y + 5 = 0 and (1, 2). 93. Find the bisector of the acute angle between the lines
73. Find the value of m so that the lines y = x + 1, 2x + y = x + y = 3 and 7x – y + 5 = 0.
16 and y = mx – 4 may be concurrent. 94. Prove that the length of the perpendiculars drawn
74. If the lines ax + y + 1 = 0, x + by + 1 = 0 and x + y + c from any point of the line 7x – 9y + 10 = 0 to the lines
= 0 (where a, b, c are distinct and different from 1) are 3x + 4y = 5 and 12x + 5y = 7 are the same.
1 1 1 95. Find the co-ordinates of the incentre of the triangle
concurrent, find the value of + + .
1- a 1- b 1- c whose sides are x + 1 = 0, 3x – 4y = 5, 5x + 12y = 27.
75 If 2a + 3b + 6c = 0, the family of straight lines 96. The bisectors of the angle between the lines
ax + by + c = 0 passes through a fixed point. Find the y = 3 x + 3 and 3 y = x + 3 3 meet the x-axis re-
co-ordinates of the fixed point. spectively, at P and Q. Find the length of PQ.

CG_01.indd 15 2/6/2017 3:57:46 PM


1.16 Coordinate Geometry Booster

97. Two opposite sides of a rhombus are x + y = 1 and x + y 4. If a and b are real numbers between 0 and 1 such that
= 5. If one vertex is (2, –1) and the angle at that vertex the points (a, 1), (1, b) and (0, 0) form an equilateral
be 45°. Find the vertex opposite to the given vertex. triangle, then a, b are
98. Find the foot of perpendicular drawn from the point (a) 2 — 3, 2 - 3 (b) 3 - 1, 3 - 1
(2, 3) to the line y = 3x + 4. (c) 2 - 1, 2 - 1 (d) None
IMAGE OF A POINT WITH RESPECT TO A STRAIGHT LINE 5. If O be the origin and Q1(x1, y1) and Q2(x2, y2) be two
points, then OQ1OQ2 cos (–Q1OQ2) is
99. Find the image of the point (–8, 12) with respect to the
(a) x1 y2 + x2 y1 (b) (x12 + y12) (x22 + y22)
line mirror 4x + 7y + 13 = 0.
100. Find the image of the point (3, 4) with respect to the (c) (x1 – x2) + (y1 – y2) (d) x1x2 + y1y2
line y = x. 6. If the sides of a triangle are 3x + 4y, 4x + 3 and 5x + 5y
101. If (–2, 6) be the image of the point (4, 2) with respect to units, where x > 0, y > 0, the triangle is
the line L = 0, find the equation of the line L. (a) right angled (b) acute angled
102. The image of the point (4, 1) with respect to the line y = x (c) obtuse angled (d) isosceles
is P. If the point P is translated about the line x = 2, the 7. A triangle is formed by the co-ordinates (0, 0), (0, 21) and
new position of P is Q. Find the co-ordinates of Q. (21, 0). Find the number of integral co-ordinates strictly
103. The image of the point (3, 2) with respect to the line x = 4 inside the triangle (integral co-ordinates of both x and y)
p (a) 190 (b) 105 (c) 231 (d) 205
is P. If P is rotated through an angle about the origin 8. The set of all real numbers x, such that x2 + 2x, 2x + 3
4
in anti-clockwise direction. Find the new position of P. and x2 + 3x + 8 are the sides of a triangle, is
104. The equations of the perpendicular bisectors of the (a) x x x x
sides AB and AC of DABC are x – y + 5 = 0 and x + 2y 9. The area of a triangle with vertices at the points
= 0, respectively. If the point A is (1, –2), find the equa- (a, b + c), (b, c + a) and (c, a + b) is
tion of the line BC. (a) 0 (b) a + b + c
(c) ab + bc + ca (d) none
REFLECTION OF A STRAIGHT LINE
10. If the vertices of a triangle ABC are (l, 2 – 2l), (–l + 1,
105. A ray of light is sent along the line x – 2y = 3. Upon 2l) and (–4 – l, 6 – 2l). If its area be 70 sq. units, the
reaching the line 3x – 2y = 5, the ray is reflected from it. number of integral values of l is
Find the equation of the line containing the reflected ray. (a) 1 (b) 2 (c) 3 (d) 4
106. A ray of light passing through the point (1, 2) is re- 11. If the co-ordinates of points A, B, C and D are (6, 3),
flected on the x-axis at a point P and passes through the (–3, 5), (4, –2) and (x, 3x), respectively and if
point (5, 3). Find the abscissa of the point P. DABC 1
107. A ray of light is travelling along the line x = 1 and gets = , then x is
DDBC 2
reflected from the line x + y = 1, find the equation of the
(a) 8/11 (b) 11/8 (c) 7/9 (d) 0
line which the reflected ray travel.
12. The area of a triangle is 5 and two of its vertices are
108. A ray of light is sent along the line x – 6 y = 8. After re-
A(2, 1), B(3, –2). The third vertex which lies on the line
fracting across the line x + y = 1, it enters the opposite side
y = x + 3 is
after turning by 15° away from the line x + y = 1. Find the
Ê 7 13 ˆ Ê 5 5ˆ
equation of the line along with the reflected ray travels. (a) Á , ˜ (b) Á , ˜
Ë2 2¯ Ë 2 2¯

LEVEL II Ê 3 3ˆ
(c) Á - , ˜
Ë 2 2¯
(d) (0, 0)
(Mixed Problems) 13. If the points (2k, k), ( k, 2k) and (k, k) with k > 0 en-
1. If P(1, 2), Q(4, 6), R(5, 7) and S(a, b) are the vertices of closes a triangle of area 18 sq. units, the centroid of the
a parallelogram PQRS, then triangle is equal to
(a) a = 2, b = 4 (b) a = 3, b= 4 (a) (8, 8) (b) (4, 4)
(c) a = 2, b = 3 (d) a = 3, b = 5 (c) (–4, –4) (d) (4 2, 4 3)
2. The extremities of the diagonal of a parallelogram are 14. If r be the geometric mean of p and q, the line px + qy
the points (3, –4) and (–6, 5). Third vertex is the point +r=0
(–2, 1), the fourth vertex is (a) has a fixed direction
(a) (1, 1) (b) (1, 0) (b) passes through a fixed point
(c) (0, 1) (d) (–1, 0) (c) forms with the axes of a triangle of
3. The centroid of a triangle is (2, 3) and two of its verti- (d) sum of its intercepts on the axes Constant area is
ces are (5, 6) and (–1, 4). Then the third vertex of the constant.
triangle is 15. A line passing through the point (2, 2) cuts the axes of
(a) (2, 1) (b) (2, –1) co-ordinates at A and B such that area OAB = k (k > 0).
(c) (1, 2) (d) (1, –2) The intercepts on the axes are the roots of the equation

CG_01.indd 16 2/6/2017 3:57:46 PM


Straight Lines 1.17

(a) x2 – kx + 2k = 0 (b) x2 – 2kx + k =0 30. The point P(a, b) and Q(b, a) lie on the lines 3x + 2y –
2
(c) x + kx+ 2k = 0 (d) x2 + kx + k = 0 13 = 0 and 4x – y – 5 = 0. The equation of the line PQ is
16. If A and B be two points on the line 3x + 4 y + 15 = 0 such (a) x – y = 5 (b) x + y = 5
that OA = OB = 9 units, the area of the triangle OAB is (c) x – y = –5 (d) x + y = –5
(a) 9 2 (b) 18 2 (c) 12 2 (d) None. 31. The sides AB, BC, CD and DA of a quadrilateral are
x + 2y = 3, x = 1, x – 3y = 4, 5x + y + 12 = 0 respectively.
17. The line segment joining the points (1, 2) and (–2, 1) is
The angle between diagonals AC and BD is
divided by the line 3x + 4y = 7 in the ratio
(a) 45° (b) 60° (c) 90° (d) 30°
(a) 3 : 4 (b) 4 : 3 (c) 9 : 4 (d) 4 : 9
32. Let PS be the median of the triangle with vertices
18. If a straight line passes through (x1, y1) and its segment
P(2, 2), Q(6, –1) and R (7, 3). The equation of the line
between the axes is bisected at this point, its equation
passing through (1, –1) and parallel to PS is
is given by
x y (a) 2x – 9y – 7 = 0 (b) 2x – 9y – 11 = 0
(a) + =2 (b) 2(xy1 + x1 y) = x1 y1 (c) 2x + 9y – 11 = 0 (d) 2x + 9y + 7 = 0
x1 y1
(c) xy1 + x1 y = x1 y1 (d) None. 33. The equation of the base of an equilateral triangle is
x + y = 2 and the vertex is (2, –1). The length of its side is
19. A straight line through the point P(3, 4) is such that its
intercept between the axes is bisected at P. Its equation is 1 3 2
(a) (b) (c) (d) 2
(a) 3x – 4y + 7 = 0 (b) 4x + 3y = 24 2 2 3
(c) 3x + 4y = 25 (d) x + y = 7 34. The distance between the lines 4x + 3y = 11 and 8x + 6y
20. If the lines 4x + 3y = 1, y = x + 5 and bx + 5y = 3 are = 15 is
concurrent, then b is (a) 7/2 (b) 7/3 (c) 7/5 (d) 7/10
(a) 1 (b) 3 (c) 6 (d) 0 Ê l l ˆ
35. A variable point Á1 + ,2+ ˜ lies in between
21. The lines ax + by + c = 0, bx + cy + a = 0 and cx + ay + Ë 2 2¯
b = 0 are concurrent if two parallel lines x + 2y = 1 and 2x + 4y = 15, the range
(a) Sa3 = 3abc (b) Sa3 = 0 of l is given by
(c) Sa = Sab
2
(d) None 5 2 4 2 5 2
22. The lines ax + 2y + 1 = 0, bx + 3y + 1 = 0 and cx + 4y + (a) 0 < l < (b) - <l<
6 5 6
1 = 0 are concurrent if a, b, c are in
4 2
(a) AP (b) GP (c) HP (d) none (c) - <l<0 (d) none
23. If the lines ax + y + 1 = 0, x + by + 1 = 0 and x + y + c = 5
36. The sum of the abcissae of all the points on the line
0 (a, b, c are distinct and not equal to 1) are concurrent, x + y = 4 that lie at a unit distance from the line 4x +
1 1 1 3y – 10 = 0 is
the value of + + is
1- a 1- b 1- c (a) – 4 (b) –3 (c) 3 (d) 4
(a) 0 (b) 1 (c) 2 (d) none 37. If the algebraic sum of the perpendicular distances from
24. The points (–a, –b), (0, 0), (a, b) and (a2, ab) are the points (2, 0), (0, 2), (1,1) to a variable straight line
(a) collinear (b) vertices of a rectangle be zero, the line passes through a fixed point whose
(c) vertices of a (d) none co-ordinates are
parallelogram (a) (1, 1) (b) (2, 2) (c) (0, 0) (d) None
25. If 25p2 + 9q2 – r2 – 30pq = 0, a point on the line px + qy 38. If a, b and c are related by 4a2 + 9b2 – 9c2 + 12ab = 0,
+ r = 0 is the greatest distance between any two lines of the fam-
(a) (–5, 3) (b) (1, 2) (c) (0, 0) (d) (5, 3) ily of lines ax + by + c = 0 is
2
26. The set of lines ax + by + c = 0 where 3a + 2b + 4c = 0 (a) 4/3 (b) ¥ 13
are concurrent at the point 3
(a) (3, 2) (b) (2, 4) (c) (3/4, ½) (d) None. (c) 3 13 (d) 0
27. If a, b, c are in AP, the straight line a + by + c = 0 will 39. If the axes are turned through an angle tan–1 2, the equa-
always pass through the point tion 4xy – 3x2 = a2 becomes
(a) (1, 1) (b) (2, 2) (c) (–2, 1) (d) (1, –2) (a) x2 – 4y2 = 2a2 (b) x2 – 4y2 = a2
2 2 2
28. The equation of the line which passes through the point (c) x + 4y = a (d) x2 – 2xy2 = a2
(–3, 8) and cuts off positive intercepts on the axes 40. The number of integral values of m for which the x co-
whose sum is 7, is ordinate of the point of intersection of the lines 3x + 4y
(a) 3x – 4y = 12 (b) 4x + 3y = 12 = 9 and y = mx + 1 is also an integer, is
(c) 3x + 4y = 12 (d) 4x – 3y = 12 (a) 2 (b) 0 (c) 4 (d) 1
29. If a pair of opposite vertices of parallelogram are (1, 3) 41. Given the family of lines
and (–2, 4) and the sides are parallel to 5x – y = 0 and a(2x + y + 4) + b(x – 2y – 3) = 0.
7x + y = 0, the equation of a side through (1, 3) is The number of lines belonging to the family at a dis-
(a) 5x – y = 2 (b) 7x + y = 10 tance 10 from any point (2, –3) is
(c) 5x – y + 14 = 0 (d) 7x + y + 10 = 0 (a) 0 (b) 1 (c) 2 (d) 4

CG_01.indd 17 2/6/2017 3:57:47 PM


1.18 Coordinate Geometry Booster

x y 53. The incentre of the triangle formed by the axes and the
42. If + = 1 be any line through the intersection of x y
c d line + = 1 is
x y x y a b
+ = 1 and + = 1 , then Ê a bˆ Ê a bˆ
a b b a (a) Á , ˜ (b) Á , ˜
Ë 2 2¯ Ë 3 3¯
1 1 1 1 1 1 1 1
(a) + = + (b) + = + Ê ab ab ˆ
c d a b a d c b (c) ,
Á 2 2 2 2˜
1 1 1 1 Ëa+b+ a +b a+b+ a +b ¯
(c) + = + (d) None.
b d a c Ê ab ab ˆ
x y (d) Á ,
43. The point of intersection of the lines + = 1 and Ë a + b + ab a + b + ab ˜¯
x y a b
+ = 1 lies on the line 54. The orthocentre of a triangle whose vertices are (0, 0),
b a (3, 4), (4, 0) is
(a) x – y = 0 7 Ê 5ˆ
(b) (x + y) (a + b) = 2ab (a) ÊÁ 3, ˆ˜ (b) Á 3, ˜
Ë 3¯ Ë 4¯
(c) (px + qy) (a + b) = (p + q)ab
Ê 3ˆ
(d) (px – qy) (a – b) = (p – q)ab (c) (5, –2) (d) ÁË 3, ˜¯
44. The equation of the right bisector of the line segment 4
joining the points (7, 4) and (–1, –2) is 55. The orthocentre of the triangle formed by the lines xy =
(a) 4x + 3y = 10 (b) 3x – 4y + 7 = 0 0 and x + y = 1 is
(c) 4x + 3y = 15 (d) none Ê 1 1ˆ Ê 1 1ˆ
(a) Á , ˜ (b) Á , ˜
45. The points (1, 3) and (5, 1) are two opposite vertices of Ë 2 2¯ Ë 3 3¯
a rectangle. The other two vertices lie on the line y = 2x Ê 1 1ˆ
+ c, the other vertices and c are (c) (0, 0) (d) ÁË , ˜¯
4 4
(a) (1, 1), (2, 3), c = 4 (b) (4, 4), (2, 0), c = –4 56. The mid-points of the sides of a triangle are (5, 0),
(c) (0, 0), ( 5, 4), c = 3 (d) none (5, 12) and (0, 12). The orthocentre of the triangle is
46. The four lines ax ± by ± c = 0 enclose a (a) (0, 0) (b) (10, 0)
(a) square
Ê 13 ˆ
(b) parallelogram (c) (0, 24) (d) Á , 8˜
Ë3 ¯
(c) rectangle
2c 2 57. One side of an equilateral triangle is the line 3x + 4y +
(d) rhombus of area
ab 8 = 0 and its centroid is at O(1, 1). The length of its side
47. The area bounded by the curves y = |x| – 1 and y = is
–|x| + 1 is (a) 2 (b) 5 (c) 6 3 (d) 7
(a) 1 (b) 2 (c) 2 2 (d) 4 58. The incentre of the triangle with vertices (1, 3) , (0, 0)
48. The area of the parallelogram formed by the lines and (2, 0) is
y = mx, y = mx + 1, y = nx and y = nx + 1 is
Ê 3ˆ Ê2 1 ˆ
| m + n| 2 (a) Á1, (b) Á ,
(a) (b) Ë 2 ¯ ˜ Ë 3 3 ˜¯
2
( m - n) | m + n|
Ê 2 3ˆ Ê 1 ˆ
1 1 (c) Á , (d) Á1,
(c) (d) Ë 3 2 ˜¯ Ë ˜

| m + n| | m - n|
59. Let P (–1, 0), Q(0, 0) and R (3, 3 3) be three points.
49. The line which is parallel to x-axis and crosses the
Then the equation of the bisector of the –PQR is
curve y = x at an angle of 45° is
3
(a) x = ¼ (b) y = ¼ (c) y = ½ (d) y = 1 (a) x+ y=0 (b) x + 3 y = 0
50. The reflection of the point (4, –13) in the line 5x + y + 2
3
6 = 0 is (c) y + x 3 = 0 (d) x + y=0
(a) (–1, –14) (b) (3, 4) 2
(c) (1, 2) (d) (–4, 13) 60. The vertices of a triangle ABC are (1, 1), (4, –2) and
51. The area enclosed within the curve |x| + |y| = 1 is (5, 5), respectively. The equation of the perpendicular
(a) 4 (b 2 (c) 1 (d) 3 dropped from C to the internal bisector of –A is
52. The incentre of the triangle formed by the lines x = 0, (a) y = 5 (b) x = 5
y = 0 and 3x + 4y = 12 is (c) 2x + 3y = 7 (d) none
61. The vertices of a triangle are A(–1, –7), B(5, 1) and
Ê 1 1ˆ Ê 1ˆ Ê1 ˆ
(a) Á , ˜ (b) (1, 1) (c) Á1, ˜ (d) Á , 1˜ C (1, 4). The equation of the internal bisector of the
Ë 2 2¯ Ë 2¯ Ë2 ¯ –ABC is

CG_01.indd 18 2/6/2017 3:57:48 PM


Straight Lines 1.19

(a) 3x – 7y = 8 (b) x – 7y + 2 = 0 5. The area of the triangle formed by the intersection of a


(c) 3x – 3y – 7 = 0 (d) none line parallel to x-axis and passing through P(h, k) with
62. The bisector of the acute angle formed between the the lines y = x and x + y = 2 is 4h2. Find the locus of the
lines 4x – 3y + 7 = 0 and 4x – 4y + 1 4 = 0 has the equa- point P.
tion 6. Two rays in the first quadrant x + y = |a| and ax – y = 1
(a) x + y = 7 (b) x Рy + 3 = 0 intersects each other in the interval a Π(a0, ). Find the
(c) 2x + y =11 (d) x + 2y = 12 value of a0.
63. The opposite angular points of a square are (3, 4) and 7. A variable line is at constant distance p from the origin
(1, –1), the other two vertices are and meets the co-ordinate axes in A and B. Show that
Ê 1 5ˆ Ê 9 1ˆ Ê 9 1ˆ Ê 7 1ˆ 1 1 9
(a) Á - , ˜ , Á , ˜ (b) Á , ˜ , Á , ˜ the locus of the centroid of the DOAB is 2 + 2 = 2 .
Ë 2 2¯ Ë 2 2¯ Ë 2 2¯ Ë 2 2¯ x y p
8. The line segment joining A(3, 0) and B(5, 2) is rotated
Ê 7 1ˆ Ê 7 1ˆ Ê 7 1ˆ Ê 7 9ˆ
(c) Á , ˜ , Á - , - ˜ (d) Á - , - ˜ , Á - , - ˜ about A in the anti-clockwise direction through an an-
Ë 2 2¯ Ë 2 2¯ Ë 2 2¯ Ë 2 2¯
gle of 45° so that B goes to C. If D(x, y) is the image of
64. A line through A(–5, –4) meets the lines x + 3y + 2 = 0, C with respect to y-axis, find the value of x + y + 7.
2x + y + 4 = 0 and x – y – 5 = 0 at B, C and D, respec- 9. Find the equation of the line passing through the point
2 2 2 (4, 5) and equally inclined to the lines 3x – 4y = 7 and
Ê 15 ˆ Ê 10 ˆ Ê 6 ˆ
tively. If Á + =
Ë AB ˜¯ ËÁ AC ¯˜ ËÁ AD ¯˜
, the equation of 5y – 12x = 6.
10. If A(3, 0) and C(–2, 5) be the opposite vertices of a
the line is square, find the co-ordinates of remaining two vertices.
(a) 2x + 3y + 22 = 0 (b) 5x – 4y + 7=0 11. For what values of the parameter m does the point
(c) 3x – 2y + 3 = 0 (d) none P(m, m + 1) lie within the DABC, the vertices of which
65. The equation of the lines through the point (2, 3) and are A(0, 3), B(–2, 0) and C(6,1)?
making an intercept of length 2 units between the lines 12. A straight line passes through the point (h, k) and this
2x + y = 3 and 2x + y = 5 are point bisects the part of the intercept between the axes.
(a) x + 3 = 0, 3x + 4y = 12 Show that the equation of the straight line is
(b) y – 2 = 0, 4x – 3y = 6 x y
(c) x – 2 = 0, 3x + 4y = 18 + =1
2h 2k
(d) none
66. A line is such that its segment between the straight lines 13. Find the values of the parameter m for which the points
5x – y = 4 and 3x + 4y = 4 is bisected at the point (1, 5). (0, 0) and (m, 3) lie on the opposite lines 3x + 2y – 6 =
Its equation is 0 and x – 4y + 16 = 0.
(a) 23x – 7y + 6 = 0 (b) 7x + 4y + 3 = 0 14. If A(0, 3) and B(–2, 5) be the adjacent vertices of a
(c) 83x – 35y + 92 = 0 (d) None square. Find the possible co-ordinates of remaining
67. A straight line through the origin O meets the parallel two vertices.
lines 4x + 2y = 9 and 2x + y + 6 = 0 at points P and Q, 15. Find the co-ordinates of two points on the line x + y
respectively. Then the point O divides the segment PQ = 3 which are situated at a distance 8 from the point
in the ratio (2, 1) on the line.
(a) 1 : 2 (b) 3 : 4 (c) 2 : 1 (d) 4 : 3 16. If a and b be variables, show that the lines (a + b)x +
(2a – b)y = 0 pass through a fixed point.
17. Determine all values of a for which the point (a, a2)
LEVEL III lies inside the triangle formed by the lines 2x + 3y –1 =
0, x + 2y – 3 = 0 and 5x – 6y – 1 = 0.
(Problems for JEE Advanced)
18. The points (1, 3) and (5, 1) are two opposite vertices of
1. Derive the conditions to be imposed on b so that (0, b) a rectangle. The other two vertices lie on the line y = 2x
should lie on or inside the triangle having sides y + 3x + c. Find c and the remaining two vertices.
+ 2 = 0, 3y – 2x –5 = 0 and 4y + x – 14 = 0 19. A vertex of an equilateral triangle is (2, 3) and the equa-
2. Find the number of integral values of m, for which the tion of the opposite side is x + y = 2. Find the equation
x-co-ordinate of the point of intersection of the lines of the other two sides.
3x + 4y = 9 and y = mx + 1 is also an integer. 20. Two consecutive sides of a parallelogram are 4x + 5y
= 0 and 7x + 2y = 0. If the equation to one diagonal is
3. Let P = (–1, 0), Q = (0, 0) and R = (3, 3 3) be three
11x + 7y = 9, find the equation of the other diagonal.
points. Find the equation of the bisector of the –PQR.
21. A ray of light coming from the point (1, 2) is reflected
4. A straight line through the point (2, 2) intersects the lines
at a point A on the x-axis and then passes through the
3 x + y = 0 and 3 x - y = 0 at the points A and B. Find point (5, 3). Find the point A.
the equation to the line AB so that the DOAB is equilateral.

CG_01.indd 19 2/6/2017 3:57:48 PM


1.20 Coordinate Geometry Booster

22. A man starts from the point P(–3, 4) and reaches the 2 Let the point B is symmetric to A (4, –1) with respect to
point Q(0, 1) after touching the line 2x + y = 7 at R. the bisector of the first quadrant. Find AB.
Find R on the line so that he travels along the shortest 3 A line segment AB through the point A(2, 0) which
path. makes an angle of 30° with the positive direction of
2
23. A ray of light is sent along the straight line y = x - 4. x-axis is rotated about A in anti-clockwise direction
3 through an angle of 15°. If C be the new position of the
On reaching the x-axis, it is reflected. Find the point of
incidence and the equation of the reflected ray. point B(2 + 3, 1) , find the co-ordinates of C.
24. If the point (a, a) is placed in between the lines |x + y| = 4. The point (1, –2) is reflected in the x-axis and then
4, find a. translated parallel to the positive direction of x-axis
25. The equations of two sides of a triangle are 3x – 2y + 6 through a distance of 3 units, find the co-ordinates of
= 0 and 4x + 5y = 20 and the orthocentre is (1, 1). Find the point in the new position.
the equation of the third side. 5. A line through the point A(2, 0) which makes an angle
26. Two vertices of a triangle are (4, –3) and (–2, 5) . If the of 30° with the positive direction of x-axis is rotated
orthocentre of the triangle is (1, 2), prove that the third about A in anti-clockwise direction through an angle
vertex is (33, 26). of 15°. Find the equation of the straight line in the new
27. The equations of the perpendicular bisectors of the position.
sides AB and AC of a DABC are x – y + 2 = 0 and x + 6. A line x – y + 1 = 0 cuts the y-axis at A. This line is
2y = 0, respectively. If the point A is (1, –2), find the rotated about A in the clockwise direction through 75°.
equation of the line BC. Find the equation of the line in the new position.
28. A line 4x + y = 1 through the point A(2, –7) meets the 7. The point (1, 1) is translated parallel to the line y = 2x
line BC whose equation is 3x – 4y + 1 = 0 at the point in the first quadrant through a unit distance. Find the
B. Find the equation of the line AC, so that AB = AC. new position of the point.
29. Find the equations of the straight lines passing through 8. Two particles start from the same point (2, –1), one
(–2, –7) and having an intercept of length 3 between moving 2 units along the line x + y = 1 and the other
the straight lines 4x + 3y = 12 and 8x + 6y = 6. 5 units along the line x – 2y = 4. If the particles move
30. If A(1, p2), B(0, 1), C(p, 0) are the co-ordinates of three towards increasing y, find their new positions and the
points, find the value of p for which the area of the distance between them.
DABC is minimum. 9. If a line AB of length 2l moves with the end A always
31. The straight lines 3x + 4y = 5 and 4x – 3y = 15 intersect on the x-axis and the end B always on the line y = 6x.
at A. Points B and C are chosen on these lines, such that Find the equation of the locus of the mid-point of AB.
AB = AC. Determine the possible equations of the line 10 The opposite angular points of a square are (3, 4) and
BC passing through the point (1, 2). (1, –1). Find the co-ordinates of the other two vertices.
32. The centre of a square is at the origin and one vertex [Roorkee, 1985]
is A(2, 1). Find the co-ordinates of other vertices of the 11. Two vertices of a triangle are (4, –3) and (–2, 5). If
square. the orthocentre of the triangle is at (1, 2). Find the co-
33. Two equal sides of an isosceles triangle are 7x – y + 3 = ordinates of the third vertex. [Roorkee, 1987]
0 and x + y – 3 = 0 and its third side passes through the 12. A line is such that its segment between the straight lines
point (1, –10). Find the equation of the third side. 5x – y – 4 = 0 and 3x + 4y – 4 = 0 is bisected at the point
34. Two opposite sides of a rhombus are x + y = 1 and (1, 5). Obtain its equation. [Roorkee, 1988]
x + y = 5. If one vertex is (2, –1) and the angle at the ver- 13. The extremities of the diagonal of a square are (1, 1)
tex is 45°. Find the vertex opposite to the given vertex. and (–2, –1). Obtain the other two vertices and the
35. Two sides of a rhombus ABCD are parallel to the lines equation of the other diagonal. [Roorkee, 1989]
y = x + 2 and y = 7x + 3. If the diagonals of the rhombus 14 A variable straight line passes through the point of in-
intersect at the point (1, 2) and the vertex A is on y-axis. tersection of the lines x + 2y = 1 and 2x – y = 1 and
Find the possible co-ordinates of A. meets the co-ordinate axes in A and B. Find the locus
of the mid-point of AB. [Roorkee, 1989]
15. A variable line, drawn through the point of intersection
LEVEL IV x y x y
of the straight lines + = 1 and + = 1 , meets
a b b a
(Tougher Problems for JEE
Advanced) the co-ordinates axes in A and B. Find the locus of the
mid-point of AB. [Roorkee, 1989]
1. The equation of two sides of a parallelogram are 16 A line joining A(2, 0) and B(3, 1) is rotated about A in
3x – 2y + 12 = 0 and x – 3y + 11 = 0 and the point of anti-clockwise direction through. If B goes to C in the
intersection of its diagonals is (2, 2). Find the equations new position, what will be the co-ordinates of C?
of other two sides and its diagonal. [Roorkee, 1989]

CG_01.indd 20 2/6/2017 3:57:49 PM


Straight Lines 1.21

17. Which pair of points lie on the same side of 3x – 8y – 7 26. Given vertices A(1, 1), B(4, –2) and C(5, 5) of a tri-
= 0? angle, find the equation of the perpendicular dropped
(a) (0, –1) and (0, 0) (b) (4, –3) and (0, 1) from C to the interior bisector of the –A.
(c) (–3, –4) and (1, 2) (d) (–1, –1) and (3, 7) [Roorkee Main,1994]
[Roorkee, 1990] 27. The co-ordinates of the foot of the perpendicular from
18 Determine the conditions to be imposed on b so that the point (2, 4) on the line x + y = 1 are
(0, b) should lie on or inside the triangle having sides (a) (1/2, 3/2) (b) (–1/2, 3/2)
y + 3x + 2 = 0, 3y – 2x – 5 = 0 and 4y + x – 14 = 0. (c) (4/3, ½) (d) (3/4, –1/2)
[Roorkee Main, 1990]
[Roorkee, 1995]
19. A ray of light is sent along the line x – 2 y – 3 = 0. Upon
28. All points lying outside the triangle formed by the
reaching the line 3x – 2y – 5 = 0, the ray is reflected
points (1, 3), (5, 0) and (–1, 2) satisfy
from it. Find the equation of the line containing the re-
flected ray. [Roorkee Main, 1990] (a) 2x + y x+y–
20. A line parallel to the straight line 3x – 4y – 2 = 0 and at (c) 2x + y x+y
a distance 4 units from it is [Roorkee, 1995]
(a) 3x – 4y + 30 = 0 (b) 4x – 3y + 12 = 0 29. In a DABC, the equation of the perpendicular bisector
(c) 3x – 4y + 18 = 0 (d) 3x – 4y + 22 = 0 of AC is 3x – 2y + 8 = 0. If the co-ordinates of the points
[Roorkee, 1991] A and B are (1, –1) and (3, 1), respectively, find the
21. The equation of a straight line passing through (–5, 4) equation of the line BC and the centre of the circum-
which cuts an intercept of 2 between the lines x + y circle of the DABC.
+ 1= 0 and x + y – 1= 0 is [Roorkee Main, 1995]
(a) x – 2y + 13 = 0 (b) 2x – y + 14 = 0 30. Two sides of a rhombus, lying in the first quadrant are
(c) x – y + 9 = 0 (d) x – y + 10 = 0 given by 3x – 4y = 0 and 12x – 5y = 0. If the length
[Roorkee, 1991] of the longer diagonal is 12, find the equations of the
22. P(3, 1), Q(6, 5) and R(x, y) are three points such that other two sides of the rhombus.
the –PRQ is a right angle and the area of DRPQ = 7, [Roorkee Main, 1996]
the number of such points R is 31. What is the equation of a straight line equally inclined
(a) 0 (b) 1 (c) 2 (d) infinite to the axes and equidistant from the points (1, –2) and
[Roorkee, 1992] (3, 4)? [Roorkee, 1997]
23. P is a point on either of the two lines y - 3| x | = 2 at 32. If the points (2a, a), (a, 2a) and (a, a) enclose a triangle
a distance of 5 units from their point of intersection, the of area 8, find the value of a.
co-ordinates of the foot of the perpendicular from P on (a) x – y – 2 = 0 (b) x + y – 2 = 0
the bisector of the angle between them are (c) x – y – 1 = 0 (d) x + y – 1 = 0
Ê 4 + 5 3ˆ Ê 4 - 5 3ˆ [Roorkee, 1997]
(a) Á 0, ˜ or Á 0, ˜ 33. One diagonal of a square 7x + 5y = 35 intercepted by
Ë 2 ¯ Ë 2 ¯
the axes. Obtain the extremities of the other diagonal.
depending on which line P is taken
[Roorkee Main,1997]
Ê 4 + 5 3ˆ 34. The equations of two equal sides AB and AC of an isos-
(b) Á 0, ˜
Ë 2 ¯ celes triangle ABC are x + y = 5 and 7x – y = 3, respec-
Ê 4 - 5 3ˆ tively. Find the equations of the side BC if the area of
(c) Á 0, ˜ the DABC is 5 sq. units.
Ë 2 ¯ [Roorkee, 1999]
Ê 5 5 3ˆ 35. Find the position of the point (4, 1) after it undergoes
(d) Á ,
Ë 2 2 ˜¯
[Roorkee, 1992] the following transformations successively:
(i) reflection about the line y = x – 1
24. If one of the diagonals of the square is along the line x (ii) translation by one unit along x-axis in the positive
= 2y and one of its vertices is A(3, 0), its sides through direction.
the vertex A are given by p
(iii) rotation through an angle about the origin in
(a) y + 3x + 9 = 0; 3y + x – 3 = 0 4
(b) y – 3x + 9 = 0; 3y + x – 3 = 0 the anti-clockwise direction.
(c) y – 3x + 9 = 0; 3y – x + 3 = 0 [Roorkee Main, 2000]
(d) y – 3x + 3 = 0; 3y + x + 9 = 0 [Roorkee, 1993] 36 Two vertices of a triangle are at (–1, 3) and (2, 5) and
25 The sides AB, BC, CD and DA of a quadrilateral have its orthocentre is at (1, 2). Find the co-ordinates of the
the equations x + 2y = 3, x = 1, x – 3y = 4 and 5x + y + third vertex.
12 = 0 respectively. Find the angle between the diago- [Roorkee Main, 2001]
nals AC and BD. [Roorkee Main, 1993]

CG_01.indd 21 2/6/2017 3:57:49 PM


1.22 Coordinate Geometry Booster

Integer Type Questions Êx +x y + y2 ˆ


(c) Á 1 2 cos w , 1 cos w ˜
Ë 2 2 ¯
1. Let the algebraic sum of the perpendicular distances
(d) none
from the points (3, 0), (0, 3) and (2, 2) to a variable
straight line be zero, the line passing through a fixed 3. If a line makes intercepts a and b on x and y axes (obvi-
point whose co-ordinates are (p, q), where 3(p + q) – 2 ously oblique axes), its equation be
is.... x y
(a) + =1
2. If the distance of the point (2, 3) from the line 2x – 3y + a cos w b cos w
9 = 0 measured along the line 2x – 2y + 5 = 0 is d 2, x y
(b) + =1
find (d + 2). a b
3. Find the number of possible straight lines passing x y
through (2, 3) and forming a triangle with co-ordinate (c) + =1
b cos w a cos w
axes, whose area is 12 sq. units.
(d) none
4. Find the number of integral values of m for which the
4. If axes are inclined at 45°, the radius of the circle x2 +
x-co-ordinate of the point of intersection of the lines
3x + 4y = 9 and y = mx + 2 is also an integer. xy + y2 – 4x – 5y – 2 = 0 is
5. Find the area of the parallelogram formed by the lines (a) 2 (b) 4 (c) 3 (d) 5
y = 2x, y = 2x + 1, y = x and y = x + 1. Passage II
6. If one side of a rhombus has end-points (4, 5) and (1, 1) Let us consider a rectangular co-ordinate system OX and OY
such that the maximum area of the rhombus is 5m sq. be rotated through an angle q in the anti-clockwise direction.
units, find m. Then we get a new co-ordinate system OX¢ and OY¢. If we
7. Find the area of a rhombus enclosed by the lines x ± 2y consider a point P(x, y) in the old co-ordinate system and
± 2 = 0. P(X, Y) in the new co-ordinate system, we can write
8. P(x, y) be a lattice point if x, y ΠN. If the number of x = X cosq РY sinq and y = X sinq + Y cosq.
lattices points lies inside of a triangle form by the line Then
x + y = 10 and the co-ordinate axes is m(m + 5), find m. 1. The value of X is
9. P(x, y) be an IIT point if x, y ΠI +. Find the number of (a) X cosq + Y sinq (b) X sinq + Y cosq
IIT points lying inside the quadrilateral formed by the (c) X cosq – Y sinq (d) X sinq – Y cosq.
lines 2x + y = 6, x + y = 9, x = 0 and y = 0.
2. The equation of a curve in a plane 17x2 – 16xy + 17y2
10. If the point P(a2, a) lies in the region corresponding to = 225.Through what angle must the axes be rotated, so
the acute angle between the lines 2y = x and 4y = x such that the equation becomes 9X 2 + 25Y 2 = 225?
that the value of a lies in (p, q), where p, q ΠN, find the
(a) 30° (b) 45° (c) 60° (d) 75°
value of (p + q + 1).
3. If the axes be rotated at 45°, the equation 17x2 –16xy
+ 17y2 = 225 reduces to AX2 + BY2 = C2, the value of
Comprehensive Link Passage
A + B + C is
Passage I (a) 59 (b) 50 (c) 58 (d) 57.
Sometimes we do not take x-axis and y-axis at 90° and as- 4. If the axes are rotated through 45°, the equation
sume that they are inclined at an angle w. Let OX and OY, 3x2 + 2xy + 3y2 = 2 reduces to
the x-axis and the y-axis respectively, are inclined at an angle (a) X 2 + 2Y 2 = 1 (b) 2X 2 + Y 2 = 1
w. Let P be a point on the plane. Draw parallel lines from P 2
(c) 2X + 3Y = 12
(d) 5X2 + 3Y 2 = 1.
parallel to y- and x-axis. We will write PN = x and PM = y and 2 2
5. The equation 4xy – 3x = a become when the axes are
will say that the co-ordinates of P are (x, y), where such axes turned through an angle tan–1 2 is
are called oblique axes. (a) x2 + 4y2 = a2 (b) x2 – 4y2 = a2
1. The distance of P(x, y) from origin must be 2 2
(c) 4x + y = a 2
(d) 4x2 – y2 = a2.
(a) x2 + y 2 (b) x 2 + y 2 - 2xy sin w Passage III
2 2 The equations of adjacent sides of a parallelogram are x + y +
(c) x + y - 2xy cos w (d) none
1 = 0 and 2x – y + 2 = 0. If the equation of one of its diagonal
2. If M(x1, y1) and N(x2, y2) be two points in oblique sys- is 13x – 2y – 32 = 0. Then the
tem, the co-ordinates of the mid-points of A and B are 1. equation of the diagonal must be
Ê x + x y + y2 ˆ
(a) Á 1 2 , 1 ˜ (a) 7x – 8y + 1 = 0 (b) 2x – y = 0
Ë 2 2 ¯ (c) 2x – y = 7 (d) 3x + 4y = 5
Êx +x y + y2 ˆ 2. area of the given parallelogram must be
(b) ÁË 1 2 sin w , 1 sin w ˜
¯
2 2 (a) 45 (b) 45/2 (c) 3 5 (d) 4 2

CG_01.indd 22 2/6/2017 3:57:50 PM


Straight Lines 1.23

3. equation of the side of the parallelogram opposite to let x = j(t) and y = y(t). Eliminating t, we get the equation of
the given side 2x – y + 2 = 0 must be the reflected curve S¢. Then
(a) 2x – y + 5 = 0 (b) 2x – y = 0 1. the image of the line 3x – y = 2 in the line y = x – 1 is
(c) 2x – y = 7 (d) x + 3y = 4 (a) x + 3y = 2 (b) 3x + y = 2
(c) x – 3y = 2 (d) x + y = 2
Passage IV
2. the image of the circle x2 + y2 = 4 in the line x + y = 2 is
The vertex C of a right-angled isosceles DABC is (2, 2) and (a) x2 + y2 – 2x – 2y = 0
the equation of the hypotenuse AB is 3x + 4y = 4. Then (b) x2 + y2 – 4x – 4y + 6 = 0
1. the equations of the sides AC and AB must be (c) x2 + y2 – 2x – 2y + 2 = 0
(a) 7y – x = 12, 7x + y = 16 (d) x2 + y2 – 4x – 4y + 4 = 0
(b) 3x – 4y + 2 = 0, 4x + 3y = 14 3. the image of the parabola x2 = 4y in the line x + y = a is
(c) x + y = 4, 2x – 3y = 10 (a) (x – a)2 = –4(y – a) (b) (y – a)2 = –4(x – a)
2
(d) x – y = 2, 3x + 2y = 5. (c) (x – a) = 4(y + a) (d) (y – a)2 = 4(x + a)
2. the area of the DABC must be 4. the image of an ellipse 9x2 + 16y2 = 144 in the line
(a) 1 sq. units (b) 2 sq. units y = x is
(a) 16x2 + 9y2 = 144 (b) 9x2 + 16y2 = 144
(c) 2 2 sq. units (d) 4 sq. units (c) 16x2 + 25y2 = 400 (d) 25x2 + 16y2 = 400
3. The in-radius of the DABC must be 5. the image of the rectangular hyperbola xy = 9 in the line
2 4 y = 3 is
(a) (b)
2+ 2 2+ 2 (a) xy + 9 = 0 (b) xy – 6x + 9 = 0
(c) xy + 6y – 9 = 0 (d) xy + 6x + 9 = 0.
2- 2 1
(c) (d)
2+ 2 2
Matrix Match
Passage V (For JEE-Advanced Examination Only)
The vertex A of a DABC is (3, –1). The equations of median
1. Match the following columns:
BE and angular bisector CF are, respectively x – 4y + 10 = 0
In DABC, AB : x + 2y = 3,
and 6x + 10y – 59 = 0. Then the
BC: 2x – y + 5 = 0, AC : x – 2 = 0
1. equation of AB must be
be the sides, then the
(a) x + y = 2 (b) 18x + 13y =41
(c) 23x + y = 70 (d) x + 4y = 0. Column I Column II
2. slope of the side BC must be (A) circumcentre of DABC is (P) Ê -7 11ˆ
(a) 1/7 (b) 1/9 (c) 2/9 (d) 3/4 (P) ÁË , ˜¯
5 5
3. length of the side AC must be
(a) 83 (b) 85 (c) 89 (d) 88 (B) centroid of DABC is (Q) Ê 13 131ˆ
ÁË , ˜
Passage VI 15 30 ¯
In a DABC, the equation of altitudes AM and BN and the side (C) orthocentre of DABC is (R) (R) Ê 19 ˆ
AB are given by the equations x + 5y = 3 and x + y = 1. Then ÁË 2, ˜¯
4
1. the equation of the third altitude CL must be
(a) 3x – y = 2 (b) 3x – y = 1 2. Match the following columns:
(c) 3x + y + 1 = 0 (d) x + 3y + 1 = 0. A line cuts x-axis at A and y-axis at B such that AB = l,
2. the equation of BC must be the loci of the
(a) 5x – y = 5 (b) 5x + y + 5 = 0 Column I Column II
(c) x – 2y = 3 (d) x + 2y +3 = 0
3. if R is the circum-radius of the triangle, 2R cos B must (A) circumcentre of DABC is (P) l2
x2 + y 2 =
be equal to 9
1 1 1 1 (B) orthocentre of DABC is (Q) l2
(a) (b) (c)
3
(d)
2 x2 + y 2 =
3 2 4
Passage VII (C) incentre of DABC is (R) x2 + y2 = 0
Let the curve be S: f (x, y) = 0 and the line mirror L: (D) centroid of DABC is (S) y = x
ax + by + c = 0. We take a point P on the given curve in para-
metric form. Suppose Q be the image or reflection of point 3. Match the following columns:
P about the line mirror L = 0, which again contains the same The vertex C of a DABC is (4, –1). The equation of
parameter. Let Q = (j(t), y(t)), where t is a parameter. Now, altitude AD and median AE are 2x – 3y + 12 = 0 and
2x+ 3y = 0, respectively then

CG_01.indd 23 2/6/2017 3:57:50 PM


1.24 Coordinate Geometry Booster

Column I Column II (C) The bisector of the angle (R) 6x – 2y = 5


(A) slope of side AB (P) –3/7 between the lines 2x + y =
6 and 2x – 4y + 7 = 0 which
(B) slope of side BC (Q) –3/2
contains the point (1, 2) is
(C) slope of side AC (R) –9/11
(D) The bisector of the angle (S) 7x + 9y = 3
4. Match the following columns: between the lines
The general equation of 2nd degree is 4x + 3y = 6
l x2 + 2y2 + 4xy + 2x + 4y + l = 0 and 5x + 12y + 9 = 0 which
containing the origin is
It represents
(E) The bisector of the angle (T) 9x –7y = 41
Column I Column II between the lines
(A) distinct lines if (P) k = 1 4x + 3y = 6
(B) parallel lines if (Q) k = 3 and 5x + 12y + 9 = 0 which
does not containing the ori-
(C) imaginary lines if (R) k = f
gin is
5. Match the following columns:
8. Match the following columns:
Column I Column II
Column I Column II
(A) If 3a + 2b + 6c = 0, the family (P) (– 2, – 3)
(A) The image of a line (P) 2x – y – 11
of lines ax + by + c = 0 passes
2x + 3y + 4 = 0 w.r.t. x-axis is =0
through a fixed point. Then
the fixed point is (B) The image of a line (Q) 5x + 3y + 7
7x – 3y – 10 = 0 w.r.t. y-axis =0
(B) The family of lines x(a + 2b) (Q) Ê 1 1 ˆ
is
+ y(a + 3b) = a + b passes ÁË , ˜¯
2 3 (C) The image of a line (R) x – 4y + 29
through a fixed point. Then
the point is 3x + 5y + 7 = 0 w.r.t. the line =0
y = x is
(C) If 4a2 + 9b2 – c2 + 12ab = 0, the (R) (2, –1)
family of straight lines ax + by (D) The image of a line (S) 7x + 3y +
+ c = 0 passes through a fixed 2x + y + 3 = 0 w.r.t. the line 10 = 0
point. Then the fixed point is x = 2 is
(E) The image of a line (T) 2x – 3y + 4
6. Match the following columns: x + 4y + 5 = 0 w.r.t. the line =0
Column I Column II y = 3 is
The area of a parallelogram formed by the lines 9. Match the following Columns:
(A) 3x – 4y + 1 = 0, 3x – 4y + 3 (P) 20/11 units
Column I Column II
= 0 , 4x – 3y – 1 = 0
and 4x – 3y – 2 = 0 is (A) The image of a point (P) (4, 5)
(2, 3) w.r.t. x-axis is
(B) x + 3y =1, 3x – 2y + 3 = 0, x (Q) 2/7 units
+ 3y + 4 = 0 (B) The image of a point (Q) (1, –1)
and 3x – 2y + 7 = 0 is (3, 4) w.r.t. y-axis is
(C) y = 2x + 3, y = 2x + 5. (R) 2/5 units (C) The image of a point (R) (2, –3)
y = 7x + 4 and y = 7 x + 5 is (5, 4) w.r.t. the line
y = x is
7. Match the following columns: (D) The image of a point (S) (–3, 4)
Column I Column II (2, 5) w.r.t. the line
x = 3 is
(A) The equation of the obtuse (P) 6x + 2y – 5
angle bisector of the lines =0 (E) The image of a point (T) (4, 5)
3x – 4y + 7 = 0 and (1, 5) w.r.t. the line
12x + 5y – 2 = 0 is y = 2 is
(B) The equation of the acute- (Q) 21x + 77y (F) The image of a point (U)
(2 3 - 1, 2 + 3)
angle bisector of the lines –101= 0 (2, 4) w.r.t. the line
x + y = 3 and 7x – y + 5 = 0 is y = 3 x is

CG_01.indd 24 2/6/2017 3:57:50 PM


Straight Lines 1.25

10. Match the following columns: 4. Two vertices of a triangle are (5, –1) and (–2, 3). If
the orthocentre of the triangle is the origin, find the co-
Column I Column II
ordinates of the third vertex. [IIT-JEE, 1979]
(A) The orthocentre of the trian- (P) (3, 1)
5. Find the equation of the line which bisects the obtuse
gle formed by the lines xy = 0
angle between the lines x – 2y + 4 = 0 and 4x – 3y + 2
and x + y = 4 is
= 0. [IIT-JEE, 1979]
(B) The orthocentre of the trian- (Q) Ê 1 1 ˆ 6. The points (–a, –b), (0, 0) and (a, b) are
gle formed by the lines ÁË - , - ˜¯ (a) collinear
4 6
x + y = 4, x – y = 2 (b) vertices of a rectangle
and 2x + 3y = 6 is
(c) vertices of a parallelogram
(C) The orthocentre of the trian- (R) (0, 0) (d) none [IIT-JEE, 1979]
gle formed the lines 7. The points (–a, –b), (0, 0), (a, b) and (a2, ab) are
6x2 + 5xy – 6y2 + 3x – 2y = 0 (a) collinear
and x + 4y = 5 is
(b) vertices of a parallelogram
11. Match the following columns: (c) vertices of a rectangle
(d) None [IIT-JEE, 1979]
Column I Column II
8. A straight line L is perpendicular to the line 5x – y = 1.
(A) A light beam emanating (P) 2x + 3y = 12 The area of the triangle formed by the line L and the
from the point (3, 10) co-ordinate axes is 5. Find the equation of the line L.
reflects from the straight [IIT-JEE, 1980]
line 2x + y = 6 and the 9. Given the four lines with the equations x + 2y = 3,
passes through the point 3x + 4y = 7, 2x + 3y = 4 and 4x + 5y = 6. Then
B(7, 2). Then the equa- (a) they all are concurrent
tion of the reflected ray is (b) they are the sides of a quadrilateral
(B) A ray of light is sent along (Q) x + 3y = 13 (c) only three lines are concurrent
2 (d) None [IIT-JEE, 1980]
the line y = x - 4 . On 10. The point (4, 1) undergoes the following three trans-
3
reaching the x-axis it is formations successively
reflected. Then the equa- (i) reflection about the line y = x
tion of the reflected ray is (ii) transformation through a distance 2 units along the
positive direction of x-axis.
(C) A ray of light is sent (R) 19x – 22y = – 9 p
along the line x – 2y + 5 = (iii) rotation through an angle about the origin in
4
0. Upon reaching the line the counterwise direction. Then the final position
3x + 2y + 7 = 0, the ray of the point is given by the co-ordinates
is reflected from it. Then Ê 1 7 ˆ
(a) Á , (b) (- 2, 7 2)
the equation of the line Ë 2 2 ˜¯
containing the reflected
ray is Ê 1 7 ˆ
(c) Á - , ˜ (d) ( 2, 7 2)
Ë 2 2¯ [IIT-JEE, 1980]
11. The points (1, 3) and (5, 1) are two opposite vertices
Questions asked in Previous Years’ of a rectangle. The other two vertices lie on the line
JEE-Advanced Examinations y = 2x + c. Find c and the remaining two vertices.
[IIT-JEE, 1981]
1. The area of a triangle is 5, two of its vertices are (2, 1)
12. The set of lines ax + by + c = 0, where 2a + 3b + 4c = 0
and (3, –2). The third vertex lies on y = x + 3. Find the
is concurrent at the point… [IIT-JEE, 1982]
third vertex. [IIT-JEE, 1978]
13. The straight lines x + y = 0, 3x + y = 4 and
2. One side of a rectangle lies along the line 4x + 7y + 5 x + 3y = 4 form a triangle which is
= 0. Two of its vertices are (–3, 1) and (1, 1). Find the (a) iscosceles (b) equilateral
equation of the other three vertices. [IIT-JEE, 1978] (c) right angled (d) none
3. A straight line segment of length l moves with its ends [IIT-JEE, 1983]
on two mutually perpendicular lines. Find the locus of 14. The ends A and B of a straight line segment of length
the point which divides the line in the ratio 1 : 2. c slide upon the fixed rectangular axes OX and OY, re-
[IIT-JEE, 1978] spectively. If the rectangle OABP be completed, show

CG_01.indd 25 2/6/2017 3:57:51 PM


1.26 Coordinate Geometry Booster

that the locus of the perpendicular drawn from P to AB x + 2y = 0, respectively. If the point A is (1, –2), find the
is x2/3 + y2/3 = a2/3. [IIT-JEE, 1983] equation of the line BC. [IIT-JEE, 1986]
15. The vertices of a triangle are [at1t2, a(t1 + t2)], Ê 8ˆ
[at2t3, a(t2 + t3)], [at3t1, a(t1 + t3)]. Find the orthocentre 28. The points Á 0, ˜ , (1, 3) and (82, 30) are the vertices
Ë 3¯
of the triangle. [IIT-JEE, 1983] of
16. The straight line 5x + 4y = 0 passes through the point of (a) an obtuse-angled triangle
intersection of the straight lines x + 2 y =10 and 2x + y (b) an acute-angled triangle
+ 5 = 0. Is it true/false? [IIT-JEE, 1983] (c) right-angled triangle
17. The co-ordinates of A, B and C are (6, 3), (–3, 5) and (d) an isosceles triangle
(4, –2) respectively and P is any point (x, y), show that (e) None [IIT-JEE, 1986]
x+ y-2 No questions asked in 1987.
the ratio of the area of the DPBC and ABC is 29. The lines 2x + 3y + 19 = 0 and 9x + 6y – 17 = 0 cut the
7
co-ordinate axes in concylic points. (T/F)
[IIT-JEE, 1983]
[IIT-JEE, 1988]
18. Two equal sides of an isosceles triangle are given by
30. Lines L1: ax + by + c = 0 and L2: ax + my + n = 0
the equations 7x – y + 3 = 0 and x + y – 3 = 0 and its
intersect at the point P and make an angle q with each
third side passes through the point (1, –10). Determine
other. Find the equation of a line L different from L2
the equation of the third side. [IIT-JEE, 1984]
which passes through P and makes the same angle q
19 If a, b and c are in AP, the straight line ax + by + c =
with L1. [IIT-JEE, 1988]
0 will always pass through a fixed point, whose co-
31. Let ABC be a triangle with AB = AC. If D is the mid-
ordinates are... [IIT-JEE, 1984]
point of BC, E the foot of the perpendicular drawn from
20 Three lines px + qy + r = 0, qx + ry + p = 0 and rx + py
D to AC and F the mid-point of DE. Prove that AF is
+ q = 0 are concurrent if
perpendicular to BE. [IIT-JEE, 1989]
(a) p + q + r = 0
32. Striaght lines 3x + 4y = 5 and 4x – 3y = 15 intersect at
(b) p2 + q2 + r2 = pq + qr + rp
the point A. Points B and C are chosen on these two
(c) p3 + q3 + r3 = 3pqr
lines such that AB = AC. Determine the possible equa-
(d) none of these [IIT-JEE, 1985]
tions of the line BC passing through the point (1, 2).
21. The orthocentre of the triangle formed by the lines [IIT-JEE, 1990]
x + y = 1, 2x + 3y = 6 and 4x – y + 4 = 0 lies in quadrant 33. A line L has intercepts a and b on the co-ordinate axes.
number... [IIT-JEE, 1985] When the axes are rotated through a given angle, keep-
22. Two sides of a rhombus ABCD are parallel to the lines ing the origin fixed, the same line L has intercepts p and
y = x + 2 and y = 7x + 3. If the diagonals of the rhombus q, then
intersect at the point (1, 2) and the vertex A is on the 1 1 1 1
y-axis, find the possible co-ordinates of A. (a) a2 + b2 = p2 + q2 (b) 2 + 2 = 2 + 2
[IIT-JEE, 1985] a b p q
x1 y1 1 a1 b1 1 1 1 1 1
(c) a2 + p2 = b2 + q2 (d) + = +
p 2 a 2 b2 q 2
23. If x2 y2 1 = a2 b2 1 , the two triangles with ver-
[IIT-JEE, 1990]
x3 y3 1 a3 b3 1
34. A line cuts the x-axis at A(7, 0) and the y-axis at
tices (x1, y1), (x2, y2), (x3, y3) and (a1, b1), (a2, b2), (a3, b3),
B(0, –5). A variable line PQ is drawn perpendicular to
must be congruent. Is it true or false?
AB cutting the x-axis in P and the y-axis in Q. If AQ and
[IIT-JEE, 1985] BP intersect in R, find the locus of R.
24. One of the diameter of the circle circumscribing the [IIT-JEE, 1990]
rectangle ABCD is 4y = x + 7. If A and B are the points
35. Find the equation of the line passing through the point
(–3, 4) and (5, 4), respectively, find the area of the rect-
(2, 3) and making intercept of length 2 units between
angle. [IIT-JEE, 1985]
the lines y + 2x = 3 and y + 2x = 5. [IIT-JEE, 1991]
25. The set of all real numbers a such that a2 + 2a, 2a + 3
36. Let the algebraic sum of the perpendicular distance
and a2 + 3a + 8 are the sides of a triangle is...
from the points (2, 0), (0, 2) and (1, 1) to a variable
[IIT-JEE, 1985]
straight line be zero, the line passes through a fixed
26. All points inside the triangle formed by the points
point whose co-ordinates are.... [IIT-JEE, 1991]
(1, 3), (5, 0) and (–1, 2) satisfy
(a) 3x + 2y 0 (b) 2x + y – 13 0 37. If the sum of the distances of a point from two mutually
(c) 2x – 3y – 12 0 (d) –2x + y perpendicular lines in a plane is 1, its locus is
(e) None [IIT-JEE, 1986] (a) square (b) circle
27. The equation of the perpendicular bisectors of the sides (c) straight line (d) two intersecting lines
AB and AC of a triangle ABC are x – y + 5 = 0 and [IIT-JEE, 1992]

CG_01.indd 26 2/6/2017 3:57:51 PM


Straight Lines 1.27

38. Determine all values of a for which the point (a, a2) 48. The number of integral values of m, for which the x-
lies inside the triangle formed by the lines 2x + 3y – 1 = co-ordinate of the point of intersection of the lines 3x +
0, x + 2y – 3 = 0 and 5x – 6y – 1 = 0. [IIT-JEE, 1992] 4y = 9 and y = mx + 1 is also an integer, is
39. The vertices of a triangle A(–1, –7), B(5, 1) and C(1, (a) 2 (b) 0 (c) 4 (d) 1
4). The equation of the bisector of the angle –ABC is... [IIT-JEE, 2001]
[IIT-JEE, 1993] 49. Let P = (–1, 0), Q = (0, 0) and R = (3, 3 3) be three
40. A line through A(–5, –4) meets the lines x + 3y + 2 = 0, points. The equation of the bisector of the angle PQR is
2x + y + 4 = 0 and x – y – 5 = 0 at the points B, C and 3
2 2 2 (a) x+ y=0 (b) x + 3 y = 0
Ê 15 ˆ Ê 10 ˆ Ê 6 ˆ 2
D, respectively. If Á + =
Ë AB ˜¯ ËÁ AC ¯˜ ËÁ AD ¯˜
, find the 3
(c) 3 x + y = 0 (d) x + y=0
equation of the line. [IIT-JEE, 1993] 2
[IIT-JEE, 2002]
41. The orthocentre of the triangle formed by the lines 50. A straight line L through the origin meets the lines
x y = 0 and x + y = 1 is x + y = 1 and x + y = 3 at P and Q, respectively.
Ê 1 1ˆ Ê 1 1ˆ Ê 1 1ˆ Through P and Q two straight lines L1 and L2 are
(a) Á , ˜ (b) Á , ˜ (c) (0, 0) (d) Á , ˜
Ë 2 2¯ Ë 3 3¯ Ë 4 4¯ drawn parallel to 2x – y = 5 and 3x + y = 5, respectively.
[IIT-JEE, 1995] Lines L1 and L2 intersect at R. Show that the locus of R
42. A rectangle PQRS has its sides PQ parallel to the line as L varies, is a straight line. [IIT-JEE, 2002]
y = mx and vertices P, Q and S on the lines y = a, x = b 51. A straight line L with negative slope passes through the
and x = – b, respectively. Find the locus of the vertex R. point (8, 2) and meets the positive co-ordinate axes at
[IIT-JEE, 1996] points P and Q. Find the absolute minimum value of
No questions asked in 1997. OP + OQ as L varies, where O is the origin.
43. The diagonals of a parallelogram PQRS are along the [IIT-JEE, 2002]
lines x + 3y = 4 and 6x – 2y = 7. Then PQRS must be a: 52. A straight line through the point (2, 2) intersects the
(a) rectangle (b) square lines 3x + y = 0 and 3x – y = 0 at the points A and
(c) cyclic quad. (d) rhombus B. The equation to the line AB so that the triangle OAB
[IIT-JEE, 1998] is equilateral, is
No questions asked in 1999. (a) x – y = 0 (b) y – 2 = 0
44. Let PS be the median of the triangle with vertices (c) x + y – 4 = 0 (d) none of these
P(2, 2), Q(6, –1) and R(7, 3). The equation of the line [IIT-JEE, 2002]
passing through (1, –1) and parallel to PS is 53. A straight line through the origin O meets the parallel
(a) 2x – 9y – 7 = 0 (b) 2x – 9y – 11 = 0 lines 4x + 2y = 9 and 2x + y + 6 = 0 at points P and Q
(c) 2x + 9y – 11 = 0 (d) 2x + 9y + 7 = 0 respectively. The point O divides the segment PQ in
[IIT-JEE, 2000] the ratio
45. A straight line through the origin O meets the parallel lines (a) 1 : 2 (b) 3 : 4 (c) 2 : 1 (d) 4 : 3.
4x + 2y = 9 and 2x + y + 6 = 0 at points P and Q, respec- [IIT-JEE, 2002]
tively. The point O divides the segment PQ in the ratio 54. The orthocentre of the triangle with vertices (0, 0), (3,
(a) 1 : 2 (b) 3 : 4 (c) 2 : 1 (d) 4 : 3 4) and (4, 0) is
[IIT-JEE, 2000] Ê 5ˆ
46. For points P = (x1, y1) and Q = (x2, y2) of the co-ordinate (a) Á 3, ˜ (b) (3, 12)
Ë 4¯
plane, a new distance d(P, Q) is defined as d(P, Q) =
Ê 3ˆ
|x1 – x2| + |y1 – y2|. (c) Á 3, ˜ (d) (3, 9)
Ë 4¯ [IIT-JEE, 2003]
Let O = (0, 0) and A = (3, 2). Prove that the set of
points in the first quadrant which are equidistant (with 55. If the equation of the locus of a point equidistant from
respect to the new distance) from O and A consists of the points (a1, b1) and (a2, b2) is
the union of a line segment of finite length and an infi- (a1 – a2)x + (b1 – b2)y + c = 0, then the values of ‘c’ is
nite ray. Sketch this set in a labelled diagram. 1 2 2 2 2
(a) (a22 + b22 - a12 - b12 ) (b) (a1 - a2 + b1 - b2 )
[IIT-JEE, 2000] 2
47. The area of the parallelogram formed by the lines 1 2
(c) (a1 + a22 + b12 + b22 ) (d) (a12 - a22 + b12 - b22 )
y = mx, y = mx + 1, y = nx and y = nx + 1 equals 2
| m + n| [IIT-JEE, 2003]
2
(a) (b) 56. The area of the triangle formed by the lines x + y = 3
( m - n) 2 | m + n| and angle bisector of the pair of straight lines
1 1
(c) (d) x2 – y2 + 2y – 1 = 0 is
| m + n| | m - n| (a) 2 (b) 4 (c) 6 (d) 8
[IIT-JEE, 2001] [IIT-JEE, 2004]

CG_01.indd 27 2/6/2017 3:57:52 PM


1.28 Coordinate Geometry Booster

57. The area of the triangle formed by the intersection of a where p π q, is


line parallel to x-axis and passing through P(h, k) with (a) a hyperbola (b) a parabola
the lines y = x and x + y = 2 is 4h2. Find the locus of the (c) an ellipse (d) a straight line
point P. [IIT-JEE, 2005] [IIT-JEE, 2009]
58. Two rays in the first quadrant x + y = | a | and No questions asked in 2010.
ax – y = 1 intersects each other in the interval a Œ (a0, 62. A straight line L through the point (3, –2) is inclined at
). Find the value of a0. [IIT-JEE, 2006] an angle 60° to the line 3 x + y = 1. If L also intersects
59. Lines L1: y – x = 0, L2: 2x + y = 0 intersect the line L3: the x-axis, the equation of L is
y + 2 = 0 at P and Q, respectively. The bisector of the
(a) y + 3 x + (2 - 3 3) = 0
acute angle between L1 and L2 intersect L3 at R.
Statement I: The ratio PR : RQ equals 2 2 : 5 . (b) y - 3 x + (2 + 3 3) = 0
Statement II: In any triangle bisector of an angle, di- (c) - x + 3 y + (3 + 2 3) = 0
vides the triangle into two similar triangles. (d) x + 3 y + (-3 + 2 3) = 0 [IIT-JEE, 2011]
[IIT-JEE, 2007]
No questions asked in 2012.
60. Consider the lines given by
63. For a > b > c > 0, the distance between (1, 1) and the
L1: x + 3y – 5 = 0, L2: 3x – ky – 1 = 0,
point of intersection of the lines ax + by + c = 0 and
and L3: 5x + 2y – 12 = 0
bx + ay + c = 0 is less than 2 2, then
Column I Column II
(a) a + b – c > 0 (b) a – b + c < 0
(a) L1, L2, L3 are concurrent if (p) k = –9
(c) a – b + c > 0 (d) a + b – c < 0
(b) One of L1, L2, L3 is parallel (q) k = –6/5
[IIT-JEE, 2013]
to at least of the other two, if
64. For a point P in the plane, let d1(P) and d2(P) be the
(c) L1, L2, L3 form a triangle, if (s) k = 5 /6
distances of the point P from the lines x – y = 0 and
(d) L1, L2, L3 do not form a (t) k = 5
x + y = 0, respectively.
triangle, if
The area of the region R consisting of all points P
[IIT-JEE, 2008]
lying in the first quadrant of the plane and satisfying 2
61. The locus of the orthocentre of the triangle formed by
£ d1(P) + d2(P) £ 4, is ...
the lines (1 + p)x – py + p(1 + p) = 0,
[IIT-JEE, 2014]
(1 + q)x – qy + q(1 + q) = 0 and y = 0,

A NSWERS

LEVEL I Ê1 ± 3 7 ± 5 3ˆ
11. Á
Ë 2
, ˜
2 ¯
(CARTESIAN CO-ORDINATES) 12. (3, 1)
13. 3/7
Ê 4 ˆ Ê 4 ˆ
1. Á 5, - tan -1 ÊÁ ˆ˜ ˜ , Á 5, p - tan -1 ÊÁ ˆ˜ ˜ 14. 12:1
Ë Ë 3¯ ¯ Ë Ë 3¯ ¯
Ê 2 ˆ Ê 5 ˆ
2. (x2 + y2)2 = a2(x2 – y2) 15. ÁË - , 0˜¯ and ÁË - , 2˜¯
3 3
3. (x2 + y2) = 2ax 16. (0, 1), (1, 0) and (3, 1)
cos 2 q sin 2q 1 17. (4, 6)
4. + 2 = 2
a2 b r Ê 10 8 ˆ
18. Á , ˜
Ë 3 3¯
2 2
5. r = 19. (–4, –15)
(4 + 3 sin 2q )
20. (2, 3)
Êa - bˆ Ê 4 5ˆ
6. 2a sin ÁË ˜ 21. ÁË , ˜¯
2 ¯ 3 6
7. 10 22. 2
8. 34 23. 46
3 24. 2
9. 25. x + y + 9 = 0, x + y – 15 = 0
10
10. 26. 132

CG_01.indd 28 2/6/2017 3:57:52 PM


Straight Lines 1.29

27. 54 17. 4x + 5y = 14
28. 0 18. (2 - 3) x - y - 2(2 - 3) = 0
29. x = 7/4 19. x–y+1=0
2304 20. x – 5y + 10 = 0
30.
3850 21. 3x – 5y + 7 = 0
31. x2 + y2 = a2 4-2
32. y2 = 4ax 22. y - 2 = ( x - 5) = - 2( x - 5)
4-5
33. y2 = 8x2
34. y2 = 2ax – a2 23. (3/2, 0), (9/4, 0), (9/4, 3/4) and (3/2, 3/4)
35. x2 – y2 = 4 24. y - 0 = 3 ( x - 2), x 3 - y - 2 3 = 0
x2 y 2 25. x+y=7
36. + =1
7 16 2 26. 2x + y = 7
2 2 l
37. x + y = 27. 3x + 2y = 12
4
38. y2 = 4x 28. x–y=7
39. 2(x2 + y2) – x – y + 1 = 0 29. x – y = 1, 2x – 3y = 12
30. 3
40. xy = x + y + x 2 + y 2 31. 2x + y = 10
44. x2 + y2 = 4 32. 3x – 8y = 0, 3x – 2y = 0
45. x2 + y2 = 3
35. x + 3 y = 10
46. 2x2 + y2 –7x + 9y + 1 = 0
36. 12x + 5y = 26
47. x2 + y2 – 8x – 10y + 5 = 0
48. (–6, –5) 37. (7, 5), (–1, –1)
49. 2x2 + y2 = 1 38. (2, 7) and (–6, –1)
50. y2=4x 39. (4, 5)
51. (–2, 3)
Ê 1 3ˆ Ê 1 3ˆ
52. 12 40. ËÁ 2 + 2 ◊ , 2 ◊ ˜ = Á2 +
¯
, ˜
53. x2 – y2 = a2 2 2 Ë 2 2¯
54. 4x2 + 2y2 = 1 41. 75°, 15°
55. (–3, 2) 42. R(–2, –1), Q(–1, 2) and S(1, –2)
Ê1 - 4 3 4 + 3ˆ Ê 3 3ˆ Ê 1 3ˆ
56. ÁË , ˜ 43. ÁË - , ˜¯ and ÁË , - ˜¯
2 2 ¯ 2 2 2 2
44. 2

LEVEL I
45. 6
46. 4 2
47. 5
(STRAIGHT LINES)
48. 5
1. 6 49. (8, 2) and (0, –4)
2. 3
54. 3x + 4y – 18 = 0
-1 Ê 3 ˆ
5. q = tan ÁË ˜¯ 55. 3x – 4y – 5 = 0
4
56. 4x + y = 9
6. 3 or –1/ 3
7. y + 3 = 0 57.
8. y = 4 58. 3x – 2y – 1 = 0
9. x = 8 59. 3x + 2y + 3 = 0
10. y = 3x + 7 60. 4x + 6y = 29
11. y = x + 5 61. x – 3y + 5 = 0.
3 62. 4x – 3y – 36 = 0
12. y = mx + c = x-6
5 6
63.
13. y = mx + c = ± x + 4 34
14. y = mx + c = ± x.
2
15. 3 x + y = 2 3 + 3 64.
3
16. 8x + 10y = 69

CG_01.indd 29 2/6/2017 3:57:53 PM


1.30 Coordinate Geometry Booster

65.
a
1
2
b
1
p
1
+ 2 = 2 LEVEL II
1 1. (c) 2. (d) 3. (b) 4. (a) 5. (a)
66. 2 6. (a) 7. (a) 8. (b) 9. (a) 10. (a)
10
1 1 1 1 11. (b) 12. (a,c) 13. (d) 14. (c) 15. (a)
67. 2
+ 2 = 2 + 2 16. (b) 17. (d) 18. (a) 19. (b) 20. (c)
a b p q
21. (a) 22. (a) 23. (b) 24. (a) 25. (a, d)
68. 50 26. (c) 27. (d) 28. (b) 29. (a, b) 30. (b)
2c 2 31. (c) 32. (d) 33. (c) 34. (d) 35. (b)
69.
| ab | 36. (a) 37. (a) 38. (b) 39. (b) 40. (a)
70. (2, 6) 41. (b) 42. (a) 43. (a,bc) 44. (c) 45. (b)
71. 0 46. (d) 47. (b) 48. (d) 49. (c) 50. (a)
72. 7x – 48y + 89= 0 51. (b) 52. (b) 53. (c) 54. (d) 55. (c)
73. m=2 56. (a) 57. (c) 58. (d) 59. (c) 60. (b)
1 1 1 61. (d) 62. (b) 63. (a) 64. (a) 65. (c)
74. + + =1 66. (c) 67. (b) 68. () 69. () 70. ()
1- a 1- b 1- c
1 1
x = and y =
75.
78. 10
3 2 LEVEL III
1 5 7
81. m = - 2 and m = 1. £b£ 2. 2
2 3 2
82. 3x – y – 7 = 0 and x + 3y – 9 = 0 3. 3x + y = 0 4. y = 2
83. (2 + 3) x - y - (2 3 + 1) = 0 and 5. 2x – y + 1 = 0, 2x + y – 1 = 0
(2 - 3) x - y + (2 3 - 1) = 0 6. a0 = 1
84. 52x + 89y + 519 = 0 8. 2(2 + 2)
85. x + 2 = 0 and 7x – 24y + 182 = 0 9.
86. 9x – 7y + 3 = 0 and 7x + 9y = 41 10. (5, 3) and (–3, 5)
87. x – 7y + 13 = 0 and 7x + 7 = 7 4
11. <m<4
88. 3x + y + 7 = 0 and x – 3y = 31 5
89. 21x + 77y = 101 and 3x – y + 3 = 0 12. ...
90. 7x + 9y = 3 13. R – (–4, 0)
91. 6x – 2y = 5 14. (–2,1) and (–4, 3)
92. 21x + 77y = 101 15. -2 2
93. 6x + 2y= 5
94. 7x – 9y + 10 = 0 Ê 3 ˆ Ê1 ˆ
17. a ŒÁË - , -1˜¯ » ÁË , 1˜¯
Ê 1 2ˆ 2 2
95. ÁË , ˜¯
3 3 18. c = –1 ; (4, 4), (2, 0)
96. PQ = 6 19. y = (2 ± 3) x - 1 ± 2 3
97. (2 - 2 2, 3 + 2 2) 20. y = x.
98. (–1/10, 37/10) 21. (13/5, 0)
99. (0, 28 22. (42/25, 91/25)
100. (4, 3). 23. (6, 0) ; 2x + 3y = 12.
101. 3x – 2y + 5 = 0 24. –2 < a < 2
102. (6, 1) 25. 26x – 122y = 1675.
Ê 3 7 ˆ 26. 27.B(–7, 6); C(11/5, 2/5)
103. ÁË , ˜ 28. 52x + 89y + 519 = 0
2 2¯
29. x + 2 = 0 and 7x + 24y + 182 = 0.
105. 29x – 2y = 31
1
13 30. p =
106. 3
5
107. y=0 31. x – 7y + 13 = 0 and 7x + y = 9.
108. (70 - 37 3) x - 13y - 153 + 74 3 = 0 32. R(42/25, 91/25)

CG_01.indd 30 2/6/2017 3:57:54 PM


Straight Lines 1.31

33. 3x + y + 7 = 0 and x – 3y – 31 = 0 COMPREHENSIVE LINK PASSAGES


34. D: (6 + 2 2, - 1 - 2 2) Passage I: 1. (c) 2. (a) 3. (b) 4. (c)
or D: (2 - 2 2, 3 + 2 2) Passage II: 1. (a) 2. (b) 3. (a) 4. (b) 5. (b)
35. (0, 0) or (0, 5/2). Passage II: 1. (b) 2. (a) 3. (c)
Passage IV: 1. (a) 2. (b) 3. (b)

LEVEL IV Passage V:
Passage VI:
1.
1.
(b)
(b)
2.
2.
(c)
(a)
3.
3.
(b)
(b)
1. Sides are: x – 3y = 3, 3x – 2y = 16, Passage VII: 1. (c) 2. (a) 3. (b) 4. (a) 5. (b).
Diagonals are x + 4y = 10, 5x – 8y + 6 = 0.
MATRIX MATCH
2. 5 2
1. (A) Æ (R), (B) Æ (Q), (C) Æ (P)
Ê 3 2 - 1 ( 2 + 1) ˆ 2. (A) Æ (Q), (B) Æ (R), (S) Æ (R), (D) Æ (P)
3. C =Á , ˜
Ë 2 2 ¯ 3. (A) Æ (R), (B) Æ (Q), (C) Æ (P)
4. (4, 2) 4. (A) Æ (P), (B) Æ (R), (C) Æ (P)
5. x–y=2 5. (A) Æ (Q), (B) Æ (R), (C) Æ (P),
6. x+ y 3- 3=0 6. (A) Æ (P), (B) Æ (Q), (C) Æ (R),
Ê 1 2 ˆ 7. (A) Æ (Q), (B) Æ (R), (C) Æ (R),
7. ÁË1 ± ,1± ˜
5 5¯ (D) Æ (S),(E) Æ (T),
8. (2 - 2, 2 - 1), (2 5 + 2, 5 - 1), 29 + 2 10 8. (A) Æ (T), (B) Æ (S), (C) Æ (Q), (D) Æ (P),
9. 9x2 – 6xy + 10y2 = 9l 2 (E) Æ (R)
9. (A) Æ (R), (B) Æ (S), (C) Æ (T),
INTEGER TYPE QUESTIONS (D) Æ (P), (E) Æ (Q), (F) Æ (U)
1. 8 2. 6 3. 3 4. 1 5. 1 10. (A) Æ (R), (B) Æ (P), (C) Æ (Q)
6. 5 7. 4 8. 4 9. 6 10. 7 11. (A) Æ (Q), (B) Æ (P), (C) Æ (R)

H INTS AND S OLUTIONS

LEVEL I 3. We have,
r = 2a cos 2q
Ê xˆ
(RECTANGULAR CARTESIAN CO-ORDINATES) fi r = 2a Á ˜
Ë r¯
1. We have, fi r = 2ax
2

Ê Ê 4ˆ ˆ fi (x2 + y2 = 2ax)
(3, - 4) fi Á 32 + (-4) 2 , tan -1 Á - ˜ ˜
Ë Ë 3¯ ¯ 4 We have,
Ê -1 Ê 4 ˆ ˆ x2 y 2
fi ÁË 5, - tan ÁË ˜¯ ˜¯ + =1
3 a 2 b2
and fi b2x2 + a2y2 = a2b2
Ê Ê 3ˆ ˆ fi b2r2cos2q + a2r2sin2q = a2b2
(-3, 4) fi Á (-3) 2 + (4) 2 , tan -1 Á - ˜ ˜
Ë Ë 4¯ ¯ r 2 cos 2 q r 2 sin 2 q
fi + =1
Ê -1 Ê 4 ˆ ˆ a2 b2
fi ÁË 5, p - tan ÁË ˜¯ ˜¯
3 cos 2 q sin 2 q 1
fi + = 2
2.. We have, a2 b2 r
r2 = a2 cos 2q 5. We have,
fi r2 = a2 (cos2q – sin2q) 2x2 + 3xy + 2y2 = 1
Ê x2 y 2 ˆ fi 2r2cos2q + 3r2sinq cosq + 2r2sin2q = 1
fi r 2 = a2 Á 2 - 2 ˜
Ër r ¯ fi r2(2cos2q + 3sinq cosq + 2sin2q = 1
r4 = a2(x2 – y2) 3
Ê ˆ
fi (x2 + y2)2 = a2(x2 – y2) fi r 2 Á 2 + sin 2q ˜ = 1
Ë 2 ¯

CG_01.indd 31 2/6/2017 3:57:55 PM


1.32 Coordinate Geometry Booster

2 13. Let the ratio be m : n.


fi r2 = Since the point lies on y-axis, so x co-ordinate will be
(4 + 3 sin 2q )
zero.
6. The required distance 7m - 3n
Thus, =0
= a 2 (cos a - cos b ) 2 + a 2 (sin a - sin b ) 2 m+n
fi 7m – 3n = 0
= a (cos a - cos b ) 2 + (sin a - sin b ) 2 m 3
fi =
= a [(cos 2a + sin 2a ) + (cos 2 b + sin 2 b ) n 7
14. Let the ratio be l : 1.
- 2(cos a cos b + sin a sin b )]
Ê 1 - 2l 3l - 2 ˆ
Thus, the point is Á , .
= a ¥ 2 - 2 cos(a - b ) Ë l + 1 l + 1 ˜¯
= a ¥ 2(1 - cos(a - b )) Ê 1 - 2l 3l - 2 ˆ
Since, the point Á , lies on the line
Ë l + 1 l + 1 ˜¯
Êa - bˆ
= 2a sin Á
Ë 2 ˜¯ 3x + 4y = 7, we get
Ê 1 - 2l ˆ Ê 3l - 2 ˆ
7. The required distance 3Á + 4Á =7
Ë l + 1 ˜¯ Ë l + 1 ˜¯
Ê 5p p ˆ
= 32 + 7 2 - 2 ¥ 3 ¥ 7 ¥ cos Á - ˜ fi 3(1 – 2l) + 4(3l – 2) = 7l + 7
Ë 4 4¯ fi 12l – 6l – 7l = 8 + 7 – 3
= 32 + 7 2 - 2 ¥ 3 ¥ 7 ¥ cos(p ) fi l = –12
Hence, the ratio is 12 : 1 externally.
= 9 + 49 + 42 = 100 = 10 15. Let A = (1, –2) and B = (–3, 4) and the line AB is tri-
8. Let P = (x, y), A = (6, –1) and B = (2, 3). sected at P and Q, respectively.
We have, Therefore P divides A and B internally in the ratio 1 : 2
PA = PB and Q divides A and B in the ratio 2 : 1.
fi (x – 6)2 + (y + 1)2 = (x – 2)2 + (y – 3)2 Ê 2 ˆ
Thus, the co-ordinates of P and Q are Á - , 0˜ and
fi x2 – 12x + 36 + y2 + 2y + 1 Ë 3 ¯
Ê 5 ˆ
= x2 – 4x + 4 + y2 – 6y + 9 ÁË - , 2˜¯ .
3
fi –12x + 36 + 2y + 1 = –4x + 4 – 6y + 9 16. Let the co-ordinates of the vertices are (x1, y1), (x2, y2),
fi –8x + 8y + 37 – 13 = 0 and (x3, y3).
fi –8x + 8y + 24 = 0 Therefore, x1 + x2 = 1, x2 + x3 = 4, x1 + x3 = 3
fi A = –8, B = 8, C = 24 fi x1 + x2 + x3 = 4
Hence, the value of A + B + C + 10 Thus, x1 = 0, x2 = 1, x3 = 3
= –8 + 8 + 24 + 10 = 34 Also, y1 + y2 = 1, y2 + y3 = 1, y3 + y1 = 2
9. We have, fi y1 + y2 + y3= 2
AB = (2 + 2) 2 + (-1 - 3) 2 = 4 2 Thus, y1 = 1, y2 = 0, y3 = 1
Hence, the vertices are (0, 1), (1, 0) and (3, 1).
BC = (2 - 4) 2 + (-1) 2 = 5 17. Let the co-ordinates of the third vertex be(x, y).
As we know that the diagonals of a parallelogram bi-
and CA = (4 + 2) 2 + (3) 2 = 45 = 3 5
sect each other.
(4 2) 2 + (3 5) 2 - ( 5) 2 x -1 1+ 2 y+2 5+3
Thus, cos A = Thus, = and =
2¥4 2 ¥3 5 2 2 2 2
fi x = 4, y = 6
32 + 45 - 5 72 3
fi cos A = = = Hence, the fourth vertex is (4, 6).
24 10 24 10 10 18. The co-ordinates of the centroid of DABC are
10. Do yourself. Ê 2 + 6 + 2 4 + 4 + 0 ˆ Ê 10 8 ˆ
ÁË , ˜¯ = ÁË , ˜¯
Ê1 ± 3 7 ± 5 3ˆ 3 3 3 3
11. ÁË , ˜
2 2 ¯ 19. Let the co-ordinates of the third vertex be (x, y).
x -1+ 5
12. Let the point be (x, y). Therefore, = 0 fi x = - 4 and
3
4+5 6-3
Thus, x = = 3, y = =1 y+4+2
2 +1 2 +1 = - 3 fi y = -15
3
Hence, the point is (3, 1). Hence, the co-ordinates of the third vertex be (–4, –15).

CG_01.indd 32 2/6/2017 3:57:55 PM


Straight Lines 1.33

20. Let a = BC, b = CA, c = AB be the lengths of the side of


1 -3 1
the given DABC. =
2 2 -2
Therefore, a = (3 - 2) 2 + (4 - 3) 2 = 2, 1
= (6 - 2) = 2 sq.u.
2
b = (3 - 1) 2 + (4 - 2) 2 = 2 2
25. Given area of a triangle is 6.
and c = (2 - 1) 2 + (3 - 2) 2 = 2 1 x -1 1- 2
Thus, the incentre are fi = ±6
2 y - 2 2 -1
Ê ax1 + bx2 + cx3 ay1 + by2 + cy3 ˆ
ÁË a + b + c , a + b + c ˜¯ x - 1 -1
fi = ±12
y-2 1
Ê 1◊ 2 + 2 ◊ 2 2 + 3 ◊ 2 2 ◊ 2 + 3 ◊ 2 2 + 4 ◊ 2 ˆ fi (x –1) + (y – 2) = ± 12
=Á , ˜
Ë 2+2 2+ 2 2+2 2+ 2 ¯ fi x + y – 3 + 12 = 0, x + y – 3 – 12 = 0
Ê 8 2 12 2 ˆ fi x + y + 9 = 0, x + y – 15 = 0
=Á ,
Ë 4 2 4 2 ˜¯
Hence, the result.
26. The required area of a quadrilateral
= (2, 3)
1 1
21. As we know that the centroid divides the orthocentre 7 -3
and the circumcentre in the ratio 2 : 1. 1
= 12 2
Thus, the centroid are 2
Ê 3 3 ˆ 7 21
2 ◊ + 1◊1 2 ◊ + 1◊1
Á 2 4 ˜ Ê 4 5ˆ 1 1
, =Á , ˜
ËÁ 2 + 1 2 + 1 ¯˜ Ë 3 6 ¯ 1
= {(-3 + 14 + 252 + 7) - (21 + 14 - 36 + 7)}
22. Clearly, it is a right-angled triangle. 2
As we know that in case of a right angled triangle, the 1
= (270 - 6) = 132 sq.u.
circumcentre is the mid-point of the hypotenuse and 2
the orthocentre is at right angle. 27. The required area of a pentagon
0 + 2 2 + 0ˆ
Thus, Circumcentre = ÊÁ , ˜ 4 3
Ë 2 2 ¯ -5 6
= (1, 1)
and 1 0 7
=
Orthocentre = (0, 0) 2 3 -6
Hence, the required distance = (1 - 0) 2 + (1 - 0) 2 -7 -2
4 3
= 2
23. The required area 1
= {(24 - 35 + 0 - 6 - 21) - ( -8 + 42 + 21 + 0 + 15)}
1 x1 - x3 x2 - x3 2
= 1
2 y1 - y3 y2 - y3 = (38 + 70) = 54 sq. u.
2
1 3 - 7 7 +1
= 28. We have,
2 -4 - 5 5 - 10
1 x1 - x2 x2 - x3
1 -4 8 2 y1 - y2 y2 - y3
=
2 -9 -5
1 a-b b-c
1 =
= (20 + 72) = 46 sq.u. 2 b+c-c-a c+a-a-b
2
1 a-b b-c
24. The required area =
2 b-a c-b
1 x1 - x3 x2 - x3
= 1 1 1
2 y1 - y3 y2 - y3 = (a - b)(b - c)
2 -1 -1
1 t -t -3 t +3-t -2 =0
=
2 t+2-t t-t-2
Hence, the result.

CG_01.indd 33 2/6/2017 3:57:56 PM


1.34 Coordinate Geometry Booster

29. Now, 34. Let the movable point be (x, y).


Y
1 x + 3 -3 - 4
ar (DDBC) =
2 3x - 3 5 + 2 M P(x, y)
1
= (7x + 21 - 21x - 21)
2
X¢ X
1 O Q(a, 0)
= (-14x) = 7x
2
and
1 6 + 3 -3 - 4
ar (DABC) = Y¢
2 3-5 5+ 2
Given condition is
1 9 -7 PQ = PM
=
2 -2 7 fi ( x - a)2 + y 2 = x
1 49 fi (x – a)2 + y2 = x2
= (63 - 14) =
2 2 fi y2 = x2 – (x – a)2 = 2ax – a2
It is given that which is the required locus of the given movable point.
7x 1 35. Let the co-ordinates of the variable point P be (x, y).
= 1 1
49 2
Then x = t + and y = t -
2 t t
2 2
7 Ê 1ˆ Ê 1ˆ
fi x= fi x2 - y 2 = Á t + ˜ - Á t - ˜ = 4
4 Ë t¯ Ë t¯
which is the required locus of P and represents a rect-
30. The required area angular hyperbola.
2
7 -2 10 36. Let P be (x, y).
1 Y
= 7 2 -10 ,
2 ¥ | C1C2C3 | P(x, y)
9 1 2

7 2 7 -2
where C1 = = -11, C2 = - = - 25, X¢ X
9 1 9 1 B(–3, 0) O A(3, 0)

7 -2
and C3 = - = - 28
7 2
1 Y¢
= (96) 2 Given condition is
2 ¥ 11 ¥ 25 ¥ 28 PA + PB = 8.
9216
= fi ( x) 2 + ( y + 3) 2 + ( x) 2 + ( y - 3) 2 = 8
2 ¥ 11 ¥ 25 ¥ 28
9216 2304 fi ( x) 2 + ( y + 3) 2 = 8 - ( x) 2 + ( y - 3) 2
= = sq. u.
15400 3850 fi (x)2 + (y + 3)2
31. Let x = acosq, y = asinq = 64 + (( x) 2 + ( y - 3) 2 ) - 16 ( x) 2 + ( y - 3) 2
Then x2 + y2 = a2cos2q + a2sin2q = a2
which is the required locus of the point P. fi 6y = 64 - 6y - 16 ( x) 2 + ( y - 3) 2
32. Let x = at2 and y = 2at. fi 12y - 64 = 16 ( x) 2 + ( y - 3) 2
Then, y2 = 4a2t2 = 4a(at2) = 4ax
which is the required locus of P. fi 3y - 16 = 4 ( x) 2 + ( y - 3) 2
33. Let the movable point P be (x, y) and the point on fi (3y – 16)2 = 16((x)2 + (y – 3)2)
y-axis be (0, y). fi 9y2 – 96y + 256 = 16(x2 + y2 – 6y + 9)
Given condition is fi 16x2 + 7y2 = 256 – 144 = 112
x 2 + y 2 = 3x x2 y 2
fi + =1
fi x2 + y2 = 9x2 7 16
fi y2 = 8x2 which is the required locus of the given point and rep-
which is the required locus of P. resents an ellipse.

CG_01.indd 34 2/6/2017 3:57:58 PM


Straight Lines 1.35

37. Let AB = l. Y 1 1 3 1
Consider the point A B Then a = + cos q , b = + sin q
P 4 2 2 4 2 2
lies on x-axis and B
2 2
lies on y-axis such that I Ê 1ˆ Ê 3ˆ
Now, 8 Á a - ˜ + 8 Á b - ˜
A = (a, 0) and X¢ O A
X Ë 4¯ Ë 4¯
B = (0, b) = cos2q + sin2q = 1
Therefore, AB = l
Ê a 3b ˆ 1 9
fi a 2 + b2 = l Y¢ fi 8Áa 2 + b 2 - - ˜ + + -1= 0
Ë 2 2¯ 2 2
fi a +b =l
2 2 2
…(i) Ê a 3b ˆ
Let the mid-point of A and B be (a, b) fi 8Áa 2 + b 2 - - ˜ +4=0
Ë 2 2¯
a b fi 2(a2 + b2) – a – 3b + 1 = 0
Thus, a = , b =
2 2 Hence, the locus of (a, b) is
fi a = 2a, b = 2b 2(x2 + y2) – x – y + 1 = 0,
From Eq. (i), we get, 40. Let OA = a and OB = b.
l2 Since the circle touches both the axes and the line AB,
a2 + b2 =
4 so the inradius of DOAB be 2.
l2
Hence, the locus of (a, b) is x 2 + y 2 = . Y
4
38. Let the point M be (a, b) and A(2a, 2b) where M is the B
mid-point of OA.
Y
A

M X¢ O X
A
X¢ X
O



From trigonometry, we can write
D
Since A lies on the curve y2 = 8x, so r= , …(i)
(2b)2 = 8(2a) s
fi 4b2 = 16a where D = area of a triangle
fi b 2 = 4a and s = semi-perimeter of the triangle.
Hence, the required locus of M is y2 = 4x. Let (x, y) be the circumcentre of DOAB, then
a b
39. We have x = and y =
x2 + y2 + x + y = 0 2 2
2 2 2 Thus, a = 2x and b = 2y.
Ê 1ˆ Ê 1ˆ Ê 1 ˆ
fi ÁË x + ˜ +Áy+ ˜ =
2 ¯ ÁË 2 ˜¯
Putting the values of a and b in Eq. (i), we get
2¯ Ë
1
Y ◊ (2x) ◊ (2y )
2= 2
Ê 2x + 2y + 4x 2 + 4y 2 ˆ
A ÁË ˜¯
2
Q P
O fi xy = x + y + x 2 + y 2
X¢ ½ X
is the required locus.
C(–½, –½) 44. x2 + y2 = 4
45. x2 + y2 = 3
46. Given curve is
Y¢ 2x2 + y2 – 3x + 5y – 8 = 0 (i)
Let the point Q be Replacing x by x – 1 and y by y + 2 in Eq. (i), we get
Ê 1 1 1 1 ˆ 2(x – 1)2 + (y + 2)2 – 3(x – 1) + 5(y +2) – 8 = 0
ÁË - 2 + cos q , - +
2
sin q ˜
¯
2 2 fi 2(x2 – 2x + 1) + (y2 + 4y + 4)
and P be (a, b) –3(x – 1) + 5(y + 2) – 8 = 0
fi 2x2 + y2 – 7x + 9y + 11 = 0

CG_01.indd 35 2/6/2017 3:57:59 PM


1.36 Coordinate Geometry Booster

47. Given curve is 3. Let m1 be the line joining the points (2, –3) and (– 5, 1)
x2 + y2 = 36 …(i) and m2 be the line joining the points (7, –1) and (0, 3).
Replacing x by x – 4 and y by y – 5 in Eq. (i), we get 1+ 3 4 3 +1 4
Thus m1 = = - and m2 = =- .
(x – 4)2 + (y – 5)2 = 36 -5 - 2 7 0-7 7
fi x2 + y2 – 8x – 10y + 16 + 25 – 36 = 0 Since, m1 = m2, so the slopes are parallel.
fi x2 + y2 – 8x – 10y + 5 = 0 4. Consider the points P = (a, b + c),
which is the required equation of the original axes. Q = (b, c + a), R = (c, a + b)
48. Let the point be (x, y). c+a-b-c a-b
Thus x – 1 = –7, y + 8 = 3 Therefore, m ( PQ) = = = -1
b-a b-a
fi x = –6, y = –5
a+b-c-a b-c
Hence, the point be (– 6, – 5). and m (QR) = = = -1
c-b c-b
49. Given equation is
Since, m(PQ) = m(QR)
3x2 + 2xy + 3y2 = 2 (i)
Replacing x by (x cos45° – y sin45°) fi Thus, the points P, Q and R are collinear.
and y by (x cos45° + y sin45°) in Eq. (i), we get 5. Let m1 and m2 be the slopes of the line joining the points
2 2 (0, 0), (2, 2) and (2, –2), (3, 5).
Ê x - yˆ Ê x + yˆ Ê x - yˆ Ê x + yˆ
3Á + 3Á + 2Á =2 2-0 5+2
Ë 2 ˜¯ Ë 2 ˜¯ Ë 2 ˜¯ ÁË 2 ˜¯ Therefore, m1 = = 1 and m2 = =7
2-0 3- 2
fi 3(x2 + y2) + (x2 – y2) = 2 Let q be the angle between them
fi 4x2 + 2y2 = 2
fi 2x2 + y2 = 1 Ê 7 -1 ˆ
tan q = Á
50. Given equation is Ë 1 + 7.1˜¯
y2 – 4x + 4y + 8 = 0 (i)
3
Replacing x by x + 1 and y by y – 2 in Eq. (i), we get fi tan q =
(y – 2)2 – 4 (x + 1) + 4 (y – 2) + 8 = 0 4
fi y2 – 4y + 4 – 4x – 4 + 4y – 8 + 8 = 0 Ê 3ˆ
fi q = tan -1 Á ˜
fi y2 = 4x Ë 4¯
which is the required transformed equation. 6. Let m be the slope of the other line.
51. If the point P (1, 2) is translated itself 2 units along the Given,
positive direction of x-axis, then P becomes (3, 2).
1
Let Z = (3, 2) = 3 + 2i m-
tan 45° = 2
Thus, the new position of P = iZ m
= i(3 + 2i) 1+
2
= –2 + 3i = (–2, 3)
52. 12 2m - 1
fi tan 45° =
53. x2 – y2 = a2 2+m
54. 4x2 + 2y2 = 1 fi (2m – 1) = ± (2 + m)
55. (–3, 2) fi (2m – 1) = (2 + m)
Ê1 - 4 3 4 + 3ˆ and (2m – 1) = – (2 + m)
56. ÁË , ˜
2 2 ¯ 1
fi m = 3 and m = -
3

LEVEL I Hence, the slopes of the other line are 3 or –1/3.


7. Equation of a line parallel to x-axis is y = k.
(STRAIGHT LINE) Which is passing through (2, –3), we have, k = –3.
y - y 10 - 4 Hence, the equation of the line is y + 3 = 0.
1. Hence, the required slope of PQ = 2 1 = =6 8. Equation of a line perpendicular to y-axis is y = k which
x2 - x1 3 - 2
2. Given slope = 2 is passing through the point (3, 4), we have, k = 4.
7-5 Hence, the equation of the line is y = 4.
fi =2
l-2 9. Hence, the equation of a line, which is equidistant from
fi l–2=1 6 + 10
the lines x = 6 and x = 10 is x = =8
fi l=3 2

CG_01.indd 36 2/6/2017 3:58:00 PM


Straight Lines 1.37

10. Given m = 3 and c = 7. 18. We have,


Hence, the required equation of the line is m = tan(15°) = (2 - 3)
y = mx + c = 3x + 7 Y
11. Here, m = tan (135°) = –1 and c = 5.
Hence, the required equation of the given line is 15°
y = mx + c = – x + 5
12. Given, 30°
Ê 3ˆ
q = tan -1 Á ˜ X
Ë 5¯ O A(2, 0)

3 Hence, the required equation of the line is


fi tan q =
5
y - 0 = (2 - 3)( x - 2)
3
fi m= and c = –6 fi
5 (2 - 3) x - y - 2(2 - 3) = 0
Hence, the equation of the given straight line is 19. Hence, the equation of a line is
3 x - x1 y - y1
y = mx + c = x - 6 =0
5 x1 - x2 y1 - y2
13. Given m = ±1 and c = 4.
x -1 y - 2
Hence, the equation of the line is fi =0
y = mx + c = ±x + 4 3 -1 4 - 2
14. Here, m = tan (45°) = 1 fi 2(x – 1) – 2(y – 2) = 0
and m = tan (135°) = 1 and c = 0 fi x – y + 1 = 0.
Hence, the equation of the bisectors is
-1 - 9 10
y = mx + c = ± x 20. Here slope of BC = = - = -5
-2 + 4 2
15. Here, m = tan(120°) = - 3 and the point (x1, y1) = (2, 3) Hence, the equation of the altitude through A is
Hence, the equation of a straight line is
1
y – y1 = m(x – x1)a y - 4 = ( x - 10)
5
fi y - 3 = - 3 ( x - 2) fi x – 5y + 10 = 0
fi 3x + y = 2 3 + 3 21. The co-ordinates of the mid points of B and C are
16. Let the points A and B are (1, 2) and (5,7), respectively. Ê 2 + 5 3 + 4ˆ Ê 7 7ˆ
ÁË , ˜ =Á , ˜
7-2 5 2 3 ¯ Ë 2 2¯
Therefore, m( AB ) = = Hence, the required equation of the median through A
5 -1 4
x -1 y - 2
and the mid-point of A and B are
is 7 7 =0
Ê1 + 5 2 + 7ˆ Ê 9ˆ -1 -2
ÁË , ˜ = Á 3, ˜ 2 2
2 2 ¯ Ë 2¯
fi 3(x – 1) – 5(y – 2) = 0
Hence, the equation of the right (perpendicular) bisec-
fi 3x – 5y + 7 = 0
tor is
9 4 22. Here, AB = 10, AC = 10, BC = 2 10
y - = - ( x - 3)
2 5 A
Let AD is the internal bi-
fi 8x + 10y = 69 sector of the angle –BAC.
17. Given, If AD is the internal bisec-
3 tor of the angle –BAC, then
cos q = -
5 BD AB 10 1
4 = = =
fi tan q = - DC AC 10 1
5 B D C
4 fi BD : DC = 1 : 1
fi m=-
5 Thus D is the mid-point of B and C, i.e. D = (4, 4)
Hence, the equation of the given line is Hence, the equation of the internal bisector is
4 4-2
y - 2 = - ( x - 1) y-2= ( x - 5) = - 2( x - 5)
5 4-5
fi 4x + 5y = 14 fi 2x + y = 12

CG_01.indd 37 2/6/2017 3:58:01 PM


1.38 Coordinate Geometry Booster

23. Let the square be PQRS, whose length of the side is k. x y


Consider the point of P be (a, 0). 26. Let the equation of the line is + = 1 …(i)
a b
B Given condition is a = 2b …(ii)
From Eqs (i) and (ii), we get,
2x + y = 2b
S R Which is passing through the point (2, 3).
Therefore, 2b = 4 + 3 = 7
7
fi b=
A(0, 0) P Q C(3, 0) 2
Then Q = (a + k, 0), R = (a + k, k) and S = (a, k). Hence, the equation of the line is 2x + y = 7.
x y
The equation of the line AB is 27. Let the equation of the line be + = 1 ,
1- 0 a b
y-0= ( x - 0) where A = (a, 0) and B = (0, b).
2-0
Consider the given line is bisected at the point M (2, 3).
fi x – 2y = 0 …(i)
a
and the equation of the line BC is Thus, 2 = fi a=4
2
0 -1 b
y -1= ( x - 2) and 3 = fi b=6
3- 2 2
fi x+y=3 …(ii) Hence, the equation of the line is
Since, the point S(a, k) on Eq. (i), we get, x y
a = 2k + = 1.
4 6
and the point R(a + k, k) on Eq. (ii), we get, fi 3x + 2y = 12
a + 2k = 3. x y
Thus, k = 3/4 and a = 3/ 2. 28. Let the equation of the line be + = 1 , where A = (a,
0) and B = (0, b) a b
Hence, the co-ordinates of P, Q, R and S are
(3/2, 0), (9/4, 0), (9/4, 3/4) and (3/2, 3/4). Given condition is a = –b
24. Let slope of AC is m. The slope of AB is 1. We have Therefore, x – y = a which is passing through (3, –4)
Y Thus, a = 7.
C Hence, the equation of the line is x – y = 7.
x y
15° B 29. Let the equation of the line is + = 1
a b
which is passing through (3, 2), so
X
O A 3 2
+ =1 …(i)
a b
Also given condition is a – b = 2 …(ii)
m -1 From Eq. (i) and (ii), we get
tan(15°) =
1+ m 3b + 2a = ab = b(b + 2)
fi 3b + 2b + 4 = b2 + 2b
fi (2 - 3)m + (2 - 3) = m - 1
fi b2 – 3b – 4 = 0
fi (1 - 3) m = (-3 + 3) fi (b – 4) (b + 1) = 0
fi b = –1, 4
fi (1 - 3) m = (-3 + 3) = 3(1 - 3)
Thus, a = 1, 6.
fi m= 3 Hence, the equation of the line are
Hence, the equation of the line AC is x – y = 1, and 2x – 3y = 12.
30. Let the equation of the line AB is
y - 0 = 3 ( x - 2) x y
2x + 3y = 6 fi + =1
3 2
fi x 3- y-2 3=0 Given lines are x y = 0
x y fi x = 0, y = 0
25. Let the equation of the line is + =1.
a b 1
Hence, the area of a triangle = ◊ 3 ◊ 2 = 3 sq. u.
Since, the line makes equal intercepts with the axes, so 2
a = b. 31. 2x + y = 10.
Thus x + y = a which is passing through (3, 4). 32. 3x – 8y = 0, 3x – 2y = 0.
Therefore a = 7. 33. Do yourself
Hence, the equation of the required line is x + y = 7. 34. Do yourself.

CG_01.indd 38 2/6/2017 3:58:02 PM


Straight Lines 1.39

35. Here, p = 5 and a = 60° Thus, the co-ordinates of Q are


Hence, the equation of the line is Ê 2 2 ˆ
xcosa + ysina = p ÁË1 + cos q , 2 + sin q ˜
¯
3 3
fi xcos (60°) + ysin (60°) = 5
Since, the point Q lies on x + y = 4, so
fi x + 3 y = 10 2 2
5 1+ cos q + 2 + sin q = 4
36. Here, p = 2 and tan q = 3 3
12
5 12 2 2
fi sin q = and cos q = fi cos q + sin q = 1
13 13 3 3
Hence, the equation of the line is 3
fi cos q + sin q =
xcosq + ysinq = p 2
fi 12x + 5y = 26. 1 1 3
fi cos q + sin q =
37. Hence, the co-ordinates of the required points are 2 2 2
(x1 ± rcosq, y1 ± rsinq). Ê pˆ Êpˆ
fi ÁË q - ˜¯ = ± ÁË ˜¯
Here r = 5, (x1, y1) = (3, 2) 4 6
3 4 3 p p
and tan q = fi cos q = , sin q = fi q = ± = 75°, 15° .
4 5 5 4 6
Thus, the points are 42. R(–2, –1), Q(–1, 2) and S(1, –2)
(x1 ± rcosq, y1 ± rsinq) Ê 3 3ˆ Ê 1 3ˆ
43. ÁË - , ˜¯ and ÁË , - ˜¯
Ê 4 3ˆ 2 2 2 2
= Á3 ± 5 ◊ , 2 ± 5 ◊ ˜
Ë 5 5¯ 44 2
= (3 ± 4, 2 ± 3) 45. 6
= (7, 5), (–1, –1) 46. 4 2
38. Here, r = 4 2 , (x1, y1) = (–2, 3) and q = 45°. 47. 5
Hence, the co-ordinates of the points are 48. 5.
(x1 ± rcosq, y1 ± rsinq) 49. (8, 2) and (0, –4)
Ê 1 1 ˆ 50. Given line is x + 3 y + 4 = 0
= Á -2 ± 4 2 ◊ ,3± 4 2 ◊ ˜
Ë 2 2¯ (i) slope intercept form is
= (–2 ± 4, 3 ± 4) Ê 1 ˆ Ê 4 ˆ
= (2, 7) and (–6, –1) fi y = Á- ˜ x + Á- ˜
Ë 3¯ Ë 3¯
39. Given point P is (3, 4). 1
Here, r = 2 and q = 45° Thus, slope = -
3
Let Q be the new position of P. 4
and y-intercept = -
Hence, the co-ordinates of Q are 3
Ê 1 1 ˆ (ii) Intercept form of x + 3 y + 4 = 0 is
ÁË 3 + 2 ◊ ,4+ 2◊ ˜ = (4, 5)
2 2¯ x y
+ =1
40. Let the new position of B is C. (-4) Ê 4 ˆ
ÁË - ˜¯
2 2
Here, r = AB = AC = (3 - 2) + (1 - 0) = 2 and 3
q = 45° + 15° = 60° where x-intercept = 4 and
Thus, the co-ordinates of C are y-intercept = 4/ 3 .
(2 + 2 cos (60∞), 0 + 2 sin(60∞))
(iii) Normal form of x + 3 y + 4 = 0 is
Ê 1 3ˆ Ê 1 3ˆ
= Á2 + 2 ◊ , 2 ◊ ˜ = Á2 + , ˜ -x - 3y = 4
Ë 2 2 ¯ Ë 2 2¯
Ê 1ˆ Ê - 3ˆ
41. Let the point P be (1, 2) and the line PQ makes an angle fi ÁË - ˜¯ x + ÁË ˜ y =1
q with the positive direction of x-axis. 4 4 ¯
2 fi x cos a + y sin a = 1,
Here, r = and (x1, y1) = (1, 2)
3 where a = p + tan -1 ( 3) = 240°

CG_01.indd 39 2/6/2017 3:58:03 PM


1.40 Coordinate Geometry Booster

51. (i) Let (x1, y1) = (2, 2) and (x2, y2) = (3, 5) and a = 2, Therefore, 3 + 0 + k = 0
b = 3, c = 4. fi k = –3
ax1 + by1 + c 2.2 + 3.2 + 4 Hence, the equation of the line is
Now, =
ax2 + by2 + c 2.3 + 3.5 + 4 3x + 2y + 3 = 0.
60. The slope of the line joining the points (1, 2) and (3, 5)
14
= >0 5-2 3
25 is = .
3 -1 2
Thus, the points (2, 2) and (3, 5) lie on the same The mid-point of the line joining the points (1, 2) and
side of the line 2x + 3y + 4 = 0.
Ê 1 + 3 2 + 5ˆ Ê 7 ˆ
(ii) Do yourself (3, 5) is Á , ˜ = Á 2, ˜ .
Ë 2 2 ¯ Ë 2¯
(iii) Do yourself.
52. As we know that, if ax1 + by1 + c and c have the same Hence, the equation of the line is
sign, then the point (x1, y1) lies on the origin side of the 7 2
y - = - ( x - 2)
line ax1 + by1 + c. 2 3
Here, 2 > 0 and fi 4x + 6y = 29
ax1 + by1 + c = 2. 2 – 7 + 2 = 6 – 7 = – 1 < 0
8-5
Thus, the point (2, –7) does not lie on the origin side of 61. Slope of BC = = -3
the line 2x + y + 2 = 0. 3- 4
53. Do yourself. Let the altitude through A is AD.
54. Equation of a line parallel to 1
3x + 4y + 5 = 0 is 3x + 4y + k = 0. Therefore, the slope of AD is .
3
which passes through (2, 3).
Hence, the equation of the altitude through A is
Therefore, 6 + 12 + k = 0
fi k = –18 1
y - 2 = ( x - 1)
Hence, the equation of the line is 3
3x + 4y – 18 = 0. fi x – 3y + 5 = 0
55 The co-ordinates of the mid-point of the line joining 62. The point of intersection of 2x + y = 8 and x – y = 10 is
the points (2, 3) and (4, –1) is (3, 1). (6, – 4).
Equation of any line parallel to 3x – 4y + 6 = 0 is Equation of any line perpendicular to
3x – 4y + k = 0 3x + 4y + 2012 = 0 is
which is passing through (3, 1). 4x – 3y + k = 0
Therefore, 9 – 4 + k = 0 which is passing through (6, – 4).
fi k = –5 Therefore, 24 + 12 + k = 0 fi k = –36
Hence, the equation of the line is Hence, the equation of the line is
3x – 4y – 5 = 0 4x – 3y – 36 = 0.
56. The slope of the line joining the points (2, 3) and 63. Hence, the distance of the point (4, 5) from the straight
-1 - 3 line 3x – 5y + 7 = 0 is
(3, – 1) is = -4
3- 2 3 ¥ 4 - 5 ¥ 5 + 7 12 - 25 + 7 6
= = .
Equation of the line parallel to the given line and pass- 2
3 +5 2 34 34
ing through (2, 1) is
y – y1 = m(x – x1) 64. Let ABC be an equilateral triangle, where A is (2, –1)
fi y – 1 = –4(x – 2) and BC is x + y = 2.
fi 4x + y = 9 Let AD be the length of perpendicular from A on BC.
57. Do yourself. 2 -1- 2 1
58. Equation of any line perpendicular to Thus, AD = =
12 + 12 2
2x + 3y – 2012 = 0 is
3x – 2y + k = 0 Therefore,
which is passing through (3, 4). AD
sin (60°) =
Therefore, 9 – 8 + k = 0 AB
fi k = –1
Hence, the equation of the line is 2 1 2
fi AB = ¥ =
3x – 2y – 1 = 0. 3 2 3
59. Equation of any line perpendicular to 2x – 3y – 5 = 0 is
2
3x + 2y – k = 0 which is passing through (1, 0). Thus, the length of the side =
3

CG_01.indd 40 2/6/2017 3:58:05 PM


Straight Lines 1.41

65. Equation of a straight line, whose intercepts are a and b 71. As we know that, the three lines aix + biy + ci = 0, i = 1,
x y 2, 3 are concurrent, if
is + = 1 .
a b a1 b1 c1
Given, the length of the perpendicular from the origin a2 b2 c2
= p. a3 b3 c3
0 + 0 -1
fi =p 2 -3 5
1 1
+ = 3 4 -7
a 2 b2
9 -5 8
1 1 1
fi + = = 2(32 – 35) + 3(24 + 63) + 5(–15 – 36)
a 2 b2 p 2
= –6 + 261 – 255
66. Clearly, both the lines are parallel =0
Hence, the required distance Hence, the three lines are concurrent.
11 72. Equation of any line passing through the point of inter-
+5 11 + 10 21 1 section of the lines
= 2 = = = 2 units.
2 2 2.5 10 10 x + 3y – 8 = 0 and 2x + 3y + 5 = 0 is
3 +4
(x – 3y + 8) + l(2x + 3y + 5) = 0 …(i)
x y which is passing through (1, 2).
67. Equation of the line L is + =1.
a b Therefore, (1 – 6 + 8) + l(2 + 6 + 5) = 0
x y 3
After rotation, the line L becomes, + =1. fi l=-
p q 13
Therefore, the lengths of perpendicular from From Eq. (i), we get
the origin to both the lines are same. Thus, 3
0 + 0 -1 0 + 0 -1 ( x - 3y + 8) - (2x + 3y + 5) = 0
= 13
1 1 1 1 fi 13(x – 3y + 8) – 3(2x + 3y + 5) = 0
+ +
a 2 b2 p2 q2 fi 7x – 48y + 89 = 0
73. On solving y = x + 1 and 2x + y = 16, we get,
1 1 1 1
fi 2
+ 2 = 2 + 2 x = 5 and y = 6.
a b p q Thus, the point of concurrency is (5, 6)
Hence, the result. Since, the given lines are concurrent, so
68. As we know that, area of a parallelogram whose sides 6 = 5m – 4
are a1x + b1y + d1 = 0, a2x + b2y + c2 = 0 fi 5m = 10
and a2x + b2y + d2 = 0 is fi m=2
Hence, the value of m is 2.
(c1 - d1 )(c2 - d 2 ) 74. Since the given lines ax + y + 1 = 0, x + by + 1 = 0 and
=
(a1b2 - a2b1 ) x + y + c = 0 are concurrent,
(4a + a ) ¥ (7a - 3a ) 20a 2 a 1 1
= = so 1 b 1 =0
(1 ¥ (-2) - 3 ¥ 3) 11
1 1 c
20
Thus, m = . a -1 0 1
11
fi 0 b -1 1 = 0
Now 11m + 30 = 20 + 30 = 50
69. Given four lines are ax + by + c = 0, 1- c 1- c c
ax + by – c = 0, ax – by + c = 0 fi (a – 1)(c(b – 1) – (1 – c)) – (1 – c)(b – 1) = 0
and ax – by – c = 0. fi c(a – 1)(b – 1) – (a – 1)(1 – c) – (1 – c)(b – 1) = 0
(c + c)(-c - c) fi c(1 – a)(1 – b) + (1 – a)(1 – c) + (1 – c)(1 – b) = 0
Hence, the area =
( a ◊ ( -b) - b ◊ a ) c 1 1
fi + + =0
1- c 1- b 1- a
4c 2 2c 2 2c 2 c 1 1
= = = fi 1+ + + =1
2ab ab | ab | 1- c 1- b 1- a
70. Hence, the points of intersection of the given lines is 1 1 1
(2, 6). fi + + =1
1- a 1- b 1- c

CG_01a.indd 41 2/6/2017 3:58:36 PM


1.42 Coordinate Geometry Booster

75. Given family of lines are fi 2m = –4 and 4m = 2


ax + by + c = 0 …(i) 1
and also the given condition is fi m = - 2 and m =
2
2a + 3b + 6c = 0 …(ii)
From Eqs (i) and (ii), we get 82. Therefore,
a b 1
ax + by - - = 0 m-
3 2 tan(45°) = 2
m
Ê 1ˆ Ê 1ˆ 1+
fi aÁ x - ˜ + bÁ y - ˜ = 0 2
Ë 3¯ Ë 2¯
2m - 1
Ê 1ˆ b Ê 1ˆ fi =1
fi ÁË x - ˜¯ + ÁË y - ˜ =0 2+m
3 a 2¯
Ê 1ˆ Ê 1ˆ 2m - 1 2m - 1
fi fi = 1 and = -1
ÁË x - ˜¯ = 0 and ÁË y - ˜¯ = 0 2+m 2+m
3 2
1 1 1
fi x = and y = fi m = 3 and m = -
3 2 3
78. We have 4a + 9b – c2 + 12ab = 0
2 2 Hence, the equation of the lines are
fi (4a2 + 9b2 + 12ab) – c2 = 0 1
y - 2 = 3( x - 3) and y - 2 = - ( x - 3)
fi (2a2 + 3b2) – c2 = 0 3
fi (2a + 3b + c)(2a + 3b – c) = 0 fi 3x – y – 7 = 0 and x + 3y – 9 = 0
fi (2a + 3b + c) = 0 and (2a + 3b – c) = 0
83. Let ABC be an equilateral triangle, where A is (2, 3)
Given family of lines are and BC is x + y = 2.
ax + by + c = 0 …(i)
Equation of any line passing through (2, 3) is
and also thegiven conditions are
y – 3 = m(x – 2).
(2a + 3b + c) = 0 …(ii)
The slope of the line x + y = 2 is (– 1).
and (2a + 3b – c) = 0 …(iii)
From Eqs (i) and (ii), we get Therefore,
m +1
ax + by – 2a – 3b = 0 tan (60°) =
fi a(x – 2) + b(y – 3) = 0 1- m
b m +1
fi ( x - 2) + ( y - 3) = 0 fi =± 3
a 1- m
fi x = 2, y = 3 m +1 m +1
fi m = 2, n = 3 fi = 3 and =- 3
1- m 1- m
From Eqs (i) and (iii), we get
ax + by + 2a + 3b = 0 fi m ( 3 + 1) = ( 3 - 1) and m ( 3 + 1) = ( 3 - 1)
fi a(x + 2) + b(y + 3) = 0
Ê 3 - 1ˆ Ê 3 + 1ˆ
b fi m=Á
fi ( x + 2) + ( y + 3) = 0 ˜ and m = Á ˜
a Ë 3 + 1¯ Ë 3 - 1¯
fi x = –2, y = –3 fi m = 2 - 3 and m = 2 + 3
fi p = –2, q = – 3
Hence, the equations of the line are
Thus, the value of
m + n + p + q +10 = 2 + 3 – 2 – 3 + 10 = 10 y - 3 = (2 - 3)( x - 2)
81. Let m be the slope of the given line and the slope of and y - 3 = (2 + 3)( x - 2)
3x – y + 5 = 0 is 3.
Therefore, fi (2 + 3) x - y - (2 3 + 1) = 0 and
m-3 (2 - 3) x - y + (2 3 - 1) = 0
tan 45° =
1 + 3m 84. Let AB: 4x + y = 1,
m-3 AC: 3x – 4y + 1 = 0
fi = ±1
1 + 3m and slope of AC is m.
m-3 m-3 The slope of AB = –4 and the slope of AC = 3/4.
fi = 1 and = -1 Let –ABC = q = –ACB
1 + 3m 1 + 3m

CG_01a.indd 42 2/6/2017 3:58:36 PM


Straight Lines 1.43

fi 9m(7m – 9) + 7(7m – 9) = 0
Ê 3 ˆ Ê 3ˆ
-m -4 - fi (7m – 9)(9m + 7) = 0
Á 4 ˜ Á 4˜
Thus, Á = 9 7
3 ˜ Á 3˜ fi m= ,-
ÁË 1 + m ˜¯ ÁË 1 - 4 ◊ ˜¯ 7 9
4 4
From Eqs (i), we get,
Ê 3 - 4m ˆ Ê -16 - 3 ˆ
fi ÁË 4 + 3m ˜¯ = ÁË 4 - 12 ˜¯ 9 7
y - 3 = ( x - 2) and y - 3 = - ( x - 2)
7 9
Ê 3 - 4m ˆ 19 fi 9x – 7y + 3 = 0 and 7x + 9y = 41
fi ÁË 4 + 3m ˜¯ = 8
which is the equations of the lines.
fi 57m + 76 = 24 – 32m 87.
fi 89m = 24 – 76 = –52
52
fi m=-
89 (1, 2)
C
B A
Hence, the equation of AC is
52
y + 7 = - ( x - 2)
89
Let AB: 3x + 4y = 5 …(i)
fi 52x + 89y + 519 = 0 and 4x – 3y = 15 …(ii)
85. Let PQ line intersects the line 4x + 3y = 12 at A and On solving Eqs (i) and (ii), we get
4x + 3y = 3 at B, respectively. x = 3 and y = 1
Equation of any line PQ passing through (–2, –7) is Thus, the co-ordinates of the point A be (3, 1).
y + 7 = m(x + 2) …(i) Equation of any line BC which passes through the point
Ê 33 - 6m 20m - 28 ˆ (1, 2) is y – 2 = m(x – 1) …(iii)
Thus, A = Á ,
Ë 4 + 3m 4 + 3m ˜¯
On solving Eqs (i) and (iii), we get the co-ordinate B,
i.e.
Ê 24 - 6m 11m - 28 ˆ Ê 4m - 3 2m + 6 ˆ
and B=Á , B= Á , .
Ë 4 + 3m 4 + 3m ˜¯ Ë 4m + 3 4m + 3 ˜¯
Given, AB = 3 fi AB2 = 9 On solving Eqs (ii) and (iii), we get the co-ordinate C,
81 81m 2 i.e.
fi + =9
(4 + 3m) 2 (4 + 3m) 2 Ê 21 - 3m 11m + 8 ˆ
C= Á , .
fi 9 + 9m2 = 16 + 24m + 9m2 Ë 4 - 3m 4 - 3m ˜¯
fi 24m = –7 Given condition is AB = AC
fi m = –7/24 and m = fi AB2 = AC2
2 2
Hence, the line PQ can be x + 2 = 0 and Ê 21 - 3m ˆ Ê 8 + 11m ˆ
fi x + 2 = 0 and 7x + 24y + 182 = 0 fi ÁË 4 - 3m - 3˜¯ + ÁË 4 - 3m + 1˜¯
86. Equation of any line passing through (2, 3) is
2 2
y – 3 = m(x – 2) …(i) Ê 4m - 3 ˆ Ê 6 + 2m ˆ
=Á -3 + +1
Equation (i) is equally inclined with the lines Ë 4m + 3 ˜¯ ÁË 4m + 3 ˜¯
3x – 4y = 7 …(ii)
(9 + 6m) 2 + (12 + 8m) 2
and 12x – 5y + 6 = 0 …(iii) fi
12 ˆ (4 - 3m) 2
Ê 3ˆ Ê
m- m-
Á 4 ˜ Á 5 ˜ (-8m - 12) 2 + (6m + 9) 2
Therefore, Á = -Á =
3 ˜ 12 ˜ (4m + 3) 2
ÁË 1 + m ˜¯ Á1 + m˜
Ë
4 5 ¯ fi (4m + 3)2(100m2 + 300m + 225)
Ê 3ˆ Ê 12 ˆ = (4 – 3m)2(100m2 + 300m + 225)
m- m-

Á 4 ˜ = -Á 5 ˜ fi 2
(100m + 300m + 225)
Á 3 ˜ Á 12 ˜
ÁË 1 + m ˜¯ Á1 + m˜
Ë [(4m + 3)2 – (4 – 3m)2] = 0
4 5 ¯
fi (4m + 12m + 9)[(4m + 3)2 – (4 – 3m)2] = 0
2
Ê 4m - 3 ˆ Ê 12 - 5m ˆ
fi ÁË 4 + 3m ˜¯ = ÁË 5 + 12m ˜¯ fi (2m + 3)2(m + 7)(7m – 1) = 0
3 1
fi 63m2 – 32m – 63 = 0 fi m = - , - 7,
2 7
fi 63m2 – 81m + 49m – 63 = 0

CG_01a.indd 43 2/6/2017 3:58:37 PM


1.44 Coordinate Geometry Booster

3 92. The given lines are 3x – 4y + 7 = 0


Put m = - in Eq. (iii), we get and –12x – 5y + 2 = 0.
2
3 Now, a1a2 + b1b2 = –36 + 20 = –16 < 0.
y - 2 = - ( x - 1) Therefore, negative one is the obtuse-angle bisector.
2
fi 3x + 2y – 7 = 0 Hence, the equation of the bisector of the obtuse angle
is
Clearly, it passes through A(3, –1).
Ê 3x - 4y + 7 ˆ Ê -12x - 5y + 2 ˆ
Thus, it can not be the equation of BC. =-
Á 2 2 ˜ Á ˜
Put m = –7 in Eq. (iii), we get, Ë 3 +4 ¯ Ë 122 + 52 ¯
y = –7(x – 1) = –7x + 7 Ê 3x - 4y + 7 ˆ Ê -12x - 5y + 2 ˆ
Thus, the equation of BC is 7x + y = 7. fi ÁË ˜¯ = - ÁË ˜¯
5 13
Also, put m = 1/7 in Eq. (iii), we get
fi 21x + 77y = 101
1
y - 2 = ( x - 1) 93. The given lines are –x – y + 3 = 0 and 7x – y + 5 = 0.
7
Now, a1a2 + b1b2 = –7 + 1 = –6 < 0.
fi x – 7y + 13 = 0
Therefore, positive one is the acute-angle bisector.
Hence, the equations of BC can be x – 7y + 13 = 0 and
Hence, the equation of the acute-angle bisector is
7x + 7 = 7.
Ê - x - y + 3 ˆ Ê 7x - y + 5 ˆ
88. 3x + y + 7 = 0 and x – 3y = 31. =
Á 2 2 ˜ Á 2 2 ˜
Do yourself. Ë 1 +1 ¯ Ë 7 +1 ¯
89. Hence, the equations of the bisectors are Ê - x - y + 3 ˆ Ê 7x - y + 5 ˆ
3x - 4y + 7 12x + 5y - 2 fi ÁË ˜¯ = ÁË ˜
= 2 5 2 ¯
32 + 42 122 + 52 fi 5(–x – y + 3) = (7x – y + 5)
Ê 3x - 4y + 7 ˆ Ê 12x + 5y - 2 ˆ fi 6x – 2y = 5
fi ÁË ˜¯ = ± ÁË ˜¯ 94. The lengths of perpendicular from any point on the line
5 13
7x – 9 y + 10 = 0 to the lines 3x + 4y = 5 and 12x + 5y
fi 13(3x – 4y + 7) = 5(12x + 5y – 2) = 7 are same if 7x – 9y + 10 = 0 is the bisector of
fi 13(3x – 4y + 7) = 5(12x + 5y – 2) 3x + 4y = 5 and 12x + 5y = 7.
and 13(3x – 4y + 7) = –5(12x + 5y – 2) Hence, the equation of bisectors is
fi 21x + 77y = 101 and 3x – y + 3 = 0 Ê -3x - 4y + 5 ˆ Ê -12x - 5y + 7 ˆ
=
90. Hence, the equation of the bisector, containing the ori- Á ˜ Á ˜
Ë 32 + 42 ¯ Ë 122 + 52 ¯
gin is
Ê -4x - 3y + 6 ˆ Ê 5x + 12y + 9 ˆ fi 13(–3x – 4y + 5) = 5(–12x – 5y + 7)
= fi 21x – 27y + 30 = 5
Á ˜ Á ˜
Ë 42 + 32 ¯ Ë 52 + 122 ¯
fi 7x – 9y + 10 = 0
fi Ê -4x - 3y + 6 ˆ Ê 5x + 12y + 9 ˆ Hence, the result.
ÁË ˜¯ = ÁË ˜¯
5 13 95. Let ABC be a triangle,
where AB: x + 1 = 0, BC: 3x – 4y = 5 and CA: 5 x + 12y
fi 13(–4x – 3y + 6) = 5(5x + 12y + 9)
= 27.
fi 7x + 9y = 3
Case I: Acute-angle bisector between AB and AC
91. The given lines are –2x – y + 6 = 0 The given lines are x + 1 = 0 and –3x + 4y + 5 = 0.
and 2x – 4y + 7 = 0. Now, a1a2 + b1b2 = 1.(–3) + 0 = –3 < 0.
Therefore, –2. 1 – 2 + 6 = –4 + 6 = 2 Thus, positive one is the acute-angle bisector.
and 2. 1 – 4. 2 + 7 = 9 – 8 = 1 Hence, the acute-angle bisector is
Thus, positive one is the equation of the bisector. Ê x + 1ˆ Ê -3x + 4y + 5 ˆ
=
Hence, the equation of bisector is ÁË 2 ˜¯ Á ˜
1 Ë 32 + 42 ¯
Ê -2x - y + 6 ˆ Ê 2x - 4y + 7 ˆ
= Ê x + 1ˆ Ê -3x + 4y + 5 ˆ
Á 2 2 ˜ Á 2 2 ˜ fi ÁË ˜ =Á
Ë 2 +1 ¯ Ë 2 + 4 ¯ ˜¯
1 ¯ Ë 5
Ê -2x - y + 6 ˆ Ê 2x - 4y + 7 ˆ fi 5(x + 1) = (–3x + 4y + 5)
fi ÁË ˜¯ = ÁË ˜¯ fi 2x – y = 5
5 2 5 …(i)
Case II: The acute-angle bisector between BC and
fi 2(–2x – y + 6) = (2x – 4y + 7)
AC.
fi 6x – 2y = 5
The given lines are –3x + 4y + 5 = 0

CG_01a.indd 44 2/6/2017 3:58:37 PM


Straight Lines 1.45

and –5 x – 12y + 27 = 0. On solving Eqs (i) and (iii), we get


Now, a1a2 + b1b2 = 15 – 48 = –23 < 0 x = 6 + 2 2, y = -1 - 2 2
Thus, the positive one is the acute-angle bisector. Thus, the co-ordinates of C can be
Hence, the acute-angle bisector is
(6 + 2 2, 1 - 2 2)
Ê -3x + 4y + 5 ˆ Ê -5x - 12y + 27 ˆ
= Again, solving Eqs (i) and (iv), we get
Á ˜ Á ˜
Ë 32 + 42 ¯ Ë 52 + 122 ¯ x = 2 - 2 2, y = 3 + 2 2

fi Ê -3x + 4y + 5 ˆ Ê -5x - 12y + 27 ˆ Thus, The co-ordinates of C may be


ÁË ˜¯ = ÁË ˜¯ (2 - 2 2, 3 + 2 2)
5 13
fi x – 8y = –5 …(ii) 98. (–1/10, 37/10)
On solving Eqs (i) and (ii), we get 99. Let the image of the point (–8, 12) be (x2, y2).
1 2 x2 + 8 y2 - 12 2(4 ◊ - 8 + 7 ◊ 12 + 13)
x= ,y= . Then = =-
3 3 4 7 (42 + 7 2 )
Ê 1 2ˆ
Hence, the in-centre is ÁË , ˜¯ .
3 3 2(-32 + 84 + 13)
=
96. The given lines are 3x - y + 3 = 0 and 65
x - 3y + 3 3 = 0 2 ¥ 65
= =2
Hence, the equation of bisectors are 65
Ê 3x - y + 3ˆ Ê x - 3 y + 3 3 ˆ x2 + 8 y - 12
fi = 2 and 2 =2
Á ˜ =Á ˜ 4 7
Ë 3 +1 ¯ Ë 1+ 3 ¯
fi x2 = 0 and y2 = 28
and ( 3 x - y + 3) = - ( x - 3 y + 3 3)
Hence, the image of the point (–8, 12) is (0, 28).
( 3 - 1) x + ( 3 - 1) y = 3( 3 - 1) 100. Hence, the image of the point (3, 4) with respect to the
fi and ( 3 + 1) x + ( 3 + 1) y = - 3( 3 + 1) line y = x is (4, 3).
fi x + y = 3 and x + y = –3 101. Let the point (4, 2) be P and the point (–2, 6) be Q.
Thus, the point P is (3, 0) and the point Q is (–3, 0) Now the mid-point of P and Q is M (1, 4).
Hence, the length of PQ = 6. Equation of a line passing through (4, 2) and (–2, 6) is
97. Give lines are AB: x + y = 1 …(i) 6-2 2
and CD: x + y = 5 …(ii) y-2= ( x - 4) = - ( x - 2)
-2 - 4 3
Slope of AB = – 1 = tan (135°)
fi 3y – 6 = –2x + 4
Since, AB makes an angle of 45° with AC, therefore it
will make an angle of (135° – 45°) or (135° + 45°), i.e. fi 2x + 13y = 10 …(i)
90° or 180° with the positive direction of x-axis. Equation of any line perpendicular to Eq. (i) is
Thus, AB is parallel to x-axis or y-axis. 3x – 2y + k = 0
Hence, x – 2 = 0 and y + 1 = 0. which is passing through M(1, 4).
Case I: Consider the lines are –x – y + 1 = 0 So k = 8 – 3 = 5.
and –x + 2 = 0 Hence, the equation of the line is
Now, a1a2 + b1b2 = 1 + 0 = 1 > 0 3x – 2y + 5 = 0.
Thus negative one is the acute-angle bisector. 102. Let the point M be (4, 1).
Hence, the equation of the acute-angle bisector of The image of the point M(1, 4) with respect to the line
–BAC is y = x be P(4, 1).
Ê - x - y + 1ˆ Ê - x + 2ˆ Now, if the point P be translated about the line x = 2,
=-
Á 2 2 ˜ ÁË ˜ then the new position of P is (4 + 2, 1).
Ë 1 +1 ¯ 12 ¯
Hence, the co-ordinates of Q is (6, 1).
fi ( 2 + 1) x + y = (2 2 + 1) …(iii) 103. Let the point P be (3, 2).
Case II: Consider the lines are –x – y + 1 = 0 and y + 1 = 0 The image of the point P with respect to the line x = 4
Now, a1a2 + b1b2 = 0 – 1 = –1 < 0 is Q(2.4 – 3, 2) i.e. Q(5, 2).
Thus, positive one is the acute-angle bisector. Let Z = 5 + 2i and R be Z1
Hence, the equation of the acute-angle bisector of Now, by rotation theorem of complex number,
–BAC is Z1 - 0 | Z - 0| i p4
fi = e
Ê - x - y + 1ˆ Ê y + 1ˆ Z - 0 | Z1 - 0|
=
Á 2 2 ˜ Á 2 ˜
Ë 1 +1 ¯ Ë 1 ¯ Z1 | Z1 - 0| i p4 ip
fi = e =e 4
fi x + ( 2 + 1) y = (1 - 2) …(iv) Z | Z - 0|

CG_01a.indd 45 2/6/2017 3:58:38 PM


1.46 Coordinate Geometry Booster

ip Ê 1 i ˆ Thus, the co-ordinates of Q is (6, 0).


fi Z1 = Z ¥ e 4 = (5 + 2i ) Á + ˜ Since, the co-ordinates of M is (1, 0) and the co-ordi-
Ë 2 2¯
nates of Q is (6, 0), so the equation of the reflected ray
1 1
fi Z1 = (5 + 5i + 2i - 2) = (3 + 7i ) is y = 0.
2 2 108. Let PM: x – 6y = 8 be the incident ray
Ê 3 7 ˆ AB: x + y = 1 is the mirror, MQ is the reflected ray
Hence, the co-ordinates of R are Á , .
Ë 2 2 ˜¯ and MS is the refracted ray.
Clearly, M is (2, –1)
104.
105. Let PM: x – 2y + 3 = 0 be the incident ray and MR be 1
m-
the reflected ray. Produced MQ in such a way that PM Now, tan 15° = 6
= MQ. Now we shall find the image of Q w.r.t. AB is R. m
1+
Co-ordinates of M are (1, – 1). 6
Let co-ordinates of Q be (3, 0) and R be (h, k).
Ê 6m - 1ˆ
Thus, R is the image of Q with respect to 3x – 2y = 5. fi (2 - 3) = ± Á
Ë m + 6 ˜¯
h-3 k -0 2(3.3 + 2.0 - 5) 8
Now = =- =- Ê 6m - 1ˆ
3 -2 2 2
(3 + 2 ) 13 fi ÁË m + 6 ˜¯ = 2 - 3 or 3-2
15 16
fi h= and k = 70 - 37 3 37 3 - 70
13 13 fi m= or
13 13
Ê 15 16 ˆ
Thus, the co-ordinates of R be Á , ˜ . Let the angle between x + y = 1 and the line through
Ë 13 13 ¯
29 M(2, –1) with the slope
Slope of MR =
2 70 - 37 3
Therefore, the equation of MR is m= be a.
13
29
y + 1 = ( x - 1)
2 70 - 37 3
+1
13 83 - 37 3
fi 29x – 2y = 31 Then tan(a ) = =
Hence, the equation of the reflected ray is 70 - 37 3 37 3 - 57
1-
29x – 2y = 31 13
106. Let N be the image of M with respect to x-axis. Thus N 83 - 37 3
is (5, –3) =
37 3 - 57
-3 - 2 5
Slope of NQ = =- and the angle between x + y = 1 and the line through
5 -1 4
Therefore, the equation QN is 37 3 - 70
M(2, –1) with slope m = be b.
5 13
y - 2 = - ( x - 1)
4 37 3 - 70
+1
fi 5x + 4y = 13 13 37 3 - 9
Then tan(b ) = =
Ê 13 ˆ 37 3 - 70 131 - 37 3
Thus, the co-ordinates of P be ÁË , 0˜¯ . 1-
5 13
13 37 3 - 9
Hence, the abscissa of the point P is . =
5 131 - 37 3
107. Let PM be the incident ray and QM be the reflected ray.
Let P(1, 5) be any point on the line x = 1 Here, tan (a) > tan (b)
Produce PM in such a way that PM = RM, fi a>b
where M = (1, 0) 70 - 37 3
Therefore, the slope of the refracted ray is .
Clearly, R = (1, –5), since, the incident ray is parallel to 13
y-axis. Hence, the equation of the refracted ray is
Now, image of R with respect to the line x + y = 1 is Q. Ê 70 - 37 3 ˆ
Let the co-ordinate of Q be (h, k). ( y + 1) = Á ˜¯ ( x - 2)
Ë 13
h - 1 k + 5 -2(1 - 5 - 1)
Now, = = =5 fi 13y + 13 = (70 - 37 3) x - 140 + 74 3
1 1 (1 + 1)
fi h = 6, k = 0 fi (70 - 37 3) x - 13y - 153 + 74 3 = 0

CG_01a.indd 46 2/6/2017 3:58:38 PM


Straight Lines 1.47

LEVEL III 3.
Y
Q
1. On solving, we get, A (3, 3 3)
the co-ordinates of A, 6

0
B and C.

5=

x+
M
Thus, A = (2, 3),

x–

4y
1

–2

=
B = (–1, 1) P(0, b)

14
X¢ X
3y
P Q(0, 0)
and C = (–2, 4). (–1, 0)
Now, the points P and B C
A lie on the same side 3x + y + 2 = 0

of the line
3x + y + 2 = 0. Since OM is the internal bisector of the angle PQR.
QM OQ 6
3.0 + b + 2 = =
So, > 0 fi b > -2 …(i) PM OP 1
6+3+ 2 Ê 3 3 3ˆ
Thus, the co-ordinates of M = ÁË - , ˜
Also, the points P and B on the same side of the line 7 7 ¯
x + 4y = 14 Therefore, the equation of OM is
0 + 4b - 14 y=- 3x
So, >0
-1 + 4 - 14 fi y+ 3x=0
fi 4b – 14 < 0 4. Equation of any line passing through (2, 2) is
7 y – 2 = m(x – 2)
fi b< …(ii) Let the points A and B are
2
Again, the points P and C lie on the same side of the Ê 2m - 2 Ê 2m - 2 ˆ ˆ
A=Á , - 3Á
line Ë m + 3 Ë m + 3 ˜¯ ˜¯
3y – 2x = 5
Ê 2m - 2 Ê 2m - 2 ˆ ˆ
3b - 5 and B=Á , 3Á
So, >0 Ëm- 3 Ë m - 3 ˜¯ ˜¯
12 + 4 - 5
Since the DOAB is equilateral, so
fi 5 OA = OB = AB
b> …(iii)
3 2 2
Ê 2m - 2 ˆ Ê 2m - 2 ˆ
From Eqs (i), (ii) and (iii), we get, fi ÁË (m + 3) ˜¯ + 3 ÁË (m + 3) ˜¯
5 7 2 2
<b< Ê 2m - 2 ˆ Ê 2m - 2 ˆ
3 2 =Á + 3Á
Ë (m - 3) ¯˜ Ë (m - 3) ˜¯
2. We have,
4 4
3x + 4y = 9 fi 2
=
(m + 3) (m - 3) 2
9 - 3x
fi 8= fi (m + 3) 2 = (m - 3) 2
4
and y = mx + 1 fi m=0
Hence the required equation of the line is y = 2.
9 - 3x 5.
fi = mx + 1 Y
4
fi 9 – 3x = 4mx + 4
fi (4m + 3)x = 5 x+y=2 y=x
5
fi x= A(1, 1)
(4m + 3) y=k P(h, k)
When m = –1, then x is –5 B C
X¢ X
When m = –2, then x is –1 O
Hence, the number of integral values of m is 2 for
which x is also an integer. Y¢

CG_01a.indd 47 2/6/2017 3:58:40 PM


1.48 Coordinate Geometry Booster

The co-ordinates of A, B and C are (1, 1), (k, k) and z3 - z1 | z3 - z1 | ip


(2 – k, k). fi = ¥e 4
z2 - z1 | z2 - z1 |
1 1 1 i p4
1 fi ( z3 - z1 ) = ( z2 - z1 ) e
Given k k 1 = 4h 2
2 1
2-k k 1 fi z3 = z1 + (5 + 2i - 3) ¥ (i + i)
2
1 0 0 1
1 Ê C2 Æ C2 - C1 ˆ fi z3 = z1 + (2 + 2i ) ¥ (i + i )
fi k 0 1 - k = 4h 2 Á 2
2 Ë C3 Æ C3 - C1 ¯˜ 2
2-k 2k - 2 k - 1 fi z3 = 3 + (1 + i ) 2 = 3 + 2(1 + i ) 2
2
1 0 1- k
fi = 4h 2 fi z3 = 3 + 2 2 i
2 2k - 2 k - 1
fi C = (3, 2 2)
0 1- k
fi = 4h 2 It is given that D is the image of C w.r.t. y-axis
k -1 k -1
Thus, D = (-3, 2 2)
fi (k – 1)2 = 4h2 So, x = - 3, y = 2 2
fi (k – 1) = ±2h Hence, the value of
Hence, the locus of P is 2x – y + 1 = 0. x+y+7
6. Given lines are x + y = | a | and ax – y = 1 = -3 + 2 2 + 7
Since two rays intersect in the first quadrant, so the
lines should be x + y = a and ax – y = 1. = 2(2 + 2)
On solving, we get, the point of intersection is (1, a – 1) 9. Let the line L: y = mx + c be equally inclined to L1:
Clearly, a0 = 1. 3x – 4y = 7 and L2: 5y – 12x = 6.
x y Since the line L = 0 is equally inclined with the lines
7. Let the variable line be + = 1 and the co-ordinates
a b L1 = 0 and L2 = 0, so we have
of O, A, B are (0, 0), (a, 0), (0, b). 3 12
-m m-
Let the centroid be G(a, b). 4 5
=
a b 3 12
Thus, a = , b = 1+ m 1+ m
3 3 4 5
fi a = 3a, b = 3b 3 - 4m 5m - 12
fi =
It is given that 4 + 3m 5 + 12m
OM = p fi 15 + 16m – 48m2 = 15m2 – 16m – 48
0 + 0 -1 fi 63m2 – 32m – 63 = 0
fi =p fi 63m2 – 81m + 49m – 63 = 0
1 1
+ fi 9m(7m – 9) + 7(7m – 9) = 0
a 2 b2
fi (7m – 9)(9m + 7) = 0
1 1 1
fi 2
+ 2 = 2 9 7
a b p fi m= ,-
7 9
1 1 1
fi 2
+ 2
= 2 Hence, the equation of the line is
(3a ) (3b ) p 9 7
y - 5 = ( x - 4) or y - 5 = - ( x - 4)
1 1 9 7 9
fi 2
+ 2 = 2
a b p fi 9x – 7y = 1 or 7x + 9y = 73
Hence, the locus of G(a, b) is 10. Let z1 = A(3, 0); z2 = B; z3 = C(2, 5) and z4 = D
1 1 9 D C(2, 5)
2
+ 2 = 2
x y p
8. Let z1 = A(3, 0), z2 = B(5, 2) and z3 = C
We have M(1, 4)
z3 - z1 z3 - z1 ip
= ¥e 4
z2 - z1 z2 - z1
A(0, 3) B

CG_01a.indd 48 2/6/2017 3:58:40 PM


Straight Lines 1.49

z3 - z2 z3 - z2 -i p -m + 4
Now, = ¥e 2 fi >0
z1 - z2 z1 - z2 4
z3 - z2 | z3 - z2 | -i p
fi m<4
fi = ¥ e 2 = -i Case III: When the points P and C lie on the same
z1 - z2 | z1 - z2 |
side of AB.
fi z3 – z2 = –iz1 + iz2 3m + 2(m + 1) - 6
fi (1 + i)z2 = z3 + iz1 So, >0
18 + 2 - 6
z3 + iz1 2 + 5i + 3i 5m - 4
fi z2 = = fi >0
1+ i 1+ i 14
2 + 5i + 3i 2(1 + 4i ) 4
fi z2 = = fi m>
1+ i 1+ i 5
2(1 + 4i ) (1 - i ) Hence, the value of m lies in
fi z2 = ¥
1+ i (1 - i ) 4
<m<4
2(1 + 4i )(1 - i ) 5
fi z2 = x y
2 12. Let the line be + = 1
fi z2 = 1 + 3i + 4 = 5 + 3i a b
Thus, B = (5, 3) It intersects the x-axis at A(a, 0) and y-axis at
B(0, b).Clearly, the point M(h, k) is the mid-point of A
Let D = (a, b)
and B.
a +5 b +3 a+0 0+b
Thus, = 1, =4 fi h= ,k =
2 2 2 2
fi a = –3, b = 5 fi a = 2h, b = 2k
Therefore, D = (–3, 5) Hence, the equation of the line is
11. First we find the equations of the sides of the triangle
x y
ABC, i.e. AB, BC and CA. + =1
3-0 2h 2k
AB: y - 3 = ( x - 0) 13. Since the points (m, 3) and (0, 0) lie on the opposite
-2 - 0
sides 3x + 2y – 6 = 0 and x – 4y + 16 = 0,
fi –2y + 6 = 3x
fi 3x + 2y = 6 3m + 6 - 6
so, <0
3 -1 0+0-6
AC: y - 3 = ( x - 0) fi 3m > 0
6-0
fi 2y – 6 = x fi m>0
fi x – 2y + 6 = 0 m - 12 + 16
1- 0 Also, <0
BC: y - 0 = ( x + 2) 0 - 0 + 16
6+2
fi m+4<0
fi 8y = x + 2
fi x – 8y + 2 = 0 fi m < –4
Hence, the values of m lies in
Case I: The points P and A lie on the same side of the
line BC R – (– 4, 0)
m - 8(m + 1) + 2 14. Let z1 = A(0, 3); z2 = B(–2, 5); z3 = C and z4 = D
So, >0
0 - 24 + 2 D C
-7m - 6
fi >0
-22 M
fi 7m + 6 > 0
6
fi m>-
7
Case II: When the points P and B lie on the same side A(0, 3) B = (–2, 5)
of AC
m - 2(m + 1) + 6 z3 - z2 z3 - z2 -i p
>0 Now, = ¥e 2
So,
-2 - 0 + 6 z1 - z2 z1 - z 2

CG_01a.indd 49 2/6/2017 3:58:41 PM


1.50 Coordinate Geometry Booster

z3 - z2 | z3 - z2 | -i p
Again, the points P and B lie on the same side of the
fi = ¥ e 2 = -i line x + 2y – 3 = 0.
z1 - z2 | z1 - z2 |
a + 2a 2 - 3
fi z3 – z2 = –iz1 + iz2 Thus, >0
1 2
fi z3 = z2 + –iz2 – iz1 + -3
3 9
fi z3 = –2 + 5i – 2i – 5 + 3 = –4 + 3i
fi 2a2 + a – 3 < 0
Thus, C = (4, 3)
and M = (–2, 3) fi (2a + 3)(a – 1) < 0
Let D = (a, b) 3
fi - < a <1 …(ii)
a-2 b +5 2
Thus, = - 2, =3 Finally, the points P and C lie on the same side of the
2 2
fi a = –2, b = 1 line 2x – 3y – 1 = 0
Therefore, D = (–2, 1) 2a + 3a 2 - 1
Thus, >0
15 Hence, the co-ordinates of the points are Ê 5ˆ Ê 7ˆ
2 Á ˜ + 3Á ˜ - 1
(x1 ± r cos b, y1 ± r sin q) Ë 4¯ Ë 8¯
= (2 ± 8 cos(135°),1 ± 8 sin(135°)) fi 3a2 + 2a – 1 > 0
Ê Ê 1 ˆ Ê 1 ˆˆ fi (3a – 1)(a + 1) > 0
= Á2 ± 8Á- ˜ , 1 ± 8 ÁË ˜
Ë Ë 2¯ 2 ¯ ˜¯ fi a < -1 or a >
1
…(iii)
= (2 2, 1 ± 2) 3
= (0, 3) and (4, –1) From Eqs (i), (ii) and (iii), we get
16. Given family of lines be Ê 3 ˆ Ê1 ˆ
a ŒÁ - , -1˜ » Á , 1˜
(a + b)x + (2a – b)y = 0 Ë 2 ¯ Ë2 ¯
fi a(x + 2y) + b(x – y) = 0 18.
b D
fi ( x + 2y ) + ( x - y ) = 0 C(5, 1)
a
fi (x + 2y) + l(x – y) = 0 M(3, 2)
fi (x + 2y) = 0, (x – y) = 0
On solving, we get the co-ordinates of the fixed point is
(0, 0).
A(1, 3) B(p, q)
Hence, the family of lines passes through a fixed point
is (0, 0). Clearly, the mid-point (3, 2) of (1, 3) and (5, 1) lies on
17. the diagonal.
A(–7, 5) So, 2 = 6 + c
fi c = –4
Equation of one diagonal is y = 2x – 4.
x+
1
y=

2y

Let B = (p, q)
+3

Now, m(BC) ¥ m(AB) = –1


3=
2x

P(a, a2)
Ê q - 1 ˆ Ê q - 3ˆ
0

fi ÁË p - 5 ˜¯ ¥ ÁË p - 1˜¯ = -1
B(1/3, 1/9) 5x – 6y – 1 = 0 C(5/4, 7/8) fi (q – 1)(q – 3) = –(p – 1)(p – 5)
fi q2 – 4q + 3 = –p2 + 6p – 5
On solving the equations, we get the co-ordinates of A,
B and C, respectively. fi p2 + q2 – 6p – 4q + 8 = 0 …(i)
Now, A and P lie on the same side of the line Also B lies on the line y = 2x – 4.
5x – 6y – 1 = 0 So, q = 2p – 4 …(ii)
5a - 6a 2 - 1 On solving Eqs (i) and (ii), we get,
Thus, >0 p = 2, 4 and q = 0, 4
5(-7) - 6(5) - 1
Hence, the co-ordinates of the other vertices are (2, 0)
fi 6a2 – 5a + 1 > 0
and (4, 4).
fi (3a – 1)(2a – 1) > 0
19. Clearly, the slope of the given line is –1.
fi 1 1
a < or a > …(i) Thus, q = 135°.
3 2

CG_01a.indd 50 2/6/2017 3:58:42 PM


Straight Lines 1.51

Therefore, the equation of the other two sides are Given equation is line AB is
y – 3 = tan (135° ± 60°)(x – 2) 2x + y = 7 …(i)
fi y – 3 = tan (195°)(x – 2) Let S be the image of the point P(–3, 4).
or y – 3 = tan (75°)(x – 2) Equation of PM is x – 2y + k = 0 which is passing
fi y – 3 = tan (15°)(x – 2) through P(–3, 4)
or y – 3 = cot (15°)(x – 2) So, –3 – 8 + k = 0
fi k = 11
fi y - 3 = (2 - 3)( x - 2)
Thus, PM is x – 2y + 11 = 0 …(ii)
or y - 3 = (2 + 3)( x - 2) Solving Eqs (i) and (ii), we get,
20. Y 3 29
x= ,y=
C
5 5
+k=0
7x + 2y Ê 3 29 ˆ
Thus, M = Á , ˜
B Ë5 5 ¯
11x 4x + 5y + k = 0
4x + 5y = 0 + 7y Let S = (a, b)
=9
3 a-3 29 b + 4
A Then = and =
5 2 5 2
7x + 2y = 0 X
O 21 38
fi a= and b =
5 5
Ê 21 38 ˆ
On solving OA and AB, we get Thus, S = Á , ˜
Ë 5 5¯
Ê 2 7ˆ
A = Á- , ˜ Therefore, the equation of SQ is
Ë 3 3¯
38
-1
y -1= 5
On solving OB and AB, we get
( x - 0)
21
Ê 5 7ˆ -0
B=Á ,- ˜ 5
Ë 3 3¯
33
Therefore, the equations of BC and AC are fi y -1= x
21
7x + 2y = 9, 4x + 5y = 9
fi 33x – 21y – 21 = 0
On solving BC and AC, we get,
fi 11x – 7y – 7 = 0 …(iii)
C = (1, 1).
On solving Eqs (ii) and (iii), we get,
Hence, the equation of the other diagonal is y = x.
42 91
21. The reflection of the point (1, 2) w.r.t. x-axis is (–1, 2) x= ,y=
25 25
Thus, the equation of line containing the reflected ray
Ê 42 91 ˆ
is Thus, R = Á , ˜
3- 2 Ë 25 25 ¯
y-2= ( x + 1)
5 +1 23. The incident ray intersects the mirror at A(6, 0).
1 Let B(0, –4) be a point on the incident ray.
fi y - 2 = ( x + 1) The reflection of the point B w.r.t. x-axis is C(0, 4).
6
Hence, the equation of the reflected ray is
fi 6y – 12 = x + 1
x y
fi x – 6y + 13 = 0 + =1
6 4
Hence, the point A is (–13, 0).
fi 2x + 3y = 12
22.
S 24. The lines |x + y| = 4 are
x + y = 4, x + y = –4
Clearly, both are parallel lines.
R 2x + y = 7
A B Now a line y = x intersects both the lines at (–2, –2) and
M (2, 2)
Thus, if the point (a, a) lies between the lines, so,
P(–3, 4) a > –2 and a < 2.
Q(0, 1)
Hence, –2 < a < 2.

CG_01a.indd 51 2/6/2017 3:58:42 PM


1.52 Coordinate Geometry Booster

25. fi 8 – 4b = 3 – 3a
A
fi 3a – 4b = –5 …(i)
F E Also, m(AC) ¥ m(BE) = –1
b+3 2-5

4x
6=
H(1, 1) fi ¥ = -1

+
+ a — 4 1+ 2

5y
2y

=
b+3

20

fi =1
3x

B D C a—4
fi a–4=b+3
Here, altitude AD passes through the intersection of AB
fi a–b=7 …(ii)
and AC.
Solving Eqs (i) and (ii), we get,
Let AD: (3x – 2y + 6) + l(4x + 5y – 20) = 0 which is
passing through H(1, 1) a = 33, b = 26
fi (3 – 2 + 6) + l(4 + 5 – 20) = 0 Hence, the third vertex is C(33, 26).
fi 7 – 11l = 0 27.
A
7 x+
fi l= 2y
11 =0 E 5=0
7 +
Thus, (3x - 2y + 6) + (4x + 5y - 20) = 0 F
–y
11 M
x
fi 61x + 13y – 74 = 0 C
B
Consider BC: 13x – 61y + k = 0
Let the co-ordinates of B and C are (x1, y1) and (x2, y2)
Now, altitude BE:
respectively.
3x - 2y + 6 13x - 61y + k Clearly the mid-point of A and B and A and C lie on the
=
3.4 - 2.5 13.4 - 61.5 perpendicular bisectors x + 2y = 0 and x – y + 5 = 0.
which is passing through H(1, 1) x1 + 1 Ê y - 2ˆ
So, + 2Á 1 =0
7 k - 48 2 Ë 2 ˜¯
So, =
2 52 - 305 fi x1 + 2y1 – 3 = 0 …(i)
7 k - 48 Ê x2 + 1ˆ Ê y2 - 2 ˆ
fi =
2 — 253 and ÁË 2 ˜¯ - ÁË 2 ˜¯ + 5 = 0
7 1675 fi x2 + y2 + 13 = 0 …(ii)
fi k = 48 - ¥ 253 = -
2 2 Ê y + 2ˆ Ê 1ˆ
Also, Á 1 Á - ˜ = -1
Hence, the equation of the BC is Ë x1 - 1 ˜¯ Ë 2 ¯
1675
13x - 61y - =0 fi 2x1 – y1 – 4 = 0
2
fi 26x – 122y – 1675 = 0 Ê y2 - 2 ˆ
and ÁË x - 1 ˜¯ (1) = - 1
26. 2
A(4, –3) fi x2 + y2 + 1 = 0 …(iv)
Solving Eqs (i) and (iii), we get the co-ordinates of B,
F E
11 2
H(1, 2) i.e. B = ÊÁ , ˆ˜
Ë 5 5¯
Solving Eqs (ii) and (iv), we get the co-ordinates of C,
i.e. C = (–7, 6)
2
D C 6-
B(–2, 5) 5 28 14
Now, m ( BC ) = =- =-
Let C = (a, b) 11 46 23
-7 -
Now, m(AB) ¥ m(CF) = –1 5
Therefore, the equation of BC is
5+3 2—b
fi ¥ = -1 14
—2 — 4 1 — a y - 6 = - ( x + 7)
23
2—b 3
fi = fi 23y – 138 = –14x – 98
1— a 4 fi 14x + 23y – 40 = 0

CG_01a.indd 52 2/6/2017 3:58:43 PM


Straight Lines 1.53

28. 30. Given points are A(1, p2), B(0, 1), C(p, 0)
A(2, –7)
1 p2
1 0 1
Area of the triangle =

1
+
y= 2 p 0
1 p2
4x

1
q q D= (1 + p 3 - p )
B C 2
3x – 4y = 1
dD 1
Let m be the slope of AC. = (3p 2 - 1)
dp 2
It is given that AB = AC.
For maximum or minimum,
Clearly BC is equally inclined with AB and AC
dD
3 3 =0
+4 m- dp
4 = 4
1
So, 3 3
1 + ¥ (- 4) 1 + ◊ m fi
2
( )
3p 2 - 1 = 0
4 4
19 4m - 3 1
fi = fi p=±
- 8 3m + 4 3
+ – +
52
fi m=-
89 – 1 1
3 3
Hence, the equation of AC is 1
52 Clearly, its area is minimum at p = .
y + 7 = - ( x - 2) 3
89 31. Clearly, the point A is (3, –1)
fi 89y + 623 = –52x + 104 Equation of BC is y – 2 = m(x – 1)
fi 52x + 89y + 519 = 0
3
29. m+
A(–2, –7)
Thus, 4 = tan 45°
3
1- m
P 4x + 3y = 12 4
3 4m + 3
fi =1
Q 4 - 3m
4x + 3y = 3
Ê 4m + 3 ˆ
fi ÁË 4 - 3m ˜¯ = ± 1
Equation of any line passing through A(–2, –7) is
y + 7 = m(x + 2). 1
Clearly, it is equally inclined to parallel lines with slope fi m = - 7,
7
–4/3.
Hence, the equation of the line BC is
Ê 4ˆ
m - Á- ˜
Ë 3¯ 1
So, tan (± q ) = y - 2 = - 7( x - 1) or y - 2 = ( x - 1)
Ê 4ˆ 7
1+ m ◊Á- ˜
Ë 3¯ x – 7y + 13 = 0 or 7x + y – 9 = 0
3 3m + 4
fi ± = . 32. Given A = (2, 1) B A(2, 1)
4 3 - 4m
So, zA = 2 + i
7
fi m=- and m = Then zB = i(2 + i) 90°
24 O
= –1 + 2i
Hence, the equation of the required lines are
B = (–1, 2)
y+7 7
,y 7 ( x 2) Therefore, C = (–2, –1) and C D
x+2 24 D = (1, –2), since O is the
x+2 1 origin.
fi = , 24y + 168 = - 7x - 14
y 7 33. Two given lines are 7x – y + 3 = 0 and x + y – 3 = 0.
fi x + 2 = 0 and 7x + 24y + 182 = 0 So the point of intersection is (0, 3)

CG_01a.indd 53 2/6/2017 3:58:44 PM


1.54 Coordinate Geometry Booster

Let m(B) = m A(0, 3) x- y+2 7x - y - 3


m(AB) = 7, m(AC) = –1 =±
1+1 49 + 1
Clearly, –ABC = –

x+
=0
x- y+2 7x - y - 3

y–
+3
ACB
2 5 2

3=
–y
m-7 m +1

0
Thus, =

7x
1 + 7m 1 - m 5(x – y + 2) = ±(7x – y – 3)
2x + 4y = 7, 12x – 6y + 13 = 0
fi 6m2 + 16m – 6 = 0 B C
Thus, slope of MA is either 2 or –1/2
fi 3m + 8m – 2 = 0
2
a-2 1
1 fi = 2 or -
fi m = - 3, 0 -1 2
3
5
Hence, the equation of the BC is So, a = 0 or
1 2
y + 10 = - 3( x - 1) or y + 10 = ( x - 1) Hence, the co-ordinates of A may be
3
Ê 5ˆ
3x + y – 7 = 0 or x – 3y – 31 = 0 (0, 0) or Á 0, ˜
Ë 2¯
34. Given lines are x + y – 1 A x+y=1 B(2, –1)
= 0 and x + y – 5 = 0.
Let the slope of BC be 45° LEVEL IV
m.
So, 1. Sides are x – 3y = 3, 3x – 2y = 16
m +1 Diagonals are x + 4y = 10, 5x – 8y + 6 = 0
tan (± 45°) = 2. Clearly B = (–1, 4), since the equation of the bisector in
1- m
the first quadrant is y = x
m +1
= ±1 Thus, AB = 25 + 25 = 50 = 5 2
1- m D x=y=5 C
m = 0, 3. We have,
p
z3 - z1 z -z i
Thus, the equation of BC may be y +1 = 0 or x – 2 = 0. = 3 1 ¥e 4
Now, the equation of the bisector of –ABC are z2 - z1 z2 - z1
p
y +1 x + y -1 z3 - z1 i 1
=± fi =e 4 = (1 + i )
1 2 z2 - z1 2
x—2 x + y -1 1
or =± fi z3 = z1 + ( z2 - z1 ) (1 + i )
1 2 2
1
The equation of the bisectors of the acute angle in two fi z3 = 2 + (2 + 2 + i - 2) (1 + i )
cases are 2
1
x + ( 2 + 1) y = (1 - 2) fi z3 = 2 + ( 2 + i ) (1 + i )
2
and ( 2 + 1) x + y = (1 + 2 2) 1
fi z3 = 2 + ( 2 + i + i 2 - 1)
On solving, we get the co-ordinates of D be 2
(6 + 2 2, - 1 - 2 2) or (2 - 2 2, 3 + 2 2) . 1
fi z3 = 2 + [( 2 - 1) + i ( 2 + 1)]
35. Let the co-ordinates of 2
A(0, a)
A be (0, a). ( 2 - 1) ( 2 + 1)
As the sides of the fi z3 = 2 + +i
2 2
rhombus are parallel to
the line y = x + 2 and y M(1, 2) 3 2 -1 ( 2 + 1)
D B fi z3 = +i
= 7x + 3, the diagonals 2 2
of the rhombus are par-
allel to the bisectors of Ê 3 2 - 1 ( 2 + 1) ˆ
fi C=Á , ˜
the angles between the Ë 2 2 ¯
given lines. C
4. Let A = (1, –2).
Equation of the bisectors of the angles between the Thus, the image of A w.r.t x-axis is, say, B(1, 2).
lines Thus, the new position of B is C(4, 2).

CG_01a.indd 54 2/6/2017 3:58:46 PM


Straight Lines 1.55

5. Clearly, the slope of the new line, m = tan (45°) = 1. We have, AB = 2l


Hence, the equation of the new line is fi AB2 = 4l2
y – 0 = 1(x – 2) fi (a – b)2 + 36b2 = 4l2
fi y=x–2 Ê k kˆ
2
Ê kˆ
2
2
fi x–y=2 fi ÁË 2h — — ˜¯ + 36 ÁË ˜¯ = 4l
3 3 3
6. Clearly, A = (0, 1) and 2
1 Ê 2k ˆ 2 2
m = tan (45° + 105°) = tan (150°) = - fi ÁË 2h — ˜¯ + 4k = 4l
3 3
2
Hence, the equation of the new line is Ê kˆ 2 2
fi ÁË h — ˜¯ + k = l
1 3
fi y -1= - ( x - 0)
3 Hence, the locus of M(h, k) is
fi 2
3y - 3 = - x Ê yˆ 2 2
ÁË x — ˜¯ + k = l
fi x + 3y = 3 3
7. Clearly, tan q = 2 fi 9x2 – 6xy + 10y2 = 9l2
2 1 10.
fi sin q = , cos q = D C(1, –1)
5 5
Hence, the required point
= (x1 + r cos q y1 + r sin q) M(2, 3/2)
Ê 1 2 ˆ
= Á1 + 1 ◊ ,1 + 1◊ ˜
Ë 5 5¯
Ê 1 2 ˆ A(3, 4) B
= Á1 + ,1+ ˜
Ë 5 5¯ Let z1 = A(3, 4), z2 = B and z3 = C(1, –1)
8. Thus, the point of intersection of the two given lines is p
z3 - z2 z -z -i
(2, –1) Now, = 3 2 ¥e 2
z1 - z2 z1 - z2
Let Q the point where P moving 2 units along the line p
x + y =1 z3 - z2 | z3 - z2 | -i
fi = ¥e 2
Ê 1 1 ˆ z1 - z2 | z1 - z2 |
So, Q = Á 2 - 2 ◊ , -1 + 2 ◊ ˜ p
Ë 2 2¯ z3 - z2 -i
fi = e 2 = -i
= (2 - 2, 2 - 1) z1 - z2
and R be the point where the point P moves 5 units fi z3 – z2 = –iz1 + iz2
along the line x – 2y = 4. fi (1 + i)z2 = z3 + iz1
Ê 2 1 ˆ z + iz1 1 - i + i (3 + 4i )
So, R = Á2 + 5◊ , -1 + 5 ◊ ˜ fi z2 = 3 =
Ë 5 5¯ (1 + i ) (1 + i )
= (2 + 2 5, 5 - 1) - 3 + 2i
fi z2 =
(1 + i )
Hence, the required distance
= QR (- 3 + 2i )(1 - i )
fi z2 =
2 2
2
= (- 2 - 2 5) + ( 2 - 5)
- 3 + 3i + 2i + 2 - 1 + 5i
fi z2 = =
= ( 2 + 2 5) 2 + ( 2 - 5) 2 2 2
= 2 + 20 + 2 + 5 Ê 1 5ˆ
Thus, the co-ordinates of B are Á - , ˜
Ë 2 2¯
= 29
9. Let A = (a, 0), B = (b, 6b) Let the co-ordinates of D be (a, b).
and M(h, k) is the mid-point of AB, where Ê 3ˆ
The mid-point of AC = M = Á 2, ˜
a+b Ë 2¯
h= , k = 3b 1
2 a-
k k Now, 2 = 2fia = 9
Thus, a = 2h - , b = 2 2
3 3

CG_01a.indd 55 2/6/2017 3:58:46 PM


1.56 Coordinate Geometry Booster

5 On solving, 20x1 – 3x2 = 52 and x1 + x2 = 2,


b+
and 2 = 3fib = 1 we get,
2 2 2 58 12
x1 = and x2 = -
Ê9 1ˆ 23 23
Hence, the co-ordinates of D are Á , ˜.
Ë2 2¯ 158 32
and so y1 = , y2 =
11. 23 23
A(4, –3)
Hence, the points A and B are
Ê 58 158 ˆ Ê 12 32 ˆ
ÁË , ˜ and ÁË - , ˜¯
F E 23 23 ¯ 23 23
H(1, 2)
83
Now, slope of AB is =
35
B(–2, 5) D C Hence, the equation of the line is
First we find the equation of BC and AC. 83x – 35y + 92 = 0
2+3 5 13.
m ( AD) = =-
1- 4 3 D C(–2, –1)
3
Now, m ( BC ) =
5 M(–1/2, 0)
Thus, the equation of BC is
3
y - 5 = ( x + 2)
5 A(1, 1) B
5y – 25 = 3x + 6 Let z1 = A(1, 1), z2 = B, z3 = C(–2, –1)
3x – 5y + 31 = 0 …(i) p
2-5 3 z3 - z2 z -z -i
Also, m ( BE ) = = - = -1 Now, = 3 2 ¥e 2
1+ 2 3 z1 - z2 z1 - z2
p
m(AC) = 1 z3 - z2 | z3 - z2 | -i
fi = ¥ e 2 = -i
Thus, the equation of AC is z1 - z2 | z1 - z2 |
y + 3 = (x + 4)
fi z3 – z2 = iz1 + iz2
x–y+1=0 …(ii)
On solving Eqs (i) and (ii), we get the co-ordinates of C fi (1 + i)z2 = z3 + iz1
are (13, 14). fi (1 + i)z2 = –2 – i + i(1 + i)
Thus, the third vertex is (13, 14).
fi (1 + i)z2 = –3
12.
-3 - 3(1 - i )
3x
=0

fi z2 = =
+4

(1 + i ) 2
–4

y–
–y

- 3 + 3i Ê 3 3 ˆ
4=

fi z2 = = Á- , ˜
5x

M(1, 5) Ë 2 2¯
0

A B 2
Ê 3 3ˆ
Thus, the co-ordinates of B are Á - , ˜ .
Ë 2 2¯
Let the co-ordinates of D be (a, b).
Let the points A and B be (x1, y1) and (x2, y2), respec-
tively. 3
a—
Thus, 5x1 – y1 – 4 = 0 and 3x2 + 4y2 – 4 = 0 Now, 2 = -1 fia = 1
2 2 2
Also, x1 + x2 = 2 and y1 + y2 = 10
3
Therefore, b+
y1 + y2 = 10 and 2 =0fib =-3
4 - 3x2 2 2
fi 5x1 - 4 + = 10 Ê 1 3ˆ
4 Therefore, the co-ordinates of D be Á , - ˜ .
Ë 2 2¯
fi 20x1 – 3x2 = 52

CG_01a.indd 56 2/6/2017 3:58:47 PM


Straight Lines 1.57

Hence, the equation of the other diagonal is ab Ê ab ˆ


y- = mÁ x -
3 3Ê 1ˆ
y + = - Áx - ˜ a+b Ë a + b ˜¯
2 2Ë 2¯
Therefore, the co-ordinates of A and B are
3 3 3 ab Ê 1ˆ ab
fi y+ =- x+
2 2 4 Á1 - ˜¯ and (1 - m)
a +bË m a+b
3 3 3
fi x+ y+ - =0 Let the mid-point be M(h, k).
2 2 4 ab Ê 1ˆ ab
3 3 Thus, h = Á1 - ˜¯ and k = (1 - m)
fi x+ y+ =0 2(a + b) Ë m 2(a + b)
2 4
2h(a + b) Ê 1ˆ 2k ( a + b )
fi 6x + 4y + 3 = 0 fi = Á1 - ˜ and = (1 - m)
ab Ë m¯ ab
14.
Y Ê 2h(a + b) ˆ 1 Ê 2k (a + b) ˆ
fi ÁË - 1˜ = - and Á - 1˜ = - m
ab ¯ m Ë ab ¯
x + 2y = 1 Eliminating m, we get
B Ê 2h(a + b) ˆ Ê 2k (a + b) ˆ
2x – y = 1
fi ÁË - 1˜ Á - 1˜ = 1
M(h, k) ab ¯Ë ab ¯
P(3/5, 1/5) Hence, the locus of M(h, k) is
A X Ê ab ˆ Ê ab ˆ ab
O
2x - 2y - =
ËÁ (a + b) ¯˜ ËÁ (a + b) ¯˜ (a + b)
16. We have,
Equation of any line passing through the point of inter-
m = tan (15°) = (2 - 3)
section of the lines x + 2y = 1 and 2x – y = 1 is
Y
1 Ê 3ˆ
y- = mÁ x - ˜
5 Ë 5¯
15°
Clearly, the co-ordinates of A and B are
Ê 3m - 1 ˆ Ê 1 - 3m ˆ 30°
ÁË , 0˜ and Á 0, ˜
5m ¯ Ë 5 ¯ X
O A(2, 0)
Let M(h, k) be the mid-point of AB
3m - 1 1 - 3m Hence, the equation of the line is
Thus, h = and k =
10m 10 y - 0 = (2 - 3)( x - 2)
1 1 — 10k fi (2 — 3) x — y - 2(2 - 3) = 0
fi m= ,m=
3 - 10h 3 -3 + 8 - 7 -2 1
17. Clearly, = = >0
1 1 3 9 - 32 - 7 - 30 15
fi m= , =
3 - 10h m 1 — 10k Thus,the points (–1, –1) and (3, 7) lie on the same side
Eliminating m, we get of the line.
18. On solving, we get the A
1 3
¥ =1 co-ordinates of A, B
3 - 10h 1 - 10k
0

and C.
5=

x+

fi (3 – 10h)(1 – 10k) = 3 Thus, A = (2, 3), B =


x–

4y
–2

(–1, 1) and C = (–2, 4).


=

Hence, the locus of the mid-point M(h, k) is P(0, b)


14
3y

(3 – 10x)(1 – 10y) = 3 Now, the points P and


15. The point of intersection of the lines A lie on the same side
of the line 3x + y + 2 = B 3x + y + 2 = 0
C
x y x y Ê ab ab ˆ
+ = 1 and + = 1 is Á , 0.
a b b a Ë a + b a + b ˜¯ 3.0 + b + 2
So, > 0 fi b > —2 …(i)
Thus, the equation of the line passing through 6+3+2
Ê ab ab ˆ Also, the points P and B on the same side of the line x
ÁË a + b , a + b ˜¯ is + 4y = 14

CG_01a.indd 57 2/6/2017 3:58:48 PM


1.58 Coordinate Geometry Booster

0 + 4b - 14 k+2
So, >0 fi =4
—1 + 4 — 14 9 + 16
fi 4b – 14 < 0 k+2
fi =4
7 5
fi b< …(ii)
2 fi k + 2 = ±20
Again, the points P and C lie on the same side of the fi k = –2 ± 20
line 3y – 2x = 5 fi k = 18, –22
3b - 5 Hence, the line is 3x – 4y + 18 = 0 and 3x – 4y – 22 = 0.
So, >0
12 + 4 - 5 21. Given lines are x + y + 1 = 0 and x + y – 1 = 0
5 Clearly both are parallel.
fi b> …(iii)
3 Distance between them is 2 .
From Eqs (i), (ii) and (iii), we get Thus, the line must be perpendicular to the given lines.
Equation of any line perpendicular to x + y + 1 = 0 is
5 7
<b< x – y + k = 0 which is passing through (–5, 4).
3 2 Thus, –5 – 4 + k = 0
19. fi k=9
N Hence, the equation of the line is x – y + 9 = 0.
22. Given DRPQ = 7
x– R
2y x y 1
–3 1
=0 fi 3 1 1 =7
2
A M 3x – 2y – 5 = 0 B 6 5 1

Let incident ray = IM and reflected ray = MR and MN x y 1


be the normal. fi 3 1 1 = 14
On solving, we get 6 5 1
M = (1, –2)
Now, slope of IM = 1/2 and slope of AB = 3/2 and slope fi x(1 – 5) – y(3 – 6) + 1(15 – 6) = ±14
of MN = –2/3. fi –4x + 3y + 9 = ±14
As we know that the normal is equally inclined with the fi –4x + 3y – 5 = 0, –4x + 3y + 23 = 0
incident ray as well as reflected ray. Clearly, infinite number of points satisfy the point R.
1 2 2 23.
+ - -m Y
Thus, 2 3 = 3
1 2 2
1- ◊ 1- ◊ m
2 3 3 M P
3 + 4 - 2 - 3m 3m + 2
fi = = R(0, 2)
6-2 3 - 2m 2m - 3
X¢ X
7 2 + 3m O
fi =
4 2m — 3

fi 14m – 21 = 8 + 12m
29 Clearly, the co-ordinates of P are
fi m=
2 x-0 y-2
= =5
Hence, the equation of the reflected ray cos 60° sin 60°
29 5 4+5 3
y+2= ( x - 1) x= ,y=
2 2 2
fi 2y + 4 = 29x – 29 Thus, the co-ordinates of M are
fi 29x – 2y – 33 = 0 Ê 4 + 5 3ˆ
20. Equation of any line parallel to 3x – 4y – 2 = 0 is 3x – 4y Á 0, 2 ˜¯
Ë
+k=0
Given distance between parallel lines = 4 24.

CG_01a.indd 58 2/6/2017 3:58:48 PM


Straight Lines 1.59

25. Thus, all the vertices lie in the half plane determined by
D(–2,–2) x – 3y = 4 C(1, –1) 3x + 2y > 0.
5x + y + 12 = 0
For the second,
2x + y – 13 > 0
x=1 2(1) + 3 – 13 = –8 £ 0
2(5) + 0 – 13 = –3 £ 0
2(–1) + 2 – 13 = –13 £ 0
A(–3, 3) x + 2y = 3 B(1, 1) Thus, all the vertices do not lie in the half plane deter-
Clearly, the diagonal AC on the line y = –x and the di- mined by 2x + y – 13 > 0.
agonal BD on the line y = x. Thus, the angle between For the third,
y = x and y = – x is right angled. 2x – 3y – 12 < 0
26. 2(1) – 3(3) – 12 = –19 £ 0
C(5, 5) 2(5) + 3(0) – 12 = –2 £ 0
2(–1) – 3(2) – 12 = –20 £ 0
4 Thus, all the vertices lie in the half plane determined by
D(31/7, 1)
2x – 3y – 12 < 0
A(1, 1) M For the fourth,
3 –2x + y £ 0
–2(1) + 3 = 1 ≥ 0
B(4, –2) –2(5) + 0 = –10 £ 0
Here AD is the internal bisector of the angle –BAC. –2(–1) + 2 = 4 ≥ 0
AB BD Thus, all the vertices do not lie in the half plane deter-
Thus, = mined by –2x + y £ 0.
AC DC
Hence, all points inside the triangle satisfy the inequali-
BD AB 3 2 3
fi = = = ties 3x + 2y > 0 and 2x – 3y – 12 < 0.
DC AC 4 2 4 29. Given OE: 3x – 2y + 8 = 0 A(1, –1)
Ê 31 ˆ AC is perpendicular to
Therefore, the co-ordinates of D = Á , 1˜ .
Ë7 ¯ OE. E
Hence, the equation of AD is y – 1 = 0 AC : 2x + 3y + k = 0 O
Thus, the equation of the perpendicular from C on the which is passing through
internal bisector AD is x = 5. A(1, –1)
27. Let the foot of perpendicular be (h, k). B(3, 1) D C
Thus, 2 – 3 + k = 0
h-2 k-4 Ê 2 + 4 - 1ˆ fi k=1
Then = = -Á
1 1 Ë 1 + 1 ˜¯ Hence, the equation of AC is 2x + 3y + 1 = 0.
Let the co-ordinates of E be (a, b)
h-2 k-4 5
fi = =- Thus, 2a + 3b + 1 = 0
1 1 2
and 3a – 2b + 8 = 0
5 5
fi h=2- ,k =4- On solving, we get,
2 2
a = –2 and b = 1
fi 1 3
h=- ,k = Therefore, E = (–2, 1)
2 2
Ê 1 3ˆ Now, E is the mid-point of A and C.
Hence, the foot of perpendicular is Á - , ˜ .
Ë 2 2¯ Thus, C = (–5, 3)
28. All the points inside the Hence, the equation of BC is
A(1, 3)
triangle will lie in the half 1- 3
fi y -1= ( x - 3)
plane determined by ax + 3+5
by + c ≥ 0 (£) if all the 1
vertices lie in it. fi y - 1 = - ( x - 3)
4
For the first,
fi x + 4y – 7 = 0
3x + 2y > 0,
3(1) + 2(3) = 9 ≥ 0 B(5, 0) C(–1, 2) Now, D = (–1, 2)
3(5) + 2(0) = 15 ≥ 0 OD is perpendicular to BC
3(–1) + 2(2) = 1 ≥ 0 OD: 4x – y + k = 0

CG_01a.indd 59 2/6/2017 3:58:49 PM


1.60 Coordinate Geometry Booster

which is passing through D(–1, 2) 32. Given,


Thus, –4 – 2 + k = 0 ar(D) = 8
fi k=6 2a a 1
Hence, the equation of OD is 4x – y + 6 = 0 1
fi a 2a 1 = 8
Now, we solve the equations 4x – y + 6 = 0 2
a a 1
and 3x – 2y + 8 = 0, we get,
Ê 4 14 ˆ 2a a 1
circumcentre is Á - , ˜ .
Ë 5 5¯ fi a 2a 1 = 16
30. Y a a 1

B
fi 2a(2a – a) + (a2 – 2a2) = 16
C fi 2a2 – a2 = 16
0
y=

fi a2 = 16
–5

A fi a = ±14
12x

y =0
O 3x – 4 X 5
33. Clearly slope of CD = B(0, 7) D
5 7
Clearly OB is the angle bisector of –AOC.
tan q =
Thus, OB: 7y – 9 x = 0 7
M(5/2, 7/2)
It is given that OB = 12 sin q cos q 1
fi = =
Let the co-ordinates of B be (h, k). 5 7 74
9 74
Now slope of OB = tan q = Thus, MD =
C A(5, 0)
7 2
sin q cos q 1
= =
9 7 130 Ê5 74 7 74 ˆ
Now, D = Á + cos q , + sin q ˜
Ê 7 9 ˆ Ë2 2 2 2 ¯
Now, B = Á 0 + 12 ◊ , 0 + 12 ◊ ˜
Ë 130 130 ¯ Ê5 74 7 7 74 5 ˆ
=Á + ¥ , + ¥ ˜
Ê 84 108 ˆ Ë2 2 74 2 2 74 ¯
B=Á ,
Ë 130 130 ˜¯ Ê5 7 7 5ˆ
=Á + , + ˜ = (6, 6)
Here BC is parallel to OA Ë2 2 2 2¯
BC: 3x – 4y + k = 0
Ê5 74 7 74 ˆ
which is passing through B. Now, C = Á - cos q , - sin q ˜
Ë2 2 2 2 ¯
180
Thus, k = -
130 Ê5 74 7 7 74 5 ˆ
=Á - ¥ , - ¥ ˜
Hence, the equation of BC is Ë2 2 74 2 2 74 ¯
180 Ê 5 7 7 5ˆ
4y — 3x = = Á - , - ˜ = (—1, 1)
130 Ë 2 2 2 2¯
Similarly, we can easily find that the equation of AB is
34.
480 A
5y - 12x + =0
130
31. The equations of the line which are equally inclined q q
with the axes are
3

x+
y=

y=

x + y = a, x – y = a

5
7x

It is given that,
3-4-a 1+ 2 - a
=± B D C
2 2
fi –1 – a = ±(3 – a) On solving x + y = 5 and 7x – y = 3, we get the co-
fi –1 – a = (3 – a), –1 – a = –3 + a ordinates of A, i.e. A = (1, 4)
fi –1 – a = –3 + a Let AB = AC = m
fi 2a = 2
Thus, BC = 2m sin q
fi a=1
Hence, the equation of the line is x – y – 1 = 0 Now, ar(DABC) = 5

CG_01a.indd 60 2/6/2017 3:58:50 PM


Straight Lines 1.61

1 fi h - 4 k -1
fi ◊ 2m sin q ◊ m cos q = 5 = = -2
2 1 -1
fi m2 sin q = 10 fi h = 2, k = 3
10 10 25 Thus, the required image is (2, 3).
fi m2 = = =
sin 2q 4/5 2 (ii) Now (2, 3) becomes (2 + 1, 3) i.e., (3, 3)
(iii) Let z1 = 3 + 3i
5
fi m= i
p
2 Then z2 = z1 ¥ e 4

Now, slope of AC = tan j = 7 1


fi z2 = (3 + 3i ) ¥ (1 + i )
1 7 2
fi cos j = , sin j = 1
50 50 = (3 + 3i ) ¥ (1 + i )
2
Ê 5 1 5 7 ˆ
Thus, C = Á1 ± ◊ ,4± ◊ ˜ 3 3
Ë 2 50 2 50 ¯ = (1 + i ) 2 = i
2 2
Ê 1 7ˆ
fi C = Á1 ± , 4 ± ˜ 3 Ê 3 ˆ
Ë 2 2¯ = i = Á 0, ˜
2 Ë 2¯
fi Ê 3 15 ˆ Ê 1 1ˆ
C = Á , ˜ or Á , ˜ 36.
Ë2 2¯ Ë2 2¯ A(–1, 3)
Also, Slope of AB = tan y = –1
1 1
cos y = - and sin y =
2 2 E
Ê 5 Ê 1 ˆ 5 1 ˆ
Thus, D = Á1 ± ◊Á- ˜ ,4± ◊ H(1, 2)
Ë 2 Ë 2¯ 2 2 ˜¯
C
Ê 5 5ˆ B(2, 5) D
= Á1 ,4± ˜
Ë 2 2¯ To find the equations of BC and AC.
Ê 3 13 ˆ Ê 7 3 ˆ 2-3 1
= Á - , ˜ or Á , ˜ BC: m( AD) = =-
Ë 2 2 ¯ Ë 2 2¯ 1+1 2
Therefore, the four possible equations of BC are fi m(BC) = 2
15 3
15 2 - 2 Ê
Equation of BC is
3ˆ 3Ê 3ˆ
y- = ÁË x - ˜¯ = - ÁË x - ˜¯ , y – 5 = 2(x – 2)
2 3 7 2 2 2
- fi 2x – y + 1 = 0 …(i)
2 2
15 3 2-5
- 3ˆ 3ˆ AC: m ( BE ) = =3
Ê Ê
= 2 2 Á x - ˜ = 4Á x - ˜ , 1- 2
3 3Ë 2 ¯ Ë 2¯
+ 1
2 2 fi m ( AC ) = -
3
1 3
1 2- 2Ê 1ˆ 1 Ê 1ˆ Equation of AC is
fi y- = ÁË x - ˜¯ = ÁË x - ˜¯ 1
2 1 7 2 3 2
- y - 3 = - ( x + 1)
2 2 3
1 13 fi 3y – 9 = –x – 1
1 2- 2 Ê 1ˆ Ê 1ˆ
and y - = Á x - ˜¯ = - 3 ÁË x - ˜¯ fi x + 3y = 8 …(ii)
2 1+3 Ë 2 2
2 2 Solving Eqs (i) and (ii), we get
35. Let (h, k) be the image of (4, 1) w.r.t. the line y = x – 1, 5 17
x= ,y=
i.e. x – y – 1 = 0 7 7
h - 4 k -1 Ê 4 - 1 - 1ˆ Ê 5 17 ˆ
Thus, = = - 2Á 2 2 ˜ Thus, the co-ordinates of the third vertex are Á , ˜ .
1 -1 Ë 1 +1 ¯ Ë7 7 ¯

CG_01a.indd 61 2/6/2017 3:58:51 PM


1.62 Coordinate Geometry Booster

Integer Type Questions fi r=4 2


1. Let the line be ax + by + c = 0 …(i) Clearly, d 2 = 4 2
Let p1, p2 and p3 are the perpendicular distances from fi d=4
the line (i). fi d+2=6
Clearly, p1 + p2 + p3 = 0 3. Equation of any line passing through (2, 3) is
3a + c 3b + c 2a + 2b + c y – 3 = m(x – 2)
fi + + =0 fi mx – y = 2m – 3
2 2 2 2
a +b a +b a 2 + b2 x y
fi + =1
fi 3a + c + 3b + c + 2a + 2b + c Ê 2m — 3 ˆ (3 — 2m)
=0 ÁË ˜
a 2 + b2 m ¯
It is given that,
5a + 5b + 3c 1 Ê 2m - 3 ˆ
fi =0 ¥Á ˜ ¥ (3 - 2m) = ± 12
a 2 + b2 2 Ë m ¯
fi 5a + 5b + 3c = 0 fi (3 – 2m)2 = 24m
5 5 fi 9 – 12m + 4m2 = 24m
fi a+ b+c=0
3 3 4m2 + 12m + 9 = 0, 4m2 – 36m + 9 = 0
Ê 5 5ˆ
which passes through a fixed point Á , ˜ . fi (2m + 3)2 = 0, 4m2 – 36m + 9 = 0
Ë 3 3¯
5 5 fi (2m + 3) = 0, 4m2 – 36m + 9 = 0
Thus, p = , q = . fi m = –3/2, D > 0
3 3
Hence, the value of So, the number of lines is 3.
3(p + q) – 2 4. We have,
3x + 4(mx + 2) = 9
Ê 5 5ˆ fi (3 + 4m)x = 1
= 3Á + ˜ - 2
Ë 3 3¯
1
= 10 – 2 = 8 fi x=
(3 + 4m)
p
2. Clearly, slope of the line 2x – 2y + 5 = 0 is 1, i.e. q = when m = –1, then x = –1
4 Thus, the number of integral values of m is 1.
Equation of any line passing through (2, 3) and making 5. Hence, the required area of the parallelogram
p (1 - 0)(1 - 0) 1
an angle is = = = 1 sq. u.
4 2 -1 (- 2 + 1)
x-2 y-3 1 -1
= =r
p
Ê ˆ Êpˆ
cos Á ˜ sin Á ˜ D
Ë 4¯ Ë 4¯ 6.
x-2 y-3
fi = =r
1 1 C
2 2 A(4, 5)
r
fi ( x - 2) = ( y - 3) = …(i)
2 q
Thus, any point on the line (i) is B(1, 1)
Ê r r ˆ
ÁË 2 + ,3+ ˜ We have
2 2¯
2 2
AB = (4 — 1) + (5 — 1) = 9 + 16 = 25 = 5
which lies on 2x – 3y + 9 = 0
Area of a rhombus = 2ar (DABC)
Ê r ˆ Ê r ˆ
So, 2Á 2 + ˜¯ - 3 ÁË 3 + ˜ +9=0 Ê1 ˆ
Ë 2 2¯ = 2 Á ¥ 5 ¥ 5 ¥ sin q ˜
Ë2 ¯
Ê 2r 3r ˆ = 25 sin q
fi ÁË - ˜ +4=0
2 2¯ Maximum area of a rhombus is 25.
r Thus, 5m = 25
fi - = -4 fi m=5
2

CG_01a.indd 62 2/6/2017 3:58:51 PM


Straight Lines 1.63

7. Here, the rhombus represented by the lines fi 0 < a < 4 and a < 0, a > 2
x + 2y + 2 = 0 fi 2<a<4
x + 2y — 2 = 0 fi a Œ (2, 4) = (p, q)
x — 2y + 2 = 0 Hence, the value of (p + q + 1)
x — 2y — 2 = 0 =2+4+1
=7
Hence, the required area of the rhombus
(2 - (- 2))(2 - (-2)) Previous Years’ JEE-Advanced Examinations
=
1 2
1. Let the third vertex be (x, y) i.e. (x, x + 3).
1 -2
2 1 1
16 1
= = 4 sq. u. It is given that 3 -2 1 = 5
4 2
x x+3 1
8(8 + 1)
8. Number of latice point = = 36 1
2 fi (4x - 4) = ± 5
Clearly, m(m + 5) = 36 2
fi m2 + 5m – 36 = 0 fi (4x – 4) = ±10
fi (m + 9)(m – 4) = 0 fi 4x = 4 ± 10
fi m = 4, –9 fi 4x = 14, –6
Hence, the value of m is 4.
7 3
9. Given lines are fi x= ,—
2 2
2x + y = 6, x + y = 9, x = 0 and y = 0.
Thus, the co-ordinates of the third vertex be
Y
Ê 7 13 ˆ Ê 3 3 ˆ
ÁË , ˜¯ or ÁË - , ˜¯
2 2 2 2
4 2. Let the equation of D C(1, 1)
DC is 4x + 7y + k
3 x+
y= = 0 which is pass-
2 5 ing through (1,
1).Thus, k = –11.
2x+

1 4x + 7y + 5 = 0
Hence, the equa-
y=
2

O X tion of DC is 4x + A(–3, 1) B
1 2 3 4
7y – 11 = 0
Clearly BC is perpendicular to AB.
From the above figure, it is clear that the number of IIT
Let the equation of BC is 7x – 4y + k = 0
points is 6.
which is passing through (1, 1).
10. Given lines are 2y = x and 4y = x
So, k = –3
x x
fi y = and y = Hence, the equation of DC is 7x – 4y – 3 = 0.
2 4 Also, AD is parallel to DC
Y Let the equation of DC is 7x – 4y + k = 0
which is passing through (–3, 1)
x
y= So, k = 25
2
Thus, the equation of DC is 7x – 4y + 25 = 0.
P(a, a2) 3.
x Y
y=
4
X B(0, b)
O 2

a2 a2 P(h, k)
We have a - > 0 and a - <0 1
4 2
fi (4a – a2) < 0 and (2a – a2) > 0 X¢
O A(a, 0)
X
fi (a2 – 4a) < 0 and (a2 – 2a) > 0
fi a(a – 4) < 0 and a(a – 2) > 0 Y¢

CG_01a.indd 63 2/6/2017 3:58:52 PM


1.64 Coordinate Geometry Booster

2a + 0 2a b+0 b 10. Reflection of the point (4, 1) w.r.t. the line y = x is


Here, h = = ,k = = (1, 4).
2 +1 3 2 +1 3
After transformation of 2 units along the positive direc-
3h
fi a= , b = 3k tion of x-axis, it becomes (3, 4).
2 Let z1 = 3 + 4i and the final position of the point is z2.
It is given that, AB = l
ip Ê 1 1 ˆ
fi a 2 + b2 = l Thus, z2 = z1 ¥ e 4 = z1 ¥ Á +i◊ ˜
Ë 2 2¯
fi a2 + b2 = l2 1
9 2 fi z2 = (3 + 4i ) ¥ (1 + i )
fi h + 9k 2 = l 2 2
4 1
fi z2 = ¥ (3 + 4i ) ¥ (1 + i )
fi 9h2 + 36k2 = 4l2 2
Hence, the locus of (h, k) is 1
fi z2 = ¥ (3 + 4i + 3i — 4)
9x2 + 36y2 = 4l2 2
4. 1
5. Here, a1 = 1, b1 = –2 and a2 = 4, b2 = –3 fi z2 = ¥ (—1 + 7i )
2
Now, a1a2 + b1b2 = 4 + 16 = 10
Ê 1 7 ˆ
Thus, the obtuse-angle bisector is fi z2 = Á - , ˜
x - 2y + 4 4x - 3y + 2 Ë 2 2¯
= 11. D
1+ 4 16 + 9 C(5, 1)
x - 2y + 4 4x - 3y + 2 M(3, 2)
fi =
5 5
x - 2y + 4 4x - 3y + 2
fi = B(p, q)
1 5 A(1, 3)

fi (4x - 3y + 2) = 5( x - 2y + 4) Clearly, the mid-point of (1, 3) and (5, 1) is (3, 2) lies


fi (4 - 5) x - (3 - 2 5) y + (2 - 4 5) = 0 on the diagonal.
So, 2 = 6 + c
6. Let A = (–a, –b), B = (0, 0), C = (a, b)
fi c = –4
b
Now, m( AB ) = = m( BC ) Equation of one diagonal is y = 2x – 4.
a Let B = (p, q)
Thus, the points A, B and C are collinear. Now, m(BC) ¥ m(AB) = –1
7. Let A = (–a, –b), B = (0, 0), C = (a, b)
fi Ê q - 1 ˆ Ê q - 3ˆ
and D = (a2, ab) ÁË p - 5 ˜¯ ¥ ÁË p - 1˜¯ = - 1
b
Now, m( AB ) = = m( BC ) = m(BC ) fi (q – 1)(q – 3) = –(p – 1)(p – 5)
a
Thus, the points A, B, C and D are collinear. fi q2 – 4q + 3 = –p2 + 6p – 5
8. Equation of any line perpendicular to 5x – y = 1 is fi p2 + q2 – 6p – 4q + 8 = 0 …(i)
x + 5y – k = 0 Also B lies on the line y = 2x – 4.
Ê kˆ So, q = 2p – 4 …(ii)
Let it intersects the x-axis at (k, 0) and y-axis at Á 0, ˜
Ë 5¯
1 k On solving Eqs (i) and (ii), we get,
Thus, ¥ k ¥ = 5 p = 2, 4 and q = 0, 4
2 5
fi k2 = 50 Hence, the co-ordinates of the other vertices are (2, 0)
fi k = ±5 2 and (4, 4).
Hence, the equation of the line L is 12. Given line is ax + by + c = 0 …(i)
x + 5y = ± 5 2 and 2a + 3b + 4c = 0
1 3
9. Now, consider the lines x + 2y = 3, 2x + 3y = 4 and fi a+ b+c=0 …(ii)
4x + 5y = 6. 2 4
1 2 -3 1 2 -3 Subtracting Eqs (ii) from (i), we get
Ê R2 Æ R2 - R1 ˆ
Now, 2 3 -4 = 1 1 -1 ÁË R Æ R - R ˜¯ Ê 1ˆ Ê 3ˆ
3 3 2
aÁ x - ˜¯ + b ÁË y - ˜ =0
4 5 -6 1 1 -1 Ë 2 4¯
=0 Ê 1ˆ b Ê 3ˆ
fi ÁË x - ˜ + Áy- ˜ =0
Thus, the above three lines are concurrent. 2¯ a Ë 4¯

CG_01a.indd 64 2/6/2017 3:58:53 PM


Straight Lines 1.65

Ê 1ˆ Ê 3ˆ First we find the equations of AD and BE.


fi ÁË x - ˜¯ = 0, ÁË y - ˜¯ = 0 1
2 4 Now, m( BC ) =
1 3 t3
fi x= ,y= fi m(AD) = t3
2 4
Ê 1 3ˆ Thus, AD is y – a(t1 + t2) = –t3x + at1t2t3
Thus, the point is concurrent at Á , ˜ .
Ë 2 4¯ Similarly, BE is y – a(t2 + t3) = –t1x + at1t2t3
13. Clearly, the given lines intersect at (2, –2), (–2, 2) and On solving the equations of AD and BE, we get
(1, 1). x = –a, y = a(t1 + t2 + t3 + t1t2t3)
Let P = (2, –2), Q = (–2, 2) and R = (1, 1)
Thus, the required orthocentre is
\ PQ = 16 + 16 = 4 2 (–a, a(t1 + t2 + t3 + t1t2t3)).
Thus, PR = 1 + 9 = 10 16. The point of intersection of the lines x + 2y =10 and
and QR = 9 + 1 = 10 . Ê 20 25 ˆ
2x + y + 5 = 0 is Á - , ˜ .
Ë 3 3¯
So, P, Q and R form an isosceles triangle.
Clearly, the line 5x + 4y = 0 passes through the point
14. Given AB = c
Ê 20 25 ˆ
Y ÁË - , ˜¯ .
3 3
B(0, k) 17. Given the co-ordinates of A, B, C and P are (6, 3),
P(h, k)
(–3, 5), (4, –2) and (x, y).
M Now, ar (DPBC)
X x y 1
O A(h, 0) 1
= -3 5 1
2
x y 4 -2 1
Let AB:+ =1 …(i)
h k 1
h =|7x + 7y - 14|
and PM : y - k = ( x - h) …(ii) 2
k
7
On solving Eqs (i) and (ii), we get = | x + y - 2|
2
h3 h3 k Ê h3 ˆ
6 3 1
x= = and y = k + ÁË 2 - h˜¯
2
h +k 2
c 2
h c 1
ar (DABC) = -3 5 1
2
h3 k3 4 -2 1
Thus, x = and y =
c2 c2 1 49
= |42 + 21 - 14| =
3 ˆ 2/3 3 ˆ 2/3 2 2
2/3 2/3 Ê h Êk
Now, x + y = Á 2 ˜ +Á 2˜ 7
Ëc ¯ Ëc ¯ | x + y — 2|
ar ( DPBC )
h2 + k 2 Thus, = 2
fi x 2/3 + y 2/3 = ar ( DABC ) 49
c 4/3 2
c2 x+ y—2
fi x 2/3 + y 2/3 = =
c 4/3 7
fi x2/3 + y2/3 = c2/3 18. Two given lines are 7x – y + 3 A(0, 3)
15. = 0 and x + y – 3 = 0. So
(at1t2, a(t1 + t2)) the point of intersection
0

A
3=

x+

is (0, 3).
y–
y+

Let m(BC) = m
3=

m(AB) = 7,
7x

F E m(AC) = –1
Clearly, –ABC = – B C
ACB
C m-7 m +1
B D Thus, =
(at2t3, a(t2 + t3)) (at1t3, a(t1 + t3)) 1 + 7m 1 - m

CG_01a.indd 65 2/6/2017 3:58:55 PM


1.66 Coordinate Geometry Booster

fi 6m2 + 16m – 6 = 0 On solving AD and BC, we get the orhocentre is


fi 3m2 + 8m – 2 = 0 Ê 3 22 ˆ
1 ÁË , ˜¯ , which lies in the first quadrant.
7 7
fi m = - 3, A(0, b)
3 0 y–
22. Since the sides of the = 7x
Hence, the equation of the BC is –2 –
rhombus are parallel to x 3
1 – =
y + 10 = - 3( x - 1) or y + 10 = ( x - 1) the lines y = x + 2 and y = y
M(1, 2)
0
3 7x + 3, so its diagonals D B
fi 3x + y – 7 = 0 or x – 3y – 31 = 0. are parallel to the bisec-
19. Given a, b, c are in AP. tors of the angles between
fi 2b = a + c these lines. C
Now, ax + by + c = 0
Ê y - x - 2ˆ Ê y - 7x - 3 ˆ
fi 2ax + 2by + 2c = 0 Thus, =±
ÁË 1 + 1 ˜¯ ÁË 1 + 49 ˜¯
fi 2ax + (a + c)y + 2c = 0
fi a(2x + y) + c (y + 2) = 0 Ê y - x - 2ˆ Ê y - 7x - 3 ˆ
c fi ÁË ˜¯ = ± ÁË ˜
fi (2x + y ) + ( y + 2) = 0 2 5 2 ¯
a
fi 5(y – x – 2) = ± (y – 7x – 3)
fi (2x + y) = 0, (y + 2) = 0
fi 2x + 4y – 7 = 0 or 12x – 6y + 13 = 0
fi x = –1, y = –2 Therefore, the slope of AM is 2 or –1/2
Thus, the given straight line passes through a fixed b-2 1
point is (1, –2). Thus, = 2 or -
0 -1 2
20. Three given lines are concurrent, if 5
fi b = 0 or
p q r 2
q r p =0 Hence, the possible co-ordinates of A are
r p q Ê 5ˆ
(0, 0) or Á 0, ˜
fi –(p3 + q3 + r3 – 3pqr) = 0 Ë 2¯
fi (p3 + q3 + r3 – 3pqr) = 0 x1 y1 1 a1 b1 1
fi (p + q + r) (p2 + q2 + r2 – pq – qr – rp) = 0 23. Given x2 y2 1 = a2 b2 1
\ (p + q + r) = 0, (p2 + q2 + r2 – pq – qr – rp) = 0
x3 y3 1 a3 b3 1
fi (p + q + r) = 0, p2 + q2 + r2 = pq + qr + rp) = 0
21. x1 y1 1 a1 b1 1
A 1 1
fi x2 y2 1 = a2 b2 1
2 2
x3 y3 1 a3 b3 1
=0

2x
+
+4

F 3y Thus, areas are the same.


–y

=
6 But it is not sure that it will be congruent.
4x

24. Q
B D C D C(h, k)
x=y=1
On solving the given equation, we get
O
Ê 3 16 ˆ Ê 3 4ˆ
A = Á - , ˜ , B = Á - , ˜ and C = (- 3, 4)
7

Ë 7 7¯ Ë 5 5¯
x+
4y =

Equation of AD is A(–3, 4) B(5, 4)


16 Ê 3ˆ
y- =Áx + ˜ P
7 Ë 7¯
Let C = (h, k).
fi 7x – 7y + 19 = 0
Slope of AB = 0 and BC is perpendicular to AB.
Equation of BC is Thus, h – 5 = 0
1 fi h=5
y - 4 = ( x + 3)
4 Clearly the mid-point of AC lies on the given diameter
fi x + 4y – 13 = 0 PQ.

CG_01a.indd 66 2/6/2017 3:58:56 PM


Straight Lines 1.67

Ê k + 4ˆ h - 3 Let the co-ordinates of B and C are (x1, y1) and (x2, y2),
Thus, 4 Á = +7 =8
Ë 2 ˜¯ 2
respectively.
Clearly the mid-point of A and B and A and C lie on the
fi k=0 perpendicular bisectors x + 2y = 0 and x – y + 5 = 0.
Therefore C is (5, 0)
x1 + 1 Ê y - 2ˆ
So, BC = 4 and AB = 8 So, + 2Á 1 =0
2 Ë 2 ˜¯
Hence, the area of the rectangle ABCD is
= AB ¥ BC = 8 ¥ 4 = 32 sq. u. fi x1 + 2y1 – 13 = 0 …(i)
25. Ê x2 + 1ˆ Ê y2 - 2 ˆ
26. All the points inside the A(1, 3) and ÁË 2 ˜¯ - ÁË 2 ˜¯ + 5 = 0
triangle will lie in the
half plane determined by fi x2 – y2 + 13 = 0 …(ii)
ax + by + c ≥ 0
Ê y1 + 2 ˆ Ê 1 ˆ
if all the vertices lie in it. Also, Á Á - ˜ = -1
Ë x1 - 1 ˜¯ Ë 2 ¯
For the first,
3x + 2y > 0, fi 2x1 – y1 – 4 = 0
B(5, 0) C(–1, 2)
3(1) + 2(3) = 9 ≥ 0 Ê y2 - 2 ˆ
3(5) + 2(0) = 15 ≥ 0 and ÁË x - 1 ˜¯ (1) = - 1
2
3(–1) + 2(2) = 2 ≥ 0
fi x2 + y2 + 1 = 0 …(iv)
Thus, all the vertices lie in the half plane determined by
3x + 2y > 0. Solving Eqs (i) and (iii), we get the co-ordinates of B,
For the second, Ê 11 2ˆ
i.e. B = Á , ˜
2x + y – 13 > 0 Ë5 5¯
2(1) + 3 – 13 = –8 £ 0 Solving Eqs (ii) and (iv), we get the co-ordinates of C.
2(0) + 0 – 13 = –3 £ 0 i.e. C = (–7, 6)
2(–1) + 2 – 13 = –13 £ 0 2
6-
Thus, all the vertices do not lie in the half plane deter- 5 28 14
Now, m( BC ) = =- =- .
mined by 2x + y – 13 > 0. 11 46 23
-7 -
For the third, 5
2x – 3y – 12 < 0 Therefore, the equation of BC is
2(1) – 3(3) – 12 = –19 £ 0 14
y-6=- ( x + 7)
2(5) – 3(0) – 12 = –2 £ 0 23
2(–1) – 3(2) – 12 = –20 £ 0 fi 23y – 138 = 14x – 98
Thus, all the vertices lie in the half plane determined by fi 14x + 23y – 40 = 0
2x – 3y – 12 £ 0.
28 The slope of
For the fourth,
–2x + y £ 0 Ê 8ˆ
ÁË 0, ˜¯ and (1, 3) and (1, 3) and (82, 30)
–2(1) + 3 = 1 ≥ 0 3
–2(5) + 0 = –10 £ 0 are same. So the given points are collinear.
–2(–1) + 2 = 4 ≥ 0 No questions asked in 1987.
Thus, all the vertices do not lie in the half plane deter- 29. We know that two lines a1x + b1y + c1 = 0 and a2x + b2y
mined by –2x + y £ 0. + c2 = 0 cut the co-ordinate axes in concyclic points if
a1a2 = b1b2.
Hence, all points inside the triangle satisfy the inequali-
ties 3x + 2y ≥ 0 and 2x – 3y – 12 £ 0. 19 17 243
Here, a1a2 = - ¥ =-
27. 2 9 18
A
19 17 243
x+ and b1b2 = - ¥ =-
2y 3 6 18
=0
E =0 Thus, the given lines cut the co-ordinate axes in con-
F 5
y+ cyclic points.
x–
M 30. According to the question, the required line passes
through the point of intersection of L1 and L2.
B C

CG_01a.indd 67 2/6/2017 3:58:56 PM


1.68 Coordinate Geometry Booster

L fi b2(a – t) = a2t
ab 2
fi t=
q a 2 + b2
q L1
Thus, the co-ordinates of E are
L2 Ê ab 2 a 2b ˆ
Á 2 , ˜
Equation of any line passing through the point of inter- Ë a + b2 a 2 + b2 ¯
section of L1 and L2 is
and the co-ordinates of F are
L1 + lL2 = 0
Ê ab 2 a 2b ˆ
fi ax + by + c + l(lx + my + n) = 0 Á , ˜
Here, L1 is equally inclined with L and L2, thus Ë 2(a 2 + b 2 ) 2(a 2 + b 2 ) ¯
m3 - m1 m - m2 Ê a 2 + b2 ˆ
= 1 Now, slope of AF = - ÁË ˜
1 + m1m3 1 + m1m2 ab ¯
a + ll a a l Ê ab ˆ
- - and slope of BE = - Á 2
b + lm b b m Ë a + b 2 ˜¯
fi =
Ê a + ll ˆ Ê a ˆ Ê aˆ Ê l ˆ Clearly, AF ^ BE.
1+ Á ˜ ÁË ˜¯ 1 + ÁË b ˜¯ ÁË m ˜¯
Ë b + lm¯ b 32. Clearly, the point A is (3, –1).
am - bl l (bl - am) Equation of BC is y – 2 = m(x – 1)
fi = 2
al + bm a + b 2 + l (bm + al ) 3
m+
a 2 + b2 4 = tan 45°
fi l=- Thus,
3
al + bm + 1 1- m
Hence, the equation of the required line is 4
4m + 3
a 2 + b2 fi =1
ax + by + c - (lx + my + n) = 0 4 - 3m
al + bm + 1
Ê 4m + 3 ˆ
31. fi ÁË 4 - 3m ˜¯ = ± 1
A(0, b)
1
fi m = - 7,
7
Hence, the equation of the line BC is
1
E y - 2 = - 7( x - 1) or y - 2 = ( x - 1)
7
F x – 7y + 13 = 0 or 7x + y – 9 = 0.
B(–a, 0) D C(a, 0) x y
33. Let L: + =1
a b
Let the co-ordinates of A, B and C are (0, b), (–a, 0) and
After rotation through a given angle is
(a, 0), respectively and D as origin.
x y
x y L: + =1
Equation of AC is + = 1 p q
a b
Ê Ê t ˆˆ So, the distance from the origin of both the lines will be
Consider the co-ordinates of E be Á t , b ÁË 1 - ˜¯ ˜ .
Ë a ¯ the same.
Ê tˆ 0 + 0 -1 0 + 0 -1
b Á1 - ˜ Thus, =
Ë a ¯ b( a - t ) 1 1 1 1
Now, m( DE ) = = + 2 + 2
t at a 2
b p 2
q
b
and m( AC ) = -
a 1 1 1 1
fi 2
+ 2
= 2
+
Since DE ^ AC a b p q2

b( a - t ) Ê b ˆ Ê 1 1ˆ Ê 1 1ˆ
fi ¥ Á - ˜ = -1 fi ÁË 2 + 2 ˜¯ = Á 2 + 2 ˜
at Ë a¯ a b Ëp q ¯

CG_01a.indd 68 2/6/2017 3:58:57 PM


Straight Lines 1.69

34. Equation of any line through P is


Y
y – 3 = m(x – 2) …(i)
Q(0, b)
The line (i) intersects the given lines at A and B, respec-
R
tively.
Thus, the co-ordinates of A and B are
P(a, 0) Ê 2m 6 - m ˆ
X A=Á ,
Ë m + 2 m + 2 ˜¯
O A(7, 0)

M Ê 2m + 2 m — 6 ˆ
and B=Á ,
Ë m + 2 m + 2 ˜¯
B(0, 5) It is given that, AB = 2
x y fi AB2 = 4
Equation of AB is + =1 2 2
7 -5 Ê 2m + 2 - 2m ˆ Ê m - 6 - 6 + mˆ
fi ÁË +Á ˜¯ = 4
x y m + 2 ˜¯ Ë m+2
Equation of PQ is + = 1
a b Ê 2 ˆ
2
Ê 2m - 12 ˆ
2
fi ÁË m + 2 ˜¯ + ÁË m + 2 ˜¯ = 4
Here, PQ is perpendicular to AB.
b 5 Ê 1 ˆ Ê m - 6ˆ
2
Thus, - ¥ = -1
a 7 fi ÁË m + 2 ˜¯ + ÁË m + 2 ˜¯ = 1
b a fi 1 + (m – 6)2 = (m + 2)2
fi = = k (say)
7 5 fi 1 + m2 – 12m + 36 = m2 + 4m + 4
Therefore, the equation of BP is 33
x y fi 16m = 33 fi m=
- =1 16
a 5 Hence, the equation of the line is
x y 33
fi - =1 y - 3 = ( x - 2)
5k 5 16
fi x – ky = 5k …(i) fi 16y – 48 = 33x – 6
fi 33x – 16y = 66 – 48 = 18
Similarly, the equation of AQ is
x y 36. Let the variable line be ax + by + c = 0.
+ =1 (2a + c) (2b + c) (a + b + c)
7 b Given + + =0
2 2
x y a +b a 2 + b2 a 2 + b2
fi + =1
7 7k fi (2a + c) + (2b + c) + (a + b + c) = 0
fi kx + y = 7k …(ii) fi 3(a + b + c) = 0
fi (a + b + c) = 0
From Eqs (i) and (ii), we get
Thus, the line ax + by + c = 0 passes through the point
Ê x ˆ Ê x ˆ (1, 1).
ÁË y + 5 ˜¯ x + y = 7 ÁË y + 5 ˜¯
37. |x| + |y| = 1
fi x2 + y2 – 7x + 5y = 0 Clearly, the locus is a square.
38. A(–7, 5)
35.
Y
x+
0
y=

2y
+3

–3

P(a, a2)
2x

=0

B
A y+
2x
= B 5x – 6y – 1= 0 C
P(2, 3) 5
y+ X (1/3, 1/9) (5/4, 7/8)
O
2x
= On solving the equations, we get the co-ordinates of A,
3
B and C, respectively.
Now, A and P lie on the same side of the line
5x – 6y – 1 = 0

CG_01a.indd 69 2/6/2017 3:58:59 PM


1.70 Coordinate Geometry Booster

5a — 6a 2 - 1 15
Thus, >0 fi = (cos q + 3 sin q )
5(-7) - 6(5) - 1 AB
fi 6a2 – 5a + 1 > 0 10
Similarly, = 2 cos q + sin q
fi (3a – 1) (2a – 1) > 0 AC
1 1 6
fi a < or a > …(i) and = cos q - sin q
3 2 AD
Again, the points P and B lie on the same side of the It is given that
2 2 2
line x + 2y – 3 = 0. Ê 15 ˆ Ê 10 ˆ Ê 6 ˆ
a + 2a 2 - 3 ÁË ˜¯ + ÁË ˜¯ = ÁË ˜
AB AC AD ¯
Thus, >0
1 2 fi (cosq + 3sinq) + (2cosq + sinq)2 = cosq – sinq)2
2
+ -3
3 9 fi 4cos2q + 9sin2q + 12sinq cosq = 0
fi 2a2 + a – 3 < 0 fi (2cosq + 3sinq)2 = 0
fi (2a + 3) (a – 1) < 0 fi (2cosq + 3sinq) = 0
3 2
fi - <a <1 …(ii) fi tan q = -
2 3
Finally, the points P and C lie on the same side of the Hence, the equation of the required line is
line 2x – 3y – 1 = 0 2
y + 4 = - ( x + 5)
2a + 3a 2 - 1 3
Thus, >0
Ê 5ˆ Ê 7ˆ fi 3y + 12 = –2x – 10
2 Á ˜ + 3Á ˜ - 1
Ë 4¯ Ë 8¯ fi 2x + 3y + 22 = 0
fi 3a + 2a – 1 > 0
2 41. As we know that the point of intersection of any two
fi (3a – 1) (a + 1) > 0 perpendicular sides of a triangle is also called ortho-
centre.
1
fi a < - 1 or a > …(iii) Given lines are xy = 0 and x + y = 1
3
fi x = 0, y = 0 and x + y = 1
From Eqs (i), (ii) and (iii), we get 42. Ans. (m2 – 1)x – my + b(m2 + 1) + am = 0
Ê 3 ˆ Ê1 ˆ 43. The given lines are x + 3y = 4 and 6x – 2y = 7 which
a Œ Á - , - 1˜ » Á , 1˜
Ë 2 ¯ Ë2 ¯ are mutually perpendicular to each other. Thus, PQRS
39. Let BD is the angle bisector of the –ABC. must be a rhombus.
44. Equation of PS is 2x + 9y = 22.
AD BA 36 + 64 10 2
Now, = = = = Equation of any line parallel to PS is
DC BC 16 + 9 5 1 2x + 9y + k = 0
Ê 1 1ˆ Which is passing through (1, –1)
Thus, the co-ordinates of D are Á , ˜
Ë 3 3¯ So, k = 7
Therefore, the equation of BD is Hence, the equation of the required line
1 is 2x + 9y + 7 = 0.
( y - 1) = ( x - 5)
7 45.
fi x – 7y + 2 = 0 46.
40. Equation of any line passing through A is 47.
Y
x+5 y+4
= = r1 , r2 , r3 y = mx + 1
cos q sin q
Let AB = r1, AC = r2, AD = r3 Q y = mx
Clearly, the point (r1cosq – 5, r1sinq – 4) lies on the line M P
x + 3y + 2 = 0 R
X¢ X
So, (r1cosq – 5) + 3(r1sinq – 4) + 2 = 0 O
y = nx + 1
15
fi r1 = y = nx
cos q + 3 sin q Y¢
15 Area of a parallelogram OPQR
fi = (cos q + 3 sin q )
r1 = 2 (ar of DOPQ)

CG_01a.indd 70 2/6/2017 3:59:00 PM


Straight Lines 1.71

1 Ê 1 m ˆ
= 2¥ ¥ OQ ¥ PM P=Á , and
2 Ë m + 1 m + 1˜¯
= OQ ¥ PM Ê 3 3m ˆ
Q=Á ,
1 Ë m + 1 m + 1˜¯
=
|m - n| Equation of the lines L1 and L2 are
2-m
48. We have 3x + 4mx + 4 = 9 L1: 2x - y =
m +1
fi x(3 + 4m) = 5
9 + 3m
5 L2 : 3x + y =
fi x= m +1
(3 + 4m)
Let the co-ordinates of R be (h, k)
When m = –1, then x = –5
11 + 2m 12 + 9m
When m = –2, then x = –1 Thus, h = ,k =
5(m + 1) 5(m + 1)
Thus, the number of possible integral values of m is 2.
49. Eliminating m from the above relations, we get
Y 5h – 15k + 25 = 0
Q fi h – 3k + 5 = 0
(3, 3 3) Hence, the locus of (h, k) is
6 x – 3y + 5 = 0
M 51. Equation of any line passing through (8, 2) is
1 y – 2 = m(x – 8)
X¢ X Put y = 0, m(x – 8) = –2.
P Q
2
(–1, 0) (0, 0) fi ( x - 8) = -
m
2
Y¢ fi x=8-
m
Since OM is the internal bisector of –PQR.
2
Thus, OP = 8 -
QM OQ 6 m
= =
PM OP 1 Put x = 0, then y = 2 – 8m
Ê 3 3 3ˆ Thus, OQ = 2 – 8m
Thus, the co-ordinates of M = ÁË - , ˜
7 7 ¯
Ê 2ˆ
Let S = OP + OQ = Á 8 - ˜ + (2 - 8m)
Therefore, the equation of OM is Ë m¯
dS 2
y=- 3x fi = -8
dm m 2
fi y + 3x = 0 For maximum or minimum,
dS
50. =0
Y dm
L2
2
Line parallel to 3x + y = 5 fi -8=0
Q L1 m2
Line parallel to 2x + y = 5
1
fi m2 =
4
R(h, k)x
+
y= 1
P fi m=±
3 2
X¢ X
O
d 2S 2
k+

L3 = mx Now, =-
y=

2
dm m3
1


Ê d 2S ˆ
fi Á 2˜ = 16 > 0
Let L: y = mx Ë dm ¯ m = - 1
Then the co-ordinates of P and Q are 2

CG_01a.indd 71 2/6/2017 3:59:02 PM


1.72 Coordinate Geometry Booster

Thus, the sum will provide us the least value. 1


The least value is = 8 + 4 + 2 + 4 = 18. Equation of OE is y = x
4
52. Equation of any line passing through (2, 2) is
Ê 3ˆ
y – 2 = m(x – 2) Thus the line BD and OE meet in Á 3, ˜
Ë 4¯
Let the points A and B are
Ê 3ˆ
Ê 2m - 2 Ê 2m - 2 ˆ ˆ Hence, the centroid is Á 3, ˜ .
A=Á , - 3Á Ë 4¯
˜˜
Ëm+ 3 Ë m + 3¯¯ 55. Let the point P be (x, y)
Ê 2m - 2 Ê 2m - 2 ˆ ˆ Given
and B = Á , 3Á ˜˜
Ëm- 3 Ë m - 3¯¯ ( x - a1 ) 2 + ( y - b1 ) 2 = ( x - a2 ) 2 + ( y - b2 ) 2
Since the triangle OAB is equilateral, so
(x – a1)2 + (y – b1)2 = (x – a2)2 + (y – b2)2
OA = OB = AB
2 2 a12 – 2a1x + b12 – 2b1y = a22 – 2a2x + b22 – 2b2y
Ê 2m - 2 ˆ Ê 2m - 2 ˆ

ÁË (m + 3) ˜¯ + 3 ÁË (m + 3) ˜¯ a12 – a22 + b12 – b22 = 2(a1 – a2)x + 2(b1 – b2)y

Ê 2m - 2 ˆ
2
Ê 2m - 2 ˆ
2 1 2
=Á + 3Á (a1 - a2 ) x + (b1 - b2 ) y = (a1 - a22 + b12 - b22 )
Ë (m - 3) ¯˜ Ë (m - 3) ˜¯ 2
4 4 1 2
fi = Thus, the value of c is (a2 - a12 + b22 - b12 ) .
2 2
(m + 3) (m - 3) 2
56. Ans. (a)
fi (m + 3) 2 = (m - 3) 2
fi m=0 57.
Y
Hence the required equation of the line is y = 2.
53. Triangles OPA and OQC are similar.
OP OA 9/4 3 y=x
Thus, = = = x+y=2
OQ OC 3 4
A(1, 1)
Y y=k P(h, k)
B B 0
X¢ X
O
P
A(9/4, 0)
X¢ X Y¢
C O
(–3, 0) 4x + 2y = 9
Q The co-ordinates of A, B and C are (1, 1), (k, k) and
D (2 – k, k).
2x + y + 6 = 0 1 1 1
Y¢ 1
Given k k 1 = 4h 2
2
54. Equation of the altitude BD is x = 3. 2-k k 1
Y
1 0 0
1 Ê C2 Æ C2 - C1 ˆ
fi k 0 1 — k = 4h 2 Á
2 Ë C3 Æ C3 - C1 ˜¯
B(3, 4) 2-k 2k — 2 k — 1

1 0 1- k
E fi = 4h 2
G 2 2k - 2 k - 1
0 1- k
O D A(4, 0) X fi = 4h 2
k -1 k -1
fi (k – 1)2 = 4h2
4-0
Slope of AB is = -4 fi (k – 1) = ±2h
3-4
Hence, the locus of P is 2x – y + 1 = 0.
1
and Slope of OE is . 58. Ans. a0 = 1
4

CG_01a.indd 72 2/6/2017 3:59:03 PM


Straight Lines 1.73

59. On solving, we get,


Y
x = pq and y = –pq
Thus, the locus of the orthocentre is x + y = 0 which is
2x + y = 0 y=x a straight line.
Bisector 62. Y

X¢ O X

y = – 3x + 1 L=0
P(–2, –2) R Q(1, –2) y+2=0
X
O 60°
Y¢ (3, –2)
PR OP 2 2
In DOPQ, = =
RQ OQ 5
But statement II is false. Equation of a line passing through (3, –2) is
60. (a) The point of intersection L1: x + 3y – 5 = 0 and (y + 2) = m(x – 3)
It is given that,
L3: 5x + 2y – 12 = 0 is P(2, 1).
m+ 3
If L1, L2 and L3 are concurrent, the point P must tan (60°) =
satisfy L2 = 0 1- m 3
Thus, 6 – k – 1 = 0 m+ 3
k = 5. fi = 3
1- m 3
(b) Let L1 and L2 are parallel.
m+ 3
1 3 fi =± 3
Then = fi k = -9 1- m 3
3 -k
fi (m + 3) = ± 3(1 - m 3)
Again, let L2 and L3 are parallel.
fi m = 0, m = 3
3 -k 6
Then = fik=- . Hence, the equation is
5 2 5
6 ( y + 2) = 3( x - 3)
(c) For k π 5, - 9, - they will form a triangle.
5 fi 3 x - y - (2 + 3 3) = 0
6 fi y - 3 x + (2 + 3 3) = 0
(d) For k = 5, - 9, - they will not form a triangle.
5 63.
64. bx + ay + c = 0
Y y=x
ax + by + c = 0

C(pq, ( + p)(1 + q))


X¢ O X

X
A(–p, 0) B(–q, 0) O

Intersection point of y = 0 with first line is A(–p, 0) Y¢


Intersection point of y = 0 with second line is B(–q, 0) Clearly, the point of intersection of the lines
Intersection point of the two lines is ax + by + c = 0 and bx + ay + c = 0 lie on the line y = x.
C(pq, (p + 1)(q + 1)) Let the point be (r, r).
Altitude from C to AB is x = pq Given (r — 1) 2 + (r — 1) 2 < 2 2
Altitude from B to AC is
q fi 2(r — 1) 2 < 2 2
y=- ( x + p)
1+ q fi 2 (r — 1) < 2 2

CG_01a.indd 73 2/6/2017 3:59:05 PM


1.74 Coordinate Geometry Booster

fi |(r — 1)| < 2 Given 2 £ d1(P) + d2(P) £ 4


fi –2 < (r – 1) < 2
a —b a+b
fi –1 < r < 3 fi 2£ + £4
Thus, (–1, –1) lies on the opposite side of origin for 2 2
both lines. fi 2 2 £ |a — b | + a + b £ 4 2
Therefore, –a – b + c < 0
fi a+b–c>0 fi 2 2 £ 2a £ 4 2 , when a > b, for P(a, b)
64. fi 2 £a £ 2 2 .
Y
2 2
Area of the region = (2 2) - ( 2)
y=x =8–2
=6

P(a, b)

X
O x= 2 x=2 2

CG_01a.indd 74 2/6/2017 3:59:05 PM


Pair of Straight Lines 2.1

C H A P T E R

2 Pair of Straight Lines

CONCEPT BOOSTER Y

2x
m
y=
1. INTRODUCTION y=
m 1x

A pair of lines is the locus of a point moving on two lines. X¢ X


O
Let the two lines be
a1x + b1y + c1 = 0
and a2x + b2y + c2 = 0

The joint equation of the given equations is
(a1x + b1y + c1) (a2x + b2y + c2) = 0 A homogeneous equation of 2nd degree in x and y is
and conversely if joint equation of two lines be ax2 + 2hxy + by2 = 0
(a1x + b1y + c1) (a2x + b2y + c2) = 0,
Ê 2h ˆ a
then their separate equations will be fi y 2 + Á ˜ x + x2 = 0
Ë b¯ b
a1x + b1y + c1 = 0 and a2x + b2y + c2 = 0
Ê 2h ˆ 4h 2 4a
Notes: -Á ˜ x ± x -
Ë b¯ b2 b
1. In order to find the joint equation of two lines make fi y=
RHS of equation of the straight lines to zero and then 2
multiply the equations. Ê Ê hˆ h2 a ˆ
2. In order to find the separate equations of two lines, = Á -Á ˜ ± - ˜x
when their joint equation is given. Simply factorize Ë Ë b¯ b2 b ¯
the joint equation and reduces into two linear factors. Ê Ê hˆ h 2 - ab ˆ
= Á -Á ˜ ± ˜x
Ë Ë b¯ b2 ¯
2. HOMOGENEOUS EQUATION Ê —h ± h 2 - ab ˆ
In an equation, if the degree of each term throughout the =Á ˜¯ x
Ë b
equation is same, it is known as homogeneous equation.
Thus ax2 + 2hxy + by2 = 0 is a homogeneous equation of Ê - h + h 2 - ab ˆ Ê - h - h 2 - ab ˆ
2nd degree and ax3 + 2hx2y + by3 = 0 is a homogeneous equa- =Á ˜ x or Á ˜¯ x
Ë b ¯ Ë b
tion of degree 3.
fi (y – m1x)(y – m2x) = 0,
3. PAIR OF STRAIGHT LINES THROUGH THE ORIGIN 2h a
where m1 + m2 = - and m1 ◊ m2 =
Theorem b b
Any homogeneous equation of 2nd degree in x and y repre- fi y – m1x = 0 or y – m2x = 0
sents two straight lines which are passing through the origin. both of which are pass through the origin.

CG_02.indd 1 2/6/2017 3:59:29 PM


2.2 Coordinate Geometry Booster

4. ANGLE BETWEEN THE LINES REPRESENTED BY Let the given equation represents the straight lines
ax2 + 2hxy + by2 = 0 y – m1x = 0 and y – m2x = 0
2h a
Let the lines are represented by Then m1 + m2 = - , m1m2 = .
b b
ax2 + 2hxy + by2 = 0
are y – m1x = 0 and y – m2x = 0 The equation of the bisectors of the angles between the given
2h straight lines are
where m1 + m2 = - A
b y - m1 x y - m2 x
=
2a 1+ m12 1 + m22
and m1 ◊ m2 =
b y - m1 x y - m2 x
and =-
Let q be the angle between them. B
1+ m12 1 + m22
m1 - m2 The joint equation of the bisectors is
Then tan q =
1 + m1m2 O
Ê y - m1 x y - m2 x ˆ Ê y - m1 x y - m2 x ˆ
fi - + =0
2 Á 2 ˜Á ˜
(m1 + m2 ) - 4m1m2 Ë 1 + m1 1 + m22 ¯ Ë 1 + m12 1 + m22 ¯
=
1 + m1m2 2 2
Ê y - m1 x ˆ Ê y - m2 x ˆ
fi =
4h 2 a Á 2 ˜ Á 2˜
-4 Ë 1 + m1 ¯ Ë 1 + m2 ¯
2
b b
= fi (y – m1x)2(1 + m22) = (y – m2x)2 (1 + m22)
a
1+ fi (1 + m22) (y2 – 2m1xy + m12x2)
b
= (1 + m12) (y2 – 2m2xy + m22x2)
2 h 2 - ab fi (m12 – m22) (x2 – y2)
=
a+b + 2(m1m2 – 1) (m1 – m2)xy = 0
Ê 2 h 2 - ab ˆ fi (m1 + m2)(x2 – y2) + 2(m1m2 – 1)xy = 0
Thus q = tan -1 Á ˜. Ê 2h ˆ 2 2 Êa ˆ
Ë a+b ¯ fi ÁË - ˜¯ ( x - y ) + 2 ÁË - 1˜¯ xy = 0
b b
(i) Condition of parallelism
In this case, q = 0 Ê 2h ˆ 2 2 Êa ˆ
fi ÁË - ˜¯ ( x - y ) = - 2 ÁË - 1˜¯ xy
fi h2 = ab b b
(ii) Condition of perpendicularity x 2 - y 2 xy
p fi =
In this case, q = a-b h
2
fi a+b=0 which is the required equation of bisectors of the lines rep-
(iii) Condition of coincidency resented by
In this case also, q = 0 ax2 + 2hxy + by2 = 0
fi h2 = ab.
6. GENERAL EQUATION OF 2ND DEGREE IS
5. BISECTORS OF ANGLES BETWEEN THE LINES ax2 + 2hxy + by2 + 2gx + 2fy + c = 0
REPRESENTED BY ax2 + 2hxy + by2 = 0 Theorem
Y L2 The general equation of 2nd degree is
A2 A1 ax2 + 2hxy + by2 + 2gx + 2fy + c = 0
L1 which represents a pair of straight lines if
abc + 2fgh – af 2 – bg2 – ch2 = 0.
q1 Proof: The general equation of 2nd degree is
q2
X¢ X ax2 + 2hxy + by2 + 2gx + 2fy + c = 0 …(i)
fi by2 + 2(hx + f)y + (ax2 + 2gx + c) = 0

- 2(hx + f ) ± 4(hx + f ) 2 - 4b(ax 2 + 2gx + c)


fi y=
2b
Y¢ - (hx + f ) ± (h 2 - ab) x 2 + 2( gh - af ) x + ( g 2 - ac)
=
b

CG_02.indd 2 2/6/2017 3:59:30 PM


Pair of Straight Lines 2.3

Equation (i) represents two straight lines if LHS of Eq (i) can


be resolved into two linear factors, therefore the expression 2 h 2 - ab
under the square root should be a perfect square. q = tan -1
(a + b)
Hence, 4(gh – af )2 – 4(h2 – ab) (g2 – ac) = 0
fi g2h2 + a2f 2 – 2afgh – h2g2 + abg2 + ach2 – a2bc = 0 Result 6
fi a(af 2 + bg2 + ch2 – 2fgh – abc) = 0 The lines represented by
fi abc + 2fgh – af – bg – ch2 = 0 ax2 + 2hxy + by2 + 2gx + 2fy + c = 0
which is the required condition. will be coincident if
h2 – ab = 0, g2 – ac = 0 and f 2 – bc = 0
Result 1
Result 7
The lines represented by
The pair of bisectors of the lines represented by
ax2 + 2hxy + by2 + 2gx + 2fy + c = 0
ax2 + 2hxy + by2 + 2gx + 2fy + c = 0 …(i)
are parallel if
is
D = abc + 2fgh – af 2 – bg2 – ch2 = 0
and h2 – ab = 0 ( x - a) 2 - ( y - b) 2 ( x - a)( x - b)
= ,
Result 2 ( a - b) h
The lines represented by where (a, b) be the point of intersection of the pair of
ax 2 + 2hxy + by2 + 2gx + 2fy + c = 0 straight lines represented by Eq. (i).
are perpendicular if Result 8
D = abc + 2fgh – af 2 – bg2 – ch2 = 0 The combined equation of the straight lines joining the ori-
and a + b = 0. gin to the points of intersection of a second degree curve
Result 3 ax2 + 2hxy + by2 + 2gx + 2fy + c = 0
The point of intersection of the lines represented by and a straight line lx + my + n = 0 is
ax2 + 2hxy + by2 + 2gx + 2fy + c = 0 Ê lx + my ˆ
ax2 + 2hxy + by2 + 2gx + 2gx Á
is obtained by Ë - n ˜¯
df df 2
= 0 and =0 Ê lx + my ˆ Ê lx + my ˆ
dx dy + 2fy Á + c ˜ =0
Ë - n ˜¯ ÁË
-n ¯
i.e 2ax + 2hy + 2g = 0
fi ax + hy + g = 0 A
and 2hx + 2by + 2f = 0
fi hx + by + f = 0
Result 4
The point of intersection of the lines represented by B
ax2 + 2hxy + by2 + 2gx + 2fy + c = 0
Ê f 2 - bc g 2 - ac ˆ O
is Á 2 , ˜
ÁË h - ab h 2 - ab ˜¯ After simplification, the above equation will be reduced to
Ax2 + 2Hxy + By2 = 0 and it will represent two straight lines,
Ê bg - hf af - gh ˆ
or Á 2 , which are passing through the origin.
Ë h - ab h 2 - ab ˜¯ Let q be the angle between them, then
Result 5
2 H 2 - AB
The angle between the lines represented by q = tan -1
ax2 + 2hxy + by2 + 2gx + 2fy + c = 0 is A+ B

E XERCISES

LEVEL I 2. Find the joint equation of the lines x = y and x + y = 1.


3. Find the separate equations of the lines represented by
(Problems Based on Fundamentals) x2 – 3xy + 2y2 = 0.
4. Find the separate equations of the lines represented by
ABC OF PAIR OF STRAIGHT LINES
x2 – y2 + 2y – 1 = 0.
1. Find the joint equation of the lines x – y + 2 = 0 and 5. Find the straight lines which are represented by
2x + y + 4 = 0. x2 + 5xy + 6y2 = 0.

CG_02.indd 3 2/6/2017 3:59:30 PM


2.4 Coordinate Geometry Booster

6. Find the area of the triangle formed by the lines 25. Find the point of intersection of the lines represented
y2 – 5xy + 6y2 = 0 and y = 6. by x2 – 5xy + 4y2 + x + 2y – 2 = 0.
7. Find the orthocentre of the triangle formed by the lines 26. Find the point of intersection of the straight lines repre-
xy = 0 and x + y = 2. sented by the equation
8. Find the circumcentre of the triangle formed by the 3x2 – 2xy – 8y2 – 4x + 18y – 7 = 0
lines xy – x – y + 1 = 0 and x + y = 4. 27. Find the point of intersection of the straight lines is
9. Find the angle between the lines represented by given by the equation
2013x2 + 2014xy – 2013y2 = 0.
3y2 – 8xy – 3x2 – 29x + 3y – 18 = 0
10. Find the angles between the lines represented by
2x 2 - 4 3 xy + 6y 2 = 0. 28. If the equation
11. Find the angle between the lines represented by 12x2 + 7xy – py2 – 18x + qy + 6 = 0
x2 + 4xy + y2 = 0. represents two perpendicular lines, then find the values
of p and q.
BISECTORS OF THE LINES 29. Prove that the four lines given by
12. Find the equation of the bisectors of the angle between 3x2 + 8xy – 3y2 = 0
the lines represented by 3x2 + 5xy + 4y2 = 0. and 3x2 + 8xy – 3y2 + 2x – 4y – 1 = 0
13. If y = mx is one of the bisectors of the lines x2 + 4xy + form a square.
y2 = 0, find the value of m. 30. Find the angle between the lines represented by
14. Prove that the lines 9x2 + 14xy + 16y2 = 0 are equally
3x2 – 2xy – 8y2 – 4x + 18y – 7 = 0
inclined to the lines 3x2 + 2xy + 4y2 = 0.
31. Find the angle between the lines represented by
15. Prove that the bisectors of the angle between the lines
ax2 + acxy + cy2 = 0 and x2 – 5xy + 4y2 + x + 2y – 2 = 0
1ˆ 2 1ˆ 2 32. If the angle between the lines represented by
Ê Ê
ÁË 2013 + ˜¯ x + xy + ÁË 2013 + ˜¯ y = 0 2x2 + 5xy + 3y2 + 7y + 4 = 0
c a
–1
are always same. is tan (m), then find m.
16. If pairs of straight lines x2 – 2pxy – y2 = 0 and x2 – 2qxy 33. Prove that the lines represented by
– y2 = 0 be such that each pair bisects the angle between x2 – 8xy + 16y2 + 2x – 8y + 1 = 0
the other pair, prove that pq = –1. is coincident
34. Find the equation of bisectors of the lines represented
GENERAL EQUATION OF A STRAIGHT LINE by x2 – 5xy + 2y2 + x + 2y – 2 = 0.
17. For what values of m, does the equation mx2 – 5xy – 6y2 35. Find the angle between the lines joining the origin to
+ 14x + 5y + 4 = 0 represents two straight lines? the points of intersection of the straight line y = 3x + 2
18. For what value of l, does the equation 12x2 – 10xy + with the curve x2 + 2xy + 3y2 + 4x + 8y – 11 = 0.
2y2 + 11x – 5y + l = 0 represents a pair of straight lines? 36. Find the equation of the straight lines joining the origin
19. If lx2 + 10xy + 3y2 – 15x – 21y + 18 = 0 represents a to the points of intersection of the line y = mx + c and
pair of straight lines, find the value of l. the curve x2 + y2 = b2.
20. Find the separate equation of lines represented by 37. Find the equation of the straight lines joining the origin
x2 – 5xy + 4y2 + x + 2y – 2 = 0 to the points of intersection of the line lx + my = 1 and
21. Prove that the equation the curve y2 = 4bx.
x 2 - 2 3 xy + 3y 2 - 3x + 3 3 y - 4 = 0 38. Find the equation of bisectors of the angles represented
by the lines 12x2 – 10xy + 2y2 + 9x + 2y – 12 = 0.
represents two parallel straight lines and also find the
39. Prove that the lines joining the origin to the points
distance between them.
of intersection of the line 3x – 2y = 1 and the curve
22. Find the equation of the straight lines passing through 3x2 + 5xy – 3y2 + 2x + 3y = 0 are perpendicular to each
the point (1, 1) and parallel to the lines represented by other.
the equation
40. If the equation ax2 – 2hxy + by2 + 2gx + 2fy + c = 0.
x2 – 5xy + 4y2 + x + 2y – 2 = 0.
23. If the equation represents two straight lines, prove that the product of
the perpendiculars drawn from the origin to the lines is
3x2 + 5xy – py2 + 2x + 3y = 0 c
represents two perpendicular straight lines,then find .
the value of p + 2010. (a - b) 2 + 4h 2
24. Prove that the equation 41. Find the value of m, if the lines joining the origin and
16x2 + 24xy + 9y2 + 40x + 30y – 75 = 0 the point of intersection of y = mx + 1 and x2 + 3y2 = 1
represents two parallel straight lines. perpendicular to one another.

CG_02.indd 4 2/6/2017 3:59:31 PM


Pair of Straight Lines 2.5

LEVEL II LEVEL III


(Mixed Problems) (Problems for JEE Advanced)
1. The orthocentre of the triangle formed by the lines 1. Find the internal angles of the triangle formed by
xy = 0 and 2x + 3y = 6 is the pair of the straight lines x2 – 4xy + y2 = 0 and the
(a) (1, 1) (b) (0, 0) (c) (1, 2) (d) (2, 1) straight line x + y + 4 6 = 0 . Give the co-ordinates of
2. The orthocentre of the triangle formed by the lines the vertices of the triangle so formed and also the area
xy – x – y + 1 = 0 and 3x + 4y = 12 is of the triangle. [Roorkee, 1983]
(a) (1,1) (b) (0, 0) (c) (2,2) (d) (3,3) 2. From a point A(1, 1), straight lines AL and AM are
3. The image of the pair of lines represented by ax2 + 2hxy drawn at right angles to the pair of straight lines
+ by2 = 0 by the line mirror y = 0 is 3x2 + 7xy + 2y2 = 0. Find the equations of the pair of
(a) ax2 + 2hxy + by2 = 0 (b) ax2 + 2hxy – by2 = 0 straight lines AL and AM. Also find the area of the quad-
rilateral ALOM, where O is the origin of co-ordinates.
(c) ax2 – 2hxy + by2 = 0 (d) ax2 – 2hxy – by2 = 0
[Roorkee, 1984]
4. The image of the pair of lines represented by ax2 + 2hxy
3. The base of a triangle passes through a fixed point (f, g)
+ by2 = 0 by the line mirror x = 0 is
and its sides are, respectively, bisected at right angles
(a) ax2 + 2hxy + by2 = 0 (b) ax2 + 2hxy – by2 = 0 by the lines y2 – 8xy – 9x2 = 0. Determine the locus of
(c) ax2 – 2hxy + by2 = 0 (d) ax2 – 2hxy – by2 = 0 its vertex. [Roorkee, 1985]
5. The point of intersection of the two lines given by 4. Show that the four straight lines given by 12x2 + 7xy +
2x2 – 5xy + 2y2 + 3x + 3y + 1 = 0 is 12y2 = 0 and 12x2 + 7xy – 12y2 – x + 7y –1 = 0 lie along
Ê 1 1ˆ Ê 1 1ˆ the sides of a square. [Roorkee, 1986]
(a) Á - , ˜ (b) Á , - ˜
Ë 2 2¯ Ë 2 2¯ 5. The distance between the two parallel lines given by
Ê 1 1ˆ the equation x 2 + 2 3 xy + 3y 2 - 3x - 3 3 y - 4 = 0 is
Ê 1 1ˆ
(c) Á - , ˜ (d) Á , - ˜ (a) 3/2 units (b) 2 units
Ë 3 3¯ Ë 3 3¯
2 2
(c) 5/2 units (d) 3 units
6. If the equation 12x + 7xy – py – 18x + qy + 6 = 0 rep- [Roorkee, 1989]
resents two perpendicular lines, the value of p + q + 7 is 6. Pair of straight lines perpendicular to each other are
(a) 15 (b) 20 (c) 25 (d) 30 represented by
7. The angle between the lines x2 + 4xy + y2 = 0 is (a) 2x2 = 2y(x + y) (b) x2 + y2 + 3 = 0
(a) p/2 (b) p/6 (c) p/3 (d) p/4 (c) 2x2 = y(2x + y) (d) x2 = 2(x – y)
8. The angle between the lines given by x2 + 2013xy – y2 [Roorkee, 1990]
= 0 is 7. If one of the lines represented by ax2 + 2hxy + by2 =
(a) p/2 (b) p/6 (c) p/3 (d) p/4 0 bisects the angle between positive directions of the
9. The area of the triangle formed by the lines y2 – 9xy + axes, then a, b, h satisfy the relation
18x2 = 0 and y = 9 is (a) a + b = 2|h| (b) (a + b)2 = 4h2
(a) 27/4 (b) 31/4 (c) 18/5 (d) 27 (c) a – b = 2|h| (d) (a – b)2 = 4h2
10. If x2 – 3xy + ly2 + 3x – 5y + 2 = 0 represents a pair of [Roorkee, 1992]
straight lines, the value of l is 8. Show that all the chords of the curve 3x2 – y2 – 2x +
(a) 1 (b) 4 (c) 3 (d) 2 4y = 0 which subtend a right angle at the origin are
11. If the equation 2x2 – 3xy – ay2 + x + by – 1 = 0 repre- concurrent. Does this result also hold for the curve
sents two perpendicular lines, then a + b + 2 is 3x2 + 3y2 – 2x + 4y = 0? [Roorkee Main, 1992]
(a) 2 (b) 5 (c) 6 (d) 3 9. A pair of straight lines drawn through the origin form
with the line 2x + 3y = 6 an isosceles triangle right
12. If xy + x + y + 1 = 0 and x + ay – 3 = 0 are concurrent,
angled at the origin. Find the equation of the pair of
the value of a + 10 is
straight lines and the area of the triangle correct to two
(a) 5 (b) 7 (c) 6 (d) 4
places of decimals. [Roorkee Main,1993]
13. The circumcentre of the triangle formed by the lines xy 10. The distance between the two parallel lines given by
+ 2x + 2y + 4 = 0 and x + y + 2 = 0 is the equation is
(a) (0, 0) (b) (–1, –1) (c) (–1, –2) (d) (–2, –2) (a) 3/2 units (b) 2 units
14. The distance between the pair of parallel lines (c) 5/2 units (d) 3 units
9x2 – 24xy + 16y2 – 12x + 16y – 12 = 0 is [Roorkee, 1994]
(a) 5 (b) 8 (c) 8/5 (d) 5/8 11. Mixed term xy is to be removed from the general equa-
15. If one of the lines given by 6x2 – xy + 4cy2 = 0 is tion of the second degree ax2 + 2hxy + by2 + 2gx + 2fy
3x + 4y = 0, then the value of c is + c = 0. One should rotate the axes through an angle
(a) 1 (b) –1 (c) –3 (d) 5 given by tan (2q) equal to

CG_02.indd 5 2/6/2017 3:59:31 PM


2.6 Coordinate Geometry Booster

a-b 2h 2h 10. If one of the lines denoted by the line pair ax2 + 2hxy +
a+b
(a) (b) (c) (d) by2 = 0 bisects the angle between the co-ordinate axes,
2h a+b 2h a-b
prove that (a + b)2 = 4h2
[Roorkee, 1996]
12. What is the conditions for the second degree polyno-
mials in x and y to represent a pair of straight lines
Integer Type Questions
ax2 + 2hxy + by2 + 2gx + 2fy + c = 0? [Roorkee, 1997] 1. If ax – 9x2y – xy2 + 4y2 = 0 represents three straight
3

13. A variable line L passing through the point B(2, 5) in- lines such that two of them are perpendicular, then find
tersects the lines 2x2 – 5xy + 2y2 = 0 at P and Q. Find the sum of the values of a.
the locus of the point R on L such that distances BP, BR 2. If the curve
and BQ are in harmonic progression. (tan2 q + cos2 q)x2 – 2xy tan q + (sin2 q)y2 = 0
[Roorkee Main, 1998] represents two straight lines, which makes angles q1
and q2 with the x-axis, find (tan q1 – tan q2).
3. If pairs of straight lines x2 – 2pxy – y2 = 0 and x2 – 2qxy
LEVEL IV – y2 = 0 be such that each pair bisects the angle between
the other pair, find the value of (pq + 4).
(Tougher Problems for JEE
4. If two of the lines represented by
Advanced) ax4 – 10x3y + 12x2y2 – 20xy3 + ay4 = 0
1. Find the area of the triangle formed by the pair of lines bisects the angle between the other two, find the value
ax2 + 2hxy + by2 = 0 and the lines lx + my = 1. of a.
2. If (a, b) be the centroid of the triangle whose sides are 5. If the equation 4x2 + 10xy + my2 + 5x + 10y = 0
represents a pair of straight lines, find m.
the lines ax2 + 2hxy + by2 = 0 and lx + my = 1, prove that
a b 2 1
= = ◊ Comprehensive Link Passages
bl - hm am - hl 3 bl 2 - 2hlm + am 2
3. A triangle has the lines ax2 + 2hxy + by2 = 0 for two of Passage I
its sides and the point (l, m) for its orthocentre. Prove Let the lines represented by 2x2 – 5xy + 2y2 = 0 be the two
that the third side has the equation sides of a parallelogram and the line 5x + 2y = 1 be one of its
(a + b)(lx + my) = am2 – 2h/m + bl2 diagonals. Then
4. If the equation 1. the one of the sides of the parallelogram passing
ax2 + 2hxy + by2 + 2gx + 2fy + c = 0 through the origin is
represents a pair of straight lines, prove that the dis- (a) x – 2y = 0 (b) x + 2y = 0
g 2 - ac (c) x – 3y = 0 (d) x + 3y = 0
tance between the lines = 2 . 2. the equation of the other diagonal is
a ( a + b)
(a) x – 2y = 0 (b) 11x – 10y = 0
5. Prove that two of the four lines represented by the joint (c) 10x – 11y = 0 (d) 10x + 11y = 0
equation ax4 + bx3y + cx2y2 + dxy3 + ay4 = 0 will bi- 3. the area of the parallelogram is
sect the angles between the other two if c + 6a = 0 and
1 1
b + d = 0. (a) sq. u. (b) sq. u.
6. Find the new equation of the curve 36 54
4(x – 2y + 1)2 + 9(2x + y + 2)2 = 25, 1 1
(c) sq. u. (d) sq. u.
if the lines 2x + y + 2 = 0 and x – 2y + 1 = 0 are taken 72 96
as the new x and y axes, respectively. Passage II
7. The lines x2 – 3xy + 2y2 = 0 are shifted parallel to A general equation of 2nd degree is
themselves so that their point of intersection comes to 8x2 + 8xy + 2y2 + 26x + 13y + 15 = 0 …(i)
(1, 1). Find the combined equation of the lines in the Then
new position. 1. the given Eq. (i) represents two straight lines, which
8. If the straight lines represented by are
ax2 + 2hxy + by2 + 2gx + 2fy + c = 0 (a) 2x + y + 5 = 0, 4x + 2y + 3 = 0
are equidistant from the origin, prove that (b) x + 2y + 5 = 0, 2x + 4y + 3 = 0
f 4 – g4 = c(bf 2 – ag2) (c) 2x – y + 5 = 0, 4x – 2y + 3 = 0
9. If the lines joining the origin and the point of intersec- (d) x – 2y + 5 = 0, 2x – 4y + 3 = 0
tion of the curves 2. the given Eq. (i) represents
a1x2 + 2h1xy + b1y2 + 2g1x = 0 (a) two perpendicular lines
and a2x2 + 2h2xy + b2y2 + 2g2x = 0 (b) two parallel lines
are mutually perpendicular, prove that (c) two intersecting lines
g1(a1 + b1) = g2(a2 + b2). (d) none of these

CG_02.indd 6 2/6/2017 3:59:31 PM


Pair of Straight Lines 2.7

3. the distance between the lines is 4. Match the following columns:


7 5 2 5 Column I Column II
(a) (b) (c) (d)
2 5 2 7 5 7 7 2 (A) The image of the pair (P) 24x2 + 10xy +
of lines represented by y2 = 0
Matrix Match 2x2 + 3xy + 3y2 = 0 with
respect to the line mirror
(For JEE-Advanced Examination Only)
y = 0 is
1. Match the following columns: (B) The image of the pair (Q) 2x2 – 3xy + 3y2
of lines represented by =0
Column I Column II x2 + 5xy + 6y2 = 0 with
(A) The separate equations of (P) x – 2 = 0, respect to the line mirror
x2 – y2 + 2y – 1 = 0 are y–2=0 x = 0 is
(B) The separate equations (Q) x – y = 0, (C) The image of the pair (R) x2 – 5xy + 6y2
of x2 – y2 + 4x + 4 = 0 are x+y+1=0 of lines represented by =0
(C) The separate equations (R) x – y + 1 = 0 x2 + 10xy + 24y2 = 0 with
of x2 – x – y2 – y = 0 are x+y–1=0 respect to the line y = x is
(D) The separate equations (S) x + y + 2 = 0
of xy – 2x – 2y + 4 = 0 are x–y+2=0
Questions asked in Previous Years’
2. Match the following columns: JEE-Advanced Examinations
Column I Column II 1. Show that all chords of the curve 3x2 – y2 – 2x + 4y = 0,
(A) The joint equation of the (P) xy – 3y – 2x + which subtend a right angle at the origin, passes through
lines x = y and x = –y is 6=0 a fixed point. Find the co-ordinates of the fixed point.
(B) The joint equation of the (Q) x2 + y2 = 0 [IIT-JEE, 1991]
lines x = 2 and y = 3 is No questions asked in between 1992 to 1993.
(C) The joint equation of the 2 2
(R) x – y + 4y – 4 2. The equations to a pair of of opposite sides of a paral-
lines x = y and x = 1 – y =0 lelogram are x2 – 5x + 6 = 0 and y2 – 6y + 5 = 0. The
is equations to its diagonals are
(D) The joint equation of the (S) x2 – y2 – x + y (a) x + 4y = 12 and y = 4x – 7
lines x + y – 2 = 0 and =0 (b) 4x + y = 13 and 4y = x – 7
x – y + 2 = 0 is (c) 4x + y = 13 and y = 4x – 7
3. Match the following columns: (d) y – 4x = 13 and y + 4x = 7 [IIT-JEE, 1994]

Column I Column II No questions asked in between 1995 to 1998.


3. Let PQR be a right angled isosceles triangle, right an-
(A) The orthocentre of the trian- (P) (11/2, 9/2)
gled at P(2, 1). If the equation of the line QR is 2x + y = 3,
gle formed by the lines xy = 0
the equations representing the pair of lines PQ and PR
and x + y = 2013 is
is
(B) The orthocentre of the trian- (Q) (–2, –2)
(a) 3x2 –3y2 + 8xy + 20x + 10y + 25 = 0
gle formed by the lines x2 – y2
+ 4y – 4 = 0 and y + 2014 = (b) 3x2 –3y2 + 8xy – 20x – 10y + 25 = 0
0 is (c) 3x2 –3y2 + 8xy + 10x + 15y + 20 = 0
(C) The circumcentre of the (R) (0, 2) (d) 3x2 –3y2 – 8xy – 10x – 15y – 20 = 0
triangle formed by the lines [IIT-JEE, 1999]
xy – y = 0 and x + y = 10 is No questions asked in 2000.
(D) The circumcentre of the (S) (0, 0) 4. Let 2x2 + y2 – 3xy = 0 be the equations of a pair of
triangle formed by the lines tangents drawn from the origin O to a circle of radius 3
xy – x – y + 1 = 0 with centre in the first quadrant. If A is one of the points
and x + y + 4 = 0 is of contact, find the length of OA. [IIT-JEE, 2001]
No questions asked in between 2002 to 2003.

CG_02.indd 7 2/6/2017 3:59:31 PM


2.8 Coordinate Geometry Booster

5. The area of the triangle formed by the angle bisectors (b) two straight lines and a circle, when a = b, and c is
of the pair of lines x2 – y2 + 2y – 1 = 0 and the line of sign opposite to a.
x + y = 3 (in sq units) is (c) two straight lines and a hyperbola, when a and b
(a) 1 (b) 2 (c) 3 (d) 4 are of the same sign and c is of sign opposite to a.
[IIT-JEE, 2004] (d) a circle and an ellipse, when a and b are of the
same sign and c is of sign opposite to that of a.
No questions asked in between 2005 to 2007.
[IIT-JEE, 2008]
6. Let a and b be non-zero real numbers. Then the equa-
tion (ax2 + by2 + c)(x2 – 5xy + 6y2) = 0 represents No questions asked in between 2009 to 2014.
(a) four straight lines, when c = 0 and a, b are of the
same signs.

A NSWERS

LEVEL I LEVEL II
1. 2x2 – y2 – xy + 8x – 2y + 8 = 0 1. (b) 2. (a) 3. (a) 4. (c) 5. (d)
2. x2 – y2 – x + y = 0 6. (b) 7. (a) 8. (a) 9. (a) 10. (d)
3. x – y = 0 and x – 2y = 0 11. (b) 12. (c) 13. (b) 14. (c) 15. (c)
4. x + y – 1 = 0 and x – y + 1 = 0
5. (x + 2y) = 0 and (x + 3y) = 0
6. 3 LEVEL III
7. (0, 0)
8. (2, 2) 1. 16 3 sq. units.; triangle is equilateral
9. 90° 2. 7/10
10. 0° 5. (c)
11. 60° 6. (a)
12. 5x2 – 2xy – 5y2 = 0 7. (d)
13. ( 3 - 2) and ( - 3 - 2) 8. 2nd part: Not Concurrent
15. ac(x2 – y2) – 2(a – c)xy = 0 9. 5(x2 – y2) – 24xy = 0; 2. 77
16. pq = –1 10. (c)
17. m=6 11. (d)
18. x – y – 1 = 0, x – 4y + 2 = 0 12. abc + 2fgh – af 2 – bg2 – ch2 = 0
19. 5/2 13. 17x – 10y = 0
20. x – y = 0 and x – 4y + 3 = 0
21. 2013
23. (2, 1) LEVEL IV
24. (1, 1)
26. q = 1, –23/2
h 2 - ab
28. q = tan–1(2) 1.
Ê 3ˆ (am 2 - 2hlm + bl 2 )
29. q = tan -1 Á ˜
Ë 5¯ 6. 4x2 = 9y2 = 5
1
32. m= 7. x2 – 3xy + 2y2 + x – y = 0
5
34. 5x2 –2xy – 5y2 – 18x + 24y + 11 = 0 INTEGER TYPE QUESTIONS
35. 7x2 – 2xy – y2 = 0
1. 1 2. 2 3. 3 4. 3
36. (c2 – b2m2)x2 + 2b2mxy + (c2 – b2)y2 = 0
5. 4
37. 4blx2 + 4bmxy – y2 = 0
( x - 14) 2 - ( y - 69/2) 2 ( x - 14)( y - 69/2) COMPREHENSIVE LINK PASSAGE
38. =
Ê 69 ˆ —5
ÁË 14 — ˜ Passage I: 1. (a) 2. (b) 3. (c)
2¯ Passage II: 1. (a) 2. (b) 3. (a)
41. ± 3

CG_02.indd 8 2/6/2017 3:59:32 PM


Pair of Straight Lines 2.9

MATRIX MATCH QUESTIONS ASKED IN IIT-JEE EXAMINATIONS


1. (A) Æ (R); (B) Æ (S); 1. (1, –2)
(C) Æ (Q); (D) Æ (P) 2. (c)
2. (A) Æ (Q); (B) Æ (P); (C) Æ (S); (D) Æ (R) 3. (b)
3. (A) Æ (S); (B) Æ (R); (C) Æ (P); (D) Æ (Q); 4. (9 + 3 10) units.
4. (A) Æ (Q); (B) Æ (R); (C) Æ (P); 5. (b)
6. (b)

H INTS A ND S OLUTIONS

LEVEL I As we know that the point of intersection of two per-


pendicular sides of a triangle is known as the othrocen-
tre of the triangle.
1. Hence, the joint equation of the given lines is
Hence, the orthocentre is (0, 0).
(x – y + 2)(2x + y + 4) = 0.
8. The given equation xy – x – y + 1 = 0 can be reduced to
fi 2x2 – y2 – xy + 8x – 2y + 8 = 0
(x – 1)(y – 1) = 0
2. Hence, the joint equation of the lines is
fi x – 1 = 0 and y – 1 = 0
(x – y)(x + y – 1) = 0.
fi x = 1 and y = 1.
fi x2 – y2 – x + y = 0
Here, ABC be a right-angled triangle and right angled
3. The given equation is
at A.
x2 – 3xy + 2y2 = 0
As we know that, the mid-point of the hypotenuse is
fi x2 – xy – 2xy + 2y2 = 0
the circumcentre of the right-angle triangle.
fi x(x – y) – 2y(x – y) = 0
Now, the mid-point of BC is (2, 2).
fi (x – y)(x – 2y) = 0
Hence, the circumcentre is (2, 2).
Hence, the separate equations of the given lines are
9. Here, a = 2013, b = –2013 and h = –1007. Since,
x – y = 0 and x – 2y = 0.
a + b = 2013 – 2013 = 0, so the angle between the lines
4. The given equation is
is 90°.
x2 – y2 + 2y – 1 = 0
fi x2 – (y2 – 2y + 1) = 0 10. Here, a = 2, b = 6 and h = - 2 3 .
fi x2 – (y2 – 1)2 = 0 Now, h2 – ab = 12 – 12 = 0.
fi (x + y – 1) (x – y + 1) = 0. Hence, the angle between them is 0°.
Hence, the separate equations of the given lines are 11. Here, a = 1, b = 1 and h = 2.
x + y – 1 = 0 and x – y + 1 = 0. Let q be the angle between them, then
5. The given equation x2 + 5xy + 6y2 = 0 can be written as Ê 2 h 2 - ab ˆ
(x + 2y)(x + 3y) = 0 tan (q ) = Á ˜
Ë a+b ¯
fi (x + 2y) = 0 and (x + 3y) = 0
Hence, the required lines are Ê 2 4 - 1ˆ p
(x + 2y) = 0 and (x + 3y) = 0 =Á ˜ = 3 = tan 3
Ë 1 + 1 ¯
6. The given equation of the lines p
y2 – 5xy + 6y2 = 0 fi q=
3
can be reduced to y = 2x and y = 3x.
Then, A = (2, 6) and B = (3, 6) Hence, the angle between them is 60°.
Hence, the area of the triangle OAB is 12. The given equation is 3x2 – 5xy + 4y2 = 0
0 0 Here, a = 3, b = 4 and h = –5/2
Hence, the equation of the bisector is
13 6 1
= = (18 - 12) = 3 sq. u.
22 6 2 x 2 - y 2 xy
=
0 0 a-b h
7. The equation xy = 0 gives to x = 0 and y = 0. x2 - y 2 xy
fi =
Clearly, OAB is a right-angled triangle and right angle 3-4 — (5/2)
at O. 2
fi - ( x 2 - y 2 ) = - xy
5
fi 5x2 – 2xy – 5y2 = 0

CG_02.indd 9 2/6/2017 3:59:32 PM


2.10 Coordinate Geometry Booster

13. Since y = mx is one of the bisector of the given lines 16. From the problem, it is clear that, the bisectors of the
x2 + 4xy + y2 = 0, then y = mx will satisfy the given angles between the lines given by
lines. x2 – 2pxy – y2 = 0 …(i)
Therefore, x2 + 4mx2 + m2x2 = 0 2 2
is x – 2qxy – y = 0 …(ii)
fi 1 + 4m + m2 = 0 The equation of the bisectors of Eq. (i) is
fi m2 + 4m + 1 = 0 x 2 - y 2 xy
=
fi (m + 2) 2 = ( 3) 2 1 - ( -1) - q
fi qx2 + 2xy + qy2 = 0 …(iii)
fi m=± 3-2
Here, Eqs (ii) and (iii) are identical.
Hence, the values of m are
Thus, comparing the co-efficients, we get
( 3 - 2) and ( - 3 - 2) 1 - 2p -1
= =
14. The given pair of lines are -q -2 q
9x2 + 14xy + 16y2 = 0 …(i) fi pq = –1
and 3x2 + 2xy + 4y2 = 0 …(ii) Hence, the result.
The two pairs will be equally inclined if the two pairs 17. The given equation is
of the straight lines have the same bisectors. mx2 – 5xy – 6y2 + 14x + 5y + 4 = 0 …(i)
The equation of the bisectors of the angle between the Here, a = m, h = –5/2, b = –6, g = 7, f = 5/2 and c = 4.
lines represented by 9x2 + 14xy + 16y2 = 0 is The Eq. (i) represents a pair of straight lines if
x 2 - y 2 xy abc – 2fgh – af 2 – bg2 – ch2 = 0
=
9 - 16 7 175 25
fi - 24m - - m + 294 - 25 = 0
fi x + xy – y = 0
2 2
2 4
The equation of the bisector of the angle between the Ê 25 ˆ 175
fi - Á 24m + m˜ = - 269
lines represented by 3x2 + 2xy + 4y2 = 0 is Ë 4 ¯ 2
x 2 - y 2 xy 121 363
= fi - m=-
3-4 1 4 2
fi x + xy – y = 0
2 2
fi m=6
Hence, the pair of lines of Eq. (i) are equally inclined to Hence, the value of m is 6.
the pair of lines of Eq. (ii). 18. Given equation is
15. The given equations of the lines are
x2 – 5xy + 4y2 + x + 2y – 2 = 0
ax2 + acxy + cy2 = 0 …(i) (x – y) (x – 4y) + x + 2y – 2 = 0 …(i)
and The joint equation can be written as
Ê 1ˆ 2 Ê 1ˆ 2
ÁË 2013 + ˜¯ x + xy + Á 2013 + ˜ y = 0 …(ii) (x – y + c1) (x – 4y + c2) = 0
c Ë a¯ (x – y) (x – 4y) + (c1 + c2)x – (4c1 + c2)y + c1c2 = 0
The equation of bisectors of Eq. (i) is …(ii)
x2 - y 2 xy Comparing Eqs (i) and (ii), we get
= (c1 + c2) = 1, (4c1 + c2) = –2, c1c2 = –2
a-c 3 - 2/2
Solving, we get
fi ac(x – y ) = 2(a – c)xy
2 2
c1 = –1, c2 = 2
fi ac(x2 – y2) – 2(a – c)xy = 0
Hence, the separate equations of the lines are
Also, the equation of bisectors of Eq. (ii) is x – y – 1 = 0 and x – 4y + 2 = 0.
x2 - y 2 xy 19. The given equation is
=
Ê 1ˆ Ê 1ˆ 1/2
ÁË 2013 + ˜¯ - ÁË 2013 + ˜ x 2 - 2 3 xy + 3y 2 - 3x + 3 3 y - 4 = 0 …(i)
c a¯
Here, a = 1, b = 3 and h = - 3
ac ( x 2 - y 2 )
fi = 2xy Now, h2 – ab = 3 – 3 = 0
(a - c) Thus, the given equation represents two parallel
fi ac(x2 – y2) = 2(a – c)xy straight lines.
fi ac(x2 – y2) – 2(a – c)xy = 0 Equation (i) can be reduces to
Hence, the result. ( x - 3 y ) 2 - 3( x - 3 y ) - 4 = 0

CG_02.indd 10 2/6/2017 3:59:32 PM


Pair of Straight Lines 2.11

Also, the given represents a pair of straight lines if


fi m2 – 3m – 4 = 0, where m = ( x - 3 y )
a h g
fi (m – 4)(m + 1) = 0
fi m = 4 and m = –1 h b f =0
g f c
fi x - 3y - 4 = 0
7
and x - 3y + 1 = 0 12 -9
2
1+ 4 5 7 q
Hence, the required distance = = . fi -12 =0
1+ 3 2 2 2
20. The given equation is
x2 – 5xy + 4y2 + x + 2y – 2 = 0 q
-9 6
fi (x – y) (x – 4y) + x + 2y – 2 = 0 …(i) 2
Thus, the Eq. (i) represents two parallel straight lines, fi q = 1, –23/2
whose joint equation is 27. We have,
(x – y + l)(x – 4y – m) = 0 3x2 + 8xy – 3y2 = 0
which is passing through (1, 1). fi 3x2 + 9xy – xy – 3y2 = 0
Therefore, l = 0 and m = 3 fi 3x(x + 3y) – y(x + 3y) = 0
Hence, the parallel lines are fi (x + 3y)(3x – y) = 0
x – y = 0 and x – 4y + 3 = 0. fi (x + 3y) = 0, (3x – y) = 0
21. The given equation is Also, 3x2 + 8xy – 3y2 + 2x – 4y – 1 = 0
3x2 + 5xy – py2 + 2x + 3y = 0 …(i) fi (x + 3y)(3x – y) + 2x – 4y – 1 = 0 …(i)
Since, the Eq. (i) represents two perpendicular straight The joint equation of the above equation can be written
lines, as
so, a + b = 0 (x + 3y + c1)(3x – y + c2)
co-efficients of x2 + co-efficients of y2 = 0 fi (x + 3y)(3x – y) + (3c1 + c2)x – (c1 – 3c2)y + c1c2 = 0
fi 3–p=0 …(ii)
fi p=3 Comparing Eqs (i) and (ii), we get
Hence, the value of p + 2010 = 2013. (3c1 + c2) = 2, (c1 – 3c2) = 4, c1c2 = –1
22. Here, a = 16, h = 12 and b = 9
Solving, we get
Now, h2 – ab = 144 – 144 = 0
c1 = 1, c2 = –1
Hence, it represents two parallel straight lines.
23. The given equation is Thus, the separate equations are
x + 3y + 1 = 0 and 3x – y – 1 = 0
x2 – 5xy + 4y2 + x + 2y – 2 = 0.
Therefore, the four lines are
df
Now, = 0 fi 2x - 5y + 1 = 0 x + 3y = 0, 3x – y = 0
dx x + 3y + 1 = 0 and 3x – y – 1 = 0
df
and = 0 fi 8y - 5x + 2 = 0 A(0, 0) B(3/10, –1/10)
dy
x + 3y = 0
Solving, we get, x = 2 and y = 1
3x – y – 1 = 0

Hence. the required point of intersection is (2, 1).


3x – y = 0

24. The given equation is


3x2 – 2xy – 8y2 – 4x + 18y – 7 = 0
Here, a = 3, h = –1, b = –8, g = –2, f = 9 and c = –7.
Therefore, h2 – ab = 1 + 24 = 25, bg – hf = 16 + 9 = 25, x + 3y + 1 = 0
af – gh = 27 – 2 = 25
D(–1/10, –3/10) C(1/5, –2/5)
Hence, the point of intersection is
Ê bg - hf af - gh ˆ Clearly, –BAD = 90°
=Á 2 ,
Ë h - ab h 2 - ab ˜¯ 1
and AB = AD =
Ê 25 25 ˆ 10
= Á , ˜ = (1, 1)
Ë 25 25 ¯ Thus, ABCD is a square
25. Do yourself
26. Clearly 12 – p = 0
fi p = 12

CG_02.indd 11 2/6/2017 3:59:33 PM


2.12 Coordinate Geometry Booster

28. We have, df
and = 0 fi —5x + 4 y + 2 = 0
Ê 2 h 2 - ab ˆ dy
tan q = Á ˜
Ë a+b ¯ Solving, we get
x = 2 and y = 1.
2 1 + 24 2¥5
= = =2 Therefore, the point of intersection is (2, 1).
3-8 -5
Hence, the equation of bisector is
q = tan–1(2)
( x - a ) 2 - ( y - b ) 2 ( x - a )( x - b )
29. The given equation is =
( a - b) h
x2 – 5xy + 4y2 + x + 2y – 2 = 0
( x - 2) 2 - ( y - 1) 2 ( x - 2)( y - 1)
Here, a = 1, h = –5/2, b = 4, g = 1/2, f = 1, c = –2. fi =
(1 - 2) ( -5/2)
Let q be the angle between them.
fi 5x2 – 2xy – 5y2 – 18x + 24y + 11 = 0
Ê 2 h 2 - ab ˆ
Then, tan q = Á ˜ which is a homogeneous equation of 2nd degree.
Ë a+b ¯ 35. The equation of the straight line is
Ê 25 ˆ y=3x+2
Á 2 4 - 4˜ 3 y - 3x
=Á ˜= fi =1 …(i)
Ë 1+ 4 ¯ 5 2
Ê 3ˆ The equation of the given curve is
fi q = tan -1 Á ˜
Ë 5¯ x2 + 2xy + 3y2 + 4x + 8y – 11 = 0 …(ii)
Ê 3ˆ
Hence, the angle between them is tan -1 Á ˜ . Making the Eq. (ii) homogeneous in the second degree
Ë 5¯ in x and y by means of Eq. (i), we get
32. We have Ê y - 3x ˆ
x 2 + 2xy + 3y 2 + 4 Á x
Ê 2 h 2 - ab ˆ Ë 2 ˜¯
tan q = Á ˜ 2
Ë a+b ¯ Ê y - 3x ˆ Ê y - 3x ˆ
+ 8Á y - 11Á =0
Ë 2 ˜¯ Ë 2 ˜¯
Ê 25 ˆ 2◊ 1 On simplification, we get
fi =Á
2 - 6 ˜ = 2 =1
4 7x2 – 2xy – y2 = 0
Á ˜ 5 5
Ë 2+3 ¯
which is the required equation of the line joining the
-1 Ê 1 ˆ origin to the points of intersection of lines (i) and (ii).
fi q = tan Á ˜
Ë 5¯ Here, a = 7, h = –1 and b = –1
—1 -1 Ê 1 ˆ Let q be the angle between them.
fi tan (m) = tan Á ˜
Ë 5¯
1 Ê 2 h 2 - ab ˆ
fi m= Then, tan q = Á ˜
5 Ë a+b ¯
33. The given equation is
Ê 2 1+ 7ˆ 2 8 2 2
x2 – 8xy + 16y2 + 2x – 8y + 1 = 0 …(i) =Á = =
Ë 7 — 1 ˜¯ 6 3
Here, a = 1, h = –4, b = 16, g = 1, f = –4 and c = 1.
Now, h2 – ab = 16 – 16 = 0, Ê 2 2ˆ
fi q = tan -1 Á
2
g – ac = 1 – 1 = 0 and Ë 3 ˜¯
f 2 – bc = 16 – 16 = 0 36. The equation of the straight line is
Hence, the Eq. (i) represents two coincident straight y = mx + c
lines. Ê y - mx ˆ
fi ÁË ˜ =1 …(i)
34. The given equation is c ¯
x2 – 5xy + 2y2 + x + 2y – 2 = 0 …(i) The equation of the given curve is
df x2 + y2 = b2 …(ii)
Now, = 0 fi 2x - 5y + 1 = 0
dx

CG_02.indd 12 2/6/2017 3:59:33 PM


Pair of Straight Lines 2.13

Making the Eq. (ii) homogeneous of the second degree 40. Let ax2 + 2hxy + by2 + 2gx + 2fy + c
in x and y by means of Eq. (i), we get = (lx + my + n)(l1x + m1y + n1)
2
Ê y - mx ˆ = ll1x2 + mm1y2 + (lm1 + l1m)xy
x2 + y 2 = b2 Á
Ë c ˜¯ + (ln1 + l1n)x + (mn1 + nm1)y + nn1
fi c2(x2 + y2) = b2(y – mx)2 Comparing the co-efficients, we get
fi (c2 – b2m2)x2 + 2b2mxy + (c2 – b2)y2 = 0 ll1 = a, mm1 = b, nn1 = c, (lm1 + ml1) = 2h,
which is the required equation of the straight lines join- (ln1 + nl1) = 2g, (mn1 + nm1) = 2f
ing the origin to the point of intersection of the lines (i) Let p1 and p 2 be the length of perpendicular from the
and (ii). origin.
37. The given line is n n1
lx + my = 1 Thus, p1 p2 = ¥
2 2
l +m l1 + m12
2

Ê lx + my ˆ
fi ÁË ˜ =1 nn1
1 ¯ =
(l + m 2 )(l12 + m12 )
2
The given curve is
nn1
y2 = 4bx …(ii) =
Making the Eq. (ii) homogeneous of the second degree l 2l12 + m 2 m12 + l 2 m12 + l12 m 2
in x and y by means of Eq. (i), we get nn1
=
y2 = 4bx(lx + my) l 2l12 +m 2
m12 + (lm1 + l1m) 2 - 2ll1mm1
fi 2
4blx + 4bmxy – y = 0 2
c
=
38. The given curve is a + b + 4h 2 - 2ab
2 2
2 2
12x – 10xy + 2y + 9x + 2y – 12 = 0 c
=
df (a - b) 2 + 4h 2
Now, = 0 gives 24x – 10y + 9 = 0
dx
41. Given curve is x2 + 3y2 = 1 …(i)
df and given line is y = mx + 1 …(ii)
and = 0 gives 5x – 2y – 1 = 0
dy Making the Eq. (i) homogeneous of the second degree
Solving, we get in x and y by means of Eq. (ii), we get
69 x2 + 3y2 = (y – mx)2
x = 14, y = fi x2 + 3y2 = y2 + m2x2 – 2mxy
2
Hence, the equation of the bisectors is fi (1 – m2)x2 + 2mxy + 2y2 = 0
Clearly, (1 – m2) + 2 = 0
( x - a ) 2 - ( y - b) 2 ( x - a )( y - b)
= fi m2 = 3
( a - b) h
fi m=± 3
( x - 14) 2 - ( y - 69/2) 2 ( x - 14)( y - 69/2)
=
Ê 69 ˆ —5 Hence, the value of m is ± 3
ÁË14 — ˜¯
2
39. The given curve is
3x2 + 5xy – 3y2 + 2x + 3y = 0 …(i)
LEVEL III
The equation of the straight line is 1. We have x2 – 4xy + y2 = 0
3x – 2y = 1 …(ii) fi y2 – 4x.y + x2 = 0
Making the Eq. (i) homogeneous of the second degree 4x ± 16x 2 - 4x 2
in x and y by means of Eq. (ii), we get fi y=
2
3x2 + 5xy – 3y2 + (2x + 3y) (3x – 2y) = 0
4x ± 2 3 x
fi 9x2 + 10xy – 9y2 = 0 = = (2 ± 2 3) x
2
clearly, they are mutually perpendicular to each other. = (2 + 2 3) x, y = (2 - 2 3) x

CG_02.indd 13 2/6/2017 3:59:33 PM


2.14 Coordinate Geometry Booster

Y
1 1
1 3
-
5 5
1
B = 0 0
2
2 1
-
5 5
1 1
X
O 1 Ê 3 + 1 + 2 + 1ˆ 7
= Á ˜¯ = sq. u.
2Ë 5 10
3. Given lines are
y2 – 8xy – 9x2 = 0
Thus fi y2 – 9xy + xy – 9x2 = 0
OB : y = (2 + 2 3) x and OA : y = (2 - 2 3) x fi y(y – 9x) + x(y – 9x) = 0
and AB : x + y + 4 6 = 0 fi (y + x)(y – 9x) = 0
p fi (y + x) = 0, (y – 9x) = 0
Clearly, the angle between OA and OB is .
3 A(a, b)
Solving, we get

=0
Ê 4 2 8 2(1 - 3) ˆ =0
O = (0, 0), A = Á , y

9x
˜ x+
Ë2— 3 (2 - 3) ¯

y–
L M
Ê 4 2 8 2(1 + 3) ˆ
and B = Á - , ˜ H
Ë 2+ 3 (2 + 3) ¯
Hence, the area of the
P(f, g) B C
1
DOAB = [64(1 + 3) + 64(1 - 3)] Equations of AB and AC are
2
1 x + 9y = (a + 9b) and x – y = (a + b)
= (64 + 64) Here, B and C are the images of A w.r.t. the lines y = 9x
2
and y = –x
= 64 sq. u.
Ê 9b - 40a 40b + 9a ˆ
So, C = ( - b , - a ) and B = Á , ˜
p Ë 41 41 ¯
Also, the angle between OA and AB is .
3 Now, P, B and C are collinear, so,
p f g 1
and also the angle between OB and AB is .
3 9b - 40a 40b + 9a
Hence, the triangle is equilateral. 1=0
41 41
2. Given lines are 3x2 + 7xy + 2y2 = 0
-b -a 1
fi 3x2 + 6xy + xy + 2y2 = 0
fi 3x(x + 2y) + (x + 2y) = 0 f g 1
fi (x + 2y)(3x + y) = 0 fi (9b - 40a ) (40b + 9a ) 41 = 0
fi (x + 2y) = 0, (3x + y) = 0 b a —1
Thus, the equation of AL and AM are f g 1
2x – y + l = 0, 3x – y + m = 0 fi (9b - 40a ) (40b + 9a ) 41 = 0
which is passing through A(1, 1) ( f + b) (g + a ) 0
l = –1, m = –2
(9b — 40a ) (40b + 9a ) f g
Hence, the equations of AL and AM are fi - 41 =0
( f + b) (g +a ) ( f + b) (g +a )
2x – y – 1 = 0, 3x – y – 2 = 0
Solving, we get fi (g + a)(9b – 40a) – (f + b)(40b + 9a)
–41(fa – gb) = 0
Ê 1 3ˆ Ê 2 1ˆ
L = Á - , ˜ and M = Á , - ˜ fi 40(a2 + b2) + 10(4g + 5f)a + 10(4f – 5g)b = 0
Ë 5 5¯ Ë 5 5¯
fi 4(a2 + b2) + (4g + 5f)a + (4f – 5g)b = 0
Hence, the area of the quadrilateral ALMO Hence, the locus of A(a, b) is
4(x2 + y2) + (4g + 5f)x + (4f – 5g)y = 0

CG_02.indd 14 2/6/2017 3:59:34 PM


Pair of Straight Lines 2.15

4. The equations of the four sides are Since the chord subtends a right angle at the origin, so
(3x + 4y) = 0, 4x – 3y = 0, (3 – 2l) + (4m – 1) = 0
(3x + 4y – 1) = 0 and 4x – 3y + 1 = 0 fi 2 – 2l + 4m = 0
fi 1 – l + 2m = 0
4x + 3y = 0 C(3/25, 4/25)
O fi l – 2m = 0
which shows that all such chords pass through a fixed

3x + 4y – 1 = 0
point (1, –2) and hence are concurrent.
3x + 4y = 0

If we repeat the process for the curve 3x2 + 3y2 – 2x +


4y = 0, we get
(3 – 2l) + (3 + 4m) = 0
fi (6 – 2l + 4m) = 0
x – 3y + 1 = 0
fi (3 – l + 2m) = 0
A(–4/25, 3/25) B(–1/25, 7/25) fi l – 2m = 3
which shows that such chords lx + my = 1 are not con-
Clearly, the angle between OA and AB is 90°.
current.
1
and OA = AB = 9.
5 Y
So, ABCD is a square.
5. We have
x 2 + 2 3 xy + 3y 2 - 3x - 3 3 y - 4 = 0 B

fi ( x + 3 y ) 2 - 3( x + 3 y ) - 4 = 0
fi a 2 - 3a - 4 = 0, a = ( x + 3 y ) A
X
fi a2 – 3a – 4 = 0 O
fi (a – 4)(a + 1) = 0
fi a = –1, 4
Suppose the lines OA and OB are
Thus, the parallel lines are
y = m1x, y = m2x respectively.
x + 3 y - 4 = 0, x + 3 y + 1 = 0 Here, OA = OB and –AOB = 90°
Hence, the distance between them is So, –OAB = 45° = –OBA
1 - ( - 4) 5 Thus,
= =
1+ 3 2 2
m1 +
tan (45°) = 3
6. We have 2x2 = 2y(x + y) 2
2x2 – 2xy – 2y2 = 0 1- m
3 1
Now, a + b 3m1 + 2
Co-eff of x2 + Co-eff of y2 fi =1
3 - 2m1
=2–2
fi (3m1 + 2) = (3 – 2m1)
=0
fi 5m1 = 1
Hence, the result. 1
7. Given lines are ax2 + 2hxy + by2 = 0 fi m1 = .
5
The line bisects the axes is y = x. 2
So, ax2 + 2hx ◊ x + bx2 = 0 m2 +
Also, tan (135°) = 3
fi ax2 + 2hx2 + bx2 = 0 2
fi a + 3h + b = 0 1 - m2
3
fi a + b = –2h 3m2 + 2
fi (a + b)2 = (–2h)2 fi = -1
3 - 2m2
fi (a + b)2 = 4h2
fi 3m2 + 2 = –3, 2m2
8. Let lx + my =1 be any chord of the curve 3x2 – y2 – 2x +
fi m2 = –5
4y = 0.
Hence, the equations of the pair of straight lines
Let the equation of the pair of straight lines and passing
Ê 1 ˆ
through the point of intersections of the chord and the ( y + 5x) Á y - x˜ = 0
Ë 5 ¯
curve is given by
(3x2 – y2) – (2x + 4y)(lx + my) = 0 fi (y + 5x)(5y – x) = 0
(3 – 2l)x2 + (4m – 1)y2 – (2m + 4l) xy = 0 fi 5(x2 – y2) – 24xy = 0

CG_02.indd 15 2/6/2017 3:59:35 PM


2.16 Coordinate Geometry Booster

Solving, we get Let P = (a, b), Q = (c, d) and R = (h, k)


the co-ordinates of A and B as and RB = r, PB = r1 and QB = r2
Ê 30 6 ˆ Ê 6 30 ˆ So, P = (2 + r1 cos q, 5 + r1 cos q)
A = Á , ˜; B = Á- , ˜
Ë 13 13 ¯ Ë 13 13 ¯ and Q = (2 + r2 cos q, 5 + r2 sin q)
Thus, the area of the triangle OAB is and R = (2 + r cos q, 5 + r sin q)
0 0 Since P lies on y = 2x,
30 6
1
1 13 13 so, = 2 cos q - sin q
= r1
2 6 30
- Also, Q lies on x = 2y
13 13
0 0 1 cos q - 2 sin q
So, =
1 Ê 900 + 36 ˆ r2 8
= Á ˜
2 Ë 169 ¯ 2 1 1
It is given that = +
1 936 469 r r1 r2
= ¥ = = 2.76 sq. u.
2 169 169 2 17 cos q - 10 sin q
10. See Q. 15 fi =
r 8
11. Let the axes be rotated through an angle q
so that x Æ x cos q – y sin q 2 17 Ê h - 2 ˆ 10 Ê k - 5 ˆ
fi = Á ˜- Á ˜
and y Æ x sin q + y sin q r 8Ë r ¯ 8Ë r ¯
Given curve is fi 16 = 17h – 34 – 10k + 50
ax2 + 2hxy + by2 + 2gx + 2fy + c = 0 fi 17h – 10k = 0
fi a(x cos q – y sin q)2 + a(x sin q + y cos q)2 Hence, the locus of R is 17x – 10y = 0.
+ 2h(x cos q – y sin q)(x sin q + y cos q)
+ 2g(x cos q – y sin q) + 2f(x sin2 q + y cos2 q)

fi (b – a) sin 2q + 2hcos 2q = 0
=0
LEVEL IV
Remove the term xy, we get 1. Let three lines be
(–a sin 2q + b sin 2q) + 2h(cos2q – sin2q) = 0 y = m1x, y = m2x, lx + my = 1
fi (b – a) sin 2q + 2h cos 2q = 0
2h 2h a
fi tan (2q ) = where m1 + m2 = - , m1m2 =
a-b b b
12. A general equation of 2nd degree represents a pair of Thus area of a triangle OAB, where
straight lines if
abc + 2fgh – af2 – bg2 – ch2 = 0 Ê 1 m1 ˆ
O = (0, 0), A = Á ,
13. We have, Ë l + mm1 l + mm1 ˜¯
2x2 – 5xy + 2y2 = 0
fi 2x2 – 4xy – xy + 2y2 = 0 Ê 1 m2 ˆ
and B=Á ,
fi 2x(x – 2y) – y(x – 2y) = 0 Ë l + mm2 l + mm2 ˜¯
fi (x – 2y)(2x – y) = 0 is
fi (x – 2y) = 0, (2x – y) = 0 0 0
fi x = 2y and y = 2x 1 m1
Y 1 l + mm1 l + mm1
=
2 1 m2
l + mm2 l + mm2
B(2, 5)
0 0
P(a, b)
R(h, k) 1Ê (m2 - m1 ) ˆ
=
Q(c, d) 2 Ë (l + mm1 )(l + mm2 ) ˜¯
Á

X 1Ê (m1 + m2 ) 2 - 4m1 m2 ˆ
O = Á 2 ˜
2 ÁË l + (m1 + m2 )lm + (m1 m2 )m 2 ˜¯

CG_02.indd 16 2/6/2017 3:59:35 PM


Pair of Straight Lines 2.17

Y
Ê 4h 2 4a ˆ
Á - ˜
1 b2 b
= Á ˜
2 Á 2 2h a 2˜
ÁË l - lm + m ˜
¯
b b B

1Ê 2 h 2 - ab ˆ y = m2x D
= Á 2 ˜ H(l, m)
2 Ë bl - 2hlm + am 2 ¯ F
A
Ê h 2 - ab ˆ E
=Á 2 ˜ sq. u y = m1x
X
Ë bl - 2hlm + am 2 ¯ O
2. Let three lines be
y = m1x, y = m2x, and lx + my = 1
Solving OA and AF, we get
2h a
where m1 + m2 = - , m1m2 = Ê l + mm2 m1 (l + mm2 ) ˆ
b b A=Á ,
Thus, the points O, A and B are Ë 1 + m1m2 1 + m1m2 ˜¯
Ê 1 m1 ˆ Solving OB and BE, we get
O = (0, 0), A = Á ,
Ë l + mm1 l + mm1 ˜¯ Ê l + mm1 m2 (l + mm1 ) ˆ
B=Á ,
Ê 1 m2 ˆ Ë 1 + m1m2 1 + m1m 2 ˜¯
and B=Á ,
Ë l + mm2 l + mm2 ˜¯ Equation of AB is lx + my + l3= 0 which is passing
through A.
1Ê 1 1 ˆ
Therefore, a = Á +
3 Ë l + mm1 l + mm2 ˜¯ Ê l + mm2 ˆ
So, l Á
Ê m (l + mm2 ) ˆ
+ mÁ 1 + l3 = 0
Ë 1 + m m ˜ Ë 1 + m1m2 ˜¯
1 Ê m1 m2 ˆ 1 2¯
and b= Á +
3 Ë l + mm1 l + mm2 ˜¯ Ê l 2 + lm (m2 + m1 ) + m 2 m1m2 ˆ
fi + l3 = 0
1 Ê 2l + (m1 + m2 ) m ˆ ËÁ 1+ m m 1 2 ¯˜
fi a= Á
3 Ë (l + mm1 )(l + mm2 ) ˜¯ Ê l 2 + lm (m2 + m1 ) + m 2 m1m2 ˆ
fi l3 = - Á
1 Ê (m + m2 )l + 2(m1m2 )m ˆ Ë 1+ m m 1 2 ¯˜
and b= Á 1
3 Ë (l + mm1 )(l + mm2 ) ¯˜ Ê 2 Ê 2h ˆ a 2ˆ
Á l - ÁË b ˜¯ lm + b m ˜
2Ê bl - hm ˆ fi l3 = - Á ˜
fi a= a
3 ËÁ bl 2 - 2hlm + am 2 ¯˜ Á 1+ ˜
Ë b ¯
2Ê am - hl ˆ 2
Ê bl - 2hlm + am ˆ 2
and b=
3 ËÁ bl 2 - 2hlm + am 2 ¯˜ fi l3 = - Á
Ë a+b ¯˜
Thus,
Hence, the equation of the third side AB is
a b 2Ê 1 ˆ
= = Ê bl 2 - 2hlm + am 2 ˆ
(bl - hm) (am - hl ) 3 ËÁ bl 2 - 2hlm + am 2 ¯˜ lx + my - Á =0
Ë a+b ¯˜
Hence, the result.
3. Let the lines be y = m1x and y = m2x fi (lx + my) (a + b) = (bl 2 – 2hlm + am2)
2h a Hence, the result.
where m1 + m2 = - , m1m2 =
b b 4. Given equation is
Equation of OA is y = m1x ax2 + 2hxy + by2 + 2gx + 2fy + c = 0 …(i)
BE is perpendicular to OA Equation (i) represents a pair of parallel straight lines
So, BE: x + m1y + l1 = 0 which is passing through lx + my + n1 = 0 and lx + my + n\2 = 0
H(l, m), we get
Thus, (lx + my + n1) (lx + my + n2)
l + mm1 + l1 = 0
= ax2 + 2hxy + by2 + 2gx + 2fy + c
fi l1 = –(l + mm1)
Comparing, the co-efficients, we get
Thus, BE: x + m1y – (l + mm1) = 0
a = l2, b = m2, n1n2 = c
Similarly, AF: x + m2y – (l + mm2) = 0

CG_02.indd 17 2/6/2017 3:59:36 PM


2.18 Coordinate Geometry Booster

and h = lm, l(n1 + n2) = 2g, m(n1 + n2) = 2f fi (n1l2 – n2l1)(n1l2 + n2l1)
Hence, the distance between the lines = (n2m1 – n1m2)(n2m1 + n1m2)
n1 - n2 fi (n1l2 – n2l1)(2g) = (n2m1 – n1m2)(2f)
= fi (n1l2 – n2l1)2(g2) = (n2m1 – n1m2)2(f 2)
l 2 + m2
fi [(n1l2 + n2l1)2 – 4n1n2l1l2(g2)]
(n1 + n2 ) 2 - 4n1n2 = (n2m1 + n1m2)2 – 4m1m2n1n2(f2)
=
l 2 + m2 fi (4g – 4ac)g2 = (4f2 – 4bc)f2
2

fi (g2 – ac)g2 = (f2 – bc)f2


g 2 - ac fi (g4 – g2ac) = (f4 – f2bc)
=2
a ( a + b) fi f4 – g4 = c(f2b – g2a)
6. We have Hence, the result.
4(x – 2y + 1)2 + 9(2x + y + 2)2 = 25 9. Given curves are
fi 4X 2 + 9Y 2 = 25, a1x2 + 2h1xy + b1y2 + 2g1x = 0 …(i)
2 2
where X = x – 2y + 1, and a2x + 2h2xy + b2y + 2g2x = 0 …(ii)
Y = 2x + y +2 Multiplying Eq. (i) by g1 and Eq. (ii) by g2, we get
which represents an ellipse. a1 g1 x 2 + 2g1h1 xy + b1 g1 y 2 + 2g12 x = 0 …(iii)
2 2
7. Given lines are x2 – 3xy + 2y2 = 0 and a2 g 2 x + 2h2 g 2 xy + b2 g 2 y + 2g 22 x
= 0 …(iv)
fi (x – y)(x – 2y) = 0 Subtracting Eq. (iii) and Eq. (iv), we get
Let the equation of the lines be (a1g1 – a2g2)x2 + 2(g1h1 – g2h2)xy
(x – y + l)(x – 2y + m) = 0 + (b1g1 – b2g2)y2 = 0
which is passing through (1, 1), i.e. These lines will be right angles to each other if
(1 – 1 + l)(1 – 2 + m) = 0 (a1g1 – a2g2) + (b1g1 – b2g2) = 0
fi (1 – 1 + l) = 0, (1 – 2 + m) = 0 fi (a1g1 + b1g1) + (a2g2 + b2g2)
fi l = 0, m = –1 fi g1(a1 + b1) = g2(a2 + b2)
Hence, the combine equation is 10. Given lines are ax2 + 2hxy + by2 = 0.
(x – y)(x – 2y + 1) = 0 The line bisects the axes is y = x.
fi (x – y)(x – 2y) + (x – y) = 0 So, ax2 + 2hx◊x + bx2 = 0
fi x2 – 3xy + 2y2 + x – y = 0 fi ax2 + 2hx2 + bx2 = 0
8. Given equation is fi a + 2h + b = 0
ax2 + 2hxy + by2 + 2gx + 2fy + c = 0 …(i) fi a + b = –2h
Let the lines represented by Eq. (i) be fi (a + b)2 = (–2h)2
l1x + m1y + n1 = 0 and l1x + m2y + n2 = 0 fi (a + b)2 = 4h2

We have
Integer Type Questions
ax2 + 2hxy + by2 + 2gx + 2fy + c
1. As we know that if ax3 + bx2y + cxy2 + dy3 = 0 repre-
= (l1x + m1y + n1)(l2x + m2y + n2)
sents two perpendicular lines then
Comparing the co-efficients, we get a2 + ac + bd + d2 = 0
l1l2 = a, m1m2 = b, n1n2 = c a2 – a – 36 + 16 = 0
l1n2 + l2n1 = 2g, l1m2 + l2m1 = 2h a2 – a – 20 = 0
(a – 5)(a + 4) = 0
and m1n2 + m2n1 = 2f,
a = 5, –4
Since they are equidistant from the origin, so Hence, the sum of the values of a is 1.
n1 n2 2. Let m1 = tan q1 and m2 = tan q2
=
2 2
l1 + m1 l2 + m22
2
2h 2 tan q
Now, m1 + m2 = - =
fi n12 (l22 + m22 ) = n22 (l12 + m12 ) b sin 2q
fi n12l22 - n22l12 = n22 m12 - n12 m22 tan 2q + cos 2q
and m1m2 =
fi n12l22 - n22l12 = n22 m12 - n12 m22 sin 2q

CG_02.indd 18 2/6/2017 3:59:36 PM


Pair of Straight Lines 2.19

Thus, 25m
tan q1 – tan q2 fi 125 - - 100 = 0
4
4 tan 2q 4(tan 2q + sin 2q ) 25m
= - fi 25 - =0
sin 4q sin 2q 4
2 25m
= tan 2q - sin 2q (tan 2q + cos 2q ) fi 25 =
sin 2q 4
2 sin q fi m=4
= sec 2q - (tan 2q + cos 2q )
sin 2q
2
= 1 — cos 2q Previous Years’ JEE-Advanced Examinations
sin q
1. Let the equation of the chord be lx + my = 1
2
= ¥ sin q The joint equation of the curve
sin q 3x2 – y2 – 2x + 4y = 0 and the chord
= 2. be lx + my = 1 to the origin is
3. From the problem, it is clear that, the bisectors of the 3x2 – y2 – (2x – 4y)(lx + my) = 0
angles between the lines given by (3 – 2l)x2 + (4l – 2m)xy + (4m – 1)y2 = 0
x2 – 2pxy – y2 = 0 …(i)
which subtends a right angle at the origin, if co-
and x2 – 2qxy – y2 = 0 …(ii) efficient of x2 + x co-efficient of y2 = 0
The equation of the bisectors of line (i) is
fi 3 – 2l + 4m – 1 = 0
x 2 - y 2 xy fi l – 2m = 1
=
1 - ( -1) - q Now, lx + my = 1
fi –qx2 + 2xy + qy2 = 0 …(iii) fi lx + my = l – 2m
Here, Eqs (ii) and (iii) are identical. fi l(x – 1) + m(y + 2) = 0
Thus, comparing the co-efficients, we get m
1 - 2p -1 fi ( x — 1) + ( y + 2) = 0
= = l
-q -2 q
fi (x – 1) + l(y + 2) = 0
fi pq = –1 fi (x – 1) = 0, (y + 2) = 0
Hence, the value of pq + 4 is 3. fi x = –1, y = –2
4. As we know that Hence, the chord passes through the fixed point is
ax4 + bx3y + cx2y2 + dx3 + ay4 = 0 (1, –2).
bisects the angle between the other two No questions asked in between 1992 to 1993.
then 6a + c + b + d = 0 2. The sides of the parallelogram are
fi 6a – 10 + 12 – 20 = 0 x2 – 5x + 6 = 0 and y2 – 6y + 5 = 0
fi 6a = 18 (x – 2)(x – 3) = 0 and (y – 1)(y – 5) = 0
fi a = 3. x = 2, x = 3 and y = 1, y = 5
5. We have
Thus, the angular points are
a h g A = (2, 1), B = (,2 5), C = (3, 5), D = (3, 1)
h b f =0 Equation of the diagonal AC is
g f c Ê 5 - 1ˆ
( y - 2) = Á ( x - 1)
4 5 5/2 Ë 3 - 2 ˜¯
fi 5 m 5 =0 fi (y – 2) = 4(x – 1)
5/2 5 0 fi y = 4x – 7
Equation of the diagonal BD is
5Ê 5m ˆ Ê 25 ˆ
fi ÁË 25 - ˜¯ - 5 ÁË 20 - ˜¯ = 0 Ê 1- 5ˆ
2 2 2 ( y - 5) = Á ( x - 2)
Ë 3 - 2 ˜¯
125 25m 125
fi - - 100 + =0 fi (y – 5) = – 4(x – 2)
2 4 2
fi 4x + y = 13.

CG_02.indd 19 2/6/2017 3:59:36 PM


2.20 Coordinate Geometry Booster

3. Ans. (b) R 2 tan a 1


Clearly, the slope of fi 2
=
1 - tan a 3
PM is –1/2
Here, PM is equally fi 6 tana = 1 – tan2a
M
inclined with PQ and 3 fi tan2a + 6tana – 1 = 0
PR. y= - 6 ± 36 + 4
+
Let the slope of PQ be 2 x fi tan (a ) =
2
m, then the slope of
Q P(2, 1) fi tan (a ) = - 3 ± 10
PR will be –1/m.
Ê 1ˆ 1 Ê 1ˆ fi tan (a ) = 10 - 3
m - Á- ˜ - - Á- ˜
Ë 2¯ 2 Ë m¯ p
Thus, = Since 0 < a < .
Ê 1ˆ Ê 1ˆ Ê 1 ˆ 4
1 + mÁ - ˜ 1 + Á - ˜ Á - ˜
Ë 2¯ Ë 2¯ Ë m¯ Thus,
2m + 1 2 - m CA 3
fi = tan a = =
2 - m 2m + 1 OA OA
fi (2m + 1)2 = (2 – m)2 3
fi OA =
fi 4m2 + 4m + 1 = 4 – 4m + m2 tan a
fi 3m2 + 8m – 3 = 0 3
fi 3m2 + 9m – m – 3 = 0 fi OA =
10 - 3
fi 3m(m + 3) – 1(m + 3) = 0
fi (m + 3)(3m – 1) = 0 fi OA = 3( 10 + 3)
1 fi OA = 9 + 3 10
fi m = - 3,
3 5. Equation of the angle bisectors of
Thus, the equations of PQ and PR are x2 – y2 + 2y – 1 = 0 is
1 x 2 - ( y — 1) 2 x( y — 1)
y - 1 = - 3( x - 2), y - 1 = ( x - 2) fi =
3 1+1 0
fi (y – 1) = –3(x – 2), 3(y – 1) = (x – 2)
fi x(y – 1) = 0
Hence, the joint equation of PQ and PR is fi x = 0 and (y –1) = 0
{3(x – 2) + (y – 1)}{(x – 2) – 3(y – 1)} = 0
Y
fi 3(x – 2)2 – 3(y – 1)2 + 8(x – 2)(y – 1) = 0
fi 3x2 – 3y2 + 8xy – 20x – 10y + 25 = 0
C(0, 3)
4. Given equations of tangents are
2x2 + y2 – 3xy = 0
(2x – y)(x – y) = 0
y = x, y = 2x
Y A(0, 1) B(2, 1)
X
O
B
C
2x

Hence, the area of the


y=

1
A DABC = ¥ AB ¥ AC
a y=x
2
1
O
X = ¥ 2 ¥ 2 = 2 sq. units
2
6. Given curve is
(ax2 + bx2 + c)(x2 – 5xy + 6y2) = 0
Let 2a be the angle between these pair of tangents.
fi (ax2 + bx2 + c) = 0, (x2 – 5xy + 6y2) = 0
m2 - m1 2 -1 1 fi (ax2 + by2 + c) = 0, (x – 2y)(x – 3y) = 0
Then tan (2a ) = = =
1 + m1m2 1 + 2 3 fi (ax2 + by2 + c) = 0, (x – 2y) = 0, (x – 3y) = 0
c

1 fi x 2 + y 2 = , ( x - 2y ) = 0, ( x - 3y ) = 0
tan (2a ) = a
3
which represents two straight lines and a circle.

CG_02.indd 20 2/6/2017 3:59:37 PM


Circle 3.1

C H A P T E R

3 Circle

CONCEPT BOOSTER Definition 4


It is the locus of a point which moves
in a plane in such a way that its dis-
1. INTRODUCTION tance from a fixed point is always
Ra
the same. The fixed point is called diu
The word ‘circle’ is derived from the Greek word kirkos s
the centre and the distance between
which means a circle, from the base ker which means to turn
the point and the centre is known as
or bend. The origins of the words ‘circus’ and ‘circuit’ are
the radius of the circle.
closely related.
The circle has been known since before the beginning of Definition 5
recorded history. Natural circles would have been observed, Circle
such as the Moon, Sun, and a short plant stalk blowing in the
wind on sand, which forms a circle-like shape in the sand.
The circle is the basis for the wheel, which, with related in-
ventions such as gears, made much of modern civilisations
possible. In mathematics, the study of the circle has helped Sphere
the development of geometry, astronomy, and calculus.

2. MATHEMATICAL DEFINITIONS In 3D geometry, the section of a sphere by a plane is a circle.


Definition 6
Definition 1 Let z be a complex number and a be a positive real number.
The intersection of a right If |z| = a, the locus of z is a circle.
circular cone and a plane is
Definition 7
a circle, in which the plane
Let z be a complex number and a ΠR+, b ΠR. If |z Рb| = a,
is perpendicular to the axis
the locus of z will be a circle, with the centre at b and the
or parallel to the base of the
radius a.
cone.
Definition 8
Definition 2
Let z1, z2 and z3 be three non-zero complex numbers such that
A circle is a conic section
z - z1
whose eccentricity is zero. = k , where k π 1. The locus of z is a circle.
z - z2
Definition 3
A conic ax2 + 2hxy + by2 + 2gx + 2fy + c = 0 represents a Definition 9
circle if Let z, z1 and z2 be three non-zero complex numbers such that
(i) a = b, |z – z1|2 + |z – z2|2 = |z1 – z2|2. The locus of z is a circle with the
(ii) h = 0, and
z +z 1
(iii) D π 0, where D = abc + 2fgh – af2 – bg2 – ch2 centre at ÊÁ 1 2 ˆ˜ and the radius | z1 - z2 |.
Ë 2 ¯ 2

CG_03.indd 1 2/6/2017 4:00:02 PM


3.2 Coordinate Geometry Booster

Definition 10 5. NATURE OF THE CIRCLE


Let z, z1 and z2 be three non-zero complex numbers such that
The nature of the circle depends on its radius.
Ê z - z2 ˆ p , the locus of z is a circle. Let the equation of a circle is x2 + y2 + 2gx + 2fy + c = 0
arg Á =
Ë z - z1 ˜¯ 2
and its radius is g2 + f 2 - c .
Definition 11
(i) If (g2 + f 2 – c) > 0, it represents a real circle.
Let z, z1 and z2 be three non-zero complex numbers such that
(ii) If (g2 + f 2 – c) < 0, it represents a virtual circle or an
Ê z - z2 ˆ
arg Á = a , where a π 0, p. The locus of z is a circle. imaginary circle.
Ë z - z1 ˜¯ (iii) If (g2 + f 2 – c) = 0, it represents a point circle.

3. STANDARD EQUATION OF A CIRCLE 6. CONCENTRIC CIRCLES


2 2 2
(i) x + y = a , where the centre is (0, 0) and the radius is Two circles having the same centre, say (h, k) and different
a. radii, say r1 and r2 respectively, are called concentric circles.
Y Thus, ( x - h) 2 + ( y - k ) 2 = r12
and ( x1 - h) 2 + ( y1 - k ) 2 = r22
P(x, y)
a y are two concentric circles.
X¢ X Y
O xM
P(x, y)
r1
r2
Y¢ Q(x1, y1)
C(h, k)
(ii) (x – h)2 + (y – k)2 = a2, where the centre is (h, k) and the
radius is ‘a’.
Y X
O
P(x, y)

7. CONCYCLIC POINTS
C(h, k)
If P, Q, R and S lie on the same circle, the points P, Q, R and
S are known as concyclic points.
X¢ X
O
S R

4. GENERAL EQUATION OF A CIRCLE


P Q
The general equation of a circle is x2 + y2 + 2gx + 2fy + c = 0,
where the centre is (–g, –f) and the radius is g 2 + f 2 - c .
As we know that the equation any of circle, when centre 8. PARAMETRIC EQUATION OF A CIRCLE
of a circle is not the origin, is
If the radius of a circle, whose centre is the origin, makes an
(x – h)2 + (y – k)2 = a2
angle q with the positive direction of x-axis, then q is called a
fi x2 – 2hx + h2 + y2 – 2ky + k2 = a2 parameter and 0 £ q < 2p.
fi x2 + y2 – 2hx – 2ky + h2 + k2 – a2 = 0 (i) If the equation of a circle be x2 + y2 = a2, its parametric
fi x2 + y2 + 2gx + 2fy + c = 0, equations are
x = a cos q, y = a sin q,
where h = –g, k = –f and c = h2 + k2 – a2
where q is a parameter.
fi x2 + y2 + 2gx + 2fy + c = 0, (ii) If the equation of a circle be (x – h)2 + (y – k)2 = a2, its
where h = –g, k = –f and parametric equations are
x = h + a cos q, y = k + a sin q,
a = h2 + k 2 - c = g 2 + f 2 - c
where q is a parameter.

CG_03.indd 2 2/6/2017 4:00:03 PM


Circle 3.3

9. DIAMETRIC FORM OF A CIRCLE will intersect each other in four concyclic points, if
a1 - b1 h1
The equation of a circle, when the end-points (x1, y1) and = .
a2 - b2 h2
(x2, y2) of a diameter are given is
(x – x1)(x – x2) + (y – y1) (y – y2) = 0.
13. INTERCEPTS MADE ON THE AXES BY A CIRCLE
10. EQUATION OF A CIRCLE PASSING THROUGH Let the circle be x2 + y2 + 2gx + 2fy + c = 0. Then the
THREE NON-COLLINEAR POINTS
(i) length of x-intercept = 2 g 2 - c
Let the equation of a circle be
(ii) length of y-intercept = 2 f 2 - c .
x2 + y2 + 2gx + 2fy + c = 0.
If three points (x1, y1), (x2, y2), (x3, y3) lie on a circle, then
14. DIFFERENT FORMS OF A CIRCLE
x12 + y12 + 2gx1 + 2fy1 + c = 0,
(i) When the circle touches the x-axis
x22 + y22 + 2gx2 + 2fy2 + c = 0 and The equation of a circle is (x – h)2 + (y – k)2 = k2.
x32 + y32 + 2gx3 + 2fy3 + c = 0. Y

Eliminating g, f and c, we get

x2 + y 2 x y 1
C(h, k)
x12 + y12 x1 y1 1
=0 k
x22 + y22 x2 y2 1 X
O P
x32 + y32 x3 y3 1

(ii) When the circle touches the y-axis


The equation of a circle is (x – h)2 + (y – k)2 = h2.
11. CYCLIC QUADRILATERAL D C
Y
If all the vertices of a quadrilateral lie
on a circle, the quadrilateral is called a
cyclic quadrilateral and the four vertices A B
are known as concyclic points. h
P C(h, k)

12. CONDITION FOR CONCYCLIC POINTS


X
A B O

O (iii) When the circle touches both the axes


The equation of a circle is

D C (x – h)2 + (y – h)2 = h2
or (x – k)2 + (y – k)2 = k2
If A, B, C and D are concyclic points, then Y
OA. OD = OB. OC,
where O is the point of intersection of the chords AD and BC,
and O is not the centre of a circle.
If the lines a1x + b1y + c1 = 0 and a2x + b2y + c2 = 0 cut the Q C(h, h)
x-axis and y-axis in four concyclic points, then
a1a2 = b1b2.
X
Two conics O P
a1x2 + 2h1xy + b1y2 + 2g1x + 2f1y + c1 = 0
and a2x2 + 2h2xy + b2y2 + 2g2x + 2f2y + c2 = 0

CG_03.indd 3 2/6/2017 4:00:04 PM


3.4 Coordinate Geometry Booster

(iv) When the circle passes through the origin and the 15. POSITION OF A POINT WITH RESPECT TO A CIRCLE
centre lies on x-axis
The equation of a circle is (i) A point (x1, y1) lies outside, on and inside of a circle
x2 + y2 = a2, if
(x – h)2 + y2 = h2
or x2 + y2 – 2hx = 0 x12 + y12 - a 2 > 0, = 0, < 0
Y
P

X
O C(h, 0)

(ii) A point (x1, y1) lies outside, on and in-


side of a circle x2 + y2 + 2gx + 2fy + c = 0, if
(v) When the circle passes through the origin and the x12 + y12 + 2gx1 + 2fy1 + c > 0, = 0, < 0 .
centre lies on y-axis
The equation of a circle is 16. SHORTEST AND LARGEST DISTANCE OF A
x2 + (y – k)2 = k2
CIRCLE FROM A POINT
or x2 + y2 – 2ky = 0
Y Let P(x1, y1) be any point and the circle be
x2 + y2 + 2gx + 2fy + c = 0.
Case I: When P lies inside the circle. A
Draw a line through P passing through the
P
C(0, k) centre and intersects the circle at A and B,
respectively. C
Shortest distance, PA = CA – CP
O X B
Longest distance, PB = CP + CB.
(vi) When the circle passes through the origin and has Case II: When P lies outside on the P
intercepts a and b on the x and y axes respectively circle.
The equation of the circle is Draw a line through P passing through A
x2 + y2 – ax – by = 0 the centre and intersects the circle at A
Y and B, respectively. C
Shortest distance = PA = CP – CA
B
Longest distance = PB = CP + CB
B(0, b)
Case III: When P lies on the circle A
Draw a line through P passing through P
X the centre and intersects the circle at A
O A(a, 0)
and B, respectively. C
Shortest distance = 0
Longest distance, PB = 2 radius. B
(vii) When the circle passes through the origin
The equation of the circle is 17. INTERSECTION OF A LINE AND A CIRCLE
x2 + y2 + 2gx + 2fy = 0
Let the circle be x2 + y2 = a2 …(i)
Y
and the line be y = mx + c …(ii)
From Eqs (i) and (ii), we get
x2 + (mx + c)2 = a2
fi (1 + m2)x2 + 2mcx + (c2 – a2) = 0
O
X (i) The line y = mx + c will intersect the circle in two dis-
tinct points, if
D>0

CG_03.indd 4 2/6/2017 4:00:05 PM


Circle 3.5

fi 4m2c2 – 4(1 + m2)(c2 – a2) > 0 Proof: We have,


fi m2c2 – [c2 + m2c2 – a2(1 + m2)] > 0 c
OD =
fi c2 – a2(1 + m2) > 0 1 + m2
fi a2(1 + m2) > c2
In DAOD, AD 2 = OA2 - OD 2
2 c2
fi a > c2
(1 + m 2 ) = a2 -
|c| (1 + m 2 )
fi a>
(1 + m 2 ) a 2 (m 2 + 1) - c 2
=
fi radius > the length of the perpendicular from the (1 + m 2 )
origin to the line y = mx + c. a 2 (m 2 + 1) - c 2
(ii) The line y = mx + c will intersect the circle in two coin- fi AD =
(1 + m 2 )
cident points if
D = 0. Thus, the length of the intercept = AB
fi 4m2c2 – 4(1 + m2)(c2 – a2) = 0 = 2AD
fi m2c2 – (1 + m2)(c2 – a2) = 0 a 2 (m 2 + 1) - c 2
=2¥
fi c2 = a2(1 + m2) (1 + m 2 )
2
c
fi a2 =
(1 + m 2 ) 19. TANGENT AND SECANT
|c|
fi a= . If a line intersects the curve in two coincident points, it is
(1 + m 2 ) called a tangent and if a line intersects the curve in two dis-
fi radius = the length of the perpendicular from the tinct points, it is called a secant.
centre of a circle to the line y = mx + c. Here, PT is a tangent and MN a secant.
(iii) The line y = mx + c will not intersect the circle if
D<0 Q
fi 4m2c2 – 4(1 + m2)(c2 – a2) < 0 Q1
fi m2c2 – [c2 + m2c2 – a2(1 + m2)] < 0
fi c2 – a2(1 + m2) < 0 Q2
fi a2(1 + m2) < c2 Q3
c2
fi a2 <
(1 + m 2 )
|c|
fi a< Different forms of the Equations of Tangents
(1 + m 2 )
1. Point form
fi radius < the length of the perpendicular from the
(i) The equation of tangent to the circle x2 + y2 = a2 at
origin to the line y = mx + c.
(x1, y1) is xx1 + yy1 = a2.
18. LENGTH OF INTERCEPT CUT OFF FROM A Proof: Let C be the centre of the circle.
Now slope of CP
LINE BY A CIRCLE y - 0 y1
= 1 = .
The length of the intercept cut off from the line y = mx + c by x1 - 0 x1
C(0, 0)
Ê a 2 (1 + m 2 ) - c 2 ˆ Here, PT is the perpen-
the circle x2 + y2 = a2 is 2 ¥ Á ˜ . dicular to CP.
Ë (1 + m 2 ) ¯
x
Thus, slope of PT = - 1 .
y1 T
P(x1, y1)
Hence, the equation of
the tangent is
O x
fi y - y1 = - 1 ( x - x1 )
y1
fi yy1 - y1 = - xx1 + x12
2
A D B
fi xx1 + yy1 = x12 + y12 = a 2

CG_03.indd 5 2/6/2017 4:00:06 PM


3.6 Coordinate Geometry Booster

(ii) The equation of tangent to the circle


c2
x2 + y2 + 2gx + 2fy + c = 0 at (x1, y1) is fi a2 =
(1 + m 2 )
x2 + y2 + xx1 + yy1 + c = 0.
Proof: Let C (–g, –f) be the centre of the circle. fi c2 = a2(1 + m2)
y + f
Now, slope of CP = 1 . which is the required condition.
x1 + g
3. Slope form
Here, PT is the perpendicular to CP. The equation of tangent with slope m to a circle x2 + y2 = a2
x +g
Thus, slope of PT = - 1 . is y = mx ± a (1 + m 2 ) and the co-ordinates of the point of
y1 + f
contact are
Hence, the equation of the tangent at (x1, y1) is Ê am a ˆ
x +g ± ,
Á 2 2 ˜
( y - y1 ) = - 1 ( x - x1 ) Ë (1 + m ) (1 + m ) ¯
y1 + f
fi xx1 + yy1 + g(x + x1) + f (y + y1) + c = 0. Note: Equation of any tangent to the circle can be consid-
Note: The equation of the tangent to the 2nd degree ered as y = mx + a 1 + m 2 .
curve ax2 + 2hxy + by2 + 2gx + 2fy + c = 0 is
T
axx1 + h(xy1 + x1y) + byy1 + g(x + x1) + f(y + y1) + c Number of tangents
=0 (i) If a point lies outside of a
circle, the two tangents can
In order to find out the equation of tangent to any 2nd be drawn. Here, TP and TQ
degree curve, the following points must be kept in your are two tangents.
mind A R B
(ii) If a point lies on the circle,
x2 is replaced by xx1, then one tangent can be
y2 is replaced by yy1, drawn. Here, ARB be a tan- P Q
xy + x y
xy is replaced by 1 1 , gent.
2 (iii) If a point lies inside the cir-
x + x1 cle, then no tangent can be
x is replaced by ,
2 drawn.
y + y1
y is replaced by , Tangents from a point to a circle
2
and c will remain c. A
This method is applicable only for a 2nd degree conic.
2. Parametric form P
The equation of a tangent to the circle x2 + y2 = a2 at (a cos q,
a sin q) is x cos q + y sin q = a
Proof: The equation of the chord joining the points (a cos q, B
a sin q) and (a cos j, a sin j) is Let the point be (x1, y1) and the circle be x2 + y2 = a2.
Êq + jˆ Êq + jˆ Êq - jˆ The equation of any tangent from a point (x1, y1) to a circle
x cos Á + y sin Á = a cos Á
Ë 2 ˜¯ Ë 2 ˜¯ Ë 2 ˜¯
.
x2 + y2 = a2 is
The equation of the tangent to the circle (y – y1) = m(x – x1) …(i)
(x – h)2 + (y – k)2 = a2 at (h + a cos q, k + a sin q) is The line (i) will be the tangent to the given circle if the
(x – h)cos q + (y – k)sin q = a length of the perpendicular from the centre of a circle =
radius of a circle
Condition of tangency
The line y = mx + c will be a tangent to the circle x2 + y2 = a2 (mx1 - y1 )
=a
if c2 = a2(1 + m2). 1 + m2
Proof: The line y = mx + c will be a tangent to the circle
fi (mx1 – y1)2 = a2(1 + m2)
x2 + y2 = a2 if radius = the length of perpendicular from the
centre of a circle to the line y = mx + c. fi m 2 x12 - 2mx1 y1 + y12 = a 2 + a 2 m 2
|c| which is a quadratic in m, gives two values of m.
fi a= Put those values of m in Eq (i), we get the required equa-
(1 + m 2 )
tions of tangents.

CG_03.indd 6 2/6/2017 4:00:07 PM


Circle 3.7

Length of tangent from a point to a circle 20. DIRECTOR CIRCLE


(i) The length of the tangent from a point (x1, y1) to the P(h, k)
The locus of the point of intersec-
circle x2 + y2 = a2 is x12 + y12 - a 2 . tion of two perpendicular tangents
to a circle is known as the director
circle. It is a concentric circle hav- A B
0) P(x1, y1) ing radius 2 times the radius of
C(0,
the original circle.
The equation of the director
circle to the circle x2 + y2 = a2 is
T x2 + y2 = 2a2.
(ii) The length of the tangent from a point (x1, y1) to the circle
21. NORMAL N
x2 + y2 + 2gx + 2fy + c = 0 is x12 + y12 + 2gx1 + 2fy1 + c .
If a line is perpendicular to the
Power of a point with respect to a circle point of contact to the tangent is
The power of a point P with re- B called a normal. P
spect to any circle is The relation between tangent
PA ◊ PB. A and normal is
From the geometry, we can write P m(T) ¥ m(N) = –1. T

PA ◊ PB = PT 2
Different Forms of the Equation of Normals
T
Thus, the power of a point is the 1. Point form
square of the length of the tan- (i) The equation of a normal to the circle x2 + y2 = a2 at
gent to a circle from that point. x y
(i) The power of a point (x1, y1) with respect to a circle (x1, y1) is = .
x1 y1
x2 + y2 = a2 is ( x12 + y12 - a 2 ) .
Proof: Let PT be a tangent and PN be a normal.
(ii) The power of a point (x1, y1) with respect to a circle Clearly PN must be a perpendicular to PT.
x2 + y2 + 2gx + 2fy + c = 0 is
N
( x12 + y12 + 2gx1 + 2fy1 + c) .

Pair of tangents T1
(i) The equation of a pair of C(0, 0)
tangents to a circle x2 + y2 P
= a2 from a point (x1, y1) is
SS1 = T2, P(x1, y1) T
T2
where S ∫ x2 + y2 – a2; Equation of tangent at P is
S1 ∫ x12 + y12 2
- a ; T ∫ xx1 + yy1 - a . 2 xx1 + yy1 = a2 …(i)
(ii) The equation of a pair of tangents to a circle Equation of normal at P is
x2 + y2 + 2gx + 2fy + c = 0 is xy1 – yx1 + k = 0 …(ii)
2
SS1 = T , which is passing through (0, 0)
where S ∫ x2 + y2 + 2gx + 2fy + c; S1 Therefore, k = 0.
Hence, the equation of the normal is xy1 – yx1 + k = 0
∫ x12 + y12 + 2gx1 + 2fy1 + c
x y
T ∫ xx1 + yy1 + g(x + x1) + f(y + y1) + c fi =
x1 y1
(iii) The angle between the pair of tangents from (x1, y1) to
(ii) The equation of a normal to the circle
the circle x2 + y2 = a2 is
x2 + y2 + 2gx + 2fy + c = 0 at (x1, y1) is
Ê a ˆ
2 tan -1 , where S1 = x12 + y12 – a2. x - x1 y - y1
ÁË S ˜¯ =
1 x1 + g y1 + f

CG_03.indd 7 2/6/2017 4:00:08 PM


3.8 Coordinate Geometry Booster

Proof: As we know that the normal always passes (ii) The equation of the chord of contact of tangents drawn
through the centre of a circle. from a point (x1, y1) to the circle
Thus, the equation of the normal at P to the circle x2 + y2 + 2gx + 2fy + c = 0
x2 + y2 + 2gx + 2fy + c = 0 is is xx1 + yy1 + g(x + x1) + f(y + y1) + c = 0
y1 + f (iii) The chord of contact exists only when if the point P not
y - y1 = ( x - x1 )
x1 + g lies inside of the circle.
x - x1 y - y1 2LR
= (iv) The length of the chord of contact, T1T2 = ,

x1 + g y1 + f R 2 + L2
where R = radius and L = length of tangent.
(iii) The equation of a normal to a conic
ax2 + 2hxy + by2 + 2gx + 2fy + c = 0 (v) Area of the triangle formed by the pair of tangents and
RL2
at (x1, y1) is its chord of contact = 2 .
R + L2
x - x1 y - y1
=
ax1 + hy1 + g hx1 + by1 + f (vi) Tangents of the angle between the pair of tangents from
2RL
the point (x1, y1) = 2 .
2. Parametric form L - R2
The equation of the normal to the circle x2 + y2 = a2 at
(vii) Equation of the circle circumscribing the triangle
x y
(a cos q, a sin q) is = DPT1T2 is
cos q sin q
(x – x1)(x + g) + (y – y1)(y + f) = 0
3. Slope form
The equation of a normal to the circle x2 + y2 = a2 is y = mx. (viii) The distance between the chord of contact of tan gents
to x2 + y2 + 2gx + 2fy + c = 0 from the origin and the
4. Normal always passes through the centre of the circle point (g, f) is
Y
g2 + f 2 - c
P(x1, y1) 2 (g 2 + f 2 )

C(h, k)
23. CHORD BISECTED AT A GIVEN POINT
(i) The equation of the chord of
X
O the circle x2 + y2 = a2 bisected
at a point (x1, y1) is given by T O
= S1, where S1 ∫ x12 + y12 - a 2
5. Line y = mx + c will be a normal to the circle x2 + y2 = and T ∫ xx1 + yy1 – a2. A B
a2 if c = 0. M
(ii) The equation of the chord of
22. CHORD OF CONTACT the circle x2 + y2 + 2gx + 2fy + c = 0 bisected at a point
(x1, y1) is given by
O T = S1, where S1 ∫ x12 + y12 + 2gx1 + 2fy1 + c and T ∫ xx1
+ yy1 + g(x + x1) + f(y + y1) + c
P
24. DIAMETER OF A CIRCLE
The locus of the mid-points of
R a system of parallel chords of a
circle is known as diameter of
From any external point, two tangents can be drawn to a giv- the circle. C
en circle. The chord joining the points of contact of the two A B
The diameter of a circle al-
tangents is called the chord of contact of tangents. ways passes through the centre
Here, QR is the chord of contact of tangents. of a circle and perpendicular to
(i) The equation of the chord of contact of tangents drawn the parallel chords.
from a point (x1, y1) to the circle x2 + y2 = a2 is Let the circle be x2 + y2 = a2 and parallel chord be y = mx
xx1 + yy1 = a2. + c.

CG_03.indd 8 2/6/2017 4:00:08 PM


Circle 3.9

Equation of any diameter to the given circle is perpendicu- (ix) Conjugate lines: Two straight lines are said to be con-
lar to the given parallel chord is my + x + l = 0 which passes jugate lines if the pole of either lies on the other.
through the centre of a circle. (x) If the lines a1x + b1y + c1 = 0 and a2x + b2y + c2 = 0 are
Thus 0+0+l=0 conjugate to each other with respect to the circle x2 + y2
fi l=0 cc
Hence, the required equation of the diameter is x + my = 0. = c2, then a1a2 + b1b2 = 1 22 .
r
(xi) If O be the centre of a circle and P be any point, OP is
25. POLE AND POLAR perpendicular to the polar of P.
If from a point P, any straight line is drawn to meet the circle (xii) If O be the centre of a circle and P be any point, then if
in Q and R and if tangents to the circle at Q and R meet in OP (produced, if it is necessary) meet the polar of P in
T, the locus of T is called the polar of P with respect to the Q, then OP ◊ OQ = (radius)2.
circle.
The point P is known as the pole of its polar. 26. COMMON CHORD OF TWO CIRCLES
Q
A
B
O O¢
A
P Polar
C B

D The chord joining the points of intersection of two given cir-


cles is called their common chord.
The equation of the common chord between two circles
R
is S1 – S2 = 0
Polar of a circle exists only when the point P lies either fi 2(g1 – g2)x + 2(f1 – f2)y = c1 – c2,
outside or inside of the given circle.
(i) The equation of a polar of the point (x1, y1) with respect where S1 ∫ x2 + y2 + 2g1x + 2f1y + c1 = 0
to the circle x2 + y2 = a2 is xx1 + yy1 = a2 and S2 ∫ x2 + y2 + 2g2x + 2f2y + c2 = 0
(ii) The equation of polar of the point (x1, y1) with respect
to the circle x2 + y2 + 2gx + 2fy + c = 0 is (i) The length of the common chord = PQ = 2 ◊ PM =
xx1 + yy1 + g(x + x1) + f(y + y1) + c = 0. 2 (C1P ) 2 - (C1M ) 2 ,
(iii) If a point P lies outside of a circle, then the polar and
where
the chord of contact of this point P are the same straight
line. C1P = radius of the circle S1 = 0 and
(iv) If a point P lies on the circle, then the polar and the C1M = Length of the perpendicular from C1 on the
tangent to the circle at P are the same straight line. common chord PQ.
(v) The pole of a line lx + my + n = 0 with respect to the (ii) The common chord PQ of two circles becomes of the
Ê a 2l a 2 m ˆ maximum length, when it is a diameter of the smaller
circle x2 + y2 = a2 is Á - ,- ˜.
Ë n n ¯ one between them.
(iii) If the circle on the common chord be a diameter, then
(vi) If the polar of a point P with respect to a circle passes
the centre of the circle passing through P and Q lie on
through Q, then the polar of Q with respect to the circle
the common chord of the two circles.
will pass through P.
(iv) If the circle S1 = 0, bisects the circumference of the
Here, the points P and Q are called the conjugate
circle S2 = 0, then their common chord will be the di-
points.
ameter of the circle S2 = 0.
(vii) Conjugate Points: Two points are said to be conju-
gate points with respect to a circle, if the polar of either
27. INTERSECTION BETWEEN TWO CIRCLES
passes through the other.
(viii) If the pole of a line L1 with respect to a circle lies on Let the two circles be (x – a)2 + (y – b)2 = r12 and (x – g)2 +
another line L2, then the pole of the other line L2 with (y – d)2 = r22, where centres are C1(a, b) and C2(g, d) and radii
respect to the same circle will lie on the first line L1. are r1 and r2, respectively.
Here, the lines L1 and L2 are conjugate lines.

CG_03.indd 9 2/6/2017 4:00:09 PM


3.10 Coordinate Geometry Booster

(i) When two circles do not intersect (iii) When two circles touch each other externally

T2

T4 P
C1 C2
D
C1 r1 T 2 C2 P

T3 Then C1C2 = r1 + r2
Thus three common tangents can be drawn.
T1
Let the point of contact be P.
Then the co-ordinates of P are
Then C1C2 > r1 + r2 Ê r1 x2 + r2 x1 r1 y2 + r2 y1 ˆ
Thus four common tangents can be drawn. ÁË r + r , r + r ˜¯
1 2 1 2
Let P be the point of intersection of two transverse tan-
gents and D be the point of intersection of two direct Hence the equation of the common transverse tangent
common tangents. is S1 – S2 = 0 which is the same as the equation of the
Then the co-ordinates of P and D are common chord.
Ê r1 x2 + r2 x1 r1 y2 + r2 y1 ˆ Let two direct common tangents intersect at D exter-
ÁË r + r , r + r ˜¯ = (h, k ) nally in the ratio r1 : r2.
1 2 1 2 Êrx -r x ry -r y ˆ
Then the co-ordinates of D are Á 1 2 2 1 , 1 2 2 1 ˜
Ê r1 x2 - r2 x1 r1 y2 - r2 y1 ˆ Ë r1 - r2 r1 - r2 ¯
and ÁË r - r , r - r ˜¯ = (l , m ) = (h, k)
1 2 1 2
Hence the equation of the direct common tangent is
respectively (y – k) = m(x – h).
The equation of the transverse common tangents is (y Now values of m can be obtained from the length of
– k) = m1(x – h) and the equation of direct common perpendicular from the centre C1 or C2 on the tangent
tangent is (y – m) = m2(x – l). is equal to r1 or r2. Put two values of m on the common
Now values of m1 and m2 can be obtained from the tangent equation, then we get the required equation of
length of the perpendicular from the centre C1 or C2 on direct common tangents.
the tangent is equal to r1 or r2. Put two values of m1 and (iv) When two circles touch each other internally
m2 on the common tangent equations, then we get the P
required results.
(ii) When two circles intersect
O
C
r1 r2
C1 C2
Q

Then C1C2 = |r1 – r2|


Then |r1 – r2| < C1C2 < r1 + r2 Thus, only one tangent can be drawn.
Thus two common tangents can be drawn. Equation of the common transverse tangent is
Let two direct common tangents intersect at D exter- S1 – S2 = 0.
nally in the ratio r1 : r2. (v) When one circle lies inside the other one
Then the co-ordinates of D are
Ê r1 x2 - r2 x1 r1 y2 - r2 y1 ˆ
ÁË r - r , r - r ˜¯ = (h, k )
1 2 1 2
C
Hence the equation of the direct common tangent is C2
y – k = m(x – h).
Now values of m can be obtained from the length of the
perpendicular from the centre C1 or C2 on the tangent
is equal to r1 or r2.
Put two values of m on the common tangent equation, then Then C1C2 > |r1 – r2|
we get the required equation of direct common tangents. Thus, no common tangent can be drawn.

CG_03.indd 10 2/6/2017 4:00:09 PM


Circle 3.11

(vi) The direct common tangents meet at a point which di- Condition of Orthogonality
vides the line joining the centres of the circles exter-
nally in the ratio of their radii.
(vii) Transverse common tangents meet at a point which P
divides the line joining the centres of the circles inter-
nally in the ratio of their radii. r1 r2
(viii) The length of an external common tangent and inter-
nal common tangent to the two circles is given by the C1 C2
length of external common tangent
Lex = d 2 - (r2 - r1 ) 2
and the length of internal common tangent Here, q = 90°, then
2 2
Lin = d - (r2 + r1 ) , r12 + r22 - d 2
fi =0
(it is applicable only when d > r1 + r2) 2r1r2
Where d is the distance between the centres of two r12 + r22 - d 2 = 0
circles and r1 and r2 are the radii of two circles where
fi r12 + r22 = d 2
C1C2 = d.
fi g12 + f12 - c1 + g 22 + f 22 - c2
29. ANGLE OF INTERSECTION OF TWO CIRCLES = g12 + f12 + g 22 + f 22 - 2( g1 g 2 + f1 f 2 )
Let the two circles be fi 2(g1g2 + f1f2) = c1 + c2
S1: x2 + y2 + 2g1x + 2f1y + c1 = 0 which is the required condition.
S2: x2 + y2 + 2g2x + 2f2y + c2 = 0
30. RADICAL AXIS
P
P

(–g1 – f1) q
A B
O1 O2
(–gz – fz)
C1 C2

Let C1 and C2 are the centres of the given circles and r1 and
r2 are the radii of the circles. The radical axis of two circles is the locus of a point which
Thus C1 = (–g1, –f1) and C2 = (–g2, –f2) moves in a plane in such a way that the lengths of the tan-
gents drawn from it to the two circles are same.
r1 = g12 + f12 - c1 , Consider
S1 : x2 + y2 + 2g1x + 2f1y + c1 = 0
and r2 = g 22 + f 22 - c2
and S2 : x2 + y2 + 2g2x + 2f2y + c2 = 0
Let d = C1C2 = ( g1 - g 2 ) 2 + ( f1 - f 2 ) 2 Let P(x1, y1) be a point such that PA = PB
Thus, PA2 = PB2
= g12 + g 22 + f12 + f 22 - 2( g1 g 2 + f1 f 2 )
fi x12 + y12 + 2g1 x1 + 2f1 y1 + c1
Ê r12 r22
+ -d 2ˆ
= x22 + y22 + 2g 2 x2 + 2f 2 y2 + c2
In C2PC2, cos a = Á ˜¯
Ë 2r1r2
fi 2(g1 – g2)x + 2(f1 – f2)y = c2 – c1
Ê r 2 + r22 - d 2 ˆ
fi cos (180° - q ) = Á 1 ˜¯ 31. PROPERTIES OF THE RADICAL AXIS
Ë 2r r 1 2
(i) The radical axis and common chord are identical.
Orthogonal Circles Since the radical axis and common chord of two circles
If the angle between two circles is 90°, then the circles are S1 = 0 and S2 = 0 are the same straight line S1 – S2 = 0,
said to be orthogonal circles. they are identical.

CG_03.indd 11 2/6/2017 4:00:10 PM


3.12 Coordinate Geometry Booster

The only difference is that, the common chord exists (v) The radical axis of three circles taken in pairs are
only if the circles intersect in two real points, while the concurrent.
radical axis exists for all pair of circles irrespective of Let
their position. S1 : x2 + y2 + 2g1x + 2f1y + c1 = 0,
(ii) The radical axis is perpendicular to the straight line S2 : x2 + y2 + 2g2x + 2f2y + c2 = 0
which joins the centres of the circles. and
P
S3 : x2 + y2 + 2g3x + 2f3y + c3 = 0
L

S
T

O
A B S1 = 0 S2 = 0

M
N
Consider
S1 : x2 + y2 + 2g1x + 2f1y + c1 = 0 S3 = 0

and S2 : x2 + y2 + 2g2x + 2f2y + c2 = 0 Now S1 – S2 : 2(g1 – g2)x + 2(f1 – f2)y = c2 – c1


Here, A(–g1, – f1) and B(–g2, –f2) are the centres of the S2 – S3 : 2(g2 – g3)x + 2(f2 – f3)y = c3 – c2
circles. S3 – S1 : 2(g3 – g1)x + 2(f3 – f1)y = c1 – c3
f - f Adding we get, both the sides are identical.
Now the slope of AB = 1 2 .
g1 - g 2 Thus three radical axes are concurrent.
Equation of the radical axis is (vi) If two circles cut a third circle othogonally, the radi-
cal axis of the two circles will pass through the cen-
2(g1 – g2)x + 2(f1 – f2)y = c2 – c1.
tre of the third circle.
g - g2
\ Slope of the radical axis is - 1 .
f1 - f 2 P A P
Clearly the product of their slopes is –1.
Hence AB and radical axis are perpendicular to each
other.
(iii) If two circles touch each other externally or inter-
nally, the common tangents itself becomes the radi-
cal axis.
Q
S
C C¢
A B
B
Radical axis 2 2
Radical axis Let S1 : x + y + 2g1x + 2f1y + c1 = 0, …(i)
2 2
(iv) The radical axis bisects common tangents of two S2 : x + y + 2g2x + 2f2y + c2 = 0 …(ii)
circles. and S3 : x2 + y2 + 2g3x + 2f3y + c3 = 0 …(iii)
P
Since (i) and (ii) both cut (iii) orthogonally, then
L 2(g1g3 + f1f3) = c1 + c3
M
2(g2g3 + f2f3) = c2 + c3
Subtracting, we get
A B 2g3(g1 – g2) + 2f3(f1 – f2) = c1 – c2 …(iv)
Also radical axis of (i) and (ii) is
S1 – S2 : 2(g1 – g2)x + 2(f1 – f2)y = c2 – c1
Q
Since it will pass through the centre of third circle, so
Radical axis we get
In this case the radical axis bisects the common tangent –2g3(g1 – g2) – 2f3(f1 – f2) = –c1 + c2

CG_03.indd 12 2/6/2017 4:00:11 PM


Circle 3.13

fi 2g3(g1 – g2) + 2f3(f1 – f2) = c1 – c2


A
which is identical with (iv).
(vii) Radical axis need not always pass through the mid- E
point of the line joining the centres of the two cir- F H
cles.
It will pass through the mid-point of the line joining the C
centres of the two circles only if they have equal radii. B D
(viii) The centre of a variable circle orthogonal to two fixed
circles lies on the radical axis of two circles.
(ix) If two circles are orthogonal, then the polar of a point
P on the first circle with respect to the second circle 2. The radical centre of the three given circles will be the
passes through the point Q, which is the other end of centre of a fourth circle, which cuts all the three circles
the diameter through P. orthogonally and the radius of the fourth circle is the
Hence the locus of a point which moves in such a way length of the tangent drawn from the radical centre of
that, its polars with respect to the circles S1 = 0, S2 = 0 the three given circles to any of these circles.
and S3 = 0 are concurrent in a circle which is orthogo- A
nal to all the three circles.
(x) Pairs of circles which do not have radical axis are con-
current.
(xi) A system of circles, every two of which have the same
O
radical axis, is called a coaxial system.

32. RADICAL CENTRE


B C
L

Let the fourth circle be (x – h)2 + (y – k)2 = r2, where


(h, k) is the centre of the circle and r be the radius. The
centre of the circle is the radical centre of the given
O
S1 = 0 S2 = 0 circles and r is the length of the tangent from (h, k) to
any of the given three circles.
M
N
33. FAMILY OF CIRCLES
S3 = 0

The radical axes of three circles, taken in pairs, meet at a


point, which is called their radical centre.
Let S1: x2 + y2 + 2g1x + 2f1y + c1 = 0,
S2: x2 + y2 + 2g2x + 2f2y + c2 = 0
and S3: x2 + y2 + 2g3x + 2f3y + c3 = 0 S1 = 0 S2 = 0
Equations of three radical axes are
S1 – S2: 2(g1 – g2)x + 2(f1 – f2)y = c2 – c1 Family of circles
S2 – S3: 2(g2 – g3)x + 2(f2 – f3)y = c3 – c2
and 1. The equation of the family of circles passing through
S3 – S1: 2(g3 – g1)x + 2(f3 – f1)y = c1 – c3 the point of intersection of two given circles S1 = 0 and
S2 = 0 is given by
Solving the three equations of radical axes, we get the re-
quired radical centre. S1 + lS2 = 0, where l is a parameter and l π –1.
2. The equation of the family of circles passing through
Properties of the Radical Centres the point of intersection of a circle S = 0 and a line
1. The radical centre of the three circles described on the L = 0 is given by
sides of a triangle as diameters is the orthocentre of the S + lL = 0, where l is a parameter.
triangle.

CG_03.indd 13 2/6/2017 4:00:11 PM


3.14 Coordinate Geometry Booster

(x – x1)(x – x2) + (y – y1)(y – y2)

x y 1
+ l x1 y1 1 = 0
x2 y2 1

L=0 P(x1, y1)


S=0
3. The equation of the family of circles touching the circle
S = 0 and the line L = 0 is
S + lL = 0, where l is a parameter.
4. The equation of the family of circles passing through
the two given points P(x1, y1) and Q(x2, y2) can be writ- Q(x1, y1)
ten in the form S=0

E XERCISES

LEVEL I 12. Find the equation of a circle passing through the points
(1, 2) and (3, 4) and touching the line 3x + y = 5.
(Problems Based on Fundamentals) 13. Find the length of intercepts to the circle x2 + y2 + 6x +
10y + 8 = 0.
ABC OF CIRCLES
14. Find the length of y-intercept to the circle x2 + y2 – x – y
1. Find the centre and the radius of the circles = 0.
(i) x2 + y2 = 16 15. Show that the circle x2 + y2 – 2ax – 2ay + a2 = 0
(ii) x2 + y2 – 8x + 15 = 0 touches both the co-ordinate axes.
(iii) x2 + y2 – x – y = 0
2. Prove that the radii of the circles POSITION OF A POINT WITH RESPECT TO A CIRCLE
x2 + y2 = 1, x2 + y2 – 2x – 6y = 6
16. Discuss the position of the points (1, 2) and (6, 0) with
and x2 + y2 – 4x – 12y = 9 are in AP.
respect to the circle x2 + y2 – 4x + 2y – 11 = 10.
3. Find the equation of the circle concentric with the cir-
17. If the point (l, –l) lies inside the circle
cle x2 + y2 – 8x + 6y – 5 = 0 and passing through the
point (–2, –7). x2 + y2 – 4x + 2y – 8 = 0, then find range of l.
4. Find the equation of the circle passing through the
SHORTEST AND LONGEST DISTANCE OF A CIRCLE FROM A POINT
point of intersection of x + 3y = 0 and 2x – 7y = 0 and
whose centre is the point of intersection of lines x + y + 18. Find the shortest and the longest distance from the
1 = 0 and x – 2y + 4 = 0. point (2, –7) to the circle
5. Find the equation of the circle touching the lines x2 + y2 – 14x – 10y – 151 = 0
4x – 3y = 30 and 4x – 3y +10 = 0 having the centre on
the line 2x + y = 0. INTERSECTION OF A LINE AND A CIRCLE
6. Let the equation of a circle is x2 + y2 – 16x – 24y + 183 19. Prove that the line y = x + 2 touches the circle x2 + y2 =
= 0. Find the equation of the image of this circle by the 2. Find its point of contact.
line mirror 4x + 7y + 13 = 0. 20. Find the equation of the tangent to the circle x2 + y2 = 4
7. Find the area of an equilateral triangle inscribed in the parallel to the line 3x + 2y + 5 = 0.
circle x2 + y2 + 2gx + 2fy + c = 0. 21. Find the equation of the tangent to the circle x2 + y2 = 9
8. A circle has radius 3 units and its centre lies on the line perpendicular to the line 4x + 3y = 0.
y = x – 1. Find the equation of the circle if it passes 22. Find the equation of the tangent to the circle x2 + y2 +
through (7, 3). 4x + 3 = 0, which makes an angle of 60° with the posi-
9. Find the point P on the circle x2 + y2 – 4x – 6y + 9 = 0 tive direction of x-axis.
such that –POX is minimum, where O is the origin and 23. If a tangent is equally inclined with the co-ordinate
OX is the x-axis. axes to the circle x2 + y2 = 4, find its equation.
10. Find the equation of the circle when the end-points of a 24. Find the equation of the tangents to the circle x2 + y2 =
diameter are (2, 3) and (6, 9). 25 through (7, 1).
11. Find the equation of a circle passing through (1, 2),
(4, 5) and (0, 9).

CG_03.indd 14 2/6/2017 4:00:12 PM


Circle 3.15

TANGENT TO A CIRCLE 44. Find the equation of the director circle to each of the
following given circles.
25. If the centre of a circle x2 + y2 = 9 is translated 2 units
(i) x2 + y2 + 2x = 0
parallel to the line x + y = 4 where x increases, find its
(ii) x2 + y2 + 10y + 24 = 0
equation.
(iii) x2 + y2 + 16x + 12y + 99 = 0
26. Find the equation of the tangents to the circle x2 + y2 =
(iv) x2 + y2 + 2gx + 2fy + c = 0.
9 at x = 2.
(v) x2 + y2 – ax – by = 0.
27. Find the equation of the tangent to the circle x2 + y2 =
16 at y = 4. NORMAL AND NORMALCY
28. Find the points of intersection of the line 4x – 3y = 10
and the circle. 45. Find the equation of the normal to the circle x2 + y2 = 9
29. Find the equation of the pair of tangents drawn to the at x = 2.
circle x2 + y2 – 2x + 4y = 0 from (0, 1). 46. Find the equation of the normal to the circle x2 + y2 +
30. Find the equation of the common tangent to the curves 2x + 4y + 4 = 0 at (–2, 1).
x2 + y2 = 4 and y2 = 4(x – 2). 47. Find the equation of a normal to a circle x2 + y2 – 4x –
31. Find the shortest distance between the circle x2 + y2 = 9 6y + 4 = 0, which is parallel to the line y = x – 3.
and the line y = x – 8. 48. Find the equation of the normal to a circle x2 + y2 – 8x –
12y + 99 = 0, which is perpendicular to the line 2x – 3y
LENGTH OF THE TANGENT TO A CIRCLE + 10 = 0.
32. Find the length of the tangent from the point (2, 3) to CHORD OF CONTACT
the circle x2 + y2 = 4.
33. Find the length of the tangent from any point on the 49. Find the equation of the chord of contact of the tan-
circle x2 + y2 = a2 to the circle x2 + y2 = b2. gents drawn from (5, 3) to the circle x2 + y2 = 25.
34. Find the length of the tangent from any point on the 50. Find the co-ordinates of the point of intersection of the
circle x2 + y2 + 2011x + 2012y + 2013 = 0 to the circle tangents at the points where the line 2x + y + 12 = 0
x2 + y2 + 2011x + 2012y + 2014 = 0. meets the circle x2 + y2 – 4x + 3y – 1 = 0.
51. Find the condition that the chord of contact from an
POWER OF A POINT W.R.T A CIRCLE external point (h, k) to the circle x2 + y2 = a2 subtends a
right angle at the centre.
35. Find the power of a point (2, 5) with respect to the cir-
52. Tangents are drawn from the point (h, k) to the cir-
cle x2 + y2 = 16.
cle x2 + y2 = a2. Prove that the area of the triangle
36. If a point P(1, 2) is rotated about the origin in an anti-
formed by the tangents and their chord of contact is
clockwise sense through an angle of 90° say at Q, then
find the power of a point Q with respect to the circle Ê (h 2 + k 2 - a 2 )3/2 ˆ
aÁ ˜.
x2 + y2 = 4. Ë (h 2 + k 2 ) ¯
53. The chord of contact of tangents drawn from a point on
PAIR OF TANGENTS
the circle x2 + y2 = a2 to the circle x2 + y2 = b2 touches
37. Find the equation of the tangent from the point (1, 2) to the circle x2 + y2 = c2 such that bm = ancp, where m, n, p
the circle x2 + y2 = 4. ΠN, find the value of m + n + p + 10.
38. Find the equation of the tangent to the circle x2 + y2 –
4x + 3 = 0 from the point (2, 3). CHORD BISECTED AT A POINT
39. Find the angle between the tangent from the point (3, 5) 54. Find the equation of the chord of the circle x2 + y2 = 25,
to the circle x2 + y2 = 25. which is bisected at the point (–2, 3).
40. If a point (1, 2) is translated 2 units through the positive 55. Find the equation of the chord of the circle x2 + y2 + 6x
direction of x-axis and then tangents drawn from that + 8y – 11 = 0, whose mid-point is (1, –1).
point to the circle x2 + y2 = 9, find the angle between 56. Find the locus of the middle points of the chords of the
the tangents. circle x2 + y2 = a2 which pass through the fixed point
(h, k).
DIRECTOR CIRCLE
57. Find the locus of the mid-point of a chord of the circle
41. Find the locus of the point of intersection of two per- x2 + y2 = 4 which subtend a right angle at the centre.
pendicular tangents to a circle x2 + y2 = 25. 58. Let AB be a chord of the circle x2 + y2 = 4 such that
42. Tangents are drawn from an arbitrary point on the line A = ( 3, 1) . If the chord AB makes an angle of 90°
y = x + 1 to the circle x2 + y2 = 9. If those tangents are about the origin in anti-clockwise direction, then find
orthogonal to each other, find the locus of that point. the co-ordinates of B.
43. Tangents are drawn from any point on the circle x2 + y2 59. Let CD be a chord of the circle x2 + y2 = 9 such that
= 20 to the circle x2 + y2 = 10. Find the angle between
C = (2 2, 1) . If the chord CD makes an angle of 60°
their tangents.

CG_03.indd 15 2/6/2017 4:00:12 PM


3.16 Coordinate Geometry Booster

about the centre of the circle in clockwise direction, 74. Two circles having radii r1 and r2 intersect orthogonal-
find the co-ordinates of D. ly, find the length of the common chord.
60. Find the locus of the mid-points of the chords of the
circle x2 + y2 = 9 which are parallel to the line 2012x + RADICAL AXIS
2013y + 2014 = 0. 75. Find the radical axis of the two circles
61. Find the locus of the mid-points of the chords of the x2 + y2 + 4x + 6y + 9 = 0
circle x2 + y2 – 4x – 6y = 0, which are perpendicular to and x2 + y2 + 3x + 8y + 10 = 0
the line 4x + 5y + 10 = 0. 76. Find the image of a point (2, 3) with respect to the radi-
cal axis of two circles x2 + y2 + 8x + 2y + 10 = 0 and
COMMON CHORD OF TWO CIRCLES x2 + y2 – 2x – y – 8 = 0.
62. Find the lengths of the common chord of the circles 77. Find the radical centre of the three circles x2 + y2 = 1,
x2 + y2 + 3x + 5y + 4 = 0 and x2 + y2 + 5x + 3y + 4 = 0. x2 + y2 – 8x + 15 = 0 and x2 + y2 + 10y + 24 = 0.
63. Find the equation of the circle whose diameter is the 78. Find the equation of a circle which cuts orthogonally
common chord of the circles every member of the circles
x2 + y2 + 2x + 3y + 1 = 0 x2 + y2 – 3x – 6y + 14 = 0,
x2 + y2 – x – 4y + 8 = 0
and x2 + y2 + 4x + 3y + 2 = 0.
and x2 + y2 + 2x – 6y + 9 = 0
INTERSECTION OF TWO CIRCLES
79 Find the equation of the circle passing through the
points of intersection of the circles
64. Find the number of tangents between the given circles x2 + y2 + 13x – 3y = 0
(i) x2 + y2 = 4 and x2 + y2 – 2x = 0 and 2x2 + 2y2 + 4x – 7y – 25 = 0
(ii) x2 + y2 + 4x + 6y + 12 = 0 and and the point (1, 1).
x2 + y2 – 6x – 4y + 12 = 0 80. If the circle x2 + y2 + 2x + 3y + 1 = 0 cuts the circle
(iii) x2 + y2 – 6x – 6y + 9 = 0 and x2 + y2 + 4x + 3y + 2 = 0 in two points, say A and B, find
x2 + y2 + 6x + 6y + 9 = 0 the equation of the circle as AB as a diameter.
(iv) x2 + y2 – 4x – 4y = 0 and 81. Find the equation of the smallest circle passing through
x2 + y2 + 2x + 2y = 0 the intersection of the line x + y = 1 and the circle
(v) x2 + y2 = 64 and x2 + y2 – 4x – 4y + 4 = 0 x2 + y2 = 9.
(vi) x 2 + y 2 - 2(1 + 2) x - 2y + 1 = 0 and 82. Find the equation of the circle, which is passing through
x2 + y2 – 2x – 2y + 1 = 0. the point of intersection of the circles
65. Find all the common tangents to the circles x2 + y2 – 6x + 2y + 4 = 0
x2 + y2 – 2x – 6y + 9 = 0 and x2 + y2 + 6x – 2y + 1 = 0. and x2 + y2 + 2x – 4y – 6 = 0
and its centre lies on the line y = x.
66. If two circles x2 + y2 + c2 = 2 and x2 + y2 + c2 = 2by
83. Find the circle whose diameter is the common chord of
1 1 1
touches each other externally, prove that 2 = 2 + 2 . the circles x2 + y2 + 2x + 3y + 1 = 0 and x2 + y2 + 4x +
a b c 3y + 2 = 0.
67. If two circles x2 + y2 = 9 and x2 + y2 – 8x – 6y + n2 =
0, where n is any integer, have exactly two common
tangents, find the number of possible values of n.
LEVEL II
(Mixed Problems)
ORTHOGONAL CIRCLES 1. The equation ax2 + 2hxy + by2 + 2gx + 2fy + c = 0 rep-
68. Find the angle at which the circles x2 + y2 + x + y = 0 resents a circle if
and x2 + y2 + x – y = 0 intersect. (a) a = b, c = 0 (b) a = b, h = 0
69. If the circles x2 + y2 + 2a1x + 2b1y + c1 = 0 and x2 + y2 (c) a = b, g = 0 (d) a = b, f = 0
+ a2x + b2y + c2 = 0 intersect orthogonally, prove that 2. The equation of the circle passing through the points
a1a2 + b1b2 = c1 + c2. (0, 0), (1, 0) and (0, 1) is
70. If two circles 2x2 + 2y2 – 3x + 6y + k = 0 and x2 + y2 – 4x (a) x2 + y2 + x + y = 0 (b) x2 + y2 – x + y = 0
+ 10y + 16 = 0 cut orthogonally, find the value of k. 2 2
(c) x + y – x – y = 0 (d) x2 + y2 – x = 0
71. A circle passes through the origin and centre lies on the 2 2
3. The equation x + y + 4x + 6y + 13 = 0 represents a
line y = x. If it cuts the circle x2 + y2 – 4x – 6y + 18 = 0 (a) point (b) circle
orthogonally, find its equation. (c) pair of straight lines (d) a pair of coincident
72. Find the locus of the centres of the circles which cut the
lines
circles x2 + y2 + 4x – 6y + 9 = 0 and x2 + y2 – 5x + 4y – 2
4. The circle x2 + y2 + 4x – 7y + 12 = 0 cuts an intecept on
= 0 orthogonally.
73. If a circle passes through the point (a, b) and cuts the y-axis is
circle x2 + y2 = 4 orthogonally, find the locus of its centre. (a) 7 (b) 4 (c) 3 (d) 1

CG_03.indd 16 2/6/2017 4:00:12 PM


Circle 3.17

5. The equation of the diameter of the circle x2 + y2 – 6x + 19. The length of the common chord of the circles x2 + y2 +
2y – 8 = 0 is 2x + 3y + 1 = 0 and x2 + y2 + 3x + 2y + 1 = 0 is
(a) x + 3y = 0 (b) x = 3y 1 3 2
(c) x = 2y (d) x + 2y = 0. (a) 3 2 (b) (c) (d)
3 2 2 3
6. The length of the tangent drawn from any point on the
circle x2 + y2 + 4x + 6y + 4 = 0 to the circle x2 + y2 + 4x 20. The number of common tangents to the circles x2 + y2
+ 6y + 11 = 0 is + 2x – 8y – 23 = 0 and x2 + y2 – 4x – 10y + 19 = 0 is
(a) 4 (b) 7 (c) 15 (d) 17 (a) 4 (b) 2 (c) 3 (d) 1
7. The value of c for which the points (2, 0), (0, 1), (0, 5) 21. The angle between the tangents drawn from the origin
and (0, c) are concyclic, is to the circle (x – 7)2 + (y + 1)2 = 25 is
(a) 1 (b) 2 (c) 3 (d) 4 (a) p/3 (b) p/6 (c) p/2 (d) p/4
8. The equation of a circle passing through the origin is x2 22. The ends of a quadrant of a circle have the co-ordinates
+ y2 – 4x + 6y = 0, the equation of the diameter is (1, 3) and (3, 1), the centre of such a circle is
(a) x = y (b) 3x +2y = 0 (a) (1,1) (b) (2, 2) (c) (2, 6) (d) (4, 4)
(c) y = 3x (d) 3x – 4y = 0 23. The line 2x – y + 1 = 0 is a tangent to the circle at the
9. The equation of the common chord of the circles point (2, 5) and the centre of the circles lies on x – 2y =
x2 + y2 – 4x + 6y = 0 and x2 + y2 – 6x + 4y – 12 = 0 is 4. The radius of the circle is
(a) x + y + 6 = 0 (b) x – y + 6 = 0 (a) 3 5 (b) 5 3 (c) 2 5 (d) 5 2
(c) x – y – 6 = 0 (d) –x + y + 6 = 0 24. Two circles of radii 4 cm and 1 cm touch each other
10. The circumcentre of the triangle whose vertices are externally and q is the angle contained by their direct
(0, 0), (2, 0) and (0, 2) is common tangents. Then sin q is
(a) (1, 2) (b) (2, 2) (a) 24/25 (b) 12/25 (c) 3/4 (d) None
(c) (1, 1) (d) (–2, –2) 25. The locus of poles whose polar with respect to x2 + y2
11. If the lines a1x + b1y + c1 = 0 and a2x + b2y + c2 = 0 cut = a2 always pass through (k, 0) is
the x-axis and y-axis in four concyclic points, then (a) kx – a2 = 0 (b) kx + a2 = 0
2
(a) a1a2 = b1b2 (b) a1b1 = a2b2 (c) ky + a = 0 (d) ky – a2 = 0
(c) a1b2 = a2b1 (d) None 26. The locus of the mid-points of the chords of the circle
12. If the circles x2 + y2 + 2g1x + 2f1y = 0 and x2 + y2 + 2g2x x2 + y2 – ax – by = 0 which subtend a right angle at
+ 2f2y = 0 touch each other, then Ê a bˆ
ÁË , ˜¯ is
(a) g1g2 + f1f2 = 0 (b) g1g2 = f1f2 2 2
(c) g1f2 = f1g2 (d) None
(a) ax + by = 0
13. Any point on the circle x2 + y2 – 4x – 4y + 4 = 0 can be
(b) ax + by = a2 + b2
taken as
(a) (2 + 2 cos q, 2 + 2 sin q) a 2 + b2
(c) x 2 + y 2 - ax - by + =0
(b) (2 – 2 cos q, 2 – 2 sin q) 8
(c) (2 – 2 cos q, 2 + 2 sin q) a 2 + b2
(d) x 2 + y 2 - ax - by - =0
(d) (2 + 2 cos q, 2 – 2 sin q) 8
14. The equation of the chord of the circle x2 + y2 = 25 27 From (3, 4), chords are drawn to the circle x2 + y2 – 4x
whose mid-point is (2, –5) is = 0. The locus of the mid-points of the chords is
(a) 3x – 2y = 11 (b) 2x – 3y = 13 (a) x2 + y2 – 5x – 4y + 6 = 0
(c) 2x + 3y = 10 (d) 3x + 2y = 11 (b) x2 + y2 + 5x – 4y + 6 = 0
15. The value of l for which the line 3x – 4l = l touches (c) x2 + y2 – 5x + 4y + 6 = 0
the circle x2 + y2 = 16 is (d) x2 + y2 – 5x – 4y – 6 = 0
(a) 4 (b) 20 (c) 15 (d) 10 2
16. The locus of the point of intersection of two perpen- 28. The lines y - y1 = m( x - x1 ) ± a 1 + m are tangents
dicular tangents to the circle x2 + y2 = 1006 is to the same circle. The radius of the circle is
(a) x2 + y2 = 2012 (b) x2 + y2 = 2020 (a) a/2 (b) a (c) 2a (d) none
2 2
(c) x + y = 2010 (d) x2 + y2 = 2000 29. The centre of the smallest circle touching the circles x2
17. The equation of the normal to the circle x2 + y2 – 2x – + y2 – 2y – 3 = 0 and x2 + y2 – 8x – 18y + 93 = 0 is
2y + 1 = 0, which is parallel to the line 2x + 4y = 3 is (a) (3, 2) (b) (4, 4) (c) (2, 7) (d) (2, 5)
(a) x + 2y = 3 (b) x + 2y + 3 = 0 30. The ends of the base of an isosceles triangle are at
(c) 2x + 4y = –3 (d) none (2, 0) and (0, 1) and the equation of one side is x = 2,
18. The image of the centre of the circle x2 + y2 – 2x – 6y + the orthocentre of the triangle is
1 = 0 to the line mirror y = x is Ê 3 3ˆ Ê5 ˆ Ê3 ˆ Ê4 7 ˆ
(a) Á , ˜ (b) Á , 1˜ (c) Á , 1˜ (d) Á , ˜
(a) (1, 3) (b) (3, 1) (c) (1, –3) (d) (–3, 1) Ë 4 2¯ Ë4 ¯ Ë4 ¯ Ë 3 12 ¯

CG_03.indd 17 2/6/2017 4:00:12 PM


3.18 Coordinate Geometry Booster

31. A rhombus is inscribed in the region common to two


circles x2 + y2 – 4x – 12 = 0 and x2 + y2 + 4x – 12 = 0 g2 + f 2 - c
(a) g2 + f 2 (b)
with two of its vertices on the line joining the centres of 2
the circles. Then the area of the rhombus is
g2 + f 2 - c g2 + f 2 - c
(a) 8 3 sq. units (b) 4 3 sq. units (c) (d)
2 g2 + f 2 2 g2 + f 2
(c) 16 3 sq. units (d) none
32. The angle between the two tangents from the origin to 40. The locus of the centres of the circles which cuts the
the circles (x – 7)2 + (y + 1)2 = 25 is circles x2 + y2 + 4x – 6y + 9 = 0 and x2 + y2 – 5x + 4y – 2
(a) p/4 (b) p/3 (c) p/2 (d) None. = 0 orthogonally is
33. The equation of the circle having normal at (3, 3) as the (a) 9x + 10y – 7 = 0 (b) x – y + 2 = 0
straight line y = x and passing through the point (2, 2) is (c) 9x – 10y + 11 = 0 (d) 9x + 10y + 7 = 0.
(a) x2 + y2 – 5x + 5y + 12 = 0 41. The locus of the centres of the circles such that the point
(b) x2 + y2 + 5x – 5y + 12 = 0 (2, 3) is the mid-point of the chord 5x + 2y + 16 = 0 is
(c) x2 + y2 – 5x – 5y – 12 = 0 (a) 2x – 5y + 11 = 0 (b) 2x + 5y – 11 = 0
(d) x2 + y2 – 5x – 5y + 12 = 0 (c) 2x + 5y +11 = 0 (d) None.
34. In a right triangle ABC, right angled at A, on the leg 42. The locus of the mid-points of the chords of the circle
p
AC as diameter, a semi-circle is described. If the chord x2 + y2 + 4x – 6y – 12 = 0 which subtends an angle of
radians at its circumcentre is 3
joining A with the point of intersection D of the hypot-
enuse and the semicircle, the length AC equals to (a) (x – 2)2 + (y + 3)2 = 6.25
AB ◊ AD (b) (x + 2)2 + (y – 3)2 = 6.25
AB ◊ AD
(a) (b) (c) (x + 2)2 + (y – 3)2 = 18.75
2
AB + AD 2 AB + AD
(d) (x + 2)2 + (y + 3)2 = 18.75
AB ◊ AD 43. If two chords of the circle x2 + y2 – ax – by = 0 drawn
(c) AB ◊ AD (d)
2
AB - AD 2 from the point (a, b) is divided by the x-axis in the ratio
2 2
2 : 1 is
35. If the circle C1 : x + y = 16 intersects another circle (a) a2 > 3b2 (b) a2 < 3b2
C2 of radius 5 in such a manner that the common chord 2
(c) a > 4b 2
(d) a2 < 4b2
is of maximum length and has a slope 3/4, the co-ordi- 44. The angle at which the circles (x – 1)2 + y2 = 10 and
nates of the centre of C2 are x2 + (y – 2)2 = 5 intersect is
Ê 9 12 ˆ Ê 9 12 ˆ (a) p/6 (b) p/4 (c) p/3 (d) p/2
(a) Á ± , ± ˜ (b) ÁË ± , ˜
Ë 5 5¯ 5 5¯ 45. Two congruent circles with centres at (2, 3) and (5, 6)
Ê 12 9 ˆ Ê 12 9ˆ which intersect at right angles has radius equal to
(c) Á ± , ± ˜ (d) Á ± , ˜
Ë 5 5¯ Ë 5 5¯ (a) 2 2 (b) 3 (c) 4 (d) none
36. Two lines p1x + q1y + r1 = 0 and p2x + q2y + r2 = 0 are 46. A circle of radius unity is centred at origin. Two parti-
conjugate lines with respect to the circle x2 + y2 = a2 if cles start moving at the same time from the point (1, 0)
(a) p1p2 + q1q2 = r1r2 (b) p1p2 + q1q2 + r1r2 = 0 and moves around the circle in opposite direction. One
of the particle moves counter-clockwise with constant
(c) a (p1p2 + q1q2) = r1r2 (d) (p1p2 + q1q2) = a2r1r2
2
speed v and the other moves clockwise with constant
37. If a circle passing through the point (a, b) cuts the circle speed 3v. After leaving (1, 0), the two particles meet
x2 + y2 = k2 orthogonally, the equation of the locus of its first at a point P and continue until they meet next at
centre is point Q. Then the co-ordinates of the point Q are
(a) 2ax + 2by – (a2 + b2 + k2) = 0 (a) (1, 0) (b) (0, 1) (c) (0, –1) (d) (–1, 0)
(b) 2ax + 2by – (a2 – b2 – k2) = 0 47. The value of c for which the set {(x, y) : x2 + y2 + 2x £ 1}
(c) x2 + y2 – 3ax – 4by – (a2 + b2 – k2) = 0 « {(x, y) : x – y + c ≥ 0} contains only one point in com-
(d) x2 + y2 – 2ax – 3by – (a2 – b2 – k2) = 0. mon is
38. Consider the circle (a) (– , –1] » [3, ) (b) {–1, 3}
S : x2 + y2 – 4x – 4y + 4 = 0. (c) {–3} (d) {–1}
If another circle of radius r less than the radius of the 48. A circle is inscribed into a rhombus ABCD with one
circle S is drawn, touching the circle S, and the co-ordi- angle 60°. The distance from the centre of the circle to
nates axes, the value of r is the nearest vertex is equal to 1. If P be any point of the
(a) 3 - 2 2 (b) 4 - 2 2 circle, then |PA|2 + |PB|2 + |PC|2 + |PD|2 is equal to
(c) 7 - 4 2 (d) 6 - 4 2 (a) 12 (b) 11 (c) 9 (d) none
39. The distance between the chords of contact of tangents 49. P is a point (a, b) in the first quadrant. If the two circles
to the circle x2 + y2 + 2gx + 2fy + c = 0 from the origin which pass through P and touch both the co-ordinates
and the point (g, f) is axes cut at right angles, then

CG_03.indd 18 2/6/2017 4:00:13 PM


Circle 3.19

(a) a2 – 6ab + b2 = 0 (b) a2 + 2ab – b2 = 0 58. The triangle formed by the lines x + y = 0, x – y = 0 and
2 2
(c) a – 4ab + b = 0 (d) a2 – 8ab + b2 = 0 lx + my = l. If l amd m vary subject to the condition l2 +
50. The range of value of a such that the angle q between m2 = 1, the locus of its circumcentre is
the pair of tangents drawn from the point Q(a, 0) to the (a) (x2 – y2)2 = x2 + y2 (b) (x2 + y2)2 = x2 – y2
circle x2 + y2 = 1 satisfies p/2 < q < p is 2 2
(c) (x + y ) = 4x y 2 2
(d) (x2 + y2)2 = x2 – y2
(a) (1, 2) (b) (1, 2) 59. Tangents are drawn to a unit circle at the origin from
each point on the line 2x + y = 4. Then the equation to
(c) ( - 2, -1) (d) ( - 2, -1) » (1, 2)
the locus of the mid-point of the chord of contact is
51. Three concentric circles of which the biggest is x2 +
(a) 2(x2 + y2) = x + y (b) 2(x2 + y2) = x + 2y
y2 = 1 have their radii in AP. If the line y = x + 1 cuts 2 2
(c) 4(x + y ) = 2x + y (d) none
all the circles in real and distinct points the interval in
60. ABCD is a square of unit area. A circle is tangent to two
which the common difference of AP will lie is
sides of ABCD and passes through exactly one of its
1 Ê 1 ˆ
(a) ÊÁ 0, ˆ˜ (b) Á 0, vertices. The radius of the circle is
Ë 4¯ Ë 2 2 ˜¯
1 1
Ê ˆ (a) 2 - 2 (b) 2 - 1 (c) (d)
(c) Á 0, 2 - 2 ˜ (d) none 2 2
Ë 4 ¯ 61. A pair of tangents are drawn to a unit circle with centre
52. A tangent is a point on the circle x2 + y2 = a2 intersects a at the origin and these tangents intersect at A enclosing
concentric circle C at two points P and Q. The tangents an angle of 60°. The area enclosed by these tangents
to the circle C at P and Q meet at a point on the circle and the arc of the circle is
x2 + y2 = b2, the equation of the circle is 2 p p
(a) - (b) 3 -
(a) x2 + y2 = ab (b) x2 + y2 = (a – b)2 3 6 3
2 2 2
(c) x + y = (a + b) (d) x2 + y2 = a2 + b2
p 3 p
53. AB is the diameter of a semicircle k, C is an arbitrary (c) - (d) 3 ÊÁ1 - ˆ˜ .
3 6 Ë 6¯
point on the semicircle (other than A or B) and S is the
centre of the circle inscribed in triangle ABC, then the 62. Two circles are drawn through the points (1, 0) and
measure of (2, –1) to touch the axis of y. They intersect at angle
(a) –ASB changes as C moves on k Ê 3ˆ Ê 4ˆ
(a) cot -1 Á ˜ (b) cot -1 Á ˜
(b) –ASB = 135 for all C Ë 4¯ Ë 5¯
(c) –ASB = 150 for all C p –1
(c) (d) tan (1)
(d) –ASB is the same for all positions of C, but r can- 2
not be determined without knowing the radius. 63. If the line x cos q + y sin q = 2 is the equation of trans-
54. Tangents are drawn to the circle x2 + y2 = 1 at the verse common tangent to the circles x2 + y2 = 4 and
points, where it is met by the circles x2 + y2 – (l + 6)x + x2 + y2 – 6 3x – 6y + 20 = 0, the value of q is
(8 – 2l)y – 3 = 0, l being the variable. The locus of the (a) 5p/6 (b) 2p/3 (c) p/3 (d) p/6
point of intersection of these tangents is 64. A circle of constant radius a passes through the origin
(a) 2x – y + 10 = 0 (b) x + 2y – 10 = 0 O and cuts the axis of co-ordinates in points P and Q,
(c) x – 2y + 10 = 9 (d) 2x + y – 10 = 0 the equation of the locus of the foot of perpendicular
1 b from O to PQ is
55. Given + = 1 and ax + by = 1 are two variable lines
x y Ê 1 1ˆ
(a) ( x 2 + y 2 ) Á 2 + 2 ˜ = 4a 2
a and b being the parameters connected by the relation Ëx y ¯
a2 + b2 = ab. The locus of the point of intersection has
Ê 1 1ˆ
the equation (b) ( x 2 + y 2 ) 2 Á 2 + 2 ˜ = a 2
(a) x2 + y2 + xy – 1 = 0 (b) x2 + y2 – xy + 1 = 0 Ëx y ¯
(c) x2 + y2 + xy + 1 = 0 (d) x2 + y2 – xy – 1 = 0 Ê 1 1ˆ
56. B and C are two fixed points having co-ordinates (c) ( x 2 + y 2 ) 2 Á 2 + 2 ˜ = 4a 2
Ëx y ¯
(3, 0) and (–3, 0) respectively. If the vertical angle BAC
is 90°, the locus of the centroid of the DABC has the 2 2 Ê 1 1ˆ 2
(d) ( x + y ) Á 2 + 2 ˜ = a .
equation Ëx y ¯
(a) x2 + y2 = 1 (b) x2 + y2 = 2
2 2 65. A square is inscribed in the circle x2 + y2 – 2x + 4y + 3
(c) x + y = 1/9 (d) x2 + y2 = 4/9.
= 0. Its sides are parallel to the co-ordinate axes. Then
Ê 1 ˆ Ê 1ˆ Ê 1ˆ Ê 1ˆ
57. If Á1, ˜ ; Á b, ˜ ; Á c, ˜ and Á d , ˜ are four distinct one vertex of the square is
Ë a¯ Ë b¯ Ë c¯ Ë d¯
(a) (1 + 2, - 2) (b) (1 - 2, - 2)
points on a circle of radius 4 units, then abcd is equal to
(a) 4 (b) ¼ (c) 1 (d) 10 (c) (1, - 2 + 2) (d) none

CG_03.indd 19 2/6/2017 4:00:14 PM


3.20 Coordinate Geometry Booster

66. The point of contact of the tangent to the circle x2 + y2 (d) Cannot be calculated unless coincident points are
= 5 at the point (1, –2) which touches the circle x2 + y2 given
– 8x + 6y + 20 = 0, is 75. The equation of the circle, which touches both the axes
(a) (2, –1) (b) (3, –1) and the straight line 4x + 3y = 6 in the first quadrant and
(c) (4, –1) (d) (5, –1) lies below it is
67. The centre of the circle passing through the point (0, 1) (a) 4(x2 + y2) – 4x – 4y + 1 = 0
and touching the curve y = x2 at (2, 4) is (b) x2 + y2 – 6x – 6y + 9 = 0
(a) (–16/5, 27/10) (b) (–16/7, 53/10) (c) x2 + y2 – 6x – y + 9 = 0
(c) (–16/5, 53/10) (d) none (d) (x2 + y2 – x – 6y) + 4 = 0
68. The locus of the mid-points of the chords of the circle 76. Circles are drawn through the point (2, 0) to cut the
x2 + y2 – 2x – 6y – 10 = 0 which passes through the intercepts of length 5 units on the x-axis. If their centres
origin is the circle lie in the first quadrant, their equation is
(a) x2 + y2 + x + 3y = 0 (b) x2 + y2 – x + 3y = 0 (a) x2 + y2 – 9x + 2ky + 14 = 0
(c) x2 + y2 + x – 3y = 0 (d) x2 + y2 – x – 3y = 0 (b) 3x2 + 3y2 + 27x – 2ky + 42 = 0
69. The equation of the circle through the points of inter- (c) x2 + y2 – 9x – 2ky + 14 = 0
section of x2 + y2 = 1, x2 + y2 – 2x – 4y + 1 = 0 and (d) x2 + y2 – 2kx – 9y + 14 = 0,
touching the line x + 2y = 0 is where k is a positive real number.
(a) x2 + y2 + x + 2y = 0 (b) x2 + y2 – x + 2y = 0 77. If the tangent to the circle x2 + y2 = 5 at (1, –2) touches
(c) x2 + y2 – x – 2y = 0 (d) 2(x2 + y2) – x – 2y = 0 the circle x2 + y2 – 8x + 6y + 20 = 0 at the point
70. y – x + 1 = 0 is the equation of the normal at (a) (2, –1) (b) (3, –1)
Ê 3 3 ˆ (c) (4, –1) (d) (5, –1)
ÁË 3 + , ˜ to which of the following circles? 78. The centre of the circle passing through the point (0, 1)
2 2¯
2 2 and touching the curve y = x2 at (2, 4) is
Ê 3 ˆ Ê 3 ˆ
(a) Á x - 3 - ˜ + ÁË y - ˜ =9 (a) (–16/5, 27/10) (b) (–16/7, 53/10)
Ë 2¯ 2¯ (c) (–16/5, 53/10) (d) none
2
Ê 3 ˆ 2
79. If the lines 2x – 4y = 9 and 6x – 12y + 7 = 0 touch a
(b) Á x - 3 - ˜ + y =6 circle, the radius of the circle is
Ë 2¯
3 17 2 5 17
(c) (x – 3)2 + y2 = 6 (a) (b) (c) (d)
(d) (x – 3)2 + (y – 3)2 = 9 5 6 5 3 3 5
71. The radical axis of two circles whose centres lie along 80. If the co-ordinates at one end of a diameter of the circle
x and y axes is x2 + y2 – 8x – 4y + c = 0 are (–3, 2), the co-ordinates of
the other end are
Ê a 2 + b2 ˆ
(a) ax - by - Á =0
Ë 4 ˜¯
(a) (5, 3) (b) (6, 2) (c) (1, –8) (d) (11, 2)
81. If a circle is inscribed in an equilateral triangle of side
Ê g2 + f 2 ˆ a, the area of the square inscribed in the circle is
(b) 2 g 2 x + 2 f 2 y - Á ˜ =0 (a) a2/6 (b) a2/3 (c) 2a2/5 (d) 2a2/3
Ë 4 ¯
82. The equations of lines joining the origin to the point of
(c) gx + fy = g4 + f 4
intersection of circle x2 + y2 = 3 and the line x + y = 2
(d) 2gx – 2fy + g2 – f 2 = 0
is
72. A circle has its centre in the first quadrant and passes
through the points of intersection of the lines x = 2 and (a) y - (3 + 2 2) x = 0 (b) x - (3 + 2 2) y = 0
y = 3. If it makes intercepts of 3 and 4 units on these (c) x - (3 - 2 2) y = 0 (d) y - (3 - 2 2) x = 0
lines respectively, its equation is 83. Two circles x2 + y2 – 10x + 16 = 0 and x2 + y2 = r2 inter-
(a) x2 + y2 – 3x – 5y + 8 = 0 sect each other at two distinct points if
(b) x2 + y2 – 4x – 6y + 13 = 0 (a) r < 2 (b) r > 8
(c) x2 + y2 – 6x – 8y + 23 = 0 (c) 2 < r < 8 (d) 2 £ r £ 8
(d) x2 + y2 – 8x – 9y + 30 = 0 84. If the equations of the tangents drawn from the origin
73. The radius of the circle passing through the points to the circle x2 + y2 – 2rx – 2hy + h2 = 0 are perpendicu-
(1, 2), (5, 2) and (5, –2) is lar, then
(a) 5 2 (b) 2 5 (c) 3 2 (d) 2 2 (a) h = r (b) h2 = r2
(c) h = –r (d) r2 + h2 = 1
74. Lines are drawn from the point (–2, –3) to meet the 2 2
85. If a circle C and x + y = 1 are orthogonal and have
circle x2 + y2 – 2x – 10y + 1 = 0. The length of the line radical axis parallel to y-axis, then C is
that meets the circle at two coincident point is (a) x2 + y2 – 1 + x = 0 (b) x2 + y2 – 1 – x = 0
(a) 4 3 (b) 16 (c) 48 2 2
(c) x + y + 1 – y = 0 (d) x2 + y2 + 1 + x = 0

CG_03.indd 20 2/6/2017 4:00:14 PM


Circle 3.21

86. The equation of the line meeting the circle x2 + y2 = a2, 94. The equation of the tangents drawn from the origin to
two points at equal distances d from a point (x1, y1) on the circle x2 + y2 + 2rx – 2hy + h2 = 0 are
the circumference is (a) x = 0 (b) y = 0
1 (c) (h2 – r2)x – 2rhy = 0 (d) (h2 – r2)x + 2rhy = 0
(a) xx1 + yy1 - a 2 + 2 = 0
2d 95. If two circles (x – 1)2 + (y – 3)2 = r2 and x2 + y2 – 8x +
1 2y + 8 = 0 intersect in two distinct points, then
2
(b) xx1 - yy1 - a + 2 = 0 (a) 2 < r < 8 (b) r < 2
2d (c) r = 2 (d) r > 2
2 1 96. The lines 2x – 3y = 5 and 3x – 4y = 7 are diameters of
(c) xx1 + yy1 + a - 2 = 0
2d a circle of area 154 sq. units. Then the equation of the
2 1 circle is
(d) xx1 + yy1 - a - 2 = 0
2d (a) x2 + y2 + 2x – 2y – 62 = 0
87. The equations of the tangents drawn from the origin to (b) x2 + y2 + 2x – 2y – 47 = 0
the circle x2 + y2 – 2rx – 2hy + h2 = 0 are (c) x2 + y2 – 2x + 2y – 47 = 0
(a) x = 0 (b) y = 0 (d) x2 + y2 – 2x + 2y – 62 = 0
97. The centre of a circle passing through the points (0, 0),
(c) (h – r )x – 2rhy = 0 (d) (h2 – r2)x + 2rhy = 0
2 2
(1, 0) and touching the circle x2 + y2 = 9 is
88. Two circles x2 + y2 = 6 and x2 + y2 – 6x + 8 = 0 are giv- (a) (3/2, ½) (b) (1/2, 3/2)
en. Then the equation of the circle through their points
(c) (1/2, ½) (d) (1/2, 1/ 2)
of intersection and the point (1, 1) is
98. The locus of the centre of a circle which touches ex-
(a) x2 + y2 – 6x + 4 = 0 (b) x2 + y2 – 3x + 1 = 0
ternally the circle x2 + y2 – 6x – 6y + 14 = 0 and also
(c) x2 + y2 – 4x + 2 = 0 (d) none
touches the y-axis is given by the equation
89. The equation of the circle passing through (1, 1) and
(a) x2 – 6x – 10y + 14 = 0
the points of intersection of x2 + y2 + 13x – 3y = 0 and
(b) x2 – 10x – 6y + 14 = 0
2x2 + 2y2 + 4x – 7y – 25 = 0 is
(c) y2 – 6x – 10y + 14 = 0
(a) 4x2 + 4y2 – 30x – 10y – 25 = 0
(d) y2 – 10x + 6y + 14 = 0
(b) 4x2 + 4y2 + 30x – 13y – 25 = 0
99. The circles x2 + y2 – 10x + 16 = 0 and x2 + y2 = r2 inter-
(c) 4x2 + 4y2 – 17x – 10y – 25 = 0
sect each other in distinct points if
(d) none
(a) r < 2 (b) r > 8
90. The centre of the circle passing through the point (0, 1)
(c) 2 < r < 8 (d) 2 £ r £ 8
and touching the curve y = x2 at (2, 4) is
100. The angle between a pair of tangents drawn from a
Ê 16 27 ˆ Ê 16 53 ˆ point P to the circle x2 + y2 + 4x – 6y + 9 sin2a + 13
(a) Á - , ˜ (b) ÁË - , ˜¯
Ë 5 10 ¯ 7 10 cos2a = 0 is 2a. Then the locus of P is
Ê 16 53 ˆ (a) x2 + y2 + 4x – 6y + 4 = 0
(c) Á - , ˜ (d) none
Ë 5 10 ¯ (b) x2 + y2 + 4x – 6y – 9 = 0
91. AB is a diameter of a circle and C is any point on the (c) x2 + y2 + 4x – 6y – 4 = 0
circumference of the circle. Then (d) x2 + y2 + 4x – 6y + 9 = 0
(a) the area of the triangle ABC is maximum when it is 101. The number of common tangents to the circles x2 + y2
isosceles. = 4 and x2 + y2 – 6x – 8y – 24 = 0 is
(b) the area of the triangle ABC is minimum when it is (a) 0 (b) 1 (c) 3 (d) 4
isosceles. 102. Let A0A1A2A3A4A5 be a regular hexagon inscribed in a
(c) the area of the triangle ABC is minimum when it is unit circle. Then the product of the lengths of the line
isosceles. segments A0A1, A0A2 and A0A4 is
(d) None (a) 3/4 (b) 3 3
92. The locus of the mid-point of a chord of the circle (c) 3 (d) 3 3 /2
x2 + y2 = 4, which subtends a right angle at the origin is 103. If two distinct chords, drawn from the point (p, q) on
(a) x + y = 2 (b) x2 + y2 = 1 the circle x2 + y2 = px + qx, where pq π 0 are bisected
2 2
(c) x + y = 2 (d) x + y = 1 by the x-axis, then
93. If a circle passes through the point (a, b) and cuts the (a) p2 = q2 (b) p2 = 8q2
circle x2 + y2 = 2 orthogonally, then the equation of the 2
(c) p < 8q 2
(d) p2 > 8q2
locus of its centre is 104. The triangle PQR is inscribed in the circle x2 + y2 = 25.
(a) 2ax + 2by – (a2 + b2 + k2) = 0 If Q and R have co-ordinates (3, 4) and (–4, 3), respec-
(b) 2ax + 2by – (a2 – b2 + k2) = 0 tively, then –QPR is
(c) x2 + y2 – 3ax – 4by + (a2 + b2 – k2) = 0
(a) p/2 (b) p/3 (c) p/4 (d) p/6.
(d) x2 + y2 – 2ax – 3by + (a2 + b2 – k2) = 0

CG_03.indd 21 2/6/2017 4:00:15 PM


3.22 Coordinate Geometry Booster

105. If the circles x2 + y2 + 2x + 2ky + b = 0 and x2 + y2 + 2ky 115. Tangents drawn from the point P(1, 8) to the circle
+ k = 0 intersect orthogonally, then k is x2 + y2 – 6x – 4y – 11 = 0 touch the circle at A and B.
(a) 2 or –3/2 (b) – 2 or – 3/2 The equation of the circumcircle of the triangle PAB is
(c) 2 or 3/2 (d) – 2 or 3/2 (a) x2 + y2 + 4x – 6y + 19 = 0
106. Let AB be a chord of the circle x2 + y2 = r2 subtending (b) x2 + y2 – 4x – 10y + 19 = 0
right angle at the centre, the locus of the centroid of the (c) x2 + y2 – 2x + 6y – 29 = 0
triangle PAB as P moves on the circle is (d) x2 + y2 – 6x – 4y + 19 = 0
(a) a parabola (b) a circle 116. Two parallel chords of a circle of radius 2 are at a dis-
(c) an ellipse (d) a pair of straight lines p
tance 3 + 1 apart. If the chords subtend, angles of
107. Let PQ and RS be tangents at the extremities of the di- 2p k
ameter PR of a circle of radius r. If PS and RQ intersect and at the centre, where k > 0, the value of [k] is,
k
a point x on the circumference of the circle, then 2r where [, ] = GIF
equals (a) 1 (b) 2 (c) 3 (d) 4
PQ + RS 117. The circle passing through the point (–1, 0) and touch-
(a) PQ ◊ RS (b)
2 ing the y-axis at (0, 2), also passes through the point
2PQ + RS PQ 2 + RS 2 Ê 3 ˆ Ê 5 ˆ
(c) (d) (a) Á - , 0˜ (b) Á - , 2˜
PQ + RS 2 Ë 2 ¯ Ë 2 ¯
108. If the tangent at the point P on the circle x2 + y2 + 6x + Ê 3 5ˆ
(c) Á - , ˜ (d) (–4, 0)
6y = 2 meets the straight line 5x – 2y + 6 = 0 at a point Ë 2 2¯
Q on the y-axis, the length of PQ is
(a) 4 (b) 2 5 (c) 5 (d) 2 5 LEVEL III
109. If a > 2b > 0, the positive value of m for which (Problems for JEE Advanced)
2
y = mx - b 1 + m is a common tangent to x + y = b
2 2 2
1. Find the number of points with integral co-ordinates
and (x – a)2 + y2 = b2 is that are interior to the circle x2 + y2 = 16.
2b a 2 - 4b 2 2. If the circle x2 + y2 + 2gx + 2fy + c = 0 cuts each of the
(a) (b)
a 2 - 4b 2 2a circles x2 + y2 = 4, x2 + y2 – 6x – 8y + 10 = 0
2b b and x2 + y2 + 2x – 4y – 2 = 0 at the extremities of a
(c) (d) diameter, find the equation of the circle.
a - 2b a - 2b 3. The equations of four circles are (x ± a)2 + (y ± a)2 =
110. The centre of the circle inscribed in a square formed by
a2. Find the radius of a circle which touches all the four
the lines x2 – 8x + 12 = 0 and y2 – 14y + 45 = 0 is
circles.
(a) (4, 7) (b) (7, 4) (c) (9, 4) (d) (4, 9)
4. A square is inscribed in the circle x2 + y2 – 10x – 6y +
111. If one of the diameters of the circle x2 + y2 – 2x – 6y + 6
30 = 0. One side of the square is parallel to y = x + 3,
= 0 is a chord to the circle with centre (2, 1), the radius
then one vertex of the square is ... .
of the circle is
5. If a chord of the circle x2 + y2 = 8 makes equal inter-
(a) 3 (b) 2 (c) 3 (d) 2 cepts of length a on the co-ordinate axes, then |a| < ... .
112. A circle is given by x2 + (y – 1)2 = 1, another circle C 6. The equation of a circle and a line are x2 + y2 – 8x + 2y
touches it externally and also the x-axis, the locus of its + 12 = 0 and x – 2y – 1 = 0. Determine whether the line
centre is is a chord or a tangent or does not meet the circle at all.
(a) {(x, y) : x2 = 4y} » {(x, y) : y £ 0} 7. If the circles (x – a)2 + (y – b)2 = c2 and (x – b)2 +
(b) {(x, y) : x2 + (y – 1)2 = 4} » {(x, y) : y £ 0} (y – a)2 = c2 touch each other, find the value of a.
(c) {(x, y) : x2 = y} » {(0, y) : y < 0} 8. Find the locus of the mid-points of the chords of the
(d) {(x, y) : x2 = y} » {(0, y) : y £ 0} circle x2 + y2 + 4x – 6y – 12 = 0, which subtends an
113. The tangent to the curve y = x2 + 6 at a point P(1, 7) p
angle of radians at its circumference.
touches the circle x2 + y2 + 16x + 12y + c = 0 at a point 3
Q. Then the co-ordinates of Q are 9. Find the equation of a circle, which touches the axis of
(a) (–6, –7) (b) (–10, –15) y at (0, 3) and cuts an intercept of 8 units on the axis of
(c) (–9, –13) (d) (–6, –11) x.
114. Let ABCD be a quadrilateral with area 18, with side 10. Find the distance between the chord of contact of tan-
AB parallel to the side CD and AB = 2 CD. Let AD be a gents to the circle x2 + y2 + 2gx + 2fy + c = 0 from the
perpendicular to AB and CD. If a circle is drawn inside origin and the point (g, f).
the quadrilateral ABCD touching all the sides, then its 11. If (1 + ax)n = 1 + 8x + 24x2 + … and a line through
radius is P(a, n) cuts the circle x2 + y2 = 4 in A and B, then PA.
(a) 3 (b) 2 (c) 3/2 (d) 1 PB = ... .

CG_03.indd 22 2/6/2017 4:00:15 PM


Circle 3.23

12. The circle x2 + y2 = 4 cuts the circle x2 + y2 + 2x + 3y – 5 30. Prove that the locus of the point of intersection of tan-
= 0 in A and B, the centre of the circle AB as diameter gents to the circle x2 + y2 = a2 at the points whose para-
is ... . p 4a 2
metric angles differ by is x 2 + y 2 = .
13. Find the length of the tangents from any point of the 3 3
circle x2 + y2 + 2gx + 2fy + c = 0 to the circle x2 + y2 + 31. Prove that the area of the triangle formed by positive
2gx + 2fy + d = 0, (d > c). axis and the normal and the tangent to the circle x2 + y2
14. Find the equation of the circle whose diameter is the = 4 at (1, 3) is 2 3.
chord x + y = 1 of the circle x2 + y2 = 4.
15. Find the equation of the image of the circle (x – 3)2 +
(y – 2)2 = 1 by the line mirror x + y = 19. LEVEL IV
16. Two circles (x – 1)2 + (y – 3)2 = r2 and x2 + y2 – 8x + 2y (Tougher Problems for JEE
+ 8 = 0 intersect in two distinct points, prove that 2 < r Advanced)
< 8.
17. If exactly two real tangents can be drawn to the circles 1. A circle of diameter 13 m with centre O coinciding
x2 + y2 – 2x – 2y = 0 and x2 + y2 – 8x – 8y + l = 0, prove with the origin of co-ordinates axes has diameter AB
that 0 < l < 24. on the x-axis. If the length of the chord AC be 5 m, find
18. Two vertices of an equilateral triangle are (–1, 0) the following:
and (1, 0) and its third vertex lies above the x-ax- (i) Equations of the pair of lines BC and BC¢.
is. Prove that the equation of the circumcircle is (ii) The area of the smaller portion bounded between
2 the circle and the chord AC.
x2 + y 2 - y -1= 0 . [Roorkee, 1983]
3
19. Find the equation of the circumcircle of the triangle 2. A circle I of radius 5 m is having its centre A at the ori-
gin of the co-ordinate axes. Two circles II and III with
formed by the lines y + 3 x = 6, y - 3 x = 6 and y
centres at B and C and radii 3 and 4 m, respectively,
= 0.
touch the circle I and also touch the x-axis to the right
20. If the equation of incircle of an equilateral triangle is
of A. Find the equations of any two common tangents
x2 + y2 + 4x – 6y + 4 = 0, prove that the equation of the
to the circles II and III. [Roorkee, 1983]
circumcircle of the triangle is x2 + y2 + 4x – 6y – 23 = 0.
3. Find the equation of a circle which is co-axial with the
21. If a square is inscribed in the circle x2 + y2 + 2gx + 2fy
circles 2x2 + 2y2 – 2x + 6y – 3 = 0 and x2 + y2 + 4x + 2y
+ c = 0 of radius r, prove that the length of its side is
+1=0 [Roorkee, 1984]
r 2. 4. Find the condition such that the four points in which
22. If r be the radius, prove that the area of the equilateral the circles x2 + y2 + ax + by + c = 0 and x2 + y2 + a¢x
triangle inscribed in the circle x2 + y2 + 2gx + 2fy + c = + b¢y + c¢ = 0 are intersected by the straight lines Ax +
3 3 By + C = 0 and A¢x + B¢y + C¢ = 0, respectively lie on
0 is ¥ r2 .
4 another circle. [Roorkee, 1986]
23. Prove that the equation of a circle with centre at the 5. Obtain the equation of the straight lines passing through
origin and passing through the vertices of an equilateral the point A(2, 0) and making an angle of 45° with the
triangle whose median is of length 3a is x2 + y2 = 4a2. tangent A to the circle (x + 2)2 + (y + 3)2 = 25.
24. A circle is inscribed in an equilateral triangle of side a. Find the equations of the circles each of radius 3 whose
Prove that the area of any square inscribed in the circle centres are on these straight lines at a distance of 5 2
is (a2, 6). from A. [Roorkee, 1987]
25. Find the co-ordinates of the point on the circle x2 + y2 – 6. A circle has radius 3 units and its centre lies on the line
12x + 4y + 30 = 0 which is farthest from the origin. y = x – 1. Find the equation of this circle if it passes
26. A diameter of x2 + y2 – 2x – 6y + 6 = 0 is a chord to the through (7, 3). [Roorkee, 1988]
circle with centre (2, 1), find the radius of it. 7. Find the equation of the circles passing through the
27. Prove that angle between two tangents from the origin point (2, 8) touching the lines 4x – 3y – 24 = 0 and
p 4x + 3y – 42 = 0 and having x co-ordinate of the centre
to the circle (x – 7)2 + (y + 1)2 = 25 is .
2 of the circle less than or equal to 8.
28. Prove that the tangents are drawn from the point (4, 3) [Roorkee, 1989]
to the circle x2 + y2 – 2x – 4y = 0 are inclined at an angle 8. The abscissa of two points A and B are the roots of
p the equation x2 + 2x – a2 = 0 and the ordinates are the
is . roots of the equation y2 + 4y – b2 = 0. Find the equation
2
of the circle with AB as its diameter. Also find the co-
29. Find the number of tangents that can be drawn from the
ordinates of the centre and the length of the radius of
point (0, 1) to the circle x2 + y2 – 2x – 4y = 0.
the circle. [Roorkee, 1989]

CG_03.indd 23 2/6/2017 4:00:16 PM


3.24 Coordinate Geometry Booster

9. The point of contact of the tangent to the circle x2 + y2 17. The radius of the circle passing through the points
= 5 at the point (1, –2) which touches the circle x2 + y2 (1, 2), (5, 2) and (5, –2) is
– 8x + 6y + 20 = 0, is (a) 5 2 (b) 2 5 (c) 3 2 (d) 2 2
(a) (2, –1) (b) (3, –1) [Roorkee, 1991]
(c) (4, –1) (d) (5, –1) 18. Find the equations of the circle passing through the
[Roorkee, 1989] points A(4, 3) and B(2, 5) and touching the axis of y.
10. The centre of the circle passing through the point (0, 1) Also find the point P on the y-axis such that the angle
and touching the curve y = x2 at (2, 4) is APB has largest magnitude.
(a) (–16/5, 27/10) (b) (–16/7, 53/10) [Roorkee Main, 1991]
(c) (–16/5, 53/10) (d) none 19. Find the radius of the smallest circle which touches the
11. The locus of the mid-points of the chords of the circle straight line 3x – y = 6 at (1, –3) and also touches the
x2 + y2 – 2x – 6y – 10 = 0 which passes through the line y = x. Compute up to one place of decimal only.
origin is the circle [Roorkee Main, 1991]
(a) x2 + y2 + x + 3y = 0 (b) x2 + y2 – x + 3y = 0 20. Lines are drawn from the point (–2, –3) to meet the
(c) x2 + y2 + x – 3y = 0 (d) x2 + y2 – x – 3y = 0 circle x2 + y2 – 2x – 10y + 1 = 0. The length of the line
[Roorkee, 1989] that meets the circle at two coincident points is
12. The equation of the circle through the points of inter- (a) 4 3 (b) 16
section of x2 + y2 = 1 and x2 + y2 – 2x – 4y + 1 = 0 and (c) 48
touches the line x + 2y = 0 is (d) Cannot be calculated unless coincident points are
(a) x2 + y2 + x + 2y = 0 (b) x2 + y2 – x + 2y = 0 given [Roorkee, 1992]
(c) x2 + y2 – x – 2y = 0 (d) 2(x2 + y2) – x – 2y = 0
[Roorkee, 1989] 21. The equation of the circle which touches both the axes
13. y – x + 3 = 0 is the equation of the normal at and the straight line 4x + 3y = 6 in the first quadrant and
Ê 3 3 ˆ lies below it, is
ÁË 3 + , ˜ to which of the following circles? (a) 4(x2 + y2) – 4x – 4y + 1 = 0
2 2¯
2 2
(b) x2 + y2 – 6x – 6y + 9 = 0
Ê 3 ˆ Ê 3 ˆ (c) 49(x2 + y2) – 420(x + y) + 900 = 0
(a) Á x - 3 - ˜ + ÁË y - ˜ =9
Ë 2¯ 2¯ (d) x2 + y2 – x – 6 + 4 = 0 [Roorkee, 1992]
2 22. Circles are drawn through the point (2, 0) to cut inter-
Ê 3 ˆ 2
(b) Á x - 3 - ˜ + y =9 cepts of length 5 units on the x-axis. If their centres lie
Ë 2¯ in the first quadrant, their equation is
(c) (x – 3)2 + y2 = 6 (a) x2 + y2 – 9x + 2ky + 14 = 0
(d) (x – 3)2 + (y – 3)2 = 9 [Roorkee, 1990] (b) 3x2 + 3y2 + 27x – 2ky + 42 = 0
14. The radical axis of two circles whose centres lie along (c) x2 + y2 – 9x – 2ky + 14 = 0
x and y axes is (d) x2 + y2 – 2kx – 9y + 14 = 0
Ê a 2 + b2 ˆ where k is a positive real number. [Roorkee, 1992]
(a) ax - by - ÁË ˜ =0 23. From a point P, tangents are drawn to the circles x2 + y2
4 ¯
+ x – 3 = 0, 3x2 + 3y2 – 5x + 3y = 0 and 4x2 + 4y2 + 8x
Ê f 2 + g2 ˆ
(b) 2g 2 x + 2f 2 y - Á ˜ =0
+ 7y + 9 = 0 are of equal lengths. Find the equation of
Ë 4 ¯ the circle through P which touches the line x + y = 5 at
(c) gx + fy = g4 + f 4 the point (6, –1). [Roorkee Main, 1992]
(d) 2gx – 2fy + g2 – f2 = 0 [Roorkee, 1990] 24. Find the equation of the system of co-axial circles that
15. Find the equations of the circle having the lines x + are the tangent at ( 2, 4) to the locus of the point of
2xy + 3x + 6y = 0 as its normals and having size just intersection of mutually perpendicular tangents to the
sufficient to contain the circle x(x – 4) + y(y – 3) = 0. conic x2 + y2 = 9. [Roorkee Main, 1993]
[Roorkee Main, 1990] 25. If the tangent to the circle x2 + y2 = 5 at (1, –2) touches
16. A circle has its centre in the first quadrant and passes the circle x2 + y2 – 8x + 6y + 20 = 0 at the point
through the points of intersection of the lines x = 2 and (a) (2, –1) (b) (3, –1)
y = 3. If it makes intercepts of 3 and 4 units on these (c) (4, –1) (d) (5, –1)
lines respectively, its equation is [Roorkee, 1994]
(a) x2 + y2 + 3x – 5y + 8 = 0 26. The centre of the circle passing through the point (0, 1)
(b) x2 + y2 – 4x – 6y + 13 = 0 and touching the curve y = x2 at (2, 4) is
(c) x2 + y2 – 6x – 8y + 23 = 0 (a) (–16/5, 27/10) (b) (–16/7, 53/10)
(d) x2 + y2 – 8x – 9y + 30 = 0 [Roorkee, 1991] (c) (–16/5, 53/10) (d) none
[Roorkee, 1994]

CG_03.indd 24 2/6/2017 4:00:16 PM


Circle 3.25

27. Find the equation of the circle which touches the circle 37. The equation of the line meeting the circle x2 + y2 = a2,
x2 + y2 – 6x + 6y + 17 = 0 externally and to which the two points at equal distances d from a point (x1, y1) on
lines x2 – 3xy – 3x + 9y = 0 are normal. the circumference is
[Roorkee Main, 1994] 2 1
(a) xx1 + yy1 - a + 2 = 0
28. If the lines 2x – 4y = 9 and 6x – 12y + 7 = 0 touch a 2d
circle, the radius of the circle is 1
2
3 17 17 (b) xx1 - yy1 - a + 2 = 0
(a) (b) (c) 2 5 (d) 2d
5 6 5 3 3 5 1
[Roorkee, 1995] (c) xx1 + yy1 + a 2 - 2 = 0
2d
29. If the co-ordinates at one end of a diameter of the circle 1
x2 + y2 – 8x – 4y + c = 0 are (–3, 2), the co-ordinates of (d) xx1 + yy1 - a 2 + 2 = 0
the other end are 2d [Roorkee, 1998]
(a) (5, 3) (b) (6, 2) 38. The equations of the tangents drawn from the origin to
(c) (1, –8) (d) (11, 2) the circle x2 + y2 + 2rx – 2hy + h2 = 0 are
[Roorkee, 1995] (a) x = 0 (b) y = 0
30. If a circle is inscribed in an equilateral triangle of side (c) (h2 – r2)x – 2rhy = 0 (d) (h2 – r2)x + 2rhy = 0
a, the area of the square inscribed in the circle is [Roorkee, 1998]
39. Find the equation of a circle which touches the
a2 a2 2a 2 2a 2
(a) (b) (c) (d) line x + y = 5 at the point (–2, 7) and cuts the circle
6 3 5 3 x2 + y2 + 4x – 6y + 9 = 0 orthogonally.
[Roorkee, 1995] [Roorkee Main, 1998]
31. The equations of lines joining the origin to the point of 40. Extremities of a diagonal of a rectangle are (0, 0) and
intersection of circle x2 + y2 = 3 and the line x + y = 2 is (4, 3). Find the equations of the tangents to the circum-
(a) y - (3 + 2 2) x = 0 (b) x - (3 + 2 2) y = 0 circle of the rectangle which are parallel to this diago-
nal. [Roorkee Main, 2000]
(c) x - (3 - 2 2) y = 0 (d) y - (3 - 2 2) x = 0 41. Find the point on the straight line y = 2x + 11 which is
[Roorkee, 1995] nearest to the circle 16(x2 + y2) + 32x – 8y – 50 = 0.
32. From a point on the line 4x – 3y = 6, tangents are drawn [Roorkee Main, 2000]
to the circle x2 + y2 – 6x – 4y + 4 = 0 which make an an- 42. A circle of radius 2 units rolls on the outer side of the
-1 Ê 24 ˆ circle x2 + y2 + 4x = 0, touching it externally. Find the
gle of tan Á ˜ between them. Find the co-ordinates
Ë 7¯ locus of the centre of this outer circle. Also find the
of all such points and the equations of tangents. equations of the common tangents of the two circles
[Roorkee Main, 1995] when the line joining the centres of the two circles
33. Two circles x + y – 10x + 16 = 0 and x2 + y2 = r2 inter-
2 2 makes an angle of 60° with x-axis.
sect each other at two distinct points if [Roorkee Main, 2000]
(a) r < 2 (b) r > 8 43.
(c) 2 < r < 8 (d) 2 £ r £ 8 P
[Roorkee, 1996]
34. A tangent is drawn from the point (4, 0) to the circle T C(h, k)
x2 + y2 = 8 touches at a point A in the first quadrant. R
Find the co-ordinates of another point B on the circle
such that AB = 4. [Roorkee Main, 1996]
35. If the equations of the tangents drawn from the origin px + qy = r Q
to the circle x2 + y2 – 2rx – 2hy + h2 = 0 are perpendicu-
lar, then Tangents TP and TQ are drawn from a point T to the
circle x2 + y2 = a2. If the point T lies on the line px + qy
(a) h = r (b) h2 = r2
= r, find the locus of the centre of the circumference of
(c) h = –r (d) h2 + r2 = 1
the triangle TPQ.
[Roorkee, 1997] [Roorkee Main, 2001]
36. If a circle C and x2 + y2 = 1 are orthogonal and have 44. Find the equation of the circle which passes through
radical axis parallel to y-axis, then C is the points of intersection of circles x2 + y2 – 2x – 6y + 6
(a) x2 + y2 – 1 + x = 0 (b) x2 + y2 – 1 – x = 0 = 0 and x2 + y2 + 2x – 6y + 6 = 0 and intersects the circle
2 2
(c) x + y + 1 – y = 0 (d) x2 + y2 + 1 + x = 0 x2 + y2 + 4x + 6y + 6 = 0 orthogonally.
[Roorkee, 1997] [Roorkee Main, 2001]

CG_03.indd 25 2/6/2017 4:00:16 PM


3.26 Coordinate Geometry Booster

3. If 16l2 + 9m2 = 24lm + 6l + 8m + 1 and if S be the equa-


Integer Type Questions
tion of the circle having lx + my + 1 = 0 as a tangent,
1. Find the number of common tangents between the cir- when the equation of director circle of S is
cles x2 + y2 = 10 and x2 + y2 – 6x – 8y = 0. (a) x2 + y2 + 6x + 8y = 25
2. If the equations x2 + y2 + 2x + 2ky + 6 = 0 and x2 + y2 + (b) x2 + y2 – 6x + 8y = 25
2ky + k = 0 intersect orthogonally, find the number of (c) x2 + y2 – 6x – 8y = 25
values of k. (d) x2 + y2 + 6x – 8y = 25
Ê 1ˆ
3. If Á mi , ˜ , i = 1, 2, 3, 4 are four distinct points on a Passage II
Ë mi¯ A circle C of radius 1 is inscribed in an equilateral triangle
circle, find the value of (m1m2m3m4 + 4). PQR. The points of contact of C with the sides PQ, QR, RP
4. If the circumference of the circle x2 + y2 – 2x + 8y – q are D, E, F respectively. The line PQ is given by the equation
= 0 is bisected by the circle x2 + y2 + 4x + 22y + p = 0, Ê 3 3 3ˆ
3 x + y = 6 and the point D is Á ,
Ë 2 2 ˜¯
. Further it is giv-
Ê p+q ˆ
find the value of Á + 2˜ .
Ë 10 ¯ en that the origin and the centre C are on the same side PQ.
5. If the two circles (x – 1)2 + (y – 3)2 = r2 and x2 + y2 – 8x 1. The equation of the circle C is
2 2
+ 2y + 8 = 0 intersect in two distinct points such that n (a) ( x - 2 3) + ( y - 1) = 1
< r < m where m, n ΠN, find the value of (m Рn). 2
2 Ê 1ˆ
6. If a straight line through P ( - 2 2, 2 2) making an (b) ( x - 2 3) + ÁË y - ˜¯ = 1
2
angle of 135° with x-axis cuts the circle x = 4 cos q, 2 2
y = 4 sin q in points A and B respectively, find the (c) ( x - 3) + ( y + 1) =1
2 2
length of the segment AB. (d) ( x - 3) + ( y - 1) = 1
7. If a circle passes through the point of intersection of the 2. Points E and F are given by
co-ordinate axes with the lines lx – y + 1 = 0 and x – 2y Ê 3 3ˆ Ê 3 1ˆ
+ 3 = 0, find the integral value of l. (a) Á , , ( 3, 0) , , ( 3, 0)
Ë 2 2 ˜¯
(b) Á
Ë 2 2 ˜¯
8. Find the radius of the circumcircle of the triangle formed
by the lines y + 3 x = 6, y - 3 x = 6 and y = 0 . Ê 3 3ˆ Ê 3 1ˆ Ê 3 3 ˆ Ê 3 1ˆ
(c) Á , , , (d) Á , , ,
9. If the circle x2 + y2 – 4x – 6y + l = 0 touches the axis of Ë 2 2 ˜¯ ÁË 2 2 ˜¯ Ë 2 2 ˜¯ ÁË 2 2 ˜¯
x, find the value of l. 3. The equation of the sides QR, PR are
10. If the straight lines y = m1x + c1 and y = m2x + c2 meet 2
the co-ordinate axes in concyclic points, find the value (a) y = x + 1, y = - 2 3 x + 1
3
of (m1m2 + 4).
1
(b) y = x + 1; y = 0
Comprehensive Link Passage 3
Ê 3ˆ Ê 3ˆ
Passage 1 (c) y = ÁË ˜¯ x + 1; y = ÁË - ˜¯ x - 1
If 7l2 – 9m2 + 8l + 1 = 0 and we have to find the equation of 2 2
circle having lx + my + 1 = 0 is a tangent and we can adjust (d) y = 3 x, y = 0
the given condition as
Passage III
16l2 + 8l + 1 = 9(l2 + m2) or (4l + 1)2 = 9(l2 + m2) The equation of the tangent and the normal to the circle x2 +
(4l + 1) y2 + 2gx + 2fy + c = 0 at (x1, x2) are
fi =3
(l 2 + m 2 ) tangent : xx1 + yy1 + g(x + x1) + f(y + y1) + c = 0
x - x1 y - y1
Thus centre of a circle = (4, 0) and radius = 3. Normal: = .
x1 + g y1 + f
Also when two non-parallel lines touching a circle, the centre
of circle lies on angle bisector of lines. Clearly, the normal always passes through the centre of the
1. If 16m2 – 8l – 1 = 0, the equation of a circle having lx + circle.
my + 1 = 0 is a tangent is 1. The equation of the tangent to the circle x2 + y2 + 4x +
(a) x2 + y2 + 8x = 0 (b) x2 + y2 – 8x = 0 6y – 12 = 0 at (1, 1) is
2 2
(c) x + y + 8y = 0 (d) x2 + y2 – 8y = 0 (a) 3x + 4y = 7 (b) 3x – 4y = 7
2 2 (c) –3x + 4y = 7 (d) –3x – 4y = 7
2. If 4l – 5m + 6l + 1 = 0, the centre and the radius of the
circle, which have lx + my + 1 = 0 as a tangent, is 2. The tangent to the circle x2 + y2 = 5 at (1, –2) also
(a) (0, 4); 5 (b) (4, 0); 5 touches the circle
(a) x2 + y2 – 8x + 6y – 20 = 0
(c) (0, 3); 5 (d) (3, 0); 5 (b) x2 + y2 + 8x + 6y – 20 = 0

CG_03.indd 26 2/6/2017 4:00:17 PM


Circle 3.27

(c) x2 + y2 – 8x + 6y + 20 = 0 6. The chord of contact of the tangents drawn from a point


(d) x2 + y2 – 8x + 9y + 20 = 0 on the circle x2 + y2 = a2 to the circle x2 + y2 = b2 touch-
3. The area of the triangle formed by the positive x-axis, es the circle x2 + y2 = c2. Then a, b, c are in
the normal and the tangent to the circle x2 + y2 = 4 at (a) AP (b) GP (c) HP (d) AGP
(1, 3) is Passage V
(a) 3 3 s.u. (b) 2 3 s.u. The line y = mx + c will be a tangent to the circle x2 + y2 = a2
(c) 4 3 s.u. (d) 5 3 s.u.
if c = ± a 1 + m 2 .
4. The extremities of a diagonal of a rectangle are (–4, 4)
The equation of any tangent to the circle x2 + y2 = a2 can
and (6, –1). A circle circumscribes the rectangle and
2
cuts an intercept AB on the y-axis. The area of the tri- be considered as y = mx + a 1 + m and the co-ordinates of
angle formed by AB and the tangents to the circle at A
Ê am a ˆ
and B is the points of contact are Á ± ,
˜ and the
363 365 Ë 1 + m2 1+ m ¯ 2
(a) s.u. (b) s.u.
8 8 length of the tangent from the point P(x1, y1) to the circle x2 +
363 365
(c) s.u. (d) s.u. y2 + 2gx + 2fy + c = 0 is x12 + y12 + 2gx1 + 2fy1 + c .
4 4
5. The equation of the normal to the circle x2 + y2 – 5x + 1. The equation of tangent to the circle x2 + y2 + 4x + 2y
2y – 48 = 0 at the point (5, 6) is = 0 from the point P(1, –2) is
(a) 14x + 5y = 10 (b) 14x – 5y = 40 (a) x – 2y – 5 = 0 (b) x + 2y + 5 = 0
(c) 5x – 14y = 40 (d) none (c) x – 2y = 0 (d) y – 3x = 0
6. If the normal to the circle x2 + y2 – 6x + 4y – 50 = 0 is 2. The equation of the tangents from the origin to the
parallel to the line 3x + 4y + 5 = 0, the equation of the circle x2 + y2 – 2x – 4y = 0 is
normal is (a) 3x – 4y = 0 (b) 4x – 3y = 0
(a) 3x + 4y + 1 = 0 (b) 3x + 4y = 2
(c) 3x + 4y = 0 (d) 4x + 3y = 0.
(c) 3x – 4y = 1 (d) 3x + 4y + 2 = 0
3. The length of the tangent from any point on the circle
Passage IV x2 + y2 – 2009x – 2010y + 2012 = 0 to the circle x2 +
Let P(x1, y1) be a point lying inside the circle S : x2 + y2 + 2gx y2 – 2009x – 2010y + 2020 = 0 is
+ 2fy + c = 0. If the tangents from P to the circle S = 0 at A
and B, then AB is called the chord of contact. (a) 2 2 (b) 3 2 (c) 5 2 (d) 2
The equation of the chord of contact is 4. If the angle between a pair of tangents from a point P
xx1 + yy1 + g(x + x1) + f(y + y1) + c = 0. to the circle x2 + y2 + 4x – 6y + 13 cos2a + 9 sin2a = 0
Let the tangents PA and PB are drawn from P(0, –2) to the is 2a. Then the equation of the locus of P is
circle x2 + y2 + 2x – 4y = 0. (a) (x + 2)2 + (y – 3)2 = 1
1. The equation of the chord of contact is (b) (x – 2)2 + (y – 3)2 = 1
(a) x – 4y + 4 = 0 (b) x – 3y – 3 = 0 (c) (x – 2)2 + (y + 3)2 = 1
(c) x + 4y + 4 = 0 (d) x + 5y + 6 = 0. (d) (x + 2)2 + (y + 3)2 = 1
2. The length of the chord AB is 5. Tangents PA and PB are drawn from P(–1, 2) to the
circle x2 + y2 – 2x – 4y + 2 = 0. The area of a triangle
60 65 12 37
(a) 2 (b) 2 (c) 2 (d) 2 PAB is
17 17 17 17
3 3 2 3 4 3
3. The area of a triangle PAB is (a) (b) (c) (d)
4 2 5 5
5 7 11 5
(a) 12 (b) 12 (c) 12 (d) 6 Passage VI
17 17 17 17 The equation of the chord of the circle x2 + y2 = a2 bisected at
4. The area of a quadrilateral PACB, where C is the centre the point (x1, y1) is T = S1, i.e. xx1 + yy1 - a 2 = x12 + y12 - a 2 .
of the circle, is 1. The equation of the chord of the circle x2 + y2 – 6x +
(a) 2 15 (b) 3 15 (c) 4 15 (d) 5 15 10y – 9 = 0 bisected at the point (–2, 4) is
5. The angle between PA and PB is (a) 5x – 9y + 40 = 0 (b) 5x – 9y + 46 = 0
(c) 3x – 4y + 46 = 0 (d) 4x – 5y + 46 = 0.
Ê 4 15 ˆ Ê 8 15 ˆ
(a) tan -1 Á (b) tan -1 Á
Ë 7 ˜¯ Ë 7 ˜¯
2. The locus of the mid-point of a chord of the circle
x2 + y2 = 4, which subtends a right angle at the origin is
Ê 16 15 ˆ Ê 6 15 ˆ (a) x + y = 1 (b) x + y = 2
(c) tan -1 Á (d) tan -1 Á
Ë 7 ˜¯ Ë 7 ˜¯ (c) x2 + y2 = 2 (d) x2 + y2 = 1

CG_03.indd 27 2/6/2017 4:00:18 PM


3.28 Coordinate Geometry Booster

3. The equation of the locus of the mid-points of the 4. The equation of the circle whose diameter is the com-
chords of the circle 4x2 + 4y2 – 12x + 4y + 1 = 0 mon chord of the circles x2 + y2 + 2x + 3y + 1 = 0 and
2p x2 + y2 + 4x + 3y + 2 = 0 is
that subtends an angle of at its centre is
3 (a) 2(x2 + y2) + 2x + 6y + 1 = 0
(a) 16(x2 + y2) – 48x + 16y + 31 = 0 (b) 2(x2 + y2) + 3x + 6y + 1 = 0
(b) 16(x2 + y2) – 16x + 48y + 31 = 0 (c) 2(x2 + y2) + 2x + 5y + 1 = 0
(c) 16(x2 + y2) – 16x – 48y + 31 = 0 (d) 2(x2 + y2) + x + 6y + 1 = 0
(d) 16(x2 + y2) – 16x – 48y – 31 = 0
4. If two distinct chords, drawn from the point (p, q) on Matrix Match
the circle x2 + y2 = px + qy (where pq π 0) are bisected (For JEE-Advanced Examination Only)
by the x-axis, then
(a) p2 = q2 (b) p2 = 8q2 1. Match the following columns:
2 2
(c) p > 8q (d) p2 < 8q2 Column I Column II
5. Let a circle be given by 2x(x – a) + 2y(y – b) = 0, (A) If the shortest and the lon- (P) L + M = 10
(a, b π 0). If two chords are bisected by the x-axis, can gest distance from the point
Ê bˆ (10, 7) to the circle
be drawn to the circle from the point Á a, ˜
Ë 2¯ x2 + y2 – 4x – 2y – 2 = 0 are
2
(a) a > b 2
(b) a > 2b2
2 L and M respectively, then
2
(c) a < 2b 2
(d) a2 < b2 (B) If the shortest and the lon- (Q) L + M = 20
6. The locus of the mid-points of the chords of the circle gest distance from the point
x2 + y2 = a2, which subtend right angle at the point (3, –6) to the circle x2 + y2
(c, 0) is – 16x – 12y – 125 = 0 are L
(a) 2(x2 + y2) – 2cx = a2 – c2 and M respectively, then
(b) 2(x2 + y2) – 2cx = a2 + c2 (C) If the shortest and the lon- (R) L + M = 30
(c) 2(x2 + y2) + 2cx = a2 + c2 gest distance from the point (S) M – L = 10
(d) 2(x2 + y2) + 2cx = a2 – c2 (6, –6) to the circle (T) M – L = 26
Passage VII x2 + y2 – 4x + 6y – 12 = 0 are
The equation of the family of circles passing through the L and M respectively, then
point of intersection of two given circles S1 = 0 and S2 = 0 is 2. Match the following columns:
given by S1 + lS2 = 0, l π –1, where l is a parameter. Column I Column II
The equation of the family of circles passing through the
(A) The radius of the circle (P) 2
point of intersection of circle S = 0 and a line L = 0 is given
x2 + y2 – 2x – 2y = 0 is
by S + lL = 0, where l is a parameter. The equation of the
family of circles touching the circle S = 0 and the line L = 0 at (B) The radius of the circle (Q) 4
their point of contact P is S + lL = 0, where l is a parameter. x2 + y2 – 4x – 4y + 4 = 0 is
(C) The radius of the circle (R)
1. The equation of the circle passing through (1, 1) and 2
the points of intersection of the circles x2 + y2 + 13x – x2 + y2 – 6x – 10y + 30 = 0 is
3y = 0 and 2(x2 + y2) + 4x – 7y – 25 = 0 is (D) The radius of the circle (S) 3
(a) 4(x2 + y2) + 30x – 13y = 25 2(x2 + y2) – 4x – 6y + 16 = 0 is
(b) 4(x2 + y2) – 30x – 13y = 25 (E) The radius of the circle (T) 5
x2 + y2 – 10x = 0 is
(c) 4(x2 + y2) – 30x + 13y = 25
(d) 4(x2 + y2) + 30x + 13y = 25 3. Match the following columns:
2. The equation of the circle passing through the point of Column I Column II
intersection of the circles x2 + y2 – 6x + 2y + 4 = 0, x2 + y2 (A) The point (l, l + 2) lies inside (P) –1
+ 2x – 4y – 6 = 0 and with its centre on the line y = x is the circle x2 + y2 = 4, the value
(a) 7(x2 + y2) – 10(x + y) = 12 of l can be
(b) 7(x2 + y2) – 10(x – y) = 12 (B) The point (l, l + 2) lies outside (Q) –1/2
(c) 7(x2 + y2) + 10(x – y) = 12 the circle x2 + y2 – 2x – 4y = 0,
(d) 7(x2 + y2) + 10(x + y) = 12 the value of l can be
3. The equation of the circle through the points of inter- (C) If both the equations (R) 3/2
section of the circle x2 + y2 – 2x – 4y + 4 = 0 and the line x2 + y2 + 2ll + 4 = 0 (S) 3
x + 2y = 4 which touches the line x + 2y = 0 is and x2 + y2 – 4ll + 8 = 0 rep- (T) 5
(a) x2 + y2 – x – 2y = 0 (b) x2 + y2 + x – 2y = 0 resent real circles, the value of
(c) x2 + y2 + x + 2y = 0 (d) x2 + y2 – x + 2y = 0 l can be

CG_03.indd 28 2/6/2017 4:00:18 PM


Circle 3.29

4. Match the following columns: 7. Match the following columns:


Column I Column II
Column I Column II
(A) Number of common tangents (P) 1
(A) If the straight lines y = a1x + b (P)
a12 + a22 = 4 to the circles x2 + y2 – 2x = 0
and y = a2x + b, (a1 π a2) and and x2 + y2 + 6x – 6y + 2 = 0 is
b ΠR meet the co-ordinate
axes in concyclic points, then (B) Number of common tangents (Q) 2
to the circles
(B) If the chord of contact of the (Q) a1 + a2 = 3
x2 + y2 – 4x – 10y + 4 = 0 and
tangents drawn from any (R) a1a2 = b x2 + y2 – 6x – 12y – 55 = 0 is
point on x 2 + y 2 = a12 to (C) Number of common tangents (R) 3
x2 + y2 = b2 touches the circle to the circles x2 + y2 – 2x – 4y
x 2 + y 2 = a22 , where (a1 π
(S) a1a2 = 1 = 0 and x2 + y2 – 8y – 4 = 0 is
a2), then (D) Number of common tangents (S) 0
to the circles
(C) If the circles (T) a1a2 = b2 x2 + y2 + 2x – 8y + 13 = 0 and
2 2
x + y + 2a1x + b = 0 and x2 + y2 – 6x – 2y + 6 = 0 is
x2 + y2 + 2a2x + b = 0 where
a1 π a2 and b Œ R cuts orthog- 8. Match the following columns:
onally, then Column I Column II
(A) Ê 1 ˆ Ê 1ˆ Ê 1ˆ (P) 2
5. Match the following columns: If Á a, ˜ , Á b, ˜ , Á c, ˜ and
Ë a¯ Ë b¯ Ë c¯
Column I Column II Ê 1ˆ
ÁË d , ˜¯ are four distinct points
(A) The lines 3x – 4y + 4 = 0 and 6x (P) 2 d
– 8y – 7 = 0 are the tangents to a on a circle of radius 2012 units,
circle, its radius is the value of abcd is
(B) The radius of the circle inscribed (Q) 3/4 (B) If a circle passes through the (Q) 1
in the triangle formed by the lines point of intersection of axes (R) –5
x = 0, y = 0 and 4x + 3y = 24 is with the lines lx – y + 1 = 0 and
(C) The radius of the circle (R) 7 x – 2y + 3 = 0, then the value
3x(x – 2) + 3y(y + 1) = 4 is of l is
(D) The lines 2x – 3y = 5 and (S) (C) If the curves (S) –4
31
3x – 4y = 7 are the diameters of a ax2 + 4xy + 2y2 + x + y + 5
circle of area 154 s.u., its radius 12 = 0 and
is ax2 + 6xy + 5y2 + 2x + 3y + 8 = 0 (T) 3
intersect at four concyclic
6. Match the following Columns: points, the value of a is
9. Match the following columns:
Column I Column II
(A) The length of the tangents from (P) Column I Column II
3/ 2
any point on the circle (A) If one of the diameters of the cir- (P) 7
x2 + y2 + 4x + 6y + 2008 = 0 cle x2 + y2 – 2x – 6y + 6 = 0 (Q) 9
to the circle (Q) 2 is a chord to the circle with centre
x2 + y2 + 4x + 6y + 2012 = 0 is (2, 1), the radius of the circle is
(B) The lengths of the common (R) 1 (B) If the tangent at the point P on the (R) 5
chord of the circles circle x2 + y2 + 6x + 6y = 2 (S) 2
x2 + y2 + 2x + 3y + 1 = 0 meets the straight line
and x2 + y2 + 3x + 2y + 1 = 0 is 5x – 2y + 6 = 0 at a point Q on the
(C) The number of tangents which (S) 5/3 y-axis, the length of PQ is
can be drawn from the point (C) If the angle between the tangents (T) 3
(2, 3) to the circle x2 + y2 = 13 is from a point P to the circle
(D) The radius of the circle (T) 4/3 x2 + y2 + 4x – 6y + 4 cos2a + 9 = 0
ax2 + (2a – 3)y2 – 4x – 7 = 0 is is 2a, the radius of the locus of
P is

CG_03.indd 29 2/6/2017 4:00:19 PM


3.30 Coordinate Geometry Booster

10. Match the following columns: 9. AB is a diameter of a circle and C is any point on the
circumference of the circle. Then
Column I Column II
(a) The area of triangle ABC is maximum when it is
(A) The locus of the point of (P) x2 + y2 – 10x – isosceles.
intersection of two per- 25 = 0 (b) The area of triangle ABC is minimum when it is
pendicular tangents to a isosceles.
circle x2 + y2 = 10 is (c) The area of triangle ABC is minimum when it is
(B) The equation of the direc- (Q) x2 + y2 – 6y + isosceles.
tor circle of the circle 1=0 (d) None [IIT-JEE, 1983]
x2 + y2 – 10x = 0 is 10. Through a fixed point (h, k) secants are drawn to the
(C) The equation of the direc- (R) x2 + y2 = 20 circle x2 + y2 = r2. Show that the locus of the mid-points
tor circle of the circle of the secants intercepted by the circle is x2 + y2 = hx +
x2 + y2 – 6y + 5 = 0 is ky [IIT-JEE, 1983]
11. The locus of the mid-point of a chord of the circle
x2 + y2 = 4, which subtends a right angle at the origin is
Questions asked in Previous Years’ (a) x + y = 2 (b) x2 + y2 = 1
2 2
JEE-Advanced Examinations (c) x + y = 2 (d) x + y = 1
[IIT-JEE, 1984]
1. Find the equation of the circle which passes through the
12. The abscissa of the two points A and B are the roots of
point (2, 0) and whose centre is the limit of the point of
the equation x2 + 2ax – b2 = 0 and their ordinates are the
intersection of the lines 3x + 5y = 1, (2 + c)x + 5c2y = 1
roots of the equation y2 + 2py – q2 = 0. Find the equa-
as c tends to 1. [IIT-JEE, 1979]
tion of the circle on AB as diameter. [IIT-JEE, 1984]
2. Two circles x2 + y2 = 6 and x2 + y2 – 6x + 8 = 0 are
given. The equation of the circle through their points of 13. The lines 3x – 4y + 4 = 0 and 6x – 8y – 7 = 0 are tan-
intersection and the point (1, 1) is gents to the same circle. The radius of the circle is…
(a) x2 + y2 – 6x + 4 = 0 (b) x2 + y2 – 3x + 1 = 0 [IIT-JEE, 1984]
(c) x2 + y2 – 4x + 2 = 0 (d) none 14. From the origin chords are drawn to the circle (x – 1)2
[IIT-JEE, 1980] + y2 = 1. The equation of the locus of the mid-points of
3. Let A be the centre of the circle x2 + y2 – 2x – 4y – 20 these chords is… [IIT-JEE, 1984]
= 0. Suppose that the tangents at the points B(1, 7) and 15. Let x2 + y2 – 4x – 2y – 11 = 0 be a circle. A pair of
D(4, –2) on the circle meet at the point C, find the area tangents from (4, 5) with a pair of radii form a quadri-
of the quadrilateral ABCD. [IIT-JEE, 1981] lateral of area… [IIT-JEE, 1985]
4. Find the equations of the circles passing through (–4, 3) 16. The equation of the line passing through the points of
and touching the lines x + y = 4 and x – y = 2. intersection of the circles 3x2 + 3y2 – 2x + 12y – 9 = 0
[IIT-JEE, 1982] and x2 + y2 + 6x + 2y – 15 = 0 is…
PA [IIT-JEE, 1986]
5. If A and B are points in the plane such that =
PB 17. From the point A(0, 3) on the circle x + 4x + (y – 3)2 =
2

k (constant) for all P on a given circle, the value of k 0. A chord AB is drawn and extended to a point M such
cannot be equal to… . [IIT-JEE, 1982] that AM = 2AB. The equation of the locus of M is…
6. The points of intersection of the line 4x – 3y – 10 = 0 [IIT-JEE, 1986]
and the circle x2 + y2 – 2x + 4y – 20 = 0, is … 18. Lines 5x + 12y – 10 = 0 and 5x – 12y – 40 = 0 touch
[IIT-JEE, 1983] a circle C1 of diameter 6. If the centre of C1 lies in the
7. The equation of the circle passing through (1, 1) and first quadrant, find the equation of the circle C2 which
the points of intersection of x2 + y2 + 13x – 3y = 0, is concentric with C1 and cuts intercepts of length 8 on
2x2 + 2y2 + 4x – 7y – 25 = 0 is these lines. [IIT-JEE, 1986]
(a) 4x2 + 4y2 – 30x – 10y – 25 = 0 19. Let a given line L1 intersects the x and y axes at P and
(b) 4x2 + 4y2 + 30x – 13y – 25 = 0 Q, respectively. Let another line L2 perpendicularly
(c) 4x2 + 4y2 – 17x – 10y – 25 = 0 cuts the x and y axes at R and S, respectively. Show that
(d) none [IIT-JEE, 1983] the locus of the point of intersection of the lines PS and
8. The centre of the circle passing through the point (0, 1) QR is a circle passing through the origin.
and touching the curve y = x2 at (2, 4) is [IIT-JEE, 1987]
Ê 16 27 ˆ Ê 16 53 ˆ 20. The circle x2 + y2 – 4x – 4y + 4 = 0 is inscribed in a tri-
(a) Á - , ˜ (b) Á - , ˜
Ë 5 10 ¯ Ë 7 10 ¯ angle which has two of its sides along the co-ordinate
axes. The locus of the circumcentre of the triangle is
Ê 16 53 ˆ
(c) Á - , ˜
Ë 5 10 ¯
(d) none x + y - xy + k ( x 2 + y 2 ) = 0, find the value of k.
[IIT-JEE, 1983] [IIT-JEE, 1987]

CG_03.indd 30 2/6/2017 4:00:19 PM


Circle 3.31

21. The area of the triangle formed by tangents from the 33. A point P is given on the circumference of a circle
points (4, 3) to the circle x2 + y2 = 9 and the line joining of radius r, a chord QR is parallel to the tangent at P.
their points of contact is… [IIT-JEE, 1987] Determine the maximum possible area of the triangle
22. A polygon of nine sides, each of length 2, is inscribed PQR. [IIT-JEE, 1990]
in a circle. The radius of the circle is… 34. Two circles each of radius 5 units, touch each other at
[IIT-JEE, 1987] (1, 2). If the equation of their common tangent is 4x +
23. If the circle C1 : x2 + y2 = 16 intersects another circle 3y = 10. Find the equations of the circles.
C2 of radius 5 in such a manner that the common chord [IIT-JEE, 1991]
is of maximum length and has a slope equal to 3/4, the 35. If a circle passes through the points of intersection of
co-ordinates of the centre C2 are… the co-ordinate axes with the lines lx – y + 1 = 0 and
[IIT-JEE, 1988] x – 2y + 3 = 0, the value of l is…
24. If a circle passes through the point (a, b) and cuts the [IIT-JEE, 1991]
circle x2 + y2 = k2 orthogonally, the equation of the lo- 36. Three circles, each of radius 5 units, touch each other
cus of its centre is externally. The tangent at their points of contact meet
(a) 2ax + 2by – (a2 + b2 + k2) = 0 at a point whose distance from a point of contact is 4.
(b) 2ax + 2by – (a2 – b2 – k2) = 0 Find the ratio of the product of the radii to the sum of
(c) x2 + y2 – 3ax – 4by + (a2 + b2 – k2) = 0 the radii of the circles. [IIT-JEE, 1992]
(d) x2 + y2 – 2ax – 3by + (a2 + b2 – k2) = 0 37. Let a circle be given by 2x(x – a) + y(2y – b) = 0, (a, b
[IIT-JEE, 1988] π 0), find the condition on a and b if two chords, each
25. The equation of the tangents drawn from the origin to bisected by the x-axis, can be drawn to the circle from
the circle x2 + y2 + 2rx – 2hy + h2 = 0 are Ê bˆ
(a) x = 0 (b) y = 0 ÁË a, ˜¯ . [IIT-JEE, 1992]
2
(c) (h2 – r2)x – 2rhy = 0 (d) (h2 – r2)x + 2rhy = 0 38. The centre of a circle passing through the points (0, 0),
[IIT-JEE, 1988] (1, 0) and touching the circle x2 + y2 = 9 is
26. Let S = x2 + y2 + 2gx + 2fy + c = 0 be a given circle. (a) (3/2, ½) (b) (1/2, 3/2)
Find the locus of the foot of the perpendicular drawn
Ê1 ˆ
from the origin upon any chord of S which subtends a (c) (1/2, ½) (d) ÁË , 2 ˜¯
2
right angle at the origin. [IIT-JEE, 1988]
27. If the two circles (x – 1)2 + (y – 3)2 and x2 + y2 – 8x + [IIT-JEE, 1992]
2y + 8 = 0 intersect in two distinct points, then 39. The locus of the centre of a circle which touches ex-
(a) 2 < r < 8 (b) r < 2 ternally the circle x2 + y2 – 6x – 6y + 14 = 0 and also
(c) r = 2 (d) r > 2 touches the y-axis is given by the equation
[IIT-JEE, 1989] (a) x2 – 6x – 10y + 14 = 0
28. The lines 2x – 3y = 5 and 3x – 4y = 7 are diameters of (b) x2 – 10x – 6y + 14 = 0
a circle of area 154 sq. units. Then the equation of the (c) y2 – 6x – 10y + 14 = 0
circle is (d) y2 – 10x + 6y + 14 = 0 [IIT-JEE, 1993]
(a) x2 + y2 + 2x – 2y – 62 = 0 40. Consider a family of circles passing through two fixed
(b) x2 + y2 + 2x – 2y – 47 = 0 points A(3, 7) and B(6, 5). Show that the chords in
(c) x2 + y2 – 2x + 2y – 47 = 0 which the circle x2 + y2 – 4x – 6y – 3 = 0 cuts the mem-
(d) x2 + y2 – 2x + 2y – 62 = 0 [IIT-JEE, 1989] bers of the family are concurrent at a point. Find the
co-ordinates of this point. [IIT-JEE, 1993]
Ê 1ˆ
29. If Á mi , ˜ i = 1, 2, 3, 4 are four distinct points on a 41. Find the co-ordinates of the point at which the circle
Ë mi ¯ x2 + y2 – 4x – 2y + 4 = 0 and x2 + y2 – 12x – 8y + 36
circle, show that m1m2m3m4 = 1 = 0 touch each other. Also find equations of common
[IIT-JEE, 1989] tangents touching the circles in distinct points.
30. The line x + 3y = 0 is a diameter of the circle x2 + y2 – [IIT-JEE, 1993]
6x + 2y = 0. Is it true/false? 42. The equation of the locus of the mid-points of chords of
[IIT-JEE, 1989] the circle 4x2 + 4y2 – 12x + 4y + 1 = 0 that subtend an
31. The area of the triangle formed by the positive x-axis 2p
and the normal and the tangent to the circle x2 + y2 = 4 angle of at its centre is… [IIT-JEE, 1993]
3
at (1, 3) is… [IIT-JEE, 1989]
32. A circle touches the line y = x at a point P such that 43. The circles x2 + y2 – 10x + 16 = 0 and x2 + y2 = r2 inter-
sect each other in distinct points if
OP = 4 2 , where O is the origin. The circle contains
(a) r < 2 (b) r > 8
the point (–10, 2) in its interior and the length of its
(c) 2 < r < 8 (d) 2 £ r £ 8
chord on the line x + y = 0 is 6 2 . Determine the equa- [IIT-JEE, 1994]
tion of the circle. [IIT-JEE, 1990]

CG_03.indd 31 2/6/2017 4:00:20 PM


3.32 Coordinate Geometry Booster

44. A circle is inscribed in an equilateral triangle of side 54. The number of common tangents to the circles x2 + y2
a. The area of any square inscribed in this circle is… = 4 and x2 + y2 – 6x – 8y – 24 = 0 is
[IIT-JEE, 1994] (a) 0 (b) 1 (c) 3 (d) 4
45. The angle between a pair of tangents drawn from [IIT-JEE, 1998]
a point P to the circle x2 + y2 + 4x – 6y + 9 sin2a + 55. C1 and C2 are two concentric circles, the radius of C2
13 cos2a = 0 is 2a. Then the locus of P is being twice that of C1. From a point P on C2, tangents
(a) x2 + y2 + 4x – 6y + 4 = 0 PA and PB are drawn to C1. Prove that the centroid of
(b) x2 + y2 + 4x – 6y – 9 = 0 the triangle PAB lies on C1. [IIT-JEE, 1998]
(c) x2 + y2 + 4x – 6y – 4 = 0 56. Let A0A1A2A3A4A5 be a regular hexagon inscribed in a
(d) x2 + y2 + 4x – 6y + 9 = 0 unit circle. Then the product of the lengths of line seg-
[IIT-JEE, 1996] ments A0A1, A0A2, A0A4 is
46. A circle passes through three points A, B and C with the (a) 3/4 (b) 3 3 (c) 3 (d) 3 3/2
line segment AC as its diameter. A line passing through [IIT-JEE, 1998]
A intersects the chord BC at a point D inside the circle. 57. If two distinct chords, drawn from the point (p, q) on
If angles DAB and CAB are a and b respectively and the circle x2 + y2 = px + qy, where pq π 0 are bisected
the distance between the point A and the mid-point of by the x-axis, then
the line segment DC is d. Prove that the area of the (a) p2 = q2 (b) p2 = 8q2
p d ¥ cos 2a 2
(c) p < 8q 2
(d) p2 > 8q2
circle is 2 2
cos a + cos b + 2 cos a cos b cos (b - a ) [IIT-JEE, 1999]
[IIT-JEE, 1996] 58. Let T1 and T2 be two tangents drawn from (–2, 0) on the
47. Find the interval in which a lies for which the circle C: x2 + y2 = 1. Determine the circles touching C
line y + x = 0 bisects two chords drawn from and having T1, T2 as their pairs of tangents. Further find
the equations of all possible common tangents to these
Ê1 + a 2 1 - a 2 ˆ
the point Á , ˜ to the circle circles, taken two at a time. [IIT-JEE, 1999]
Ë 2 2 ¯ 59. The triangle PQR is inscribed in the circle x2 + y2 = 25.
2( x 2 + y 2 ) - (1 + a 2) x - (1 - a 2) y = 0 . If Q and R have co-ordinates (3, 4) and (–4, 3) respec-
[IIT-JEE, 1996] tively, then –QPR is … [IIT-JEE, 2000]
48. Intercepts on the line y = x by the circle x2 + y2 – 2x = 0 (a) p/2 (b) p/3
is AB. Equation of the circle with AB as diameters is… (c) p/4 (d) p/6
[IIT-JEE, 1996] 60. If the circles x2 + y2 + 2x + 2ky + 6 = 0 and x2 + y2 + 2ky
+ k = 0 intersect orthogonally, then k is
49. Let C be any circle with the centre (0, 2) . Prove that
(a) 2 or –3/2 (b) –2 or –3/2
at the most two rational points can be there on C. ( A
(c) 2 or 3/2 (d) –2 or 3/2
rational point is a point for which both the co-ordinates
[IIT-JEE, 2000]
are rational numbers.) [IIT-JEE, 1997]
61. Let AB be a chord of the circle x2 + y2 = r2 subtending
50. Consider a curve ax2 + 2hxy + by2 = 1 and a point P on right angle at the centre, the locus of the centroid of the
the curve. A line drawn from the point P intersects the triangle PAB as P moves on the circle is
curve at point Q and R. If the product PQ ◊ PR is inde- (a) a parabola (b) a circle
pendent of the slope of the line, show that the curve is (c) an ellipse (d) a pair of straight lines
a circle. [IIT-JEE, 1997] [IIT-JEE, 2001]
51. For each natural number k, let Ck denotes the circle with 62. Let 2x2 + y2 – 3xy = 0 be the equation of pair of tangents
radius k centimetres and centre at the origin. On the drawn from the origin O to a circle of radius 3 with
circle Ck, a particle moves k centimetres in the counter- centre is in the first quadrant. If A is one of the points
clockwise direction. After completing its motion on of contact, find the length of OA. [IIT-JEE, 2001]
Ck, the particle moves to Ck+1 in the radial direction. 63. Let PQ and RS be tangents at the extremities of the di-
The particle starts at (1, 0). If the particle crosses the ameter PR of a circle of radius r. If PS and RQ intersect
positive direction of the x-axis for the first time on the a point x on the circumference of the circle, then 2r
circle Cn, then n = … [IIT-JEE, 1997] equals
52. The chords of contact of the pair of tangents drawn PQ + RS
(a) PQ ◊ RS (b)
from each point on the line 2x + y = 4 to the circle x2 + 2
y2 = 1 pass through the point… [IIT-JEE, 1997] 2PQ ◊ RS PQ 2 + RS 2
53. Two vertices of an equilateral triangle are (–1, 0) and (c) (d)
PQ + RS 2
(1, 0) and its third vertex lies above the x-axis, the [IIT-JEE, 2001]
equation of its circumcircle is… [IIT-JEE, 1997]

CG_03.indd 32 2/6/2017 4:00:20 PM


Circle 3.33

64. Let C1 and C2 be two circles with C2 lying inside C1. tact intersect at P. Find the distance between point P
A circle C lying inside C1 touches C1 internally and C2 and the point of contact. [IIT-JEE, 2005]
externally. Identify the locus of the centre of C. 75. A circle is given by x2 + (y – 1)2 = 1, another circle C
[IIT-JEE, 2001] touches it externally and also the x-axis, the locus of its
65. If the tangent at the point P on the circle x2 + y2 + 6x + centre is
6y = 2 meets the straight line 5x – 2y + 6 = 0 at a point (a) {(x, y) : x2 = 4y} » {(x, y) : y £ 0}
Q on the y-axis, the length of PQ is (b) {(x, y) : x2 + (y – 1)2 = 4} » {(x, y) : y £ 0}
(a) 4 (b) 2 5 (c) 5 (d) 3 5 (c) {(x, y) : x2 = y} » {(0, y) : y < 0}
[IIT-JEE, 2002] (d) {(x, y) : x2 = y} » {(0, y) : y £ 0}
66. If a > 2b > 0, the positive value of m for which [IIT-JEE, 2005]
76. The tangent to the curve y = x2 + 6 at a point P(1, 7)
y = mx - b 1 + m 2 is a common tangent to x2 + y2 = touches the circle x2 + y2 + 16x + 12y + c = 0 at a point
b2 and (x – a)2 + y2 = b2 is Q. Then the co-ordinates of Q are
2b a 2 - 4b 2 (a) (–6, –7) (b) (–10, –15)
(a) (b) (c) (–9, –13) (d) (–6, –11)
a 2 - 4b 2 2a
[IIT-JEE, 2005]
2b b
(c) (d)
a - 2b a - 2b Comprehension Link Passage
[IIT-JEE, 2002]
67. Tangents are drawn from P(6, 8) to the circle x2 + y2 Let ABCD be a square of side 2 units. C2 is the circle through
= r2. Find the radius of the circle such that the area of vertices A, B, C, D and C1 in the circle touching all the sides
the triangle formed by the tangents and the chord of of the square ABCD, L is a line through A.
contact is maximum. [IIT-JEE, 2003] 77. If P is a point on C1 and Q in another point on C2, then
68. If In represents area of n-sided regular polygon in- PA2 + PB 2 + PC 2 + PD 2
is equal to
scribed in a unit circle and On be the area of the n- QA2 + QB 2 + QC 2 + QD 2
sided regular polygon circumscribing it, prove that (a) 0.75 (b) 1.25 (c) 1 (d) 0.5
O È

Ê 2I ˆ 78. A circle touches the line L and the circle C1 externally
I n = n Í1 + 1 - Á n ˜ ˙ .
2 ÎÍ Ë n ¯ ˚˙ such that both the circles are on the same side of the
[IIT-JEE, 2003]
line, the locus of the centre of the circle is
69. The centre of the circle inscribed in a square formed by
(a) ellipse (b) hyperbola
the lines x2 – 8x + 12 = 0 and y2 – 14y + 45 = 0 is
(c) parabola (d) pairs of straight lines
(a) (4, 7) (b) (7, 4)
79. A line M through A is drawn parallel to BD. Point S
(c) (9, 4) (d) (4, 9)
moves such that its distances from the line BD and the
[IIT-JEE, 2004]
vertex A are equal. If locus of S cuts M at T2 and T3 and
70. If one of the diameters of the circle x2 + y2 – 2x – 6y + 6
AC at T1, the area of DT1T2T3 is
= 0 is a chord to the circle with centre (2, 1), the radius
(a) 1/2 sq. unit (b) 2/3 sq. units
of the circle is
(c) 1 sq. unit (d) 2 sq. units
(a) 3 (b) 2 (c) 3 (d) 2 [IIT-JEE, 2006]
[IIT-JEE, 2004] 80. Let ABCD be a quadrilateral with area 18, with side
71. Find the centre and the radius of the circle formed by AB parallel to the side CD and AB = 2CD. Let AD be
all the points represented by z = x + iy satisfying the perpendicular to AB and CD. If a circle is drawn inside
|z - a | the quadrilateral ABCD touching all the sides, its radius
relation = k (k π 1) , where a and b are constant
|z - b| is [IIT-JEE, 2007]
complex numbers given by a = a1 + ia2, and b = b1 + (a) 3 (b) 2 (c) 3/2 (d) 1.
ib2. [IIT-JEE, 2004] 81. Tangents are drawn from the point (17, 7) to the circle
72. | z - 1| = 2 is a circle inscribed in a square whose one x2 + y2 = 169.
Statement 1: The tangents are mutually perpendicular.
vertex is 2 + i 3 . Find the remaining vertices. Statement 2: The locus of the point from which mutu-
[IIT-JEE, 2005] ally perpendicular tangents can be drawn to the given
73. Find the equation of the circle touches the line 2x + circle is x2 + y2 = 338. [IIT-JEE, 2007]
3y + 1 = 0 at the point (1, –1) and is orthogonal to the 82. Consider L1: 2x + 3y + (p – 3) = 0
circle which has the line segment having end-points L2: 2x + 3y + (p + 3) = 0,
(0, –1) and (–2, 3) as the diameter. [IIT-JEE, 2004] where p is a real number and
74. Three circles of radii 3, 4 and 5 units touches each other C: x2 + y2 + 6x – 10y + 30 = 0
externally and the tangents drawn at the point of con-

CG_03.indd 33 2/6/2017 4:00:20 PM


3.34 Coordinate Geometry Booster

Statement 1: If L1 is a chord of circle C, then L2 is not ÏÊ 3 ˆ Ê 5 3 ˆ Ê 1 1 ˆ Ê 1 1 ˆ ¸


always the diameter of the circle C. S = ÌÁ 2, ˜ , Á , ˜ , Á , - ˜ , Á , ˜ ˝ ,
Statement 2: If L1 is a diameter of circle C, the line L2 ÓË 4 ¯ Ë 2 4 ¯ Ë 4 4 ¯ Ë 8 4 ¯ ˛
is not a chord of circle C. [IIT-JEE, 2008] the number of point(s) in S lying inside the smaller part
is… [IIT-JEE, 2011]
Comprehension 90. The locus of the mid-point of the chord of contact of
tangents drawn from points lying on the straight line
A circle C of radius 1 is inscribed in an equilateral triangle 4x – 5y = 20 to the circle x2 + y2 = 9 is
PQR. The points of contact of C with the side PQ, QR, RP (a) 20(x2 + y2) – 36x + 45y = 0
are D, E, F, respectively. The line PQ is given by the equa- (b) 20(x2 + y2) + 36x – 45y = 0
tion 3 x + y = 6 and the point D is (3 3/2, 3/2) . Further it (c) 20(x2 + y2) – 20x + 45y = 0
is given that the origin and the centre C are on the same side (d) 20(x2 + y2) + 20x – 45y = 0 [IIT-JEE, 2012]
PQ. 91. A tangent PT is drawn to the circle x2 + y2 = 4 at the
83. The equation of the circle C is
point P( 3, 1) . A straight line L, perpendicular to PT
(a) ( x - 2 3) 2 + ( y - 1) 2 = 1 is a tangent to the circle (x – 3)2 + y2 = 1
(b) ( x - 2 3) 2 + ( y - 1/2) 2 = 1 (i) A possible equation of L is
(c) ( x - 3) 2 + ( y + 1) 2 = 1 (a) x — 3 y = 1 (b) x + 3 y = 1
2 2
(d) ( x - 3) + ( y - 1) = 1 [IIT-JEE, 2008] (c) x - 3 y = -1 (d) x + 3 y = 5
84. Points E and F are given by (ii) A common tangent to the two circles is
(a) ( 3/2, 3/2), ( 3, 0) (a) x = 4 (b) y = 2
(b) ( 3/2, 1/2), ( 3, 0) (c) x + 3 y = 4 (d) x + 2 2 y = 6
[IIT-JEE, 2012]
(c) ( 3/2, 3/2), ( 3/2, 1/2)
No questions asked in 2013.
(d) (3/2, 3/2), ( 3/2, 1/2) [IIT-JEE, 2008] 92. A circle S passes through the point (0, 1) and is or-
85. Equations of the sides QR, PR are thogonal to the circles (x – 1)2 + y2 = 16 and x2 + y2 = 1.
(a) y = (2/ 3) x + 1, y = - (2/ 3) x - 1 Then the
(a) radius of S is 8 (b) radius of S is 7
(b) y = (1/ 3) x + 1, y = 0
(c) centre of S is (–7, 1) (d) centre of S is (–8, 1)
(c) y = ( 3/2) x + 1, y = ( - 3/2) x - 1 [IIT-JEE, 2014]
(d) y = 3 x, y = 0 [IIT-JEE-2008] 93. The common tangents to the circle x2 + y2 = 2 and the
86. Tangents are drawn from a point P(1, 8) to the circle parabola y2 = 8x touch the circle at the points P, Q and
x2 + y2 – 6x – 4y – 11 = 0 touch the circle at the points A the parabola at the points R, S. Then the area of the
and B. The equation of the circumcircle of the triangle quadrilateral PQRS is
PAB is (a) 3 (b) 6 (c) 9 (d) 15
(a) x2 + y2 – 6x – 4y – 19 = 0 [IIT-JEE, 2014]
(b) x2 + y2 – 6x – 10y – 19 = 0 94. Let RS be the diameter of the circle x2 + y2 = 2, where
(c) x2 + y2 – 2x + 6y – 29 = 0 S is the point (1, 0).
(d) x2 + y2 – 6x – 4y + 19 = 0 [IIT-JEE, 2009] Let P be a variable point (other than R and S) on the
87. The locus of the point (h, k) for which the line hx + ky circle and tangents to the circle at S and P meet at the
= 1 touches the circle x2 + y2 = 4 is.... point Q. The normal to the circle at P intersects a line
[IIT-JEE, 2009] drawn through Q parallel to RS at point E. Then the
88. Two parallel chords of a circle of the radius 2 are at locus of E passes through the point(s)
a distance 3 + 1 apart. If the chords subtend, at the Ê1 1 ˆ 1 1
p 2p (a) Á , (b) ÊÁ , ˆ˜
centre angles of and , where k > 0, the value of Ë 3 3 ˜¯ Ë 4 2¯
k k
[k] is…, where [,] = GIF [IIT-JEE, 2010] Ê1 1 ˆ Ê 1 1ˆ
(c) Á , - ˜ (d) ÁË , - ˜¯
89. The straight line 2x – 3y = 1 divides the circular region Ë3 3¯ 4 2
x2 + y2 £ 6 into two parts. If [IIT-JEE-2016]

CG_03.indd 34 2/6/2017 4:00:21 PM


Circle 3.35

A NSWERS

LEVEL I 38. y = 3 ± 2 2( x - 2)
Ê 5ˆ
1. (i) centre is (0, 0) and radius is 4 39. 2 tan -1 Á ˜
Ë 3¯
(ii) centre is ( 4, 0) and radius is 1
Ê 1 1ˆ 1 Ê 3ˆ
(iii) centre is Á , ˜ and radius is 40. 2 tan -1 Á ˜
Ë 2 2¯ 2 Ë 2¯
3. x2 + y2 – 8x + 6y = 27 41. x2 + y2 = 50
4. x2 + y2 + 4x – 2y = 0 42. x2 + y2 = 18
2 2
Ê 5ˆ Ê 10 ˆ 43. 90°
5. ÁË x - ˜¯ + Á y + ˜ = 16 44. (i) x2 + y2 + 2x – 1 = 0
7 Ë 7¯
(ii) x2 + y2 + 10y + 23 = 0
6. x2 + y2 + 4y = 23 (iii) x2 + y2 + 16x + 12y + 98 = 0
3 3 (iv) x2 + y2 + 2gx + 2fy – g2 – f2 + 2c = 0
7. ¥ ( g 2 + f 2 - c) 1
4 (v) x 2 + y 2 - ax - by - (a 2 + b 2 ) = 0
4
8. x2 + y2 – 8x – 6y + 16 = 0
and x2 + y2 – 14x – 12y + 76 = 0. 5 5
45. y = x and y = - x
Ê 36 15 ˆ 2 2
9. Á , ˜ 46. 3x + y + 5 = 0
Ë 13 13 ¯
47. x – y + 1 = 0
10. x2 + y2 – 8x – 12y + 39 = 0 48. 3x + 2y = 24
11. 2(x2 + y2) – 13x – 33y – 135 = 0 49. 5x + 3y = 25
12. x2 + y2 – 5x – 5y + 10 = 0 50. (1, –2)
13. 2 17 51. a2(x2 + y2) = (hx + ky)2
14. 1 Ê (h 2 + k 2 - a 2 )3/2 ˆ
15. the centre is (a, a) and the radius is a. 52. a Á ˜
16. the point (1, 2) lies inside of the circle and (6, 0) lies Ë (h 2 + k 2 ) ¯
outside of the circle. 53. m = 2, n = 1 and p = 1
17. (–1, 4) 54. 2x – 3y + 13 = 0
18. 2, 28 55. 4x + 3y = 17
19. (–1, 1) 56. x2 + y2 = hx + ky
57. x2 + y2 = 2
20. 3x + 2y ± 2 13 = 0
21. 3x – 4y + 15 = 0 and 3x – 4y – 15 = 0 58. ( -1, 3)
22. y = 3 x + (2 3 ± 2) Ê 3 1 ˆ
59. Á 2 + , - 3˜
23. y = ±x ± 4 Ë 2 2 ¯
24. 4x – 3y = 25 and 3x + 4y = 25 60. 2012x + 2013y = 0
25. x2 + y 2 - 2 2x - 2 2 y - 5 = 0 61. 5x – 4y + 2 = 0
62. 4
26. 2x + 5 y = 9 and 2x - 5 y = 9
63. 2(x2 + y2) + 2x + 6y + 1 = 0
27. y=4
64. (i) 1 (ii) 4 (iii) 4 (iv) 3 (v) 0
Ê 11 ˆ Ê 13 ˆ
28. ÁË - 2, - ˜¯ and ÁË 4, ˜¯ 65. D.C.T: y = 4, 4x – 3y = 0
10 10 T.C.T: x = 0 and 3x + 4y = 10
29. x + 2y – 1 = 0 and 2x – y + 1 = 0 1 1 1
30. x=2 66. 2 + 2 = 2
a b c
31. (4 2 - 3) 67. 4
32. 3 68. 90°
33. a 2 - b2 70. k = 4
18 18
34. 1 71. x 2 + y 2 - x - y = 0
35. 13 5 5
36. 1 72. 9x – 10y + 11 = 0
37. y = 2, 4x – 3y + 2 = 0 73. 2ax + 2by – (a2 + b2 + 4) = 0

CG_03.indd 35 2/6/2017 4:00:21 PM


3.36 Coordinate Geometry Booster

2r1r2 11. 16
74. 12. (2/13, 3/13)
r12 + r22 13. d -c
75. x – 2y – 1 = 0 14. –6
76. (3, 2) 15. (x – 17)2 + (y – 16)2 = 1
Ê 5ˆ 19. x2 + y2 – 4y – 12 = 0
77. ÁË 2, - ˜¯
2 25. (9, –3)
78. x2 + y2 – 2x – y – 8 = 0 26. 3
79. (4x2 + 4y2 + 30x – 13y – 25) = 0 29. 0
80. (x2 + y2 + 2x + 6y + 1) = 0
81. x2 + y2 – x – y – 8 = 0
10 10 12
LEVEL IV
82. x 2 + y 2 - x - y - =0 1. (i) 24y = ±5(2x + 13)
7 7 7
Ï169 -1 Ê 120 ˆ ¸
83. 2(x2 + y2) + 2x + 6y + 1 = 0. (ii) Ì sin Á ˜¯ - 15˝ sq. m.
Ó 8 Ë 169 ˛
2.
LEVEL II 3. 4x2 + 4y2 + 6x + 10y – 1 = 0
4. (a – a¢)(BC¢ – CB¢) + (b – b¢)(CA¢ – AC¢)
1. (b) 2. (c) 3. (b) 4. (b) 5. (a) + (c – c¢)(AB¢ – BA¢) = 0
6. (b) 7. (a) 8. (b) 9. (a) 10. (c) 2 2
5. (x – 1) + (y – 7) = 9
11. (a) 12. (a) 13. (a) 14. (b) 15. (b) or (x – 7)2 + (y – 1)2 = 9
16. (a) 17. (a) 18. (b) 19. (c) 20. (c)
6. x2 + y2 – 8x – 6y + 16 = 0
21. (c) 22. (b) 23. (a) 24. (a) 25. (a)
26. (d) 27. (a) 28. (b) 29. (d) 30. (b) and x2 + y2 – 14x – 12y + 76 = 0
31. (a) 32. (c) 33. (d) 34. (b) 35. (b) 7. x2 + y2 – 2x – 6y – 12 = 0
36. (a) 37. (a) 38. (d) 39. (c) 40. (c) 8. a 2 + b2 + 5
41. (a) 42. (b) 43. (a) 44. (b) 45. (b)
9. (3, –1)
46. (d) 47. (d) 48. (b) 49. (c) 50. (d)
51. (c) 52. (a) 53. (b) 54. (a) 55. (b) 10. (c)
56. (a) 57. (c) 58. (a) 59. (c) 60. (c) 11. (d)
61. (b) 62. (c) 63. ( c) 64. ( c) 65. (d) 12. (c)
66. (b) 67. (c) 68. (d) 69. (c) 70. (c) 13. (c)
71. (d) 72. (a) 73. (d) 74. (a) 75. (b) 14. (d)
76. (d) 77. (b) 78. (c) 79. (b) 80. (d) 15. x2 + y2 + 6x – 3y – 45 = 0
81. (a) 82. (b) 83. (c) 84. (c) 85. (b, d)
16. x2 + y2 – 8x – 9y + 30 = 0
86. (a) 87. (a, c) 88. (d) 89. (b) 90. (c)
91. (a) 92. (c) 93. (a) 94. (a, c) 95. (a) 17. 2 2
96. (c) 97. (d) 98. (b) 99. (c) 100. (d) 18. (0, 3), x2 + y2 – 4x – 6y + 4 = 0
101. (b) 102. (c) 103. (d) 104. (c) 105. (a) 19. B = (3 - 2 5, 3 - 2 5)
106. (b) 107. (a) 108. (c) 109. (a) 110. (a) 20. (a)
111. (c) 112. (d) 113. (a) 114. (b) 115. (b)
21. (c)
116. (c) 117. (d)
22. 49(x2 + y2) – 420(x + y) + 900 = 0

LEVEL III
22. (c)
23. x2 + y2 – 7x + 7y + 12 = 0
1. 45 24. x 2 + y 2 - 4 2 x - 42 = 0
3. a ( 2 - 1), 2 2a 25. (b)
4. (3, 3); (7, 3) 26. (c)
5. 4 27. x2 + y2 – 6x – 2y + 1 = 0
6. (3/25, 4/25) 28. (b)
7. a=b±c 2 29. (d)
8. (x + 2)2 + (y – 3)2 = 6.25 a2
9. x2 + y2 – 2x – 4y + 4 = 0 30.
6
( g 2 + f 2 - c) 31. (d)
10.
2 g2 + f 2 32. (0, –2), (6, 6)

CG_03.indd 36 2/6/2017 4:00:22 PM


Circle 3.37

33. (c) Passage II: 1. (d) 2. (a)


34. B = (2, –2) or (–2, 2) Passage III: 1. (a) 2. (c) 3. (b) 4. (a)
35. (a) 5. (b) 6. (a)
36. (a, d) Passage IV: 1. (a) 2. (a) 3. (a) 4. (a)
37. 5. (a) 6. (b)
38. (d) Passage V: 1. (a) 2. (a) 3. (a) 4. (a) 5. (a)
39. x2 + y2 + 7x – 11y + 38 = 0 Passage VI 1. (b) 2. (c) 3. (a) 4. (c)
40. 8y – 6x ± 25 = 0 5. (b) 6. (a)
Passage VII: 1. (a) 2. (a) 3. (a) 4. (a)
Ê 9 ˆ
41. Á - , 2˜
Ë 2 ¯
MATRIX MATCH
42. y - ( 3 - 1) = 3( x - ( 3 - 1)) 1. (A) Æ (Q); (B) Æ (T); (C) Æ (P, S)
and y - ( 3 - 1) = 3( x + ( 3 + 1)) 2. (A) Æ (R); (B) Æ (P); (C) Æ (P); (D) Æ (R);
2 2 Ê 10 ˆ
(E) Æ (T)
43. x + y + Á ˜ x - 6y + 6 = 0 3. (A) Æ (P, Q); (B) Æ (S, T); (C) Æ (R)
Ë 3¯
4. (A) Æ (S); (B) Æ (T); (C) Æ (R);
5. (A) Æ (Q); (B) Æ (P); (C) Æ (S); (D) Æ (R)
INTEGER TYPE QUESTIONS 6. (A) Æ (Q); (B) Æ (P); (C) Æ (R); (D) Æ (R)
1. 1 2. 2 3. 5 4. 7 5. 6 7. (A) Æ (R); (B) Æ (S); (C) Æ (P); (D) Æ (Q)
6. 8 7. 2 8. 4 9. 4 10. 5 8. (A) Æ (Q); (B) Æ (P); (C) Æ (S)
9. (A) Æ (T); (B) Æ (R); (C) Æ (S)
COMPREHENSIVE LINK PASSAGE 10. (A) Æ (R); (B) Æ (P); (C) Æ (Q)
Passage I: 1. (b) 2. (a) 3. (b)

H INTS AND S OLUTIONS

LEVEL I and r3 = 4 + 36 + 9 = 7
Therefore, r1 + r3 = 7 + 1 = 8 = 2(4) = 2r2.
1. (i) The given equation of circle is x2 + y2 = 16 Thus, r1, r2, r3 are in AP.
Hence, the centre is (0, 0) and the radius = 4. 3. Since the circle is concentric, so the centre of the circle
(ii) The given equation of a circle is is the same as
x2 + y2 – 8x + 15 = 0 x2 + y2 – 8x + 6y – 5 = 0
fi (x – 4)2 + y2 = 16 – 15 = 1 Let CP is the radius.
Hence, the centre is (4, 0) and the radius = 1.
Then CP = ( - 4 - 2) 2 + ( - 7 + 3) 2
(iii) The given equation of a circle is
x2 + y2 – x – y = 0 = 36 + 16 = 52 = 2 13
2 2 2 Hence, the equation of the circle is
Ê 1ˆ Ê 1ˆ 1 1 1 Ê 1 ˆ
fiÁ x- ˜ + Á y - ˜ = + = =Á ( x - 4) 2 + ( y + 3) 2 = (2 13) 2
Ë 2 ¯ Ë 2 ¯ 4 4 2 Ë 2 ˜¯
Ê 1 1ˆ 1 fi x2 + y2 – 8x + 6y = 27
the centre is Á , ˜ and the radius is .
Ë 2 2¯ 2 4. The point of intersection of x + 3y = 0 and 2x – 7y = 0
2. The equations of given circles are is P(0, 0) and the point of intersection of x + y + 1 = 0
and
x2 + y2 = 1 …(i)
x – 2y + 4 = 0 is C (–2, 1).
x2 + y2 – 2x – 6y = 6 …(ii)
Thus, CP is the radius, i.e. CP = 4 + 1 = 5
and
Hence, the equation of the circle is
x2 + y2 – 4x – 12y = 9 …(iii)
( x + 2) 2 + ( y - 1) 2 = ( 5) 2
Let r1, r2 and r3 are the radii of the circles (i), (ii) and
(iii). fi x2 + y2 + 4x – 2y = 0
Then r1 = 1, 5. Clearly, the radius of the circle is
1 30 - ( -10) 1 40
r2 = g 2 + f 2 - c = 1 + 9 + 6 = 4 = = ¥ =4
2 42 + 32 2 5

CG_03.indd 37 2/6/2017 4:00:22 PM


3.38 Coordinate Geometry Booster

Let (h, k) be the centre of the circle. 8. Let the centre of the circle be (h, k) such that k = h – 1.
Thus 2h + k = 0
fi k = –2h
8h - 3k - 30
Also, =4 C(h, k)
42 + 32 1
8h - 3(— 2h) - 30 x–
fi =4 y =
P(7, 3)
5
14h - 30
fi =4
5 We have,
fi 14h – 30 = ±20 (h – 7)2 + (k – 3)2 = 32
fi 14h = 30 ± 20 = 50, 10 fi (h – 7)2 + (h – 4)2 = 32
25 5 fi h2 – 11h + 28 = 0
fi h= , h = 7, 4 and k = 6, 3
7 7
Hence, the equation of a circle can be
50 10 (x – 7)2 + (y – 6)2 = 32
So, k = - , -
7 7 & (x – 4) + (y – 3)2 = 32
Hence, the equation of the circle is fi x2 + y2 – 8x – 6y + 16 = 0
2 2
Ê 25 ˆ Ê 50 ˆ and x2 + y2 – 14x – 12y + 76 = 0
ÁË x - ˜¯ + ÁË y + ˜¯ = 16 9.
7 7
2 2 Y
Ê 5ˆ Ê 10 ˆ
or ÁË x - ˜¯ + ÁË y + ˜¯ = 16
7 7
6. The centre of the circle C L
Q
x2 + y2 – 16x – 24y + 183 = 0 is C(8, 12). 2
Let the centre of the new circle be C¢(h, k). R
3 P
h - 8 k - 12 2( - 32 + 84 + 13) q
Now, = =- X
—4 7 16 + 49 O N M
fi h = 0, k = –2
Hence, the equation of the new circle is Let –POX = q,
2 2
x + ( y + 2) = (3 3) 2 then –NCP = q
fi x2 + Y2 + 4Y = 23 Here, CP = 2, OC = 22 + 32 = 13
7. Let ABC be an equilateral tri- A OP = OC 2 - OP 2 = 13 - 4 = 9 = 3
angle such that OM is perpen-
dicular on BC. C = (2, 3) and ON = 2
Now, OM = ON + MN = ON + PR
Here, OB = g2 + f 2 - c O fi OP cos q = 2 + 2 sin q
Clearly, –BOM = 60°. 60° fi 3 cos q = 2 + 2 sin q
B M C fi 9 ocs2 q = 4 + 4 sin2 q + 8 sin q
Ê p ˆ BM fi 9 – 9 sin2 q = 4 + 4 sin2 q + 8 sin q
Now, sin Á ˜ =
Ë 3 ¯ OB fi 13 sin2 q + 8 sin q – 5 = 0
fi 13 sin2 q + 13 sin q – 5 sin q – 5 = q
3 3
fi BM = OB = ¥ g2 + f 2 - c fi 13 sin q (sin q + 1) – 5(sin q + 1) = 0
2 2 fi (sin q + 1) (13 sin q – 5) = 0
3 5
Thus, ar ( DABC ) = ¥ ( BC ) 2 fi sin q =
4 13

3 12
= ¥ (2BM ) 2 fi cos q =
4 13
Therefore,
3 3
= ¥ ( g 2 + f 2 - c) Ê 36 15 ˆ
4 P = (OP cos q , OP sin q ) = Á , ˜
Ë 13 13 ¯

CG_03.indd 38 2/6/2017 4:00:23 PM


Circle 3.39

10. Hence, the equation of the circle is 5 5


(x – 2)(x – 6) + (y – 3)(y – 9) = 0 Thus, f = –5 – g = –5 + = - and c = 10
2 2
fi x2 + y2 – 8x – 12y + 39 = 0 Hence, the equation of the required circle is
11. Hence, the equation of the circle be
Ê 5ˆ Ê 5ˆ
x2 + y2 + 2gx + 2fy + c = 0 …(i) x 2 + y 2 - 2 Á ˜ x - 2 Á ˜ + 10 = 0
Ë 2¯ Ë 2¯
which is passing through (1, 2), (4, 5) and (0, 9).
Thus when (x, y) = (1, 21) 1 + 4 + 2g + 8f + c = 0 fi x2 + y2 – 5x – 5y + 10 = 0
fi 2g + 8f + c = –5 …(ii) 13. The lengths of the x-intercept
when (x, y) = (4, 5) = 2 g2 - c = 2 9 - 8 = 2
16 + 25 + 8g + 10f + c = 0
fi 8g + 10f + c = –41 …(iii) and the y-intercept
when (x, y) = (0, 9) = 2 f 2 - c = 2 25 - 8 = 2 17
0 + 81 + 18f + c = 0
14. The length of the y-intercept
fi 18f + c = –81 …(iv)
Eqs (iii) – Eqs (ii), we get 1 1
=2 f2-c =2 - 0 = 2 ¥ =1
6g + 2f = –36 4 2
fi 3g + f = –18 …(v) 15. The given equation of a circle is
Eqs (iii) – Eqs (iv), we get x2 + y2 – 2ax – 2ay + a2 = 0
8g – 8f = 40 fi (x – a)2 + (y – a)2 = a2.
fi g–f=5 …(vi) Thus, the centre is (a, a) and the radius is a.
From Eqs (v) and (vi), we get 16. Let N = x2 + y2 – 4x + 2y – 11
4g = –13 The values of N at (1, 2) is
fi g = –13/4 1 + 4 – 4 + 4 – 11 = 5 – 11 = –6 < 0
and f = g – 5 = –13/4 – 5 and at (6, 0) is
36 + 0 – 0 + 12 – 11 = 37 > 0
fi f = –33/4
Thus, the point (1, 2) lies inside of the circle and (6, 0)
Now, from Eq. (iv), we get,
lies outside of the circle.
c = –81 – 18f
17. Since, the point (l, – l) lies inside the circle x2 + y2 – 4x
= –81 – 297/2 = –135/2 + 2y – 8 = 0, so
Put all these values of f, g and c in Eq. (i), we get l2 + l2 – 4l – 2l – 8 < 0
13 33 135 fi 2l2 – 6l – 8 < 0
x2 + y 2 - x - y- =0
2 2 2 fi l2 – 3l – 4 < 0
fi 2(x2 + y2) – 13x – 33y – 135 = 0 fi (l – 4)(l + 1) < 0
12. Let the equation of the circle is fi –1 < l < 4
x2 + y2 + 2gx + 2fy + c = 0 Thus, the range of l is (–1, 4).
which is passing through (1, 2) and (3, 4). 18. Let N = x2 + y2 – 14x – 10y – 151
Now, when (x, y) = (1, 2) 1 + 4 + 2g + 4f + c = 0 The value of N at (2, –7) is
fi 2g + 4f + c = –5 (i) 4 + 49 – 28 + 98 – 151 = 153 – 151 – 28
And, when (x, y) = (3, 4) 9 + 16 + 6g + 8f + c = 0 = 2 – 28 = –26.
fi 6g + 8f + c = –25 (ii) Thus, the point P(2, –7) lies inside of the circle.
Eq. (ii) – Eq. (i), we get The centre of the circle is C(7, 5) and the radius is
4g + 4f = –20 = CA = CB = 15.
fi g + f = –5 (ii) Now,
- 3g - f — 5 2 2 CP = (7 - 2)2 + (5 + 7) 2
Also, = g + f -c
32 + 12 = 25 + 144 = 169 = 13
2 2
= ( g + 1) + ( f + 2) Hence, the shortest distance,
PA = CA – CP = 15 – 13 = 2
fi (3g + f + 5)20 = 10{(g + 1)2 + (g + 3)2} and the longest distance
fi (2g – 5 + 5)2 = 10(g2 + 2g + 1 + g2 + 6g + 9)) = PB = CP + CB = 13 + 15 = 28.
fi 4g2 = 10(2g2 + 8g + 10) 19. The line y = x + 2 touches the circle x2 + y2 = 2, if the
fi 4g 2 + 20g + 25 = 0 length of the perpendicular from the centre to the given
fi (2g + 5)2 = 0 line is equal to the radius of a circle.
fi g = –5/2

CG_03.indd 39 2/6/2017 4:00:23 PM


3.40 Coordinate Geometry Booster

The line (i) will be tangent to the circle x2 + y2 = 4, if


Thus, the radius of the circle is 2 .
the length of the perpendicular from the centre (0, 0) to
The length of the perpendicular from the centre (0, 0)
the circle is equal to the radius (= 2) of the given circle.
to the line x – y + 2 = 0 is
0+0+c
0-0+2 2 Therefore, =2
= = 2 = radius. 12 + 11
2
1 +1 2 2
Hence, the line y = x + 2 touches the circle x2 + y2 = 2. fi c = ±4
On solving y = x + 2 and x2 + y2 = 2, we get x2 + (x + 2)2 Hence, the equation of the tangents becomes
=2 y = ±x ± 4
fi x2 + x2 + 4x + 4 – 2 = 0 24. The equation of any line passing through (7, 1) is
fi 2x2 + 4x + 2 = 0 y – 1 = m(x – 7) …(i)
fi x2 + 2x + 1 = 0 The line (i) will be a tangent to the circle x2 + y2 = 25,
fi (x + 1)2 = 0 if the length of the perpendicular from the centre of the
fi x = –1, y = 1 + 2 = 1 given circle is equal to the radius of the same circle.
Thus, the co-ordinates of the point of contact is (–1, 1). 0 - 0 - (7m - 1)
Therefore, =5
20. The equation of any line parallel to the line m2 + 1
3x + 2y + 5 = 0 is 3x + 2y + k = 0 …(i)
The line (i) will be a tangent to the circle x2 + y2 = 4, fi (7m – 1)2 = 25(m2 + 1)
if the length of the perpendicular from the centre to the fi 12m2 – 7m – 12= 0
line (i) is equal to the radius of the circle. fi 12m2 – 16m + 9m – 12 = 0
3.0 + 2.0 + k fi (3m – 4)(4m + 3) = 0
Therefore, =2
32 + 22 4 3
fi m= ,-
3 4
fi k = ± 2 13 Hence, the equation of the tangents are
Hence, the equation of the tangents to the given circle
4 3
are 3x + 2y ± 2 13 = 0. y - 1 = ( x - 7) and y - 1 = - ( x - 7)
3 4
21. The equation of any line perpendicular to the line
4x + 3y = 0 is 3x – 4y + k = 0 …(i) fi 4x – 3y = 25 and 3x + 4y = 25
The line (i) will be a tangent to the circle x2 + y2 = 9, 25. The centre of a circle is (0, 0) and the radius is 3. Here
if the length of the perpendicular from the centre to the OC = 2.
line (i) is equal to the radius of the circle. Y
3.0 - 4.0 + k
Therefore, =3 x+
32 + 42 y=
4
fi k = ±5
O
Hence, the equation of the tangents are X¢ X
3x – 4y + 15 = 0 and 3x – 4y – 15 = 0 C
22. Let the equation of line be
y = mx + c = 3 x + c …(i)

( m = tan (60°) = 3) We shall find the co-ordinates of C.
The line (i) will be a tangent to the circle x2 + y2 + 4x + Let C be (x, y).
3 = 0, if the length of the perpendicular from the centre
(–2, 0) to the line (i) is equal to the radius (= 1) of a Then x = 2 cos ( - 45°) = 2
circle. and y = 2 sin ( - 45°) = - 2.
-2 3 - 0 + c Thus, the co-ordinates of C becomes ( 2, - 2) .
Therefore, =1
3+1 Hence, the equation of a circle is

fi c - 2 3 = ±2 ( x - 2) 2 + ( y + 2) 2 = 32
fi c=2 3±2 fi x2 + y 2 - 2 2 x - 2 2 y - 5 = 0
Hence, the equation of the tangents becomes 26. When x = 2, then y = ± 5 .
y = 3 x + (2 3 ± 2) Therefore, the points are (2, 5) and (2, - 5) .
23. A line is equally inclined to the co-ordinate axes is Thus, the equation of the tangents at (2, 5) and (2, - 5)
y = ±x + c …(i) are 2x + 5 y = 9 and 2x - 5 y = 9 .

CG_03.indd 40 2/6/2017 4:00:24 PM


Circle 3.41

27. When y = 4, then x = 0. From the graph, it is clear that the equation of the com-
Therefore, the point is (0, 4). mon tangent is x = 2.
Hence, the equation of the tangent to the given circle at x2 + y2 – 2x + 4y – 20 = 0
(0, 4) is x ◊ 0 + y ◊ 4 = 16 31.
fi y = 4. Y
28. The given equations are D
B
4x – 3y = 10 …(i) X¢ O X
2 2
and x + y – 2x + 4y – 20 = 0 …(ii) P
On solving Eqs (i) and (ii), we get Q
C
2
2 Ê 4x - 3 ˆ Ê 4x - 3 ˆ
x +Á - 2x + 4 Á - 20 = 0
Ë 10 ˜¯ Ë 10 ˜¯
A

Let AB: y = x – 4 and CD be a tangent parallel to AB.
fi 9x2 + (4x – 10)2 – 18x + 48x – 120 – 180 = 0
fi 25x2 – 50x – 200 = 0 0-0-8 8
Now OQ = = =4 2
fi x2 – 2x – 8 = 0 12 + 12 2
fi (x – 4)(x + 2) = 0 and OP = radius of a circle = 3.
11 13 Thus, the shortest distance = PQ
fi x = –2, 4 and y = - , .
10 10 = OQ – OP
Hence, the points of intersection are = (4 2 - 3)
Ê 11 ˆ Ê 13 ˆ 32. Hence, the length of the tangent from the point (2, 3) to
ÁË - 2, - ˜¯ and ÁË 4, ˜¯ the circle x2 + y2 = 4 is
10 10
4+9-4 = 9 =3
29. The given circle is x2 + y2 – 2x + 4y = 0
fi (x – 1)2 + (y + 2)2 = 5 …(i) 33. Let P(x1, y1) be any point on the circle x2 + y2 = a2.
2 2 2
The equation of any line passing through (0, 1) is Therefore, x1 + y1 = a .
y – 1 = m(x – 0) …(ii) Now, the length of the tangent from (x1, y1) to the circle
The line (ii) will be a tangent to the circle (i), then the x2 + y2 = b2 is
length of the perpendicular from the centre (1, –2) to
the line (ii) is equal to the radius of a circle (i). x12 + y12 - b 2 = a 2 - b 2 .
m + 2 +1 34. Let P(x1, y1) be any point on
Therefore, = 5
2 x2 + y2 + 2011x + 2012y + 2013 = 0
m +1
Therefore,
fi (m + 3)2 = 5(m2 + 1)
fi m2 + 5m + 9 – 5m2 – 5 = 0 x12 + y12 + 2011x1 + 2012y1 + 2013 = 0
fi 2m2 – 3m – 2 = 0 Now, the length of the tangent from (x1, y1) to the circle
x2 + y2 + 2011x + 2012y + 2014 = 0 is
fi (m – 2)(2m + 1) = 0
1 x12 + y12 + 2011x1 + 2012y1 + 2014
fi m=- ,2
2 = - 2013 + 2014
Hence, the equation of the tangents are =1
1
y - 1 = - x and y - 1 = 2x 35. The power of a point (2, 5) with respect to a circle
2 x2 + y2 = 16 is (4 + 25 – 16) = 13.
fi x + 2y – 1 = 0 and 2x – y + 1 = 0 36. Let the point P represents a complex number Z. i.e. Z =
30. The given curves are x2 + y2 = 4 and y2 = 4(x – 2). 1+ 2i
Then, from the rotation theorem,
Y
Q = iZ = i(1 + 2i) = i – 2 = –2 + 1.i = (–2, 1)
Thus, the power of a point Q with respect to a circle is
4 + 1 – 4 = 1.
X 37. Hence, the equation of a pair of tangents to a circle
O x2 + y2 = 4 from a point (1, 2) is SS1 = T 2
fi (x2 + y2 – 4)(1 + 4 – 4) = (x + 2y – 4)2
fi (x2 + y2 – 4) = (x2 + 4y2 + 16 + 4xy – 8x – 16y)
X=2

CG_03.indd 41 2/6/2017 4:00:24 PM


3.42 Coordinate Geometry Booster

fi 3y2 + 4(x – 4)y + 4(5 – 2x) = 0 43. Clearly, the equation x2 + y2 = 20 is the director circle
of the circle x2 + y2 = 10.
- 4( x - 4) ± 16( x - 4) 2 - 48(5 - 2x)
fi y= Hence, the angle between the pair of tangents is 90°.
6 44. (i) The given circle x2 + y2 + 2x = 0 can be reduced to
- 4( x - 4) ± 4 x 2 - 8x + 16 - 15 + 6x (x + 1)2 + y2 = 1.
fi y= Hence, the equation of the director circle to the
6
circle (x + 1)2 + y2 = 1 is
- 4( x - 4) ± 4( x - 1) (x + 1)2 + y2 = 2 ¥ 1 = 2
fi y=
6 fi x2 + y2 + 2x – 1 = 0
fi y = 2, 4x – 3y + 2 = 0 (ii) The given circle x2 + y2 + 10y + 24 = 0 can be re-
38. Hence, the equation of the pair of tangents are SS1 = T2 duced to x2 + (y + 5)2 = 1
fi (x2 + y2 – 4x + 3)(4 + 9 – 8 + 3) Hence, the equation of the director circle to the
= (2x + 3y – 2(x + 2) + 3)2 circle x2 + (y + 5)2 = 1 is
fi 8(x2 + y2 – 4x + 3) = (3y – 1)2 x2 + (y + 5)2 = 2 ¥ 1 = 1
fi x2 + y2 + 10y + 23 = 0
fi 8(x2 + y2 – 4x + 3) = 9y2 – 6y + 1
(iii) The given circle
fi y2 – 8x2 + 32x – 6y – 23 = 0 x2 + y2 + 16x + 12y + 99 = 0
fi y2 – 6y – (8x2 – 32x + 23) = 0 can be reduced to
6 ± 36 + 4(8x 2 - 32x + 23) (x + 8)2 + (y + 6)2 = 64 + 36 – 99 = 1
fi y= Hence, the equation of the director circle to the
2 circle (x + 8)2 + (y + 6)2 = 1
6 ± 32x 2 - 128x + 128 is (x + 8)2 + (y + 6)2 = 2 ¥ 1 = 2
fi y= fi x2 + y2 + 16x + 12y + 98 = 0
2
(iv) The given circle
6 ± 32( x 2 - 4x + 4) x2 + y2 + 2gx + 2fy + c = 0 can be reduced to
=
2 ( x + g )2 + ( y + f )2 = ( g 2 + f 2 - c )2
6 ± 4 2 ( x - 2)
= Hence, the equation of the director circle to the
2
2 2 2 2 2
circle ( x + g ) + ( y + f ) = ( g + f - c ) is
fi y = 3 ± 2 2( x - 2)
(x + g)2 + (y + f )2 = 2(g2 + f 2 – c)
39. Hence, the angle between the tangent from the point
(3, 5) to the circle x2 + y2 = 25 is fi x2 + y2 + 2gx + 2fy – g2 – f 2 + 2c = 0.
(v) The given circle x2 + y2 – ax – by = 0 can be re-
Ê a ˆ Ê 5 ˆ
2 tan -1 = 2 tan -1 2
ÁË S ˜¯ ÁË 9 + 25 - 25 ˜¯ Ê aˆ
2
Ê bˆ
2 Ê
a 2 + b2 ˆ
1
duced to Á x - ˜ + Á y - ˜ = Á ˜¯
Ë 2¯ Ë 2¯ Ë 2
Ê 5ˆ
= 2 tan -1 Á ˜
Ë 3¯ Hence, the equation of the director circle to the
40. After translation the point (1, 2) becomes circle
2
2 Ê
a 2 + b2 ˆ
(1+2, 2) = (3, 2). 2
Ê aˆ Ê bˆ
Hence, the angle between the tangent from the point ÁË x - ˜ + ÁË y - ˜ = Á ˜¯ is
2¯ 2¯ Ë 2
(3, 2) to the circle x2 + y2 = 9 is
2 2
Ê a ˆ Ê 3 ˆ Ê aˆ Ê bˆ Ê a 2 + b2 ˆ
2 tan -1 = 2 tan -1 ÁË x - ˜¯ + ÁË y -
˜¯ = ÁË ˜
ÁË S ˜¯ ÁË 9 + 4 - 9 ˜¯ 2 2 2 ¯
1
1
Ê 3ˆ fi x 2 + y 2 - ax - by - (a 2 + b 2 ) = 0
= 2 tan -1 Á ˜ 4
Ë 2¯
45. When x = 2, y = ± 5 .
41. The locus of the point of intersection of two perpen-
dicular tangents to a circle is the director circle of the So, the points are (2, 5) and (2, - 5) .
given circle. The equation of the normal to the circle at
Thus, the equation of the director circle to the circle (2, 5) and (2, - 5) are
x2 + y2 = 25 is x2 + y2 = 2 ¥ 25 = 50 x y x y
= and = -
42. Clearly, the locus of the arbitrary point is the director 2 5 2 5
circle of the given circle x2 + y2 = 9.
Hence, the equation of the director circle to the circle 5 5
fi y= x and y = - x
x2 + y2 = 9 is x2 + y2 = 2 ¥ 9 = 18. 2 2

CG_03.indd 42 2/6/2017 4:00:25 PM


Circle 3.43

46. The equation of the normal to the given circle at (–2, 1) Since, the line (i) makes a right angle at the centre, so
is (a2 – h2) + (a2 – k2) = 0
x - x1 y - y1 fi h2 + k2 = 2a2
fi =
x1 + g y1 + f which is the required condition.
x + 2 y -1 52. Let AB be the chord of contact.
=
—2 + 1 1 + 2 Then AB : hx + ky = a2
fi 3x + 6 = –y + 1 h2 + k 2 - a 2
fi 3x + y + 5 = 0 Now, PM =
h2 + k 2
47. The centre of a circle x2 + y2 – 4x – 6y + 4 = 0 is (2, 3).
The equation of a normal parallel to x – y – 3 = 0 is The length of the chord of contact,
x–y+k=0 …(i) 2LR
AB = ,
As we know that the normal always passes through the L2 + R 2
centre of a circle.
Therefore, 2 – 3 + k = 0 fi k = 1. where L = the length of the tangent and R be the radius
Hence, the equation of the normal is of the circle.
x–y+1=0 Therefore,
48. The centre of a circle 2a ¥ h 2 + k 2 - a 2
AB =
x2 + y2 – 8x – 12y + 99 = 0 is (4, 6). (h 2 + k 2 - a 2 ) + a 2
The equation of the normal which is perpendicular to
2a ¥ h 2 + k 2 - a 2
2x – 3y + 10 = 0 is 3x + 2y – k = 0 …(i) =
h2 + k 2
which is passing through (4, 6).
Thus, the area of the D PAB
Therefore, 12 + 12 – k = 0 fi k = 24.
1 Ê 2a ¥ h + k - a ˆ Ê h 2 + k 2 - a 2 ˆ
2 2 2
Hence, the equation of the normal is 3x + 2y = 24.
= ¥Á ˜ ¥Á
49. The equation of the chord of contact of tangents from 2 ÁË h2 + k 2 ˜¯ Ë h 2 + k 2 ˜¯
(5, 3) to the circle x2 + y2 = 25 is 5x + 3y = 25.
50. Let the point be (x1, y1). Ê (h 2 + k 2 - a 2 )3/2 ˆ
= aÁ ˜
Clearly, 2x + y + 12 = 0 ...(i) Ë (h 2 + k 2 ) ¯
is the chord of contact of the tangents to the circle 53. Let P(a cos q, a sin q) be any point on the circle x2 + y2
x2 + y2 – 4x + 3y – 1 = 0 from (x1, y1). = a2.
The equation of the chord of contact of tangents to the Therefore, QR:
circle x2 + y2 – 4x + 3y – 1 = 0 from the point (x1, y1) is ax cos q + ay sin q = b2 …(i)
3 Since the line (i) be a tangent to the circle x + y = c2,
2 2
xx1 + yy1 – 2(x + x1) + (y + y 1) – 1 = 0 …(ii)
2 so
Clearly Eqs (i) and (ii) are identical. 0 + 0 - b2
=c
3 3 a 2cos 2q + a 2sin 2q
x1 - 2 y1 + 2 2 y1 - 2x1 - 1
Therefore, = = fi b2 = ac
2 1 12
Thus, m = 2, n = 1 and p = 1.
fi x1 – 2y1 = 5 and 4x1 + 21y1 = –38 Hence, the value of m + n + p + 10 = 14.
fi x1 = 1, y1 = –2 54. The equation of the chord of the circle x2 + y2 = 25
Hence, the point is (1, –2). bisected at (–2, 3) is
T = S1
51. The equation of the chord of contact QR is
fi –2x + 3y – 25 = 4 + 9 – 25
hx + ky = a2 …(i) fi –2x + 3y = 13
The chord of contact subtends a right angle at the cen- fi 2x – 3y + 1 3 = 0
tre of the circle x2 + y2 = a2. 55. Hence, the equation of the chord of the circle x2 + y2 +
2
x 2 + y 2 Ê hx + ky ˆ 6x + 8y – 11 = 0, whose mid-point is (1, –1) is T = S1

Ë a 2 ˜¯ fi x – y + 3(x + 1) + 4(y – 1) – 11
Therefore,
a2
= 1 + 1 + 6 + 8 –11
fi a2(x2 + y2) = (hx + ky)2 fi 4x + 3y – 1 = 16
fi x2 (a2 – h2) + y2(a2 – k2) – 2hkxy = 0 fi 4x + 3y = 17

CG_03.indd 43 2/6/2017 4:00:25 PM


3.44 Coordinate Geometry Booster

56. Let the chord AB is bisected at C(x1, y1) 61. Obviously, the locus of the mid-point of the chords of
the circle is the diameter of the given circle.
The equation of AB is xx1 + yy1 = x12 + y12 Here, the centre of the circle x2 + y2 – 4x – 6y = 0 is
which is passing through (h, k), so (2, 3).
hx1 + ky1 = x12 + y12 The equation of the diameter of the circle, which is per-
pendicular to the line 4x + 5y + 10 = 0 is 5x – 4y + k = 0
Thus, the locus of (x1, y1) is which is passing through (2, 3).
x2 + y2 = hx + ky Therefore, k = 12 – 10 = 2.
57. Let M(h, k) is the mid-point of AB Hence, the equation of the diameter is
In DOAM, 5x – 4y + 2 = 0
OM OM 62. The equation of the common chord of the circle is
sin (45°) = = O 2x – 2y = 0
AM 2
OM = 2 sin (45°) fi x=y
fi A B
M On solving y = x and the circle
1 x2 + y2 + 3x + 5y + 4 = 0,
=2¥ = 2
2 we get the points of intersection.
fi OM 2 = 2 Thus, x = - 2 + 2 = y and x = - 2 - 2 = y
fi h2 + k2 = 2 Let the common chord be PQ, where
Hence, the locus of (h, k) is x2 + y2 = 2.
58. Let A represents a complex number Z. P = ( - 2 + 2, - 2 + 2) and
i.e. Z = 3 + i Q = ( - 2 - 2, - 2 - 2)
Here, –AOB = 90°. Thus, the length of the common chord, PQ
By rotation theorem, we can say that
= ( - 2 - 2 + 2 - 2) 2 + ( - 2 - 2 + 2 - 2) 2
B = iZ = i ( 3 + i ) = –1 + i 3
Hence, the point B is (-1, 3). = 8 + 8 = 16 = 4
59. Let C represents a complex number Z and D be Z1. 63. The equation of the common chord of the given circles
Here, –COD = 60° is 2x + 1 = 0.
Z - 0 Z1 - 0 -i p
By rotation theorem, 1 = ¥e 3 On solving 2x + 1 = 0 and
Z -0 Z -0
x2 + y2 + 2x + 3y + 1 = 0,
Z1 |Z1| -i p -i p
fi = ¥e 3 =e 3 we get the points of intersection.
Z |Z |
Thus, the points of intersection are
- i p3
fi Z1 = Ze 1 3
x = - and y = - ± 2
Ê1 3ˆ 2 2
fi Z1 = (2 2 + i ) Á - i ˜
Ë2 2 ¯ Let PQ be the common chord, where
Ê 3ˆ Ê1 ˆ Ê 1 3 ˆ
fi Z1 = Á 2 + ˜¯ + i Á - 3˜ P = Á — , - + 2 ˜ and
Ë 2 Ë2 ¯ Ë 2 2 ¯
Hence, the co-ordinates of D are Ê 1 3 ˆ
Ê ˆ Q = Á — , - - 2˜
3 1 Ë 2 2 ¯
Á 2 + 2 , 2 - 3˜
Ë ¯ The mid-point of P and Q is the centre of the new cir-
60. As we know that the locus of the mid-points of the cle.
chord of the circle is the diameter of the circle. Thus, Ê 1 3ˆ
Thus, centre is C = Á — , - ˜
the equation of the diameter which is parallel to Ë 2 2¯
2012x + 2013y + 2014 = 0
is 2012x + 2013y + k = 0 Now, radius r = CP = 2.
which is passing through the centre of the circle x2 + y2 Hence, the equation of the circle is
= 9. 2 2
Ê 1ˆ Ê 3ˆ
Therefore, k = 0 ÁË x + ˜ + ÁË y + ˜ =2
Hence, the equation of the diameter is 2¯ 2¯
2012x + 2013y = 0 fi 2(x2 + y2) + 2x + 6y + 1 = 0

CG_03.indd 44 2/6/2017 4:00:26 PM


Circle 3.45

64. (i) The given circles are x2 + y2 = 4 …(i) (vi) The given circles are
2 2
and x + y – 2x = 0 (ii) x 2 + y 2 - 2(1 + 2) x - 2y + 1 = 0
Now, C1(0, 0) and C2(1, 0) and r1 = 2 and r2 = 1. and x2 + y2 – 2x – 2y + 1 = 0
Thus, C1C2 = 1 = r2 – r1 Now C1: (1 + 2, 1) , C2: (1, 1),
Hence, both the circles touch each other internally. r1 = 2 + 1 and r2 = 1.
Thus, the number of common tangent = 1.
Therefore, C1C2 = ( 2) 2 = 2
(ii) The given circles are x2 + y2 + 4x + 6y + 12 = 0
and r1 – r2 = 2 .
and x2 + y2 – 6x – 4y + 12 = 0
Thus,C1C2 = r1 – r2
Now, C1: (–2, –3), C2: (3, 2), r1 = 1 and r2 = 1
Hence, the number of common tangents = 1.
Therefore,
65. The given circles are
2 2 x2 + y2 – 2x – 6y + 9 = 0
C1C2 = (3 + 2) + ( - 3 - 2)
and x2 + y2 + 6x – 2y + 1 = 0
= 25 + 25 = 50 = 5 2 Now, C1: (1, 3), C2: (–3, 1), r1 = 1 and r2 = 3.
and r1 + r 2 = 1 + 1 = 2. Therefore,
Thus, C1C2 > r1 + r2 2 2
C1C2 = ( - 3 - 1) + (1 - 3)
Hence, the number of common tangents = 4.
= 16 + 4 = 20 = 2 5
(iii) The given circles are
and r1 + r2 = 1 + 3 = 4
x2 + y2 – 6x – 6y + 9 = 0
andx2 + y2 + 6x + 6y + 9 = 0 Then, C1C2 > r1 + r2
Now, C1 = (3, 3), C2 = (–3, –3), r1 = 3 and r2 = 3. Thus, both the circles do not intersect.
Therefore, Hence, the number of common tangents = 4, in which
2 are direct common tangents and another 2 are trans-
C1C2 = (—3 - 2) 2 + ( -3 - 3) 2 verse common tangents.
= 36 + 36 = 72 = 6 2 Here, the point M divides C2(–3, 1) and C1(1, 3) inter-
and r1 + r2 = 3 + 3 = 6 nally in the ratio 3 : 1.
Thus, C1C2 > r1 + r2 Then, the co-ordinates of M are
Hence, the number of common tangents = 4. Ê 3.1 - 1.3 3.3 + 1.1ˆ Ê 5ˆ
(iv) The given circles are ÁË 3 + 1 , 3 + 1 ˜¯ = ÁË 0, ˜

x2 + y2 – 4x – 4y = 0
Also, the point P divide C2(–3, 1) and C1 (1, 3) exter-
and x2 + y2 + 2x + 2y = 0
nally in the ratio 3 : 1.
Now, C1 = (2, 2), C2 = (–1, –1), r1 = 2 2 and Then, the co-ordinates of P are
r2 = 2 Ê 3.1 + 1.3 3.3 - 1.1ˆ
Therefore, ÁË 3 - 1 , 3 - 1 ˜¯ = (3, 4)
2 2
C1C2 = ( -1 - 2) + ( -1 - 2) Case I: Direct common tangents
= 9 + 9 = 18 = 3 2 Any line through (3, 4) is
y – 4 = m(x – 3) …(i)
and r1 + r2 = 2 2 + 2 = 3 2
fi mx – y + 4 – 3m = 0
Thus, C1C2 = r1 + r2 If it is a tangent to the circle,
Hence, the number of common tangents = 3.
m - 3 + 4 - 3m
(v) The given circles are x2 + y2 = 64 =1
m2 + 1
and x2 + y2 – 4x – 4y + 4 = 0
Now, C1: (0, 0), C2: (2, 2), r1 = 8 and r2 = 2 fi (1 – 2m)2 = (m2 + 1)
Therefore, fi 3m2 – 4m = 0
2 2
fi m(3m – 4) = 0
C1C2 = (2 - 0) + (2 - 0) 4
fi m = 0, .
= 4+4 =2 2 3
and r1 + r 2 = 8 + 2 = 10 Hence, the direct common tangents are
Therefore, C1C2 < r1 + r2 y = 4, 4x – 3y = 0.
Thus, one circle lies inside of the other. Case II: Transverse common tangents
Hence, the number of common tangents = 0. Any line through (0, 5/2) is

CG_03a.indd 45 2/6/2017 4:01:00 PM


3.46 Coordinate Geometry Booster

5 fi –21 < – n2 < 39


y - = m( x - 0) …(i) fi –39 < n2 < 21
2
fi 0 < n < 21
fi 2mx – 2y + 5 = 0
fi n = 1, 2, 3, 4
If it is a tangent to a circle, then
5 Case II: When 5 < 3 + 25 - n 2
m ◊1 - 3 +
2 =1 Then, 25 - n 2 > 2
m2 + 1 fi 25 – n2 > 4
fi (2m – 1)2 = 4(m2 + 1) fi n2 < 21
fi 4m2 – 4m + 1 = 4(m2 + 1) fi n < 21
fi 4m + 3 = 0
fi n = 1, 2, 3, 4.
3
fi m ,m Hence, the number of integers = 4.
4 68. The given circles are
Hence, the equation of transverse tangents are x = 0 and x2 + y2 + x + y = 0
3x + 4y = 10. and x2 + y2 + x – y = 0
66. Given circles are x2 + y2 + c2 = 2ax Here, g1 = –1/2, f1 = –1/2, g2 = –1/2, f2 = 1/2, c1 = 0,
fi ( x - a)2 + y 2 = ( a 2 - c 2 )2 …(i) c2 = 0
Now, 2(g1 g2 + f1 f2) = 2(–1/4 + 1/4) = 0 and c1 + c2 = 0.
and x2 + y2 + c2 = 2by Hence, the angle between the given circles is 90°.
fi x 2 + ( y — b) 2 = ( b 2 - c 2 ) 2 …(ii) 69. The given circles are
x2 + y2 + 2a1x + 2b1y + c1 = 0
Now C1 : (a, 0), C2 : (0, b), and x2 + y2 + a2x + b2y + c2 = 0
r1 = a 2 - c 2 and r2 = b 2 - c 2 Here, g1 = a1, f1 = b1, g2 = a2/2, f2 = b2/2, c1 = c1 and c2
= c2
Since both the circles touch each other externally, then Since the given two circles are orthogonal, so
C1C2 = r1 + r2 2(g1g2 + f1f2) = c1 + c2
fi a 2 + b2 = a 2 - c2 + b2 - c2 Ê a b ˆ
fi 2 Á a1 ◊ 2 + b1 ◊ 2 ˜ = c1 + c2
Ë 2 2¯
fi a2 + b2 = a2 – c2 + b2 – c2 + 2( a 2 - c 2 )( b 2 - c 2 )
fi (a1 ◊ a2 + b1 ◊ b2) = c1 + c2
fi 2c 2 = 2( a 2 - c 2 )( b 2 - c 2 ) Hence, the result.
fi c4 = (a2 – c2)(b2 – c2) = a2b2 – c2(a2 + b2) + c4 70. The given circles are
fi a2b2 – c2(a2 + b2) = 0 2x2 + 2y2 – 3x + 6y + k = 0
1 1 1 3 k
fi 2
+ 2 = 2 fi x 2 + y 2 - x + 3y + = 0
a b c 2 2
67. The given circles are 2 2
and x + y – 4x + 10y + 16 = 0
x2 + y2 = 9 …(i) Since, the two given circles are orthogonal, then
and x2 + y2 – 8x – 6y + n2 = 0 2(g1g2 + f1f2) = c1 + c2
fi ( x - 4) 2 + ( y - 3) 2 = ( 25 - n 2 ) 2 …(ii) Ê 3 3 ˆ k
fi 2 Á - ¥ - 2 + ¥ 5˜ = + 16
Ë 4 2 ¯ 2
Here, C1: (0, 0), C2: (4, 3), r1 = 3 and r2 = 25 - n 2
k
Since the circles have only two common tangents, so fi + 16 = 18
2
we can write,
k
|r1 – r2| < C1C2 < r1 + r2 fi = 18 — 16
2
fi |3 - 25 - n 2 | < 5 < 3 + 25 - n 2 fi k=4
Hence, the value of k is 4.
Case I: When |3 - 25 - n 2 | < 5
71. Let the equation of the circle be
Then - 5 < 3 - 25 - n 2 < 5 x2 + y2 + 2gx + 2fy = 0 …(i)
2
Therefore, the centre of the circle is (–g, – f).
fi - 5 - 3 < - 25 - n < 5 — 3 Since, the centre lies on the line y = x, so
fi — 2 < 25 - n 2 < 8 –f = –g
fi f=g
fi 4 < 25 – n2 < 64

CG_03a.indd 46 2/6/2017 4:01:01 PM


Circle 3.47

The Eq. (i) is orthogonal to 75. Hence, the equation of the radical axis is
x2 + y2 – 4x – 6y + 18 = 0, (x2 + y2 + 4x + 6y + 9) –
so, 2[g(–2) + f(–3)] = 0 + 18 (x2 + y2 + 3x + 8y + 10) = 0
fi 2(–5g) = 18 ( f = g) fi (4x – 3x) + (6y – 8y) + (9 – 10) = 0
9 fi x – 2y – 1 = 0
fi g=- = f
5 76. Hence, the equation of the radical axis is
Hence, the equation of the circle is (x2 + y2 + 8x + 2y + 10) – (x2 + y2 + 7x + 3y + 10) = 0
18 18 fi (8x – 7x) + (2y – 3y) = 0
x2 + y 2 - x - y = 0 fi y=x
5 5
Hence, the image of (2, 3) with respect to the line y = x
72. Let the equation of the circle be is (3, 2).
x2 + y2 + 2gx + 2fy + c = 0 …(i) 77. Let S1 : x2 + y2 = 1 …(i)
The equation of the circle (i) is orthogonal to S2 : x2 + y2 – 8x + 15 = 0 …(ii)
x2 + y2 + 4x – 6y + 9 = 0 and S3 : x2 + y2 + 10y + 24 = 0 …(iii)
and x2 + y2 – 5x + 4y – 2 = 0 Eq. (i) – Eq. (ii), we get
Thus, (4g – 6f) = c + 9 …(ii) 8 x = 16 fi x = 2
and (–5g + 4f) = c – 2 …(iii) Eq. (i) – Eq. (iii), we get
Subtracting, we get –9g + 10f + 11 = 0 –10 y = 25 fi y = –5/2
fi 9(–g) – 10(–f) + 11 = 0
Ê 5ˆ
Hence, the locus of the centre is Hence, the radical centre is Á 2, - ˜ .
Ë 2¯
9x – 10y + 11 = 0
73. Let the equation of the circle be 78. Let the equation of the circle be
x2 + y2 + 2gx + 2fy + c = 0 …(i) x2 + y2 + 2gx + 2fy + c = 0 …(i)
which is passing through (a, b). The circle (i) is orthogonal to
Therefore, x2 + y2 – 3x – 6y + 14 = 0,
a2 + b2 + 2ga + 2fb + c = 0 …(ii) x2 + y2 – x – 4y + 8 = 0,
It is given that the circle (i) is orthogonal to x2 + y2 = 4, and x2 + y2 + 2x – 6y + 9 = 0
so 2(g ◊ 0 + f ◊ 0) = c – 4 fi c = 4 Therefore,
From Eq. (ii), we get Ê Ê 3ˆ ˆ
2 Á g ◊ Á - ˜ - 3f ˜ = c + 14
a2 + b2 + 2g ◊ a + 2f ◊ b + 4 = 0 Ë Ë 2¯ ¯
Hence, the locus of (–g, –f) is fi (–3g – 6f) = c + 14 …(ii)
2ax + 2by – (a2 + b2 + 4) = 0
74. Let C1 and C2 be the centres of the two orthogonal cir- È Ê 1ˆ ˘
2 Í g ◊ Á - ˜ - 2f ˙ = c + 8
cles with radii r1 and r2, respectively. Î Ë 2 ¯ ˚
Here –C1PC2 = 90° and let fi (2g – 4f) = c + 8 …(iii)
–PC1C2 = q and –PC2C1 = 90° – q and
Thus, 2(g ◊ 1 – 3 ◊ f) = c + 9
PM PM fi (2g – 6f) = c + 9 …(iv)
sin (q ) = and sin (90° - q ) = .
r1 r2 Eq. (iii) – Eq. (ii), we get
Squaring and adding, we get 5g + 2f = –6 …(v)
2 2
Eq. (iii) – Eq. (iv), we get
Ê PM ˆ Ê PM ˆ 2f = – 1 fi f = –1/2
fi ÁË r ˜¯ + ÁË r ˜¯ = 1
1 2 Put the value of f in Eq. (v), we get,
Ê 1 1ˆ g = –1
PM 2 Á 2 + 2 ˜ = 1 Also, put the values of f and g in Eq. (iv), we get
Ë r1 r2 ¯
c = –8.
r12 r22 Hence, the equation of the circle is
fi PM 2 =
r1 + r22
2 x2 + y2 – 2x – y – 8 = 0
r1r2 79. Equation of any circle passing through the point of in-
fi PM = tersection of the circles
r12 + r22 x2 + y2 + 13x – 3y = 0
Hence, the length of the common chord, and 2x2 + 2y2 + 4x – 7y – 25 = 0
2r1r2 is (x2 + y2 + 13x – 3y)
PQ = 2PM = + l(2x2 + 2y2 + 4x – 7y – 25) = 0 …(i)
r12 + r22
which is passing through (1, 1).

CG_03a.indd 47 2/6/2017 4:01:01 PM


3.48 Coordinate Geometry Booster

Therefore, Ê l - 3ˆ
(1 + 1 + 13 – 3) + l(2 + 2 + 4 – 7 – 25) = 0 fi x2 + y 2 + 2 Á x
Ë l + 1 ˜¯
fi 12 – 24l = 0
1 Ê 1 - 2l ˆ Ê 2(2 - 3l ) ˆ
fi l= + 2Á ˜ y+Á =0
2 Ë l +1 ¯ Ë l + 1 ˜¯
Put the value of l in Eq. (i), we get
Ê 3 — l 2l — 1ˆ
1 So its centre is Á , .
( x 2 + y 2 + 13x - 3y ) + (2x 2 + 2y 2 + 4x - 7y - 25) = 0 Ë l + 1 l + 1 ˜¯
2
Since, the centre lies on the line y = x,
fi (2x2 + 2y2 + 26x – 6y) + (2x2 + 2y2 + 4x – 7y – 25) = 0
2l — 1 3 — l
fi (4x2 + 4y2 + 30x – 13y – 25) = 0 =
l +1 l +1
80. The equation of radical axis is
(x2 + y2 + 2x + 3y + 1) fi 3l = 4
–(x2 + y2 + 4x + 3y + 2) = 0 fi l = 4/3
fi (2x + 3y + 1) – (4x + 3y + 2) = 0 Hence, the equation of the circle is
fi 2x + 1 = 0 10 10 12
x2 + y 2 - x - y - = 0.
1 7 7 7
fi x=-
2 83. The given circles are x2 + y2 + 2x + 3y + 1 = 0 and
Thus, the equation of the circle is x2 + y2 + 4x + 3y + 2 = 0.
(x2 + y2 + 2x + 3y + 1) + l(2x + 1) = 0 Hence, the equation of the common chord is 2x + 1 = 0.
Therefore, the equation of the circle is
fi (x2 + y2 + 2(l + 1)x + 3y + (1 + l)) = 0
(x2 + y2 + 2x + 3y + 1) + l(x2 + y2 + 4x + 3y + 2) = 0
Since AB is diameter of the circle, so the centre lies on …(i)
it. fi (1 + l)x2 + (1 + l)y2 + 2(1 + 2l)x + 3(1 + l)y
Therefore, –2l – 2 + 1 = 0 + (1 + 2l) = 0
1 Ê 1 + 2l ˆ (1 + 2l )
fi l=- . fi x2 + y 2 + 2 Á x + 3y + =0
2 Ë (1 + l ) ˜¯ (1 + l )
Hence, the equation of the circle is Ê 1 + 2l 3 ˆ
So, its centre is Á - ,- ˜
1 Ë l +1 2¯
( x 2 + y 2 + 2x + 3y + 1) - (2x + 1) = 0
2 Since 2x + 1 = 0 is the diameter, so centre lies on it.
fi 2 2
(x + y + 2x + 6y + 1) = 0 Therefore,
Ê 1 + 2l ˆ
81. Any circle passing through the point of intersection of 2Á - +1= 0
Ë l + 1 ˜¯
the given line and the circle is
x2 + y2 – 9 + l(x + y – 1) = 0 fi –2 – 4l + l + 1 = 0
fi x2 + y2 + lx + ly – (9 + l) = 0 1
fi l=-
3
Ê l lˆ
So the centre is Á - , - ˜ . Hence, the equation of the circle (i) is
Ë 2 2¯
Ê l lˆ 2(x2 + y2) + 2x + 6y + 1 = 0
Since, the circle is smallest, so the centre Á - , - ˜
Ë 2 2¯
lies on the chord x + y = 1.
l l
Therefore, - - = 1 fi l = –1. LEVEL III
2 2
Hence, the equation of the smallest circle is 1. Given circle is x2 + y2 = 16.
x2 + y2 – 9 – (x + y – 1) = 0 Y
fi x2 + y2 – x – y – 8 = 0
82. The equation of any circle passing through the point of
intersection of the given circle is
(x2 + y2 – 6x + 2y + 4) + l(x2 + y2 + 2x – 4y – 6) = 0 X¢ O X
fi (1 + l)x2 + (1 + l)y2 + 2(l – 3)x + 2(1 – 2l)y
+ 2(2 – 3l) = 0

CG_03a.indd 48 2/6/2017 4:01:02 PM


Circle 3.49

Thus, the number of integral points inside the circle 4. Given circle is
=5+7+7+7+7+7+5 x2 + y2 – 10x – 6y + 30 = 0 S R
= 45 fi (x – 5)2 + (y – 3)2
C(5, 3)
2. Let S1: x2 + y2 + 2gx + 2fy + c = 0 = 25 + 9 – 30
S2: x2 + y2 = 4 fi (x – 5)2 + (y – 3)2 = 4
S3: x2 + y2 – 6x – 8y + 10 = 0 P Q
Clearly, CP = radius = 2
and S4: x2 + y2 + 2x – 4y – 2 = 0 Let PQ is parallel to y = x + 3
Now, S1 – S2 = 0 Therefore, the co-ordinates of P and R are obtained by
2gx + 2fy + c + 4 = 0 …(i)
x-5 y-3
Centre of a circle x2 + y2 = 4 lies on (i) fi = = ±2
So, c + 4 = 0 cos (45°) sin (45°)
c = –4 …(ii) x-5 y-3
Again, S1 – S3 = 0 fi = = ±2
1 1
2(g + 3)x + 2(f + 4)y + (c – 10) = 0 …(iii)
2 2
So, the centre of a circle x2 + y2 – 6x – 8y + 10 = 0 lies
2
on (ii), so fi x -5= y -3= ±
fi 2(g + 3)3 + 2(f + 4)4 + (–4 – 10) = 0 2
fi 2(g + 3)3 + 2(f + 4)4 + –14 = 0 fi x -5= y -3= ± 2
fi 6g + 8f + 18 + 32 – 14 = 0
fi 6g + 8f + 36 = 0 fi x = 5 ± 2, y = 3 ± 2
fi 3g + 4f + 18 = 0 …(iv) Thus, R = (5 + 2, 3 + 2)
Also, S1 – S4 = 0
and Q = (5 - 2, 3 - 2) .
fi 2(g – 1)x + 2(f + 2)y + (c + 2) = 0
So, centre (–1, 2) lies on (iii). 5. Clearly,
Thus, –2(g – 1) + 4(f + 2) + (–4 + 2) = 0 (2 2) 2 + (2 2) 2 = a 2
fi –2(g – 1) + 4(f + 2) – 2 = 0 fi a 2 = (2 2) 2 + (2 2) 2
fi (g – 1) – 2(f + 2) + 1 = 0
fi a2 = 8 + 8 = 16
fi g – 2f = 4 …(v) fi |a| = 4
On solving, we get 6. Given circle is
g = –2 and f = –3 x2 + y2 – 8x + 2y + 12 = 0
Hence, the equation of the circle is fi (x – 4)2 + (y + 1)2 = 16 + 1 – 12
x2 + y2 – 4x – 6y – 4 = 0 fi (x – 4)2 + (y + 1)2 = 5
3. Given circle is (x ± a)2 + (y ± a)2 = a2 As we know that the line is a chord, tangent or does not
Y meet the circle at all, if
p < r, p = r or p > 4,
where p is the length of the perpendicular from the cen-
R(a, –a) Q(a, –a) tre to the line.
4 + 2 -1 5
So, p = = = 5=r
X¢ X 1+ 4 5
Thus, the line be a tangent to the circle.
S(–a, –a) P(–a, –a) 7. It is given that
C1C2 = r1 + r2

fi (a - b) 2 + (b - a ) 2 = c + c

Hence, the radius of the smallest circle fi 2(a - b) 2 = 2c


fi 2(a – b)2 = 4c2
= ( a 2 + a 2 - a ), ( a 2 + a 2 + a )
fi (a – b)2 = 2c2
= a 2 - a, a 2 + a
fi a - b = ±c 2
= ( 2 - 1)a, ( 2 + 1)a fi a=b±c 2

CG_03a.indd 49 2/6/2017 4:01:02 PM


3.50 Coordinate Geometry Booster

8. Given circle is 10. The equation of the chord of contact of tangents to the
x2 + y2 + 4x – 6y – 12 = 0 circle
(x + 2)2 + (y – 3)2 = 52 x2 + y2 + 2gx + 2fy + c = 0
Thus, C = (–2, 3) and CA = 5 = CB from (0, 0) is
p x ◊ 0 + y ◊ 0 + g(x + 0) + f(y + 0) + c = 0
and –CAB =
3 gx + fy + c = 0 …(i)
Hence, the required distance from the point (g, f) to the
Ê ˆ MC
p
Now, sin Á ˜ = chord of contact (i)
Ë 3¯ 5 C(–2, 3)
Ê g2 + f 2 + cˆ
3 MC = Á
2 ˜
.
fi = 2
Ë g + f ¯
2 5
A M(h, k) B 11. We have,
5 3
fi MC = (1 + ax)n = 1 + 8x + 24x2 + ◊◊◊
2
n(n - 1)
75 1 + n(a x) + (a x) 2 + ◊◊◊ = 1 + 8x + 24x 2 + ◊◊◊
fi MC 2 = 2
4 Comparing the co-efficients, we get
75
fi (h + 2) 2 + (k - 3) 2 = fi
na (na - a )
4 na = 8, = 24
2
Hence, the locus of M(h, k) is 8(8 - a )
= 24
75 2
( x + 2) 2 + ( y - 3) 2 =
4 fi (8 – a) = 6
9. fi a = 2, n = 4
Y Thus, the point P is (2, 4)
Therefore, PA ◊ PB
= (PT)2 = 4 + 16 – 4 = 16
12. Given circles are x2 + y2 = 4
C(0, 3)
and x2 + y2 + 2x + 3y – 5 = 0
O A X Hence, the common chord is
B
2x + 3y = 1 …(i)
Thus, the equation of the circle is
Let the equation of the circle be S + lL = 0
x2 + y2 + 2gx + 2fy + c = 0 …(i) fi (x2 + y2 – 4) + l(2x + 3l – 1) = 0
which touches the y-axis at C. fi x2 + y2 + 2lx + 3ly – (l + 4) = 0
Put x = 0 in Eq. (i), we get
So centre of the circle is
y2 + 2fy + c = (y – 3)2
Ê 3l ˆ
fi y2 + 2fy + c = y2 – 6y + 9 ÁË — l , — ˜¯
fi 2fy + c = –6y + 9 2
Comparing the co-efficients, we get Since the chord 2x + 3y = 1 is a diameter of the circle, so
2f = –6, c = 9 9l
fi f = –3, c = 9 - 2l - =1
2
Intercepts on x-axis is
fi –13l = 2
fi 2 g2 - c = 8 2
fi l=-
2 13
fi g -c =4
Hence, the equation of the circle is
fi g2 - 9 = 4 Ê 2ˆ Ê 2ˆ Ê 2ˆ
x2 + y 2 + 2 Á - ˜ x + 3Á - ˜ y - Á 4 - ˜ = 0
fi g2 – 9 = 16 Ë 13 ¯ Ë 13 ¯ Ë 13 ¯
fi g2 = 25 fi 13(x2 + y2) – 4x – 6y – 50 = 0
fi g = ±5
Ê 2 3ˆ
Hence, the equation of the circle is Therefore, the centre of the circle is Á , ˜ .
Ë 13 13 ¯
x2 + y2 ± 10x – 6y + 9 = 0

CG_03a.indd 50 2/6/2017 4:01:03 PM


Circle 3.51

13. Let (h, k) be a point lies on the circle and (x – 4)2 + (y – 4)2 = (32 – l)
x2 + y2 + 2gx + 2fy + c = 0 It is given that |r1 – r2| < C1C2 < r2 + r2
Thus, the length of the tangent from (h, k) | 2 - 32 - l | < 3 2 < ( 2 + 32 - l )
2 2
= h + k + 2gh + 2fk + d Now, 3 2 < ( 2 + 32 - l )
2 2
= (h + k + 2gh + 2fk ) + d fi 32 - l > 2 2
= -c + d fi 32 – l > 8
fi l < 24 …(i)
= d -c and | 2 - 32 - l | < 3 2
14. Equation of the circle is fi - 4 2 < - 32 - l < 2 2
S + lL = 0 fi —2 2 < 32 - l < 4 2
fi (x2 + y2 – 4) + l(x + y – 1) = 0 fi 8 < (32 – l) < 32
fi x2 + y2 + lx + ly – (l + 4) = 0 fi –24 < –l < 0
Ê l lˆ fi 0 < l < 24 …(ii)
So centre of the circle is Á - , - ˜ From Eqs (i) and (ii), we get
Ë 2 2¯
0 < l < 24
Since the chord x + y = 1 is a diameter of the circle, so 18. Since two vertices of an equilateral triangle are
l l B(–1, 0) and C(1, 0).
- - =1
2 2 So, the third vertex must lie on the y-axis.
fi –l = 1 Let the third vertex be A(0, b).
fi l = –1 Now, AB = BC = CA
Hence, the equation of the circle is fi AB2 = BC2 = AC2
x2 + y2 – x – y – 3 = 0 fi 1 + b2 = 4 = 1 + b2
15. Now the image of the centre (3, 2) to the line x + y = 19 fi b2 = 4 – 1
is obtained by fi b= 3
a -3 b -2 Ê 3 + 2 - 19 ˆ Thus, the third vertex is A = (0, 3)
= = - 2Á 2 2 ˜
1 1 Ë 1 +1 ¯ As we know that in case of an equilateral triangle,
a -3 b -2 Ê 1 ˆ
= = 14 Circumcentre = Centroid = Á 0, ˜
1 1 Ë 3¯
a = 17, b = 16 Hence, the equation of the circumcircle is
2 2
Hence, the equation of the new circle is Ê 1 ˆ 2 Ê 1 ˆ
fi ( x — 0) 2 + Á y - ˜¯ = (1 - 0) + ÁË 0 - ˜
(x – 17)2 + (y – 16)2 = 1 Ë 3 3¯
16. Given circles are 2
Ê 1 ˆ 4
(x – 1)2 + (y – 3)2 = r2 …(i) fi x2 + Á y - ˜ =
Ë 3¯ 3
and x2 + y2 – 8x + 2y + 8 = 0
19. The vertices of the triangle are
fi (x – 4)2 + (y + 1)2 = 16 + 1 – 8
fi (x – 4)2 + (y + 1)2 = 9 …(ii) A = (0, 6), B = (2 3, 0), C = (0, 2 3)
Two circles (i) and (ii) intersect, if Let P(h, k) be the circumcentre, then
|r1 – r2| < C1C2 < r1 + r2 PA = PB = PC
fi PA2 = PB2 = PC2
fi |r — 3| < (1 - 4) 2 + (3 + 1) 2 < r + 3
fi h 2 + (k - 6) 2 = (h - 2 3) 2 + k 2
fi |r – 3| < 5 < r + 3
fi |r – 3| < 5 and r + 3 > 5 = h 2 + (k - 2 3) 2
fi –5 < (r – 3) < 5, r > 2 On solving, we get
fi –2 < r < 5 + 3, r > 2 h = 0 and k = 2
fi r < 8, r > 2 Thus, radius, r = h 2 + (k - 6) 2 = 4
fi 2<r<8 Hence, the equation of the circle is
17. Since there are two real tangents drawn, so the circles x2 + (y – 2)2 = 16
intersect. fi x2 + y2 – 4y – 12 = 0
Given circles are (x – 1)2 + (y – 1)2 = 2

CG_03a.indd 51 2/6/2017 4:01:03 PM


3.52 Coordinate Geometry Booster

20. Given circle is 24. Let ABC be an equilateral triangle with AB = a.


x2 + y2 + 4x – 6y + 4 = 0 P
(x + 2)2 + (y – 3)2 = 32
So, C = (–2, 3) and r = 3 C(–2, 3)
As we know that the centroid D C
divides the median in the ratio O
2:1
Here, circumradius = 2 and in- A B
radius = 6 Q M R
Hence, the equation of the circumcircle is Let PM = p
(x + 2)2 + (y – 3)2 = 62 PM p
fi x2 + y2 + 4x – 6y – 23 = 0 Now, sin (60°) = =
PQ a
21. Given circle is p 3
fi =
x2 + y2 + 2gx + 2fy + c = 0 D C a 2
We have, 3
a2 + a2 = (r + r)2 fi p= a
O a 2
fi a2 = 2r2 r Here O is the centroid.
fi a2 = 2r2 So the centroid divides the median in the ratio 2 : 1
A a B p
fi a=r 2 Thus, OM =
3
Hence, the result. p
fi r=
22. Given circle is 3
x2 + y2 + 2gx + 2fy + c = 0 2p 2 3 a
fi 2r = = ¥ a=
3 3 2 3
Here, OA = r, –AOM = 60°
Let x be the side of a square.
Now, C
Thus,
AM AM 2
sin (60°) = = Ê a ˆ a2
OA r x2 + x2 = Á ˜ =
Ë 3¯ 3
O
AM 3 a2
fi = fi 2 x2 =
r 2 3
A M B
3 a2
fi AM = r fi x2 =
2 6
a2
fi AB = 2AM = r 3 Area of the square =
6
Hence, the area of an equilateral triangle is 25. Given circle is
x2 + y2 – 12x + 4y + 30 = 0
3
= ¥ ( AB) 2 fi (x – 6)2 + (y + 2)2 = 36 + 4 – 30
4
fi ( x - 6) 2 + ( y + 2) 2 = ( 10) 2
3
= ¥ 3r 2 Any point on the circle is
4
P(6 + 10 cos q , - 2 + 10 sin q )
3 3 2
= r Let d be the distance from the origin
4
Thus, d = OP
23. Given centre of the circle is (0, 0). d 2 = (6 + 10 cos q ) 2 + ( - 2 + 10 sin q ) 2
As we know that the centroid divides the median in the
ratio 2 : 1 = 50 + 4 10(3 cos q - sin q )
\ Circumradius = 2a Ê 3 1 ˆ
= 40 + 4.10 Á cos q - sin q ˜
Hence, the equation of the circle is Ë 10 10 ¯
x2 + y2 = (2a)2 = 40 + 40(cos q cos a – sin q sin a)
fi x2 + y2 = 4a2 = 40 + 40(cos(q + a))
= 40 + 40, when cos(q + a) = 1 = cos 0°
= 80 and q = –a

CG_03a.indd 52 2/6/2017 4:01:04 PM


Circle 3.53

Hence, the point is fi mx – y – m = 0 …(ii)


= (6 + 10 cos q , - 2 + 10 sin q ) If (ii) is a tangent of (i), then
Ê 3 Ê 1 ˆˆ m-2-m
= Á 6 + 10 ◊ , - 2 + 10 ◊ Á - fi = 5
Ë 10 Ë 10 ˜¯ ˜¯ m2 + 1
= (6 + 3, –2 – 1) -2
= (9, –3) fi = 5
m2 + 1
26.
27. The equation of any tangent through origin is fi 5(m2 + 1) = 2
y = mx 2
If y = mx be a tangent to the given circle, then fi (m 2 + 1) =
5
2 2 3
7m + 1 fi m = -1= -
=5 5 5
m2 + 1 fi m=j
So, no real tangents can be drawn.
fi 25(m2 + 1) = (7m + 1)2 Thus, number of tangents = 0.
fi 25m2 + 25 = 46m2 + 14m + 1 30. The equation of the tangent to the circle x2 + y2 = a2
fi 24m2 – 14m – 24 = 0 È Êp ˆ Êp ˆ˘
at (a cos q , a sin q ) and Ía cos Á + q ˜ , a sin Á + q ˜ ˙
Let its roots are m1, m2. Î Ë 3 ¯ Ë 3 ¯ ˚
So, product of the roots = –1 are x cos q + y sin q = a
fi m1 ◊ m2 = –1 Êp ˆ Êp ˆ
and x cos ÁË + q ˜¯ + y sin ÁË + q ˜¯ = a
p 3 3
fi q= Let (h, k) be the point of intersection.
2
On solving, we get
28. Given circle is
Ê Êp ˆ ˆ
x2 + y2 – 2x – 4y = 0 2a Á sin Á + q ˜ - sin q ˜
Ë Ë 3 ¯ ¯
fi (x – 1)2 + (y – 2)2 = 5 …(i) h=
Equation of any tangent passing through (4, 3) is 3
y – 3 = m(x – 4) Ê Êp ˆ ˆ
2a Á cos Á + q ˜ - cos q ˜
fi mx – y + (3 – 4m) = 0 …(ii) Ë Ë 3 ¯ ¯
and k=
If (ii) is a tangent of (i), then 3
Now, squaring and adding, we get
m - 2 + 3 - 4m
fi = 5 3h 2 3k 2
m2 + 1 + =1
4a 2 4a 2
1 — 3m
= 5 4a 2
fi h2 + k 2 =
m2 + 1 3
Hence, the locus of (h, k) is
fi 5(m2 + 1) = (1 – 3m)2
fi 5m2 + 5 = 1 – 6m + 9m2 4a 2
x2 + y 2 =
fi 4m2 – 6m – 4 = 0 3
fi 2m2 – 3m – 2 = 0 31. The equation of tangent to the circle x2 + y2 = 4 at
Let its roots are m1, m2. (1, 3) is x + 3 y = 4
So, product of the roots = –1 Y
fi m1 ◊ m2 = –1
B
p
fi q= P(1, 3)
2
29. Given circle is
x2 + y2 – 2x – 4y = 0
X
fi (x – 1)2 + (y – 2)2 = 5 …(i) O M A
The equation of any tangent passing through (0, 1) is
y = m(x – 1)

CG_03a.indd 53 2/6/2017 4:01:05 PM


3.54 Coordinate Geometry Booster

(ii) Area of the sector OAC


Clearly, A = (4, 0), B = (0, 4/ 3)
Thus, the area of the triangle OPA is 1 2
= r q
1 2
= ¥ 4 ¥ 3 = 2 3 sq. u. 2
2 1 Ê 13 ˆ -1 Ê 120 ˆ
= ÁË ˜¯ sin ÁË ˜
2 2 169 ¯
LEVEL IV =
169 Ê 120 ˆ
sin -1 Á sq. m.
8 Ë 169 ˜¯
1. (i) Here, AB = 13 m and AC = 5 m
Y 1 2
Area of the triangle OAC = r sin q
2
2
C 1 Ê 13 ˆ Ê 120 ˆ
= Á ˜ Á ˜
2 Ë 2 ¯ Ë 169 ¯
q
X¢ X = 15 sq. m.
B O A
Thus, the area of the smaller portion bounded by the
circle and the chord AC.
C¢ = Area of sector OAC – Area of DAOC
Ï169 -1 Ê 120 ˆ ¸
Y¢ =Ì sin Á ˜ - 15˝ sq. m.
Ó 8 Ë 169 ¯ ˛
Ê 13 13 ˆ
Let the co-ordinates of C be Á cos q , sin q ˜ 2. Here, the co-ordinates of the centres A, B and C are
Ë2 2 ¯
It is given that AC = 5 (0, 0), ( 55, 3) and ( 65, 4).
fi AC2 = 25 Y
2 2
Ê 13 13 ˆ Ê 13 ˆ
fi Á cos q - ˜ + Á sin q ˜ = 25
Ë2 2¯ Ë2 ¯
238 119 B
fi cos q = = D
338 159 X¢
A
X
E
2
Ê 119 ˆ 120 C
fi sin q = 1 - Á ˜ =
Ë 159 ¯ 169

-1 Ê 120 ˆ 3. The equation of the radical axis is
fi q = sin Á
Ë 169 ˜¯ (x2 + y2 + 4x + 2y + 1)
The co-ordinates of C and C¢ will become Ê 3ˆ
- Á x 2 + y 2 — x + 3y — ˜ = 0
Ê 119 60 ˆ Ê 119 60 ˆ Ë 2¯
ÁË , ˜ and Á ,- ˜
26 13 ¯ Ë 26 13 ¯ 5
fi 5x - y + = 0
Ê 13 ˆ 2
Thus, the co-ordinates of B are Á - , 0˜
Ë 2 ¯ 10x – 2y + 5 = 0 …(i)
The equation of the co-axial circle is
Hence, the equations of the pair of lines BC and BC¢
Ê 2 2 3ˆ
are ÁË x + y - x + 3y - ˜¯ +
2
± (60/13) Ê 13 ˆ
y —0= ÁË x + ˜¯ l(x2 + y2 + 4x + 2y + 10) = 0
Ê 119 13 ˆ 2
ÁË + ˜
26 2¯ Ê 4l - 1ˆ Ê 2l + 3 ˆ Ê l - 3/2 ˆ
x2 + y 2 + Á ˜ x+Á ˜ y+Á =0
15 Ê 13 ˆ Ë l +1 ¯ Ë l +1 ¯ Ë l + 1 ˜¯
fi y=± Á x + ˜¯ …(ii)
36 Ë 2
È 1 Ê 4l - 1ˆ 1 Ê 2l + 3 ˆ ˘
5Ê 13 ˆ Thus, Í Á ˜, Á ˜˙
fi y = ± Áx + ˜
Ë Î2 Ë l + 1 ¯ 2 Ë l + 1 ¯˚
12 2¯
Clearly the centre lies on the radical axis.
fi 24y = ±5(2x + 13)

CG_03a.indd 54 2/6/2017 4:01:06 PM


Circle 3.55

Ê 1 Ê 4l - 1ˆ ˆ Ê 1 Ê 2l + 3 ˆ ˆ 1
10 Á Á + 2Á Á y - 0 = 7( x - 2) and y - 0 = ( x - 2)
So ˜ ˜ ˜ +5=0
Ë 2 Ë l +1 ¯¯ Ë 2 Ë l + 1 ¯ ˜¯ 7
fi l = 1. fi 7x + y = 14 and x – 7y = 2
Put the value of l = 1 in Eq. (ii), we get Now, the centres of the circles lie on the lines AB and
4x2 + 4y2 + 6x + 10y – 1 = 0. AC at a distance of 5 2 units.
4. A circle passing through the point of intersection of x-2 y-0
x2 + y2 + ax + by and Ax + By + C = 0 is Thus C1: = =5 2
1 7
x2 + y2 + ax + by + c + l(Ax + By + C) = 0 …(i) 5 2 5 2
and a circle passing through the point of intersection
x2 + y2 + a¢x + b¢y + c¢ = 0 and (x, y) = (1, 7)
A¢x + B¢y + C¢ = 0 is x-2 y-0
and C2: = =5 2
x2 + y2 + a¢x + b¢y + c¢ + l¢ (a¢x + B¢y + C¢) = 0 7 1
…(ii) 5 2 5 2
Since the point of intersection lie on the circle, so,
(x, y) = (9, 1)
a + lA = a¢ + l¢A¢
b + lB = b¢ + l¢B¢ Hence, the equations of the circles are
and c + lC = c¢ + l¢C¢ (x – 1)2 + (y – 7)2 = 9
Eliminating l and l¢, we get or (x – 7)2 + (y – 1)2 = 9
6. Let the centre of the circle be (h, k) such that k = h – 1.
A A¢ a — a ¢
We have, (h – 7)2 + (k – 3)2 = 32
fi B B ¢ b — b¢ = 0
fi (h – 7)2 + (h – 4)2 = 32
C C ¢ c — c¢ fi h2 = 11h + 28 = 0
B¢ b — b¢ A¢ a — a ¢ A¢ a — a ¢ fi h = 7, 4 and k = 6, 3
fi A -B +C =0
C ¢ c — c¢ C ¢ c — c¢ B¢ b — b¢ Hence, the equation of a circle can be
(x – 7)2 + (y – 6)2 = 32
fi AB ¢ (c - c ¢ ) - AC ¢ (b - b ¢ ) - BA¢ (c - c ¢ ) and (x – 4)2 + (y – 3)2 = 32
+ BC ¢ (a - a ¢ ) + CA¢ (b - b¢ ) - CB ¢ (a - a ¢ ) = 0 fi x2 + y2 – 8x – 6y + 16 = 0
fi (a - a ¢ )( BC ¢ - CB ¢ ) + (b - b ¢ )(CA¢ - AC ¢ ) and x2 + y2 – 8x – 6y + 16 = 0
+ (c - c ¢ )( AB ¢ - BA¢ ) = 0 7. Here, CM = CN
Y
5. The equation of the tangent to the given circle
(x + 2)2 + (y + 3)2 = 25 at A(2, 0) is
4x – 3y – 8 = 0 …(i) P(2, 8)
N
P
B C(h, k)
X¢ X
O

A(2, 0) M

T Y¢
C
Let m and m¢ be the slopes of the lines AB and AC. 4h - 3k - 24 4h + 3k - 42
fi =
m - (4/3) 16 + 9 16 + 9
Thus, tan (45°) =
1 + m ◊ (4/3) fi (4h – 3k – 24) = ±(4h + 3k – 42)
m¢ - (4/3) fi k=3
and tan (135°) =
1 + m¢ ◊ (4/3) Also, r = CM
On solving, we get fi r2 = CM 2
1 Ê 4h + 3k - 42 ˆ
2
m = - 7, m¢ = fi (h — 2) 2 + (k - 8) 2 = Á ˜¯
7 Ë 5
Therefore, the equations of the lines AB and AC are

CG_03a.indd 55 2/6/2017 4:01:06 PM


3.56 Coordinate Geometry Booster

2 The equation of the normal is


Ê 4h + 9 - 42 ˆ
fi (h — 2) 2 + (3 - 8) 2 = Á ˜¯ 1
Ë 5 ( y - 4) = - ( x - 2)
2 4
Ê 4h — 33 ˆ
fi (h — 2) 2 + 25 = Á fi 4(y – 4) = –(x – 2)
Ë 5 ˜¯
fi x + 4y = 18
On solving we get, Let (h, k) be the centre of the circle.
h=2 Thus, h + 4 k = 18 …(i)
Thus, the equation of the circle is Also, CP = CQ
fi h2 + (k – 1)2 = (h – 2)2 + (k – 4)2
(x – 2)2 + (y – 3)2 = 25
fi –2k + 1 = –4h + 4 – 8k + 16
fi x2 + y2 – 2x – 6y – 12 = 0 fi 4h + 6k = 19 …(ii)
8. Let x1, x2 are the roots of x2 + 2x – a2 = 0. On solving Eqs (i) and (ii), we get
Then x1 + x2 = –2, x1x2 = –a2 16 53
Similarly, y1 + y2 = –4, y1y2 = –b2 h=- ,k =
5 10
Hence, the equation of the circle is Ê 16 53 ˆ
Hence, the required centre is Á , ˜ .
(x – x1)(x – x2) + (y – y1)(y – y2) = 0 Ë 5 10 ¯
x2 – (x1 + x2)x + x1x2 + y2 – (y1 + y2)y + y1y2 = 0 11. Let the mid point be M(h, k).
x2 + 2x – a2 + y2 + 4y – b2 = 0 Equation of the chord bisected at M is
x2 + y2 + 2x + 4y – (a2 + b2) = 0 T = S1
Therefore, the centre of the circle = (–1, –2) and the hx + ky – (x + h) – 3(y + k) = h2 + k2 – 2h – 6k
2 2 which is passing through the origin. So,
radius = a + b + 5 . –h – 3k = h2 + k2 – 2h – 6k
9. Equation of the tangent to the circle x2 + y2 = 5 at (1, fi h2 + k2 – h – 3k = 0
–2) is x – 2y = 5. Hence, the locus of (h, k) is
Given circle is x2 + y2 – x – 3y = 0
x2 + y2 – 8x + 6y + 20 = 0 12. The equation of a circle passing through the points of
Centre = (4, –3) and radius = 5 . intersection of x2 + y2 = 1
Let the point of contact be (h, k). x2 + y2 – 2x – 4y + 1 = 0 is
Thus, h – 2 k = 5 …(i) (x2 + y2 – 2x – 4y + 1) + l(x2 + y2 – 1) = 0
and (h – 4)2 + (k + 3)2 = 5 fi (1 + l)x2 + (1 + l)y2 – 2x – 4y + (1 – l) = 0
fi h2 + k2 – 8h + 6k = 20 = 0 …(ii)
2 4 (1 - l )
On solving Eqs (i) and (ii), we get fi x2 + y 2 - x- y+ = 0 …(i)
h = 3, k = –1 (1 + l ) (1 + l ) (1 + l )
Hence, the point of contact is (3, –1). Ê 1 2 ˆ
Centre = Á ,
10. Given curve is y = x2 Ë (1 + l ) (1 + l ) ˜¯
Y 2 2 2
Ê 1 ˆ Ê 2 ˆ Ê (1 - l ) ˆ
Radius = Á ˜ +Á ˜ +Á
Ë (1 + l ) ˜¯
N T Ë (1 + l ) ¯ Ë (1 + l ) ¯

C(h, k) Equation (i) touching the straight line x + 2y = 0. So,


the length of perpendicular from the centre is equal to
Q(0, 1) P(2, 4) the radius of a circle.
2 2 2
X¢ X 5 Ê 1 ˆ Ê 2 ˆ Ê (1 — l ) ˆ
O Thus, = Á ˜ +Á ˜ +Á
(1 + l ) Ë (1 + l ) ¯ Ë (1 + l ) ¯ Ë (1 + l ) ˜¯
fi (1 – l)2 + 5 = 5
fi l=1
Hence, the equation of the circle is
dy x2 + y2 – x – 2y = 0
fi = 2x
dx 13. As we know that the normal always passes through the
Ê dy ˆ centre of the circle.
Thus, m = Á ˜ =4 So, the equation of the circle is
Ë dx ¯ /(2,4)
(x – 3)2 + y2 = 6

CG_03a.indd 56 2/6/2017 4:01:07 PM


Circle 3.57

14 Let the equations of the two given circles are Also, (h – 1)2 + (k – 2)2 = (h – 5)2 + (k – 2)
x2 + y2 + 2gx + g2 = 0 fi (h – 1)2 = (h – 5)2
and x2 + y2 + 2fy + f 2 = 0 fi h2 – 2h + 1 = h2 – 10h + 25
whose centres lie on x and y axes. fi 8h = 24
Thus, the equation of the radical axis is fi h = 3.
2gx + g2 – 2fy – f 2 = 0 Thus, the centre is (3, 0)
fi 2gx – 2fy + g2 – f 2 = 0 Hence, the radius of the circle is 2 2.
15. Equations of the normals of the circle are 18. Let the co-ordinates of P be (0, k) and the centre of the
x + 2xy + 3x + 6y = 0 circle be C(h, k).
fi (x + 2y)(x + 3) = 0
fi (x + 2y) = 0, (x + 3) = 0 Y
Ê 3ˆ
Thus, the centre of the circle is C1: Á - 3, ˜ . A
Ë 2¯
Given circle is
x(x – 4) + y(y – 3) = 0 P
fi x2 + y2 – 4x – 3y = 0 B
Ê 3ˆ 5 X¢ X
Centre is C2: Á 2, ˜ and the radius is . O
Ë 2¯ 2
According to the questions, we get Y¢
C1C2 = r1 – r2
Now, CA = CB = CP
2 5 fi CA2 = CB2 = CP2
fi ( - 3 - 2) + 0 = r —
2 fi (h – 4)2 + (k – 3)2 = (h – 2)2 + (k – 5)2 = h2
5 15
fi r =5+ = Now,
2 2 (h – 4)2 + (k – 3)2 = (h – 2)2 + (k – 5)2
Hence, the equation of the circle is fi –8h – 6k = –4h + 4 – 10k
2 2
Ê 3ˆ Ê 15 ˆ fi 4h – 4k + 4 = 0
( x + 3) 2 + Á y - ˜ = Á ˜
Ë 2¯ Ë 2¯ fi h–k+1=0 …(i)
fi x2 + y2 + 6x – 3y – 45 = 0 and (h – 2)2 + (k – 1)2 = h2
16. Here A = (2, 3), B = (6, 3) and D = (2, 6) fi (h – 2)2 + (h – 4)2 = h2
Thus, the centre of the circle is (4, 9/2) and the radi- fi h2 – 4h + 4 + h2 – 8h + 16 = h2
us = 5/2. fi –4h + 4 + h2 – 8h + 16 = 0
Y fi h2 – 12h + 20 = 0
fi (h – 2) = 0, (h – 10) = 0
D(2, 6)
fi (h – 2) = 0, (h – 10) = 0
fi h = 2, 10
C(4, 9/2) Thus, k = h + 1 = 3, 11
Hence, the point on y-axis is P (0, 3).
y=3 Thus, the equation of the circle is
A(2, 3) B(6, 3) (x – 2)2 + (y – 3)2 = 9
X fi x2 – 4x + 4 + y2 – 6y + 9 = 9
O
x=2 fi x2 + y2 – 4x – 6y + 4 = 0
19. Let the centre of the circle be C(h, k).
Y
Hence, the equation of the circle is
2
Ê 9ˆ 25
( x - 4) 2 + Á y - ˜¯ =
P(3, 3)
Ë 2 4 y=x
fi x2 + y2 – 8x – 9x + 30 = 0 O
X¢ X
17. Let the centre be (h, k) B 3x – y = 6
Thus, (h – 1)2 + (k – 2)2 = (h – 5)2 + (k – 2)2
A(1, –3)
= (h – 5)2 + (k + 2)2
Now, (h – 5)2 + (k – 2)2 = (h – 5)2 + (k + 2)2
fi (k – 2)2 = (k + 2)2 Y¢
fi k2 – 4k + 4 = k2 + 4k + 4
Here, PA = 2 10
fi 8k = 0
fi k=0 Let B be (a, a).

CG_03a.indd 57 2/6/2017 4:01:07 PM


3.58 Coordinate Geometry Booster

Thus, PB = 2 10 81 25
fi x 2 - 9x + + y 2 - 2ky =
fi (a – 3)2 + (a – 3)2 = 40 4 4
fi (a – 3)2 = 20
fi x2 + y2 – 9x – 2ky + 14 = 0
fi (a - 3) = ± 2 5
23. Let P be (h, k)
fi a =3± 2 5
fi a=3-2 5
Hence, B = (3 - 2 5, 3 - 2 5) .
C B
20. The length of the line = the length of the tangent B(h, k)
= 4 + 9 + 4 + 30 + 1
= 48 = 4 3 Q(6, –1)
A
21. Let the centre be C(h, h)
Y y =5
x+

Here, PA = PB = PC
M
h fi PA2 = PB2 = PC2
4x + 3y = 6
h C(h, h) fi h2 + k2 + h – 3 = 3(h2 + k2) – 5h + 3k

O X = 4(h2 + k2) + 8h + 7k + 9
On solving, we get
Y¢ h = 0, k = –3.
Now, CM = h So, the point P is (0, –3).
4h + 3h Let the centre of the new circle be (a, b).
fi =6
5
We have,
fi 7h = 30
30 a+b-5
fi h= = (a - 6) 2 + (b + 1) 2
7 2
Hence, the equation of the circle is = a 2 + (b + 3) 2
2 2 2
Ê 30 ˆ Ê 30 ˆ Ê 30 ˆ On solving, we get
fi ÁË x - ˜¯ + ÁË y - ˜¯ = ÁË ˜¯
7 7 7 7 7
a = ,b= -
60 900 60 2 2
fi x2 - x+ + y2 - y=0
7 49 7
5 2
60 60 900 and radius is r =
fi x2 + y 2 - x- y+ =0 2
7 7 49
Hence, the equation of the circle is
fi 49(x2 + y2) – 420(x + y) + 900 = 0
2 2 2
Ê 7ˆ Ê 7ˆ Ê5 2ˆ
22. Hence, the equation of the circle is ÁË x - ˜¯ + ÁË y + ˜¯ = ÁË ˜
2 2 2 2 2 ¯
Ê 9ˆ 2 Ê9 ˆ 2
ÁË x - ˜¯ + ( y - k ) = Á - 2˜ + k fi x2 + y2 – 7x + 7y + 12 = 0
2 Ë2 ¯
Y 24. Clearly, the locus of the mutually perpendicular tan-
gents to the circle is the director circle. So its equation
is x2 + y2 = 18.
Thus, its centre is (0, 0) and the radius = 3 2 .
C(9/2, k)
Let the equation of the co-axial system be
x2 + y2 + 2gx + c = 0
O A(2, 0) B(7, 0)
X whose centre is (–g, 0) and the radius = g2 - c .

CG_03a.indd 58 2/6/2017 4:01:08 PM


Circle 3.59

fi 4(y – 4) = –(x – 2)
fi x + 4y = 18
P( 2, 4) Let (h, k) be the centre of the circle.
C(0, 0) C¢(–g, 0) Thus, h + 4 k = 18 …(i)
Also, CP = CQ
fi h2 + (k – 1)2 = (h – 2)2 + (k – 4)2
fi –2k + 1 = –4h + 4 – 8k + 16
—g + 0 fi 4h + 6k = 19 …(ii)
Clearly, = 2
2 On solving (i) and (ii), we get
fi g = -2 2 16 53
h=- ,k =
Also, CC¢ = r1 + r2 5 10
fi g = 3 2 + g2 - c Ê 16 53 ˆ
Hence, the required centre is Á - , ˜ .
Ë 5 10 ¯
fi —2 2 = 3 2 + 8 - c
27. Given equation of normals are
fi —5 2 = 8 - c x2 – 3xy – 3x + 9y = 0
fi 8 – c = 50 fi (x – 3y)(x – 3) = 0
fi c = –42
fi (x – 3y) = 0, (x – 3) = 0
Hence, the equation of the co-axial circle is
As we know that the normal always passes through the
x 2 + y 2 - 4 2 x - 42 = 0 .
centre of a circle.
25. Equation of the tangent to the circle x2 + y2 = 5 at
(1, –2) is x – 2y = 5. Thus, centre, C1 = (3, 1)
Given circle is Also, given circle is
x2 + y2 – 8x + 6y + 20 = 0 x2 + y2 – 6x + 6y + 17 = 0
fi (x – 3)2 + (y + 3)2 = 1
Centre = (4, –3) and the radius = 5.
So the centre C2 = (3, –3) and r2 = 1
Let the point of contact be (h, k).
Since the circle touches externally, so
Thus, h – 2 k = 5 …(i)
and (h – 4)2 + (k + 3)2 = 5 C1C2 = r1 + r2
fi h2 + k2 – 8h + 6k + 20 = 0 …(ii) fi 4 = r1 + 1
On solving Eqs (i) and (ii), we get fi r1 = 3
h = 3, k = – 1 Hence, the equation of the circle is
Hence, the point of contact is (3, –1). (x – 3)2 + (y – 1)2 = 9
26. Given curve is fi x2 + y2 – 6x – 2y + 1 = 0
Y 28. Given lines are 2x – 4y = 9
N 7
T and 2x - 4y + = 0
3
7
C(h, k) 9+
Q(0, 1)
1 3
P(2, 4) Hence, the radius =
2 4 + 16
X¢ X
O
1 Ê 34 ˆ
=
2 ÁË 3 20 ˜¯
1 Ê 34 ˆ 17
y = x2 = Á ˜ =
Ë
2 6 5 ¯ 6 5
dy
fi = 2x 29. Let the co-ordinates of the other end be (a, b).
dx
Given that the centre of the given circle = (4, 2).
Ê dy ˆ
Thus, m = Á ˜ =4 a-3 b+2
Ë dx ¯ /(2,4) Thus, = 4, =2
2 2
The equation of the normal is
a = 11, b = 2
1
( y - 4) = - ( x - 2) Hence, the other end be (11, 2).
4

CG_03a.indd 59 2/6/2017 4:01:09 PM


3.60 Coordinate Geometry Booster

30. Hence, the equation of the line is


P
Ê 2 - 1ˆ
y=Á ˜ x = (3 — 2 2) x
Ë 2 + 1¯
D C
32. Given circle is x2 + y2 – 6x – 4y + 4 = 0
O Y

A B
Q M R P(h, k)
B
Let PM = p
PM p
Now, sin (60°) = = C
PQ a A
p 3 X
O
fi =
a 2
3 So, the centre is (3, 2) and the radius is 3.
fi p= a
2 Let P be (h, k)
Here O is the centroid. Thus, 4h – 3k = 6 …(i)
So the centroid divides the median in the ratio 2 : 1.
Let –CPB = q
p
Thus, OM = BC
3 Then sin q =
p PC
fi r= 3
3 = …(ii)
2p 2 3 a (h - 3) + (k - 2) 2
2

fi 2r = = ¥ a=
3 3 2 3 From Eqs (i) and (ii), we get
Let x be the side of a square. 3
sin q = …(iii)
2 2 2
Ê a ˆ a Ê 4h — 6 ˆ
Thus, x 2 + x 2 = Á ˜ = (h - 3) 2 + Á - 2˜
Ë 3¯ 3 Ë 3 ¯
2
a
fi 2 x2 = It is given that,
3 Ê 24 ˆ
2q = tan -1 Á ˜
2 a2 Ë 7¯
fi x =
6 24
a2 fi tan (2q ) =
Area of a square = 7
6
7
31. Given circle is x2 + y2 = 3 fi cos (2q ) =
25
and the line is x + y = 2
7
Now, fi 1 — 2 sin 2 (q ) =
x2 + (2 – x)2 = 3 25
2 7 18
fi x2 + x2 – 4x + 4 = 3 fi 2 sin (q ) = 1 — =
25 25
fi 2x2 – 4x + 1 = 0 9
fi sin 2 (q ) =
4± 8 1 25
fi x= =1±
4 2 3
fi sin (q ) =
1 5
fi x =1+
2 From Eq. (iii), we get
1 1 3 3
Also, y = 2 — x = 2 - 1 - =1- =
2 2 5 2
Ê 4h — 6 ˆ
Thus, the point of intersection is (h - 3) 2 + Á - 2˜
Ë 3 ¯
Ê 1 1 ˆ
ÁË1 + ,1 - 2
˜ Ê 4h — 6 ˆ
2 2¯ fi (h - 3) 2 + Á - 2˜ = 25
Ë 3 ¯

CG_03a.indd 60 2/6/2017 4:01:10 PM


Circle 3.61

Ê 4h — 12 ˆ
2 Ê pˆ Ê 3p ˆ
fi (h - 3) 2 + Á = 25 fi cos Á q — ˜ = cos Á ˜
Ë 3 ˜¯ Ë 4¯ Ë 4¯
p
fi 9(h – 3)2 + (4h – 12)2 = 225 fi q = p or —
2
fi 25h2 – 150h = 0
Hence, the point B be (–2, 2) or (2, –2).
fi h = 0, 6
35. Given circle is x2 + y2 – 2rx – 2hy + h2 = 0
If h = 0, then k = –2.
fi (x – r)2 + (y – h)2 = r2.
If h = 6, then k = 6.
So, the centre is (r, h) and radius is r.
Hence, the points are (0, –2), (6, 6).
It is possible only when r = h.
33. Here, C1 = (5, 0), r1 = 3, C2 = (0, 0), r2 = r
It is given that two circles intersect.
So, |r1 – r2| < C1C2 < r1 + r2
fi |3 – r| < 5 < 3 + r C(r, h)
fi |3 – r| < 5, 5 < 3 + r r C(r, r)
fi –5 < (r – 3) < 5, r > 2
fi –2 < r < 8, r > 2
fi 2<r<8
34. Given circle is x2 + y2 = 8
Y 36. Let the equation of the circle C be
x2 + y2 + 2gx + 2fy + c = 0 …(i)
Given circle is x2 + y2 = 1
fi x2 + y2 – 1 = 0 …(ii)
A
Since two circles are orthogonal, so
2(g ◊ 0 + f ◊ 0) = c – 1
B
X fi c=1
X¢ O P(4, 0)
Also, the equation of radical axis is
B
2gx + 2fy + c + 1
Y¢ fi 2gx + 2fy + 1 + 1 = 0
Let the point A be (h, k). fi gx + fy + 1 = 0
The equation of the tangent at A is hx + ky = 8 which is It is given that the radical axis parallel to y-axis, so,
passing through P(4, 0). f = 0, g ΠR Р{0}
So, 4h = 8 fi h = 2 1
Thus, x = -
Now PA = length of the tangent = 2 2 g
fi PA2 = 8 Hence, the equation of the circle is
fi (h – 4)2 + k2 = 8 fi x2 + y2 + 2gx + 1 = 0
fi (2 – 4)2 + k2 = 8 fi x2 + y2 + x + 1 = 0
fi k2 = 8 – 4 or x2 + y2 – x + 1 = 0
fi k=2 37. Ans.
Hence, the point A is (2, 2). 38. Given circle is (x + r)2 + (y – h)2 = r2.
Now, for the point B, The equation of any tangent passing through origin is
x-2 y-2 y = m x. i.e. m x – y = 0
= =4 As we know that the length of the perpendicular from
cos q sin q
the centre of the circle to the tangent is equal to the
fi x = 2 + 4 cos q, y = 2 + 4 sin q
radius of a circle.
Thus, the point B is (2 + 4 cos q, 2 + 4 sin q).
Since the point B lies on the circle x2 + y2 = 8, so - rm - h
Thus, =r
(2 + 4 cos q)2 + (2 + 4 sin q)2 = 8 m2 + 1
fi 16 cos q + 16 sin q + 16 = 0
fi ( - rm - h) 2 = (r m 2 + 1) 2
fi cos q + sin q + 1 = 0
fi cos q + sin q = –1 fi (rm + h)2 = r2(m2 + 1)
1 1 1 fi r2m2 + 2rmh + h2 = r2m2 + r2
fi cos q + sin q = —
2 2 2 fi 2rmh + h2 = r2

CG_03a.indd 61 2/6/2017 4:01:10 PM


3.62 Coordinate Geometry Booster

and c = 27 – 2f = 27 + 11 = 38
r 2 - h2
fi m= Hence, the equation of the circle is
2rh x2 + y2 + 7x – 11y + 38 = 0.
Hence, the equation of the circle is Ê 3ˆ 5
40. Here, the centre = Á 2, ˜ and the radius =
Ê r 2 - h2 ˆ Ë 2¯ 2
y=Á x
Ë 2rh ˜¯ Therefore, the equation of the circle is
2
fi (r2 – h2)x – 2rhy = 0 Ê 3ˆ 25
( x — 2) 2 + Á y - ˜¯ =
fi (h2 – r2)x + 2rhy = 0 Ë 2 4
39. Let the equation of the circle be Y
x2 + y2 + 2gx + 2fy + c = 0 …(i)
Also (i) is orthogonal to
x2 + y2 + 4x – 6y + 9 = 0
Thus, 2(g1g2 + f1f2) = c1 + c2 M
P
fi 2(g ◊ 2 + f ◊ ( –3)) = c + 9
fi 4g – 6f = c + 9 …(ii) C
X
O

C(–g, –f)
The equation of the tangent parallel to the diagonal is
x+y=5 4y – 3x + k = 0
A 5
P(–2, 7) Now, CM =
2
Also, CP ^ PA Ê 3ˆ
7+ f 4 Á ˜ - 3.2 + k
Thus, ¥ -1 = -1 Ë 2¯ 5
g-2 fi =
5 2
7+ f 25
fi =1 fi k=±
g-2 2
fi 7+f=g–2 Hence, the equations of the tangents are
fi f=g–9 …(ii) 25
4y - 3x ± =0
From Eqs (i) and (ii), we get 2
4g – 6f = c + 9 fi 8y – 6x ± 25 = 0
fi 4(f + 9) – f = c + 9
Ê 1ˆ
fi 36 – 2f = c + 9 41. Clearly, the centre of the circle is Á -1, - ˜
Ë 4¯
fi c = 27 – 2f …(iii)
Again, CP = Radius
-g - f -5
= g2 + f 2 - c C
12 + 12
fi (g + f + 5)2 = 2(g2 + f 2 – c) + 11
Q 2x
fi (2f + 14)2 = 2((9 + f )2 + f 2 – (27 – 2f)) P y=
fi 2(f + 7)2 = (2f 2 + 18f + 81 – (27 – 2f ))
fi 2(f + 7)2 = (2f 2 + 20f + 54) Given line is y = 2x + 11
fi (f + 7)2 = (f 2 + 10f + 27) fi 2x – y + 11 = 0 …(i)
fi f2 + 14f + 49 = (f 2 + 10f + 27) The equation of CP is –x – 2y + k = 0
fi 4f = 27 – 49 fi x + 2y – k = 0
22 11 Ê 1ˆ
fi f =- =- which is passing through the centre C Á -1, - ˜ . so,
4 2 Ë 4¯
1
11 7 -1 - = k
Thus, g = f + 9 = 9 - = 2
2 2

CG_03a.indd 62 2/6/2017 4:01:11 PM


Circle 3.63

3 43. If R be the radius of the circumcircle


fi k=-
2 ph + qk - r
3 so, R= …(i)
Hence, the line CP is x + 2y + = 0 p2 + q2
2
fi 2x + 4y + 3 = 0 …(ii) Since TP and TQ are tangents to the circle x2 + y2 =
On solving Eqs (i) and (ii), we get a2, so the circumcircle through T will pass through its
9 centre.
x=- ,y=2
2 Thus, R = h 2 + k 2 ...(ii)
Ê 9 ˆ
Hence, the required point is Á - , 2˜ . From Eqs (i) and (ii), we get
Ë 2 ¯
2 2
42. Given circle is x + y + 4x = 0 ph + qk — r
= h2 + k 2
(x + 2)2 + y2 = 4 2
p +q 2

So, the centre is (–2, 0) and the radius is = 2.


fi ( ph + qk — r ) 2 = ( p 2 + q 2 )(h 2 + k 2 )
The equation of the line joining the centres of the cir-
cles is Hence, the locus of (h, k) is
y + 2 = m(x – 0) ( px + qy — r ) 2 = ( p 2 + q 2 )( x 2 + y 2 )
y + 2 = 3 x, m = tan (60°) = 3 44. Let S1: x2 + y2 – 2x – 6y + 6 = 0
S2: x2 + y2 + 2x – 6y + 6 = 0
Y
and S3 : x2 + y2 + 4x + 6y + 6 = 0
T3 Let the equation of the circle passing through the point
of intersection S1 and S2 is
C2 S4: S1 + lS2 = 0
R T2
P
fi ( x 2 + y 2 — 2x — 6y + 6) + l ( x 2 + y 2 + 2x — 6y + 6) = 0
Q
X¢ X
C1(2, 0) O Ê 2l - 2 ˆ Ê - 6l - 6 ˆ Ê 6l + 6 ˆ
T1 fi x2 + y 2 + Á x+Á yz + Á =0
Ë l + 1 ˜¯ Ë l + 1 ˜¯ Ë l + 1 ˜¯
Y¢ …(i)
Also, S4 is orthogonal to S3.
Here, let the centre C2 be (h, k).
Thus, 2(g1g2 + f1f3) = c1 + c2
h+2 k -0
Thus, = =4
1 3 Ê 2l - 2 ˆ Ê - 6l - 6 ˆ Ê 6l + 6 ˆ
fi 2.2 Á + 2.3 Á =6+Á
2 2 Ë l + 1 ˜¯ Ë l + 1 ˜¯ Ë l + 1 ˜¯
fi h = 0, k = 2 3 On solving, we get
Therefore, C2 = (h, k ) = (0, 2 3) l2 + 6l + 8 = 0
Hence, the locus of the centre of the outer circle is fi (l + 2)(l + 4) = 0
( x - 0) 2 + ( y - 2 3) 2 = 4 fi l = –2, –4
Put the values of l = –2, –4, we get
x2 + y 2 - 4 3 y - 8 = 0
x2 + y2 + 6x – 6y + 6 = 0
The equation of the common tangent T1 is
Ê 10 ˆ
— x - 3y + k = 0 or x 2 + y 2 + Á ˜ x - 6y + 6 = 0
Ë 3¯
which is passing through P (- 1, 3).
So, k = 2
Hence, the equation of the common tangent T1 is Integer Type Questions
x + 3y = 2 . 1. Clearly, C1C2 = 5
Clearly, the co-ordinates of Q and R are Here, r1 = 10, r2 = 5
( 3 - 1, 3 - 1) and ( - 3 - 1, 3 + 1) Thus, C1C2 = 5 = r1 – r2
Hence, the equation of the tangents T2 and T3 are So, the number of common tangents = 1
2. We have, 2(g1g2 + f1f2) = c1 + c2
y - ( 3 - 1) = 3[ x - ( 3 - 1)]
fi 2(1.0 + k ◊ k) = k + 6
and y - ( 3 - 1) = 3[ x + ( 3 + 1)] fi 2k2 – k – 6 = 0

CG_03a.indd 63 2/6/2017 4:01:11 PM


3.64 Coordinate Geometry Booster

fi (k – 2)(2k + 3) = 0 fi l ◊ 1 = (–1)(–2)
fi k = 2, –3/2 fi l=2
3. Let the equation of the circle be 8.
2 2
x + y + 2gx + 2fy + c = 0 A(0, 6)

Ê 1ˆ
and Á m, ˜ be a variable point lies on the circle
Ë m¯
1 2f C(0, 2)
Thus, m 2 + 2
+ 2gm + +c=0 B(–2 3, 0) B(2 3, 0)
m m
fi m4 + 1 + 2gm3 + 2fm + cm2 = 0
fi m4 + 2gm3 + cm2 + 2fm + 1 = 0 Clearly, radius = 4.
It has four roots, say m1, m2, m3 and m4. 9. Given circle is
x2 + y2 – 4x – 6y + l = 0
Thus, m1m2m3m4 = 1
fi m1m2m3m4 + 4 = 5 fi ( x - 2) 2 + ( y - 3) 2 = ( 13 - l ) 2
4. The equation of the common chord is Clearly, 13 - l = 3
6x + 14y + p + q = 0 fi 13 – l = 9
Here, the centre of the 1st circle (1, –4) lies on the com- fi l=4
mon chord. 10. Clearly, m1m2 = 1
fi m1m2 + 4 = 5
So, 6 – 56 + p + q = 0
p + q = 50
Previous Years’ JEE-Advanced Examinations
Ê p+q ˆ
Hence, the value of Á + 2˜ = 7
Ë 10 ¯ 1. Given lines are 3x + 5y – 1 = 0,
5. We have, |r1 – r2| < C1C2 < r1 + r2 (2 + c)x + 5c2y – 1 = 0
fi |r – 3| < C1C2 < r + 3 On solving, we get
fi |r – 3| < 5 < r + 3 x y 1
fi = =
fi –5 < (r – 3) < 5, r + 3 > 5 - 5 + 5c 2 - (2 + c) + 3 15c 2 - 5(2 + c)
fi –2 < r < 8, r > 2 x y 1
fi 2<r<8 2
= = 2
5(c - 1) - (c - 1) 5(3c - c - 2)
Clearly, n = 2, m = 8
(c 2 - 1) - (c - 1)
Hence, the value of (m – n) is 6. fi x= ,y=
6. The equation of any line passing through P is (3c 2 - c - 2) 5(3c 2 - c - 2)
(c - 1)(c + 1) - (c - 1)
y — 2 2 = - ( x + 2 2) fi x= ,y=
(c - 1)(3c + 2) 5(c - 1)(3c + 2)
fi y = –x
(c + 1) 1
and the equation of the circle is fi x= ,y=-
2 2
(3c + 2) 5(3c + 2)
x + y = 16
when c tends to 1, then
Y
P(–2 2, 2 2) 2 1
x= ,y=-
5 25
A
Now, radius,
X¢ O X
2 2
Ê 2ˆ Ê 1ˆ
r = Á2 - ˜ + Á0 + ˜
B Ë 5¯ Ë 25 ¯
Y¢ 64 1 1601
= + =
Clearly, AB = 8 25 625 625
7. As we know that if the lines a1x + b1y + c1 = 0 and Hence, the equation of the circle is
a2x + b2y + c2 = 0 cut the x and y axes in four concyclic 2 2
Ê 2ˆ Ê 1ˆ 1601
points, then ÁË x - ˜¯ + ÁË y + ˜¯ = .
a1a2 = b1b2 5 25 625

CG_03a.indd 64 2/6/2017 4:01:12 PM


Circle 3.65

2. The equation of a circle passing through the point of fi 9y2 + 60y + 100 + 16y2 –24y – 80 + 64y – 320 = 0
intersection of x2 + y2 = 6 and x2 + y2 – 6x + 8 = 0 is fi 25y2 + 100y – 300 = 0
fi (x2 + y2 – 6) + l(x2 + y2 –6x + 8) = 0). fi y2 + 4y – 12 = 0
which is passing through (1, 1). fi (y + 6)(y – 2) = 0
So, (2 – 6) + l(10 – 6) = 0 fi y = 2, –6
fi l=1 when y = 2, –6, then x = 4, –2
Hence, the equation of the circle is Hence, the points of intersection are (4, 2), (–2, –6)
(x2 + y2 – 6) + 1 ◊ (x2 + y2 – 6x + 8) = 0 7. Equation of any circle passing through the point of in-
fi 2(x2 + y2) – 6x + 2 = 0 tersection of the given circles is
fi (x2 + y2) – 3x + 1 = 0 (2x2 + 2y2 + 4x – 7y – 25) +l(x2 + y2 + 13x – 3y) = 0
which is passing through the point (1, 1).
3. The equations of the tangents at B and D are
fi –24 + 12l = 0
y = 7 and 3x – 4y = 20 fi l=2
B(1, 7) Hence, the equation of the circle is
(2x2 + 2y2 + 4x – 7y – 25)
+ 2(x2 + y2 + 13x – 3y) = 0
C
A fi 4x + 4y + 30x – 13y – 25 = 0
2 2
(1, 2) 8. Ans. (b)
9. Do yourself
D(4, –2) 10. Equation of the chord bisected at M is
ax + by = a2 + b2
Thus, the co-ordinates of C are (16, 7).
Hence, the area of the quadrilateral ABCD = 2(DABC)
Ê1 ˆ O
= 2 Á ¥ AB ¥ BC ˜
Ë2 ¯
= AB ◊ BC B
P(h, k) A M(a, b)
= (5 ¥ BC)
= 5 ¥ 15
which is passing through P(h, k).
= 75 sq. u.
So, ah + bk = a2 + b2
4.
Hence, the locus of M(a, b)
P(–4, 3) hx + ky = x2 + y2
x+y=4 11. Let AB be a chord of the circle.
C(h, k) Draw a perpendicular from the centre O to the chord
N AB at M. Then AM = BM.
Now, –AOM = 45°
OM OM
fi cos 45° = = O
M x–y=2 OA 2
Clearly, CM = CN OM 1 A M(h, k) B
fi =
h+k -4 h-k -2 2 2
fi =
2 2 fi OM2 = 2
fi (h + k – 4) = ±(h – k – 2) fi h2 + k2 = 2
fi h = 3, k = 1 Hence, the locus of M(h, k) is
Now, r = (3 + 4) 2 + (1 - 3) 2 = 53 x2 + y2 = 2
12. Given equation is x2 = 2ax – b2 = 0
Hence, the equation of the circle is
(x – 3)2 + (y – 1)2 = 53 Let its roots are x1 and x2.
fi x2 + y2 – 6x –2y– 43 = 0 Thus, x1 + x2 = 2a, x1 ◊ x2 = –b2
5. Ans. k π 1 Similarly, y1 + y2 = –2p, y1 ◊ y2 = –q2
6. Given circle is Hence, the equation of the circle is
x2 + y2 – 2x + 4y – 20 = 0 fi (x – x1)x – x2) + (y – y1)(y – y2) = 0
2
fi Ê 3y + 10 ˆ 2 Ê 3y + 10 ˆ fi x2 – (x1 + x2)x + x1x2 + y2 – (y1 + y2)y + y1y2 = 0
ÁË ˜ + y - 2 ÁË ˜ + 4y - 20 = 0
4 ¯ 4 ¯ fi x2 + 2ax – b2 + y2 + 2py – q2 = 0
fi (3y + 10)2 + 16y2 – 8(3y + 10) + 64y – 320 = 0 fi x2 + y2 + 2ax + 2py – (b2 + q2) = 0

CG_03a.indd 65 2/6/2017 4:01:12 PM


3.66 Coordinate Geometry Booster

13. Diameter = Distance between two parallel lines 17. Let the co-ordinates of M be (h, k).
Ê Ê 7ˆ ˆ
ÁË 4 - ÁË - 2 ˜¯ ˜¯ , k)
= M(h
32 + 42
B
3
= A(0, 3)
2
3 Clearly, B is the mid-point of AM.
Hence, the radius is.
4 h k + 3ˆ
Thus, ÊÁ ,
14. Equation of the chord bisected at M(h, k) is T = S1 Ë 2 2 ˜¯
Since B lies on x2 + 4x + (y – 3)2 = 0. So
2 2
Ê hˆ Ê hˆ Ê k + 3 ˆ
C(1, 0) ÁË ˜¯ + 4 ÁË ˜¯ + ÁË - 2˜ = 0
¯
2 2 2

B fi h2 + 8h + (k – 3)2 = 0
O M(h, k)
Hence, the locus of M (h, k) is
fi xx1 + yy1 - ( x + x1 ) = x12 + y12 - 2x1 x2 + 8x + (y – 3)2 = 0
fi hx + ky – (x + h) = h2 + k2 – 2h 18. Since the lines 5x + 12y = 10 and 5x – 12y = 40 touch
the circle C1, its centre lies on one of the angle bisec-
which is passing through origin. tors of the given lines.
So, 0 + 0 – 0 – h = h2 + k2 – 2h
fi h2 + k2 – h = 0 5x + 12y — 10 5x - 12y — 40
=
Hence, the locus of M(h, k) is 25 + 144 25 + 144
x2 + y2 – x = 0.
15.
D(4, 5)
B
C
5x – 12y = 40
C 3
A
5x + 12y = 10
A 4 M B

Here, C = (2, 1) and r = 4 fi |5x + 12y – 10| = |5x – 12y – 40|


\ Area of quadrilateral ACBD = 2(DACD) fi (5x + 12y – 10) = ±(5x – 12y – 40)
1 fi x = 5 and y = –5/4
¥4¥2= 2¥
2 Since the centre lies on the first quadrant, let its co-
= 8 sq. u. ordinates be (5, k).
16. The equation of the line of intersection is We have CM = 3
Ê 2 2 2 ˆ 5.5 + 12k - 10
ÁË x + y - x + 4y - 3˜¯ fi =3
3 25 + 144
fi –(x2 + y2 + 6x + 2y – 15) = 0 15 + 12k
fi =3
Ê2 ˆ 13
fi - Á + 6˜ x + (4 - 2) y + 12 = 0
Ë3 ¯ fi (15 + 12k) = ±39
20 9
fi - x + 2 y + 12 = 0 fi k = 2, -
3 2
10 Thus, k = 2 and r = 32 + 42 = 5
fi - x+ y+6=0
3 Hence, the equation of the circle is
fi –10x + 3y + 18 = 0 (x – 5)2 + (y – 2)2 = 25
fi 10x – 3y – 18 = 0. fi x2 + y2 – 10x – 4y + 4 = 0

CG_03a.indd 66 2/6/2017 4:01:13 PM


Circle 3.67

19. 21. The equation of the chord of contact is


Y 4x + 3y = 9
\ Length of the tangent PQ = PR
= 16 + 9 - 9 = 4
Q
M(h, k) 256 12
Now, QM = 16 - =
S 25 5
Q
X¢ R X
O P
L2 = 0 L1 = 0
Y¢ P(4, 3) M
20. Given circle is
x2 + y2 – 4x – 4y + 4 = 0
fi (x – 2)2 + (y – 2)2 = 4 R
Hence, the area of the triangle PQR
B(0, b) = 2(DPQM)
1 12 16
M =2¥ ¥ ¥
2 2 5 5
192
C(2, 2) =
25
O 22.
A(a, 0)

x y
Let AB: + =1
a b C
Now, CM = 2
q
2 2 9
+ -1
fi a b =2 A B
1 1 M
+
a 2 b2 Here, AB = 2. So, AM = 1
2 2 Ê p ˆ AM 1
+ -1 In DACM, sin Á ˜ = =
a b Ë 9 ¯ AC r
fi = ±2
1 1 1 Êpˆ
+ fi r= = cosec Á ˜
a 2 b2 Êpˆ Ë 9¯
sin Á ˜
2 2 Ë 9¯
+ -1
fi a b = -2 23.
1 1
+
a 2 b2
[since the origin and (2, 2) lie on the same side of the
3
line AB] O¢
O
Ê a bˆ 4
Here, circumcentre = M = ÁË , ˜¯ 5 C1
2 2
Therefore the locus of the circumcentre is A
C2
1 1 1 1
fi + -1= -2 2
+ 2 Clearly, OO ¢ = 25 — 16 = 3
x y 4x 4y
3
1 1 1 1 It is given that, tan q =
fi + -1= - 2 + 2 4
x y x y sin q cos q 1
fi = =
3 4 5
fi x + y - xy + x 2 + y 2 = 0
Let the co-ordinates of the centre O¢ = (x, y) of the cir-
Thus, the value of k is 1. cle C2.

CG_03a.indd 67 2/6/2017 4:01:15 PM


3.68 Coordinate Geometry Booster

x-0 y-0 Equation of PQ is


Therefore, = = ±3 h
4 3 y — k = — ( x - h)
5 5 k
12 9 fi hx + ky = h2 + k2
fi x=± ,y=±
5 5 hx + ky
Ê 12 9 ˆ fi =1 …(i)
Thus, O ¢ = Á ± , ± ˜ . h2 + k 2
Ë 5 5¯
The equation of the pair of lines joining the point of
24. The equation of a circle be intersection of (i) with S = 0 is
x2 + y2 + 2gx + 2fy + c = 0 …(i) 2
Ê hx + ky ˆ Ê hx + ky ˆ
x 2 + y 2 + (2gx + 2fy ) Á 2 2˜
+ cÁ 2 =0
Ë h + k 2 ˜¯
which passes through (a, b). So
Ëh +k ¯
a2 + b2 + 2ga + 2fb + c = 0 …(ii) …(ii)
2 2 2
Also (i) is orthogonal to x + y = k . So As the line (ii) are at right angles, so
2(g ◊ 0 + f ◊ 0) = c – k2 (h 2 + k 2 )
Ê 2gh + 2fk ˆ
fi c = k2 1+1+ Á 2 + c =0
Ë h + k 2 ˜¯
…(iii)
(h 2 + k 2 ) 2
From Eqs (ii) and (iii), we get
c
a2 + b2 + 2ga + 2fb + k2 = 0 fi (h 2 + k 2 ) + gh + fk + = 0
fi a2 + b2 – 2(–g)a – 2(–f)b + k2 = 0 2
Hence, the locus of the centre (–g, –f) is Hence, the locus of M(h, k) is
a2 + b2 – 2xa – 2yb + k2 = 0 c
fi ( x 2 + y 2 ) + gx + fy + = 0
fi 2ax – 2by – (a2 + b2 + k2) = 0 2
25. Given circle is (x + r)2 + (y – h)2 = r2 27. Given two circles intersect. We have
Equation of any tangent passing through origin is |r1 – r2| < C1C2 < r1 + r2
y = mx fi mx – y = 0 fi |r – 3| < 5 < r + 3
As we know that the length of the perpendicular from fi |r – 3| < 5, 5 < r + 3
the centre of the circle to the tangent is equal to the fi –5 < (r – 3) < 5, r > 2
radius of a circle.
fi –2 < r < 8, r > 2
- rm - h
Thus, =r fi 2<r<8
m2 + 1 28. Let r be the radius of the circle.
fi ( - rm - h) 2 = (r m 2 + 1) 2 So, pr2 = 154
fi 22
(rm + h)2 = r2(m2 + 1) fi ¥ r 2 = 154
fi r2m2 + 2rmh + h2 = r2m2 + r2 7
fi 2rmh + h2 = r2 fi r2 = 7 ¥ 7
fi r=7
r 2 - h2
fi m= Now, the centre of the circle is the point of intersection
2rh of 2x – 3y = 5 and 3x – 4y = 7. So
Hence, the equation of the circle is C = (1, –1)
Ê r 2 - h2 ˆ Hence, the equation of the circle is
y=Á x
Ë 2rh ˜¯ (x – 1)2 + (y + 1)2 = 72
fi (r2 – h2)x – 2rhy = 0 fi x2 + y2 – 2x + 2y – 47 = 0
fi (h2 – x2)x + 2rhy = 0 29 Let the equation of the circle be
26. Let PQ be the chord of the given circle which subtends x2 + y2 +2gx + 2fy + c = 0
a right angle at the origin O and M(h, k) be the foot of Ê 1ˆ
Let Á m, ˜ be an arbitrary point on the circle.
the perpendicular from O on this chord PQ. Ë m¯
P 1 2f
fi m 2 + 2 + 2gm + +c=0
m m
fi m4 + 1 + 2gm3 + 2fm + cm2 = 0
O M(h, k) fi m2 + 2gm3 + cm2 + 2fm + 1 = 0
Let m1, m2, m3, m4 be the roots.
1
Q Thus, m1 ◊ m2 ◊ m3 ◊ m4 = = 1 .
1

CG_03a.indd 68 2/6/2017 4:01:15 PM


Circle 3.69

30. Given circle is x2 + y2 – 6x + 2y = 0


Centre is (3, –1). Also, r = CM 2 + (3 2) 2
Clearly, the centre (3, –1) satisfies the equation of the fi r = CM 2 + 18
diameter x + 3y = 0.
31. fi r2 = CM2 + 18
Y (h + k )2
fi r2 = + 18
B 2
(h — k ) 2 (h + k )2
P fi = + 18
2 2
X (h — k ) 2 (h + k )2
X¢ O M A fi - = 18
2 2
fi –2hk = 18
fi hk = –9 …(iii)
Y¢ From Eq. (ii), we get
The equation of tangent to the circle x2 + y2 = 4 at h+k=8
9
(1, 3) is fi h— =8
h
x + 3y = 4
fi h2 – 8h – 9 = 0
2 2
The equation of normal to the circle x + y = 4 at fi (h – 9)(h + 1) = 0
(1, 3) is fi h = –1, 9
y = 3x and k = 9, –1
Hence, the equation of the circle is
Thus, area of the triangle OPA
(x + 1)2 + (y – 9)2 = 50
1
= ¥ OA ¥ PM or
2
(x – 9)2 + (y + 1)2 = 50
1
= ¥4¥ 3 3 3
2 33. Ans. ¥ r 2 sq. units
= 2 3 sq. u. 4
34.
32. Let C(h, k) be the centre and r be the radius of the given 4x + 3y = 10
circle.
Y
5 5
C1 P (1, 2) C2
O
X¢ Q X
P M
x–y=0 R
C The equation of the line where both the centres lie is
x+y=0
3x – 4y + k = 0 which is passing through the point.
Thus, k = 5
Hence, the required line is 3x – 4y + 5 = 0
Y¢ 3
Clearly, tan q =
h-k 4
Thus, =r …(i)
sin q cos q 1
2 fi = =
3 4 5
Given OP = 4 2, QR = 6 2 Therefore, the co-ordinates of C1 and C2 can be ob-
So, QM = MR = 3 2 tained from
x -1 y - 2
Clearly, CM = OP = 4 2 fi = = ±5
4 3
h+k 5 5
=4 2 …(ii) fi x = 1 ± 4, y = 2 ± 3
2
fi x = 5, –3; y = 5, –1

CG_03a.indd 69 2/6/2017 4:01:16 PM


3.70 Coordinate Geometry Booster

Therefore, the equations of the required circles are 37. Given circle is
(x – 5)2 + (y – 5)2 = 25 2x(x – a) + y(2y – b) = 0
or Y
(x + 3)2 + (y + 1)2 = 25
35. Here, A, B, C and D are concyclics.
P(a, b/2)

D(0, 1)
M(c, 0) X
lx – y + 1 = 0 O

=0
y+3 B(0, 3/2)
x–2 O fi 2x2 – 2ax + 2y2 – by = 0
A(–3, 0) C – 1 , 0 fi 2x2 + 2y2 – 2ax – by = 0
l The equation of the chord bisected at M is T = S1
Ê y + y1 ˆ
Thus, OA.OC = OB.OD fi 2xx1 + 2yy1 - a ( x + x1 ) - b Á
Ë 2 ˜¯
1 3
fi - 3. - = ◊ 1
l 2 = 2x12 + 2y12 - 2ax1 - by1
fi l=2 Ê y + 0ˆ
fi 2x ◊ c + 0 - a( x + c) - b Á
36. Consider three circles with centres at A, B and C with Ë 2 ˜¯
radii r1, r2, r3 respectively, which touch each other ex- = 2c2 + 0 – 2ac – 0
ternally at P, Q, R. fi 4cx + 2ax – 2ac – by = 4c2 – 4ac
which is passing through P(a, b/2). So
C b2
r3
r3 4ca - 2a 2 — 2ac - = 4c 2 - 4ac
2
P R
r1
O fi 8ca – 4a2 – 4ac – b2 = 18c2 – 8ac
r2
A fi 8c2 – 12ac + (4a2 + b2) = 0
r1 Q r
2 B Now, D > 0
fi 144a2 – 32(4a2 + b2) > 0
fi 9a2 – 2(4a2 + b2) > 0
fi 9a2 + 8a2 – 2b2 > 0
Let the common tangents at P, Q, R meet each other at
O. fi a2 > 2b2
38.
Then OP = OQ = QR = 4 Y
Also, OP ^ AB, OQ ^ AC, OR ^ BC
Here, O is the incentre of the triangle ABC.
For DABC, C(h, k)
(r + r ) + (r3 + r2 ) + (r1 + r3 )
s= 1 2 = r1 + r2 + r3 X¢ X
2 O A(1, 0)

and D = (r1 + r2 + r3 )r1r2 r3


D
Now, from the relation r = , we get Y¢
s
Clearly, OC = OA
(r1 + r2 + r3 )r1r2 r3
=4 fi OC2 = CA2
r1 + r2 + r3
fi (h2 + k2) = (h – 1)2 + k2
r1r2 r3
fi =4 fi (h2 + k2) = h2 – 2h + 1 + k2
r1 + r2 + r3
fi –2h + 1 = 0
r1r2 r3 16
fi = 16 = 1
r1 + r2 + r3 1 fi h=
2
fi (r1r2r3) : (r1 + r2 + r3) = 16 : 1

CG_03a.indd 70 2/6/2017 4:01:17 PM


Circle 3.71

Also, C1C2 = |r1 – r2| fi –5x – 6y + 56 = 0, 2x + 3y – 27 = 0


23
fi OC = 3 - h 2 + k 2 fi x = 2, y =
3
Ê 23 ˆ
fi h2 + k 2 = 3 - h2 + k 2 Hence, the co-ordinates of the point is Á 2, ˜ .
Ë 3¯
fi 2 h2 + k 2 = 3 41. Given circles are
x2 + y2 – 4x – 2y + 4 = 0
9
fi h2 + k 2 = fi (x – 2)2 + (y – 1)2 = 1
4
and (x – 6)2 = (y – 4)2 = 42
fi 9 9 1
k2 = - h2 = - = 2 fi (x – 6)2 + (y – 4)2 = 42
4 4 4
Here, C1 = (2, 1), r1 = 3; C2 = (6, 4), r2 = 4
fi k= 2
1
Hence, the centre is ÊÁ , 2 ˆ˜ . Now, C1C2 = (6 - 2) 2 + (4 - 1) 2 = 5
Ë2 ¯
= r1 + r2
39. Given circle is
N
x2 + y2 – 6x – 6y + 14 = 0 M
fi (x – 3)2 + (y – 3)2 = 4 1 4
P
Y C1(2, 1) C2(6, 1)

4.2 + 1.6 4.1 + 1.4 ˆ Ê 14 8 ˆ


Therefore, D = ÊÁ , =Á , ˜
C1(h, k) C2(3, 3) Ë 4 +1 4 + 1 ˜¯ Ë 5 5 ¯
Ê 4.2 - 1.6 4.1 — 1.4 ˆ Ê 2 ˆ
and P = Á , = Á , 0˜
X Ë 4 -1 4 - 1 ˜¯ Ë 3 ¯
O
The equations of the tangent through P is
Ê 2ˆ
fi y - 0 = mÁ x - ˜
We have, Ë 3¯
C1C2 = r1 + r2 fi 3y = m(3x – 2)
fi 3mx – 3y – 2m = 0
fi (h - 3) 2 + (k - 3) 2 = h + 2 Now, C1M = 1
fi (h – 3)2 + (k – 3)2 = (h + 2)2 3m.2 - 3.1 - 2m
fi =1
fi h2 – 6h + 9 + k2 – 6k + 9 = h2 + 4h + 4 9m 2 + 9
fi k2 – 10h – 6k + 14 = 0
4m - 3
Hence, the locus of (h, k) is fi =1
y2 – 10x – 6y + 14 = 0 3 m2 + 1
40. The equation of any circle passing through A(3, 7) and fi (4m – 3)2 = 9(m2 + 1)
fi 16m2 – 24m + 9 = 9m2 + 9
B(6, 5) is
fi 7m2 – 24m = 0
fi m(7m – 24) = 0
x y 1 fi m = 0, (7m – 24) = 0
( x - 3)( x - 6) + ( y - 7)( y - 5) + l 3 7 1 = 0 24
fi m = 0, m =
6 5 1 7
Hence, the equations of tangents are
fi S1:(x – 3)(x – 6) + (y – 7)(y – 5) + l(2x + 3y – 27) = 0
24
fi S1: x2 + y2 – 9x – 12y + 53 + l(2x + 3y – 27) = 0 y = 0 and y = ( x - 3)
7
…(i)
fi y = 0 and 24x – 7y – 72 = 0
The equation of the common chord of (i)
42. Given circle is
and S2: x2 + y2 – 4x – 6y – 3 = 0 …(ii)
4x2 + 4y2 – 12x + 4y + 1 = 0
is S1 – S2 = 0. x2 + y2 – 3x + y + (1/4) = 0
fi –5x = 6y + 56 + k(2x + 3y – 27) = 0 x2 + y2 – 3x + y + (1/4) = 0

CG_03a.indd 71 2/6/2017 4:01:17 PM


3.72 Coordinate Geometry Booster

Let x be the side of a square.


3 1 9 1 1 3
Thus, C = ÊÁ , - ˆ˜ , r = + - = 2
Ë 2 2¯ 4 4 4 2 Ê a ˆ a2
Thus, x 2 + x 2 = Á ˜ =
In DACM, Ë 3¯ 3
CM a2
cos (60°) = fi 2 x2 =
AC 3
CM 1
fi =
3/2 2 C(3/2, 1/2) a2
fi x2 =
3 3/2 60° 6
fi CM =
4 a2
A M(h, k) B Area of a square =
9 6
fi CM 2 =
16 45. Given circle is
2 2 x2 + y2 + 4x – 6y + 9 sin2 a +13 cos2 a = 0
Ê 3ˆ Ê 1ˆ 9
fi ÁË h - ˜¯ + ÁË k - ˜¯ = (x + 2)2 + (y – 3)2 = 4 – 4 cos2 a
2 2 16
(x + 2)2 + (y – 3)2 = (2 sin a)2
Hence, the locus of M(h, k) is
2 2
(x + 2)2 + (y – 3)2 = (2 sin a)2
Ê 3ˆ Ê 1ˆ 9
ÁË x - ˜¯ + Á y - ˜¯ = Let the P be (h, k).
2 Ë 2 16
A
43. Here, C1 = (5, 0), r1 = 3, C2 = (0, 0), r2 = r
It is given that two circles intersect. P(h, k) a
C(–2, 3)
So, |r1 – r2| < C1C2 < r1 + r2
fi |3 – r| < 5 < 3 + r B
fi |3 – r| < 5, 5 < 3 + r
We have
fi –5 < (r – 3) < 5, r > 2
AC
fi –2 < r < 8, r > 2 sin a =
PC
fi 2<r<8
2 sin a
44. Let PM = p fi sin a =
(h + 2) 2 + (k - 3) 2
P fi (h + 2)2 + (k – 3)2 = 4
fi h2 + k2 + 4h – 6k + 9 = 0
Hence, the locus of (h, k) is
D C x2 + y2 + 4x – 6y + 9 = 0
46. Let r be the radius of a circle, then AC = 2r
O

A B
Q M R

PM p
Now, sin (60°) = =
PQ a
p 3
fi =
a 2
3
fi p= a
2
Here O is the centroid. Since, AC is the diameter –ABC = 90°
So the centroid divides the median in the ratio 2 : 1. In DABC, BC = 2r sin b, AB = 2r cos b
p BD = AB tan a = 2r cos b tan a
Thus, OM = AD = AB sec a = 2r cos b sec a
3
p DC = BC – BD = 2r sin b – 2r cos b tan a
fi r= Since E is the mid-point of DC , so
3
DC
2p 2 3 a DE = = r sin b - r cos b tan a
fi 2r = = ¥ a= 2
3 3 2 3

CG_03a.indd 72 2/6/2017 4:01:19 PM


Circle 3.73

Now, in DADC , AE is the median Hence, the required equation of the circle is
2 2
2( AE + DE ) = AD + AC 2 2 x2 + y2 – 2x + (x – y) = 0
fi x2 + y2 – x – y = 0
2(d 2 + r 2 (sin b - cos b tan a ) 2 )
49. Given C is the circle with centre at (0, 2)
= 4r 2 cos 2 b sec 2a + 4r 2 and radius r (say)
d 2 cos 2a Then x 2 + ( y - 2) 2 = r 2
r2 =
cos 2a + cos 2 b + 2 cos a cos b cos( b - a ) ( y - 2) 2 = r 2 - x 2
2
Hence, the area of a circle = pr
( y - 2) = ± r 2 - x 2
p d 2 cos 2a
2 2
cos a + cos b + 2 cos a cos b cos (b - a ) y = 2 ± r 2 - x2
The only rational value of y is 0
Ê1 + a 2 1 - a 2 ˆ
47. Let (p, q) = Á , ˜ Suppose the possible value of x for which y is 0 is x1.
Ë 2 2 ¯ Certainly, y – x1 will also give the value of y as 0. Thus,
Given circle is 2(x2 + y2) – 2px – 2qy = 0 atmost there are two rational points which satisfy the
A(p, q) equation of the circle.
50. Let the point P be (h, k).
M(h, –h) x- p y-q
x+ Thus, = =r …(i)
B y= cos q sin q
0
fi x = p + r cos q, y = q + r sin q
The point P lies on the curve.
So, a(p + r cos q)2 + b(q + r sin q)2
The equation of the chord bisected at M is + 2h(p + r cos q)(q + r sin q) = 1
2(hx – hy) – p(x + h) – q(y – h) fi (a2 cos2 q + 2h cos q sin q + b2 sin2 q) r2
= 2(h2 + h2) – 2ph + 2qh +2[p(a cos q + h sin q) r + q(h cos q + b sin q)]r
+ ap2 + 2hpq + bq2 – 1 = 0 …(ii)
which is passing through A(p, q).
Let PQ = r1 and PR = r2.
fi 2(hp – hq) – p(p + h) – q(q – h) = 4h2 – 2ph + 2qh
Also, let r1 and r2 are the roots of Eq. (ii)
fi 3hp – 3hq – p2 – q2 – 4h2 = 0
2(ap 2 + 2hpq + bq 2 - 1)
fi 4h2 + 3(q – p)h + (p2 + q2) = 0 Thus, r1r2 =
(a + b) + 2h sin 2q + (a - b) cos 2q
Since chords are distinct, so
D>0 Since the product of PQ and PR is the independent of q
so, h = 0, a = b and a π 0.
fi 9(q – p)2 – 16(p2 + q2) > 0
So the given product becomes
fi 9( - a 2) 2 - 8(1 + 2a 2 ) > 0 1
fi 18a2 – 16a2 – 8 > 0 x2 + y 2 =
a
fi 2a2 – 8 > 0 which represents a circle.
fi a2 – 4 > 0 51.
fi (a + 2)(a – 2) > 0 52. Any point on the line 2x + y = 4 can be considered as
P(a, 4 – 2a).
fi a < –2, a > 2
The equation of the chord of contact of the tangent to
Thus, a Œ (– , 2) » (2, ) the circle x2 + y2 = 1 from P is
48. The equation of any circle passing through the point of xx1 + yy1 – 1 = 0
intersection of x2 + y2 – 2x = 0 and y = x is fi ax + (4 – 2a)y – 1 = 0
x2 + y2 – 2x + l(x – y) = 0 fi a(x – 2y) + (4y – 1) = 0
fi x2 + y2 + (l – 2)x – ly = 0 1
fi ( x - 2y ) + (4y - 1) = 0
Ê 2 — l lˆ a
Its centre is Á , ˜ 1
Ë 2 2¯ fi ( x - 2y ) + l (4y - 1) = 0, l =
a
The centre lies on y = x Thus, x – 2y = 0, 4y – 1 = 0
2—l l 1 1
So, = fi x= ,y= .
2 2 2 4
Ê1 1ˆ
fi l=1 Hence, the required point is Á , ˜.
Ë2 4¯

CG_03a.indd 73 2/6/2017 4:01:20 PM


3.74 Coordinate Geometry Booster

53. Since two vertices of an equilateral triangle are h = 2x


B(–1, 0) and C(1, 0). Similarly, k = 2y
So, the third vertex must lie on the y-axis. Putting in (i), we get,
Let the third vertex be A(0, b) 4x2 + 4y2 = 4r2
Now, AB = BC = CA x2 + y2 = r2
fi AB2 = BC2 = AC2 Hence, the locus is x2 + y2 = r2
fi 1 + b2 = 4 = 1 + b2 56. Here A0A1 = 1
fi b2 = 4 – 1 12 + 12 - A0 A22
fi b= 3 fi cos (120°) =
2.1.1
Thus, the third vertex is A = (0, 3).
1 12 + 12 - A0 A22
As we know that in case of an equilateral triangle, fi - =
2 2
Ê 1 ˆ A3
Circumcentre = Centroid = Á 0, ˜ fi A0A22 = 3 A4
Ë 3¯
Hence, the equation of the circumcircle is fi A0 A2 = 3
O A2
Ê 1 ˆ
2
Ê 1 ˆ
2 Similarly, A0 A4 = 3
2
( x — 0) + Á y - ˜ = (1 - 0) 2 + Á 0 - ˜
A5
Ë 3¯ Ë 3¯ Hence, the value of
2 A0 A1 A0 A2 A0 A4
Ê 1 ˆ 4
fi x2 + Á y - ˜ = . A0
A1
Ë 3¯ 3 = 1◊ 3 ◊ 3
2 2 = 3.
54. Given circles are x + y = 4
and x2 + y2 – 6x – 8y – 24 = 0 57. Y
Here C1 = (0, 0), r1 = 2 ; C2 = (3, 4), r2 = 7
Now, C1C2 = 5 = r2 – r1
So, two circles touch each other internally. B(p, q)
Thus, the number of common tangents = 1
55. Let P(h, k) be on C2
X
So, h2 + k2 = 4r2 ...(i) O M(h, 0)

The equation of the chord bisected at M(h, 0) is


Ê x + x1 ˆ Ê y + y1 ˆ
xx1 + yy1 - p Á - qÁ
Ë 2 ˜¯ Ë 2 ˜¯
= x12 + y12 - px1 - qy1
Ê x + hˆ Ê y + 0ˆ
fi hx + 0 - p Á - qÁ
Ë 2 ˜¯ Ë 2 ˜¯
= h2 + 0 – ph – 0
fi 2hx – px – ph – qy = 2h2 – 2ph
fi 2hx – px – qy = 2h2 – ph
Chord of contact of P w.r.t C1 is hx + ky = r2 fi 2h2 – 2hx – ph + (px + qy) = 0
It intersects C1 x2 + y2 = a2 in A and B.
which is passing through (p, q)
Eliminating y, we get,
2
So, 2h2 – 2ph – ph + (p2 + q2) = 0
Ê r 2 - hx ˆ fi 2h2 – 3ph + (p2 + q2) = 0
x2 + Á = r2
Ë k ˜¯ Clearly, D > 0
(h 2 + k 2 ) x 2 - 2r 2 hx + r 2 (r 2 - k 2 ) = 0 fi 9p2 – 8(p2 + q2) > 0
fi p2 – 8q2 > 0
4r 2 x 2 - 2r 2 hx + r 2 (r 2 - k 2 ) = 0
fi p2 > 8q2
h k 58. The given circle is x2 + y2 = r2 ...(i)
Thus, x1 + x2 = , y1 + y2 =
2 2 Centre is (0, 0) and radius = 1
Let (x, y) be the centroid of DPAB Let T1 and T2 be the tangents drawn from (–2, 0) to the
h 3h circle (i)
Thus, 3x = x1 + x2 + h = + h =
2 2

CG_03a.indd 74 2/6/2017 4:01:21 PM


Circle 3.75

Let m be the slope of the tangent, then the equations of For the circles (iii) and (iv), there will be four com-
tangents are mon tangents of which 2 are direct and another two are
y – 0 = m(x + 2) transverse common tangents.
mx – y + 2m = 0 ...(ii) In two triangles, DC1XN, DC2YN
C1 N 3
= =9
C2 N 1/3
Thus, N divides C1C2 in the ratio 9:1

2m
Clearly, =1
m2 + 1
Clearly, N lies on x-axis
1
m=±
3 Ê 4 ˆ
N = Á - , 0˜
Thus, the two tangents are Ë 5 ¯
T1: x + 3 y = 2 Ê 4ˆ
Any line through N is y = m Á x + ˜
Ë 5¯
T2 : x - 3 y = 2
5mx – 5y + 4m = 0
Now any other circle touching (i) and T1, T2 is such that If is is tangent to the circle (iii), then
its centre lies on x-axis 20m + 4m
Let (h, 0) be the centre of such circle =3
Thus, OC1 = OA + AC1 2m 2 + 25m
|h| = 1 + |AC1| 5
m=±
But AC1 = Perpendicular distance from (h, 0) to the 39
tangents Hence, the required tangents are
h+2 5 Ê 4ˆ
|h| = 1 + y=±
2 Á x + ˜¯
39 Ë 5
h+2 59. Let z1 = Q = 3 + 4i
|h| - 1 =
2 Then z2 = R = iz = i(3 + 4i) = 3i – 4
h 2 + 4h + 4 = – 4 + 3i
h2 - 2 | h | + 1 =
4 = (–4, 3)
4 p
h=- ,4 Thus, –QOR = .
3 2
p
Hence, the centres of the circles are Clearly, –QPR = .
Ê 4 ˆ 4
ÁË - , 0˜¯ , (4, 0) 60. The given circles are orthogonal. So,
3
2(g1g2 + f1f2) = c1 + c2
Radius of the circle with centre (4, 0) is 4 – 1 = 3 and the
fi 2(1.0 + k◊k) = k + 6
Ê 4 ˆ 4 1
radius of the circle with centre Á - , 0˜ is - 1 = fi 2k2 = k + 6
Ë 3 ¯ 3 3
Thus, two possible circles are fi 2k2 – k – 6 = 0
(x – 4)2 + y2 = 9 ...(iii) fi 2k2 – 4k + 3k – 6 = 0
2
Ê 4ˆ 2 1 fi 2k(k – 2) + 3(k – 2) = 0
and ÁË x + ˜¯ + y = ...(iv)
3 9 fi (k – 2)(2k + 3) = 0
Since (i) and (iii) are two touching circles, so they have fi (k – 2) = 0, (2k + 3) = 0
three common tangents T1, T2 and x = 1 3
Similarly, common tangents of (i) and (iv) are T1, T2 fi k = 2, -
2
and x = –1

CG_03a.indd 75 2/6/2017 4:01:23 PM


3.76 Coordinate Geometry Booster

61. 64.
Y

B(0, r) C r
P(h, k)
C1
G(h, k) r1 R(0, 0)
X¢ X
O A(r, 0)
r2
Q(a, b)
P
C2

Let A = (r, 0), B = (0, r) Let C1 be the circle with centre R(0, 0) and radius r.
and P = (r cos q, r sin q).
Thus, its equation is x2 + y2 = r2.
1 1
We have h = (r + r cos q ), k = (r + r sin q ) Let C2 be (x – a)2 + (y – b)2 = r12
3 3
and C be (x – h)2 + (y – k)2 = r22.
2 2
Ê rˆ Ê rˆ r2 2 2 It is given that PR = r – r1 and QR = r + r2
\ ÁË h — ˜¯ + ÁË k — ˜¯ = (cos q + sin q )
3 3 9 h 2 + k 2 = r - r1 and (h — a ) 2 + (k - b) 2 = r1 + r2
Hence, the locus of G(h, k) is Adding, we get
2 2
Ê rˆ Ê rˆ r2 h 2 + k 2 + (h — a ) 2 + (k - b) 2 = r + r2
ÁË x — ˜¯ + ÁË y — ˜¯ =
3 3 9
Thus, the locus of P(h, k) is
which represents a circle.
x 2 + y 2 + ( x — a ) 2 + ( y - b) 2 = r + r2
62. Ans. OA = 9 + 3 10
which represents an ellipse with foci at R(0, 0) and
63. Let –SPR = q
S
Q(a, b) and the length of the major axis is r + r2.
65. Clearly, the point Q is (0, 3).

6 =0 Q(0, 3)
Q
x+
–2
X 5x
X¢ X
P

q O
P r r R

Now, the length of the tangent PQ from Q to the circle
x2 + y2 + 6x + 6y = 2 is
Êp ˆ PQ = 0 + 9 + 0 + 18 - 2 = 5
Then –QRP = Á - q ˜ , –PQR = q
Ë2 ¯ 66. Given common tangent is
Êp ˆ PQ y = mx - b 1 + m 2
In DPQR, tan Á - q ˜ =
Ë2 ¯ PR
PQ fi mx - y - b 1 + m 2 = 0
fi cot q = …(i)
2r Now, the length of the perpendicular from the 2nd cir-
RS RS
Also, in DPRS, tan q = = …(ii) cle is equal to the radius of the circle.
PR 2r
From Eqs (i) and (ii), we get am - b 1 + m 2
fi =b
PQ RS m2 + 1
◊ =1
2r 2r
fi am - b 1 + m 2 = b m 2 + 1
fi 4r2 = PQ ◊ RS
fi 2r = PQ ◊ RS fi am = 2b 1 + m 2

CG_03a.indd 76 2/6/2017 4:01:24 PM


Circle 3.77

fi a2m2 = 4b2(1 + m2) 68. From Figure (i),


fi a2m2 = 4b2 + 4b2m2 1 Ê 2p ˆ
I n = n ◊ ◊ (OA1 ) ◊ (OA1 ) sin Á ˜
fi (a2 – 4b2) m2 = 4b2 2 Ë n ¯
4b 2 p Ê 2p ˆ
fi m2 = 2 = sin Á ˜
(a - 4b 2 ) 2 Ë n ¯
From figure (ii)
4b 2
fi m= B1 B2 = 2(B1 L)
( a - 4b 2 )
2

2b Êpˆ
fi m= = 2(OL) tan Á ˜
Ë n¯
(a 2 - 4b 2 )
Êpˆ
67. Given circle is x2 + y2 = r2 = 2 ◊ 1 ◊ tan Á ˜
Ë n¯
A
Êpˆ
= 2 tan Á ˜
Ë n¯
q q
Ê 1 ˆ Êpˆ
P(6, 8) O Thus, On = n Á ( B1 B2 )(OL)˜ = n tan Á ˜
M Ë2 ¯ Ë n¯
I n (n /2) sin (2q )
B Now, = ,
On n tan q
2 2
We have OP = 6 + 8 = 10 p 2 tan q 1
where q = = ◊ = cos2 q
BM = r cos q, OM = r sin q n (1 + tan 2q ) 2 tan q
p 1
where 0 < q < = (2 cos 2q )
2 2
r
Also, sin q = 1
10 = (1 + cos 2q )
2
If A denotes the area of the triangle PAB, then


A = 2ar (DPBM) Ê 2I ˆ
= Á1 + 1 - Á n ˜ ˜
1 2Ë Ë n ¯ ¯
= 2 ¥ ¥ PM ¥ BM
2
On Ê

= PM ¥ BM Ê 2I ˆ
In = Á1 + 1 - Á n ˜ ˜
2 Ë Ë n ¯ ¯
= (OP – OM) ◊ BM
= (10 – r sin q) ◊ r cos q 69. Given x2 – 8x + 12 = 0 and y2 + 14y + 45 = 0
= (10 – 10 sin q ◊ sin q)(10 sin q ◊ cos q) fi (x – 2)(x – 6) = 0 and (y – 5)(y – 9) = 0
= 100 sin q cos3 q fi x = 2, x = 6 and y = 5, y = 9
dA y=9 C(6, 9)
fi = 100 cos q cos3q - 300 sin 2q cos 2q
dq
Ê 1 ˆÊ 1 ˆ
= 300 cos 4q Á - tan q ˜ Á + tan q ˜ x=6
Ë 3 ¯Ë 3 ¯ X=2

dA
For maxima and minima, = 0 gives
dq
1
- tan q = 0 A(2, 5) y = 5
3
p Ê 2 + 6 5 + 9ˆ
fi q= Hence, the centre is Á , ˜ = (4, 7)
6 Ë 2 2 ¯
Ï p 70. The equation of a circle C is
> 0:0 <q <
dA ÔÔ 6 (x – 2)2 + (y – 1)2 = r2 …(i)
Thus, =Ì
dq Ô p p Given circle is
< 0: <q <
ÔÓ 6 2 x2 + y2 – 2x – 6y + 6 = 0 …(ii)
p The equation of the common chord is
Therefore A is maximum, when q= and
6 –2x + 4y + 5 – r2 – 6 = 0
Êpˆ
r = 10.sin Á ˜ = 5 which is a diameter of the circle (ii).
Ë 6¯

CG_03a.indd 77 2/6/2017 4:01:25 PM


3.78 Coordinate Geometry Booster

Thus, –2.1 + 4.3 + 5 – r2 – 6 = 0 By the rotation theorem,


fi 9 – r2 = 0 Ê z2 - z0 ˆ Ê z2 - z0 ˆ ip /2
fi r=3 ÁË z - z ˜¯ = ÁË z - z ˜¯ e = i
1 0 1 0
z -a
71. Given =k fi (z2 – z0) = i(z1 – z0)
z-b
fi z2 = z0 + i(z1 – z0)
| z - a |2
fi = k2
| z - b |2 fi z2 = 1 + i (2 + i 3 - 1) = 1 + i - 3
( z - a )( z - a ) fi z2 = (1 - 3) + i
fi = k2
( z - b )( z - b ) Also, z4 = 2 - z2 = 2 — (1 - 3) + i
2
fi ( z - a )( z - a ) = k ( z - b )( z - b ) z4 = (1 + 3) - i

fi | z|2 - a z - a z + |a |2 = k 2 (| z|2 - b z - b z + |b |2 ) 73. The equation of the family of circle is
2 2 2 2
fi (1 - k ) | z| - (a - k b ) z - (a - b k ) z (x – 1)2 + (y + 1)2 + l(2x + 3y + 1) = 0
+ (|a |2 - k |b |2 ) = 0 x2 + y2 – 2x + 2y + 2 + l(2x + 3y + 1) = 0
x2 + y2 + 2 (l – 1) x + (3l + 2) y + (l + 2) = 0 …(i)
(a - k 2 b ) (a - b k 2 )
fi | z|2 - z- z The equation (i) is orthogonal to the circle
(1 - k 2 ) (1 - k 2 )
x(x + 2) + (y + 1)(y – 3) = 0
(|a |2 - k |b |2 )
+ =0 x2 + y2 + 2x – 2y – 3 = 0
(1 - k 2 )
Therefore,
(a - k 2 b ) È (3l + 2) ˘
Thus, the centre of a circle is = . 2 Í(l - 1) ◊ 1 + ◊ ( -1) ˙ = l - 1
(1 - k 2 ) Î 2 ˚
and its radius
fi 2l – 2 – 3l – 2 = l – 1
2 2
(a - k b ) Ê |a |2 - k 2 bb ˆ 3
= - Á ˜ fi l=-
(1 - k 2 ) Ë (1 - k 2 ) ¯ 2
Hence, the equation of the circle is
Ê (a - k 2 b ) ˆ Ê (a - k 2 b ) ˆ Ê |a |2 - k 2 bb ˆ 3
= Á ˜Á ˜ -Á ˜ ( x - 1) 2 + ( y + 1) 2 — (2x + 3y + 1) = 0
Ë (1 - k 2 ) ¯ Ë (1 - k 2 ) ¯ Ë (1 - k 2 ) ¯ 2
k (a - b ) fi 2(x2 + y2) – 4x + 4y + 4 – 6x – 9y – 3 = 0
= fi 2(x2 + y2) – 10x – 5y + 1 = 0
1 - k2
74. Let A, B, C be the centres of the 3- given circles.
72.
Q(z2) P(2 + i 3)
(z 1 )
A 3
5
3
B
(z 0 ) P
O(1, 0)
4 5
4
C
R(z3) S(z4)

Since the centre of a square coincides with the centre of Clearly P is the incentre of the DABC.
a circle, so
D s ( s - a )( s - b)( s - c)
z1 + z3 Thus, r = =
=1 s s
2
fi z 1 + z3 = 2 ( s - a )( s - b)( s - c)
fi r=
fi z3 = 2 — z1 = 2 - (2 + i 3) = - i 3 s
p 5.4.3
Here, –z1 z0 z2 = fi r= = 5, since s = 12
2 12

CG_03a.indd 78 2/6/2017 4:01:25 PM


Circle 3.79

75. Given C1 = (0, 1)


Y

C r 1 r2
C2
X¢ X
O

Let C2 = (x, y) PA2 + PB 2 + PC 2 + PD 2


Then r1 + r2 = C1C2 Then
QA2 + QB 2 + QC 2 + QD 2
fi 1 + | y| = x 2 + ( y - 1) 2 1+1+ 5 + 5
=
fi 1 + y2 + 2|y| = x2 + y2 – 2y + 1 2[( 2 - 1) + 1] + 2[( 2 + 1) 2 + 1]
2

fi x2 = 2|y| + 2y 12 3
= = = 0.75
fi x2 = 4y if y ≥ 0 16 4
Thus, the locus of its centre is 78. Let C ¢ be the said circle touching C1 and L, so that C1
{(x, y) : x2 = 4y} » {(0, y) : y £ 0} and C ¢ are on the same side of L.
Let us draw a line T parallel to L at a distance equal to
76. The equation of the tangent to the curve y = x2 + 6 at
the radius of the circle C1, on opposite side of L
P(1, 7) is
Then the centre of C ¢ is equivalent from the centre of
2x – y + 5 = 0 …(i) C1 and from line T
Y

P(1, 7)

X¢ X
C(–8, –6)


Here CQ is perpendicular to PQ. Locus of centre of C ¢ is a parabola
The equation of CQ is –x – 2y + k = 0 79. Since S is equidistant from A and line BD, it traces a
parabola.
which is passing through the centre (–8, –6). So,
8 + 12 + k = 0
k = –20
The equation of CQ is –x – 2y – 20 = 0
x + 2y + 20 = 0 …(ii)
On solving Eqs. (i) and (ii), we get
x = –6 and y = –7.
Therefore, the co-ordinates of Q are (–6, –7)
77. Without loss of genrality, we can assume the square
ABCD with its vertices A(1, 1), B(–1, 1), C(–1, 1), Clearly, AC is the axis, A(1, 1) is the focus and
D(1, –1)
Ê 1 1ˆ
Let P be (0, 1) and Q at ( 2, 0) T1 Á , ˜ is the vertex of the parabola.
Ë 2 2¯

CG_03a.indd 79 2/6/2017 4:01:28 PM


3.80 Coordinate Geometry Booster

1 Let its roots are m1, m2.


AT1 = and T2T3 = latus rectum of parabola Therefore, m1m2 = –1
2
fi the tangents are mutually perpendicular.
1
=4¥ =2 2 As we know that the point of intersection of two mutu-
2 ally perpendicular tangents is the director circle.
Thus, the area of the DT1T2T3 So, the equation of the director circle is
1 1 x2 + y2 = 338
= ¥ ¥ 2 2 = 1 Sq unit.
2 2 Therefore, the Statement II is the correct explanation of
Note. No questions asked in 2015 the Statement I.
80. Given AB || CD ◊ CD = 2AB. 82. Given circle is (x + 3)2 + (y – 5)2 = 4
So, the radius = 2
D R C Distance between the parallel lines L1 and L2 is
( p + 3) - ( p - 3) 6
r
Q = < radius (2)
4+9 13
S O
So, the Statement II is false, but the Statement I is true.

A P B Comprehension
Let AB = 2a, CD = a Y Q
and the radius of the circle be r.
Let the circle touches AB at P, BC at Q, AD at R E D
3 3, 3
and CD at S. 2 2
C
Then AR = AP = r, BP = BQ = a – r, 1 3x + y = 6
DR = DS = r and CQ = CS = 2a – r X¢ X
R F P
In DBEC,
BC2 = BE2 + EC2

fi (a – r + 2a – r)2 = (2r)2 + a2 83. Co-ordinates of C are
fi (3a – 2r)2 = (2r)2 + a2
3 3 3
fi 9a2 + 4r2 – 12ar = 4r2 +a2 x- y-
fi 2 = 2 = -1
3 Êpˆ Êpˆ
fi a= r cos Á ˜ sin Á ˜
2 Ë 6¯ Ë 6¯
Also, ar(Quad. ABCD) = 18 3 3 3 3 1
fi ar(Quad. ABED) + ar(DBCE) = 18 fi x- =- ,y- =-
2 2 2 2
1
fi a ◊ 2r + ◊ a ◊ 2r = 18 fi x = 3, y = 1
2
fi 3ar = 18 Thus, C = ( 3, 1)
3 Hence, the equation of the circle is
fi 3 ¥ ¥ r 2 = 18
2 ( x - 3) 2 + ( y - 1) 2 = 1
fi r2 = 4 84. Clearly, the point F is ( 3, 0).
fi r=2
Now the co-ordinates of E are
Thus, the radius is r = 2.
81. The equation od any tangent to the given circle is x— 3 y -1
fi = =1
y = mx + a 1 + m 2 cos (150°) sin (150°)

which is passing through (17, 7). x — 3 y -1


fi = =1
Thus, 7 = 17m + 13 1 + m 2 3 1
-
2 2
fi (7 – 17m)2 = 169(1 + m2)
fi 49 + 289 m2 – 238m = 169(1 + m2) 3 1
fi x— 3=- , y =1-
fi 120m2 – 238m – 120 = 0 2 2

CG_03a.indd 80 2/6/2017 4:01:29 PM


Circle 3.81

3 1 Êpˆ
fi x= 3- ,y= Similarly, OM 2 = 2 cos Á ˜
2 2 Ëk¯
It is given that,
3 1 OM1 + OM2 = 2
fi x= ,y=
2 2
Êpˆ Êpˆ
fi 2 cos Á ˜ + 2 cos Á ˜ = ( 3 + 1)
Ê 3 1ˆ Ë 2k ¯ Ëk¯
Therefore, E = Á , , F = ( 3, 0)
Ë 2 2 ˜¯
Êpˆ Ê p ˆ Ê 3 + 1ˆ
fi cos Á ˜ + cos Á ˜ = Á
Ë 2k ¯ Ë k ¯ Ë 2 ˜¯
85. Co-ordinates of Q are ( 3, 3).
3 Êqˆ Ê 3 + 1ˆ
fi cos Á ˜ + cos (q ) = Á
The equation of QR is y - 0 = ( x - 0) Ë 2¯ Ë 2 ˜¯ Êpˆ
3 where Á ˜ = q
Ë k¯
fi y = 3x
and the equation of RP is y = 0. Êqˆ Êqˆ Ê 3 + 1ˆ
fi cos Á ˜ + 2 cos 2 Á ˜ - 1 = Á
86. Given circle is x2 + y2 – 6x – 4y – 11 = 0 Ë 2¯ Ë 2¯ Ë 2 ˜¯

Êqˆ Ê q ˆ Ê 3 + 3ˆ
B fi 2 cos 2 Á ˜ + cos Á ˜ - Á ˜ =0
Ë 2¯ Ë 2¯ Ë 2 ¯
C P Ê 3 + 3ˆ Êqˆ
fi 2b 2 + b - Á = 0, b = cos Á ˜
Ë 2 ˜¯ Ë 2¯
A fi 4b 2 + 2b - ( 3 + 3) = 0

Therefore, the centre is (3, 2) —2 ± 4 + 16(3 + 3)


fi b=
Since CA and CB are perpendicular to PA and PB. 8
So, CP is the diameter of the circumcircle PAB.
—1 ± 1 + 4(3 + 3)
Thus, the equation of the circumcircle triangle PAB is fi b=
(x – 3)(x – 1) + (y – 2)(y – 8) = 0 2
fi x2 + y2 – 4x – 10y + 19 = 0 —1 ± 1 + 4 3 + 12
87. Clearly, the length of the perpendicular from the centre fi b=
2
of the circle is equal to the radius of the circle.
h◊0 + k ◊0 - 1 —1 ± (1 + 2 3) 2 —1 ± (1 + 2 3)
Thus, =2 fi b= =
h2 + k 2 2 2
1
fi (h 2 + k 2 ) = 3 - 3 -1
4 fi b= ,
Hence, the locus of the (h, k) is 2 2
1
x2 + y 2 = 3
4 fi b=
2
88.
M2
D C Êqˆ 3 Êpˆ
fi cos Á ˜ = = cos Á ˜
Ë 2¯ 2 Ë 6¯
O Êqˆ Êpˆ
fi cos Á ˜ = cos Á ˜
Ë 2¯ Ë 6¯

A B p
M1 fi q=
3
In DOM1A,
p p p
fi =
OA = 2, –AOM1 = k 3
2k
Ê p ˆ OM1 Êpˆ fi k=3
cos Á ˜ = fi OM1 = 2 cos Á ˜
Ë 2k ¯ 2 Ë 2k ¯ fi [k] = 3

CG_03a.indd 81 2/6/2017 4:01:30 PM


3.82 Coordinate Geometry Booster

89. Let L: 2x – 3y = 1 91. (i) Equation of the tangent to the circle x2 + y2 = 4


Y at P( 3, 1) is 3 x + y = 4
i.e PT: 3 x + y = 4
Now, m( PT ) = - 3
1
So, m( L) = .
3
X¢ X The line L is y = m( x - 3) ± a 1 + m 2

1
O

=
3y
x– 1 1
fi y= ( x - 3) ± 1 1 +
:2

3 3
L

1 2
fi y= ( x - 3) ±

3 3
x - 5 x -1
and S: x2 + y2 £ 6 fi y= ,
3 3
If L > 0 and S < 0, the points lie on the smaller part.
fi x - 3 y - 5 = 0, x - 3 y - 1 = 0
Ê 3ˆ Ê 1 1ˆ
Thus, the points Á 2, ˜ and Á , - ˜ lie inside the tri- (ii) Given circles are
Ë 4¯ Ë 4 4¯
angle. x2 + y2 = 4, (x – 3)2 + y2 = 1
90. The equation of the chord bisected at P(h, k) is
M
hx + ky = h2 + k2 …(i)
2
Y
C1(0,0) C2(3,0) P(6, 0)

M

O , k) B
P(h Clearly, the point of intersection is (6, 0).
A The equation of the direct common tangent is
Q a 4a , –4 y – 0 = m(x – 6)
L 5 fi mx – y + 6m = 0
Y¢ Now, C1M = 2
Ê 4a ˆ 6m
Let any point on line LM be Á a , - 4˜ . fi =2
Ë 5 ¯ m2 + 1
The equation of the chord of contact is fi m2 + 1 = 9m2
Ê 4a ˆ fi 8m2 = 1
ax + Á - 4˜ y = 9 … (ii)
Ë 5 ¯ 1
fi m=± .
Comparing Eqs (i) and (ii), we get 2 2
h k h2 + k 2 Hence, the equation of the common tangents are
= = 1
a 4a - 4 9 y=± ( x - 6) and x = 2
5 2 2
92. Given circles are x2 + y2 – 2x – 15 = 0
20h
fi a= and x2 + y2 = 1
4h - 5k
Let the equation of the circle is
h h2 + k 2 x2 + y2 + 2gx + 2fy + c = 0
Therefore, =
20h 9 It passes through (0, 1), so
4h - 5k 1 + 2f + c = 0
4h - 5k h 2 + k 2 Applying condition of orthogonality, we have
fi =
20 9 –2g = c – 15, 0 = c – 1
fi 20(h + k ) = 9(4h – 5)
2 2 fi c = 1, g = 7, f = –1
Hence, the locus of P(h, k) is Thus, r = 49 + 1 — 1 = 7
20(x2 + y2) = 9(4x – 5y) and centre = (–7, 1)

CG_03a.indd 82 2/6/2017 4:01:31 PM


Circle 3.83

93. Thus, the area of the equadrilateral PQRS


Y
R(2, 4) 1
= ¥ (2 + 8) ¥ 3 = 15
P(–1, 1) 2
94. Ans. (a, c)
X¢ X
O
Q(–1, –1)

S(2, –4)

The equation of any tangent to the parabola can be con-
a 2
sidered as y = mx + = mx + .
m m
i.e. m2x – my + 2 = 0
As we know that the length of the perpendicular from
the centre to the tangent to the circle is equal to the
radius of a circle.
2
Thus, = 2
m + m2
4
Ê a
E=Á
ˆ
,a
4
m +m =2 2 Ë tan q ˜¯
m4 + m2 – 2 = 0 cos q + a sin q = 1
(m2 + 2)(m2 – 1) = 0 Êqˆ
m = ±1 a = tan Á ˜
Ë 2¯
Hence, the equation of the tangents are
Hence, the locus is y2 = 1 – 2x.
y = x + 2, y = –x – 2
Ê1 1 ˆ Ê1 1 ˆ
Therefore, the points P, Q are (–1, 1), (–1, –1) and R, S Thus, the required points are Á , ˜ and Á , - ˜ .
are (2, 4) and (2, –4) respectively. Ë 3 3¯ Ë3 3¯

CG_03a.indd 83 2/6/2017 4:01:33 PM


CG_03a.indd 84 2/6/2017 4:01:33 PM
Parabola 4.1

C H A P T E R

4 Parabola

CONCEPT BOOSTER (ii) Parabola


The section of a right circular cone by a plane which is paral-
lel to a generator of a cone is called a parabola.
1. INTRODUCTION
It is believed that the first definition of a conic section is due
to Menaechmus (died 320 BC). His work did not survive and
is only known through secondary accounts. The definition Generator
used at that time differs from the one commonly used today.
It requires the plane cutting the cone to be perpendicular to
one of the lines (a generatrix), that generates the cone as a Parabola
surface of revolution. Thus the shape of the conic is deter-
mined by the angle formed at the vertex of the cone (between
two opposite generatrices). If the angle is acute, the conic is
an ellipse; if the angle is right, the conic is a parabola; and if Plane
the angle is obtuse, the conic is a hyperbola.
Note: The circle cannot be defined in this way and was not (iii) Ellipse
considered as a conic at this time.
The section of a right circular cone by a plane which is nei-
Euclid (300 BC) is said to have written four books on con- ther parallel to a generator of a cone nor parallel or perpen-
ics but these were lost as well. Archimedes (died 212 BC)
dicular to the axis of a cone is called an ellipse.
is known to have studied conics, having determined the area
bounded by a parabola and an ellipse. The only part of this
work to survive is a book on the solids of revolution of conics.

2. BASIC DEFINITIONS
(i) Circle
The section of a right circular cone by a plane which is paral- Ellipse
lel to its base is called a circle.
Plane

(iv) Hyperbola
The section of a double right circular cone by a plane which
is parallel to the axis of a cone is called a hyperbola.
Circle
Plane

CG_04.indd 1 2/6/2017 4:01:58 PM


4.2 Coordinate Geometry Booster

Latus rectum
Any chord passing through the focus and perpendicular to the
axis is known as latus rectum of the conic section.
Centre
The point which bisects every chord of the conic passing
Hyperbola through it, is called the centre of the conic section.

4. RECOGNITION OF CONICS
A general equation of 2nd degree is
ax2 + 2hxy + by2 + 2gx + 2fy + c = 0,
3. CONIC SECTION
a h g
It is the locus of a point which moves in a plane in such a way
that its distance from a fixed point to its perpendicular dis- where D= h b f
tance from a fixed straight line is always constant. The fixed g f c
point is called the focus of the conic and this fixed straight
= abc + 2fgh – af2 – bg2 – ch2
line is called the directrix of the conic and this constant ratio
is known as the eccentricity of the conic. It is denoted as e. a h
and H= = ab - h 2
h b
M P There are two types of conics.
(i) Degenerate conic, and
(ii) Non-degenerate conic.
We use the term degenerate conic sections to describe the
S(Focus) single point, single line and pair of lines and the term non-
Directirix degenerate conic sections to describe those conic sections
that are circles, parabolas, ellipses or hyperbolas.
A non-degenerate conic represents
(i) a circle if, D π 0, h = 0, a = b
(ii) a parabola if D π 0, H = 0
Conic Section with Respect to Eccentricity (iii) an ellipse, if D π 0, H < 0
(iv) a hyperbola, if D π 0, H > 0
(i) If e = 0, the conic section is called a circle (v) a rectangular hyperbola, if D π 0, H > 0 and a + b = 0.
(ii) If e = 1, the conic section is called a parabola. Now, the centre of the conics is obtained by
(iii) If e < 1, the conic section is called an ellipse.
df
(iv) If e > 1, the conic section is called a hyperbola. = 2ax + 2hy + 2g = 0
dx
(v) If e = 2 , the conic section is called a rectangular hy-
perbola. df
and = 2hx + 2by + 2f = 0 .
dy
Some Important Definitions to Remember fi ax + hy + g = 0, hx + by + f = 0
Axis: The straight line passing through the focus and perpen- Solving the above equations, we get the required centre of
dicular to the directrix is called the axis of the conic section. the given conic.
Vertex: The point of intersection of the conic section and the
axis is called the vertex of the conic section. 5. EQUATION OF CONIC SECTION
Double ordinate
Any chord, which is perpendicular to the axis of the conic
M P
section, is called a double ordinate of the conic section.
Focal chord
Any chord passing through the focus is called the focal chord
of the conic section.
S(Focus)
Focal distance Directirix
The distance between the focus and any point on the conic is
known as the focal distance of the conic section.

CG_04.indd 2 2/6/2017 4:01:58 PM


Parabola 4.3

Let the focus be (h, k), directrix be ax + by + c = 0 and the Definition 3


eccentricity is e. In algebra, the parabolas are frequently encountered as
SP
Then the equation of the conic section is =e graphs of quadratic functions, such as y = ax2 + bx + c or
PM
x = ay2 + by + c.
ax + by + c
fi ( x - h) 2 + ( y - k ) 2 = e Definition 4
a 2 + b2 It is a section of a conic, whose eccentricity is 1.
Ê (ax + by + c) 2 ˆ Definition 5
fi ( x - h) 2 + ( y - k ) 2 = e 2 Á ˜
Ë a 2 + b2 ¯ A plane curve formed by the intersection of a right circular
cone and a plane parallel to an element of the cone is called
which is the general equation of the conic section.
parabola.
6. PARABOLA
The term parabola comes from Greek word, para ‘alongside,
nearby, right up to,’ and bola, from the verb ballein means Generator
‘to cast, to throw.’ Understandably, parallel and many of its
derivatives start with the same root. The word parabola may
Parabola
thus mean ‘thrown parallel’ in accordance with the definition.

7. MATHEMATICAL DEFINITIONS
Plane
Definition 1
It is the locus of a point which moves in a plane in such a Definition 6
way that its distance from a fixed point is equal to its distance A conic ax2 + 2hxy + by2 + 2gx + 2fy + c = 0 represents a
from a fixed straight line. The fixed point S is called the focus parabola if
and the fixed straight line OM is called the directrix. (i) D π 0
(ii) h2 – ab = 0, where
Y
P D = abc + 2fgh – af 2 – bg2 – ch2
M
8. STANDARD EQUATION OF A PARABOLA
X Y
O S P
M

X¢ O X
Z S

Definition 2
Let D be a line in the plane and F a fixed point not on D. A Y¢
parabola is the collection of points in the plane that are equi- Let S be the focus and MN be the directrix of the parabola.
distant from F and D. The point F is called the focus and the Draw SZ perpendicular to ZM and let O be the mid-point
line D is called the directrix. of FN.
Y Thus OS = OZ
So O lies on the parabola.
Consider O as the origin and OX and OY as x and y axes,
D respectively.
Let OS = OZ = a.
X Then the co-ordinates of F is (a, 0) and the equation of
O F
ZM is x + a = 0.
Now from the definition of the parabola, we get,
SP = PM
fi SP2 = PM2

CG_04.indd 3 2/6/2017 4:01:59 PM


4.4 Coordinate Geometry Booster

fi (x – a)2 + (y – 0)2 = (x + a)2 (vi) Length of the latus rectum: 4a


fi y2 = 4ax (vii) Extremities of the latus rectum are: L(a, 2a), L¢(a, –2a)
which is the required equation of a parabola. This is also (viii)Equation of the latus rectum is: x = a
known as horizontal parabola or right-ward parabola. (ix) Parametric equations of the parabola:
y2 = 4ax are x = at2 and y = 2at
9. IMPORTANT TERMS RELATED TO PARABOLA (x) Focal distance: x + a
(xi) Any point on the parabola can be considered as
Y (at2, 2at).
M P
Parabola openning leftwards
i.e y2 = –4ax
X¢ X Y
Z O S
L(–a, 2a)
M
P(x, y)

Focus X¢ X
It is the fixed point with reference to which the parabola is S(–a, 0) O
constructed. Here, S is the focus.
Directrix
x–a=0
It is a straight line outside the parabola. Here ZM is the L¢(–a, –2a)
directrix. Y¢
Axis of symmetry (i) Vertex is (0, 0)
It is the line which is perpendicular to the directrix and passes (ii) Focus is (–a, 0)
through the focus. It divides the parabola into two equal halves. (iii) Equation of the directrix is x – a = 0
Vertex (iv) Equation of the axis is y = 0
It is the point on the axis of symmetry that intersects the pa- (v) Equation of the tangent at the vertex is x = 0
rabola when the turn of the parabola is the sharpest. (vi) Length of the latus rectum is 4a
The vertex is halfway between the directrix and the focus. (vii) Extremities of the latus rectum are:
L(–a, 2a), L¢(a, –2a)
Focal chord
(viii) Equation of the latus rectum is x = –a
It is any chord that passes through the focus.
(ix) Parametric equations of the parabola:
Latus rectum y2 = –4ax are x = –at2 and y = 2at
It is that focal chord which is perpendicular to the axis of
(x) Focal distance is x – a
symmetry. The latus rectum is parallel to the directrix. Half
(xi) Any point on the parabola can be considered as
of the latus rectum is called the semi-latus rectum.
(–at2, 2at).
Focal parameter
Parabola opening upwards
The distance from the focus to the directrix is called the focal
i.e. x2 = 4ay
parameter. Y
Focal distance S
The distance between any point on the parabola to the focus (0, a)
is called the focal distance. Here, SP is the focal distance. P
X¢ X
Parametric equation O
From the equation of the parabola, we can write y+a=0 Z M
y 2x
= =t Y¢
2a y (i) Vertex is (0, 0)
Then x = at2, y = 2at, where t is a parameter. (ii) Focus is (0, a)
The equations x = at2 and y = 2at are called the parametric (iii) Equation of the directrix is y + a = 0
equations and the point (at2, 2at) is also referred to as the (iv) Equation of the axis is x = 0
point t. (v) Equation of the tangent at the vertex is y = 0
(i) Vertex is: (0, 0) (vi) Length of the latus rectum 4a
(ii) Focus is: (a, 0) (vii) Extremities of the latus rectum are L(2a, a), L¢(–2a, a)
(iii) Equation of the directrix is: x + a = 0. (viii) Equation of the latus rectum is y = a
(iv) Equation of the axis is: y = 0 (ix) Parametric equations of the parabola x2 = 4ay are
(v) Equation of the tangent at the vertex is: x = 0 x = –2at and y = at2

CG_04.indd 4 2/6/2017 4:01:59 PM


Parabola 4.5

(x) Focal distance is y + a 11. EQUATION OF A PARABOLA WHEN THE VERTEX IS


(xi) Any point on the parabola can be considered as
(h, k) AND AXIS IS PARALLEL TO x-AXIS
(2at, at2).
Parabola Opening Downwards The equation of the parabola y2 = 4ax can be written as
i.e. x2 = –4ay (y – 0)2 = 4a(x – 0)
Y The vertex of the parabola is O(0, 0). Now the origin is
Z M
shifted to V(h, k) without changing the direction of axes, its

O
X equation becomes (y – k)2 = 4a(x – h)
P Y
S

Y¢ y=k
V(h, k)
(i) Vertex is (0, 0)
(ii) Focus is (0, –a) X
O
(iii) Equation of the directrix is y – a = 0
(iv) Equation of the axis is x = 0
Thus its focus is F(a + h, k), latus rectum = 4a and the
(v) Equation of the tangent at the vertex is y = 0
equation of the directrix is
(vi) Length of the latus rectum is 4a x = h – a, i.e. x + a – h = 0
(vii) Extremities of the latus rectum are: The parametric equation of the curve (y – k)2 = 4a(x – h)
L(2a, –a), L¢(–2a, –a) are x – h + at2 and y = k + 2at.
(viii) Equation of the latus rectum is y = –a
(ix) parametric equations of the parabola x2 = –4ay are 12. EQUATION OF A PARABOLA WHEN THE VERTEX IS
x = 2at and y = –at2 (h, k) AND AXIS IS PARALLEL TO y-AXIS
(x) Any point on the parabola can be considered as
Y
(2at, –at2).
(xi) Focal distance is y – a
y=k
10. GENERAL EQUATION OF A PARABOLA V(h, k)
X¢ X
O
Y P(x, y) x=h

The equation of a parabola with the vertex V(h, k) is
M S(h, k) (x – h)2 = 4a(y – k)
Thus, its focus is F(h, a + k), latus rectum = 4a and the
Z
X¢ X equation of the directrix is
O
y = k – a, i.e. y + a – k = 0
The parametric equation of the curve (x – h)2 = 4a(y – k)
are x = h + 2at and y = k + at2.

Note: The equation of a parabola, whose axis is parallel to
Let S(h, k) be the focus and lx + my + n = 0 is the equation of y-axis can also be considered as y = ax2 + bx + c.
the directrix and P(x, y) be any point on the parabola.
Then, Polar form of a Parabola
SP = PM In polar coordinates, the equation of a parabola with param-
lx + my + n eters r and q and the centre (0, 0) is given by
fi ( x - h) 2 + ( y - k ) 2 =
(l 2 + m 2 ) 2a
r=-
1 + cos q
(lx + my + n) 2
fi ( x - h) 2 + ( y - k ) 2 =
(l 2 + m 2 ) 13. FOCAL CHORD
fi m x + l y – 2lmxy + (term)x
2 2 2 2
Any line passing through the focus and intersects the parabola
+ (term)y + (constant term) = 0 in two distinct points, it is known as focal chord of the parabola.
fi (mx – ly) + 2gx + 2fy + c = 0
2
Any point on the parabola y2 = 4ax can be considered as
2
which is the general equation of a parabola. (at , 2at).

CG_04.indd 5 2/6/2017 4:02:00 PM


4.6 Coordinate Geometry Booster

14. POSITION OF A POINT RELATIVE TO A PARABOLA Equation of the Tangent to a Parabola in Different
Forms
Consider the parabola y2 = 4ax and the point be (x1, y1).
Y P(x1, y1)
(i) Point form
The equation of the tangent to the parabola y2 = 4ax at
L (x1, y1) is
yy1 = 2a(x + x1)
X
O M Now the given equation is
y2 = 4ax.
Differentiating with respect to x, we get
The point (x1, y1) lies outside, on and inside of the pa- dy
2y = 4a
rabola y2 = 4axy2 = 4ax according as dx
y12 – 4ax1 > 0, = 0, < 0 dy 2a
fi =
dx y
15. INTERSECTION OF A LINE AND A PARABOLA dy
Ê ˆ 2a
Now m = ÁË ˜¯ = .
Let the parabola be y2 = 4ax dx ( x1, y1 ) y1
M
and the line be y = mx + c. T R Thus the equation of tangent is
Eliminating x between fi yy1 – y12 = 2ax – 2ax1
these two equations, we get fi yy1 = 2ax – 2ax1 + 4ax1
Ê y - cˆ = 2a (x + x1)
y 2 = 4a Á
Ë m ˜¯
P
which is the required equation of the tangent to the pa-
fi my2 – 4ay + 4ac = 0 rabola y2 = 4ax at (x1, y1).
N (ii) Parametric form
The given line will cut the Q
The equation of the tangent to the parabola y2 = 4ax at
parabola in two distinct, co-
(at2, 2at) is
incident and imaginary points according as
yt = x + at2
D > 0, = 0, < 0
fi 16a2 – 16amc > 0, = 0, < 0 The equation of the tangent to the parabola y2 = 4ax at
fi a > cm, a = cm, a < cm (x1, y1) is
yy1 = 2a(x + x1)
Condition of tangency: The line y = mx + c will be a tangent
fi y ◊ 2at = 2a(x + at2)
a
to the parabola y2 = 4ax, if c = . fi yt = x + at2
m
(iii) Slope form
The equation of any tangent to the parabola can be consid-
The equation of the tangent to the parabola y2 = 4ax at
a
ered as y = mx + . Ê a 2a ˆ
m ÁË 2 , ˜¯ is
m m
The co-ordinates of the point of contact in terms of m is
Ê a 2a ˆ a
y = mx + , where m = slope
ÁË 2 , ˜¯ m
m m
The equation of the tangent to the parabola y2 = 4ax at
16. TANGENT (x1, y1) is
yy1 = 2a(x + x1) …(i)
If a line intersects the parabola in two coincident points, it is
known as the tangent to a parabola. 2a 2a
Here, m = fi y1 =
y1 m
Q Since the point (x1, y1) lies on the parabola y2 = 4ax, we
have,
Q1
y12 = 4ax1.
Q2
4a 2
fi 4ax1 =
Q3 m2
Q4 a
T fi x1 =
P
m2

CG_04.indd 6 2/6/2017 4:02:00 PM


Parabola 4.7

Putting the values of x1 and y1 in Eq. (i), we get 17. NORMAL


Ê 2a ˆ Ê a ˆ It is a line which is perpendicular to the point of contact to
y ◊ Á ˜ = 2a Á x + 2 ˜
Ë m¯ Ë m ¯ the tangent.
a
fi y = mx + N
m
(iv) Condition of tangency
The line y = mx + c will be a tangent to the parabola
a P
y2 = 4ax is c = .
m
Note: Any tangent to the parabola can be considered T
a Here PT is a tangent and PN is a normal.
as y = mx + .
m
Equation of Normals to the Parabola in Different
(v) Director circle Forms
The locus of the point of intersection of two perpen-
(i) Point form
dicular tangents to a parabola is known as the director
The equation of the normal to the parabola y2 = 4ax at
circle.
(x1, y1) is
(vi) The equation of the pair of tangents can be drawn to the y - y1 x - x1
parabola from the point (x1, y1). =
y1 2a
M(h, k) The equation of the tangent to the parabola y2 = 4ax at
(x1, y1) is
yy1 = 2a(x + x1) …(i)
2a
Slope of the tangent is m(T ) =
P(x1, y1) y1
y1
N Slope of the normal is m( N ) = -
2a
Thus the equation of the normal is
Let (h, k) be any point on either of the tangents drawn y1
from (x1, y1). y - y1 = - (x - x1)
2a
The equation of the line joining (x1, y1) and (h, k) is
(ii) Parametric form
k - y1
y - y1 = ( x - x1 ) The equation of the normal to the parabola
h - x1 y2 = 4ax at (at2, 2at) is
k - y1 hy - kx1 y = –tx + 2at + at3
fi y= x+ 1
h - x1 h - x1 As we know that the equation of the normal to the pa-
a rabola y2 = 4ax at (x1, y1) is
If this be a tangent, it must be of the form y = mx + .
m y - y1 x - x1
k - y1 a hy1 - kx1 =-
Thus, m = and = y1 2a
h - x1 m h - x1
Replacing x1 by at2 and y1 by 2at, we get
Therefore, by multiplication we get
y - 2at x - at 2
Ê k - y1 ˆ Ê hy1 - kx1 ˆ =-
a=Á 2at 2a
Ë h - x1 ˜¯ ÁË h - x1 ˜¯ fi y = –tx + 2at + at3
fi a(h – x1)2 = (k – y1)(hy1 – kx1) which is the required equation of the normal to the giv-
Hence, the locus of the point (h, k) is en parabola.
fi a(x – x1)2 = (y – y1)(xy1 – yx1) (iii) Slope form
fi (y2 – 4ax)(y12 – 4ax1) = {yy1 – 2a(x + x1)}2. The equation of the normal to the parabola
y2 = 4ax at (ax2, –2am) is
fi SS1 = T2
y = mx – 2am – am3
where, S: (y2 – 4ax), S1:(y12 – 4ax1)
As we know that the equation of the normal to the pa-
and T:{yy1 – 2a(x + x1)}. rabola

CG_04.indd 7 2/6/2017 4:02:00 PM


4.8 Coordinate Geometry Booster

y2 = 4ax at (at2, 2at) is y2 = 4ax is


y = –tx + 2at + at3 …(i) yy1 = 2a(x + x1)
The slope of the normal is
m = –t Q
fi t = –m
Putting the values of m in Eq. (i), we get
y = mx – 2am – am3 P Chord of contact
which is the required equation of the normal to the pa-
rabola y2 = 4ax at (am2, –2am).
R
(iv) Condition of normal
The line y = mx + c will be a normal to the parabola
y2 = 4ax, if c = –2am – am3 and the co-ordinates of the 19. CHORD BISECTED AT A GIVEN POINT
point of contact are (am2, –2am).
(v) Co-normal points Y
A
In general, three normals can be drawn from a point to
a parabola and their feet (points) where they meet the
parabola are called the co-normal points.
Y X
O M

A B

P(h, k)
The equation of the chord of the parabola
O X y2 = 4ax is bisected at the point (x1, y1) is
C T = S1
fi yy1 – 2a(x + x1) = y12 – 4ax1
B where
T: y1 – 2a(x + x1), S:y12 – 4ax1
Here A, B and C are three co-normal points.
Let P(h, k) be any given point y2 = 4ax be a parabola. 20. DIAMETER
The equation of any normal to the parabola The locus of the mid-points of a system of parallel chords to a
y2 = 4ax is parabola is known as the diameter of the parabola.
y = mx – 2am – am3
Y
which passes through P(h, k). Then
k = mh – 2am – am3
fi am3 + (2a – h)m + k = 0
which is a cubic equation in m. So it has three roots. M(h, k) y = 2a/m
Thus, in total, three normals can be drawn from a point X
lies either outside or inside of a parabola. O

Notes:
1. We can draw one and only one normal to a pa-
rabola, if a point lies on the parabola.
2. From an external point to a parabola, only one
normal can be drawn. The equation of the diameter to the parabola y2 = 4ax
bisecting a system of parallel chords with slope m is

18. CHORD OF CONTACT 2a


y=
m
The chord joining the points of contact of two tangents drawn
Let (h, k) be the mid-point of the chord y = mx + c of the
from an external point to a parabola is known as the chord of
parabola y2 – 2ah.
contact.
The equation of the chord of contact of tangents drawn Then, T = S1
from a point (x1, y1) to the parabola fi ky – 2a(x + h) = k2 – 2ah

CG_04.indd 8 2/6/2017 4:02:01 PM


Parabola 4.9

2a 21. REFLECTION PROPERTY OF A PARABOLA


Now slope =
k All rays of light coming from the positive direction of x-axis
2a 2a and parallel to the axis of the parabola are reflected through
m= fi k= the focus of the parabola
k m
2a Y
Hence the locus of the mid-point (h, k) is y = .
m P
M
Note: Any line which is parallel to the axis of the parabola
drawn through any point on the parabola is called the di- X
O S
ameter of the parabola and its equation is the y-coordinate
of that point. Z

E XERCISES

LEVEL I 12. Find the equation of a parabola whose vertex is (1, 2)


and the axis is parallel to x-axis and also passes through
(Problems based on the point (3, 4).
Fundamentals) 13. If the axis of a parabola is parallel to y-axis, the ver-
tex and the length of the latus rectum are (3, 2) and 12
1. What conic does ax + by = 1 represent? respectively, find its equation.
2. If the conic x2 – 4xy + ly2 + 2x + 4y + 10 = 0 represents
a parabola, find the value of l. PROPERTIES OF THE FOCAL CHORD

3. If the conic 16( x 2 + ( y - 1) 2 ) = ( x + 3 y - 5) 2 repre- 14. If the chord joining P(at12, 2at1) and is the focal chord,
sents a non-degenerate conic, write its name and also prove that t1t2 = –1.
find its eccentricity. 15. If the point (at2, 2at) be the extremity of a focal chord
4. If the focus and the directrix of a conic be (1, 2) and of the parabola y2 = 4ax, prove that the length of the
2
x + 3y + 10 = 0 respectively and the eccentricity be Ê 1ˆ
focal chord is a Á t + ˜ .
1 Ë t¯
, then find its equation.
2 16. If the length of the focal chord makes an angle q with
5. Find the equation of a parabola, whose focus is (1, 1) the positive direction of x-axis, prove that its length is
and the directrix is x – y + 3 = 0. 4a cosec2q.
17. Prove that the semi-latus rectum of a parabola y2 = 4ax
ABC OF PARABOLA is the harmonic mean between the segments of any
focal chord of the parabola.
6. Find the vertex, the focus, the latus rectum, the direc- 18. Prove that the length of a focal chord of the parabola
trix and the axis of the parabolas varies inversely as the square of its distance from the
(i) y2 = x + 2y + 2 vertex.
(ii) y2 = 3x + 4y + 2 19. Prove that the circle described on the focal chord as the
(iii) x2 = y + 4x + 2 diameter touches the tangent to the parabola.
(iv) x2 + x + y = 0 20. Prove that the circle described on the focal chord as the
7. If the focal distance on a point to a parabola y2 = 12x is diameter touches the directrix of the parabola.
6, find the co-ordinates of that point.
8. Find the equation of a parabola, whose focus (–6, –6) POSITION OF A POINT RELATIVE TO A PARABOLA
and the vertex is (–2, –2). 21. If a point (l, –l) lies in an interior point of the parabola
9. The parametric equation of a parabola is x = t 2 + 1 and y2 = 4x, find the range of l.
y = 2t + 1. Find its directrix. 22. If a point (l, 2) is an exterior point of both the parabo-
10. If the vertex of a parabola be (–3, 0) and the directrix las y2 = (x + 1) and y2 = –x + 1, find the value of l.
is x + 5 = 0, find its equation.
11 Find the equation of the parabola whose axis is parallel INTERSECTION OF A LINE AND A PARABOLA
to y-axis and which passes through the points (0, 2),
23. If 2x + 3y + 5 = 0 is a tangent to the parabola y2 = 8x,
(–1, 0) and (1, 6).
find the co-ordinates of the point of contact.

CG_04.indd 9 2/6/2017 4:02:01 PM


4.10 Coordinate Geometry Booster

24. If 3x + 4y + l = 0 is a tangent to the parabola y2 = 12x, 47. Prove that the orthocentre of any triangle formed by
find the value of l. three tangents to a parabola lies on the directrix.
25. Find the length of the chord intercepted by the parabola 48. Prove that the equation of the director circle to the
y2 = 4ax and the line y = mx + c. parabola y2 = 4ax is x + a = 0.
49. Find the equation of the director circle to the following
TANGENT AND TANGENCY parabolas:
26. Find the point of intersection of tangents at P(t1) and (i) y2 = x + 2 (ii) x2 = 4x + 4y
2
Q(t2) on the parabola y2 = 4ax. (iii) y = 4x + 4y – 8
27. Find the equation of tangent to the parabola y2 = 2x +
NORMAL AND NORMALCY
5y – 8 at x = 1.
28. Find the equation of the tangent to the parabola y2 = 8x 50. Find the point of intersection of normals at P(t1) and
having slope 2 and also find its point of contact. Q(t2) on the parabola y2 = 4ax.
29. Two tangents are drawn from a point (–1, 2) to a pa- 51 Find the relation between t1 and t2, where the normal at
rabola y2 = 4x. Find the angle between the tangents. t1 to the parabola y2 = 4ax meets the parabola y2 = 4ax
30. Find the equation of the tangents to the parabola again at t2.
y = x2 – 3x + 2 from the point (1, –1). 52. If the normal at t1 meets the parabola again at t2, prove
31. Find the equation of the common tangent to the that the minimum value of t22 is 8.
parabolas y2 = 4ax and x2 = 4ay. 53. If two normals at t1 and t2 meet again the parabola y2 =
32. Find the equation of the common tangent to the 4ax at t3, prove that t1t2 = 2.
parabola y2 = 4ax and x2 = 4by. 54. Find the equation of the normal to the parabola y2 = 4x
33. Find the equation of the common tangent to the at the point (1, 2).
parabola y2 = 16x and the circle x2 + y2 = 8. 55. Find the equation of the normal to the parabola y2 = 8x
34. Find the equation of the common tangents to the at m = 2.
parabolas y = x2 and y = –(x – 2)2. 56. If x + y = k is a normal to the parabola y2 = 12x, find the
35. Find the equation of the common tangents to the curves value of k.
y2 = 8x and xy = –1. 57. If the normal at P(18, 12) to the parabola y2 = 8x cuts it
36. Find the equation of the common tangent to the circle again at Q, prove that 9PQ = 80 10 .
x2 + y2 – 6y + 4 = 0 and the parabola y2 = x. 58. Find the locus of the point of intersection of two nor-
37. Find the equation of the common tangent touching the mals to the parabola y2 = 4ax, which are at right angles
circle x2 + (y – 3)2 = 9 and the parabola y2 = 4x above to one another.
the x-axis. 59. If lx + my + n = 0 is a normal to the parabola y2 = 4ax,
38. Find the points of intersection of the tangents at the prove that al3 + 2alm2 + m2n = 0.
ends of the latus rectum to the parabola y2 = 4x. 60. If a normal chord subtends a right angle at the vertex
39. Find the angle between the tangents drawn from a point of the parabola y2 = 4ax, prove that it is inclined at an
(1, 4) to the parabola y2 = 4x. angle of tan -1 ( 2) to the axis of the parabola.
40. Find the shortest distance between the line y = x – 2 and 61. At what point on the parabola y2 = 4x, the normal
the parabola y = x2 + 3x + 2. makes equal angles with the axes?
41. Find the shortest distance from the line x + y = 4 and the 62. Find the length of the normal chord which subtends an
parabola y2 + 4x + 4y = 0. angle of 90° at the vertex of the parabola y2 = 4x.
42. If y + b = m1(x + a) and y + b = m2(x + a) are two tan- 63. Prove that the normal chord of a parabola y2 = 4ax at
gents of the parabola y2 = 4ax, find the value of m1m2. the point (p, p) subtends a right angle at the focus.
43. The tangent to the curve y = x2 + 6 at a point (1, 7) 64. Show that the locus of the mid-point of the portion of
touches the circle x2 + y2 + 16x + 12y + c = 0 at Q. Find the normal to the parabola y2 = 4ax intercepted between
the co-ordinates of Q. the curve and the axis is another parabola.
65. Find the shortest distance between the curves y2 = 4x
44. Two straight lines are perpendicular to each other. One
and x2 + y2 – 12x + 31 = 0.
of them touches the parabola y2 = 4a(x + a) and the
66. Find the shortest distance between the curves x2 + y2 +
other touches y2 = 4b(x + b). Prove that the point of 12y + 35 = 0 and y2 = 8x.
intersection of the lines lie on the line x + a + b = 0.
45. Prove that the area of the triangle formed by three CO-NORMAL POINT
points on a parabola is twice the area of the triangle
67. Prove that the algebraic sum of the three concurrent
formed by the tangents at these points.
normals to a parabola is zero.
46. Prove that the circle circumscribing the triangle formed
68. Prove that the algebraic sum of the ordinates of the feet
by any three tangents to a parabola passes through the of three normals drawn to a parabola from a given point
focus. is also zero.

CG_04.indd 10 2/6/2017 4:02:01 PM


Parabola 4.11

69. Prove that the centroid of the triangle formed by the 85. Prove that the locus of the mid-points of the focal chord
feet of the three normals lies on the axis of the parabo- of the parabola is another parabola.
la. Also find the centroid of the triangle. 86. Prove that the locus of the mid-points of the chord of a
70. If three normals drawn from a given point (h, k) to any parabola passes through the vertex is a parabola.
parabola be real, prove that h > 2a. 87. Prove that the locus of the mid-points of a normal
71. If three normals from a given point (h, k) to any pa- chords of the parabola y2 = 4ax is y4 – 2a(x – 2a)y2 +
rabola y2 = 4ax be real and distinct, prove that 27ak2 < 8a4 = 0
4(h – 2a)3. 88. Prove that the locus of the mid-point of a chord of a
72. If a normal to a parabola y2 = 4ax makes an angle q parabola y2 = 4ax which subtends a right angle at the
with the axis of the parabola, prove that it will cut the vertex is y2 = 2a(x – 4a).
-1 Ê tan q ˆ
89. Prove that the locus of the mid-points of chords of the
curve again at an angle of tan Á
Ë 2 ˜¯ parabola y2 = 4ax which touches the parabola y2 = 4bx
.
is y2(2a – b) = 4a2x.
73. Prove that the normal chord to a parabola y2 = 4ax at 90. Find the locus of the mid-point of the chord of the pa-
the point whose ordinate is equal to its abscissa, which rabola y2 = 4ax, which passes through the point (3b, b).
subtends a right angle at the focus of the parabola. 91. Prove that the locus of the mid-points of all tangents
74. Prove that the normals at the end-points of the latus drawn from points on the directrix to the parabola
rectum of a parabola y2 = 4ax intersect at right angle on y2 = 4ax is y2(2x + a) = a(3x + a)2.
the axis of the parabola and their point of intersection
is (3a, 0). DIAMETER OF A PARABOLA
75. If S be the focus of the parabola and the tangent and the
92. Prove that the tangent at the extremity of a diameter of
normal at any point P meet the axes in T and G respec-
a parabola is parallel to the system of chords it bisects.
tively, prove that ST = SG = SP.
93. Prove that tangents at the end of any chord meet on the
76. From any point P on the parabola y2 = 4ax, a perpen-
diameter which bisects the chords.
dicular PN is drawn on the axis meeting at N, the nor-
mal at P meets the axis in G. Prove that the sub-normal REFLECTION PROPERTY OF A PARABOLA
NG is equal to its semi-latus rectum.
77. The normal to the parabola y2 = 4ax at a point P on it, 94. A ray of light moving parallel to the x-axis gets reflect-
meets the x-axis in G, prove that P and G are equidis- ed from a parabolic mirror whose equation is (y – 4)2
tant from the focus S of the parabola. = 8(x + 1). After reflection, the ray passes through the
78. The normal at P to the parabola y2 = 4ax meets its axis point (a, b), find the value of a + b + 10.
at G. Q is another point on the parabola such that QG 95. A ray of light is moving along the line y = x +2, gets
reflected from a parabolic mirror whose equation is
is perpendicular to the axis of the parabola. Prove that
y2 = 4(x + 2). After reflection, the ray does not pass
QG2 – PG2 = constant.
through the focus of the parabola. Find the equation of
CHORD OF CONTACT
the line which containing the reflected ray.

79. Find the equation of the chord of contact to the tangents


from the point (2, 3) to the parabola y2 = 4x.
80. Find the chord of contact of the tangents to the parabola
LEVEL II
(Mixed Problems)
y2 = 12x drawn through the point (–1, 2).
81. Prove that the locus of the point of intersection of two 1. Three normals to the parabola y2 = x are drawn through
tangents to a parabola y2 = 4ax which make a given a point (c, 0), then
angle q with one another is y2 – 4ax = (x + a)2 tan2q. (a) c = 1/4 (b) c = 1/2 (c) c > 1/2 (d) none
82. Prove that the length of the chord of contact of tan- 2. The line which is parallel to x-axis and crosses the
gents drawn from (h, k) to the parabola y2 = 4ax is curve y = x at an angle of 45° is
1 2
|(k + 4a 2 )(k 2 - 4ah)|1/2 . (a) x = 1/4 (b) y = 1/4 (c) y = 1/2 (d) y = 1
a 3. Consider a circle with its centre lying on the focus of
83. Prove that the area of the triangle formed by the tan- the parabola y2 = 2px such that it touches the directrix
gents from the point (h, k) to the parabola y2 = 4ax and of the parabola, the point of intersection of the circle
(k 2 - 4ah)3/2 and the parabola is
a chord of contact is .
2a Êp ˆ Êp ˆ
(a) Á , p˜ (b) Á , - p˜
Ë2 ¯ Ë2 ¯
CHORD BISECTED AT A POINT
84. Find the equation of the chord of the parabola y2 = 8x Ê p ˆ Ê p ˆ
(c) Á - , p˜ (d) Á - , - p˜
which is bisected at (2, 3). Ë 2 ¯ Ë 2 ¯

CG_04.indd 11 2/6/2017 4:02:02 PM


4.12 Coordinate Geometry Booster

4. If the line x – 1 = 0 is the directrix of the parabola 15. If a π 0 and the line 2bx + 3cy + 4d = 0 passes through
y2 – kx + 8 = 0, one of the value of k is the point of intersection of the parabolas y2 = 4ax and
(a) 1/8 (b) 8 (c) 4 (d) 1/4 x2 = 4ay, then
5. If x + y = k is a normal to the parabola y2 = 12x, the (a) d2 + (2b – 3c)2 = 0 (b) d2 + (2b + 3c)2 = 0
value of k is 2
(c) d + (3b + 2c) = 0 2
(d) d2 + (3b – 2c)2 = 0
(a) 9 (b) 3 (c) –9 (d) –3 16. The locus of the vertices of the family of parabolas
6. The equation of the directrix of the parabola y2 + 4y +
a2 x2 a2 x
4x + 2 = 0 is y= + - 2a is
(a) x = –1 (b) x = 1 2 2
(c) x = –3/2 (d) x = 3/2 105 3
(a) xy = (b) xy =
7. The equation of the common tangent touching the 64 4
circle (x – 3)2 + y2 = 9 and the parabola y2 = 4x above 35 64
the x-axis is (c) xy = (d) xy =
16 105
(a) 3 y = 3x + 1 (b) 3 y = - ( x + 3)
17. The angle between the tangents to the curve y = x2 – 5x
(c) 3 y = ( x + 3) (d) 3 y = - (3x + 1) + 6 at the point (2, 0) and (3, 0) is
8. The locus of the mid-point of the line segment joining p p p p
the focus to a moving point on the parabola y2 = 4ax is (a) (b) (c) (d)
2 3 6 4
another parabola with directrix
18. The equation of a tangent to the parabola y2 = 8x is
(a) x + a = 0 (b) x = –a/2
y = x + 2. The point on this from which the other tangent
(c) x = 0 (d) x = a/2 to the parabola is perpendicular to the given tangent is
9. The focal chord of y2 = 16x is a tangent to (x – 6)2 + y2 (a) (–2, 0) (b) (–1, 1) (c) (0, 2) (d) (2, 4)
= 2, the possible values of the slope of this chord are 19. A parabola has the origin as its focus and the line x = 2
(a) 1, –1 (b) –1/2, 2 as its directrix. The vertex of the parabola is at
(c) –2, 1/2 (d) 1/2, 2 (a) (0, 1) (b) (2, 0)
10. The tangent to the parabola y = x2 + 6 touches the circle (c) (0, 2) (d) (1, 0)
x2 + y2 + 16x + 12y + c = 0 at the point 20. The length of the chord of the parabola y2 = x which is
bisected at the point (2, 1) is
(a) (–6, –9) (b) (–13, –9)
(c) (–6, –7) (d) (13, 7) (a) 2 3 (b) 4 3 (c) 3 2 (d) 2 5
11. The axis of a parabola is along the line y = x and the 21. Two mutually perpendicular tangents to the parabola
y2 = 4ax meet the axis in P1 and P2. If S be the focus of
distance of its vertex from the origin is 2 and that
1 1
from its focus is 2 2 . If the vertex and the focus both the parabola, + is
l ( SP1 ) l ( SP2 )
lie in the first quadrant, then the equation of the pa-
rabola is 4 2 1 1
(a) (b) (c) (d)
(a) (x + y)2 = (x + y – 2) (b) (x – y)2 = (x + y – 2) a a a 4a
(c) (x – y)2 = 4(x + y – 2) (d) (x – y)2 = 8(x + y – 2) 22. Which of the following equations represents a para-
12. The equations of the common tangents to the parabola bolic profile, represented parametrically, is
y = x2 and y = –(x – 2)2 is (a) x = 3 cos t, y = 4 sin t
(a) y = 4(x – 1) (b) y = 0 Êtˆ
(c) y = –4(x – 1) (d) y = –10(3x + 5) (b) x 2 - 2 = - 2 cos t , y = 4 cos 2 Á ˜
Ë 2¯
13. The tangent PT and the normal PN to the parabola (c) x = tan t , y = sec t
y2 = 4ax at a point P on it meet its axis at points T and
Êtˆ Êtˆ
N, respectively. The locus of the centroid of the triangle (d) x = 1 - sin t , y = sin Á ˜ + cos Á ˜
Ë 2¯ Ë 2¯
PTN is a parabola whose
2a
(a) vertex is ÊÁ , 0ˆ˜
23. The points of contact Q and R of tangent from the point
(b) directrix is x = 0 P(2, 3) on the parabola y2 = 4x are
Ë 3 ¯
(a) (9, 6), (1, 2) (b) (1, 2), (4, 4)
(c) latus rectum is 2a (d) focus is (a, 0)
14. The normal at the point (bt12, 2bt1) on a parabola meets Ê1 ˆ
(c) (4, 4), (9, 6) (d) (9, 6), Á , 1˜
the parabola again at (bt22, 2bt2), then Ë4 ¯
2 2 24. A tangent is drawn to the parabola y2 = 4x at the point
(a) t2 = - t1 + (b) t2 = t1 - P whose abscissa lies in [1, 4]. The maximum possible
t1 t1
2 area of the triangle formed by the tangents at P ordinate
2
(c) t2 = t1 + (d) t2 = - t1 - of the point P and the x-axis is
t1 t1 (a) 8 (b) 16 (c) 24 (d) 32

CG_04.indd 12 2/6/2017 4:02:02 PM


Parabola 4.13

25. The length of the normal chord y2 = 4x which makes an and OQ as diameters intersect in R. If q1, q2 and j are
p the angles made with the axis by the tangents at P and
angle of with the axis of x is Q on the parabola and by OR, the value of cot (q1) + cot
4
(a) 8 (b) 8 2 (c) 4 (d) 4 2 (q2) is
26. The co-ordinates of the end points of a focal chord (a) –2 tan (j) (b) –2 tan (p – j)
of a parabola y2 = 4ax are (x1, y1) and (x2, y2), then (c) 0 (d) –2 cot (j)
(x1x2 + y1y2) is 36. The tangent at P to a parabola y2 = 4ax meets the
(a) 2a2 (b) –3a2 (c) –a2 (d) 4a2 directrix at U and the base of the latus rectum at V, then
2 SUV (where S is the focus) must be a/an
27. If the normal to a parabola y = 4ax at P meets the curve
again at Q and if PQ and the normal at Q makes angles (a) right D (b) equilateral D
a and b, respectively with the x-axis, then tan a + tan b (c) isosceles D (d) right isosceles D
is 37. Two parabolas y2 = 4a(x – m1) and x2 = 4a(y – m2)
(a) 0 (b) –2 (c) –1/2 (d) –1 always touch one another, the quantities m1 and m2 are
28. If the normal to the parabola y2 = 4ax at the point with both variables. The locus of their points of contact has
parameter t1 cuts the parabola again at the point with the equation
parameter t2, then (a) xy = a2 (b) xy = 2a2
(a) 2 £ t22 £ 8 (b) 2 £ t22 £ 4 (c) xy = 4a 2
(d) none
(c) t2 ≥ 4
2
(d) t22 ≥ 8 38. If a normal to a parabola y2 = 4ax makes an angle
2
29. A parabola y = ax + bx + c crosses the x-axis at (a, 0)
j with its axis, it will cut the curve again at an angle
and (b, 0) both to the right of the origin.
—1 Ê 1 ˆ
A circle also passes through these two points. The (a) tan–1(2 tan j) (b) tan Á tan j ˜
Ë2 ¯
length of a tangent from the origin to the circle is
—1 Ê 1 ˆ
bc b c (c) cot Á tan j ˜ (d) none
(a) (b) ac2 (c) (d) Ë2 ¯
a a a
39. The vertex of a parabola is (2, 2) and the co-ordinates
30. Two parabolas have the same focus. If their directrices of its extremities of the latus rectum are (–2, 0) and
are the x-axis and the y-axis respectively, the slope of (6, 0). The equation of the parabola is
their common chord is
(a) y2 – 4y + 8x – 12 = 0 (b) x2 + 4x + 8y – 12 = 0
(a) 1, –1 (b) 4/3 (c) 3/4 (d) none
(c) x2 – 4x + 8y – 12 = 0 (d) x2 + 4x – 8y + 20 = 0
31. The straight line joining the point P on the parabola
y2 = 4ax to the vertex and the perpendicular from the 40. The length of the chord of the parabola y2 = x which is
bisected at the point (2, 1) is
focus to the tangent at P intersect at R. Then the locus
of R is (a) 2 3 (b) 4 3 (c) 3 2 (d) 2 5
(a) x2 + 2y2 – ax = 0 (b) x2 + y2 – 2ax = 0 41. If the tangent and the normal at the extremities of a
2 2
(c) 2x + 2y – ax = 0 (d) 2x2 + y2 – 2ay = 0 focal chord of a parabola intersect at (x1, y1) and (x2, y2)
32. A normal chord of the parabola y2 = 4x subtending a respectively, then
right angle at the vertex makes an acute angle q with (a) x1 = x2 (b) x1 = x2
the x-axis, then q is (c) y1 = y2 (d) x2 = y1
(a) tan–12 (b) sec -1 ( 3) 42. If the chord of contact of tangents from a point P to the
-1
(c) cot ( 3) (d) none parabola y2 = 4ax touches the parabola xy = 4by, then
33. C is the centre of the circle with centre (0, 1) and the the locus of P is a/an
radius unity of the parabola y = ax2. The set of values of (a) circle (b) parabola
a for which they meet at a point other than the origin is (c) ellipse (d) hyperbola
1 43. The latus rectum of a parabola whose focal chord PSQ
(a) a > 0 (b) 0 < a < is such that SP = 3 and SP = 2 is given by
2
1 1 1 (a) 24/5 (b) 12/5 (c) 6/5 (d) none
(c) <a< (d) a > 44. If two normals to a parabola y2 = 4ax intersect at right
4 2 2
angles, then the chord joining then feet passes through
34. TP and TQ are two tangents to the parabola y2 = 4ax at
a fixed point whose co-ordinates are
P and Q. If the chord PQ passes through the fixed point
(a) (–2a, 0) (b) (a, 0) (c) (2a, 0) (d) None
(–a, b), the locus of T is
45. The straight line passing through (3, 0) and cutting the
(a) ay = 2b(x – b) (b) by = 2a(x – a)
(c) by = 2a(x – a) (d) ax = 2a(y – b) curve y = x orthogonally is
35. Through the vertex O of the parabola y2 = 4ax two (a) 4x + y = 18 (b) x + y = 9
chords OP and OQ are drawn and the circles on OP (c) 4x – y = 6 (d) None

CG_04.indd 13 2/6/2017 4:02:03 PM


4.14 Coordinate Geometry Booster

46. PQ is a normal chord of the parabola y2 = 4ax at P. A 8. If the focal chord of y2 = 16x is a tangent to (x – 6)2 + y2
being the vertex of the parabola. Through P a line is = 2, find the possible values of the slope of the chord.
drawn parallel to AQ meeting the x-axis in R. Then the 9. Find the points of intersection of the tangents at the end
length of AR is of the latus rectum of the parabola y2 = 4x.
(a) the length of latus rectum. 10. If the tangent at the point P(2, 4) to the parabola
(b) the focal distance of the point P. y2 = 8x meets the parabola y2 = 8x + 5 at Q and R, find
(c) 2 ¥ Focal distance of the point P. the mid-point of QR.
(d) the distance of P from the directrix. 11. If y + b = m1(x + a) and y + b = m2(x + a) are two tan-
47. The locus of the point of intersection of the perpen- gents to the parabola y2 = 4ax, prove that m1m2 = –1.
dicular tangents of the curve y2 + 4y – 6x – 2 = 0 is 12. Find the equation of the common tangent to the circle
(a) 2x = 1 (b) 2x + 3 = 0 (x – 3)2 + y2 = 9 and the parabola y2 = 4x above the
(c) 2y + 3 = 0 (d) 2x + 5 = 0 x-axis.
13. Find the equation of the common tangent to the curves
48. The length of the focal chord of the parabola y2 = 4ax
y2 = 8x and xy = –1.
at a distance p from the vertex is
14. Find the common tangents of y = x2 and y = –x2 + 4x – 4.
2a 2 a3 4a 3 p3 15. Consider a circle with the centre lying on the focus of
(a) (b) 2 (c) 2
(d)
p p p a the parabola y2 = 2px such that it touches the direc-
49. The locus of a point such that two tangents drawn from trix of that parabola. Find a point of intersection of the
it to the parabola y2 = 4ax are such that the slope of one circle and the parabola.
is double the other is 16. If the normal drawn at a point (at12, 2at1) of the
9 2 9 parabola y2 = 4ax meets it again at (at22, 2at2), prove
(a) y 2 = ax (b) y = ax that t12 + t1t2 + 2 = 0.
2 4
17. Three normals to the parabola y2 = x are drawn through
(c) y2 = 9ax (d) x2 = 4ay
a point (c, 0), find c.
50. The point on the parabola y2 = 4x which are closest to 18. A tangent to the parabola y2 = 8x makes an angle 45°
the curve x2 + y2 – 24y + 128 = 0 is with the straight line y = 3x + 5. Find the equation of
(a) (0, 0) (b) (2, 2) the tangent and its point of contact.
(c) (4, 4) (d) none 19. Find the equations of the normal to the parabola y2 =
4ax at the ends of the latus rectum. If the normal again
meets the parabola at Q and Q¢, prove that QQ¢ = 12a.
LEVEL III 20. Prove that from any point P(at2, 2at) on the parabola
(Problems for JEE Advanced) y2 = 4ax, two normals can be drawn and their feet
Q and R have the parameters satisfying the equation
1. If two ends of a latus rectum of a parabola are the points l2 + lt + 2 = 0.
(3, 6) and (–5, 6), find its focus. 21. Find the locus of the points of intersection of those nor-
2. Find the locus of the focus of the family of parabolas mals to the parabola x2 = 8y which are at right angles to
a2 x2 a2 x each other.
y= + - 2a . 22. Two lines are drawn at right angles, one being a tangent
3 2
3. Find the equation of the parabola whose vertex and the to y2 = 4ax and the other to x2 = 4by. Show that the
focus lie on the axis of x at distances a and a1 from the locus of their points of intersection is the curve
origin, respectively. (ax + by)(x2 + y2) + (bx – ay)2 = 0.
4. A square has one vertex at the vertex of the parabola 23. Prove that the locus of the centroid of an equilat-
y2 = 4ax and the diagonal through the vertex lies eral triangle inscribed in the parabola y2 = 4ax is
along the axis of the parabola. If the ends of the other 9y2 = 4a(x – 8a).
diagonal lie on the parabola, find the co-ordinates of 24. Prove that the locus of the mid-points of chords of the
the vertices of the square. parabola y2 = 4ax which subtends a right angle at the
5. Find the equation of the parabola whose axis is y = x, vertex is y2 = 2a2(x – 4).
25. Prove that the locus of a point that divides a chord of
the distance from origin to vertex is 2 and the dis-
slope 2 of the parabola y2 = 4x internally in the ratio
tance from the origin to the focus is 2 2 . 1 : 2 is a parabola. Find the vertex of the parabola.
6. If a π 0 and the line 2bx + 3cy + 4d = 0 passes through 26. From a point A, common tangents are drawn to the
the points of intersection of the parabola y2 = 4ax and
a2
x2 = 4ay, prove that d2 + (2b + 3c)2 = 0. circle x 2 + y 2 = and the parabola y2 = 4ax. Find the
7. Two straight lines are perpendicular to each other. One 2
area of the quadrilateral formed by the common tan-
of them touches the parabola y2 = 4a(x + a) and the
gents, the chord of contact of the circle and the chord
other y2 = 4b(x + b). Prove that their points of intersec-
of contact of the parabola.
tion lie on the line x + a + b = 0.

CG_04.indd 14 2/6/2017 4:02:03 PM


Parabola 4.15

27. Normals are drawn from the point P with slopes m1, 7. The ordinates of points P and Q on the parabola
m2, m3 to the parabola y2 = 4x. If the locus of P with y2 = 12x are in the ratio 1 : 2. Find the locus of the points
m1m2 = a is a part of the parabola, find the value of a. of intersection of the normals to the parabola at P and Q.
28. The tangent at a point P to the parabola y2 – 2y – 4x + [Roorkee Main, 1998]
5 = 0 intersects the directrix at Q. Find the locus of a 8. Find the equations of the common tangents of the circle
point R such that it divides PQ externally in the ratio x2 + y2 – 6y + 4 = 0 and y2 = x. [Roorkee, 1999]
1 9. Find the locus of points of intersection of tangents
:1 . drawn at the ends of all normals chords to the parabola
2 y2 = 4(x – 1).
x2 [Roorkee Main, 2001]
29. Prove that the curve y = - + x + 1 is symmetric 10. Find the locus of the trisection point of any double
2
ordinate y2 = 4ax.
with respect to the line x = 1. And also prove that it is
11. Find the locus of the trisection point of any double
symmetric about its axis. ordinate x2 = 4by.
30. Three normals are drawn from the point (14, 7) to the 12. Find the shortest distance between the curves y2 = 4x
parabola y2 – 16x – 8y = 0. Find the co-ordinates of the and x2 + y2 – 12x + 31 = 0.
feet of the normals. 13. Find the radius of the circle that passes through the
31. Find the locus of the foot of the perpendicular drawn origin and touches the parabola at (a, 2a).
from a fixed point to any tangent to a parabola. 14. Find the condition if two different tangents of y2 = 4x
32. Find the locus of the point of intersection of the are the normals to x2 = 4by.
normals to the parabola y2 = 4ax at the extremities of 15. A circle is drawn to pass through the extremities of the
a focal chord. latus rectum of the parabola y2 = 8x. It is also given that
the circle touches the directrix of the parabola. Find the
radius of the circle.
LEVEL IV
(Tougher Problems for JEE Integer Type Questions
Advanced) 1. Find the maximum number of common chords of a
parabola and a circle.
1. From the point (–1, 2), tangent lines are drawn to the
2. If the straight lines y – b = m1(x + a) and y – b =
parabola y2 = 4x. Find the equation of the chord of con-
m2(x + a) are the tangents of y2 = 4ax, find the value of
tact. Also find the area of the triangle formed by the
(m1m2 + 4).
chord of contact and the tangents.
p
[Roorkee Main, 1994] 3. A normal chord of y2 = 4ax subtends an angle at
2. The equation y2 – 2x – 2y + 5 = 0 represents 2
(a) a circle with centre (1, 1) the vertex of the parabola. If its slope is m, then find the
(b) a parabola with focus (1, 2) value of (m2 + 3).
(c) a parabola with directrix x = 3/2 4. Find the slope of the normal chord of y2 = 8x that gets
(d) a parabola with directrix x = –1/2 bisected at (8, 2).
[Roorkee, 1995] 5. Find the maximum number of common normals of y2 =
3. A ray of light is coming along the line y = b from the 4ax and x2 = 4by.
6. Find the length of the latus rectum of the parabola
positive direction of x-axis and strikes a concave mir-
whose parametric equation are given by x = t2 + t + 1
ror whose intersection with the xy-plane is a parabola
and y = t2 – t + 1.
y2 = 4ax. Find the equation of the reflected ray and
7. If the shortest distance between the curves y2 = x – 1
show that it passes through the focus of the parabola.
and x2 = y – 1 is d, find the value of (8d2 – 3).
Both a and b are positive. [Roorkee Main, 1995] 8. If m1 and m2 be the slopes of the tangents that are drawn
4. If a tangent drawn at a point (t2, t2) on the parabola from (2, 3) to the parabola y2 = 4x, find the value of
y2 = 4x is the same as the normal drawn at a point Ê 1 1 ˆ
( 5 cos j , 2 sin j ) on the ellipse 4x2 + 5y2 = 20, find ÁË m + m + 2˜¯ .
1 2
the values of t and j. [Roorkee, 1996] 9. Let P(t1) and Q(t2) are two points on the parabola y2 =
5. The equations of normals to the parabola y2 = 4ax at the 4ax. If the normals at P and Q meet the parabola again
point (5a, 2a) are at R, find the value of (t1t2 + 3).
(a) y = x – 3a (b) y = x + 3a 10. Find the length of the chord intercepted between the
(c) y + 2x = 12a (d) 3x + y = 33a parabola y2 = 4x and the straight line x + y = 1.
[Roorkee, 1997] 11. If x + y = k is a normal to the parabola y2 = 12x, find the
6. Find the locus of the points of intersection of those nor- value of k.
mals to the parabola x2 = 8y which are at right angles to 12. Find the number of distinct normals drawn from the
each other. [Roorkee Main, 1997] point (–2, 1) to the parabola y2 – 4x – 2y – 3 = 0.

CG_04.indd 15 2/6/2017 4:02:03 PM


4.16 Coordinate Geometry Booster

(iii) The equation of the directrix of the parabola is


Comprehensive Link Passages
(a) 6x + 7 = 0 (b) 4x + 11 = 0
Passage I (c) 3x + 11 = 0 (d) 2x + 13 = 0
y = x is a tangent to the parabola y2 = ax2 + c. Passage V
(i) If a = 2, the value of c is The normals at three points P, Q, R on the parabola y2 = 4ax
(a) 1 (b) –1/2 (c) 1/2 (d) 1/8 meet at (a, b).
(ii) If (1, 1) is a point of contact, the value of a is (i) The centroid of DPQR is
(a) 1/ 4 (b) 1/3 (c) 1/ 2 (d) 1/6 a - 2a ˆ Ê 2a - 4a ˆ
(iii) If c = 2, the point of contact is (a) ÊÁ , 0˜ (b) Á , 0˜
Ë 3 ¯ Ë 3 ¯
(a) (3, 3) (b) (2, 2) (c) (6, 6) (d) (4, 4).
Êa bˆ Êa + b b -aˆ
Passage II (c) Á , ˜ (d) Á , ˜
Ë 3 3¯ Ë 3 3 ¯
Consider, the parabola whose focus is at (0, 0) and the tan-
gent at the vertex is x – y + 1 = 0. (ii) The orthocentre of DPQR is
(i) The length of the latus rectum is Ê bˆ Ê bˆ
(a) Á a + 6a, ˜ (b) Á a + 3a, ˜
Ë 2¯ Ë 2¯
(a) 4 2 (b) 2 2 (c) 8 2 (d) 3 2
Ê bˆ Ê bˆ
(c) Á a - 6a, - ˜ (d) Á a - 3a, ˜
(ii) The length of the chord of the parabola on the x-axis is Ë 2¯ Ë 2¯
(a) 4 2 (b) 2 2 (c) 8 2 (d) 3 2 (iii) The circumcentre of DPQR is
(iii) Tangents drawn to the parabola at the extremities of the Ê a + 2a b ˆ Ê a + 2a b ˆ
(a) Á ,- ˜ (b) Á , ˜
chord 3x + 2y = 0 intersect at an angle Ë 2 4¯ Ë 2 4¯
p p p
(a) (b) (c) (d) none Êa bˆ Êa bˆ
6 3 2 (c) Á , ˜ (d) Á , ˜
Ë 4 4¯ Ë 2 2¯
Passage III
Two tangents on a parabola are x – y = 0 and x + y = 0. If (2, 3) Passage VI
is the focus of the parabola. Consider the circle x2 + y2 = 9 and the parabola y2 = 8x. They
(i) The equation of the tangent at the vertex is intersect at P and Q in the first and the fourth quadrants, re-
(a) 4x – 6y + 5 = 0 (b) 4x – 6y + 3 = 0 spectively. The tangents to the circle at P and Q intersect the
(c) 4x – 6y + 1 = 0 (d) 4x – 6y + 3/2 = 0 x-axis at R and tangents to the parabola at P and Q intersect
(ii) The length of the latus rectum of the parabola is the x-axis at S.
6 10 2 9 2 (i) The ratio of the areas of Ds PQS and PQR is
(a) (b) (c) (d) (a) 1: 2 (b) 1 : 2 (c) 1 : 4 (d) 1 : 8
3 13 13 13
(iii) If P and Q are ends of the focal chord of the parabola, (ii) The radius of the circumcircle of DPRS is
then is (a) 5 (b) 3 3 (c) 3 2 (d) 2 3
2 13 2 13 2 13 (iii) The radius of the incircle of DPQR is
(a) (b) 2 13 (c) (d) (a) 4 (b) 3 (c) 8/3 (d) 2
3 3 7
Passage VII
Passage IV
If P is a point moving on a parabola y2 = 4ax and Q is a mov-
If l, m are variable real numbers such that 5l2 + 6m2 – 4lm + 3l
ing point on the circle x2 + y2 – 24ay + 128a2 = 0. The points
= 0, the variable line lx + my = 1 always touches a fixed pa-
P and Q will be closest when they lie along the normal to the
rabola, whose axis is parallel to x-axis.
parabola y2 = 4ax passing through the centre of the circle.
(i) The vertex of the parabola is
(i) If the normal at (at2, 2at) of the parabola passes through
Ê 5 4ˆ Ê 7 3ˆ
(a) ÁË - , ˜¯ (b) ÁË - , ˜¯ the centre of the circle, the value of t must be
3 3 4 4 (a) 1 (b) 2 (c) 3 (d) 4
Ê 5 7ˆ Ê 1 3ˆ (ii) The shortest distance between P and Q must be
(c) ÁË , - ˜¯ (d) ÁË , - ˜¯
6 6 2 4 (a) a ( 2 - 1) (b) 2a ( 5 - 1)
(ii) The focus of the parabola is
(c) 4a ( 5 - 1) (d) 4a ( 5 + 1)
Ê 1 7ˆ Ê 1 4ˆ
(a) Á , - ˜ (b) ÁË , ˜¯ (iii) When P and Q are closest, the point P must be
Ë 6 6¯ 3 3
(a) (1, 2a) (b) (2a, 2 2a )
Ê 3 3ˆ Ê 3 3ˆ
(c) Á , - ˜ (d) ÁË - , ˜¯ (c) (4a, 4a) (d) (5a, 4a)
Ë 2 2¯ 4 4

CG_04.indd 16 2/6/2017 4:02:04 PM


Parabola 4.17

(iv) When P and Q are closest, the point Q must be 2. Match the following columns:
Ê 4a 8a ˆ Ê 4a 8a ˆ Column I Column II
(a) Á , 6a - ˜ (b) Á , 12a - ˜
Ë 5 5¯ Ë 5 5¯ (A) The point, from which perpen- (P) (–1, 2)
Ê 4a 8a ˆ Ê 4a 12a ˆ dicular tangents can be drawn
(c) Á , - 12a - ˜ (d) Á , - 8a - ˜ to the parabola y2 = 4x, is
Ë 5 5¯ Ë 5 5¯
(B) The point, from which only one (Q) (3, 2)
(v) When P and Q are closest and the diameter QR of the
normal can be drawn to the pa-
circle is drawn through Q, the x-cordinate of Q is
rabola y2 = 4x, is
2a 8a 4a 6a
(a) - (b) - (c) - (d) - (C) The point, at which chord x – y (R) (–1, –5)
5 5 5 5 + 1 = 0 of the parabola y2 = 4x
Passage VIII is bisected, is
If a source of light is placed at the fixed point of a parabola (D) The point, from which tangents (S) (5, –2)
and if the parabola is reflecting surface, the ray will bounce cannot be drawn to the parabola
back in a line parallel to the axis of the parabola. y2 = 4x, is
(i) A ray of light is coming along the line y = 2 from the
positive direction of x-axis and strikes a concave mirror 3. Match the following columns:
whose intersection with the xy plane is a parabola y2 = Column I Column II
8x, the equation of the reflected ray is (A) The equation of the director (P) 2y – 1 = 0
(a) 2x + 3y = 4 (b) 3x + 2y = 6 circle to the parabola y2 = 12x is
(c) 4x + 3y = 8 (d) 5x + 4y = 10
(B) The equation of the director (Q) x – 2 = 0
(ii) A ray of light moving parallel to the x-axis gets reflect-
circle to the parabola x2 = 16y is
ed from a parabolic mirror whose equation is y2 + 10y –
4x + 17 = 0. After reflection, the ray must pass through (C) The equation of the director (R) y + 4 = 0
the point circle to the parabola
(a) (–2, –5) (b) (–1, –5) y2 + 4x + 4y = 0 is
(c) (–3, –5) (d) (–4, –5) (D) The equation of the director (S) x + 3 = 0
(iii) A ray of light is coming along the line x = 2 from the circle to the parabola
positive direction of y-axis and strikes a concave mirror y = x2 + x + 1 is
whose intersection with the xy plane is a parabola x2 =
4y, the equation of the reflected ray after second reflec- 4. Match the following columns:
tion is Column I Column II
(a) 2x + y = 1 (b) 3x – 2y + 2 = 0 (A) Number of distinct normals can (P) 3
(c) y = 1 (d) none be drawn from (–2, 1) to the pa-
(iv) Two rays of light coming along the line y = 1 and rabola y2 – 4x – 2y – 3 = 0 is
y = –2 from the positive direction of x-axis and strikes a
(B) Number of distinct normals can (Q) 1
concave mirror whose intersection with the xy plane is
be drawn from (2, 3) to the pa-
a parabola y2 = x at A and B, respectively. The reflected
rabola y = x2 + x + 1 is
rays pass through a fixed point C, the area of DABC is
(a) 21/ 8 s.u. (b) 19/2 s.u. (C) Number of distinct normals can (R) 0
(c) 17/2 s.u. (d) 15/2 s.u. be drawn from (–5, 3) to the pa-
rabola y2 – 4x – 6y – 1 = 0 is
Matrix Match (D) Number of tangents can be drawn (S) 2
(For JEE-Advanced Examination Only) from (1, 2) to the parabola
y2 – 2x – 2y + 1 = 0 is
1. Match the following columns: AB is a chord of the pa-
rabola y2 = 4ax joining A(at12, 2at1) and B(at22, 2at2). 5. Match the following columns:
Normals are drawn at points P, Q and R lying on the
Column I Column II
parabola y2 = 4x which intersect at (3, 0).
(A) AB is a normal chord, if (P) t2 = 2 – t1
Column I Column II
(B) AB is a focal chord, if (Q) t1t2 = –4
(A) Area of DPQR (P) 2
(C) AB subtends 90° at (R) t1t2 = –1
(0, 0), if (B) Radius of the circumcircle DPQR (Q) 5/2
(D) AB is inclined at 45° to 2
(S) t1 + t1t2 + 2 = 0 (C) Centroid of DPQR (R) (5/2, 0)
the axis of the parabola (D) Circumcentre of DPQR (S) (2/3, 0)

CG_04.indd 17 2/6/2017 4:02:05 PM


4.18 Coordinate Geometry Booster

6. Match the following columns: 2. A is a point on the parabola y2 = 4ax. The normal at
A cuts the parabola again at B. If AB subtends a right
Column I Column II
angle at the vertex of the parabola, find the slope of AB.
(A) If y + 3 = m1(x + 2) and (P) m1 + m2 = 0 [IIT-JEE -1982]
y + 3 = m2(x + 2) are two
No questions asked in 1983.
tangents to the parabola
y2 = 8x, then 3. Find the equation of the normal to the curve x2 = 4y
which passes through the point (1, 2). [IIT-JEE, 1984]
(B) If y = m1x + c1 and y = m2x (Q) m1 + m2 = –1
4. Three normals are drawn from the point (c, 0) to the
+ c2 are two tangents to a
curve y2 = x. Show that c must be greater than 1/2. One
parabola y2 = 4a(x + a),
normal is always the x-axis. Find c for which the other
then
two normals are perpendicular to each other.
(C) If y = m1(x + 4) + 2013 and (R) m1, m2 = –1 [IIT-JEE, 1991]
y + 1 = m2(x + 4) + 2014 5. Through the vertex O of the parabola y2 = 4x, chords
are two tangents to the OP and OQ are drawn at right angles, show that for
parabola y2 = 16x, then all positions of P, PQ cuts the axis at the parabola at a
(D) If y = m1(x – 2) + 2010 (S) m1 – m2 = 0 fixed point. Also find the locus of the mid-point of PQ.
and y = m1(x – 2) + 2010 [IIT-JEE, 1994]
are two tangents to the 6. Consider a circle with its centre lying on the focus of
parabola y2 = –8x, then the parabola y2 = 2px such that it touches the direc-
trix of the parabola. Then a point of intersection of the
7. Match the following columns:
circle and the parabola is
Column I Column II
Êp ˆ Ê p ˆ Ê p pˆ
(A) If 2x + y + l = 0 is a normal to the (P) (a) Á , p˜ or Á - , p˜ (b) Á , ˜
9 Ë2 ¯ Ë 2 ¯ Ë 2 2¯
parabola y2 = –8x, then l is
Ê p ˆ Ê p pˆ
(B) If x + y = k is a normal to the pa- (Q) 24 (c) Á - , p˜ (d) Á - , - ˜
Ë 2 ¯ Ë 2 2¯
rabola y2 = 12x, then k is
[IIT-JEE, 1995]
(C) If 2x – y – c = 0 is a tangent to the (R) 17
7. Show that the locus of a point that divides a chord of
parabola y2 = 16x, then c is
slope 2 of the parabola y2 = 4x internally in the ratio
(D) If y = 4x + d is a tangent to the (S) 2 1 : 2 is parabola. Also find its vertex. [IIT-JEE, 1995]
parabola y2 = 4(x + 1), then 4d is 8. Points A, B, C lie on the parabola y2 = 4ax. The tangent
to the parabola at A, B and C taken in pair intersect at
8. Match the following columns
the points P, Q and R. Determine the ratio of the areas
Column I Column II of Ds ABC and PQR. [IIT-JEE, 1996]
(A) The equation of the directrix (P) x – 2y + 4 9. From a point A, common tangents are drawn to the
of the parabola =0 a2
circle x 2 + y 2 = and the parabola y2 = 4ax. Find
y2 + 4x + 4y + 2 = 0 is 2
(B) The equation of the axis of (Q) 2x – 3 = 0 the area of the quadrilateral formed by the common
the parabola tangents drawn from A and the chord of contact of the
x2 + 4x + 4y + 2013 = 0 is circle and the parabola. [IIT-JEE, 1996]
(C) The equation of the tangent to (R) x + 3 = 0 No questions asked in between 1997 to 1999.
the parabola y2 = 4x from the 10. If x + y = k is normal to y2 = 12x, then k is
point (2, 3) is (a) 3 (b) 9 (c) –9 (d) –3
(D) The equation of the directrix (S) x+2=0 [IIT-JEE, 2000]
of the parabola y2 = 4x + 8 is 11. If the line x – 1 = 0 is the directrix of the parabola y2 =
kx – 8, one of the value of the k is
(a) 1/8 (b) 8 (c) 4 (d) 1/4
Questions asked in Previous Years’ [IIT-JEE, 2000]
JEE-Advanced Examinations 12. The equation of the directrix of the parabola y2 + 4y +
4x + 2 = 0 is
1. Suppose that the normals drawn at three different (a) x = –1 (b) x = 1
points on the parabola y2 = 4x pass through the point (c) x = –3/2 (d) x = 3/2
(h, k). Show that h > 2a. [IIT-JEE, 1981] [IIT-JEE, 2001]

CG_04.indd 18 2/6/2017 4:02:05 PM


Parabola 4.19

13. The equation of the common tangent touching the 22. The equation of the common tangent to the parabola
circle (x – 3)2 + y2 = 9 and the parabola y2 = 4x above y = x2 and y = – (x – 2)2 is/are
the x-axis is (a) y = 4(x – 1) (b) y = 0
(a) y 3 = (3x + 1) (b) y 3 = - x - 3 (c) y = –4(x – 1) (d) y = –30x – 50
[IIT-JEE, 2006]
(c) y 3 = x + 3 (d) y 3 = - (3x + 1) 26. Match the following:
[IIT-JEE, 2001] Normals are drawn at points P, Q and R lying on the
14. The locus of the mid-point of the line segment joining parabola y2 = 4x which intersect at (3, 0).
the focus to a moving a point on the parabola y2 = 4x is
another parabola with directrix Column I Column II
(a) x = –a (b) x = –a/2 (i) Area of DPQR (A) 2
(c) x = 0 (d) x = a/2 (ii) Radius of circumcircle of (B) 5/2
[IIT-JEE, 2002] DPQR
15. The equation of the common tangent to the curve y2 = (iii) Centroid of DPQR (C) (1, 0)
8x and xy = –1 is
(a) 3y = 9x + 2 (b) y = 2x + 1 (iv) Circumcentre of DPQR (D) (2/3, 0)
(c) 2y = x + 8 (d) y = x + 2 [IIT-JEE, 2006]
[IIT-JEE, 2002] 24. Comprehension
16. The focal chord to y2 = 16x is tangent to (x – 6)2 + y2 = 2, Consider the circle x2 + y2 = 9 and the parabola y2 =
the possible values of the slope of this chord, are 8x. They intersect at P and Q in the first and the fourth
(a) {–1, 1} (b) {–2, 2} quadrants, respectively. The tangents to the circle at P
(c) {–2, 1/2} (d) {2, –1/2} and Q intersect the x-axis at R and tangents to the pa-
[IIT-JEE, 2003] rabola at P and Q intersect the x-axis at S.
17. Let C1 and C2 be, respectively, the parabolas x2 = y – 1 (i) The ratio of the areas of Ds PQS and PQR is
and y2 = x – 1, let P be any point on C1 and Q be any (a) 1: 2 (b) 1 : 2 (c) 1 : 4 (d) 1 : 8
point on C2. Let P1 and Q1 be the reflections of P and (ii) The radius of the circumcircle of DPRS is
Q respectively, with respect to the line y = x. Prove that
P1 lies on C2, P1 lies on C1 and PQ ≥ min[PP1, QQ1]. (a) 5 (b) 3 3 (c) 3 2 (d) 2 3
Hence or otherwise determine points P0 and Q0 on the (iii) The radius of the incircle of DPQR is
parabolas P0 and Q0, C1 and C2, respectively such that (a) 4 (b) 3 (c) 8/3 (d) 2
P0Q0 £ PQ for all pairs of points (P1, Q) with P on C1 [IIT-JEE, 2007]
x2
and Q on C2 [IIT-JEE, 2003] 25. Statement 1: The curve y = - + x + 1 is symmetric
18. Three normals with slopes m1, m2 and m3 are drawn with respect to the line x = 1. 2
from a point P not on the axis of the parabola y2 = 4x. Statement 2: A parabola is symmetric about its axis.
If m1m2 = a, results in the locus of P being a part of the [IIT-JEE, 2007]
parabola, find the value of a. [IIT-JEE, 2003] 26. Let P(x1, y1), Q(x2, y2), y1 < 0, y2 < 0, be the end-points
19. The angle between the tangents drawn from the point of the latus rectum of the ellipse x2 + 4y2 = 4. The equa-
(1, 4) to the parabola y2 = 4x is tion of the parabola with the latus rectum PQ are
p p p p
(a) (b) (c) (d) (a) x 2 + 2 3 y = 3 + 3 (b) x 2 - 2 3 y = 3 + 3
6 4 3 2
[IIT-JEE, 2004] (c) x 2 + 2 3 y = 3 - 3 (d) x 2 - 2 3 y = 3 - 3
20. At any point P on the parabola y2 – 2y – 4x + 5 = 0, a [IIT-JEE, 2008]
tangent is drawn which meets the directrix at Q. Find 27. The tangent PT and the normal PN to the parabola
the locus of point R which divides QP externally in the y2 = 4ax at a point P on it meet axes at points T and N,
1 respectively. The locus of the centroid of DPTN is a
ratio :1 . [IIT-JEE, 2004] parabola whose
2
(a) vertex is (2a/3, 0) (b) directrix is x = 0
No questions asked in 2005. (c) latus rectum is 2a/3 (d) focus is (a, 0)
21. The axis of a parabola is along the line y = x and the [IIT-JEE, 2009]
distances of its vertex and focus from the origin are 28. Let A and B be two distinct points on the parabola y2 =
2 and 2 2 , respectively. If the vertex and the focus 4x. If the axis of the parabola touches a circle of radius
both lie in the first quadrant, the equation of the pa- r having AB as the diameter, the slope of the line join-
rabola is ing A and B can be
(a) (x – y)2 = (x – y – 2) (b) (x – y)2 = (x + y – 2) 1 1 2 2
(a) - (b) (c) (d) -
(c) (x – y)2 = 4(x + y – 2) (d) (x – y)2 = 8(x + y – 2) r r r r
[IIT-JEE, 2006] [IIT-JEE, 2010]

CG_04.indd 19 2/6/2017 4:02:06 PM


4.20 Coordinate Geometry Booster

29. Let L be a normal to the parabola y2 = 4x. If L passes 36. The value of r is
through the point (9, 6), then L is given by 1 t2 + 1 1 t2 - 1
(a) y – x + 3 = 0 (b) y + 3x – 33 = 0 (a) - (b) (c) (d)
t t t t
(c) y + x – 15 = 0 (d) y – 2x + 12 = 0
[IIT-JEE, 2104]
[IIT-JEE, 2011]
37. If st = 1, the tangent at P and the normal at S to the
30. Consider the parabola y2 = 8x, Let D1 be the area of the
parabola meet at a point whose ordinate is
triangle formed by the end-points of its latus rectum
Ê1 ˆ (t1 + 1) 2 a (t 2 + 1) 2
and the point P Á , 2˜ on the parabola, and D2 be the (a) (b)
Ë2 ¯ 2t 3 2t 3
area of the triangle formed by drawing tangents at P
D a (t 2 + 1) 2 a (t 2 + 2) 2
and at the end-points of the latus rectum. Then 1 is... (c) (d)
D2 t3 t3
[IIT-JEE, 2011] [IIT-JEE, 2014]
31. Let S be the focus of the parabola y2 = 8x and let PQ be 38. Let the curve C be the mirror image of the parabola y2
the common chord of the circle x2 + y2 – 2x – 4y = 0 and = 4x with respect to the line x + y + 4 = 0. If A and B are
the given parabola. The area of DPQS is ... the points of intersection of C with the line y = –5, then
[IIT-JEE, 2012] the distance between A and B is... [IIT-JEE-2015]
2
Comprehension 39. If the normals of the parabola y = 4x drawn at the end
Let PQ be a focal chord of a parabola y2 = 4ax. The tangents points of its latus rectum are tangents to the circle (x –
to the parabola at P and Q meet at a point lying on the line 3)2 + (y + 2)2 = r2, then the value of r2 is...
y = 2x + a, a > 0. [IIT-JEE-2015]
32. The length of the chord PQ is 40. Let P and Q be distinct points on the parabola y2 = 2x
(a) 7a (b) 5a (c) 2a (d) 3a such that a circle with PQ as diameter passes through
33. If the chord PQ subtends an angle q at the vertex of y2 the vertex O of the parabola. If P lies in the first quad-
= 4ax, then tan q is rant and the area of the triangle DOPQ is 3: 2, then
2 7 2 7 2 5 2 5 which of the following is (are) the coordinates of P?
(a) (b) - (c) (d) -
3 3 3 3 (a) (4, 2 2) (b) (9, 3 2)
[IIT-JEE, 2013] Ê1 1 ˆ
(c) Á , (d) (1, 2)
37. Match Matrix Ë 4 2 ˜¯
A line L: y = mx + 3 meets y-axis at E(0, 3) and the arc [IIT-JEE-2015]
of the parabola y2 = 16x, 0 £ y £ 6 at the point F(x0, y0). 2 2
41. The circle C1 : x + y = 3, with centre at O, intersects
The tangent to the parabola at F(x0, y0) intersects the the parabola x2 = 2y at the point P in the first quadrant.
y-axis at G(0, y1). The slope m of the line L is chosen Let the tangent to the circle C1 at P touches other two
such that the area of DEFG has a local maximum. circles C2 and C3 at R2 and R3, respectively.
Match List I and List II and select the correct answers
Suppose C2 and C3 have equal radii 2 3 and centres
using the code given below the lists.
Q2 and Q3, respectively.
List I List II
P m= 1. 1/2 If Q2 and Q3 lie on the y-axis, then
Q Maximum area of DEFG is 2. 4 (a) Q2Q3 = 12 (b) R2 R3 = 4 6
R y0 = 3. 2 (c) ar ( DOR2 R3 ) = 6 2 (d) ar ( DPQ2Q3 ) = 4 2
S y1 = 4. 1
[IIT-JEE, 2013] [IIT-JEE-2016]
35. The common tangents to the circle x2 + y2 = 2 and the 42. Let P be the point on the parabola y2 = 4x which is at
parabola y2 = 8x touch the circle at the points P, Q and the shortest distance from the center S of the circle x2 +
the parabola at the points R, S. Then the area of the y2 – 4x – 16y + 64 = 0. Let Q be the point on the circle
quadrilateral PQRS is dividing the line segment SP internally. Then
(a) 3 (b) 6 (c) 9 (d) 15 (a) SP = 2 5
[IIT-JEE, 2014] (b) SQ:SP = ( 5 + 1):2
Comprehension (c) the x-intercept of the normal to the parabola at P is
Let a, r, s, t be non-zero real numbers. 6.
1
Let P(at2, 2at), Q, R(ar2, 2ar) and S(as2, 2as) be distinct (d) the slope of the tangent to the circle at Q is
points on the parabola y2 = 4ax. Suppose that PQ is the focal 2
chord and lines QR and PK are parallel, where K is the point [IIT-JEE-2016]
(2a, 0).

CG_04.indd 20 2/6/2017 4:02:06 PM


Parabola 4.21

A NSWERS

LEVEL I 33.
34.
y = ±x ± 4
y = 4x – 4
35. y=x+2
1. Parabola
36. x + 2y = 1
2. l = 4
1 37. x - 3y + 3 = 0
3.
2 38. L(1, 2) and L¢(1, –2) is (1, 0)
4. 20{x2 + y2 – 2x – 4y + 5} p
39.
= (x2 + 9y2 + 100 + 6xy + 20x + 6y) 3
5. x2 + 2xy + y2 + 2x + 2y + 4 = 0 3
40.
Ê 11 ˆ 2
6. (i) V: (–3, 1), S: Á - , 1˜ , L.R. = 1, A: y =1
Ë 4 ¯ 41. 2 2
Ê 5 ˆ 42. –1
(ii) V: (–2, 2).; S: Á - , 2˜ ; L.R. = 3, A: y = 2
Ë 4 ¯ 43. (–2, –9)
44. x+a+b=0
Ê 23 ˆ
(iii) V: (2, -6), S: Á 2, — ˜ ; L.R. = 1, A: x = 2 1 2
Ë 4¯ 45. a (t1 - t2 )(t2 - t3 )(t3 - t1 )
2
Ê 1 1ˆ
(iv) V: Á — , ˜ ; S: (–1/2, 3/4) ; L.R. =1, 46. x2 + y2 – a(1 + t2t3 + t3t1 + t1t2)x
Ë 2 2¯
– a(t1 + t2 + t3 – t1t2t3)y
A: 2y – 1 = 0.
+ a2(t2t3 + t3t1 + t1t2) = 0
7. (3, 6) and (3, –6)
8. x2 – 2xy + y2 + 32x + 32y + 76 = 0 Ê Ê 1 1 1 1 ˆˆ
47. Á - a, a Á + + +
9. x=0 Ë Ë m1 m2 m3 m1m2 m3 ˜¯ ˜¯
10. y2 = 8x + 24 = 8(x + 3) 48. x + a = 0
11. y = x2 + 3x + 2 49. (i) 4x + 9 = 0
12. (y – 2)2 = 2(x – 1) (ii) y + 2 = 0
13. (x – 3)2 = 12(y – 1) (iii) x = 0
2 2
14. t1t2 = –1 50. x = 2a + a (t1 + t2 + t1t2 )
2
Ê 1ˆ and y = – at1t2(t1 + t2)
15. a Á t + ˜
Ë 2¯ 2
51. t2 = - t1 -
16. 4a cosec2 q t1
18. (0, at) 52. ≥8
21. (0, 4) 53. 2
22. –3 < x < 3 54. x+y=3
23. (1/2, –2) or (25/2, –10) 55. y = 2x – 24
24. 16 56. k=9
4 9PQ = 80 10
25. 2
1 + m 2 a (a - mc) 57.
m 58. y2 = a(x – 3a)
26. (at1t2, a(t1 + t2)) 59. al3 + 2alm2 + nm2 = 0
27. 2x + y – 4 = 0 and 2x – y + 1 = 0 60. q = tan -1 ( 2)
Ê1 ˆ Ê1 ˆ 61. (m2, – 2m) = (1, 2)
28. Á , 2˜ or Á , - 2˜
Ë2 ¯ Ë2 ¯ 62. PQ = 6 3
p 64. y2 = a(x – a)
29. q =
2 65. 5
30. y = x – 2 and y = –3x + 2 66. 2 5 -1
31. x + y + a = 0
Ê 2h - 4a ˆ
Ê a1/3 ˆ Ê b1/3 ˆ 69. ÁË , 0˜
32. y = Á - 1/3 ˜ x + a Á - 1/3 ˜ 3 ¯
Ë b ¯ Ë a ¯

CG_04.indd 21 2/6/2017 4:02:07 PM


4.22 Coordinate Geometry Booster

70. h > 2a Êp ˆ Êp ˆ
71. 27ak2 < 4(h – 2a)3 15. Á , p˜ , Á , - p˜
Ë2 ¯ Ë2 ¯
-1 Ê tan q ˆ 17. c > 1/2
72. j = tan Á
Ë 2 ˜¯ 18. x – 2y + 8 = 0, (8, 8)
74. (3a, 0)
79. 2x – 3y + 4 = 0 19. 12a
80. y = 3x – 3 21. x2 = 2(y – 6)
81. (y2 – 4ax) = (x + a)2 tan2 q 22. (ax + by)(x2 + y2) + (bx – ay)2 = 0
1 23. 9y2 = 4a(x – 8a)
82. ¥ (k 2 - 4ah)(k 2 + 4a 2 )
|a| 24. y2 = 2a2(x – 4)
(k 2 - 4ah)3/2 Ê 2 8ˆ
83. 25. Á , ˜
2a Ë 9 9¯
84. 4x – 3y + 1 = 0 15a 2
85. y2 = 2a (x – a) 26.
4
86. y2 = 2ax 27. a = 2
87. y4 – 2a(x – 2a)y2 + 8a4 = 0 28. (y – 1)2(x + 1) + 4 = 0
88. y4 = 2a(x – 4a) 30. (3, –4), (0, 0), (8, 16)
89. (2a – b)y2 = 4a2x ( x - h) a ( y - k )
90. y2 – 2ax – by + 6ab = 0 31. y = - -
( y - k) ( x - h)
91. y2(2x + a) = a(3x + a)2
93. y = a(t1 + t2) 32. y2 = a(x – 3a)
94. 15
95. x – 7y + 10 = 0
LEVEL IV
LEVEL II 1. 8 2 s.u.
2. (c)
1. (c) 2. (c) 3. (b) 4. (c) 5. (a) 3. (b2 – 4a2)y – 4abx + 4a2b = 0
6. (d) 7. (c) 8. (c) 9. (a) 10 () 1 Ê 1 ˆ
11. (d) 12. (b) 13. (a) 14. (d) 15. (b) 4. t = ± , j = cos -1 Á - ˜
5 Ë 5¯
16. (a) 17. (a) 18. (a) 19. () 20. (d)
21. (c) 22. (b) 23. (b) 24. (b) 25. (b) 5. (a, c)
26. (b) 27. (b) 28. (d) 29. (d) 30. (a) 6. x2 = 2(y – 6)
31. (b) 32. (b) 33. (d) 34. (c) 35. (a) 3/2
Ê x - 6ˆ
36. (c) 37. (c) 38. (b) 39. (c) 40. (d) 7. y + 18 Á =0
41. (c) 42. (d) 43. (a) 44. (b) 45. (a) Ë 21 ˜¯
46. (c) 47. (d) 48. (c) 49. (a) 50. (c) 8. x – 2y + 1 = 0
9. (2 – x)y2 = (5y2 + 32)
9y2 = 4ax
LEVEL III
10.
11. 9x2 = 4by

1. (4, 0) 12. ( 21 - 5)
2. 4ya2 + 8a3 – xa4 + 4x = 0 5a
13.
3. y2 = 4(a1 – a)(x – a) 2
4. (0, 0), (4a, 4a), (4a, – 4a), (8a, 0) 1
5. (x – y)2 = 8(x + y – 2) 14. |b| <
2 2
8. m = 1, –1
15. 4
9. (–1, 0)
10. (2, 4) INTEGER TYPE QUESTIONS
12. y 3= x+3 1. 6
13. y=x+2 2. 3
14. y = 0, y = 4(x – 1) 3. 5

CG_04.indd 22 2/6/2017 4:02:07 PM


Parabola 4.23

4. 2 Ê 2 8ˆ
5. 6 7. Á , ˜
Ë 9 9¯
6. 2
8. 2 : 1
7. 6
8. 5 15a 2
9.
9. 5 4
10. 8 10. (b)
11. 9 11. (c)
12. 3 12. (d)
13. (c)
COMPREHENSIVE LINK PASSAGES 14. (c)
15. (d)
Passage I: (i) (d) (ii) (c) (iii) (d) 16. (a)
Passage II: (i) (b) (ii) (a) (iii) (c) 17.
Passage III: (i) (d) (ii) (b) (iii) (c) 18. a = 2
Passage IV: (i) (a) (ii) (b) (iii) (c) 19. (c)
Passage V: (i) (b) (ii) (c) (iii) (a) 20. (y – 1)2(x + 1) + 4 = 0
Passage VI: (i) (c) (ii) (b) (iii) (d) 21. (d)
Passage VII: (i) (a) (ii) (c) (iii) (c) (iv) (b) (v) (c) 22. (a, b)
Passage VII: (i) (c) (ii) (b) (iii) (d) 23. (i) Æ A; (ii) Æ B; (iii) Æ D; (iv) Æ C
24. (i) - (c) (ii)-(b) (iii)-(d)
MATRIX MATCH 25. (a)
1. (A) Æ (S); (B) Æ (R); (C) Æ (Q); (D) Æ (P) 26. (b, c)
2. (A) Æ (P, R); (B) Æ (P,R); (C) Æ (Q); (D) Æ (Q, S) 27. (a, d)
3. (A) Æ (S); (B) Æ (R); (C) Æ (Q); (D) Æ (P) 28. (c, d)
4. (A) Æ (Q); (B) Æ (P); (C) Æ (Q); (D) Æ (R) 29. (a, b, d)
5. (A) Æ (P); (B) Æ (Q); (C) Æ (S); (D) Æ (R) 30. 2
6. (A) Æ (R); (B) Æ (R); (C) Æ (R); (D) Æ (R) 31. 4
7. (A) Æ (Q); (B) Æ (P); (C) Æ (S); (D) Æ (R) 32. (b)
8. (A) Æ (Q); (B) Æ (S);(C) Æ (P); (D) Æ (R) 33. (d)
34. m = 1
QUESTIONS ASKED IN PREVIOUS YEARS’ JEE-ADVANCED 35. (d)
EXAMINATIONS 36. (d)
37. (b)
2. ± 2 38. 4
3. x+y=3 39. 2
4. 3/4 40. (a, c)
5. y2 = 2x – 8 41. (a, b, c)
6. (a) 42. (a, c, d)

H INTS AND S OLUTIONS

LEVEL I a2x2 – 2abxy + b2y2 – 2ax – 2by + 1 = 0


Here A = a , B = b2 and H = –ab
2

1. The given equation is Now, H2 – AB = a2b2 – a2b2 = 0


ax + by = 1 Hence, it represents a parabola.
2. Here, a = 1, h = –2 and b = l.
fi ( ax + by ) 2 = 1
Since, it represents a parabola, so
fi ax + by + 2 abxy = 1
h2 – ab = 0
fi (ax + by — 1) 2 = (2 abxy ) 2 fi 4–l=0
fi a2x2 + b2y2 + 1 + 2abxy – 2ax – 2by fi l=4
= 4abxy

CG_04.indd 23 2/6/2017 4:02:07 PM


4.24 Coordinate Geometry Booster

3. The given conic is Latus rectum: 4a = 1


2 2
16( x + ( y - 1) ) = ( x + 3 y - 5) 2 Directrix: X + a = 0
1
2 fi x+3=
2 2 1 Ê x + 3 y - 5ˆ 4
fi ( x + ( y - 1) ) = Á ˜ …(i)
4Ë 1+ 3 ¯ 11
fi x= -
which represents an ellipse. 4
Now, Eq. (i) can also be written as fi 4x + 11 = 0
SP2 = e2 ¥ PM2 Axis: Y = 0
1 fi y–1=0
Thus, the eccentricity is . fi y=1
2
(ii) The given equation is
4. Let S be the focus, PM be the directrix and the eccen-
y2 = 3x + 4y + 2
tricity = e
fi y2 – 4y = x2 + 2
From the definition of conic section, we get
fi y2 – 4y + 4 = 2x + 6
SP
=e fi (y – 2)2 = 3(x + 2)
PM fi Y2 = 3X,
fi SP = e ¥ PM where X = x + 2 and Y = y – 2
fi SP2 = e2 ¥ PM2 Vertex: (0, 0)
2
1 Ê x + 3y + 10 ˆ fi X = 0, Y = 0
fi ( x - 1) 2 + ( y - 2) 2 = ¥
2 ÁË 1 + 9 ˜¯ fi x + 2 = 0 and y – 2 = 0
fi x = –2 and y = 2
fi 20{(x – 1)2 + (y – 2)2} = (x + 3y + 10)2
Hence, the vertex is (–2, 2)
fi 20{x2 + y2 – 2x – 4y +5}
Focus: (a, 0)
= (x2 + 9y2 + 100 + 6xy + 20x + 6y)
fi X = a, Y = 0
5. Let S be the focus and PM be the directrix.
3
From the definition of conic section, it is clear that, fi x+2= ,y–2=0
SP = PM 4
5
fi SP2 = PM2 fi x = - and y = 2
2 4
Ê x - y + 3ˆ Ê 5 ˆ
fi ( x - 1) 2 + ( y - 1) 2 = Hence, the focus is Á - , 2˜
ÁË 1 + 1 ˜¯
Ë 4 ¯
fi 2{(x – 1)2 + (y – 1)2} = (x – y + 3)2 Latus rectum: 4a = 3
fi 2(x2 + y2 – 2x – 2y + 2) Directrix: X + a = 0
= (x2 + y2 + 9 – 2xy –6x – 6y) fi x + 2 = 3/4
fi x + 2xy + y2 + 2x + 2y + 4 = 0
2 fi x = –5/3
6. (i) The given equation is fi 3x + 5 = 0
y2 = x + 2y + 2 Axis: Y = 0
fi y2 – 2y = x + 2 fi y–2=0
fi (y – 1)2 = x + 3 fi y=2
fi Y2 = X, where X = x + 3, Y = y – 1 (iii) The given equation is
Vertex: V(0, 0) x2 = y + 4x + 2
fi X = 0, Y = 0 fi x2 – 4x = y + 2
fi x + 3 = 0, y – 1 = 0 fi x2 – 4x + 4 = y + 6
fi x = –3, y = 1 fi (x – 2)2 = y + 6
Hence, the vertex is (–3, 1) fi X2 = Y,
Focus: (a, 0) where X = x – 2 and Y = y + 6
\ X = a, Y = 0 Vertex: (0, 0)
1 fi X = 0, Y = 0
fi x + 3 = , y -1= 0 fi x – 2 = 0 and y + 6 = 0
4
1 fi x = 2 and y = –6
fi x = - 3, y = 1 Hence, the vertex is (2, –6)
4
Focus: (0, a)
11
fi x = - , y =1 fi X = 0, Y = a
4 fi x – 2 = 0 and y + 6 = 1/4
Ê 11 ˆ fi x = 2 and y = –23/4
Hence, the focus is Á - , 1˜
Ë 4 ¯

CG_04.indd 24 2/6/2017 4:02:08 PM


Parabola 4.25

Ê 23 ˆ When x = 3, y2 = 12 ¥ 3 = 36
Hence, the focus is Á 2, - ˜ fi y = ±6
Ë 4¯
Hence, the co-ordinates of the points are (3, 6) and
Latus rectum: 4a = 1
(3, –6).
Directrix: Y + a = 0
8. Let the vertex be V and the focus be S.
fi y + 6 = 1/4
The equation of axis is x – y = 0.
fi 4y + 23 = 0
Y
Axis: X = 0
fi x–2=0
fi x=2
(iv) The given equation is
x2 + x + y = 0 O
X¢ X
fi x2 + x = –y V(–2, –2)
2
Ê 1ˆ 1 Ê 1ˆ F(–6, 6)
fi ÁË x + ˜¯ = - y + = - ÁË y - ˜¯
2 4 4
fi X2 = –Y, where Y¢
1 1
X = x + ,Y = y - Let the point Q is the point of intersection of the axis
2 2
and the directrix.
Vertex: (0, 0) fi X = 0, Y = 0 Clearly, V is the mid-point of Q and S.
1 1 Then Q is (2. 2).
fi x + = 0, y - = 0
2 2 As we know that the directrix is perpendicular to the
1 1 axis of the parabola. So, the equation of the directrix is
fi x=— ,y=
2 2 x+y–k=0
Ê 1 1ˆ which is passing through (2, 2).
Hence, the vertex is Á - , ˜ .
Ë 2 2¯ Therefore, k = 4.
Focus : (0, –a) Hence, the equation of the directrix is
fi X = 0, Y = –a x+y–4=0
fi x + 1/2 = 0, y – 1/2 = 1/4 Thus the equation of the parabola is
fi x = –1/2, y = 3/4 Ê x + y - 4ˆ
Hence, the focus is (–1/2, 3/4) ( x + 6) 2 + ( y + 6) 2 =
ÁË 1 + 1 ˜¯
Latus rectum: 4a = 1
Directrix: Y – a = 0 fi 2((x + 6)2 + (y + 6)2) = (x + y – 4)2
fi y – 1/2 – 1/4 = 0 fi 2(x2 + y2 + 12x + 12y + 36)
fi y – 3/4 = 0 = (x2 + y2 + 16 + 2xy + 8x – 8y)
fi 4y – 3 = 0 fi x – 2xy + y2 + 32x + 32y + 76 = 0
2

Axis: Y = 0 fi y – 1/2 = 0 fi 2y – 1 = 0 9. The given equations are x = t2 + 1 and y = 2t + 1


7. Let the point be (x, y) Eliminating t, we get
Y (y – 1)2 = 4(x – 1)
fi Y2 = 4X, where X = (x – 1)
P and Y = (y – 1)
Hence, the equation of the directrix is
X+a=0
O S
X fi x–1+1=0
fi x = 0.
10. Let the vertex be V and the focus be S.
Let Q be the point of intersection of the axis and the
directrix.
The given equation is Clearly, Q be (–5, 0) and V be the mid-point of S and Q.
y2 = 12x Then focus S is (–1, 0).
fi 4a = 12 Hence, the equation of the parabola is
fi a = 12/4 = 3
Ê x + 5ˆ
Given focal distance = 6 ( x + 1) 2 + y 2 = Á
\ x+a=6 Ë 12 ˜¯
fi x+3=6 fi (x + 1)2 + y2 = (x + 5)2
fi x=6–3=3 fi y2 = 8x + 24 = 8(x + 3)

CG_04.indd 25 2/6/2017 4:02:08 PM


4.26 Coordinate Geometry Booster

11. Let the equation of the parabola be fi t1t2 + 1 = 0


y = ax2 + bx + c …(i) fi t1t2 = –1
which is passing through (0, 2), (–1. 0) and (1, 6). So which is the required relation.
c = 2, a + c = b, a + b + c = 6 15. Since one extremity of the focal chord is P(at2, 2at),
Solving, we get Êa 2a ˆ
then the other extremity will be Q Á 2 , - ˜ .
a = 1, b = 3 and c = 2 Ët t ¯
Hence, the equation of the parabola is y = x2 + 3x + 2.
Y
12. Let the equation of the parabola be (y – k)2 = 4a(x – h),
where vertex is (h, k). P(at2, 2at)
Then the equation becomes
(y – 2)2 = 4a(x – 1)
which is passing through (3, 4).
X
O S(a, 0)
1
Therefore, 8a = 4 fi a =
2
Hence, the equation of the parabola is Q(at12, 2at1)
(y – 2)2 = 2(x – 1)
13. Let the equation of the parabola be (x – H)2 = 4a(y – k),
Thus, PQ = SP + SQ
where vertex is (h, k).
Êa ˆ
Thus the equation becomes = (at 2 + a ) + Á 2 + a˜
(x – 3)2 = 4a(y – 2) Ët ¯
Also it given that, the length of the latus rectum = 12 2
Ê 1 ˆ Ê 1ˆ
fi 4a = 12 fi a = 3. = a Á t 2 + 2 + 2˜ = a Á t + ˜
Ë t ¯ Ë t¯
Hence, the equation of the parabola is
(x – 3)2 = 12(y – 1). 2
2 16. Now, slope of PQ = = tan q
14. Let y = 4ax be a parabola, if PQ be a focal chord. 1
t-
Y t
1
P fi 2 cot q = t -
t
2
Ê 1ˆ
X Thus, PQ = a Á t + ˜
O S Ë t¯
Q ÈÊ 1ˆ 2 ˘
= a ÍÁ t - ˜ + 4˙
ÎË t¯ ˚

Consider any two points on the parabola (at12, 2at1) and = a(4 cot2 q + 4)
(at22, 2at2). = 4a cosec2 q
Êa 2a ˆ
Since PQ passes through the focus S(a, 0), so P, S, Q 17. S = (a, 0), P = (at 2 , 2at ) and Q = Á 2 , - ˜
are collinear. Ët t ¯
Thus, m(PS) = m(QS) a
Thus, SP = a + at 2 , SQ = a +
2at1 - 0 0 - 2at2 t2
fi =
at12 - a a - at22 Now, Harmonic mean of SP and SQ
2SP ◊ SQ 2
2at1 2at = =
fi = 2 2 SP + SQ 1 1
2 +
at1 - a at2 - a SP SQ
2t1 2t2 1 1 1 1 1 + t2 1
fi = = + = + = =
SP SQ a + at 2 a 2
t12 - 1 t22 - 1 a + 2 a (1 + t ) a
t
t1 t 2 2
fi = 2 Thus, = = 2a
t12 - 1 t22 - 1 1 1 1
+
fi t1(t22 – 1) = t2(t12 – 1) SP SQ a
fi t1t2(t2 – t1) + (t2 – t1) = 0 = semi-latus rectum.

CG_04.indd 26 2/6/2017 4:02:09 PM


Parabola 4.27

18. 21. Since, the point (l, –l) lies inside of the parabola
Y y2 = 4x, then l2 – 4l < 0
fi l(l – 4) < 0
P(at2, 2at) fi 0<l<4
Hence, the range of l is (0, 4).
22. Since the point (l, 2) is an exterior point of both the
parabolas
X Y
O S(a, 0)
P(l, 2)
Q(at12, 2at1)
X
O

The equation of the focal chord SP:


2at - 0
y-0= 2 ( x - a)
at - a y2 = (x + 1) and y2 = –(x – 1),
fi y(t2 – 1) = 2tx – 2at So we have
4 – x – 1 > 0 and 4 + x – 1 > 0
fi 2tx – (t2 – 1)y – 2at = 0 fi 3 – x > 0 and 3 + x > 0
Let d be the distance of the focal chord SP from the fi x – 3 < 0 and x + 3 > 0
vertex (0, 0) to the parabola y2 = 4ax. fi x < 3 and x > –3
fi –3 < x < 3
(0 - 0 - 2at ) 23. The given line is
Then d =
4t 2 + (t 2 - 1) 2 2x + 3y + 5 = 0 …(i)
and the parabola is y2 = 8x …(ii)
2at 2a Since (i) is a tangent to the parabola y2 = 8x, so
= =
(t 2 + 1) Ê t + 1ˆ 2
ÁË ˜ Ê - 2x - 5ˆ
t¯ ÁË ˜ = 8x
3 ¯
2
Ê 1ˆ 4a 2 4a 3 fi (2x + 5)2 = 72x
Also, PQ = a Á t + ˜ = a ¥ 2 = 2 fi 4x2 + 20x + 25 = 72x
Ë t¯ d d
fi 4x2 – 52x + 25 = 0
1 fi 4x2 – 2x – 50x + 25 = 0
Thus, PQ a
d2 fi 2x(2x – 1) – 25(2x – 1) = 0
Hence the length of the focal chord varies inversely as fi (2x – 1)(2x – 25) = 0
the square of its distance from the vertex of the given fi x = 1/2, x = 25/2
parabola. Ê -1 - 5 ˆ
When x = 1/2, then y = Á = -2
19. Let the circle described on the focal chord SP, where Ë 3 ˜¯
S = (a, 0) and P = (at2, 2at). Also, when x = 25/2, then y = –10
The equation of the circle is Hence, the points of contact are
(x – at2)(x – a) + (y – 2at)(y – 0) – 0 (1/2, –2) or (25/2, –10)
Solving it with y-axis, x = 0, we have 24. The given parabola is
y2 – 2aty + a2t2 = 0 y2 = 12x …(i)
Clearly, it has equal roots. fi 4a = 12
So the circle touches the y-axis. fi a=3
Also, the point of contact is (0, at). The given line is 3x + 4y + l = 0
20. The equation of the circle described on AB as diameter 3 l
fi y=- x- …(ii)
is 4 4
Ê aˆ Ê 2a ˆ Since, the line (ii) is a tangent to the parabola (i), so
( x - at 2 ) Á x - 2 ˜ + ( y - 2at ) Á y + ˜ = 0 a
Ë t ¯ Ë t ¯ c=
Put x = a, we have m
2 l 3
Ê 1ˆ Ê 1ˆ fi - = = -4
y 2 - 2a Á t - ˜ y + a 2 Á t - ˜ = 0 4 Ê 3ˆ
Ë t ¯ Ë t¯ -
ÁË ˜¯
4
Clearly, it has equal roots. fi l = 16
Hence the circle touches the directrix at x = –a. Hence, the value of l is 16.

CG_04.indd 27 2/6/2017 4:02:09 PM


4.28 Coordinate Geometry Booster

25. Let the equation of the parabola be fi y = 2, 3


y2 = 4ax Thus, the points are (1, 2) and (1, 3).
and the line be y = mx + c. Hence, the equations of tangents can be at (1, 2) and
Solving the above equations, we get (1, 3) be
(mx + c)2 = 4ax 5
2y = ( x + 1) + ( y + 2) - 8
fi m2x2 + (2mc + 4a)x + c2 = 0 2
Let the line y = mx + c intersects the parabola in two 5
and 3y = ( x + 1) + ( y + 3) - 8
real and distinct points, say (x1, y1) and (x2, y2). 2
Thus (x1 – x2)2 = (x1 + x2)2 – 4x1x2 fi 2x + y – 4 = 0 and 2x – y + 1 = 0
28. Let the equation of the tangent be
4(mc - 2a) 2 4c 2 16a(a - mc)
= - 2 = , y = 2x + c …(i)
m4 m m4 If the equation (i) be a tangent to the parabola, then
and y1 – y2 = m(x1 – x2) a 2
Thus, the required length c = = = 1.
m 2
= ( y1 - y2 ) 2 + ( x1 - x2 ) 2 Thus, the equation of the tangent is
y = 2x + 1 …(ii)
= ( x1 - x2 ) 2 m 2 + ( x1 - x2 ) 2 The given parabola is y2 = 8x …(iii)
Solving (ii) and (iii), we get
= 1 + m 2 ( x1 - x2 )
(2x + 1)2 = 8x
4 fi 4x2 + 4x + 1 = 8x
= 2
1 + m 2 a (a - mc)
m fi (2x – 1)2 = 0
26. Let the parabola be y2 = 4ax and the two points on the 1
fi x=
parabola are P (at12 , 2at1 ) and Q(at22 , 2at2 ) , respec- 2
tively. When x = 1/2, then y = ± 2
Hence, the point of contacts are
P(t1)
Ê1 ˆ Ê1 ˆ
ÁË , 2˜¯ or ÁË , - 2˜¯
2 2
R(at1, at2) a(t1, t2) 29. The equation of line from (–1, 2) is
(y – 2) = m(x + 1)
fi mx – y + (m + 2) = 0
Q(t2) fi y = mx + (m + 2) …(i)
The line (i) will be a tangent to the parabola y2 = 4x, if
The equation of the tangent at P (at12 , 2at1 ) and 1
(m + 2) =
Q(at22 , 2at2 ) are m
fi m2 + 2m – 1 = 0
t1 y = x + at12 …(i) which is a quadratic in m.
and t2 y = x + at2 …(ii) Let its roots are m1, m2.
Thus, m1 + m2 = –2 and m1m2 = –1
Solving these equations, we get
Let q be the angle between them.
x = at1t2, y = a(t1 + t2)
Hence the co-ordinates of the point of intersection of m2 - m1
Then, tan (q ) =
the tangents are (at1t2, a(t1 + t2)). 1 + m1m2
Notes:
(m1 + m2 ) 2 - 4m1m2
1. x-co-ordinate is the geometric mean of the x- =
co-ordinates of P and Q. 1 + m1m2
2. y-co-ordinate is the arithmetic mean of the y-
p
co-ordinates of P and Q. fi tan (q) = = tan
2
27. The given parabola is y2 = 2x + 5y – 8. p
fi q=
when x = 1, y2 = 5y – 6 2
fi y2 – 5y + 6 = 0 p
Hence, the angle between the tangents is q = .
fi (y – 2)(y – 3) 2

CG_04.indd 28 2/6/2017 4:02:10 PM


Parabola 4.29

30. Let the equation of the tangent be fi mx2 = 5m2x + 4ab


(y + 1) = m(x – 1) fi mx2 – 4bm2x – 4ab = 0
fi y = mx – (m + 1) …(i) Since it has equal roots, so
Equation (i) be a tangent to the parabola D=0
y = x2 – 3x + 2, 16b2m4 + 16abm = 0
then mx – m – 1 = x2 – 3x + 2 fi 16bm(bm 3 + a) = 0
fi x2 – (m + 3)x + (m + 3) = 0
a
Since it has equal roots, so fi m3 = -
D=0 b
fi (m + 3)2 – 4(m + 3) = 0 a1/3
fi (m + 3)(m + 3 – 4) = 0 fi m=-
b1/3
fi (m + 3)(m – 1) = 0
fi m = 1, –3 Hence, the equation of the common tangent be
Hence, the equation of the tangents are Ê a1/3 ˆ Ê b1/3 ˆ
y = x – 2 and y = –3x + 2 y = Á - 1/3 ˜ x + a Á - 1/3 ˜
Ë b ¯ Ë a ¯
31. The given parabolas are
33. Let the equation of the tangent to the parabola
y2 = 4ax and x2 = 4ay
a y2 = 16x is
Let the equation of the tangent be y = mx +
. 4
m y = mx +
If it is a tangent to the parabola x2 = 4ay, then m
Ê aˆ fi m2x – my + 4 = 0 …(i)
x 2 = 4a Á mx + ˜
Ë m¯ Y
fi mx2 + 4am2x + 4a2 M
fi mx2 – 4am2x – 4a2 = 0
Now D = 0 gives,
16a2m2 + 16a2m = 0 O
X
fi 16a2m (m3 + 1) = 0
fi m(m3 + 1) = 0
fi m = 0 and –1
Since m = 0 will not satisfy the given tangent, so
If the Eq. (i) be a tangent to the circle x2 + y2 = 8, the
m = –1
length of the perpendicular from the centre to the tan-
Hence, the equation of the common tangent be
gent is equal to the radius of the circle.
y = –x – a
fi x+y+a=0 Therefore,
32. The given parabolas are y2 = 4ax and x2 = 4by. 0-0+4
=2 2
Y m4 + m2
fi 8(m4 + m2) = 16
fi (m4 + m2 – 2) = 0
P fi (m4 + 2m2 – m2 – 2) = 0
X fi (m2 + 2)(m2 – 1) = 0
O fi (m2 – 1) = 0
Q fi m = ±1
Hence, the common tangents are y = ± x ± 4.
34. Any point on the parabola y = x2 is (t, t2).
Let the equation of the tangent be Now the tangent at (t, t2) is
a 1
y = mx + …(i) xx1 = ( y + y1 )
m 2
Since the equation (i) is also a tangent to the parabola 1
fi tx = ( y + t 2 )
x2 = 4by, so 2
Ê aˆ fi 2tx – y – t2 = 0
x 2 = 4b Á mx + ˜
Ë m¯

CG_04.indd 29 2/6/2017 4:02:10 PM


4.30 Coordinate Geometry Booster

If it is a tangent to the parabola, 37. Any tangent to the parabola y2 = 4x is


y = – (x – 2)2, then a
y = mx +
2tx – t2 = – (x – 2)2 m
fi 2tx – t2 = –x2 + 4x – 4 1
fi y = mx +
fi x2 + 2(2 – t)x + (t2 – 4) = 0 m
fi m x – my + 1 = 0
2
…(i)
Since it has equal roots,
If it is a tangent to the circle x2 + (y – 3)2 = 9 the length
D=0
of the perpendicular from the centre to the tangent is
4(2 – t)2 – 4(t2 – 4) = 0 equal to the radius of the circle.
fi (2 – t)2 – (t2 – 4) = 0 Therefore,
fi t=2 3m 2 + 1
=3
Hence, the equation of the common tangent is m4 + m2
y = 4x – 4
fi (3m2 + 1)2 = 9(m4 + m2)
35. Let the equation of the tangent to the parabola y2 = 8x is fi (9m4 + 6m2 + 1) = 9(m4 + m2)
2 fi 3m2 = 1
y = mx + …(i)
m
Ê 1 ˆ
If it is a tangent to the curve xy = –1, then fi m= ±Á ˜
Ë 3¯
Ê 2ˆ
x Á mx + ˜ = -1 Since, the tangent touches the parabola above x-axis,
Ë m¯ so it will make an acute angle with x-axis, so that m is
fi m2x2 + 2x + m = 0 positive.
Since it has equal roots, so, 1
Thus m = .
D=0 3
fi 4 – 4m3 = 0 Hence, the common tangent is x - 3 y + 3 = 0 .
fi m3 = 1 38. The equation of the given parabola is y2 = 4x.
fi m=1 We have, 4a = 4 fi a = 1
Let the end-points of the latus rectum are L(a, 2a) and
Hence, the equation of the common tangent is y = x + 2.
L¢(a, –2a).
36. Any point on the parabola y2 = x can be considered as Therefore L = (1, 2) and L¢ = (1, –2).
(t2, t). As we know that the point of intersection to the tangents
The equation of the tangent to the parabola y2 = x at 2
at (at12 , 2at1 ) and (at2 , 2at2 ) to the parabola y2 = 4ax is
(t2, t) is
1 Ê Ê t1 + t2 ˆ ˆ
yy1 = ( x + x1 ). ÁË at1t2 , a ÁË 2 ˜¯ ˜¯
2
Thus, the point of intersection of the tangents at L(1, 2)
1
fi yt = (x + t2 ) and L¢(1, –2) is (1, 0).
2 39. The equation of the tangent to the parabola
fi x + 2yt – t2 = 0 …(i) y2 = 4x …(i)
2 2
If it is a tangent to the circle x + y – 6y + 4 = 0, then from (1, 4) is
(2yt – t2)2 + y2 – 6y + 4 = 0 y – 4 = m(x – 1)
fi 4y2t2 + t4 – 4yt3 + y2 – 6y + 4 = 0 fi y = mx + (4 – m) …(ii)
2
fi (4t2 + 1)y2 – 2(2t3 + 3)y + (t4 + 4) = 0 Since (ii) is a tangent to the parabola y = 4x, so
Since it has equal roots, so a
c=
D=0 m
fi 4(2t3 + 1)2 – 4(4t2 + 1)(t4 + 4) = 0 1
fi (4 - m) =
fi (2t3 + 1)2 – (4t2 + 1)(t4 + 4) = 0 m
fi t4 – 12t3 + 16t2 – 5 = 0 fi 4m – m2 – 1 = 0
fi m2 – 4m + 1 = 0
fi t=1
It has two roots, say m1 and m2.
Hence, the equation of the common tangent is
Therefore, m1 + m2 = 4 and m1m2 = 1
x + 2y = 1.

CG_04.indd 30 2/6/2017 4:02:10 PM


Parabola 4.31

Let q be the angle between the tangents When y = 0, then x = 0.


m2 - m1 Thus, the point on the parabola is (0, 0).
Then tan (q ) = Hence, the required shortest distance
1 + m1m2
0+0-4 4
= = =2 2
(m1 + m2 ) 2 - 4m1m2 1+1 2
=
1 + m1m2 42. The given tangents are y + b = m1(x + a)
and y + b = m2(x + a)
12 p Therefore, both the tangents pass through (–a, –b)
= = 3 = tan
2 3 which is a point lying on the directrix of the parabola.
p Thus, the angle between them is 90°.
fi q=
3 Hence, the value of m1m2 is –1.
p 43. The equation of the tangent to the curve y = x2 + 6 at
Hence, the angle between the tangents is .
3 (1, 7) is
40. The shortest distance between a line and the parabola 1
means the shortest distance between a line and a tan- ( y + y1 ) = xx1 + 6
2
gent to the parabola parallel to the given line.
1
Y fi ( y + 7) = x + 6
2
fi 2x – y – 5 = 0 …(i)
Y

X¢ P O
X
P(1, 7)
Q
X¢ X
Y¢ Q O

Thus, the slopes of the tangent and the line will be the
same.
Therefore,

2x + 3 = 1
The given circle is
fi x = –1
When x = –1, then y = 0. ( x + 8) 2 + ( y + 6) 2 = ( 100 - c ) 2 …(ii)
Hence, the point on the parabola is (–1, 0). If the line (i) be a tangent to the circle (ii), the length of
Thus, the required shortest distance the perpendicular from the centre of the circle is equal
-1 - 0 - 2 3 to the radius of the circle.
= =
1+1 2 -16 + 6 - 5
Therefore, = 100 - c
41. The shortest distance between a line and the parabola 22 + 1
means the shortest distance between a line and a tan-
gent to the parabola parallel to the given line. fi 45 = 100 – c
Y fi c = 100 – 45 = 5
Thus, the equation of the circle is
(x + 8)2 + (y + 6)2 = 45
Q
fi x2 + y2 + 16x + 12y + 55 = 0 …(iii)
Solving Eqs (i) and (iii), we get
X x2 + (2x – 5)2 + 16x + 12(2x – 5) + 55 = 0
O
P fi x2 + 4x2 + 25 – 20x + 16x
+ 24x – 60 + 55 = 0
fi 5x2 + 20x + 20 = 0
fi x2 + 4x + 4 = 0
fi (x + 2)2 = 0
Thus, the slopes of the tangent and the line will be the fi x = –2
same.
4 When x = –2, then y = 2x – 5 = –4 – 5 = –9.
Therefore, - = -1 fi y = 0 Hence, the point Q is (–2, –9).
2y + 4

CG_04.indd 31 2/6/2017 4:02:11 PM


4.32 Coordinate Geometry Booster

44. Any tangent to y2 = 4(x + a) is Since it passes through the above three points, we have
a a 2t22t32 + a 2 (t2 + t3 ) 2 + 2gat2t3 + 2fa(t2 + t3) + c = 0
y = m1 ( x + a ) + …(i)
m1 …(ii)
Also, any tangent to y2 = 4b(x + b) is a 2t12t32 + a 2 (t1 + t3 ) 2 + 2gat1t3 + 2fa(t1 + t3) + c = 0
b …(iii)
y = m2 ( x + b) + …(ii) and
m2
a 2t12t22 + a 2 (t1 + t2 ) 2 + 2gat1t2 + 2fa(t1 + t2) + c = 0
Since, two tangents are perpendicular, so …(iv)
m1m2 = –1
Subtracting Eq. (iii) from Eq. (ii) and dividing by a(t2
1 – t1), we get
fi m2 = -
m1
a (t32 (t1 + t2 ) + t1 + t2 + 2t3 ) + 2gt3 + 2f = 0
From Eq. (ii), we get
similarly from Eqs (iii) and (iv), we get
1
y = - ( x + b) - bm1 …(iii)
m1 a (t12 (t3 + t2 ) + t3 + t2 + 2t1 ) + 2gt1 + 2f = 0
Now subtracting Eq. (i) and Eq. (iii), we get From these two equations, we get
1 a 2g = – a(1 + t2t3 + t3t1 + t1t2),
m1 ( x + a ) + ( x + b) + + bm1 = 0
m1 m1 2f = – a(t1 + t2 + t3 – t1t2t3)
Ê 1ˆ Ê 1ˆ Ê 1ˆ Substituting these values of g and f in Eq. (ii), we get
fi ÁË m1 + m ˜¯ x + ÁË m1 + m ˜¯ a + ÁË m1 + m ˜¯ b = 0
1 1 1 c = a2(t2t3 + t3t1 + t1t2).
fi x+a+b=0 Thus, the equation of the circle is
Hence, the result. x2 + y2 – a(1 + t2t3 + t3t1 + t1t2)x
45. Let the three points of the parabola be
– a(t1 + t2 + t3 – t1t2t3)y
P(at12 , 2at1 ), Q(at22 , 2at2 ) and R (at32 , 2at3 )
+ a2 (t2t3 + t3t1 + t1t2) = 0
and the points of intersection of the tangents at these
points are A(t2 t3, a(t2 + t3)), B(t1t3, a(t1 + t3)) and 47. Let the equations of the three tangents be
A(t1t2, a(t1 + t2) a
y = m1 x + …(i)
Now, m1
at12 2at1 1 a
y = m2 x + …(ii)
1 2 m2
ar(DPQR) = at2 2at2 1 a
2 and y = m3 x + …(iii)
at32 2at3 1 m3
= a2(t1 – t2)(t2 – t3)(t3 – 1) The point of intersection of (ii) and (iii) is
Also, Ê a Ê 1 1 ˆˆ
at2t3 a (t2 + t3 ) 1 Á m m , a ÁË m + m ˜¯ ˜
1 Ë 2 3 2 3 ¯
ar(DABC) = at3t1 a (t3 + t1 ) 1
2 The equation of any line perpendicular to (i) and
at1t2 a (t1 + t2 ) 1 Y
1
= a 2 (t1 - t2 )(t2 - t3 )(t3 - t1 )
2 M P(x1, y1)
Hence, the result.
46. Let P, Q and R be the points on the parabola y2 = 4ax at T O N Q
X
which tangents are drawn and let their co-ordinates be
P(at12 , 2at1 ), Q(at22 , 2at2 ) and R (at32 , 2at3 ).
The tangents at Q and R intersect at the point The equation of any tangent to the parabola at (x1, y1) is
A[at2t3, a(t2 + t3)] (y – y1) = m(x – x1),
Similarly, the other pairs of tangents at the points Ê dy ˆ
where m = Á ˜
B[at1t3, a(t1 + t3)] and C[at1t2, a(t1 + t2)] Ë dx ¯ ( x , y )
1 1
Let the equation to the circle be
passes through the point of intersection of tangents
x2 + y2 + 2gx + 2fy + c = 0 …(i) (ii) and (iii) is

CG_04.indd 32 2/6/2017 4:02:11 PM


Parabola 4.33

Ê 1 1ˆ 1 Ê a ˆ We have,
y - aÁ + ˜ = - Áx -
Ë m2 m3 ¯ m1 Ë m2 m3 ˜¯ 4a = 1
1
x È 1 1 1 ˘ fi a=
i.e. y+ = aÍ + + ˙ …(iv) 4
m1 Î m2 m3 m1m2 m3 ˚ Hence, the equation of the director circle is
Similarly the equation to the straight line through the X+a=0
point of intersection of (iii) and (i) and perpendicular 1
to (ii) is fi x+2+ =0
4
x È 1 1 1 ˘
y+ = aÍ + + …(v) fi 4x + 9 = 0
m2 m m m ˙
Î 3 1 1 2 m3 ˚
m (ii) The given parabola is
and the equation of the straight line through the point of x2 = 4x + 4y
intersection of (i) and (ii) and perpendicular to (iii) is fi (x2 – 4x + 4) = 4y + 4 = 4(y + 1)
x fi (x – 2)2 = 4(y + 1)
È1 1 1 ˘
y+ = aÍ + + ˙ …(vi) fi X2 = 4Y,
m3 Î m1 m2 m1m2 m3 ˚ where X = x – 2
The point which is common to the straight lines (iv), and Y=y+1
(v) and (vi), i.e. the orthocentre of the triangle is We have, 4a = 4
Ê Ê 1 1 1 1 ˆˆ fi a=1
Á - a, a ÁË m + m + m + m m m ˜¯ ˜ Hence, the equation of the director circle is
Ë 1 2 3 1 2 3 ¯ Y+a=0
Hence, the point lies on the directrix. fi y+1+1=0
48. fi y+2=0
Y (iii) The given parabola is
y2 = 4x + 4y – 8
fi y2 – 4y + 4 = 4x – 8 + 4
P(at12, 2at1)
fi (y – 2)2 = 4x – 4 = 4(x – 1)
fi Y 2 = 4X,
where X = (x – 1)
X
O and Y = (y – 2)
We have 4a = 4 fi a = 1
Hence, the equation of the director circle is
x+a=0
Q(at22, 2at2) X+a=0
fi x–1+1=0
2 fi x=0
The equations of tangents at P and Q are yt1 = x + at1
50. Let the parabola be y2 = 4ax
and yt2 = x + at2. P
and two points on the pa-
The point of intersection of these tangents is 2
rabola are P (at1 , 2at1 ) and
[at1t2, a(t1 + t2)]
Q(at22 , 2at2 ) . A
Let this point be (h, k).
The equation of the nor-
1 1
The slope of the tangents are m1 = and m2 = . mals at P (at12 , 2at1 ) and Q
t1 t2
Q(at22 , 2at2 ) are
Since these two tangents are perpendicular, so
m1m2 = –a y = - t1 x + 2at1 + at13 …(i)
and
1 1 y = - t2 x + 2at2 + at23 …(ii)
fi ◊ = -1
t1 t2
Solving Eqs (i) and (ii), we get
fi t1 ◊ t2 = –1
x = 2a + a (t12 + t22 + t1t2 )
fi h = –a P
Thus the locus of the points of intersection is x + a = 0 and y = –at2t2(t1 + t2).
which is the directrix of the parabola y2 = 4ax. 51. Let the parabola be y2 = 4ax
49. (i) The given parabola is and the two points on the pa-
y2 = x + 2 rabola are P (at12 , 2at1 ) and
fi Y 2 = X, Q(at22 , 2at2 ) .
Q
where X = x + 2 and Y = y

CG_04.indd 33 2/6/2017 4:02:12 PM


4.34 Coordinate Geometry Booster

The equation of the normal to the parabola at The line (i) will be a normal to the given parabola, if
k = –2am – am3 = 6 + 3 = 9.
P (at12 ,
2at1 ) is y = - t1 x + 2at1 + at13 which meets the
Hence, the value of k is 9.
parabola again at Q(at22 , 2at2 ) .
57. The equation of the parabola is y2 = 8x.
Thus, 2at2 = - at1t22 + 2at1 + at13 We have, 4a = 8 fi a = 2
fi 2a (t2 - t1 ) + at1 (t22 - t12 ) = 0 Y
fi (t2 – t1)[2a + at1(t2 + t1)] = 0
P
fi [2 + t1(t2 + t1)] = 0
fi t12 + t1t2 + 2 = 0
2 X
fi t2 = - t1 - O
t1
which is the required condition.
52. As we know that if the normal at t1 meets the parabola Q
again at t2, then The equation of the normal to the given parabola at
2 P(8, 12) is
t2 = - t1 - y
t1 y - y1 = - 1 ( x - x1 )
2 2a
Ê 2ˆ
fi t22 = Á - t1 - ˜ 12
Ë t1 ¯ fi y - 12 = - ( x - 18)
4
4
= t12 + + 4≥4+4=8 fi y – 12 = –3x + 54
t12 fi 3x + y = 66
Hence, the result. Q Solving y = 66 – 3x and the parabola y2 = 8x, we get
53. Since the normal at t1
44 242
meets the parabola at P x=- and y =
2 3 9
t3, so t3 = - t1 - . 44 242 ˆ
t1 Hence, the co-ordinates of Q is ÊÁ - , .
Ë 3 9 ˜¯
2
Similarly, t3 = - t2 -
t2 R 242
2 2
Thus, PQ = ÊÁ ˆ Ê 44 ˆ
- 18˜ + Á - - 12˜
2 2 Ë 9 ¯ Ë 3 ¯
Thus, - t1 - = - t2 -
t1 t2 6400 6400
= +
Ê 2 2 ˆ 2(t - t ) 81 9
fi (t1 - t2 ) = Á - ˜ = 1 2
Ë t1 t2 ¯ t1t2 1 1
= 80 +
81 9
fi t1t2 = 2
80
54. The equation of the normal is = ¥ 10
9
y
y - y1 = - 1 ( x - x1 ) , where a = 1 fi 9PQ = 80 10
2a
58. The equations of normals at P(t1) and Q(t2) are
2
fi y - 2 = - ( x - 1) = - x + 1 y = –t1x + 2at1 + at13
2
and y = –t2x + 2at2 + at23
fi x+y=3
55. The equation of the normal to the parabola y2 = 4ax is
P
y = mx – 2am – am3
Here, a = 2 and m = 2.
R
Therefore, y = 2x – 8 – 16 = 2x – 24 X
O
Hence, the equation of the normal is y = 2 x – 24.
56. The given parabola is y2 = 12x. Q
We have, 4a = 12 fi a = 3.
The given line x + y = k fi y = –x + k …(i) Since these two normals are at right angles, so t1t2 = –1.

CG_04.indd 34 2/6/2017 4:02:13 PM


Parabola 4.35

Let M(h, k) be the point of intersection of two normals. 62. If the normal at P(t1) meets the parabola at Q(t2), then
Then, h = 2a + a(t12 + t1t2 + t22 ) 2
t2 = - t1 - …(i)
and k = – at1t2(t1 + t2) t1
fi h = 2a + a {(t1 + t2)2 – 2t1t2} Y
and k = –at1t2(t1 + t2)
fi h = 2a + a{(t1 + t2)2 + 2} P
and k = a(t1 + t2)
Eliminating t1 and t2, we get,
k2 = a(h – 3a) O
X
Hence, the locus of M(h, k) is y2 = a(x – 3a).
59. The given line is
lx + my = 0 Q
fi my = –lx – n Since the normal chord subtends an angle of 90° at the
Ê lˆ Ê nˆ vertex, then
fi y = Á- ˜ x + Á- ˜
Ë m¯ Ë m¯ t1t2 = –4
As we know that the line y = mx + c will be a normal to From Eq. (i), we get
the parabola y2 = 4ax if t12 + t1t2 + 2 = 0
c = –2am – am3
3
fi t12 – 4 + 2 = 0
Ê nˆ Ê lˆ Ê lˆ fi t12 – 2 = 0
fi ÁË - ˜¯ = - 2a Á - ˜ - a Á - ˜
m Ë m ¯ Ë m¯ 2
Ê 2ˆ
fi al + 2alm + nm = 0
3 2 2 Also, t22 = Á -t1 - ˜
Ë t1 ¯
Hence, the result.
4
60. The equation of the parabola is y2 = 4ax. = t12 + +4
If the normal at P(t1) meets the parabola again at Q(t2), t12
then =2+2+4=8
2 Therefore,
t2 = - t1 -
t1 PQ2 = a2(t12 – t22)2 + 4a2(t1 – t2)2
fi t1t2 = –t12 – 2 = 1 ◊ (2 - 8) 2 + 4( 2 + 2 2) 2
fi t12 + t1t2 + 2 = 0 …(i) = 36 + 72 = 108
The chord joining t1, t2 subtends a right angle at the fi PQ = 6 3
vertex, so the product of their slopes = –1 63. The equation of the parabola is y2 = 4ax.
2 2 Y
fi ◊ = -1
t1 t2
fi t1t2 = –4 …(ii) P
From Eqs (i) and (ii), we get
t12 – 4 + 2 = 0 X
O
fi t12 = 2
fi t1 = 2
fi tan q = 2 Q
fi q = tan -1 ( 2) Let the normal chord be PQ, where P(t1) and Q(t2).
61. The given equation of the parabola is y2 = 4x. Since the abscissa and ordinate of the point (p, p) are
We have 4a = 4 fi a = 1. same, then
The equation of the normal to the parabola 2at1 = at12
y2 = 4x at (am2, –2am) is fi t1 = 2
y = mx – 2am – am3 = mx – 2m – m3 If the normal at P(t1) meets the parabola Q(t2), then
Since, the normal makes equal angles with the axes, so 2
m = ±1 t2 = - t1 -
t1
Thus, the points are (m2, –2m) = (1, 2)
fi t2 = –2 – 1 = –3

CG_04.indd 35 2/6/2017 4:02:13 PM


4.36 Coordinate Geometry Booster

Let S(a, 0) be the focus of the parabola y2 = 4ax. fi m3 – 4m = 0


2at 4 4 fi m = 0, –2, 2
Then the slope of SP = 2 1 = =
at1 - a 4 - 1 3 Therefore, P is (4, –4) or (4, 4) and let C(6, 0) be the
centre of the circle and Q be a point on the circle.
2at2
and the slope of SQ = Therefore,
at22 - a
CP = (6 - 4) 2 + (4 - 0) 2 = 20 = 2 5
—6 —6 3
= = =-
9 -1 8 4 and CQ = 5
It is clear that Thus, the shortest distance = CP – CQ
4 3
m( SP ) ¥ m(SQ) = ¥ - = -1 = 2 5- 5= 5
3 4 66. The given equation of the parabola is y2 = 8x.
Hence, the result.
We have, 4a = 8 fi a = 2.
64. The given equation of the parabola is y2 = 4ax.
The equation of the normal at P(am2, –2am) is Y
y = mx – 2am – am3 …(i)
Let Q be a point on the axis of the parabola.
Put y = 0 in Eq. (i), we get X
O
x = 2a + am2 P
Hence, the co-ordinates of the point Q is (2a + am2, 0).
Let M(h, k) be the mid-point of the normal PQ. Q
am 2 + 2a + am 2
Then, h = O
2
2am
and k = - = - am
2 The equation of the normal to the parabola
Eliminating m, we get
y2 = 8x at P(4m2, –4m) is
k2
h=a+ y = mx – 4m – 2m3 …(i)
a The given equation of the circle is
fi a2 + k2 = ah x2 + y2 + 12y + 35 = 0
Hence, the locus of M(h, k) is fi x2 + (y + 6)2 = 1
a2 + y2 = ax Thus, the centre of the circle is C(0, –6).
fi y2 = a(x – a) As we know that the shortest distance between a circle
65. The given equation of the parabola is y2 = 4x. and the parabola lies along the common normal.
The equation of the normal at P(m2, –2m) to the
Therefore, the normal always passes through the centre
parabola y2 = 4x is
of the circle. So
y = mx – 2m – m3 …(i) –6 = –4m – 2m3
The given equation of the circle is fi m3 + 2m – 3 = 0
x2 + y2 – 12x + 31 = 0 fi m=1
fi (x – 6)2 + y2 = 5 …(ii)
Thus, the point P is (4, –4).
Y Let Q be any point on the circle.
Then CQ = 1 and
P
Q
CP = (4 - 0) 2 + ( - 4 + 6) 2 = 20 = 2 5
Hence, the shortest distance = PQ
X
O C = CP – CQ = 2 5 - 1 .
67. The equation of any normal to a parabola y2 = 4ax is
y = mx – 2am – am3
which meets at a point, say (h, k).
The shortest distance between the parabola and the Thus, am3 + (2a – h)m + k = 0.
circle lies along the common normal. which is a cubic equation in m.
Therefore, the centre of a circle passes through the nor- So it has three roots, say m1, m2 and m3.
mal, so we have Therefore, m1 + m2 + m3 = 0
0 = 6m – 2m – m3 Hence the result.

CG_04.indd 36 2/6/2017 4:02:14 PM


Parabola 4.37

68. Let the ordinates of A, B and C be y1, y2 and y3 respec-


4(h - 2a )3
tively. fi k2 - <0
27a
Then, y1 = –2am1, y2 = –2am2, y3 = –2am3.
Thus, fi 27ak2 < 4(h – 2a)3
y1 + y2 + y3 = –2am3 – 2am2 – 2am3 Hence, the result.
= –2a(m1 + m2 + m3) = –2a.0 = 0 72. Let the normal at P (at12 , 2at1 ) be y = –t1x + 2at1 + at13
69. If A(x1, y1), B(x2, y2) and C(x3, y3) be the vertices of a Thus slope of the normal = tan q = –t1
DABC, then its centroid is It meets the parabola again at Q(at22, 2at2)
Ê x1 + x2 + x3 y1 + y2 + y3 ˆ 2
ÁË , ˜¯ Then t2 = - t1 - .
3 3 t1
Êx +x +x ˆ Now the angle between the normal and the parabola =
= Á 1 2 3 , 0˜
Ë 3 ¯ angle between the normal and the tangent at Q. If j be
Hence the centroid lies on the axis of the parabola. the angle between them, then
Also, m - m2
tan j = 1
Ê x1 + x2 + x3 ˆ 1 2 2 2 1 + m1 m2
ÁË ˜¯ = (am1 + am2 + am3 )
3 3 1
-t1 -
aÏ Ê 2a - h ˆ ¸ 2h - 4a t2
= Ì0 - 2 Á =
3Ó Ë a ˜¯ ˝˛ 3 =
Ê 1ˆ
1 + (- t1 ) Á ˜
Ê 2h - 4a ˆ Ë t2 ¯
Thus, the centroid of DABC is Á , 0˜ .
Ë 3 ¯ t1t2 + 1
=-
70. When three normals are real, then all the three roots of t2 - t1
am3 + (2a – h)m + k = 0 are real. Ê 2ˆ
t1 Á -t1 - ˜ + 1
Let its three roots are m1, m2, m3. Ë t1 ¯
For any real values of m1, m2, m3, =-
2
- t1 - - t1
m12 + m22 + m32 > 0 t1
fi (m1 + m2 + m3)2 – 2(m1m2 + m2m3 + m3m1) > 0 - t12 - 1
=-
Ê 2a - h ˆ Ê 1 + t12 ˆ
fi 0 - 2Á >0
Ë a ˜¯ - 2Á ˜
Ë t1 ¯
fi h – 2a > 0 t1
fi h > 2a =-
2
71. Let f(m) = am3 + (2a – h)m + k
tan q
fi f ¢(m) = 3am2 + (2a – h) 2a =
2
If f(m) has three distinct roots, so f ¢(m) has two distinct
Ê tan q ˆ
fi j = tan -1 Á
Ë 2 ˜¯
roots.
Let two distinct roots of f ¢(m) = 0 are a and b.
73. The equation of the normal to the parabola y2 = 4ax at
Ê h — 2a ˆ Ê h — 2a ˆ
Thus, a = Á and b = - Á P(at2, 2at) is
Ë 3 ˜¯ Ë 3 ˜¯
.
y = –tx + 2at + at3
Now, f(a)f(b) = 0 Y
f(a) f(–a) = 0
fi (aa3 + (2a – h)a + k)(–aa3 – (2a – h)a + k) < 0 P

fi k – ((aa + (2a – h)) a ) < 0


2 2 2 2
S
2
X
2Ê h - 2a ˆ Ê h - 2a ˆ O
fi k -Á + (2a - h)˜ Á <0
Ë 3 ¯ Ë 3 ˜¯
2 Q
Ê 4a - 2h ˆ Ê h - 2a ˆ
fi k2 - Á ˜ Á ˜ <0
Ë 3 ¯ Ë 3 ¯

CG_04.indd 37 2/6/2017 4:02:15 PM


4.38 Coordinate Geometry Booster

It is given that at2 = 2at fi t = 2. Thus, SP = PM = x1 + a,


Thus, the co-ordinates of P is (4a, 4a) and focus is SG = AG – AS = x1 + 2x – a = x1 + a.
S(a, 0). and ST = AS +AT = a + x1.
Also, the normal chord meets the parabola at some Hence, SP = SG = ST.
point, say Q. Then the co-ordinates of Q is (9a, –6a). 76. The normal at P(t) is y = –tx + 2at + at3.
Now, Y
4a 4
Slope of SP = m1 = =
3a 3
P
-6a 3
and the slope of SQ = m2 = =- .
8a 4
X
4 4 O N G
Thus, m1 ¥ m2 = ¥ - = -1
3 3
Hence, the result.
74. Let the latus rectum be LSL¢, where L = (a, 2a) and
L¢ = (a, –2a). It meets the x-axis at G.
Y Thus the co-ordinates of G be (2a + at2, 0).
Also N is (at2, 0).
L Thus NG = 2a + at2 – at2 = 2a = semi-latus rectum.
M 77. The normal at P(t) is y = –tx + 2at + at3
Y
X
O S P(3a, 0)
B
Z L¢

O S G
Normal at L(a, 2a) is x + y = 3a …(i)
Normal at L¢(a, –2a) is x – y = 3a …(ii)
Clearly, (ii) is perpendicular to (i).
Solving, we get
x = 3a and y = 0 Thus, S is (a, 0), G is (2a + at2, 0) and P is (at2, 2at).
Hence, the point of intersection is (3a, 0). Now, SP = a + x = a + at2 = a(12 + t2)
75. SG = 2a + at2 – a = a + at2 = a(1 + t2) = SP
Y Thus, P and G are equidistant from the focus.
78.
P(x1, y1)
Y
Q
P
X
T O S(a, 0) N G

X
O G

Let P(x1, y1) be any point on the parabola y2 = 4ax.


The equation of any tangent and any normal at P(x1, y1) The normal at P(t) is y = –tx + 2at + at3
are
Thus G is (2a + at2, 0) and P is (at2, 2at).
y - y1 x - x1
yy1 = 2a(x + x1) and =- Now, PG2 = 4a2 + 4a2t2 …(i)
y1 2a
Q is a point on the parabola such that QG is perpen-
Since the tangent and the normal meet its axis at T and dicular to axis so that its ordinate is QG and abscissa is
G, respectively, so the co-ordinates of T and G are (–x1, the same as of G.
0) and (x1 + 2a, 0), respectively. Hence, the point Q is (2a + at2, QG).

CG_04.indd 38 2/6/2017 4:02:15 PM


Parabola 4.39

But Q lies on the parabola y2 = 4ax. Now,


Now, QR = ( at12 - at22 ) 2 + (2at1 - 2at2 ) 2
QG2 = 4a(2a + at2)
= 8a2 + 4a2t2 = |a (t1 - t2 )| (t1 + t2 ) 2 - 4
= (4a2 + 4a2t2) + 4a2
= PG2 + 4a2 Ê k 2 4h ˆ Ê k 2 ˆ
= |a| Á 2 - ˜ Á 2 + 4˜
fi QG – PG2 = 4a2 = constant.
2
Ëa a ¯ Ëa ¯
Hence, the result.
1
79. The equation of the chord of contact of the tangents = ¥ (k 2 - 4ah)(k 2 + 4a 2 )
|a|
from the point (2, 3) to the parabola y2 = 4x is
83. Let tangents are drawn from P(h, k) to the parabola
yy1 = 2(x + x1)
y2 = 4ax, intersects the parabola at Q and R.
fi 3y = 2(x + 2)
fi 2x – 3y + 4 = 0 Q
80. The equation of the chord of contact of the tangents to
the parabola y2 = 12x drawn through the point (–1, 2) is
2y = 6(x – 1) P
fi y = 3x – 3
81. Let the point of intersection of the tangents be R(a, b). R
The equation of the tangent to the parabola at
P (at12 , 2at1 ) and Q(at22 , 2at2 ) are
Then the chord of contact of the tangents to the given
Thus, the slopes of the tangents are parabola is QR.
1 1 Then QR is
m1 = and m2 =
t1 t2 yk = 2a(x + h)
fi 2ax – yk + 2ah = 0
Then the point of intersection of the two tangents be
[at1t2, a(t1 + t2)]. Therefore PM = the length of perpendicular from
Therefore a = at1t2 and b = a(t1 + t2). P(h, k) to QR
Let q be the angle between the two tangents. Then 2ah - k 2 + 2ah
=
1 1 k 2 + 4a 2
-
t1 t2 t -t
tan (q ) = = 2 1 k 2 - 4ah
1 t1t2 + 1 = -
1+ k 2 + 4a 2
t1t2
k 2 - 4ah
fi (1 + t1t2 ) tan (q ) = ( (t1 + t2 ) 2 - 4t1t2 ) =
k 2 + 4a 2
fi (1 + t1t2)2 tan2 q = (t1 + t2)2 – 4t1t2
Thus, the area (DPQR)
Ê aˆ
2
b 2 4a b 2 - 4aa 1
fi 2
ÁË1 + ˜¯ tan q = 2 - = = ◊ QR ◊ PM
a a a a2 2
fi (a + a)2 tan2 q = (b2 – 4aa) 1 1 (k 2 - 4ah)
= ¥ (k 2 - 4ah)(k 2 + 4a 2 ) ¥
2 | a| k 2 + 4a 2
Hence, the locus of (a, b) is
(y2 – 4ax) = (x + a)2 tan2 q (k 2 - 4ah)3/2
= if a > 0
82. The equation of the parabola is y2 = 4ax and the point 2a
(h, k) be P. 84. The equation of the chord of the parabola y2 = 4x,
Let the tangents from P touch the parabola at which is bisected at (2, 3) is
Q(at12 , 2at1 ) and R(at22 , 2at2 ) , then P is the point of T = S1
intersection of the tangents. fi yy1 – 2a(x + x1) = y12 – 4ax1
Therefore, h = at1t2 and k = a(t1 + t2) fi 3y – 4(x + 2) = 9 – 8.2 = 7.
h k fi 3y – 4x – 1 = 0
fi t1t2 = and (t1 + t2 ) =
a a fi 4x – 3y + 1 = 0

CG_04.indd 39 2/6/2017 4:02:16 PM


4.40 Coordinate Geometry Booster

85. Let the equation of the parabola be y2 = 4ax. 88. Let QR be the chord and M(a, b) be the mid-point of it.
The equation of the chord of the parabola, whose Y
mid-point (x1, y1) is A
T = S1.
fi yy1 – 2a(x + x1) = y12 – 4ax1
If it is a focal chord, then it will pass through the focus M
X
O
(a, 0) of the parabola.
Therefore,
B
0. y1 – 2a(x + x1) = y12 – 4ax1
fi y12 = –2a2 – 2ax1 + 4ax1 = 2a(x1 – a) Then the equation of the chord of a parabola
Hence, the locus of (x1, y1) is y2 = 2a(x – a). y2 = 4ax at M(a, b) is
86. Let the equation of the parabola be y2 = 4ax. T = S1
Let VP be any chord of the parabola through the vertex fi yb – 2a (x + a) = b2 – 2aa
and M(h, k) be the mid-pont of it. fi yb – 2ax = b2 – 2aa …(i)
Then, the co-ordinates of P becomes (2h, 2k). Let V(0, 0) be the vertex of the parabola.
Since P lies on the parabola, so The combined equation of VQ and VR, making homo-
geneous by means of (i), we have
(2k)2 = 4a ◊ (2h)
Ê yb - 2ax ˆ
fi 4k2 = 8ah y 2 = 4ax ¥ Á 2
Ë b - 2aa ˜¯
fi k2 = 2ah
Hence, the locus of (h, k) is y2 = 2ax. fi y2(b2 – 2aa) – 4abxy + 8a2x2 = 0
87. Equation of the normal at any point (at2, 2at) of the Since, the chord QR subtends a right angle at the ver-
tex, so we have
parabola y2 = 4ax is
co-oefficient of x2 +co-efficient of y2 = 0
y = –tx + 2at + at3 …(i) fi (b2 – 2aa) + 8a2 = 0
Lep PQ be the normal, whose mid-point is M(a, b). fi b2 = 2a(a – 4a)
Therefore, Hence, the locus of M(a, b) is
T = S1 y2 = 2a(x – 4a)
fi yb – 2a(x + a) = b2 – 3aa 89. Let QR be the chord of the parabola and M(a, b) be its
fi yb = 2a(x + a) = (b2 – 4aa) …(ii) mid-point. Then the equation of the chord QR bisected
at M(b, b) is
Equations (i) and (ii) are identical.
T = S1
1 t 2at + at 3 fi yb – 2a(x + a) = b2 – 4aa
Therefore, = = 2
b —2a b - 2aa fi yb = 2a(x + a) + (b2 – 4aa)
fi yb = 2ax + (b2 – 2aa) …(i)
2a t 2at + at 3
fi t=- and = 2 If the Eq. (i) be a tangent to the parabola y2 = 4bx, then
b —2a b - 2aa
b
c=
From the above two relations, eliminating t, we get m
3
Ê 2a ˆ Ê - 2a ˆ Ê - 2a ˆ Ê b 2 - 2aa ˆ b bb
ÁË - b ˜¯ 2a ÁË b ˜¯ + a ÁË b ˜¯ fi ÁË b ˜ = =
¯ (2a /b ) 2a
=
—2a b 2 - 2aa fi (b – 2a)b2 + 4a2a = 0
2 Hence, the locus of M(a, b) is
Ê Ê - 2a ˆ ˆ
fi (b 2 - 2aa ) = - 2a Á 2a + a Á ˜ (2a – b)y2 = 4a2x
Ë Ë b ˜¯ ¯ 90. Let the mid-point of the chord be M(h, k).
fi b2(b2 – 2aa) = –2a(2ab2 + 4a3) Then the equation of the chord at M(h, k) is
T = S1
fi b2(b2 – 2aa) = –4a2b2 – 8a4)
fi yk – 2a(x + h) = k2 – 4ah
fi b4 – 2a(a – 2a)b2 + 8a4 = 0
which passes through the point (3b, b).
Hence the locus of M(a, b) is Then, bk – 2a(3b + h) = k2 – 4ah.
y4 – 2a(x – 2a)y2 + 8a4 = 0 fi k2 – 2ah – bk + 6ab = 0

CG_04.indd 40 2/6/2017 4:02:16 PM


Parabola 4.41

Hence, the locus of M(h, k) is Now,


y2 – 2ax – by + 6ab = 0 Slope of AB
91. The equation of the given parabola is 2a (t2 - t1 ) 2
y2 = 4ax …(i) = m( AB ) = 2 2
=
a (t2 - t1 ) 1 t2
t +
The equation of the tangent at P (at2, 2at) is
The equation of the diameter is
yt = x + at2 …(ii)
2a
The equation of the directrix of the parabola y= fi y = a (t1 + t2 ) …(i)
m
y2 = 4ax is x + a = 0 …(iii) 2 2
Now the tangents at A = (at1 , 2at1 ) and B = (at2 , 2at2 )
Solving Eqs (ii) and (iii), we get
meet at N[at1t2, a(t1 + t2)].
a (t 2 - 1) Thus, N lies on y = a(t1 + t2).
x = –a and y =
t 94. As we know that all rays of light parallel to x-axis of
Thus, the point on the directrix, say Q, whose co-ordi- the parabola are reflected through the focus of the pa-
Ê a (t 2 - 1) ˆ rabola.
nates are Á - a, ˜¯ . The equation of the given parabola is
Ë t
(y – 4)2 = 8(x + 1)
Let M(h, k) be the mid-point of P and Q. Then
fi Y2 = 8X,
at 2 - a a (t 2 - 1) 2at
h= and k = + where Y = y – 4 and X = x + 1
2 2t 2
Now the focus of the parabola is (a, 0).
2h + a
fi t2 = and 4k2t2 = a2(3t2 – 1)2 Therefore,
a
X = a and Y = 0
Eliminating t, we get
2 fi x + 1 = 2 and y – 4 = 0
Ê 2h + a ˆ Ê Ê 2h + a ˆ ˆ
4k 2 Á ˜ = a2 Á 3Á ˜ -1 fi x = 1 and y = 4
Ë a ¯ Ë Ë a ¯ ˜¯
Hence, the focus is (1, 4).
fi 4k2(2h + a) = a(6h + 3a – a)2 Thus a = 1 and b = 4
fi k2(2h + a) = a(3h + a)2 Now, a + b + 10 = 1 + 4 + 10 = 15.
Hence, the locus of M(h, k) is 95. Let the line y = x + 2 intersects the parabola at P.
y2(2x + a) = a(3x + a)2
Solving the line y = x + 2 and the parabola y2 = 4(x + 2),
92. Let y = mx + c represents the system of parallel chords.
we get
The equation of the diameter to the parabola y2 = 4ax is
the point P is (2, 4).
2a
y= . Now the equation of the tangent to the parabola
m
Ê a 2a ˆ y2 = 4(x + 2) at P(2, 4) is
The diameter meets the parabola y2 = 4ax at Á 2 , ˜ .
Ëm m¯ yy1 = 2(x + x1) + 8
The equation of the tangent to the parabola y2 = 4ax at fi 4y = 2(x + 2) + 8
Ê a 2a ˆ a fi x – 2y + 6 = 0
ÁË 2 , ˜¯ is y = mx + . Let IP be the incident ray, PM be the reflected ray and
m m m
PN be the normal
which is parallel to y = mx + c. As we know that the normal is equally inclined with the
93. incident ray as well as the reflected ray.
A
Now the slope of IP = 1, slope of normal PN = –2 and
let the slope of the reflected ray = m. Then
1+ 2 -2 - m
=
O M
1 - 2 1 - 2m
N
1
fi m=
7
Hence, the equation of the reflected ray is
1
B y - 2 = ( x - 4)
7
Let AB be the chord, where A = (at12 , 2at1 ) and fi 7y – 14 = x – 4
B = (at22 , 2at2 ) . fi x – 7y + 10 = 0

CG_04.indd 41 2/6/2017 4:02:17 PM


4.42 Coordinate Geometry Booster

LEVEL III Now, m(OA) ¥ m(OC) = –1


2at - 2at
fi ¥ = -1
Ê 3 - 5 6 + 6ˆ at 2 at 2
1. Clearly, focus = Á , ˜ = ( -1, 6) 2t - 2t
Ë 2 2 ¯ fi ¥ 2 = -1
t2 t
a2 x2 a2 x fi t =42
2. Given y = + - 2a
3 2 fi t=2
Hence, the co-ordinates of the vertices are O = (0, 0);
fi 6y = 2a2x2 + 3a2x – 12a
A = (4a, 4a); B = (8a, 0), and C = (4a, –4a).
Ê 3 ˆ 5. Let P = (x, y).
fi 2a 2 Á x 2 + x˜ = 6y + 12a = 6( y + 2a )
Ë 2 ¯ Y

Ê 2 3 ˆ 3
fi ÁË x + x˜¯ = 2 ( y + 2a )
2 a P
S(2 2, 2 2)
2
Ê 3ˆ 3 9
fi ÁË x + ˜¯ = 2 ( y + 2a) +
4 a 16 M V( 2, 2)
X¢ X
x+
2 2ˆ
Ê 3ˆ 3 Ê 3a =
x y=
fi ÁË x + ˜¯ = 2 ÁË y + 2a + ˜ y 0
4 a 16 ¯

fi 2 3
X = 2Y, We have,
a
SP = PM
3 3a 2
where X = x + , Y = y + 2a + fi SP2 = PM2
4 16 2
Ê x + yˆ
Let focus = (h, k) fi ( x - 2 2) 2 + ( y - 2 2) 2 = Á
Ë 2 ˜¯
3
X = 0 and Y = 2
4a fi 2[( x - 2 2) 2 + ( y - 2 2) 2 ] = ( x + y ) 2
3 3a 2 3
fi x + = 0 and y + 2a + = fi ( x - y ) 2 = 8 2( x + y - 2 2)
4 16 4a 2
3 3a 2 3 6. Clearly, the point of intersection is (4a, 4a) which satis-
fi h = — , k + 2a + = fies the straight line
4 16 4a 2
2bx + 3cy + 4d = 0
—4ha 2 —4h fi 2b(4a) + 3c(4a) + 4d = 0
fi k + 2a + = 2
16 4a fi 2ab + 3ac + d = 0
fi 16ka2 + 32a3 – 4ha4 + 16h = 0 fi a(2b + 3c) + d = 0
fi 4ka2 + 8a3 – ha4 + 4h = 0 fi a(2b + 3c) = –d
Hence, the locus of (h, k) is fi a2(2b + 3c)2 = d2
4ya2 + 8a3 – xa4 + 4x = 0 2
Êdˆ
3. Clearly, the equation of the parabola is fi (2b + 3c) 2 = Á ˜
y2 = 4(a1 – a)(x – a) Ë a¯
4. Let A = (at2, 2at), C = (at2, – 2at) fi d2 = a2(2b + 3c)2
Y 7. Any tangent to the parabola y2 = 4a(x + a) is
A a
y = m1 ( x + a ) + …(i)
m1


and to the parabola y2 = 4b(x + b) is
X
O B b
y = m2 ( x + b) + …(ii)
m2
C Since two tangents are perpendicular, so
Y¢ m1m2 = –1 …(iii)
and B = (2at2, 0)

CG_04a.indd 42 2/6/2017 4:02:56 PM


Parabola 4.43

b fi x2 – 4x – 1 = 0
Thus, y = m2 ( x + b) + fi ( x - 2) 2 = ( 5) 2
m2
1 fi x=2± 5
y=— ( x + b) - bm1 …(iv)
m1
Thus, y = 4 ± 5
Eliminating m1 from Eqs (i) and (iv), we get
a 1 Therefore, Q = (2 + 5, 4 + 5)
m1 ( x + a ) + = - ( x + b) - bm1
m1 m1 R = (2 - 5, 4 - 5)
and
1
fi m1 ( x + a + b) = - ( x + a + b) Thus, the mid-point of Q and R is (2, 4).
m1 11. Clearly, both the lines pass through (–a, –b) which a
Ê 1ˆ point lying on the directrix of the parabola.
fi ( x + a + b) Á m1 + ˜ = 0 Thus, m1m2 = –1, since tangents drawn from any point
Ë m1 ¯
on the directrix are always mutually perpendicular.
fi (x + a + b) = 0 ( m1 π 0) 12. The equation of any tangent to the parabola y2 = 4x is
fi (x + a + b) = 0 1
Hence, the result. y = mx +
m
8. Let the focal chord be y = mx + c which is passing
through the focus. So fi my = m2x + 1
0 = 4m + c fi m2x – my + 1 = 0 …(i)
fi c = –4m Also (i) is a tangent to the circle (x – 3)2 + y2 = 9
Thus, the focal chord is y = mx – 4m Thus, the length of the perpendicular from the centre to
mx – y – 4m = 0 …(i) the tangent is equal to the radius of the circle.
(i) is tangent to the circle (x – 6)2 + y2 = 2, so 3m 2 + 1
So, =3
6m - 4m m4 + m2
= 2
m2 + 1
fi (3m2 + 1)2 = 9(m4 + m2)
2m fi 9m4 + 6m2 + 1 = 9(m4 + m2)
fi = 2
m2 + 1 fi 3m2 = 1
1
fi 4m2 = 2(m2 + 1) fi m=±
3
fi 2m2 = 2
Hence, the equation of the common tangent which lies
fi m2 = 1
x
fi m = ±1 above x-axis is y = + 3
9. The co-ordinates of the latus rectum are 3
L = (a, 2a) = (1, 2) and L¢ = (a, –2a) = (1, –2) fi x - 3y + 3 = 0
The equation of the tangent at L is
yy1 = 2(x + x1) 13. The equation of any tangent to the parabola y2 = 8x is
fi 2y = 2(x + 1) 2
y = mx + …(i)
fi y = (x + 1) …(i) m
The equation of the tangent at L is
Since (i) is also a tangent of xy = –1, so
–2y = 2(x + 1)
fi y = –(x + 1) …(ii) Ê 2ˆ
x Á mx + ˜ = -1
Solving Eqs (i) and (ii), we get Ë m¯
x = –1, y = 0
2x
Hence, the point of intersection is (–1, 0). fi mx 2 + +1= 0
10. The equation of the tangent at P to the parabola m
y2 = 8x is fi m2x2 + 2x + m = 0
4y = 4(x + 2) Since it will provide us equal roots, so
fi y = (x + 2) …(i) D=0
Given parabola is y2 = 8x + 5 …(ii) fi 4 – 4m3 = 0
Solving Eqs (i) and (ii), we get fi m3 = 1
(x + 2)2 = 8x + 5 fi m=1
fi x2 + 4x + 4 = 8x + 5 Hence, the equation of the common tangent is y = x + 2.

CG_04a.indd 43 2/6/2017 4:02:56 PM


4.44 Coordinate Geometry Booster

14. Given parabola is y = x2. p p 3p


So, 4a = 1 fi x=- ± p= ,-
2 2 2
a = 1/4
p
The equation of any tangent is when x = , then, y = ± p
2
a 1
x = my + = my + Thus the point of intersection are
m 4m
Êp ˆ Êp ˆ
which is also a tangent of ÁË , p˜¯ and ÁË , - p˜¯
2 2
y = –x2 + 4x – 4
2 1 16. Let the parabola be y2 = 4ax and the two points on the
fi x = m( - x + 4x - 4) + parabola are
4m
fi 4mx = 4m2(–x2 + 4x – 4) + 1 P(at12 , 2at1 ) and Q(at22 , 2at2 )
fi –4m2x2 + 4(4m2 – m) + (1 – 16m2) = 0
fi 4m2x2 – 4(4m2 – m)x – (1 – 16m2) = 0 P
Since it will provide us equal roots, so
D=0
fi 16(4m2 – m)2 + 16m2(1 – 16m2) = 0
fi (4m2 – m)2 + m2(1 – 16m2) = 0
fi 16m4 – 8m3 + m2 + m2 – 16m4 = 0
fi –8m3 + 2m2 = 0
fi 4m3 – m2 = 0 Q
1
fi m = 0, The equation of the normal to the parabola at
4
Hence, the equation of the common tangents are P(at12 , 2at1 ) is
y = 0 and y = 4(x – 1) y = - t1 x + 2at1 + at13
15. Given parabola is y2 = 2px.
which meets the parabola again at Q(at22 , 2at2 ) )
Êp ˆ
So, the focus is Á , 0˜ . 2 3
Ë2 ¯ Thus, 2at2 = - at1t2 + 2at1 + at1

Y
fi 2a (t2 - t1 ) + at1 (t22 - t12 ) = 0
fi (t2 – t1)[2a + at1(t2 + t1)] = 0
fi [2 + t1(t2 + t1)] = 0
fi t12 + t1t2 + 2 = 0
X¢ C(p/2, 0) X Hence, the result.
O
17. Given parabola is y2 = x.
So, 4a = 1
x = p/2
1
Y¢ fi a=
4
Clearly, the equation of the circle is The equation of any normal to the parabola
2
Ê pˆ 1 1
2
ÁË x — ˜¯ + y = p
2
y = mx - m - m3
2 2 4
2 which is passing through (c, 0). So
Ê pˆ 2
fi ÁË x — ˜¯ + 2px = p 1 1
2 mc - m - m3 = 0
2 4
p2 fi 4mc – 2m – m3 = 0
fi x 2 - px + + 2px = p 2
4 fi m3 + 2(1 – 2c) = 0
fi m2 + 2(1 – 2c) = 0
p2
fi x 2 + px + = p2 m2
4 fi (1 - 2c) = —
2
2
Ê pˆ
fi ÁË x + ˜¯ = p
2 m2
2 fi (1 - 2c) = — <0
2
Ê pˆ 1
fi ÁË x + ˜¯ = ± p fi c>
2 2

CG_04a.indd 44 2/6/2017 4:02:57 PM


Parabola 4.45

18. The equation of any tangent to the parabola y2 = 8x is 20. Prove that from any point P(at2, 2at) on the parabola
2 y2 = 4ax, two normals can be drawn and their feet
y = mx +
m Q and R have the parameters satisfying the equation
Clearly, l2 + lt + 2 = 0.
m-3 21. Given parabola is x2 = 8y.
tan (± 45°) = We have 4a = 8
1 + 3m
fi a=2
m-3
fi = ±1 The equation of the normal to the parabola x2 = 8y at
1 + 3m
( - 2am1 , am12 ), ( - 2am2 , am22 ) are
m-3 m-3
fi = 1, = -1 x = m1 y - 2am1 - am13 …(ii)
1 + 3m 1 + 3m
3
1 and x = m2 y - 2am2 - am2 …(i)
fi m = - 2,
2 Let (h, k) be the point of intersection.
Hence, the equations of tangents are Thus, h = –a(m1 + m2) …(iii)
x 2 2
and k = 2a + a (m1 + m2 - 1)
y = - 2x - 1, y = + 4 …(iv)
2 From Eqs (iii) and (iv), we get
fi y = –2x – 1, x – 2y + 8 = 0
Solving y2 = 8x and y = –2x – 1 we get, the point of x 2 y - 2a y — 2a — a y - 3a
= -1= =
a2 a a a
Ê1 ˆ
intersection is Á , - 2˜ . fi x = a(y – 3a)
2
Ë2 ¯
fi x2 = 2(y – 6)
Again, solving y2 = 8x and x – 2y + 8 = 0 we get the which is the required locus.
point of intersection is (8, 8).
22. The equation of the tangent at P (at12 , 2at1 ) to the
Ê1 ˆ
Thus, the point of contacts are Á , - 2˜ and (8, 8). parabola y2 = 4ax is
Ë2 ¯
19. Clearly, the ends of a latus rectum are L(a, 2a) and yt1 = x + at12
L¢ = (a, –2a).
fi x - yt1 + at12 = 0
Y¢ Q¢ fi at12 - yt1 + x = 0 …(i)
L
Also, the equation of the tangent at Q(2bt2 , bt22 ) to the
X¢ O X parabola x2 = 4by is
xt2 = y + bt22 …(ii)

Q It is given that the tangents (i) and (ii) are perpendicu-
Y¢ lar, so
The equation of the normal at L is Ê 1ˆ
ÁË t ˜¯ ◊ t2 = -1
y = mx – 2am – am3 1
putting m = –1, we get fi t2 = –t1
y = –x + 2a + a Equation (ii) reduces to
fi x + y = 3a
- xt1 = y + bt12
The equation of the normal at L is
y = mx – 2am – am3 fi xt1 + y + bt12 = 0
putting m = 1, we get
fi bt12 + xt1 + y = 0 …(iii)
y = x – 2a – a
fi x – y = 3a Solving Eqs (i) and (iii), we get
On solving x + y = 3a and y2 = 4ax, we get t12 t1 1
Q = (9a, –6a) = =
- ( x 2 + y 2 ) bx - ay ax + by
Again, solving x – y = 3a and y2 = 4ax, we get
Q¢ = (9a, –6a) Eliminating t1, we get
(ax + by)(x2 + y2) + (bx – ay)2 = 0
Thus, the length of QQ¢ = (9a — 9a) 2 + (6a + 6a) 2
which is the required locus of the point of intersection
= 12a of two tangents.
Hence, the result.

CG_04a.indd 45 2/6/2017 4:02:58 PM


4.46 Coordinate Geometry Booster

23. Let P, Q, and R be three points on the parabola. fi 9b2 = 4a(a – 8a)
Y Hence, the locus of G(a, b) is 9y2 = 4a(x – 8a)
P(t1)
24. Let P = (at12 , 2at1 ) and Q = (at22 , 2at2 ) .

Y
X¢ X P
O
Q(t2)
M(h, k)
X¢ X
O
R(t3)

Q
Let the co-ordinates of the centroid be G(a, b).

a (t 2 + t 2 + t 2 )
Clearly, a = 1 2 3 2a (t2 - t1 ) 2
3 Slope of PQ = =
2 2
a (t2 - t1 ) (t1 t2 )
+
2a (t1 + t2 + t3 )
and b= Equation of PQ is
3
2
Now y - 2at1 = ( x - at12 )
(t1 + t2 )
2a (t1 - t2 ) 2
Slope of PQ = 2 2
=

a (t1 - t2 ) (t1 + t2 ) 2x – (t1 + t2)y = –2at1t2 …(i)
2a (t1 - t3 ) 2 2
and the slope of PR = 2 2
= Now, m1 = m(OP) =
a (t1 - t3 ) (t +
1 t3 ) t1
Clearly, –QPR = 60° 2
and m2 = m(OQ) =
2 2 t2
-
t1 + t2 t1 + t3
Thus, tan (60°) = As OP is perpendicular to OQ, so
2 2
1+ ◊ m1m2 = –1
t1 + t2 t1 + t3
4
fi = -1
2(t3 - t2 ) t1t2
fi 3=
(t1 + t2 )(t2 + t3 ) + 4 fi t1t2 = –4 …(ii)
From Eqs (i) and (ii), we get
3 [(t1 + t2 )(t2 + t3 ) + 4] = 2(t2 - t3 ) …(i)
2(x – 4a) – (t1 + t2) y = 0
Similarly, –Q = 60° and –R = 60° Let M(h, k) be the mid-point of PQ. So
Thus, 3 [(t1 + t2 )(t1 + t3 ) + 4] = 2(t3 - t1 ) …(ii) a a
h = (t12 + t22 ) and k = (2t1 + 2t2 )
2 2
and 3 [(t1 + t3 )(t2 + t3 ) + 4] = 2(t1 - t2 ) …(iii)
a
Adding Eqs (i), (ii) and (iii), we get fi h = (t12 + t22 ) and k = a (t1 + t2 )
2
3(t1t2 + t2t3 + t3t1 ) + (t12 + t22 + t32 + 12) = 0 Now, k2 = a2(t1 + t2)2
3a
fi 3(t1t2 + t2t3 + t3t1 ) + + 12 = 0 fi k 2 = a 2{(t12 + t22 ) + 2t1t2 }
a
a fi k2 = a2(2h + 2(–4))
fi (t1t2 + t2t3 + t3t1 ) + + 4 = 0 Hence, the locus of M(h, k) is
a
fi a(t1t2 + t2t3 + t3t1) + a + 4a = 0 y2 = 2a2(x – 4)
25. Let AB be a chord of a parabola, in which
fi 2a(t1t2 + t2t3 + t3t1) + 2a + 8a = 0
A = (t12 , 2t1 ), B = (t22 , 2t2 )
fi a{(t1 + t2 + t3 ) 2 - (t12 + t22 + t32 )} + 2a + 8a = 0
Slope of AB = 2
Ï 9b 2 3a ¸
fi aÌ 2 - ˝ + 2a + 8a = 0 2
Ó 4a a ˛ fi =2
t1 + t2
9b 2
fi - 3a + 2a + 8a = 0 fi t1 + t2 = 1
4a

CG_04a.indd 46 2/6/2017 4:02:58 PM


Parabola 4.47

Y
A Ê a aˆ Ê a aˆ
Therefore, the points P, Q are Á - , ˜ , Á - , - ˜
Ë 2 2¯ Ë 2 2¯
and R, S are (a, 2a) and (a, –2a) respectively.
P(h, k)
Thus, the area of the equadrilateral PQRS
X¢ X
O 1
= ( PQ + RS ) ¥ LM
2
B 2
1 Êa ˆ 15a
= ¥ ( a + 4a ) ¥ Á + a ˜ =
Y¢ 2 Ë2 ¯ 4

Let P be a point which divides AB internally in the ratio 27. Let the point P be (h, k).
1:2 The equation of any normal to the parabola y2 = 4x is
y = mx – 2m – m3 which is passing through P. So
2t12 + t22 4t + 2t2 k = mh – 2m – m3
So, h = and k = 1
3 3 fi m3 + (2 – h)m + k = 0 …(i)
2 2 which is a cubic equation in m.
3h = (2t1 + t2 ) and 3k = (4t1 + 2t2 )
Let its roots are m1, m2, m3.
Eliminating t1 and t2, we get
Thus, m1 + m2 + m3 = 0
2
Ê 8ˆ 4Ê 2ˆ m1m2 + m2 m3 + m3m1 = (2 - h)
ÁË k - ˜¯ = ÁË h - ˜
9 9 9¯ m1m2 m3 = - k
Thus, the locus of P(h, k) is It is given that m1m2 = a, so
2
Ê 8ˆ 4Ê 2ˆ k
ÁË y - ˜¯ = ÁË x - ˜¯ m3 = -
9 9 9 a
Ê 2 8ˆ Since m3 is root of Eq. (i), so
Hence, the vertex is Á , ˜ .
Ë 9 9¯ m33 + (2 - h)m3 + k = 0
26. The equation of any tangent to the parabola can be con- k3 Ê kˆ
sidered as fi — + (2 - h) Á - ˜ + k = 0
a3 Ë a¯
a
y = mx +
m k2 Ê 1ˆ
fi — + (2 - h) Á - ˜ + 1 = 0
Y a 3 Ë a¯
R(a, 2a)
fi –k2 – (2 – h)a2 + a3 = 0
P(–a/2, a/2) fi k2 + (2 – h)a2 – a3 = 0
Hence, the locus of P is
X¢ M y2 + (2 – x)a2 – a3 = 0
O X
which represents a parabola.
Q(–a/2, –a/2) clearly, a = 2, since a = 2 satisfies the given parabola
y2 = 4x.
S(a, –2a)
28. Given parabola is

y2 – 2y – 4x + 5 = 0
i.e. m2x – my + a = 0 fi (y – 1)2 = 4x – 4 = 4(x – 1)
As we know that the length of perpendicular from the fi Y 2 = 4X
centre to the tangent to the circle is equal to the radius where X = (x – 1), Y = (y – 1)
of a circle. So, the directrix is
a a X+a=0
Thus, = fi (x – 1) + 1 = 0
4
m +m 2 2
fi x=0
fi m4 + m2 = 2 Any point on the parabola is
fi m4 + m2 – 2 = 0 P(1 + t2, 2t + 1)
fi (m2 + 2)(m2 – 1) = 0 The equation of the tangent at P is
fi m = ±1 t(y – 1) = x – 1 + t2
Hence, the equation of the tangents are which meets the directrix x = 0 at
y = x + a, y = –x – a

CG_04a.indd 47 2/6/2017 4:02:59 PM


4.48 Coordinate Geometry Booster

Ê 1ˆ (am2 – 1, 4 – 2am)
Q Á 0, 1 + t - ˜ i.e. (4m2 – 1, 4 – 8m)
Ë t¯
which is passing through (14, 7). So
Let the co-ordinates of R be (h, k). 3 = 15m – 8m – 4m3
1
Since it divides QP externally in the ratio :1 , so Q is fi 4m3 – 7m + 3 = 0
2 fi 4m3 – 4m2 + 4m2 – 4m – 3m + 3 = 0
the mid-point of R and P.
fi 4m2(m – 1) + 4m(m – 1) – 3(m – 1) = 0
h + 1 + t2 1 k + 1 + 2t fi (m – 1)(4m2 + 4m – 3) = 0
\ = 0 and 1 + t - =
2 t 2 fi (m – 1) = 0, (4m2 + 4m – 3) = 0
2 2 fi (m – 1) = 0, (4m2 + 6m – 2m – 3) = 0
fi t = - (h + 1) and t =
1- k fi (m – 1) = 0, (2m – 1)(2m + 3) = 0
4 fi (m – 1) = 0, (2m – 1) = 0, (2m + 3) = 0
Thus, + (h + 1) = 0 1 3
(k - 1) 2 fi m = 1, , -
2 2
fi (k – 1)2(h + 1) + 4 = 0
Hence, the locus of R(h, k) is Hence, the feet of the normals are
(y – 1)2(x + 1) + 4 = 0 (3, –4), (0, 0), (8, 16)
( x + 1) 2 31. Let the parabola be y2 = 4ax.
29. Now, f ( x + 1) = - +x+2 The equation of any tangent to the given parabola is
2
a
- x 2 - 2x - 1 + 2x + 4 - x 2 + 3 y = mx + …(i)
= = m
2 2
2 Let the fixed point be (h, k).
- (1 - x)
Also, f (1 - x) = +1- x +1 The equation of any line passing through P and perpen-
2 dicular to (i) is
- 1 + 2x - x 2 + 4 - 2 x - x 2 + 3 1
= = y - k = - ( x - h) …(ii)
2 3 m
x2 Eliminating m between Eqs (i) and (ii), we get
Thus, y = - + x + 1 is symmetric about the line ( x - h) a ( y - k )
2 y=- -
x=1 ( y - k ) ( x - h)
x2 which is the required locus.
Also, given curve is y = - + x +1
2 32. Let P(at12 , 2at1 ) and (at22 , 2at2 ) be the extremities of
fi 2y = –x2 + 2x + 2 the focal chord.
fi x2 – 2x = –2y + 2 Thus, t1t2 = –1
Ê 3ˆ Let the points of intersection of the normals at P and Q
fi ( x - 1) 2 = - 2y + 3 = —2 Á y — ˜ are R(a, b).
Ë 2¯
Then a = a (t12 + t22 + t1t2 + 2)
fi X2 = –2Y
Ê 3ˆ = a (t12 + t22 — 1 + 2)
X = ( x - 1), Y = Á y — ˜
Ë 2¯ = a (t12 + t22 + 1)
Axis is X = 0
= a[(t1 + t2)2 – 2t1t2 + 1]
fi x–1=0
fi x=1 = a[(t1 + t2)2 + 3] …(i)
which is already proved that the given curve is sym- and b = –at1t2(t1 + t2)
metric about the line x = 1. = a(t1 + t2) …(ii)
Hence, the result. From Eqs (i) and (ii), we get,
30. Given parabola is y2 – 16x – 8y = 0 Ê b2 ˆ
fi y2 – 8y = 16x a = a Á 2 + 3˜
fi (y – 4)2 = 16(x + 1) Ëa ¯
fi Y2 = 16X Ê b2 ˆ
where Y = y – 4, X = x + 1 =Á + 3a˜
Ë a ¯
The equation of any normal to the parabola
Y = mX – 2am – am3 at (am2, –2am) fi b2 = a(a – 3a)
y – 4 = m(x + 1) – 8m – 4m3 at Hence, the locus of (a, b) is y2 = a(x – 3a)

CG_04a.indd 48 2/6/2017 4:02:59 PM


Parabola 4.49

LEVEL IV 3.
4. The equation of the tangent to the parabola y2 = 4x at
(t2, 2t) is yy1 = 2(x + x1)
1. The equation of the chord of contact is
fi 2t ◊ y = 2(x + t2)
yy1 – 2(x + x1) = 0
fi t ◊ y = (x + t2)
fi 2y – 2(x – 1) = 0
fi x – ty + t2 = 0 …(i)
fi y = (x – 1)
and the equation of the normal to the ellipse is
Y
a 2 x b2 y
A - = a 2 - b2
x1 y1
P(–1, 2) 5x 4y
M fi - = 5 - 4 =1

O
X 5 cos j 2 sin j
B
5x 2y
fi - =1
cos j sin j

2
Equations (i) and (ii) are identical, so
On solving y = 4x and y = x – 1, we get, the co-ordi-
nates of the points A and B. 1 -t -t2
= =
Thus, A = (3 - 2 2, 2 - 2 2) 5 -2 1
cos j sin j
and B = (3 + 2 2, 2 + 2 2)
cos j t sin j
fi = = -t2
Now, the length AB = 8 5 2
Therefore, the area of DPAB
1 fi cos j = - 5t 2 , sin j = - 2t
= ¥ PM ¥ AB
2 Now, cos2 j = 5t4
1 —1 — 2 — 1 fi 1 – sin2 j = 5t4
= ¥ ¥8 fi 1 – 4t2 = 5t4
2 2
1 4 fi 5t4 + 4t2 – 1 = 0
= ¥ ¥8 fi (t2 + 1)(5t2 – 1) = 0
2 2
fi (5t2 – 1) = 0
= 8 2 s.u. 1
fi t=±
2. Given curve is y2 – 2x – 2y + 5 = 0 5
fi y2 – 2y = 2x – 5 1 1
fi (y – 1)2 = 2x – 5 + 1 Now, cos j = - 5 ¥ = -
5 5
fi (y – 1)2 = 2x – 4 = 2(x – 2)
Ê 1 ˆ
fi Y2 = 2X fi j = cos -1 Á - ˜
Ë 5¯
where X = x – 2, Y = y – 1
which represents a parabola. 5. The equation of the normal to the parabola
Now, the focus = (a, 0) y2 = 4ax is y = mx – 2am – am3
fi X = a, Y = 0 which is passing through (5a, 2a).
1 fi 2a = 5am – 2am – am3
fi x - 2 = , y -1= 0 fi 2 = 5m – 2m – m3
2
fi m3 – 3m + 2 = 0
5
fi x = , y =1 fi m3 – m2 + m2 – m – 2m + 2 = 0
2
fi m2(m – 1) + m(m – 1) – 2(m – 1) = 0
Ê5 ˆ fi (m – 1)(m2 + m – 2) = 0
Hence, the focus is Á , 1˜ .
Ë2 ¯ fi (m – 1) = 0, (m2 + m – 2) = 0
Also, the directrix: X + a = 0 fi (m – 1) = 0, (m – 1)(m + 2) = 0
1 fi m = 1, –2
fi x-2+ =0
2 Hence, the equation of the normals are
3 y = x – 2a – a = x – 3a
fi x=
2 and y = –2x + 4a + 8a = –2x + 12a.

CG_04a.indd 49 2/6/2017 4:02:59 PM


4.50 Coordinate Geometry Booster

6. Let the point of intersection be P(h, k). Therefore,


The equation of any normal to the given parabola is x = 2a + a (t12 + 4t12 + 2t12 ) = 2a + 7at12
x = my – 2am – am3
fi x = my – 4m – 2m3 Ê x - 2a ˆ
( a = 2) fi t12 = Á
which is passing through P. Ë 7a ˜¯
fi h = mk – 4m – 2m3 1/2
fi Ê x - 2a ˆ
fi 2m3 + (4 – k)m + h = 0 t1 = Á
Ë 7a ˜¯
Let its roots be m1, m2, m3.
m 1 + m2 + m 3 = 0 and y = - at1 ◊ 2t1 (t1 + 2t1 ) = - 6at13
4-k
m1m2 + m1m3 + m2 m3 = 3/2
2 Ê x - 2a ˆ
fi y = - 6a Á
h Ë 7a ˜¯
and m1m2 m3 = -
2 3/2
Ê x - 6ˆ
fi y = -18 Á
Now, m1m2 m3 = -
h Ë 21 ˜¯
2
3/2
h Ê x - 6ˆ
fi m3 = , fi y + 18 Á =0.
Ë 21 ˜¯
( m1m2 = –1)
2
Also, m1 + m2 = –m3 8. Given circle is x2 + (y – 3)2 = 5.
4-k The equation of any tangent to the parabola y2 = x.
and m1m2 + m1m3 + m2 m3 =
2 can be considered as
4-k 1
fi —1 + (m1 + m2 )m3 = y = mx +
2 4m
4-k fi 4my = 4m2x + 1
fi —1 - m32 = fi 4m2x – 4my + 1 = 0
2
h2 4 - k Now, OM = 5
fi —1 - =
4 2 0 - 12m + 1
fi = 5
4 + h2 k — 4 16m 4 + 16m 2
fi =
4 2 fi (1 – 12m)2 = 5(16m4 + 16m2)
fi 4 + h2 = 2k – 8 fi (1 – 12m)2 = 5(16m4 + 16m2)
fi h2 = 2(k – 6)
fi 1 – 24m + 144m2 = 80m4 + 80m2
Hence, the locus of P is x2 = 2(y – 6).
7. Let the parabola be y2 = 4ax and two points on the fi 80m4 – 64m2 + 24m – 1 = 0
2 fi 80m4 – 40m3 + 40m3 – 20m2
parabola are P(at1 , 2at1 ) and Q(at22 , 2at2 ) ., where
a=3 –44m2 + 22m + 2m – 1 = 0
It is given that fi 40m3(2m – 1) + 20m2(2m – 1)
2at1 1 –22m(2m – 1) + 1(2m – 1) = 0
=
2at2 2 fi (2m – 1)(40m3 + 20m2 – 22m + 1) = 0
t1 1 1
fi = fi m=
t2 2 2
fi 2t1 = t2 Hence, the equation of the common tangent be
The equation of the normal at P (at12 , 2at1 ) and x 1
y= +
Q(at22 , 2at2 ) are 2 2
fi 2y = x + 1,
y = - t1 x + 2at1 + at13 …(i)
fi x – 2y + 1 = 0
and 9. The equation of the normal to the given parabola
y = - t2 x + 2at2 + at23 …(ii) y2 = 8(x – 1) is
y = m(x – 1) – 2am – am3
Solving Eqs (i) and (ii), we get fi y = m(x – 1) – 4m – 2m3
x = 2a + a (t12 + t22 + t1t2 ) fi y = mx – 5m – 2m3 …(i)
Let the co-ordinates of the point of intersection of the
and y = –at1t2(t1 + t2) tangents be P(h, k).

CG_04a.indd 50 2/6/2017 4:03:00 PM


Parabola 4.51

x + x1 ˆ Given parabola is
Thus, yy1 = 8 ÊÁ -8
Ë 2 ˜¯ y2 = 4x
dy
fi yk = 4(x + h) – 8 = 4(x + h – 2) fi 2y =4
4 dx
fi y = ( x + h — 2)
k dy 2
fi =
dx y
4 4
fi y= x + (h — 2) …(ii) Also, given circle is
k k
x2 + y2 – 12x + 31 = 0
Comparing Eqs (i) and (ii), we get dy
4 4 fi 2x + 2y - 12 = 0
m = , (h — 2) = - (5m + 2m3 ) dx
k k dy
3 fi x+ y -6=0
4 Ê 4 Ê 4ˆ ˆ dx
fi ( h - 2) = - Á 5 ◊ + 2 ◊ ÁË ˜¯ ˜
k Ë k k ¯ dy 6 — x
fi =
Ê 2 dx y
fi Ê 4ˆ ˆ
(h - 2) = - Á 5. + 2. Á ˜ ˜ Since the tangents are parallel, so their slopes are the
Ë Ë k¯ ¯
same.
fi (h – 2)k2 = –(5k2 + 32) 2 6—x
Hence, the locus of P(h, k) is Thus, =
y y
(2 – x)y2 = (5y2 + 32)
10. Let AB be a double ordinate, where fi x=4
When x = 4, then y2 = 16
A= (at12 , 2at1 ), B = (at22 , 2at2 )
fi y = ±4
Let P(h, k) be the point of trisection. Then Thus, the point Q is (4, 4).
3h = 2at2 + at2 and 3k = 4at – 2at Therefore, the shortest distance,
fi 3h = 3at2 and 3k = 2at PQ = CQ – CP
fi h = at2 and 3k = 2at = (6 - 4) 2 + (4 - 0) 2 - 1
Solving, we get
= 20 - 1
h 3k
t 2 = and t =
a 2a = (2 5 - 1)

Ê 3k ˆ
2
h 13. Let the parabola be y2 = 4ax
fi ÁË ˜¯ = The equation of the tangent to the parabola at (a, 2a) is
2a a
y ◊ 2a = 2a(x + a)
fi 9k 2 h fi y=x+a
= fi x–y–a=0
4a 2 a
The equation of a circle touching the parabola at (a, 2a)
fi 9k2 = 4ah is
Hence, the locus of P(h, k) is (x – a)2 + (y – 2a)2 + l(x – y – a) = 0
9y2 = 4ax which is passing through (0, 0). So
11. Do yourself. a2 + 4a2 – la = 0
12. Given circle is x2 + y2 – 12x + 31 = 0 fi l = 5a
fi (x – 6)2 + y2 = 5 Thus, the required circle is
The centre is (6, 0) and the radius is 5. x2 + y2 – 7ax + ay = 0
2 2
Ê 7a ˆ Ê aˆ 50a 2 5a
Y Hence, the radius = Á ˜ + Á ˜ = =
Q Ë 2¯ Ë 2¯ 4 2
P 14. The tangent to y2 = 4x in terms of m is
1
X y = mx +
O C(6, 0) m
and the normal to x2 = 4by in terms of m is
b
y = mx + 2b + 2
m

CG_04a.indd 51 2/6/2017 4:03:00 PM


4.52 Coordinate Geometry Booster

If these are the same line, then Clearly,


1 b 4(t1 + t2) = 4
= 2b + 2
m m fi (t1 + t2) = 1
2 2
fi 2bm – m + b = 0
2
Now, m = m( AB) = = =2
For two different tangents, we get t1 + t2 1
D>0 Hence, the slope of the normal chord is 2.
fi 1 – 8b2 > 0 5. Normals to y2 = 4ax and x2 = 4by in terms of m are
fi 8b2 < 1 y = mx – 2am – am3
1 b
fi b2 < and y = mx + 2b + 2
8 m
1 For a common normal,
fi |b| < b
2 2 2b + 2 = - 2am - am3
which is the required condition. m
15. Given parabola is y2 = 8x fi 2bm2 + b + 2am3 + am5 = 0
Extremities of the latus rectum are (2, 4) and (2, –4). fi am5 + 2am3 + 2b2 + b = 0
Since any circle is drawn with any focal chord as its Thus, the number of common normals is 5.
diameter touches the directrix, the equation of the re- 6. We have, x + y = 2(t2 + 1)
quired circle is and x – y = 2t
(x – 2)(x – 2) + (y – 4)(y + 4) = 0 Eliminating t, we get,
fi x2 + y2 – 4x – 12 = 0 ÊÊ x - yˆ 2 ˆ
Hence, the radius = 4 + 12 = 4 . x + y = 2Á Á ˜ + 1˜
ËË 2 ¯ ¯
( x - y)2
fi x+ y= +2
Integer Type Questions 2
1. A circle and a parabola can meet at most in four points. fi 2(x + y) = (x – y)2 + 4
Thus, the maximum number of common chords is fi (x – y)2 – 2(x + y) + 4 = 0
4¥3 fi (x – y)2 = 2(x + y – 2)
4
C2 = =6. Comparing with y2 = 4ax, we get
2
4a = 2
2. Clearly, both the lines pass through (–a, b) which a Thus, the length of latus rectum is 2.
point lying on the directrix of the parabola. 7. Both the given curves are symmetrical about the line
Thus, m1m2 = –1, since tangents are drawn from any y = x.
point on the directrix always mutually perpendicular. If the line AB is the shortest distance then at A and B the
Hence, the value of (m1m2 + 4) is 3. slopes of the curve should be equal to 1.
3. Let AB be a normal chord, where dy 1
For y2 = x – 1, = =1
A = (at12 , 2at1 ), B = (at22 , 2at2 ) dx 2y
1
Now, the normal at A meets the parabola again at B, so fi y=
2
2
t2 = - t1 - and t1t2 = –4 Y
t1
x
Solving, we get y=
t12 = 2 B

2 A
Thus, m = m( AB) = X
t1 + t2
2
fi m= = - t1 = 2
( - 2/t1 ) 1 5
Then x = +1= .
fi (m + 3) = 2 + 3 = 5
2
4 4
4. Let AB be a normal chord, where Ê 5 1ˆ Ê 1 5ˆ
Therefore, A = Á , ˜ and B = Á , ˜
A= (at12 , 2at1 ), B = (at22 , 2at2 ) Ë 4 2¯ Ë 2 4¯

CG_04a.indd 52 2/6/2017 4:03:01 PM


Parabola 4.53

Hence, the shortest distance,


x = 1 - ( - 2 + 2 2) = 3 - 2 2
2 2
Ê 5 1ˆ Ê 1 5ˆ when y = - 2 - 2 2 , then
d = Á - ˜ +Á - ˜
Ë 4 2¯ Ë 2 4¯
x = 1 - ( - 2 - 2 2) = 3 + 2 2
2 2
Ê 3ˆ Ê 3ˆ 18 3 2 Let the chord be AB,
= Á ˜ +Á ˜ = =
Ë 4¯ Ë 4¯ 16 4 where A = (3 - 2 2, - 2 + 2 2),
Hence, the value of
and B = (3 + 2 2, - 2 - 2 2)
(8d2 – 3)
=9–3=6 Hence, the length of the chord AB
8. The equation of any tangent to the parabola y2 = 4x is = (4 2) 2 + (4 2) 2 = 32 + 32 = 8
a 1
y = mx + = mx + 11. Given L: y = –x + k
m m
Here, m = –1 and c = k
which is passing through (2, 3). So The line L will be a normal to the parabola, if
1 c = –2am – am3
3 = 2m +
m fi k = –2 ¥ 3 ¥ (–1) – 3 ¥ (–1)3
fi 2m2 – 3m + 1 = 0 fi k=6+3=9
fi 2m2 – 2m – m + 1 = 0 Hence, the value of k is 9.
fi 2m(m – 1) – 1(m – 1) = 0 12. Given parabola is
fi (m – 1)(2m – 1) = 0 y2 – 4x – 2y – 3 = 0
fi m = 1 and 1/2 fi y2 – 2y = 4x + 3
1 fi (y – 1)2 = 4(x + 1)
fi m1 = 1, m2 =
2 The equation of any normal to the given parabola is
Ê 1 1 ˆ (y – 1) = m(x + 1) – 2am – am3
Hence, the value of Á + + 2˜ = 1 + 2 + 2 = 5.
Ë m1 m2 ¯ fi (y – 1) = m(x + 1) – 2m – m3
2 which is passing through (–2, 1), so
9. Let the co-ordinates of the point R be (at3 , 2at3 ) . (1 – 1) = m(–2 + 1) – 2m – m3
The normal at P meets the parabola again at R, so fi 0 = –m – 2m – m3
2 fi m3 + 3m = 0
t3 = - t1 - …(i)
t1 Hence, the number of distinct normals is 3.
and the normal at Q meets the parabola again at R, so
2 Previous Years’ JEE-Advanced Examinations
t3 = - t2 - …(ii)
t2 1. The equation of the normal to the parabola y2 = 4ax is y
From Eqs (i) and (ii), we get = mx – 2am – am3 which is passing through (h, 0). So
2 2 mh – 2am – am3 = k
—t1 - = - t2 -
t1 t2 fi am3 + (2a – h)m + k = 0
2 2 2(t2 - t1 ) Let its roots are m1, m2 and m3.
fi t2 — t1 = - =
t1 t2 t1t2 Thus, m1 + m2 + m3 = 0
2a - h
2(t2 - t1 ) m1m2 + m2m3 + m3m1 =
fi t2 — t1 = a
t1t2 k
and m1m2 m3 = -
fi t1t2 = 2 a
Hence, the value of (t1t2 + 3) = 5 For any real values of m1, m2, m3,
10. Solving, we get,
y2 = 4(1 – y) m12 + m22 + m32 > 0
fi y2 = 4 – 4y fi (m1 + m2 + m3)2 – 2(m1m2 + m2m3 + m3m1) > 0
fi y2 + 4y – 4 = 0
fi (y + 2)2 = 8 Ê 2a - h ˆ
fi 0 - 2Á >0
Ë a ˜¯
fi y = -2 ± 2 2
fi h – 2a > 0
when y = - 2 + 2 2 , then fi h > 2a

CG_04a.indd 53 2/6/2017 4:03:01 PM


4.54 Coordinate Geometry Booster

2. Let the point A be (at2, 2at). 3


fi c=
The equation of the normal at A is 4
y = –tx + 2at + at3 5. Let P = (t12 , 2t ) and Q = (t22 , 2t2 )
and the equation of the pair of lines OA and OB is
Y
Ê y + tx ˆ
y 2 = 4ax Á
Ë 2at + at 3 ¯˜
P

fi (t3 + 2t)y2 = 4tx2 + 4xy


p X¢ X
O
As –AOB = , we get
2
co-efficient of x2 + co-efficient of y2 = 0 Q
fi –4t + t3 + 2t = 0

fi t3 – 2t = 0
fi t(t2 – 2) = 0 The equation of PQ is
fi t = 0, t = ± 2 2x – (t1 + t2)y = –2t1t2 …(i)
2
Clearly, t π 0, so t = ± 2 Now, m1 = m(OP) =
t1
Thus, slope of normal = - t = ± 2 .
2
3. The equation of the normal to the given parabola is and m2 = m(OQ) =
x = my – 2am – am3 t2
fi x = my – 2m – m3 As OP is perpendicular to OQ, so
which is passing through (1, 2) m1m2 = –1
1 = 2m – 2m – m3 4
fi = -1
fi m3 = –1 t1t2
fi m = –1 fi t1t2 = –4 …(ii)
Hence, the equation of the normal is From Eqs (i) and (ii), we get
x = –y + 2 + 1 2(x – 4) – (t1 + t2)y = 0
fi x+y=3 which is passing through a fixed point (4, 0) on the axis
4. The equation of the normal to the given parabola is of the parabola.
y = mx – 2am – am3 Let M(h, k) be the mid-point of PQ.
1 1 1 1
fi y = mx - 2 ◊ m - m3 Thus, h = (t12 + t22 ) and k = (2t1 + 2t2 )
4 4 2 2
1 1 1 2 2
fi y = mx - m - m3 fi h= (t1 + t2 ) and k = (t1 + t2 )
2 4 2
fi 4y = 4mx – 2m – m3 Now, k2 = (t1 + t2)2
which is passing through (c, 0), so
fi k 2 = (t12 + t22 ) + 2t1t2
0 = 4mc – 2m – m3
fi m3 + 2m – 4mc = 0 fi k2 = 2h + 2(–4)
fi m3 + 2(1 – 2c)m = 0 Hence, the locus of M(h, k) is
fi m = 0, m2 + 2(1 – 2c) = 0 y2 = 2(x – 4) = 2x – 8
fi m = 0, m2 = 2(2c – 1) 6. Given parabola is y2 = 2px. So
fi m = 0, m = ± 2(2c — 1) Êp ˆ
the focus is ÁË , 0˜¯ .
2
So, one normal is always the x-axis.
Y
Let m1 = 2(2c — 1) and m2 = - 2(2c — 1)
It is given that,
m1m2 = –1
fi 2(2c – 1) = 1 X¢ X
O C(p/2, 0)
1
fi (2c - 1) =
2
x = p/2
1 3
fi 2c = 1 + = Y¢
2 2

CG_04a.indd 54 2/6/2017 4:03:02 PM


Parabola 4.55

Clearly, the equation of the circle is 2


2 Ê 8ˆ 4Ê 2ˆ
Ê pˆ 2 2 ÁË y - ˜¯ = ÁË x - ˜¯
ÁË x — ˜¯ + y = p 9 9 9
2
2 Ê 2 8ˆ
Ê pˆ Hence, the vertex is Á , ˜ .
2 Ë 9 9¯
fi ÁË x — ˜¯ + 2px = p
2
8. Let the three points of the parabola be
p2 P(at12 , 2at1 ), Q(at22 , 2at2 ) and R(at32 , 2at3 ),
fi x 2 - px + + 2px = p 2
4
and the points of intersections of the tangents at these
2 p2 points are A[t2t3, a(t2 + t3)], B[t1t3, a(t1 + t3)] and A[t1t2,
fi x + px + = p2
4 a(t1 + t2)]
2 Now,
Ê pˆ
fi ÁË x + ˜¯ = p
2
at12 2at1 1
2 1 2
ar(DPQR) = at2 2at2 1
Ê pˆ 2
fi ÁË x + ˜¯ = ± p at32 2at3 1
2
= a2(t1 – t2)(t1 – t3)(t3 – t1)
p p 3p Also,
fi x=- ± p= ,-
2 2 2 at2 t3 a (t2 + t3 ) 1
1
p ar(DABC) = at3t1 a (t3 + t1 ) 1
when x = , then, y = ± p 2
2 at1t2 a(t1 + t2 ) 1
Thus the point of intersection are 1 2
= a (t1 - t2 )(t2 - t3 )(t3 - t1 )
Êp ˆ Êp ˆ 2
ÁË , p˜ and Á , -
¯ Ë2

¯
2 DPQR
\ =2
7. Let AB be a chord of a parabola, in which DABC
9.
A = (t12 , 2t1 ), B = (t22 , 2t2 )
10. The equation of the normal to the parabola y2 = 12x
Slope of AB = 2 is y = mx – 2am – am3
2 fi y = mx – 6m – 3m3 …(i)
fi =2
t1 + t2 Given normal is y = –x + k …(ii)
fi t1 + t2 = 1 Equations (i) and (ii) are identical, so
m = –1
Y
A and k = –6m – 3m3 = 6 + 3 = 9
Hence, the value of k is 9.
11 Given parabola is y2 = kx – 8
P(h, k)
Ê 8ˆ
X¢ fi y2 = k Á x - ˜
O
X Ë k¯
Here, 4a = k
B k
fi a=
Y¢ 4
Let P be a point which divides AB internally in the ratio Ê 8ˆ k
So, the directrix is Á x - ˜ + = 0
1 : 2. So Ë k¯ 4
2t12 + t22 4t + 2t2 Given directrix is x – 1 = 0.
h= and k = 1
3 3
8 k
fi 2 2
3h = (2t1 + t2 ) and 3k = (4t1 + 2t2 ) Thus, - =1
k 4
Eliminating t1 and t2, we get
fi 32 – k2 = 4k
2
Ê 8ˆ 4Ê 2ˆ fi k2 + 4k – 32 = 0
ÁË k - ˜¯ = ÁË h - ˜¯ fi (k + 8)(k – 4) = 0
9 9 9
fi k = 4, –8
Thus, the locus of P(h, k) is

CG_04a.indd 55 2/6/2017 4:03:02 PM


4.56 Coordinate Geometry Booster

12. Given parabola is If it is a tangent to the curve xy = –1, then


y2 + 4y + 4x + 2 = 0 Ê 2ˆ
x Á mx + ˜ = -1
fi (y + 2)2 = –4x – 2 + 4 Ë m¯
Ê 1ˆ fi m2x2 + 2x + m = 0
fi ( y + 2) 2 = - 4x + 2 = — 4 Á x — ˜ It has equal roots. So,
Ë 2¯
D=0
1 fi 4 – 4m3 = 0
So, the directrix is x - = a =1
2 fi m3 = 1
3 fi m =1
fi x= Hence, the equation of the common tangent is y = x + 2.
2
16. Ans. (a)
13. Any tangent to the parabola y2 = 4x is
For the parabola, y2 = 16x , focus = (4, 0)
a Let m be the slope of the focal chord.
y = mx +
m So, its equation is y = m(x – 4) ...(i)
1 which is a tangent to the circle
fi y = mx + (x – 6)2 + y2 = 2
m
where centre = (6, 0) and radius = 2 .
fi m2x – my + 1 = 0 …(i) Length of perpendicular from (6, 0) to (i) is equal to r
If it is a tangent to the circle x2 + (y – 3)2 = 9 the length
6m - 4m
of the perpendicular from the centre to the tangent is = 2
equal to the radius of the circle. So m2 + 1
2m
3m 2 + 1 = 2
=3 m2 + 1
m4 + m2
4m2 = 2(m2 + 1)
fi 2 2
(3m + 1) = 9(m + m )4 2
2m2 = (m2 + 1)
fi (9m4 + 6m2 + 1) = 9(m4 + m2) m2 = 1
fi 3m2 = 1 m = ±1
17 Given that
Ê 1 ˆ C1: x2 = y – 1
fi m= ±Á ˜
Ë 3¯ C2: y2 = x – 1
Since, the tangent touches the parabola above x-axis, Let P( x1 , x12 + 1) on C1 and Q( y22 + 1, y2 ) on C2.
so it will make an acute angle with x-axis, so that m is
positive.
1
Thus m =
3
Hence, the common tangent is x - 3 y + 3 = 0 .
14. Ans. (c)
If (h, k) be the mid point of line joining the focus (a,
a + at 2
0) and Q(at2, 2at) on the parabola, then h = ,
k = at. 2
Now, the reflection of the point P in the line y = x can
Eliminating ‘t’, we get, be obtained by interchanging the values of abscissa and
Ê k2 ˆ the ordinate.
2h = a + a Á 2 ˜
Ëa ¯ Thus, the reflection of the point P( x1 , x12 + 1) is
Ê aˆ P1 ( x12 + 1, x1 )
fi k 2 = 2a Á h - ˜ and the reflection of the point Q( y22 + 1, y2 ) is
Ë 2¯
Ê aˆ a Q1 ( y2 , y22 + 1)
Now, directrix: Á x - ˜ = - It can be seen clearly that, P1 lies on C2 and Q1 on C1
Ë 2¯ 2
fi x = 0. Now, PP1 and QQ1 both are perpendicular to the mirror
15. Let the equation of the tangent to the parabola y2 = 8x is line y = x.
2 Also, M is the mid point of PP1
y = mx + …(i)
1
m Thus, PM = PP1
2

CG_04a.indd 56 2/6/2017 4:03:04 PM


Parabola 4.57

In triangle PML, PL > PM and m1m2m3 = –k


1 It is given that m1m2 = a. So
PL > PP1 ...(i)
2 k
m3 = -
1 a
Similarly, LQ > QQ1 ...(ii)
2 Since m3 is the roots of (i), so
Adding (i) and (ii), we get, m33 + (2 - h) m3 + k = 0
1 3
PL + LQ > ( PP1 + QQ1 ) Ê kˆ Ê kˆ
2 fi ÁË — ˜¯ + (2 - h) ÁË - ˜¯ + k = 0
a a
1
PQ > ( PP1 + QQ1 ) 3
2 Ê kˆ Êkˆ
fi - Á ˜ - (2 - h) Á ˜ + k = 0
PQ is more than the mean of PP1 and QQ1 Ë a¯ Ëa¯
PQ ≥ min (PP1, QQ1) k 2 (2 - h)
fi + -1= 0
Let min (PP1, QQ1) = PP1 a3 a
then PQ 2 ≥ PP12 fi k2 + (2 – h)a2 – a3 = 0
Hence, the locus of P(h, k) is
= ( x12 + 1 - x1 ) 2 + ( x12 + 1 - x1 ) 2 y2 + (2 – x)a2 – a3 = 0
= 2( x12 + 1 - x1 ) 2 = f ( x1 ) As this locus is a part of the parabola y2 = 4x so, a2 = 4
and –2a2 + a3 = 0
Now, f ¢ ( x1 ) = 4( x12 + 1 - x1 )(2x1 - 1) Thus, a = 2.
ÊÊ 2 19. The equation of any tangent to the given parabola can
1ˆ 3ˆ
= 4 Á Á x1 - ˜¯ + ˜¯ (2x1 - 1) be considered as
ËË 2 4
a 1
1 y = mx + = mx +
f ¢ ( x1 ) = 0 gives x1 = m m
2
which is passing through (1, 4). So
1 1
Also, f ¢ ( x1 ) < 0 if x1 < 4=m+
2 m
1 fi m2 – 4m + 1 = 0
and f ¢ ( x1 ) > 0 if x1 >
2 Let its roots are m1, m2.
1 \ m1 + m2 = 4 and m1m2 = 1
Thus, f(x1) is minimum when x1 = Let q be the angle between them. Then
2
1 m2 - m1
Thus, if at x1 = at P is P0 on C1 tan q =
2 1 + m1m2
Ê 1 Ê 1ˆ 2 ˆ Ê 1 5ˆ
So, P0 = Á , Á ˜ + 1˜ = Á , ˜ (m2 + m1 ) 2 - 4m1m2
Ë 2 Ë 2¯ ¯ Ë 2 4¯ =
1 + m1m2
Similarly Q0 on C2 will be image of P0 with respect to
the line y = x 16 — 4 2 3
= =
Ê 5 1ˆ 1+1 2
So, Q0 = Á , ˜
Ë 4 2¯
p
18. Let the point P be (h, k). = 3=
3
The equation of any normal to the given parabola is p
y = mx – 2am – am3 fi q=
3
fi y = mx – 2m – m3, since a = 1
20. Given parabola is
which is passing through P. So
y2 – 2y – 4x + 5 = 0
k = mh – 2m – m3 fi (y – 1)2 = 4x – 4 = 4(x – 1)
fi m3 + (2 – h)m + k = 0 …(i) fi Y2 = 4X
Let its roots be m1, m2, m3. where X = (x – 1), Y = (y – 1)
So, m1 + m2 + m3 = 0 So, the directrix is
m1m2 + m2m3 + m3m1 = (2 – h) X+a=0

CG_04a.indd 57 2/6/2017 4:03:04 PM


4.58 Coordinate Geometry Booster

fi (x – 1) + 1 = 0 Since it has equal roots, so


fi x=0 fi D=0
Any point on the parabola is 4(2 – t)2 – 4(t2 – 4) = 0
P(1 + t2, 2t + 1) fi (2 – t)2 – (t2 – 4) = 0
The equation of the tangent at P is fi t = 2, 0
t(y – 1) = x – 1 + t2 Hence, the equation of the common tangent is
y = 4x – 4, y = 0
which meets the directrix x = 0 at
23.
Ê 1ˆ
Q Á 0, 1 + t - ˜ Y
Ë t¯
Let the co-ordinates of R be (h, k). P
Q
1
Since it divides QP externally in the ratio :1 , so Q is
2 X¢ X
the mid-point of R and P. Thus O M(3, 0)

h + 1 + t2 1 k + 1 + 2t
= 0 and 1 + t - = R
2 t 2

2 2
fi t = - (h + 1) and t = Equation of any normal to the given parabola is
1- k
y = mx – 2am – am3 …(i)
4 Let P = (am12 , - 2am1 ), Q = (am22 , - 2am2 )
Thus, + (h + 1) = 0
(k - 1) 2 and R = (am32 , - 2am3 )
fi (k – 1)2(h + 1) + 4 = 0 Equation (i) passing through (3, 0)
So, 0 = 3m – 2am – am3
Hence, the locus of R(h, k) is
m3 – m = 0, ( a = 1)
(y – 1)2(x + 1) + 4 = 0 m(m + 1)(m – 1) = 0
21. Clearly, the vertex is (1, 1) and the focus is S(2, 2) and m = –1, 0, 1
the directrix is x + y = 0. Thus, m1 = –1, m2 = 0, m3 = 1
Let P be (x, y) Now, P = (m12 , 2m1 ) = (1, - 2)
Now, SP = PM
fi SP2 = PM2 Q = (m22 , 2m2 ) = (0, 0)

Ê x + yˆ
2
and R = (m32 , 2m3 ) = (1, 2)
fi ( x - 2) 2 + ( y - 2) 2 = Á
Ë 2 ˜¯ (i) Area of DPQR
fi 2[(x – 2) + (y – 2) ] = (x + y)
2 2 2
1 -2 1
fi 2(x2 + y2 – 4x – 4y + 8) = x2 + y2 + 2xy 1 1
= 0 0 1 = ¥ 4 ¥1- 2
fi (x2 + y2 – 2xy) = 8(x + y + 2) 2 2
1 2 1
fi (x – y)2 = 8(x + y + 2)
22. Any point on the parabola y = x2 is (t, t2). (ii) Radius of circumcircle of DPQR = 2
Now tangent at (t, t2) is (iii) Centroid of DPQR
1 Ê m 2 + m22 + m32 2(m1 + m2 + m3 ) ˆ
xx1 = ( y + y1 ) =Á 1 , ˜¯
2 Ë 3 3
1 Ê2 ˆ
fi tx = ( y + t 2 ) = Á ,0˜
2 Ë3 ¯
fi 2tx – y – t2 = 0
(iv) Clearly, DPQR is a right-angled triangle and right
If it is a tangent to the parabola, y = –(x – 2)2, then angle at Q.
2tx – t2 = –(x – 2)2 Thus, Circumcentre of DPQR
fi 2tx – t2 = – x2 + 4x – 4 = Mid-point of the hypotenuse PR
fi x2 + 2(2 – t)x + (t2 – 4) = 0 = (1, 0).

CG_04a.indd 58 2/6/2017 4:03:04 PM


Parabola 4.59

24. 26. Given ellipse is


Y x2 + 4y2 = 4
x2 y 2
P(1, –2 2) fi + =1
4 1
Y
N(–30) L¢ L
X¢ X
S(–1, 0) O M(1, 0) R(9, 0)
S¢ S
X¢ X
V
Q(1, –2 2)

Q P


(i) Co-ordinates of P and Q are
P = (1, 2 2), Q = (1, - 2 2) b2 1 3
Thus, e = 1 - 2
= 1- =
1 a 4 2
Area of DPQR = ¥ 4 2 ¥ 8 = 16 2
2 Foci: S = (ae, 0) = ( 3, 0)
1
Area of DPQS = ¥ 4 2 ¥ 2 = 4 2 and S ¢ = (— ae, 0) = ( - 3, 0)
2
End-points of latus recta:
ar ( DPQS ) 4 2 1
Thus, = = Ê b2 ˆ Ê 1ˆ
ar ( DPQR) 16 2 4 and L = Á ae, ˜ = Á 3, ˜
Ë a ¯ Ë 2¯
abc 2 3 ¥ 6 2 ¥ 10 Ê b2 ˆ Ê 1ˆ
(ii) R = = =3 3 L ¢ = Á — ae, ˜ = Á - 3, ˜
4D Ê1 ˆ Ë a ¯ Ë 2¯
4 ◊ Á ¥ 10 ¥ 2 2 ˜
Ë2 ¯
Ê 1ˆ Ê 1ˆ
Thus, P = Á 3, - ˜ and Q = Á - 3, - ˜
(iii) r = D Ë 2¯ Ë 2¯
s As we know that, the focus is the mid-point of the
1 P and Q.
¥4 2 ¥8
2 1
=
1 Thus, the focus of a parabola is ÊÁ 0, - ˆ˜ .
¥ (6 2 + 6 2 + 4 2) Ë 2¯
2 The length of PQ = 2 3
32 2
= 3
16 2 Now, 4a = 2 3 fi a =
2
=2
Thus, the vertices of a desired parabola
25. Given curve is
Ê 1 ˆ Ê 1 3ˆ
x2 = Á 0, - ± a˜ = Á 0, - ± ˜
y=- + x +1 Ë 2 ¯ Ë 2 2 ¯
2
Therefore, two desired parabolas are
1Ê 1 1ˆ
fi y = - Á x2 - x - ˜ Ê Ê 1 3ˆˆ
2Ë 2 2¯ fi x 2 = ± 4a Á y - Á - ± ˜˜
Ë Ë 2 2 ¯¯
1 3
fi y = - ( x - 1) 2 + Ê 1 3ˆ
2 2 fi x2 = 2 3 Á y + + ˜
Ë 2 2 ¯
Ê 3ˆ 1 2 or
fi ÁË y — ˜¯ = - ( x - 1)
2 2 Ê 1 3ˆ
x2 = - 2 3 Á y + - ˜
Ë 2 2 ¯
which is symmetric about the line x =1.
fi x 2 = 2 3 y + (3 + 3)
Note: A function f(x) is symmetric about the line x =
1 then, f(1 – x) = f(x + 1) or x 2 = - 2 3 y + (3 — 3)

CG_04a.indd 59 2/6/2017 4:03:05 PM


4.60 Coordinate Geometry Booster

27. Let the co-ordinates P be (at2, 2at). fi 6 = 9m – 2m – m3


Y fi m3 – 7m + 6 = 0
P fi m3 – m2 + m2 – m – 6m + 6 = 0
fi m2(m – 1) + m(m – 1) – 6(m – 1) = 0
fi (m – 1)(m2 + m – 6) = 0
X¢ X fi (m – 1)(m – 2)(m + 3) = 0
T O S N fi m = 1, 2, –3
Thus, the equation of normal can be
y = x – 3, y = 2x – 12, y + 3x – 33 = 0.
Y¢ 30.
The equation of PT is yt = x + at2 Y
So, T is (–at2, 0). L(2, 4)
and the equation of PN is y = –tx + 2at + at3 A
So, N is (2a + at2, 0).
B(0, 2) P(1/2, 2)
Let the centroid be G(h, k). X

at 2 - at 2 + 2a + at 2 2at
Thus, h = and k = C
3 3 x=2
2
2a + at 2at M(2, –4)
fi h= and k = Y¢
3 3
2 D LPM
Ê 3h - 2a ˆ Ê 3k ˆ Clearly, =2
fi ÁË ˜ =Á ˜ D ABC
a ¯ Ë 2a ¯
D1
Ê 2a ˆ 9k 2 fi =2
fi 3Á h - ˜ = D2
Ë 3 ¯ 4a
31. The parabola is x = 2t2, y = 4t
4a Ê 2a ˆ Solving it with the circle, we get
fi k2 = ÁË h - ˜¯
3 3 4t4 + 16t2 – 4t2 – 16t = 0
Hence, the locus of G(h, k) is fi t4 + 3t2 – 4t = 0
fi t(t3 + 3t – 4) = 0
4a Ê 2a ˆ
y2 = ÁË x - ˜¯ fi t = 0, 1
3 3 So, the points P and Q are (0, 0) and (2, 4), respec-
Ê 2a ˆ tively which are also diametrically opposite points on
So the vertex is Á , 0˜ and focus is (a, 0).
Ë 3 ¯ the circle.
2 2 The focus is S = (2, 0)
28. Let A = (t1 , 2t1 ), B = (t2 , 2t2 )
0 0 1
Y 1
The area of DPQS = 2 4 1
B 2
C 2 0 1
A 1

O M
X = ¥2¥4=4
2
Êa 2a ˆ
32. Let P = (at 2 , 2at ), Q = Á 2 , - ˜
Ët t ¯

Ê t2 + t2 ˆ Ê Ê 1ˆ ˆ
Then C = Á 1 2 , t1 + t2 ˜ and R = Á - a, a Á t - ˜ ˜
Ë 2 ¯ Ë Ë t¯¯
Clearly, |t1 + t2| = r Y
fi (t1 + t2) = ±r P
2(t - t ) 2 2
Now, m( AB) = 22 21 = =±
(t2 - t1 ) (t1 + t2 ) r R
29. Here a = 1 X¢ X
The equation of normal to the parabola y2 = 4x
is y = mx – 2am – am3 Q
fi y = mx – 2m – m3
which is passing through (9, 6). Y¢

CG_04a.indd 60 2/6/2017 4:03:06 PM


Parabola 4.61

Since R lies on y = 2x + a, so Now,


Ê 1ˆ 0 3 1
aÁt - ˜ = -a 1
Ë t¯ ar(DEFG) = 0 4t 1
2
Ê 1ˆ 4t 2 8t 1
fi ÁË t - ˜¯ = -1
t 1
= [4t 2 (3 - 4t )]
2 2 2
fi Ê 1ˆ Ê 1ˆ = 2t2(3 – 4t)
ÁË t + ˜¯ = ÁË t - ˜¯ + 4 = 1 + 4 = 5
t t = (6t2 – 8t3)
2 dA
Ê 1ˆ fi = 12t - 24t 2 = 12t (1 - 2t )
Thus, PQ = a Á t + ˜ = 5a dt
Ë t¯
For maximum or minimum,
Êa 2a ˆ dA
33. Here, P = (at 2 , 2at ), Q = Á 2 , - ˜ = 0 gives t = 0, 1/2
Ët t ¯ dt
Y – + –
0 1/2
P So, t = 1/2 has a point of local maxima.
Thus, G = (0, 4t) = (0, 2) fi y1 = 2
F = (x0, y0) = (4t2, 8t) = (1, 4) fi y0 = 4
Ê 3 1ˆ 1 1

O q
X Area = 2 Á - ˜ = 2 ¥ =
S Ë 4 2¯ 4 2
So, y = mx + 3 passes through (1, 4).
Q Thus, m = 1.
35. The equation of any tangent to the parabola can be con-
Y¢ sidered as
a 2
1 y = mx + = mx +
Now, t - = -1 m m
t
Y
2 R(2, 4)
Ê 1ˆ
fi ÁË t + ˜¯ = 1 + 4 = 5 P(–1, 1)
t

fi Ê 1ˆ
ÁË t + ˜¯ = 5 X¢ O X
t
Q(–1, –1)
Ê2 ˆ 2 Ê t + 1ˆ
+ 2t ÁË ˜
Á ˜ t¯ 2 5
tan q = Á t
S(2, –4)
= =-
Ë 1 - 4 ˜¯

-3 3
i.e. m2x – my + 2 = 0
34. The tangent at F(4t2, 8t), is As we know that the length of the perpendicular drawn
yy1 = 8(x + x1) from the centre to the tangent to the circle is equal to
fi y ◊ 8t = 8(x + 4t2) the radius of a circle.
fi y ◊ t = (x + 4t2) 2
Thus, = 2
Y m + m2
4

E(0, 3) fi m4 + m2 = 2
F fi m4 + m2 – 2 = 0
G(0, y1) fi (m2 + 2)(m2 – 1) = 0
fi m = ±1
X¢ X
O Hence, the equation of the tangents are
y = x + 2, y = –x – 2
Therefore, the points P, Q are (–1, 1), (–1, –1) and R, S
Y¢ are (2, 4) and (2, –4) respectively.
Thus, the area of the equadrilateral PQRS
Put x = 0, then y = 4t
1
Thus, pt G is (0, 4t) = ¥ (2 + 8) ¥ 3 = 15
2

CG_04a.indd 61 2/6/2017 4:03:06 PM


4.62 Coordinate Geometry Booster

36. Given P = (at2, 2at) Ê 1 ˆ


Since PQ is a focal chord, so the co-ordinates of Q are fi 2yt = a Á t 2 + 2 + 2˜
Ë t ¯
Êa 2a ˆ
ÁË 2 , - ˜¯ . Ê 1ˆ
2
t t fi 2yt = a Á t + ˜
Also, R = (ar2, 2ar), S = (as2, 2as) Ë t¯
and K = (2a, 0) 2
a Ê 1ˆ a (t 2 + 1) 2
It is given that, fi y= Á t + ˜ =
2t Ë t¯ 2t 3
m(PK) = m(QR)
2a 38. Image of y = –5 about the line x + y + 4 = 0 is x = 1
2ar + Hence, the required distance AB = 4
0 - 2at t
fi =
2a - at 2 ar 2 - a 39. Equation of normals are
t2 x + y = 3 and x – y = 3
1 Hence, the distance from (3, –2) on both the normals is
r+
t t r
fi =
t2 - 2 r2 - 1 3- 2-3
Thus, =r
t2 2
1
r+ fi r2 = 2
t t
fi = 40.
t 2 - 2 Ê r + 1ˆ Ê r - 1ˆ
ÁË ˜Á ˜
t¯ Ë t¯
t 1
fi 2
=
t -2 Ê 1ˆ
ÁË r - ˜¯
t
Ê 1 ˆ 2
fi ÁË r - ˜¯ t = (t - 2)
t
fi rt – 1 = (t2 – 2)
Ê t 2 - 1ˆ Ê 1ˆ
fi r=Á = t- ˜
Ë t ˜¯ ÁË t¯
Ê 16a 8a ˆ
Ê a 2a ˆ Clearly, P = (at2, 2at) and Q = Á 2 , - ˜
37. Now, S = (as 2 , 2as ) = Á 2 , ˜ Ë t t ¯
Ët t ¯
Tangent at P, y ◊ t = x + at2 …(i) Area of the triangle OPQ = 3 2
Tangent at S, yy1 = 2a(x + x1) 1
◊ OP ◊ OQ = 3 2
2
2a Ê aˆ
fi y◊ = 2a Á x + 2 ˜ 1 - 4a 16
t Ë t ¯ at t 2 + 4 ¥ +4 =3 2
2 t t2
Ê aˆ
fi y◊= tÁx + 2˜ t2 - 3 2 t + 4 = 0
Ë t ¯
Ê t2 ˆ
2a 1Ê aˆ P = (at 2 , 2at ) = Á , t ˜
Normal at S, y - = - Áx - 2˜ Ë2 ¯
t tË t ¯
when t = 2, P = (1, 2)
Ê aˆ
fi y ◊ t - 2a = - Á x - 2 ˜
Ë t ¯ when t = 2 2, P = (4, 2 2)
Ê aˆ 41. Equation of tangent at P ( 2, 1) is
fi y ◊ t = - Á x - 2 ˜ + 2a …(ii)
Ë t ¯ 2x+ y=3
Solving (i) and (ii), we get
If centre of C2 at (0, a) and the radius equal to 2 3
a
fi yt - at 2 = 2 + 2a - yt a -3
t fi 2 3=
a 3
fi 2yt = at 2 + 2 + 2a
t fi a = –3, 9

CG_04a.indd 62 2/6/2017 4:03:08 PM


Parabola 4.63

(a) Q2Q3 = 12 (d) ar(DPQ2Q3)


(b) R2R3 = length of the transverse common tangent 1
= ¥ 12 ¥ 2 = 6 2
= (Q2Q3 ) 2 - (r1 + r2 ) 2 2
= (12) 2 - (2 3 + 2 3) 2 42. Equation of normal of parabola is
y + tx = 2t + t3
=4 6

Normal passes through S(2, 8)


8 + 2t = 2t + t3
t3 = 8
t=2
Hence, P = (4, 4) and SQ = radius = 2

(c) Area of DOR2R3


1 r
= ¥ R2 R3 ¥ ^ distance from O to the line
2
1
= ¥4 6¥ 3=6 2
2

CG_04a.indd 63 2/6/2017 4:03:09 PM


CG_04a.indd 64 2/6/2017 4:03:09 PM
Ellipse 5.1

C H A P T E R

5 Ellipse

CONCEPT BOOSTER Definition 2


An ellipse is the set of all points in a plane, the sum of whose
distances from two fixed points is a constant, i.e.
1. INTRODUCTION SP + S¢P = 3a
An oval is generally regarded as any ovum (egg)-shaped
smooth, convex closed curve. The word convex means any P
chord connecting two points of the curve lies completely
within the curve, and smooth means that the curvature does
not change rapidly at any point. The ellipse is a typical oval, S¢ S
but a very particular one with a shape that is regular and can
be exactly specified.
It has two diameters at right angles that are lines of sym-
metry. It is best to reserve the word ellipse for real ellipses, Definition 3
and to call others ovals. A diameter is any chord through the A conic
centre of the ellipse. The diameters that are lines of symme-
ax2 + 2hxy + by2 + 2gx + 2fy + c = 0
try are called the major axis (2a), and the minor axis (2b),
where a > b. If a = b, we have the very special ellipse, the represents an ellipse if
circle, which has enough special properties that it should be (i) D π 0 and
distinguished from an ellipse, though, of course, it has all
the properties of an ellipse in addition to its own remarkable (ii) h2 – ab < 0, where
properties. a h g
A vertex of a curve is a point of maximum or minimum D= h b f
radius of curvature. An ellipse has vertices at the ends of
the major axis (minimum) and at the ends of the minor axis g f c
(maximum). Definition 4
Let z, z1 and z3 be three complex numbers such that |z – z1| +
2. MATHEMATICAL DEFINITIONS |z – z2| = k, where k > |z1 – z2| and k be a positive real number,
the locus of z is an ellipse.
Definition 1
Y
It is the locus of a point which moves
in a plane in such a way that its dis- M P P(z)
tance from a fixed point is a constant
ratio from a fixed straight line. This S (Focus) X¢ X
A¢ F1(z1) O F2(z2) A
ratio is always less than 1. This fixed
point is called the focus and the fixed Z (Directrix)
straight line is called the directrix. The
constant ratio is called the eccentricity.

CG_05.indd 1 2/6/2017 4:04:48 PM


5.2 Coordinate Geometry Booster

3. EQUATION OF AN ELLIPSE
x2 y 2
fi + = 1, b 2 = a 2 (1 - e 2 ) , …
Y a 2 b2
This is the standard equation of an ellipse.
B P
M¢ M
X¢ X 4. PROPERTIES OF AN ELLIPSE
Z¢ A¢ S¢ C S N A Z
B¢ Y

Y¢ B P
M¢ M
Let S be the focus and ZM be the directrix of the ellipse. Draw X¢ X
L¢ A¢ S¢ C S N A L
SZ ^ ZM. Divide SZ internally and externally in the ratio
e : 1 (e < 1) and let A and A¢ be the internal and external B¢
point of division.

Then SA = e AZ …(i)
and SA¢ = e A¢Z …(ii) Centre
Clearly A and A¢ will lie on the ellipse. A point inside the ellipse which is the mid-point of the line
Let AA¢ = 2a and take C be the mid-point of AA¢ as origin. segment linking the two foci, i.e. the intersection of the major
Thus, CA = CA¢ = a …(iii) and minor axes. Here C = (0, 0).
Let P(x, y) be any point on the ellipse referred to CA and Major/minor axis
CB as co-ordinate axes. The longest and the shortest diameters of an ellipse are
Adding Eqs (i) and (ii), we get known as the major axis and the minor axis respectively. The
SA + SA¢ = e(AZ + A¢Z) length of the major axis is equal to the sum of the two genera-
tor lines.
fi AA¢ = e(CZ – CA + CA¢ + CZ)
Here, major axis = AA¢ = 2a,
fi AA¢ = e(2CZ) and minor axis = BB¢ = 2b
fi 2a = 2eCZ
Semi-major/Half the major axis
a
fi CZ = The distance from the centre to the farthest point on the el-
e lipse is known as the semi-major axis.
a
Thus the directrix ZM is x = CZ = . Semi-minor axis/Half the minor axis
e
Again subtracting Eqs (i) from (ii), we get The distance from the centre to the closest point on the ellipse
is known as the semi-minor axis.
SA – SA¢ = e(A¢Z – AZ)
fi (CA¢ + CS) – (CA – CS) = e(AA¢) Directrices
LM and L¢M¢ are two directrices of the ellipse.
fi 2CS = e(AA¢)
a a
fi 2CS = e(2a) Thus LM: x = and L¢M¢: x = - .
e e
fi CS = ae 2a
The distance between two directrices: LL ¢ =
Thus the focus is S(CS, 0) = S(ae, 0) e
Foci (Focus points)
Now draw PM ^ ZM,
The two points that define the ellipse is known as the foci.
SP
=e Here S = (ae, 0) and S¢ = (–ae, 0)
PM
Distance between two foci: SS¢ = 2ae
fi SP2 = e2PM2
2 Perimeter
2 a
2 ˆ 2Ê The perimeter is the distance around the ellipse, i.e.
fi ( x - ae) + y = e Á - x˜
Ëe ¯
È3 ˘
Perimeter = p ¥ Í (a + b) + ab ˙ .
fi x2 + a2e2 – 2aex + y2 = a2 – 2aex + e2x2 Î2 ˚
It is not easy to calculate.
fi x2(1 – e2) + y2 = a2(1– e2)
x2 y2 Area
fi + =1 The number of square units it takes to fill the region inside an
a 2 a 2 (1 - e 2 )
ellipse is called the area of an ellipse.

CG_05.indd 2 2/6/2017 4:04:49 PM


Ellipse 5.3

Thus the co-ordinates of L and L¢ are


x2 y 2
If the equation of ellipse is + = 1 , then its area is pab.
a 2 b2 Ê b2 ˆ Ê b2 ˆ
ÁË ae, ˜¯ and ÁË ae, - ˜¯ .
If the equation of ellipse is a a
2p Hence the length of the latus rectum,
Ax2 + Bxy + Cy2 = 1, then its area is .
4AC - B 2 2b 2
LL¢ = .
a
Chord
A line segment linking any two points on an ellipse is known Relation amongst a, b, and e:
as the chord of the ellipse. b2 = a2(1 – e2)
Focal chord Eccentricity (e)
A chord of the ellipse passing through its focus is called the
focal chord. b2 a 2 - b2
e = 1- =
Focal distances a2 a2
Let P(x, y) be any point on the ellipse. Auxiliary circle
Êa ˆ The circle described on the major axis of an ellipse as diam-
Here, SP = ePM = e Á - x˜ = a - ex
Ëe ¯ eter is called an auxilliary circle.
Êa ˆ Relation to a circle
S¢P = ePM¢ = e ÁË + x˜¯ = a + ex
e A circle is actually a special case of an ellipse. In an ellipse,
Now, SP + S¢P = a – ex + a + ex = 2a = constant. if you make the major and minor axes of the same length, the
Thus the sum of the focal distances of a point on the ellipse result is a circle, with both foci at the center C.
is constant.
5. PARAMETRIC EQUATION OF AN ELLIPSE
Notes
Focal distances are also known as focal radii of the ellipse. Y

Vertices P
The vertices of the ellipse are the points where the ellipse N
meets its major axis. f
X¢ M X
Here, A = (a, 0) and A¢ = (–a, 0) are the vertices of the ellipse. C
Co-vertices
The co-vertices of the ellipse are the points where the ellipse
meets its minor axis. Y¢
Here, B = (0, b) and B¢ = (0, –b) are the co-vertices of the x2 y 2
ellipse. Let the equation of the ellipse be + = 1.
a 2 b2
Double ordinate The equation of its auxilliary circle is x2 + y2 = a2.
It is a chord perpendicular to the major axis and intersects the Let Q be a point on the auxilliary circle x2 + y2 = a2 such
curve in two distinct points. that QP produced is perpendicular to the x-axis.
Latus rectum Thus P and Q are the corresponding points on the ellipse
It is a double ordinate, perpendicular to the major axis and and the auxilliary circle.
passes through the foci. Here LSL¢ and L1SL1¢ are two latus Let –QCA = j, where 0 £ j < 2p
recta. Let Q = (a cos j, a sin j) and P = (a cos j, y).
Length of the LR x2 y 2
Since P lies on the ellipse + = 1 , so we can write
Let the co-ordinates of L and L¢ be (ae, y1) and (ae, –y1). a 2 b2
x2 y 2 a 2cos 2j y 2
Since L lies on the ellipse 2 + 2 = 1 , so we have + 2 =1
a b a2 b
a 2e 2 y12 fi y2 = b2 sin2 j
+ 2 =1
a2 b fi y = b sin j
Ê b2 ˆ b4 since P lies in the 1st quadrant.
fi y12 = b 2 (1 - e 2 ) = b 2 Á 2 ˜ = 2
Ëa ¯ a Thus the parametric equations of the ellipse are
fi y1 = b2/a x = a cos j, y = b sin j.

CG_05.indd 3 2/6/2017 4:04:49 PM


5.4 Coordinate Geometry Booster

Notes Proof
B
P(r cos q, r sin q)
Any point, say P, on the ellipse can be considered as (a cos
j, b sin j). Since the point is known when f is given, then r
it is often called ‘the point f’ or P(j).. A¢ A
S¢ C S

6. IMPORTANT PROPERTIES RELATED TO CHORD AND



FOCAL CHORD We have, SP = a – ex and S¢P = a + ex.
(i) Equation of the chord joining the points P(j1) and Thus,
Q(j2) SP. S¢P = (a2 – e2x2) = (a2 – a2e2 cos2 q)
P(f1) = a2 + (b2 – a2) cos2 q
= a2(1 – cos2 q) + b2 cos2 q
= a2 sin2 q + b2 cos2 q
(iv) If PQ be a focal chord and S, S¢ are the foci of an el-
Q(f2)
lipse, the perimeter of the triangle described by DS¢PQ
The equation of the chord joining the points P(a cos j, is 4a.
b sin j1) and Q(a cos j2, b sin j2) is Proof
x Ê j + j2 ˆ y Ê j1 + j 2 ˆ P
cos Á 1 ˜¯ + sin ÁË ˜
a Ë 2 b 2 ¯
Ê j - j2 ˆ S¢ S
= cos Á 1
Ë 2 ˜¯
(ii) The length of a radius vector from the centre drawn in Q
a given direction We have, perimeter of the DS¢PQ
B P(r cos q, r sin q) = S¢P + S¢Q + PQ
= (S¢P + S¢Q) + (SP + SQ)
r = (S¢P + SP) + (S¢Q + SQ)
A¢ A = 2a + 2a = 4a
C
(v) The length of the focal chord of an ellipse which makes
2ab 2
B¢ an angle q with the major axis is 2 2 .
As we know that the equation of the ellipse is a sin q + b 2cos 2q
Proof
x2 y 2
+ = 1. P
a 2 b2
Put x = r cos q, y = r sin q, we have q
2 2 2 2 S
r cos q r sin q
2
+ = 1.
a b2
Q
2 a 2b 2
fi r = 2 2 Let the chord be PQ, where P = (x1, y1), Q = (x2, y2) and
b cos q + a 2sin 2q S(ae, 0) be the focus.
ab The chord PQ be
fi r=
b cos q + a 2sin 2q
2 2
(y – 0) = tan q (x – ae) …(i)
which is the required distance from the centre of the Now PQ = SP + SQ
= a – ex1 + a – ex2 = 2a = e(x1 + x2)
x2 y 2
point P(q) on the ellipse 2 + 2 = 1 . Let the equation of the ellipse be
a b
x2 y 2
(iii) Product of the focal radii of an ellipse from any point + =1 …(ii)
a 2 b2
x2 y 2
P(q) on the ellipse 2 + 2 = 1 is Now PQ = SP + SQ
a b
= a – ex1 + a – ex2
a2 sin2 q + b2 cos2 q. = 2a – e(x1 + x2)

CG_05.indd 4 2/6/2017 4:04:50 PM


Ellipse 5.5

From Eqs (i) and (ii), we get Ê a - bˆ


cos Á
x 2 tan 2q ( x - ae) 2 Ë 2 ˜¯
+ =1 fi =e
a2 b2 Ê a + bˆ
cos Á
fi b2x2 + a2 tan2 q(x – ae)2 = a2b2 Ë 2 ˜¯
fi b2x2 + a2 tan2 q(x2 – 2aex + a2e2) = a2b2 Ê a - bˆ Ê a + bˆ
cos Á ˜ + cos Á
fi (b2 + a2 tan2 q)x2 – 2a3e tan2 q x Ë 2 ¯ Ë 2 ˜¯ e - 1
fi =
+ a2(a2e2 tan2 q – b2) = 0 Ê a - bˆ Ê a + bˆ e + 1
cos Á - cos
Let its roots are x1 and x2. Ë 2 ˜¯ ÁË
2 ¯
˜

2a 3e tan 2q Ê aˆ Ê bˆ e - 1
Then x1 + x2 = fi tan Á ˜ tan Á ˜ =
Ë 2¯ Ë 2¯ e +1
b 2 + a 2 tan 2q
Therefore, PQ = 2a – e(x1 + x2) Hence, the result.
Ê 2a 3e tan 2q ˆ (vii) If a and b are the eccentric angles of the extremities
= 2a - e Á 2 ˜ x2 y 2
Ë b + a 2 tan 2q ¯ of a focal chord of the ellipse 2 + 2 = 1 , then the
a b
Ê b 2 + a 2 tan 2q - a 2e 2 tan 2q ˆ eccentricity of the ellipse is
= 2a Á ˜
Ë b 2 + a 2 tan 2q ¯ sin a + sin b
2 2 2
Ê b + a (1 — e ) tan q ˆ 2 sin (a + b )
= 2a Á ˜
Ë b 2 + a 2 tan 2q ¯ Proof
Ê b 2 + b 2 tan 2q ˆ P
= 2a Á 2 ˜
Ë b + a 2 tan 2q ¯
S
Ê b 2 (1 + tan 2q) ˆ
= 2a Á 2 ˜
Ë b + a 2 tan 2q ¯
Q
2ab 2
= 2 2 The equation of the chord joining the points P(a) and
a sin q + b 2cos 2q
P(b) is
Hence the result. x Ê a + bˆ y Ê a + bˆ Ê a - bˆ
cos Á ˜ + sin Á ˜ = cos Á
(vi) If P(a) and P(b) are the extremities of a focal chord, a Ë 2 ¯ b Ë 2 ¯ Ë 2 ˜¯
then
which is passing through (ae, 0),
Êaˆ Ê bˆ e -1
tan Á ˜ tan Á ˜ = .
Ë 2¯ Ë 2¯ e +1 We have,
Ê a + bˆ Ê a — bˆ
Proof e cos Á = cos Á
P(a) Ë 2 ˜¯ Ë 2 ˜¯
Ê a + bˆ Ê a + bˆ
fi e ¥ 2 sin Á cos Á
Ë 2 ˜¯ Ë 2 ˜¯
Ê a + bˆ Ê a - bˆ
= 2 sin Á cos Á
Q(b) Ë 2 ˜¯ Ë 2 ˜¯
Let the equation of the ellipse be fi e ¥ sin (a + b) = sin a + sin b
2 2
x y sin a + sin b
+ = 1. fi e=
a 2 b2 sin (a + b)
Equation of the chord PQ is Hence, the result.
x Êa + bˆ y Êa + bˆ Êa - bˆ
cos Á + sin Á = cos Á
a Ë 2 ˜¯ b Ë 2 ˜¯ Ë 2 ˜¯ 7. POSITION OF A POINT WITH RESPECT TO AN ELLIPSE
which is passing through the focus (ae, 0), then The point (x1, y1) lies outside, on or inside the ellipse
x2 y 2
Êa + bˆ Êa - bˆ + = 1 according as
e cos Á = cos Á a 2 b2
Ë 2 ˜¯ Ë 2 ˜¯

CG_05.indd 5 2/6/2017 4:04:50 PM


5.6 Coordinate Geometry Booster

x12 y12 10. VARIOUS FORMS OF TANGENTS


+ -1> , = , < 0
a 2 b2 (i) Point form
x2 y 2
8. INTERSECTION OF A LINE AND AN ELLIPSE The equation of the tangent to the ellipse 2 + 2 = 1
a b
at (x1, y1) is
x2 y 2
The line y = mx + c intersects the ellipse + = 1 into two xx1 yy1
a 2 b2 + 2 =1
a2 b
(i) real and distinct points if x2 < a2m2 + b2.
(ii) coincident points if c2 = a2m2 + b2. (ii) Parametric form
x2 y 2
(iii) imaginary points if c2 > a2m2 + b2. The equation of the tangent to the ellipse 2 + 2 = 1
at (a cos q, b sin q) is a b
Also
(iv) The line y = mx + c will be a tangent to the given ellipse x y
cos q + sin q = 1
if a b
c2 = a2m2 + b2.
(iii) Slope form
(v) The co-ordinates of the point of contact is x2 y 2
Ê a 2 m b2 ˆ The equation of the tangent to the ellipse 2 + 2 = 1
a b
ÁË ± , ± ˜ which is also known as the m-point on in terms of the slope m is
c c¯
the ellipse. y = mx + a 2 m 2 + b 2
(vi) The equation of any tangent to the ellipse can be con-
(iv) The co-ordinates of the points of contact are
sidered as y = mx + a 2 m 2 + b 2 .
Ê a2m b2 ˆ
Á ,± ˜.
(vii) The line lx + my + n = 0 will be a tangent to the ellipse Ë a 2 m2 + b2 a 2 m2 + b2 ¯
x2 y 2
+ = 1, if a2l2 + b2m2 = n2
a 2 b2 (v) The point of intersection of the tangents at P(q) and
Q(j) is
9. THE LENGTH OF THE CHORD INTERCEPTED Ê Ê q + jˆ Ê q + jˆ ˆ
BY THE ELLIPSE ON THE LINE Y = mx + C. Á a cos ÁË 2 ˜¯ b sin ÁË 2 ˜¯ ˜
Á , ˜
Let the equation of the ellipse be Á cos Ê q — j ˆ cos Ê q — j ˆ ˜
ÁË ÁË ˜ ÁË ˜
2 ¯ 2 ¯ ˜¯
x2 y 2
+ =1
a 2 b2
11. DIRECTOR CIRCLE
x 2 (mx + c) 2
fi + =1 The locus of the point of intersec-
P(h, k)
a2 b2
tion of two perpendicular tangents
fi (a2m2 + b2)x2 + 2a2mcx + a2(c2 – b2) = 0 to an ellipse is known as the direc-
Let its roots are x1, x2. tor circle.
a 2 (c 2 - b 2 ) The equation of the director cir-
2a 2 mc
Then x1 + x2 = - and x ◊ x = .
a 2m2 + b2
1 2
a 2 m2 + b2 x2 y 2
cle to an ellipse 2
+ 2 = 1 is x2 +
a b
2ab a 2 m 2 + b 2 - c 2 y2 = a2 + b2.
Thus, x1 - x2 = .
a 2 m2 + b2
12. PAIR OF TANGENTS
Hence the length of the chord
Equation of a pair of tangents from a point (x1, y1) to an
= ( x1 - x2 ) 2 + ( y1 - y2 ) 2 x2 y 2
ellipse 2 + 2 = 1 is
= ( x1 - x2 ) 1 + m 2 a b
Ê x2 y 2 ˆ Ê x12 y12 ˆ Ê xx1 yy1 ˆ 2
2ab ¥ 1 + m 2 a 2 m 2 + b 2 - c 2 1 1
= Á 2 + - ˜Á 2 + - ˜ = ÁË 2 + 2 - 1˜¯
a 2 m2 + b2 Ëa b2 ¯Ëa b2 ¯ a b

CG_05.indd 6 2/6/2017 4:04:51 PM


Ellipse 5.7

P Let Q(a, b) be any point in the xy-plane.


Equation (i) passes through Q(a, b), so we have
aa sec q – bb cosec q = a2 – b2
Ê 2 Ê qˆ ˆ Ê 2 Ê qˆ ˆ
Á 1 + tan ÁË 2 ˜¯ ˜ Á 1 + tan ÁË 2 ˜¯ ˜
Q R
2 2
fi aaÁ ˜ - bb Á ˜ =a -b
Á 1 - tan 2 Ê q ˆ ˜ Á 2 tan Ê q ˆ ˜
ÁË ÁË ˜¯ ˜¯ ÁË ÁË ˜¯ ˜¯
2 2
13. VARIOUS FORMS OF NORMALS Ê Ê qˆˆ Ê qˆ Ê Ê qˆˆ
fi 2aaÁ1 + tan 2 Á ˜ ˜ tan Á ˜ - b b Á1 - tan 4 Á ˜ ˜
T Ë Ë 2¯ ¯ Ë 2¯ Ë Ë 2¯ ¯
Ê qˆ Ê Ê qˆˆ
P = 2(a 2 - b 2 ) tan Á ˜ Á1 - tan 2 Á ˜ ˜
Ë 2¯ Ë Ë 2¯ ¯
fi bbt4 + 2(a2 – b2 + aa)t3 –2(a2 – b2 – aa)t – bb = 0
…(ii)
Ê qˆ
where t = tan Á ˜
N Ë 2¯

Here, PT be a tangent and PN be a normal. The above equation will give four values of t say, t1, t2, t3, t4.
The angle between the tangent and the normal is 90°. Corresponding to these four values of t, we will get 4
(i) Point form points, say A, B, C, D on the ellipse, the normals to which
x2 y 2 pass through the point Q(a, b).
The equation of the normal to the ellipse 2 + 2 = 1
a b Hence, in general four normals can be drawn from any
at the point (x1, y1) is point to an ellipse.
a 2 x b2 y (i) Co-normal points
- = a 2 - b2 . Let P, Q, R, S are four points on the ellipse. If the nor-
x1 y1
mals at these points meet at a point, say M, then these
(ii) Parametric form four points are known as co-normal points.
x2 y 2
The equation of the normal to an ellipse 2 + 2 = 1 at
the point (a cos q, b sin q) is a b R Q

ax sec q – by cosec q = a – b .
2 2
M
(iii) Slope form
The equation of the normal in terms of slope is
m( a 2 - b 2 )
y=mx P
a 2 + b2 m2 S
x2 y 2
(iv) The line y = mx + c is a normal to an ellipse 2 + 2 = 1 (ii) If a, b, g, d be the eccentric angles of the four points
if a b
on the ellipse such that the normals at these points are
2 2 2 2
m (a - b ) concurrent, then a + b + g + d = (2n + 1) p, n Π1.
c2 = .
(a 2 + b 2 m2 ) As we know that if four normals are concurrent at a
point, say, M(a, b), then
(v) The straight line lx + my + n = 0 is a normal to an
bbt2 + 2(a2 – b2 + aa)t3 –2(a2 – b2 – aa)t = bb = 0,
x2 y 2
ellipse 2 + 2 = 1 if Ê qˆ
a b where t = tan Á ˜
Ë 2¯
a 2 b 2 (a 2 - b 2 )2
+ = Êa b g dˆ S1 - S3
l 2 m2 n2 Now, tan Á + + +
Ë2 2 2 ˜¯ =
2 1 - S2 + S4
14. NUMBER OF NORMALS ARE DRAWN TO AN - 2[aa + (a 2 - b 2 )] 2[ab - (a 2 - b 2 )]
+
ELLIPSE FROM A POINT TO ITS PLANE bb bb
=
1- 0 -1
x2 y 2 =
The equation of the normal to the ellipse + = 1 at P(q) is
a 2 b2 Ê a b g dˆ
fi cot Á + + + ˜ = 0
ax sec q – by cosec q = a – b2 2
…(i) Ë 2 2 2 2¯

CG_05.indd 7 2/6/2017 4:04:51 PM


5.8 Coordinate Geometry Booster

Êa b g dˆ Ê 2n + 1ˆ Properties related to pole and polar


fi cot Á + + + ˜¯ = 0 = cot ÁË ˜p (i) The polar of the focus is the directrix.
Ë2 2 2 2 2 ¯
(ii) Any tangent is the polar of the point of contact.
Ê a b g d ˆ Ê 2n + 1ˆ
fi ÁË + + + ˜¯ = ÁË ˜p (iii) The pole of a line lx + my + n = 0 with respect to the
2 2 2 2 2 ¯
x2 y 2 Ê a 2l b 2 m ˆ
fi a + b + g + d = (2n + 1)p. ellipse 2 + 2 = 1 is Á - ,- ˜.
a b Ë n n ¯
Hence, the result.
(iv) The pole of a given line is the same as the point of in-
15. CHORD OF CONTACT tersection of tangents at its extremities.
(v) If the polar of P(x1, y1) passes through Q(x2, y2), the
P polar of Q(x2, y2) goes through P(x1, y1) and such points
are said to be conjugate points.
R
(vi) If the pole of a line lx + my + n = 0 lies on the another
line l¢x + m¢y + n¢ = 0, the pole of the second line will lie
Q on the first and such lines are said to be conjugate lines.

Chord of contact 18. DIAMETER


The equation of the chord of contact of tangents drawn from The locus of the mid-points of a system of parallel chords of
a point (x1, y1) to an ellipse an ellipse is called a diameter and the point where the diam-
x2 y 2 xx yy eter intersects the ellipse is called the vertex of the diameter.
2
+ 2 = 1 is 21 + 21 = 1
a b a b B

16. CHORD BISECTED AT A GIVEN POINT (h


,k
)
A¢ M A
The equation of the chord bi- B
sected at a point (x1, y1) to an
x2 y 2 M(h, k) y = mx + c
ellipse 2 + 2 = 1 is B¢
a b A
2 2 Let (h, k) be the mid-point of the chord, then
xx1 yy1 x y
+ 2 - 1 = 12 + 12 - 1
a 2
b a b x2 y 2
y = mx + c of the ellipse 2 + 2 = 1 .
i.e. T = S1 a b
Then T = S1
17. POLE AND POLAR xh yk h 2 k 2
fi + = +
a 2 b2 a 2 b2
The equation of the polar of an ellipse
b2h
x2 y 2 fi k=-
+ =1 a2m
a 2 b2 b2 x
Hence, the locus of the mid-point is y = - .
from a point (x1, y1) is a2m
xx1 yy1 19. CONJUGATE DIAMETERS
+ 2 = 1,
a2 b
where S P

(x1, y1) is the pole of polar.


Q C
B R
A Q
P
C
Polar Two diameters are said to be conjugate when each bisects all
chords parallel to the other.
Q¢ If y = m1x and y = m2x be two conjugate diameters of an
D
b2
ellipse, then m1m2 = - 2 .
a

CG_05.indd 8 2/6/2017 4:04:52 PM


Ellipse 5.9

Let PQ and RS be two conjugate diameters. Proof


Then the co-ordinates of the four extremities of two con- D P
jugate diameters are
P(a cos j, b sin j),
C
Q(–a cos j, – b sin j),
S(– a sin j, b cos j),
Let CP and CD be two conjugate semi-diameters of an
and R(a sin j, –b cos j)
x2 y 2
Properties of conjugate diameters ellipse + = 1 and the eccentric angle of P is f.
(i) Prove that the eccentric angles of the ends of a pair a 2 b2
p
of conjugate diameters of an ellipse differ by a right Thus the eccentric angle of D is + j .
angle. 2
Proof Therefore the co-ordinates of P and D are (a cos j, b
sin j) and
R(f¢) P(f)
Ê Êp ˆ Êp ˆˆ
ÁË a cos ÁË 2 + j˜¯ , b sin ÁË 2 + j˜¯ ˜¯

C i.e. (–a sin j, b cos j).


Thus CP2 + CD2
Q S = (a2 cos2 j + b2 sin2 j) + (a2 sin2 j + b2 cos2 j)
= a2 + b2
Let PCQ and RCS be two conjugate diameters of an Hence, the result
x2 y 2 (iii) Prove that the product of the focal distances of a point
ellipse + = 1. on an ellipse is equal to the square of the semi-diameter,
a 2 b2 which is conjugate to the diameter through the point.
Then the co-ordinates are P(a cos j, b sin j) and Proof
R(a cos j¢, a sin j¢). D P
b
Now, m1 = Slope of CP = tan j and
a
S(–æ, 0) C S(æ, 0)
b
m2 = Slope of CR = tan j ¢
a
Since the diameters PCQ and RCS are conjugate diam-
Let CP and CD be the conjugate diameters of the
eters, then
ellipse.
b2 Let P = (a cos j, b sin j), then the co-ordinates of D is
m1 ◊ m2 = -
a2 (–a sin j, b cos j).
Thus,
b2 b2 SP ◊ S¢P = (a – ae cos j) ◊ (a + ae cos j)
fi tan j tan j ¢ = -
a2 a2 = a2 – a2e2 cos2 j
fi tan j tan j¢ = –1 = a2 – (a2 – b2) cos2 j
= a2 sin2 j + b2 cos2 j
Êp ˆ
fi tan j = - cot j¢ = tan Á + j¢˜ = CD2
Ë2 ¯
Hence, the result.
p (iv) Prove that the tangent at the extremities of a pair of
fi j= + j¢
2 conjugate diameters form a parallelogram whose area
is constant and equal to the product of the axes.
p
fi j - j¢ = Proof
2 Y
Hence, the result.
R P
(ii) Prove that the sum of the squares of any two conjugate
semi-diameters of an ellipse is constant and equal to
the sum of the squares of the semi-axes of the ellipse, X¢ X
C
i.e.
CP2 + CD2 = a2 + b2. Q B

CG_05.indd 9 2/6/2017 4:04:53 PM


5.10 Coordinate Geometry Booster

Let PCQ and RCS be two conjugate diameters of the (v) Equi-conjugate diameters
x2 y 2 B
ellipse 2 + 2 = 1 . R P
a b
Then the co-ordinates of P, Q, R, and S are A¢ A
P(a cos j, b sin j), Q(–a cos j, – b sin j), C
R(–a sin j, b cos j) and S(a sin j, –b cos j) respec-
tively. B¢
Equations of tangents at P, R, Q and S are
Two conjugate diameters are said to be equi-conjugate
x y diameters if their lengths are equal.
cos j + sin j = 1,
a b
a 2 + b2
x y i.e. CP = CR =
- sin j + cos j = 1, 2
a b
x y
- cos j - sin j = 1
a b
20. REFLECTION PROPERTY OF AN ELLIPSE
x y If an incoming light ray passes through one focus (S) strikes
and sin j - cos j = 1
a b the concave side of the ellipse, it will get reflected towards
Thus, the tangents at P and Q are parallel. other focus.
Also the tangents at R and S are are parallel. Y¢
Hence, the tangents at P, R, Q, S form a parallelogram.
Area of the parallelogram = MNM¢N¢ P
= 4(the area of the parallelogram CPMR)
ab X¢ X
= 4 ¥ a 2cos 2j + b 2sin 2j ¥ S¢ C S
a 2cos 2j + b 2sin 2j
= 4ab
= constant
Hence, the result. Y¢

E XERCISES

LEVEL I 5. Find the equation of the ellipse whose axes are co-ordi-
nate axes and foci are (±2, 0) and the eccentricity is 1/2.
(Problems based on Fundamentals) 6. If the distance between the foci of an ellipse is equal
to the length of its latus rectum, the eccentricity is
ABC OF AN ELLIPSE 5 -1
.
2
1. Find the centre, vertices, co-vertices, lengths of major x2 y 2
and minor axes, eccentricity, lengths of latus rectum, 7. Find the eccentricity of the ellipse 2 + 2 = 1 whose
a b
equation of directrices and the end-points of a latus latus rectum is half of its major axis.
recta. 8. Find the eccentric angle of a point on the ellipse
(i) 9x2 + 16y2 = 144
x2 y 2
(ii) 2x2 + 3y2 – 4x – 12y + 8 = 0 + = 1 whose distance from the centre of the
6 2
2. Find the sum of the focal distances of any point on the ellipse is 5 .
ellipse 16x2 + 25y2 = 400. 9. Find the area of the greatest rectangle that can be in-
x2 y2 x2 y 2
3. If the equation + = 1 represents of an scribed in the ellipse + = 1.
10 - a a — 4 16 9
ellipse such that the length of the interval, where a lies, 10. Find the locus of a point whose co-ordinates are given
is m, find m. by x = 3 + 4 cos q cy = 2 + 3 sin q.
4. If (5, 12) and (24, 7) are the foci of an ellipse passing 11. If PSQ is a focal chord of an ellipse 16x2 + 25y2 = 400
through the origin, find the eccentricity of the ellipse. such that SP = 8, find the length of SQ.

CG_05.indd 10 2/6/2017 4:04:54 PM


Ellipse 5.11

28. If p be the length of perpendicular from the focus S of


x2 y 2
12. Find the area bounded by the curve + £ 1 and x2 y 2
x y 16 9 the ellipse + = 1 on the tangent at P, prove that
the line + ≥ 1 . a 2 b2
4 3
b 2 2a
= -1.
p 2 SP
POSITION OF A POINT W.R.T. AN ELLIPSE
29. If p be the perpendicular from the centre of an
13. Find the location of the point (2, 3) with respect to the ellipse upon the tangent at any point P on it and
x2 y 2 r be the distance of P from the centre, prove that
ellipse + = 1.
4 3 a 2b 2
14. If (l, –l) be an interior point of an ellipse 4x2 + 5y2 = 1 = a 2 + b2 - r 2 .
p2
such that the length of the interval, where l lies, is m,
30. Prove that the locus of the mid-points of the portion
where m ΠQ+, find the value of (3m Р2)2013 + 2013.
x2 y 2
of the tangents to the ellipse 2 + 2 = 1 intercepted
TANGENT AND TANGENCY a b
a 2 b2
15. Find the number of tangents drawn from a point (2, 3) between the axes is 2 + 2 = 4 .
to an ellipse 4x2 + 3y2 = 12. x y
16. Find the equations of the tangents drawn from the point 31. Prove that the portion of the tangent to the ellipse inter-
cepted between the curve and the directrix subtends a
(2, 3) to the ellipse 9x2 + 16y2 = 144.
right angle at the corresponding focus.
17. If the line 3x + 4y = 5 touches the ellipse 9x2 + 16y2 = 32. Prove that in an ellipse, the perpendicular from a focus
144, find the points of contact. upon any tangent and the line joining the centre of the
18. For what value of l does the line y = x + l touches the ellipse to the point of contact meet on the correspond-
ellipse 9x2 + 16y2 = 144? ing directrix.
19. Find the equations of the tangents to the ellipse 33. Prove that the co-ordinates of those points on the
x2 y 2 x2 y 2
+ = 1 having slope 2. ellipse 2 + 2 = 1 tangents at which make equal an-
3 4 a b
20. Prove that the locus of the feet of perpendicular drawn Ê a2 b2 ˆ
gles with the axes is Á ± ,±

.
2 2 2
from the centre upon any tangent to the given ellipse is Ë a +b a +b ¯
r2 = a2 cos2 q + b2 sin2 q.
34. Find the locus of the point of intersection of two per-
21. A circle of radius r is concentric with the ellipse
x2 y 2
x2 y 2 pendicular tangents to the ellipse + = 1.
+ = 1 . Prove that the common tangent is inclined 16 9
a 2 b2 2 2
-1 Ê r - b ˆ 35. Tangents are drawn from any point P on the parabola
to the major axis at an angle of tan Á 2 ˜ .
Ë a - r2 ¯ x2 y 2
(y – 2)2 = 4(x – 1) to the ellipse + = 1 , which are
22. Prove that the tangents at the extremities of latus rectum 4 1
mutually perpendicular to each other. Find the locus of
of an ellipse intersect on the corresponding directrix.
the point P.
23. Prove that the locus of the mid-points of the portion of
36. Find the equations of the pair of tangents to the ellipse
the tangents to the given ellipse intercepted between
2x2 + 3y2 = 1 from the point (1, 1).
the axes is 4r2 = a2 sin2 q + b2 cos2 q.
37. If the tangents are drawn from a point (1, 2) to the el-
x2 y 2 lipse 3x2 + 2y2 = 5, find the angle between the tangents.
24. A tangent to the ellipse 2 + 2 = 1 meets the ellipse
a b
x2 y 2
+ = a + b in the points P and Q, prove that the NORMAL AND NORMALCY
a b
tangents at P and Q are at right angles. 38. Find the equation of the normal to the ellipse 4x2 + 9y2
Ê 4ˆ
25. Prove that the product of the perpendiculars drawn = 20 at Á1, ˜ .
from the foci upon any tangent to an ellipse is constant, Ë 3¯
i.e. b2. 39. Find the equation of the normal to the ellipse 5x2 + 3y2
26. If an ellipse slides between two perpendicular straight = 137 at the point whose ordinate is 2.
lines, prove that the locus of its centre is a circle. 40. Find the equation of the normal to the ellipse
27. Prove that the locus of the feet of the perpendiculars x2 y 2
from foci upon any tangent to an ellipse is an auxiliary + = 1 at the negative end of the latus rectum.
9 4
circle.

CG_05.indd 11 2/6/2017 4:04:55 PM


5.12 Coordinate Geometry Booster

41. If the normal at the point P(q) to the ellipse 5x2 + 14y2 CHORD OF CONTACT/CHORD BISECTED AT A POINT
= 70 intersects it again at the point Q(2q), prove that 51. Prove that the locus of the point, the chord of contact
3 cos q + 2 = 0.
x2 y 2
42. The normal at an end of a latus rectum of an ellipse of tangents from which to the ellipse 2 + 2 = 1
a b
x2 y 2 subtends a right angle at the centre of the ellipse is
+ = 1 passes through one extremity of the minor
a 2 b2 x2 y 2 1 1
axis, then prove that e4 + e2 – 1 = 0. 4
+ 4 = 2 + 2.
a b a b
43. Prove that the tangent and the normal at any point of
an ellipse bisect the the external and internal angles be- 52. Prove that the locus of the point, from which the chord
tween the focal distances of the point. of contact of tangents are to be drawn to the ellipse
x2 y 2 x2 y 2 1
44. The normal at any point P on the ellipse 2 + 2 = 1 touches the circle x2 + y2 = c2 is 4 + 4 = 2 .
a b a b c
meets the major and minor axes at G and G¢, respec- 53. The perpendicular tangents are drawn to the ellipse
tively and CF is perpendicular upon the normal from x2 y 2
the centre C of the ellipse, show that PF ◊ PG = b2 and + = 1 . Prove that the locus of the mid-point of
a 2 b2 2
PF × PG¢ = a2. Ê x2 y 2 ˆ Ê x2 + y 2 ˆ
x2 y 2 the chord of contact is Á 2 + 2 ˜ = Á 2 ˜.
45. The normal at a point P(q) on the ellipse 2 + 2 = 1 Ëa b ¯ Ë a + b2 ¯
a b
meets the axes of x and y at M and N, respectively, 54. Tangents are drawn from any point on the circle x2 + y2
show that x2 + y2 = (a + b)2. x2 y 2
= c2 to the ellipse 2 + 2 = 1 . Prove that the locus of
x2 y 2 a b
46. An ordinate PN of an ellipse 2 + 2 = 1 meets the the mid-points of the chord of contact is
a b
2
auxiliary circle in Q. Show that the locus of the point Ê x2 y 2 ˆ Ê x2 + y 2 ˆ
of intersection of the normals at P and Q is the circle x2 ÁË 2 + 2 ˜¯ = ÁË ˜
c b c2 ¯
+ y2 = (a + b)2.
55. Tangents PA and PB are drawn from a point P to the
47. Prove that the tangent of the angle between CP and the
x2 y 2
Ê a 2 - b2 ˆ ellipse 2 + 2 = 1 . The area of the triangle formed
normal at P(q) is Á ¥ sin 2q and its greatest a b
Ë 2ab ˜¯ by the chord of contact AB and axes of co-ordinates are
Ê a 2 - b2 ˆ constant. Prove that the locus of P is a hyperbola.
value is Á
Ë 2ab ˜¯
.
56. Prove that the locus of the mid-points of the normal
48. Prove that in an ellipse, the distance between the centre x2 y 2
chords of the ellipse 2 + 2 = 1 is
and any normal does not exceed the difference between a b
6 6 2 2 2
the semi-axes of the curve. Êa b ˆÊx y ˆ 2 2 2
49. The tangent and the normal at any point P of an el- Á 2 + 2 ˜ ÁË 2 + 2 ˜¯ = (a - b )
Ëx y ¯ a b
x2 y 2
lipse + = 1 cut its major axis in points Q and R, 57. Prove that the locus of the mid-points of the chords
a 2 b2
x2 y 2
respectively. If QR = a, show that the eccentric angle of of the ellipse 2 + 2 = 1 which touch the auxiliary
the point P is satisfying the equation a b
circle x2 + y2 = a2 is
e2 cos2 j + cos j – 1 = 0 2
50. If the normals at P(x1, y1), Q(x2, y2) and R(x3, y3) on 2Ê x2 y 2 ˆ Ê x2 y 2 ˆ
a Á 4 + 4˜ =Á 2 + 2˜
x2 y 2 Ëa b ¯ Ëa b ¯
the ellipse + = 1 are concurrent, show that
a 2 b2 58. Prove that the locus of the mid-points of the chords of
x1 y1 x1 y1 x2 y 2
the ellipse 2 + 2 = 1 which subtends a right angle at
x2 y2 x2 y2 = 0 a b
the centre of the ellipse is
x3 y3 x3 y3 2
Ê x2 y 2 ˆ Ê x2 y 2 ˆ Ê 1 1ˆ
and if points P(a), Q(b) and R(g), prove that ÁË 4 + 4 ˜¯ = ÁË 2 + 2 ˜¯ ÁË 2 + 2 ˜¯
a b a b a b
sec a cosec a 1 59 The eccentric angles of two points P and Q on the
sec b cosec b 1 = 0 ellipse differ by p/2. Prove that the locus of the mid-
sec g cosec g 1 x2 y 2
point of PQ is 2 + 2 = 2 .
a b

CG_05.indd 12 2/6/2017 4:04:56 PM


Ellipse 5.13

60. Prove that the locus of the mid-point of the chord of 72. Find the co-ordinates of the four extremities of two
contact of the perpendicular tangents to the ellipse x2 y 2
2 2 2 2ˆ2 2 2ˆ
conjugate diameters of an ellipse + = 1.
x y Êx y Êx + y a 2 b2
2
+ 2 = 1 is ÁË 2 + 2 ˜¯ = Á 2 ˜.
a b a b Ë a + b2 ¯ 73. Prove that the sum of the squares of any two conjugate
61. Prove that the locus of the mid-points of the focal semi-diameters of an ellipse is constant and equal to
Ê x2 y 2 ˆ the sum of the squares of the semi-axes of the ellipse,
chords of the ellipse Á 2 + 2 ˜ = 1 is i.e. CP2 + CD2 = a2 + b2.
Ëa b ¯
74. Prove that the product of the focal distances of a point
b2x2 + a2y2 = ab2xe
on an ellipse is equal to the square of the semi-diam-
62. Prove that the locus of the mid-points of the chords of eter, which is conjugate to the diameter through the
Ê x2 y 2 ˆ point. i.e. SP ◊ S¢P = CD2 .
the ellipse Á 2 + 2 ˜ = 1 which are tangents to the
Ëa b ¯ 75. Prove that the locus of the poles of the line joining the
x2 y 2 eccentricities of two conjugate diameters is the ellipse
ellipse + = 1 is x2 y 2
a2 b 2 + = 2.
2 a 2 b2
Ê x2 y 2 ˆ Ê x 2a 2 y 2 b 2 ˆ
ÁË 2 + 2 ˜¯ = ÁË 4 + 4 ˜¯ 76. If P and D be the ends of the conjugate diameters of an
a b a b
ellipse, find the locus of the mid-point of PD.
POLE AND POLAR x2 y 2
77. For the ellipse + = 1 , find the equation of the
63. Prove that the polar of the focus of an ellipse is the a 2 b2
directrix. diameter conjugate to ax – by = 0.
64. Find the pole of a given line lx + my + n = 0 with re- x2 y 2
78. If the point of intersection of the ellipses + =1
x2 y 2 2 2 a 2 b2
spect to an ellipse 2 + 2 = 1 . x y
a b and 2 + 2 = 1 be at the extremities of the conjugate
c d
65. Prove that the pole of a given line is the same as the a 2 b2
point of intersection of tangents at its extremities. diameters of the former, prove that 2 + 2 = 1 .
c d
66. Find the pole of the straight line x + 4y = 4 with respect
79. If CP and CD are the conjugate diameters of the ellipse
to the ellipse x2 + 4y2 = 4.
x2 y 2
67. Find the locus of the poles of the tangents to + = 1 , prove that the locus of the orthocentre of
x2 y 2 a 2 b2
+ = 1 with respect to the concentric ellipse c2x2 the DPCD is 2(b2y2 + a2x2)2 = (a2 – b2)2 (b2y2 – a2x2)2 .
a 2 b2
+ d2y2 = 1. 80. Prove that the tangents at the extremities of a pair of
68. The perpendicular from the centre of an ellipse conjugate diameters form a parallelogram whose area
is constant and equal to the product of the axes.
x2 y 2
+ = 1 on the polar of a point with respect to the 81. Show that the tangents at the ends of conjugate diam-
a 2 b2 x2 y 2
ellipse is constant and equal to c. Prove that the locus eters of the ellipse 2 + 2 = 1 intersect on the ellipse
a b
x2 y 2 1 x2 y 2
of the point is the ellipse 4 + 4 = 2 . + = 2.
a b c a 2 b2
69. Show that the equation of the locus of the poles of nor- 82. Find the eccentricity of the ellipse if y = x and 2x + 3y =
x2 y 2 0 are the equations of a pair of its conjugate diameters.
mal chords of the ellipse 2 + 2 = 1 is
a b
2 2 2 2 2 6 2 6 2 REFLECTION PROPERTY OF AN ELLIPSE
(a – b ) x y = a y + b x
70. If the polar with respect to y2 = 4x touches the ellipse 83. A ray is emanating from the point (–3, 0) is incident on
x2 y 2 the ellipse 16x2 + 25y2 = 400 at the point P with ordi-
+ = 1 , find the locus of its pole. nate 4. Find the equation of the reflected ray after first
a2 b2
reflection.
84. A ray is coming along the line x – y + 2 = 0 on the
CONJUGATE DIAMETERS ellipse 3x2 + 4y2 = 12. After striking the elliptic mirror,
71. Prove that the equation of the diameter of the ellipse it is then reflected. Find the equation of the line con-
x2 y 2 b2 x taining the reflected ray.
+ = 1 is y = - .
a 2 b2 a2m

CG_05.indd 13 2/6/2017 4:04:56 PM


5.14 Coordinate Geometry Booster

LEVEL II 12. The equation of common tangent between the ellipses


x2 y 2 x2 y 2
(Mixed Problems) + = 1 and + = 1 is
9 4 2 1
1. Tangents are drawn from a point on the circle x2 + y2 (a) x = 3 (b) y = 2
x2 y 2 (c) x = 1 (d) not defined
= 50 to the ellipse + = 1 , the tangents are at the
30 20 x2 y 2
angle is 13. If the equation of the tangent to the ellipse 2 + 2 = 1
p p p p x y a b
(a) (b) (c) (d) is + = 2 , its eccentric angle is
4 3 2 8 a b
2. The centre of the ellipse 4x2 + 9y2 – 8x – 36y + 4 = 0 is (a) 45° (b) 60° (c) 90° (d) 22.5°
(a) (2, 4) (b) (3, 2) (c) (1, 2) (d) (0, 1) 14. The equations of the tangents to the ellipse 3x2 + y2 = 3
3. The co-ordinates of the foci of the ellipse 4x2 + 9y2 – 8x making equal intercepts on the axes are
– 36y + 4 = 0 is (a) y = ±x ±2 (b) y = ±x ±4
(a) (1 ± 5, 2) (b) (2 ± 5, 2) (c) y = ±x ±5 (d) y = ±x ±7
(c) (± 5, 2) (d) (1 ± 5, 3) 15. The number of real tangents can be drawn from (3, 5)
to the ellipse 3x2 + 5y2 = 15 is
x2 y2
4. The equation + = 1 represents an ellipse (a) 4 (b) 2 (c) 1 (d) 0
if 10 - a 4 - a 16. The number of normals that can be drawn from a point
(a) a < 4 (b) a > 4 to a given ellipse is
(c) 4 < a < 10 (d) a > 10 (a) 4 (b) 2 (c) 1 (d) 0
x2 y 2 17. The set of possible values of m for which a line with the
5. Lep P be a variable point on the ellipse + =1 x2 y 2
25 16 slope m is a common tangent to the ellipse 2 + 2 = 1
with foci at F and F¢. If A be the area of the DPFF¢, the and the parabola y2 = 4ax is b c
maximum value of A is
(a) 12 s.u. (b) 24 s.u. (a) (3, 5) (b) (2, 3) (c) (1, 3) (d) (0, 1)
(c) 36 s.u. (d) 48 s.u. 18. The angle between the normals of the ellipse 4x2 + y2 =
6. The eccentricity of the ellipse (10x – 5)2 + (10y – 5)2 = 5, at the intersection of 2x + y = 3 and ellipse is
(3x + 4y – 1)2 is (a) tan–1(3/5) (b) tan–1(3/4)
–1
1 1 1 1 (c) tan (4/3) (d) tan–1(4/5)
(a) (b) (c) (d) 19. If the latus rectum of an ellipse x2 tan2 j + y2 sec2 j =
2 3 2 4
1 is 1/2, then f is
7. If the line y = x + l touches the ellipse 9x2 + 16y2 = 144, (a) p/2 (b) p/6 (c) p/3 (d) 5p/12
the value of l is x2 y 2
(a) ±5 (b) ±4 (c) ±7 (d) ±3 20. If pair of tangents are drawn to the ellipse + =1
16 9
8. The equation of the tangents to the ellipse 3x2 + 4y2 = from a point P, so that the tangents are at right angles to
12 which are perpendicular to the line y + 2x = 4 is/are each other, the possible co-ordinates of the point P are
(a) x – 2y + 4 = 0 (b) x – 2y + 7 = 0 (a) (3 2, 7) (b) (5, 0)
(c) x – 2y – 4 = 0 (d) x – 2y – 7 = 0 (c) (3, 4) (d) (2 5, 5)
9. The product of the perpendiculars from the foci of any 21. The eccentricity of the ellipse whose pair of a conju-
tangent to an ellipse b2x2 + a2y2 = a2b2 is gate diameters are y = x and 3y = –2x is
(a) a2 (b) b2 (c) 2b2 (d) 2a2 (a) 1/ 3 (b) 2/3 (c) 1/3 (d) 1/5
x2 y 2 22. The minimum length of intercept of any tangent of an
10. The number of tangents to the ellipse + =1
16 9 x2 y 2
from the point (4, 3) is ellipse 2 + 2 = 1 between the axes is
(a) 0 (b) 1 (c) 2 (d) 3 a b
11. If the normal at an end of a latus rectum of an ellipse (a) 2a (b) 2b (c) a + b (d) a – b
x2 y 2 x2 y 2
+ = 1 passes through one extremity of the minor 23. The eccentric angle of a point on the ellipse + =1
a 2 b2 whose distance from the centre is 2, is 6 2
axis, then e2 is (a) p/4 (b) p/6 (c) p/2 (d) p/6
3 -1 3 +1 2 2
x y
(a) (b) 24. If 2 + 2 = 1 represents an ellipse,
2 2 l - l - 6 l - 6l + 5
5 +1 5 -1 then l lies in
(c) (d)
2 2

CG_05.indd 14 2/6/2017 4:04:57 PM


Ellipse 5.15

(a) (– , –2) (b) (1, ) 34. An ellipse has OB as semi-minor axis, F, F¢ are its foci
(c) (3, ) (d) (5, ) and the angle FBF¢ is a right angle. The eccentricity of
25. The eccentricity of the ellipse ax2 + by2 + 2gx + 2fy + c the ellipse is
= 0, if its axis is parallel to x-axis, is (a) 1/2 (b) 1/ 2 (c) 1/3 (d) 1/4
a+b a-b 35. The sum of the focal distances from any point on the
(a) (b)
4 2 ellipse 9x2 + 16y2 = 144 is
b a (a) 32 (b) 18 (c) 16 (d) 8
(c) -1 (d) 1 - 36. If P = (x, y), F1 = (3, 0), F2 = (–3, 0) and 16x2 + 25y2 =
a b
400, then PF1 + PF2 is equal to
26. S and T are the foci of an ellipse and B is an end of the
(a) 8 (b) 6 (c) 10 (d) 12
minor axis. If STB is an equilateral triangle, the eccen-
37. The number of values of c such that the straight line
tricity of the ellipse is
x2
(a) 1/4 (b) 1/3 (c) 1/2 (d) 2/3 y = 4x + c touches the ellipse + y 2 = 1 is
27. The centre of the ellipse 4
( x + y - 1) 2 ( x - y ) 2 (a) 0 (b) 1 (c) 2 (d) infinite
+ = 1 is
9 16 38. The area of the quadrilateral formed by the tangents at
(a) (0, 0) (b) (1, 1) (c) (2, 1) (d) (1, 2) x2 y 2
the points of latus recta to the ellipse + = 1 is
x2 y 2 9 5
28. Let P be a variable point on the ellipse 2 + 2 = 1 (a) 27/4 s.u. (b) 9 s.u.
a b
with foci F1 and F2. If A be the area of the DPF1F2, the (c) 27/2 s.u. (d) 27 s.u.
maximum value of A is x2
39. Tangents are drawn to the ellipse + y 2 = 1 at
ea ab e 27
(a) (b) (c) aeb (d)
b e ab (3 3 cos q , sin q ) , where 0 < q < p/2. The value of
29. If the angle between the lines joining the foci of an q for which the sum of intercepts on the axes made by
ellipse to an extremity of a minor axis is 90°, the this tangent is minimum is
eccentricity of the ellipse is (a) p/3 (b) p/4 (c) p/8 (d) p/6
(a) 1/8 (b) 1/4 (c) 1/ 2 (d) 1/ 3 40. If tangents are drawn to the ellipse x + 2y2 = 2, the
2

locus of the mid-point of the intercept made by the tan-


30. On the ellipse 4x2 + 9y2 = 1, the points at which the
gents between the co-ordinate axes is
tangents are parallel to the line 8x = 9y are
1 1 1 1
Ê 2 1ˆ Ê 2 1ˆ (a) 2
+ 2 =1 (b) 2
+ 2 =1
(a) Á , ˜ (b) Á - , ˜ 2x 4y 4x 2y
Ë 5 5¯ Ë 5 5¯
x2 y 2 x2 y 2
Ê 2 1ˆ Ê 2 1ˆ (c) + =1 (d) + =1
(c) Á - , - ˜ (d) Á , - ˜ 2 4 4 2
Ë 5 5¯ Ë 5 5¯
41. The minimum area of the triangle formed by the tan-
31. The equation of the largest circle with centre (1, 0) that
can be inscribed in the ellipse x2 + 4y2 = 36 is x2 y 2
gent to the ellipse 2 + 2 = 1 and the co-ordinate
(a) 3x2 + 3y2 – 6x – 8 = 0 axes is a b
(b) 3x2 + 3y2 + 6x – 8 = 0
(c) 2x2 + 2y2 + 5x – 8 = 0 a 2 + b2
(a) ab s.u. (b) s.u.
(d) 2x2 + 2y2 – 5x – 8 = 0 2
x2 y 2 ( a + b) 2 a 2 + ab + b 2
32. Let E be the ellipse 2 + 2 = 1 and C be the circle (c) s.u. (d) s.u.
a b 2 2
x2 + y2 = 9. Let P and Q be the points (1, 2) and (2, 1)
respectively. Then 42. The point on the curve x2 + 2y2 = 6 whose distance
(a) Q lies inside C but outside E from the line x + y = 7 is minimum, is
(b) Q lies outside both C and E (a) (1, 2) (b) (1, 3) (c) (2, 1) (d) (3, 1)
(c) P lies inside both C and E 43. The equation of the common tangent in 1st quadrant to
(d) P lies inside C but outside E x2 y 2
the circle x2 + y2 = 16 and the ellipse + = 1 is
33. The radius of the circle passing through the foci of the 25 4
x2 y 2
ellipse + = 1 and having its centre at (0, 3) is (a) 2x + y 3 = 4 7 (b) 2x - y 3 = 4 7
16 9
(a) 4 (b) 3 (c) 5 (d) 7 (c) 3x - y 3 = 4 7 (d) 3x + y 3 = 4 7

CG_05.indd 15 2/6/2017 4:04:57 PM


5.16 Coordinate Geometry Booster

44. Let P(x1, y1) and Q(x2, y2), y1 < 0, be the end-points of
x2 y 2
the latus rectum of the ellipse x2 + 4y2 = 4. The equa- 50. The number of points on the ellipse + = 1 from
50 20
tion of the parabolas with latus rectum PQ are
which pair of perpendicular tangents are drawn to the
2
(a) x 2 + 2 3 y = 3 + 3 (b) x - 2 3 y = 3 + 3 x2 y 2
2 ellipse + = 1 is
(c) x 2 + 2 3 y = 3 - 3 (d) x - 2 3 y = 3 - 3 16 9
45. The line passing through the extremity A of the major (a) 0 (b) 2 (c) 1 (d) 4
axis and extremity B of the minor axis of the ellipse
x2 + 9y2 = 9 meets the auxiliary circle at the point M. y2
51. The eccentricity of the ellipse ( x - 3) 2 + ( y - 4) 2 =
The area of the triangle with vertices at A, M and the is 9
origin is
3 1 1 1
31 29 21 27 (a) (b) (c) (d)
(a) (b) (c) (d) 2 3 3 2 3
10 10 10 10 2 2
x y
46. The normal at a point P on the ellipse x2 + 4y2 = 16 52. For an ellipse + = 1 with vertices A and A¢,
9 4
meets the x-axis at Q. If M is the mid-point of the seg- tangents are drawn at the point P in the first quadrant
ment PQ, the locus of M intersects the latus rectum of meets the y-axis in Q and the chord A¢P meets the
the given ellipse at the points y-axis in M. If O be the origin, then OQ2 – MQ2 is
Ê 3 5 2ˆ Ê 3 5 19 ˆ (a) 9 (b) 13 (c) 4 (d) 5
(a) Á ± ,± ˜ (b) ÁË ± ,± ˜
Ë 2 7¯ 2 4 ¯ x2 y 2
53. The line lx + my + n = 0 will cut the ellipse 2 + 2 = 1
Ê 1ˆ Ê 4 3ˆ a b
(c) Á ± 2 3, ± ˜ (d) ÁË ± 2 3, ± ˜ in points whose eccentric angles differ by p/2, if
Ë 7¯ 7 ¯
(a) a2l2 + b2n2 = 2m2 (b) a2m2 + b2l2 = 2n2
47. Tangents are drawn from the point P(3, 4) to the ellipse
(c) a2l2 + b2m2 = 2n2 (d) a2n2 + b2m2 = 2l2.
x2 y 2
+ = 1 touching the ellipse at A and B. Then the 54. A circle has the same centre as an ellipse and passes
9 4
through the foci F1 and F2 of the ellipse such that two
co-ordinates of A and B are
curves intersect in 4 points. Let P be any of their points
(a) (3, 0) and (0, 2)
of intersection. If the major axis of the ellipse is 17 and
Ê 8 2 161 ˆ Ê 9 8ˆ the area of the triangle F1F2 is 30, the distance between
(b) ÁË - , ˜ and Á - , ˜
5 15 ¯ Ë 5 5¯ the foci is
(a) 11 (b) 12 (c) 13 (d) None
Ê 8 2 161 ˆ
(c) Á - , and (0, 2)
Ë 5 15 ˜¯
55. The point O is the centre of the ellipse with major axis
AB and minor axis CD and the point F is one focus of
Ê 9 8ˆ the ellipse. If OF = 6 and the diameter of the inscribed
(d) (3, 0) and Á - , ˜
Ë 5 5¯ circle of DOCF is 2, the product of (AB)(CD) is
48. The maximum area of an isosceles triangle inscribed in (a) 65 (b) 52 (c) 78 (d) none
x2 y 2 x2 y 2
an ellipse + = 1 with its vertex at one end of the 56. A tangent having slope –4/3 to the ellipse + =1
a 2 b2 18 32
major axis is intersects the major and minor axes in points A and B,
respectively. If C is the centre of the ellipse, the area of
3 3 3 2 the triangle ABC is
(a) ab (b) ab
4 4 (a) 12 s.u. (b) 24 s.u.
(c) 36 s.u. (d) 48 s.u.
2 3 3 3
(c) ab (d) ab x2 y2
5 5 57. The common tangent to the ellipse 2 + =1
a + b2 b2
49. The point (a, b) on the ellipse 4x2 + 3y2 = 12, in the x2 y2
and 2 + 2 = 1 is
first quadrant, so that the area enclosed by the lines a a + b2
y = x and y = b, x = a and the x-axis is maximum, is
(a) ay = bx + a 4 - a 2b 2 + b 4
Ê3 ˆ Ê 3ˆ Ê 3ˆ Ê 3ˆ
(a) Á , 1˜ (b) Á1, ˜ (c) Á 2, ˜ (d) Á 3, ˜ (b) by = ax - a 4 + a 2b 2 + b 4
Ë2 ¯ Ë 2¯ Ë 2¯ Ë 2¯

CG_05.indd 16 2/6/2017 4:04:58 PM


Ellipse 5.17

(c) ay = bx - a 4 + a 2b 2 + b 4 Ê 12 ˆ
(c) ( , 7) ÁË 7, ˜¯
5
(d) by = ax + a 4 - a 2b 2 + b 4 (d) no such value of a exists
58. The normal at a variable point P on an ellipse x2 y 2
66. If the ellipse 2 + 2 = 1 is inscribed in a rectangle
x2 y 2 a b
+ = 1 of eccentricity e meets the axes of the el-
a 2 b2 whose length : breadth is 2 : 1, the area of the rectangle
lipse in Q and R, the locus of the mid-point of QR is a is
conic with an eccentricity e¢ such that 4 2 4 2
(a) (a + b 2 ) (b) (a + b 2 )
(a) e¢ is independent of e 7 3
(b) e¢ =1 (c) e¢ = e (d) e¢ = 1/e 12 2 8
(c) (a + b 2 ) (d) (a 2 + b 2 )
59. An ellipse is drawn with major and minor axes of 5 5
lengths 10 and 8, respectively. Using one focus as the 67. The length of the side of the square which can be
centre, a circle is drawn that is the tangent to the el- made by four perpendicular tangents to the ellipse
lipse, and no part of the circle being outside of the el- x2 y 2
lipse. The radius of the circle is + = 1 is
7 11
(a) 3 (b) 2 (c) 2 2 (d) 5
(a) 10 (b) 8 (c) 6 (d) 5
60. A common tangent to 9x2 + 16y2 = 144; y2 = x – 4 and 2 2
x y
x2 = y2 – 12x + 32 = 0 is 68. If the tangent to the ellipse 2 + 2 = 1 makes angles
(a) y = 3 (b) x + 4 = 0 a b
a and b with the major axis such that tan a + tan b = l,
(c) x = 4 (d) y + 3 = 0
the locus of their points of intersection is
61. The area of the rectangle formed by the perpendiculars
from the centre of the standard ellipse to the tangent (a) x2 + y2 = a2 (b) x2 + y2 = b2
and the normal at its point whose eccentric angle is p/4 (c) x2 = 2llx = a2 (d) l(x2 – a2) = 2xy.
is 69. If a – b = c, the locus of the points of intersection of
Ê a 2 - b2 ˆ Ê a 2 - b2 ˆ 1 tangents at
(a) Á 2 ˜ ab (b) Á 2 ˜ P(a cos a, b sin a) and Q(a cos b, b sin b)
Ë a + b2 ¯ Ë a + b 2 ¯ ab
x2 y 2
Ê a 2 + b2 ˆ 1 Ê a 2 + b2 ˆ to the ellipse + = 1 is
(c) Á 2 ˜ (d) Á 2 ˜ ab a 2 b2
Ë a - b 2 ¯ ab Ë a - b2 ¯ (a) a circle (b) a straight line
62. An ellipse having foci at (3, 3) and (–4, 4) and passing (c) an ellipse (d) a parabola
through the origin has eccentricity equal to x2 y2
(a) 3/7 (b) 2/7 (c) 5/7 (d) 3/5 70. If the eccentricity of the ellipse 2 + 2 = 1 is
a +1 a + 2
63. A bar of length 20 units moves with its ends on two 1
fixed straight lines at right angles. A point P marked on , the latus rectum of the ellipse is
6
the bar at a distance of 8 units from one and describes a 5 10 8 7
conic whose eccentricity is (a) (b) (c) (d)
6 6 6 6
2 4
(a) 5/9 (b) (c) (d) 3/5
3 9
64. If maximum distance of any point on the ellipse
x2 + 2y2 + 2xy = 1 from its centre be r, the value of r is
LEVEL III
(Problems for JEE Advanced)
(a) 3 + 3 (b) 2 + 2
x2 y 2
2 1. Let P be a variable point on the ellipse + =1
(c) (d) 2- 2 a 2 b2
3- 5 with foci F1 and F2. If A be the area of DPF1F2, find the
x2 y2
65. If the ellipse + = 1 is inscribed in a maximum value of A.
a 2 - 7 13 - 5a
2. Let d be the perpendicular distance from the centre of
square of side length a 2 , then a is
x2 y 2
6 the ellipse + = 1 to the tangent drawn at a point
(a) a 2 b2
5
Ê 13 ˆ P on the ellipse. If F1 and F2 be the two foci of the
(b) ( , 7) Á 7, ˜
Ë 5¯ ellipse, prove that

CG_05.indd 17 2/6/2017 4:04:59 PM


5.18 Coordinate Geometry Booster

16. Find the equation of the common tangent in first


Ê b2 ˆ
( PF1 - PF2 ) 2 = 4a 2 Á1 - 2 ˜ quadrant to the circle x2 + y2 = 16 and the ellipse
Ë d ¯
x2 y 2
3. Find the radius of the circle passing through the foci of + = 1. Also find the length of the intercept of
25 4
x2 y 2 the tangent between the co-ordinate axes.
the ellipse + = 1 and having its centre at (0, 3).
16 9 x2 y 2
4. If a tangent drawn at a point (t2, 2t) on the parabola 17. P and Q are two points on the ellipse + = 1 such
25 9
y2 = 4x is the same as the normal drawn at a point
that sum of their ordinates is 3. Find the locus of the
( 5 cos j , 2 sin j ) on the ellipse 4x2 + 5y2 = 20. Find
points of intersection of tangents at P and Q.
the values of t and j. 18. From any point P lying in the first quadrant on the
5. An ellipse has OB as a minor axis. F and F¢ are its foci
x2 y 2
and the angle FBF¢ is a right angle. Find the eccentric- ellipse + = 1 , PN is drawn perpendicular to the
25 16
ity of the ellipse.
major axis and produced at Q so that NQ equals to PS,
6. A tangent to the ellipse x2 + 4y2 = 4 meets the ellipse
where S is a focus. Prove that the locus of Q is 3x + 5y
x2 + 2y2 = 6 at P and Q. Prove that the tangents at P and + 25 = 0.
Q of the ellipse x2 + 2y2 = 6 are at right angles. x2 y 2
7. Find the co-ordinates of the points P on the ellipse 19. If a tangent of slope 2 of the ellipse 2 + 2 = 1 is the
a b
x2 y 2 normal to the circle x2 + y2 + 4x + 1 = 0, find the maxi-
+ = 1 , for which the area of the DPON is maxi-
a 2 b2 mum value of ab.
mum, where O denotes the origin and N, the foot of the 20. If the maximum distance of any point on the ellipse
perpendicular from O to the tangent at P. x2 + 2xy + 2y2 = 1 from its centre be r, find r.
8. Find the angle between the pair of tangents drawn to 21. Prove that the locus of the mid-points of the chords of
the ellipse 3x2 + 2y2 = 5 from the point (1, 2). x2 y 2
the ellipse 2 + 2 = 1 which are of constant length 2c
x2 y 2 a b
9. If the normal at P(q) to the ellipse + = 1 inter-
14 5 2 2 2 2
Êb x + a y ˆ 1 Ê x2 y 2 ˆ
2 is Á 4 2 ˜ = Á 1 - - ˜.
sects it again at Q(2q), prove that cos q = — . Ë a y + b4 x2 ¯ c2 Ë a 2 b2 ¯
3
Ê 1 2 ˆ 22. Chords of the ellipse touch the parabola ay2 = –2b2x.
10. If Á , ˜ be the mid-point of the chord of the ellipse Prove that the locus of their poles is the parabola
Ë 5 5¯
ay2 = 2b2x.
x2 y 2
+ = 1 , find its length. 23. An ellipse is rotated through a right angle in its own
25 16 plane about its centre, which is fixed. Find the locus of
11. Prove that, in an ellipse, the perpendicular from a focus the point of intersection of a tangent to the ellipse in its
upon any tangent and the line joining the centre of the original position with the tangent at the same point of
ellipse to the point of contact meet on the correspond- the curve in its new position.
ing directrix.
24. The tangents drawn from a point P to the ellipse make
12. Find the area of the quadrilateral formed by the tan-
angles q1 and q2 with the major axis. Find the locus of
gents at the end-points of latus rectum to the ellipse
P for which q1 + q2 = 2a.
x2 y 2
+ = 1. 25. Find the length of the sides of a square which can be
9 5 made by four perpendicular tangents to the ellipse
x2
13. A tangent is drawn to the ellipse + y 2 = 1 at x2 y 2
p 27 + = 1.
(3 3 cos q , sin q ) , where 0 < q < . 16 9
2 26. Find the locus of the point which divides the double
Find the value of q such that the sum of the intercepts
x2 y 2
on axes made by the tangent is minimum. ordinates of the ellipse + = 1 in the ratio 1 : 2
14. If tangents are drawn to the ellipse x2 + 2y2 = 2, find internally. 9 4
the locus of the mid-point of the intercept made by the 27. An ellipse has the points (1, –1)and (2, –1) as its foci
tangents between the co-ordinate axes. and x + y = 5 as one of its tangent. Find the point where
15. Find the minimum area of the triangle formed by the the line be a tangent to the ellipse from the origin.
x2 y 2 28. An ellipse is sliding along the co-ordinate axes. If the
tangent to the ellipse 2 + 2 = 1 and the co-ordinate foci of the ellipse are (1, 1) and (3, 3), find the area of
a b
axes. the director circle of the ellipse.

CG_05.indd 18 2/6/2017 4:04:59 PM


Ellipse 5.19

29. What are the values of the parameter q for points where 9. If the eccentric angles of points P and Q on the ellipse
x2 y 2 p
the line bx = ay intersects the ellipse 2 + 2 = 1 ? be q and + q and a be the angle between the nor-
a b 2
30. Prove that the sum of the squares of the reciprocals of mals at P and Q, prove that the eccentricity e is given
two perpendicular radius vectors of an ellipse is con-
2 1 — e2
stant. by tan a = .
31. Find the eccentricity of the ellipse e 2 (sin 2 2q )
4(x – 2y + 1)2 + 9(2x + y + 2)2 = 25. 10. The tangent and the normal at any point P of an el-
32. If two vertices of a rectangle lie on y = 2x + c and other x2 y 2
two vertices are (0, 4) and (–1, 2). Find c and other two lipse 2 + 2 = 1 cut its major axis in points T and T¢
a b
vertices such that the area of the ellipse inscribed in the respectively such that TT¢ = a. Prove that the eccentric
5p angle of the point P is given by c2 cos2 j + cos j – 1
rectangle is .
2 = 0.
11. A variable point P on an ellipse of eccentricity e is
LEVEL IV joined to its foci S and S¢. Prove that the locus of the
in-centre of the DPSS¢ is an ellipse whose eccentricity
(Tougher Problems for JEE
Advanced) 2e
is .
1+ e
1. Find the maximum area of an isosceles triangle in- 12. The eccentric angle of any point P measured from the
x2 y 2 semi-major axis CA is f. If S be the focus nearest to A
scribed in the ellipse 2 + 2 = 1 with its vertex at one
a b and –ASP = q, prove that
end of the major axis. [Roorkee, 1994]
Ê qˆ 1+ e Ê jˆ
2 tan Á ˜ = tan Á ˜
2. If a tangent drawn at a point (t , 2t) on the parabola Ë 2¯ 1- e Ë 2¯
y2 = 4x is the same as the normal drawn at a point
13. Prove that the locus of the centroid of an equilateral
( 5 cos j , 2 sin j ) on the ellipse 4x2 + 5y2 = 20. Find
the values of t = j. x2 y 2
inscribed in an ellipse 2 + 2 = 1 is
3. Find the equation of the largest circle with centre (1, 0) a b
that can be inscribed in the ellipse x2 + 4y2 = 16. (a 2 + 3b 2 ) 2 2 (3a 2 + b 2 ) 2 2
[Roorkee, 1999] x + 2 2 y =1
a 2 (a 2 - b 2 )2 a (a - b 2 )2
4. Find the condition so that the line px + qy = r inter-
14. The tangent at a point P(a cos j, b sin j) of an ellipse
x2 y 2
sects the ellipse 2 + 2 = 1 in points whose eccentric x2 y 2
a b + = 1 meets its auxiliary circle in two points,
p a 2 b2
angles differ by . [Roorkee, 2001] the chord joining which subtends a right angle at the
4
centre. Prove that the eccentricity of the ellipse is
5. A point moves so that the sum of the squares of its dis-
1
tances from two intersecting straight lines is constant. .
Prove that its locus is an ellipse. 1 + sin 2 q
6. Prove that the line joining the extremities of any pair of 15. Prove that the locus of the point of intersection of tan-
diameters of an ellipse which are at right angles, will gents to an ellipse at two points whose eccentric angles
touch a fixed circle. differ by a constant a is an ellipse
x2 y 2
7. If P be any point on the ellipse 2 + 2 = 1 whose or- x2 y 2 Êaˆ
a b + 2 = sec 2 Á ˜
a 2
b Ë 2¯
dinate is y¢, prove that the angle between the tangent at
2
16. If two concentric ellipses be such that the foci of
—1 Ê b ˆ one be on the other end and e and e¢ be their eccen-
P and the focal distance of P is tan Á
Ë aey ¢ ˜¯
.
tricities. Prove that the angle between their axes is
x2 y 2 Ê e2 + e ¢ 2 - 1 ˆ
8. Prove that the ellipse 2 + 2 = 1 and the circle x2 + y2
a b cos -1 Á ˜.
Ë ee ¢ ¯
Ê a - bˆ
= ab intersect at an angle tan -1 Á .
Ë ab ˜¯

CG_05.indd 19 2/6/2017 4:05:00 PM


5.20 Coordinate Geometry Booster

17. If the normals at the four points (x1, y1), (x2, y2), (x3y3) 8. If F1 and F2 be the feet of perpendiculars from the foci
x2 y 2 x2 y 2
and (x4, y4) on the ellipse 2 + 2 = 1 are concurrent, S1 and S2 of an ellipse + = 1 on the tangent at any
prove that a b 5 3
point P on the ellipse, find the value of (S1F1) ◊ (S1F2).
Ê1 1 1 1ˆ
( x1 + x2 + x3 + x4 ) Á + + + =1
Ë x1 x2 x3 x4 ˜¯ 9. Find the minimum length of the intercept of any tan-
x2 y 2
18. If q be the difference of the eccentric angles of two gent to the ellipse + = 1 between the co-ordi-
nate axes. 16 9
points on an ellipse, the tangent at which are at right
angles. Prove that ab sin q = d1d2, where d1, d2 are the 10. Find the number of distinct normals that can be drawn
semi-diameters parallel to the tangents at the points and x2 y2
a, b are the semi-axes of the ellipse. to the ellipse + = 1 from the point (0, 6).
169 25
x2 y 2
19. From any point on the conic + = 4 , tangents are
a 2 b2 Comprehensive Link Passage
x2 y 2
drawn to the conic + = 1.
a 2 b2 Passage I
Prove that the normals at the points of contact meet on x2 y 2
Let C: x2 + y2 = r2 and E: + = 1 intersect at four dis-
1 16 9
the conic a 2 x 2 + b 2 y 2 = (a 2 - b 2 ) 2 .
4 tinct points A, B, C, D. Their common tangents form a paral-
20. Show that the locus of the centre of the circle which lelogram EFGH.
x2 y 2 1. If ABCD is a square, then r is
cuts the ellipse 2 + 2 = 1 in a fixed point (h, k) and 12 2 12 12
a b (a) (b) (c) (d) none
two other points at the extremities of a diameter is 5 5 5 5
2(a2x2 + b2y2) = (a2 – b2)(hx – ky). 2. If EFGH is a square, then r is
(a) 20 (b) 12 (c) 15 (d) none
Integer Type Questions 3. If EFGH is a square, the ratio of
ar (Circle C )
1. Find the minimum area of the triangle formed by the is
ar (circumcircle D EFG )
x2 y 2
tangent to the ellipse + = 1. 9 3 1
9 4 (a) (b) (c) (d) none
x2 y 2 16 4 2
2. Let P be a variable point on the ellipse + =1
5 1 Passage II
with foci F1 and F2. If A be the area of the triangle A curve is represented by C:
DPF1F2, find the maximum value of A. 21x2 – 6xy + 29y2 + 6x – 58y – 151 = 0
Ê 16 ˆ 1. The eccentricity of the given curve is
3. If the tangent at ÁË 4 cos j , sin j ˜ to the
¯ 1 1 2 2
11 (a) (b) (c) (d)
2 2 3 3 3 5
ellipse 16x + 11y = 256 is also a tangent to the circle
2. The length of axes are
x2 + y2 – 2x – 15 = 0, find the number of values of f.
(a) 6, 2 6 (b) 5, 2 5 (c) 4, 4 5 (d) None
4. Find the area of a parallelogram formed by the tangents
at the extremities of a pair of conjugate diameters of an 3. The centre of the conic C is
(a) (1, 0) (b) (0, 0) (c) (0, 1) (d) None.
x2
ellipse + 16y 2 = 1 . Passage III
9 x2 y 2
Let F and F¢ be the foci of the ellipse 2 + 2 = 1 whose ec-
5. Find the number of integral values of a for which the a b
x2 y2 centricity is e, P is a variable point on the ellipse.
equation + = 1 represents an ellipse.
a - 10 4 - a 1. The locus of the incentre of the DPFF¢ is a/an
6. Find the area of the greatest rectangle that can be in- (a) ellipse (b) hyperbola
scribed in an ellipse x2 + 4y2 = 4. (c) parabola (d) circle
2. The eccentricity of the locus of P is
7. If P(x, y), F1 = ( 7, 0), F2 = ( - 7, 0) and 9x2 + 16x2
= 144, find the value of (PF1 + PF2 – 2). 2e 2e
(a) (b) (c) 1 (d) none
1- e 1+ e

CG_05.indd 20 2/6/2017 4:05:00 PM


Ellipse 5.21

3. The maximum area of the rectangle inscribed in the el- Passage VI


lipse is Tangents are drawn from the point P(3, 4) to the ellipse
2abe 2 2abe abe x2 y 2
(a) (b) (c) (d) none + = 1 touching the ellipse at points A and B.
1+ e 1+ e 1+ e 9 4
1. The co-ordinates of A and B are
Passage IV
(a) (3, 0) and (0, 2)
x2 y 2
The ellipse + = 1 is such that it has the least area but Ê 8 2 161 ˆ Ê 9 8ˆ
a 2 b2 (b) Á - , and Á - , ˜
Ë 5 15 ˜¯ Ë 5 5¯
contains the circle (x– 1)2 + y2 = 1.
Ê 8 2 161 ˆ
1. The eccentricity of the ellipse is (c) Á - , and (0, 2)
Ë 5 15 ˜¯
2 1 1
9 8
(d) (3, 0) and ÊÁ - , ˆ˜
(a) (b) (c) (d) none
3 3 2
Ë 5 5¯
2. The equation of the auxiliary circle of the ellipse is
2. The orthocentre of the DPAB is
13
(a) x 2 + y 2 = (b) x2 + y2 = 5 8 Ê 7 25 ˆ
2 (a) ÊÁ 5, ˆ˜ (b) Á , ˜
Ë 7¯ Ë5 8 ¯
9
(c) x 2 + y 2 = (d) none Ê 8 7ˆ
2 Ê 11 8 ˆ
(c) Á , ˜ (d) ÁË , ˜¯
3. The length of the latus rectum of the ellipse is Ë 5 5¯ 25 5
(a) 2 (b) 1 (c) 3 (d) 5/2 3. The equation of the locus of the point whose distances
Passage V from the point P and the line AB are equal is
The conic section is the locus of a point which moves in a (a) 9x2 + y2 – 6xy – 54x – 62y + 241 = 0
plane in such a way that, the ratio of its distance from a fixed (b) 9x2 + y2 + 6xy – 54x + 62y + 241 = 0
point to a fixed straight line is constant. The fixed point is (c) 9x2 + 9y2 – 6xy – 54x – 62y + 241 = 0
called focus and the fixed straight line is called the directrix. (d) x2 + y2 – 2xy – 27x – 31y + 120 = 0
The constant ratio is called the eccentricity. It is denoted by e.
If e is less than 1, the conic section is called an ellipse. A Matrix Match
line joining two points on the ellipse is called the chord of (For JEE-Advanced Examination Only)
the ellipse. If through a point C, any chord of an ellipse is 1. Match the following columns:
bisected, the point C is called the centre of the ellipse. Let the equation of the curve is x2 + 2y2 + 4x + 12y = 0.
Let the equation of the curve is
ax2 + 2hxy + by2 + 2gx + 2fy + c = 0 Column I Column II
(A) The centre is (P)
It will represents an ellipse if h2 < ab and D π 0, where ( - 2 - 11, -)
a h g (B) The focus is (Q) (–2, –3)
D= h b f (C) One extremity of the (R)
( 11 - 2, - 3)
g f c major axis is
(D) The latus rectum is (S)
Ê 11 ˆ
1. The length of the longest chord of the ellipse x2 + xy + ÁË - 2 + , - 2˜
2 ¯
y2 = 1 is
1 (T)
22
(a) 2 (b) (c) 2 2 (d) 1
2
2. The length of the chord passing through the centre 2. Match the following columns:
and perpendicular to the longest chord of the ellipse ( x - 3) 2 ( y - 2) 2
x2 + xy + y2 = 1 is Let the equation of the ellipse is + = 1.
25 16
1 3 2 1
(a) (b) (c) 2 (d) Column I Column II
2 2 3 3
(A) The centre is (P) (8, 2)
3. There are exactly n integral values of l for which the (B) One extremity of major axis is (Q) (3, 2)
equation x2 + lxy + y2 = 1 represents an ellipse, then n
(C) One of the foci is (R) (6, 2)
must be
(a) 0 (b) 1 (c) 2 (d) 3 (D) One extremity of minor axis is (S) (3, 6)
4. The centre of the ellipse x2 + xy + 2y2 = 1 is (E) The length of latus rectum is (T) 10
(a) (0, 0) (b) (1, 1) (c) (1, 2) (d) (2, 1) (F) The focal distance is (U) 6.4

CG_05.indd 21 2/6/2017 4:05:01 PM


5.22 Coordinate Geometry Booster

3. Match the following columns: (B) The length of PQ is (Q)


12 3
For all real p, the line 2px + 2 1 - p 2 = 1 touches a
fixed ellipse. (C) Area of DTPQ is (R)
9 3
Column I Column II (D) Area of quadrilateral (S) x + 2y = 2
(A) The eccentricity is (P) 1/2 OPTQ is (T) 2x + y = 1
(B) The latus rectum is (Q) 3/2
(C) The focal distance is (R) 2
Questions asked in Previous Years’
(D) One extremity of major axis (S) (0, 1) JEE-Advanced Examinations
is
(E) One of the foci is (T) (0, 3/2) 1. The locus of a point whose distance from (–2, 0) is 2/3
9
times its distance from the line x = - is a/an
4. Match the following columns: 2
A tangent having slope –4/3 touches the ellipse (a) ellipse (b) parabola
x2 y 2 (c) hyperbola (d) none [IIT-JEE, 1994]
+ = 1 at point P and intersects the major and x2 y 2
18 32 2. Let E be the ellipse + = 1 and C be the circle
minor axes at A and B respectively, where O is the cen- 9 4
tre of the ellipse. x2 + y2 = 9. Let P and Q be the points (1, 2) and (2, 1)
respectively, then
Column I Column II (a) Q lies inside C but outside E
(A) The distance between the parallel (P) 24 (b) Q lies outside both C and E
tangents having slopes –4/3 is (c) P lies inside both C and E
(B) Area of DAOB is (Q) 7/24 (d) P lies inside C but outside E [IIT-JEE, 1994]
x2 y 2
(C) If the tangent in first quadrant (R) 48/5 3. Let P be a variable point on the ellipse 2 + 2 = 1
touches the ellipse at (h, k), the a b
value of hk is with foci F1 and F2. If A is the area of DPF1F2, find the
maximum value of A. [IIT-JEE, 1994]
(D) If the equation of the tangent in- (S) 12
4. The radius of the circle passing through the foci of the
tersecting positive axes is lx + my
= 1, the value of l + m is x2 y 2
ellipse + = 1 , and having its centre at (0, 3) is
16 9
5. Match the following columns: 1 7
Let M(t, t + 1) is a point moving in a straight line and (a) 4 (b) 3 (c) (d)
2 2
C1, C2, C3 be three conics whose equations are x2 + y2 =
[IIT-JEE, 1995]
2, y2 = 8x and x2 + 2y2 = 1. Let M lies within the interior
of C1, C2, C3. 5. Let d be the perpendicular distance from the centre of
x2 y 2
Column I Column II the ellipse 2 + 2 = 1 to the tangent drawn at a point
a b
(A) C1 is (P) P on the ellipse.
(3 - 2 2, 3 + 2 2)
If F1 and F2 are two foci of the ellipse, show that
(B) C2 is (Q) Ê 1ˆ
ÁË -1, - ˜¯ Ê b2 ˆ
3 ( PF1 - PF2 ) 2 = 4a 2 Á1 - 2 ˜ . [IIT-JEE, 1995]
Ë d ¯
(C) C3 is (R)
Ê -1 - 3 -1 + 3 ˆ No questions asked in 1996.
ÁË , ˜
2 2 ¯ 6. A tangent to the ellipse x2 + 4y2 = 4 meets the ellipse
x2 + 2y2 = 6 at P and Q. Prove that the tangents at P and
6. Match the following columns: Q of the ellipse x2 + 2y2 = 6 are at right angles.
Tangents are drawn from (2, 3) to the ellipse [IIT-JEE, 1997]
x2 y 2 7. An ellipse has OB as a semi-minor axis. F and F¢ are
+ = 1.
4 3 its foci, and the ellipse FBF¢ is a right angle. Find the
eccentricity of the ellipse. [IIT-JEE, 1997]
Column I Column II
8. If P = (x, y) be any point on 16x2 + 25y2 = 400 with foci
(A) The equation of PQ is (P)
3 F1 = (3, 0) and F2 = (–3, 0), then PF + PF2 is
3¥ (a) 8 (b) 6 (c) 10 (d) 12
2
[IIT-JEE, 1998]

CG_05.indd 22 2/6/2017 4:05:01 PM


Ellipse 5.23

9. Find the co-ordinates of all points P on the ellipse 1 2 1 2


x2 y 2 (a) (a + b 2 ) s.u. (b) (a + b 2 + ab) s.u.
+ = 1 , for which the area of DPON is maxi- 2 3
a 2 b2 1
mum, where O denotes the origin and N, the foot of the (c) (a + b) 2 s.u. (d) ab s.u.
2 [IIT-JEE, 2005]
perpendicular from O to the tangent at P.
[IIT-JEE, 1999] No questions asked in between 2006-2007.
10. On the ellipse 4x2 + 9y2 = 1, the points at which the 17. Let P(x1, y1), Q(x2, y2), y1 < 0, y2 < 0, be the end-points
tangents are parallel to the line 9y = 8x are of the latus rectum of the ellipse x2 + 4y2 = 4. The equa-
Ê 2 1ˆ Ê 2 1ˆ tion of the parabola with latus rectum PQ are
(a) Á , ˜ (b) Á - , ˜
Ë 5 5¯ Ë 5 5¯ (a) x 2 + 2 3 y = 3 + 3 (b) x 2 - 2 3 y = 3 + 3
Ê 2 1ˆ Ê 2 1ˆ (c) x 2 + 2 3 y = 3 - 3 (d) x 2 - 2 3 y = 3 - 3
(c) Á - , - ˜ (d) Á , - ˜
Ë 5 5¯ Ë 5 5¯
[IIT-JEE, 2008]
[IIT-JEE, 1999]
2
18. The line passing through the extremity A of the major
x y2 axis and extremity B of the minor axis of the ellipse
11. Let P be a point on the ellipse 2 + 2 = 1 0 < b < a.
a b x2 + 9y2 = 9 meets its auxiliary circle at the point M.
Let the line parallel to y-axis passing through P meet Then the area of the triangle with vertices at A, M and
the circle x2 + y2 = a2 at the point Q such that P and the origin O is
Q are on the same side of x-axis. For two positive real (a) 31/10 (b) 29/10
numbers r and s, find the locus of the point R on PQ (c) 21/10 (d) 27/10
such that PR : RQ = r : s and p varies over the two [IIT-JEE, 2009]
ellipses. [IIT-JEE, 2001] 19. The normal at P on the ellipse x2 + 4y2 = 16 meets the
12. Prove that, in an ellipse, the perpendicular from a focus x-axis at Q. If M is the mid-point of the line segment
upon any tangent and the line joining the centre of the PQ, the locus of M intersects the latus rectums of the
ellipse to the point of contact meet on the correspond- given ellipse at the points
ing directrix. [IIT-JEE, 2002] (a) (± 3 5 /2, ± 2/7) (b) (± 3 5 /2, ± 19 /7)
13. The area of the quadrilateral formed by the tangents at (c) (± 2 3, ± 1/7) (d) (± 2 3, ± 4 3/7)
x2 y 2 [IIT-JEE, 2009]
the end-points of latus rectum to the ellipse + = 1,
is 9 5
(a) 27/4 s.u. (b) 9 s.u. Comprehension
(c) 27/2 s.u. (d) 27 s.u. Tangents are drawn from the point P(3, 4) to the ellipse
[IIT-JEE, 2003] x2 y 2
14. If tangents are drawn to the ellipse x2 + 2y2 = 2, the + = 1 touching the ellipse at points A and B.
9 4
locus of the mid-point of the intercept made by the tan-
gents between the co-ordinate axes is 20. The co-ordinates of A and B are
(a) (3, 0) and (0, 2)
1 1 1 1
(a) + 2 =1 (b) + 2 =1
2x 2
4y 4x 2
2y Ê 8 2 16 ˆ Ê 9 8ˆ
(b) ÁË - , ˜ and ÁË - , ˜¯
5 15 ¯ 5 5
x2 y 2 x2 y 2
(c) + =1 (d) + =1 Ê 8 2 16 ˆ
2 4 4 2 (c) Á - , and (0, 2)
Ë 5 15 ˜¯
[IIT-JEE, 2004]
Ê 9 8ˆ
15. Find the equation of the common tangent in first quad- (d) (3, 0) and Á - , ˜
2 2 Ë 5 5¯
x y
rant to the circle x2 + y2 = 16 and the ellipse + = 1. 21. The orthocentre of DPAB is
25 4
Ê 8ˆ Ê 7 25 ˆ
Also find the length of the intercept of the tangent be- (a) Á 5, ˜
Ë 7¯
(b) ÁË , ˜¯
tween the co-ordinate axes. [IIT-JEE, 2005] 5 8
Ê 11 8 ˆ Ê 8 7ˆ
16. A triangle is formed by a tangent to the ellipse (c) Á , ˜ (d) ÁË , ˜¯
Ë 5 5¯ 25 5
x2 y 2
+ = 1 and the co-ordinate axes. The area of the [IIT-JEE, 2010]
a 2 b2 22. The equation of the locus of the point whose distances
triangle cannot be less than from the point P and the line AB are equal, is

CG_05.indd 23 2/6/2017 4:05:02 PM


5.24 Coordinate Geometry Booster

(a)9x2 + y2 – 6xy – 54x – 62y + 241 = 0 The m1 is the slope of T1 and m2 is the slope of T2, then
(b)x2 + 9y2 + 6xy – 54x + 62y – 241 = 0 Ê 1 ˆ
the value of Á 2 + m22 ˜ is... [IIT-JEE-2015]
(c)9x2 + 9y2 – 6xy – 54x – 62y – 241 = 0 Ë m1 ¯
(d)x2 + y2 – 2xy + 27x + 31y – 120 = 0.
27. Let E1 and E2 be two ellipses whose centers are at the
[IIT-JEE, 2011]
2 2
origin. The major axes of E1 and E2 lie along the x-axis
x y and the y-axis, respectively. Let S be the circle x2 + (y
23. The ellipse E1 : + = 1 is inscribed in a rectangle
9 4 – 1)2 = 2. The straight line x + y = 3 touches the curves
R whose sides are parallel to the co-ordinate axes. S, E1 ad E2 at P, Q and R, respectively. Suppose that
Another ellipse E2: passing through the point (0, 4)
2 2
circumscribes the rectangle R. The eccentricity of the PQ = PR = . If e1 and e2 are the eccentricities of
ellipse E2 is 3
E1 and E2, respectively, then the correct expression(s)
2 3 1 3 is(are)
(a) (b) (c) (d)
2 2 2 4 43 7
(a) e12 + e22 = (b) e1e2 =
[IIT-JEE, 2012] 40 2 10
24. A vertical line passing through the point (h, 0) inter- 5 3
x2 y 2 (c) | e12 - e22 | = (d) e1e2 =
sects the ellipse + = 1 at the points P and Q. Let 8 4
4 3 [IIT-JEE-2015]
the tangents to the ellipse at P and Q meet at R. 28. Comprehension
If D(h) = area of DPQR, Let F1(x1, 0) and F2(x2, 0), for x1 < 0 and x2 > 0, be the
D1 = max D (h) x2 y 2
1 £ h £1 foci of the ellipse + =1
2 9 8
and D 2 = min D (h) Suppose a parabola having vertex at the origin and fo-
1 £ h £1
2 cus at F2 intersects the ellipse at point M in the first
8 quadrant and at point N in the fourth quadrant.
then D1 - 8D 2 =........ (i) The orthocentre of the triangle F1MN is
5 [IIT-JEE, 2013]
Ê 9 ˆ Ê2 ˆ
25. If the normal from the point P(h, 1) on the ellipse (a) Á - , 0˜ (b) Á , 0˜
Ë 10 ¯ Ë3 ¯
x2 y 2
+ = 1 is perpendicular to the line x + y = 3, then
6 3 Ê9 ˆ Ê2 ˆ
(c) Á , 0˜ (d) Á , 6 ˜
Ë 10 ¯ Ë3 ¯
the value of h is... [IIT-JEE, 2014]
x2 y 2 (ii) If the tangents to the ellipse at M and N meet at R
26. Suppose that the foci of the ellipse + = 1 are (f1, and the normal to the parabola at M meets the x-
9 5
0) where (f2, 0). Let P1 and P2 be two parabolas with a axis at Q, then the ratio of area of the triangle MQR
common vertex at (0, 0) and with foci at (f1, 0) and (2f2, to area of the quadrilateral MF1NF2 is
0), respectively. Let T1 be a tangent to P1 which passes (a) 3:4 (b) 4:5 (c) 5:8 (d) 2:3
through (2f2, 0) and T2 be a tangent to P2 which passes [IIT-JEE-2016]
through (f1, 0).

A NSWERS

LEVEL I 6. e =
5 -1
2
2. 10 7. 0
3. 4 < a < 10 p
1 8. q =
4. 6
3 9. 24
x2 y 2 ( x - 3) 2 ( y — 2) 2
5. + =1 10. + =1
16 12 16 9

CG_05.indd 24 2/6/2017 4:05:02 PM


Ellipse 5.25

40 Ê a 2l b 2 m ˆ
11. SQ = 64. Á - ,- ˜
3 Ë n n ¯
12. (9p – 6) s.u. 66. (1, 1)
x2 y 2 67. a2c4 x2 + b2d4 y2 = 1
13. + =1
4 3
x2 y 2 1
14. 2013 68. + =
15. 2 a 4 b4 c2
1 a 6 b6
16. y = - x + 13 69. + = (a 2 - b 2 )2
2 x2 y 2
Ê ± ( - 48) ± 36 ˆ 70. 4a2x2 = y2b2 + 4a2a2
17. ÁË , ˜
5 5 ¯ b2 x
71. y = -
18. l and ±5 a2m
19. y = 2x ± 14 72. P(a cos j, b sin j),
34. x2 + y2 = 25 Q(–a cos j, –b sin j),
35. x2 + y2 = 25 S(–a sin j, b cos j),
36. 4x2 + 3y2 – 12xy + 4x + 6y – 3 = 0 and R(a sin j, –b cos j)
cosec a sec a 1 x2 y 2
75 + =2
50. cosec b sec b 1 = 0 a 2 b2
cosec g sec g 1 x2 y 2 1
76. + =
Ê x2 y 2 ˆ 1 a 2 b2 2
1
51. Á 4 + 4 ˜ = 2 + 2 77. a3y + b3x = 0
Ëa b ¯ a b
a 2 b2
x2 y 2 1 78. 2 + 2 = 2
52. + = c d
a 4 b4 c2
2 2 79. 2(a x + b2y2)3 = (a2 – b2)(a2x2 – b2y2)2
2 2

2 2 Êx y2 ˆ 2 2
53. (a + b ) Á 2 + 2 ˜ = ( x + y ) x2 y 2
Ëa b ¯ 81. + =2
2 a 2 b2
Ê x2 y 2 ˆ Ê c2 ˆ 1
54. Á 2 + 2 ˜ = Á 2 ˜ 82.
Ëa b ¯ Ë x + y2 ¯ 3
a 2b 2 83. 4x + 3y = 12
55. x ◊ y =
l 84. x + y = 2
2
Ê a 6 b6 ˆ Ê x 2 y 2 ˆ 2 2 2
56. Á 2 + 2 ˜ Á 2 + 2 ˜ = (a - b )
Ëx y ¯Ëa b ¯ LEVEL II
2 2ˆ 2 2ˆ2
2Ê x y Êx y
57. a Á 4 + 4 ˜ = Á 2 + 2 ˜ 1. (c) 2. (c) 3. (a) 4. (c) 5. (a)
Ëa b ¯ Ë a b ¯ 6. (c) 7. (a) 8. (a) 9. (b) 10. (c)
2
Ê x2 y 2 ˆ Ê 1 1 ˆ Ê x2 y 2 ˆ 11. (d) 12. (d) 13. (a) 14. (a) 15. (b)
58. Á 2 + 2 ˜ ÁË 2 + 2 ˜¯ = ÁË 4 + 4 ˜¯ 16. (a, b) 17. (d) 18. (a) 19. (a) 20. (a,b,c,d)
Ëa b ¯ a b a b
21. (a) 22. (c) 23. (a) 24. (a) 25. (d)
Ê x2 y 2 ˆ 1 26. (c) 27. (b) 28. (c) 29. (c) 30. (b)
59. Á 2 + 2 ˜ =
Ëa b ¯ 2 31. (a) 32. (d) 33. (a) 34. (b) 35. (d)
2 36. (c) 37. (c) 38. (d) 39. (d) 40. (a)
Ê x2 + y 2 ˆ Ê x2 y 2 ˆ
60. Á 2 ˜ =Á + ˜ 41. (a) 42. (c) 43. (a) 44. (a) 45. (d)
Ë a + b2 ¯ Ë a 2 b2 ¯ 46. (c) 47. (d) 48. (a) 49. (a) 50. (d)
61. b2x2 + a2y2 = ab2 xc 51. (b) 52. (c) 53. (c) 54. (c) 55. (a)
2 56. (b) 57. (b) 58. (a) 59. (b) 60. (c)
a 2 x2 b 2 y 2 Ê x2 y 2 ˆ 61. (a) 62. (c) 63. (d) 64. (c) 65. (c)
62. + 4 =Á 2 + 2˜
a4 b Ëa b ¯ 66. (d) 67. (d) 68. (d) 69. (c) 70. (b)
a
63. x =
e

CG_05.indd 25 2/6/2017 4:05:03 PM


5.26 Coordinate Geometry Booster

LEVEL III 30.


1
r12
1 1 1
+ 2 = 2 + 2
r2 a b
2 2
1. b (a - b ) 5
2. 4 31.
3
2 1
4. j = p - tan 2, t = - , j = p + tan -1 (2) , 32. c = 14 or –6, A = (–4, 6), D = (3, 0)
5
1 p
t=
5
;j = ± ,t = 0
2 LEVEL IV
1
5. 3 3
2 1. ab
4
Ê a2 b2 ˆ
7. Á , Ê3 ˆ
2 2 2 2 ˜ 2. 1 s.u., Á , 1˜
Ë (a + b ) (a + b ) ¯ Ë2 ¯
121 11
8. ( x - 1) 2 + y 2 = 3. ( x - 1) 2 + y 2 =
9 3
2 2 2 2 2 2Êpˆ
9. a p + b q = r sec ÁË ˜¯ Êpˆ
8 4. r 2 = cos 2 Á ˜ ¥ (a 2 p 2 + b 2 q 2 )
Ë 8¯
12. 27 s.u.
p
13. INTEGER TYPE QUESTIONS
6
1 1 1. 6 s.u.
14. + =1
2x 2 4y 2 2. 2 s.u.
15. ab s.u. 3. 2
4. 3 s.u.
2 7 14 5. 5
16. Y = — x+4 ;
3 3 3 6. 4 s.u.
17. x2 + y2 = 1 7. 6
8. 3
19. 4
9. 7
2 10. 3
20.
3- 5
COMPREHENSIVE LINK PASSAGES
Ê b2 x2 + a 2 y 2 ˆ 1 Ê x2 y 2 ˆ Passage I.: 1(a) 2(d) 3(c)
21. Á 4 2 4 2˜
= 2 Á1 - 2 - 2 ˜
Ëa y +b x ¯ c Ë a b ¯ Passage II: 1(b) 2(a) 3(c)
Passage III: 1(a) 2(b) 3(a)
Ê 2b 2 ˆ Passage IV: 1(a) 2(c) 3(b)
22. y 2 = Á x
Ë a ˜¯ Passage V: 1(c) 2(c) 3(d) 4(a)
23. (x2 + y2)(x2 + y2 – a2 – b2 = 2xy(a2 – b2) Passage VI: 1(a) 2(c) 3(a) 4(a)
24. {(x2 – y2) + (b2 + a2)} tan (2a) + 2xy MATRIX MATCH
25. 5 2 1. (A) Æ (Q); (B) Æ (P, R); (C) Æ (S); (D) Æ (T)
2. (A) Æ (Q); (B) Æ (P); (C) Æ (R); (D) Æ (S);
x 2 9y 2 (E) Æ (U); (F) Æ (T)
26. + =1
9 4 3. (A) Æ (Q); (B) Æ (P); (C) Æ (R); (D) Æ (S);
27. (E) Æ (T)
28. 4. (A) Æ (R); (B) Æ (P); (C) Æ (S); (D) Æ (Q)
p 5p 5. (A) Æ (R); (B) Æ (Q); (C) Æ (P)
29. or
4 4 6. (A) Æ (S); (B) Æ (P); (C) Æ (R); (D) Æ (Q)

CG_05.indd 26 2/6/2017 4:05:03 PM


Ellipse 5.27

H INTS AND S OLUTIONS

LEVEL I (c) Co-vertices: (0, ±b)


fi X = 0, Y = ±3
fi x – 1 = 0, y – 2 = ±3
1. (i) The given equation of the ellipse is 9x2 + 16y2 = 144 fi x = 1, y = 2 ± 3
x2 y 2 fi x = 1, y = 5, –1
fi + =1 Hence, the co-vertices are (1, 5), (1, –1).
16 9
(d) The length of the major axis = 2a = 6
(a) Centre: C(0, 0) (e) The length of the minor axis = 2b = 4
(b) Vertices: A(a, 0) A¢(–a, 0) = A(4, 0) and A¢(– (f) The length of the latus rectum =
4, 0)
2b 2 18 9
(c) Co-vertices: B = (0, b) and B¢ =(0, –b) = =
fi B = (0, 3) and B¢ = (0, –3) a 4 2
(d) Length of the major axis: 2a = 8 (g) Eccentricity,
(e) Length of the minor axis: 2b = 6 b2 2 1
e= 1- 2
= 1- =
b2 9 7 a 3 3
(f) Eccentricity = e = 1- 2
= 1- =
a 16 4 (h) Equations of the directrices:
a 3
2b 2 18 9 X =± =± = ±3 3
(g) Lengths of the latus rectum = = = e 1/ 3
a 4 2
(h) Equations of the directrices: fi x =1± 3 3
a (i) End-points of the latus recta:
x=±
e Ê b2 ˆ Ê b2 ˆ
4 16 L Á ae, ˜ ; L ¢ Á ae, - ˜
fi x=± =± Ë a¯ Ë a¯
7 7
Ê 9ˆ Ê 9ˆ
4 = L Á 3, ˜ ; L ¢ Á 3, - ˜
Ë 2¯ Ë 2¯
(i) End-points of the latus recta:
2. The given equation of the ellipse is
Ê b2 ˆ Ê b2 ˆ 16x2 + 25y2 = 400
L Á ae, ˜ , L ¢ Á ae, — ˜
Ë a¯ Ë a¯ x2 y 2
fi + =1
Ê 9ˆ Ê 9ˆ 25 16
= L Á 7, ˜ , L ¢ Á 7, - ˜
Ë 4 ¯ Ë 4¯ Thus, the sum of the focal distances
= 2a = 10 units
(ii) The given equation of the ellipse is x2 y2
3. The given equation of an ellipse is + =1
2x2 + 3y2 – 4x – 12y + 8 = 0 10 - a a — 4
fi 2(x2 – 2x + 1) + 3(y2 – 4y + 4) = 6 fi (a – 4)x2 + (10 – a)y2 – (a – 4)(10 – a) = 0
fi 2(x – 1)2 + 3(y – 2)2 = 6 Since, the given equation represents an ellipse,
h2 – ab < 0
( x — 1) 2 ( y — 2) 2 fi 0 – (a – 4)(10 – a) < 0
fi + =1
3 2 fi (a – 4)(a – 10) < 0
fi 4 < a < 10
X2 Y2
fi + = 1 , where X = x – 1, Y = y – 2 Hence, the length of the interval = 10 – 4 = 6.
3 2 Thus, the value of m is 6.
(a) Centre: (0, 0) 4. Let the foci of the ellipse are S and S¢ respectively.
fi X = 0, Y = 0 Then S = (5, 12) and S¢ = (24, 7).
fi x – 1 = 0 and y – 2 = 0 Ê 29 19 ˆ
Thus, the centre of the ellipse is C Á , ˜ .
fi x = 1 and y = 2 Ë 2 2¯
Hence, the centre is (1, 2). 1 1
(b) Vertices : (±a, 0) Now OC = ¥ 292 + 192 = ¥ 1202
2 2
fi X = ±a, Y = 0 We have,
fi x – 1 = ±3, y – 2 = 0 SS¢ = 386
fi x = 1 ± 3, y = 2
fi x = 4, –2, y = 2 386 386 386 1
fi e= = = =
Hence, the vertices are (4, 2) and (–2, 2) 2a 1202 1202 3

CG_05.indd 27 2/6/2017 4:05:04 PM


5.28 Coordinate Geometry Booster

5. Let S(2, 0) and S¢(–2, 0) are two foci of the ellipse and fi 4 cos2 q = 3
C(0, 0) be the centre of the ellipse.
3 3
We have, fi cos q = =
SS¢ = 4 4 2
fi 2ae = 4 p
fi q=
1 6
fi 2a ¥ = 4 fia=4 p
2 Hence, the eccentric angle is .
6
Ê 1ˆ
Also, b 2 = a 2 (1 - e 2 ) = 16 Á1 - ˜ = 12 9. Let Q be any point
Ë 4¯ S P
fi on the given ellipse,
b = 12 = 2 3
whose co-ordinates
Hence, the equation of the ellipse is are (4 cos2 q, 3 sin2 N C M
x2 y 2 q).
+ =1 Thus, PQ = 8 cos q R Q
a 2 b2
and QR = 6 sin q
x2 y 2
fi + =1 Thus, the area of the rectangle PQRS
16 12 = PQ ¥ QR
2b 2 = 48 sin q cos q
6. Given 2ae =
a = 24 sin 2q
fi 2
b =a e2 Hence, the area of the greatest rectangle
fi a2(1 – c2) = a2c p
= 24 sq.u. at q =
fi (1 – c2) = c 4
fi c2 + c – 1 = 0 10. The co-ordinates of the given point are
5 -1 x = 3 + 4 cos q, y = 2 + 3 sin q
fi e=
2 ( x - 3) (y — 2)
fi = cos q , = sin q
Hence, the result 4 3
x2 y 2
7. The equation of the given ellipse is 2 + 2 = 1 ( x - 3) 2 ( y — 2) 2
a b fi + = cos 2q + sin 2q = 1
1 42 32
It is given that the length of the latus rectum = ¥ the
length of the major axis. 2 ( x - 3) 2 ( y — 2) 2
fi + =1
16 9
2b 2
fi =a which is the required locus.
a
11. The equation of the ellipse is
fi b=a
16x2 + 25y2 = 400
As we know that,
x2 y 2
b2 fi + =1
eccentricity = 1 - = 1-1 = 0 25 16
a2 As we know that, if SP and SQ are the focal segments
x2 y 2 1 1 1
8. Let any point on the ellipse + = 1 is of a focal chord PSQ, then + =
6 2 SP SQ a
P( 6 cos q, 2 sin q) and C be the centre of the el- B
lipse. P

B
P(q) A¢ A
C S

A¢ A
C B¢ Q

1 1 1
fi + =
B¢ 8 SQ 5
Therefore,
1 1 1 8-5 3
CP = 5 fi = - = =
SQ 5 8 40 40
fi 6 cos2 q + 2 sin2 q = 5

CG_05.indd 28 2/6/2017 4:05:05 PM


Ellipse 5.29

40 3 = 2m + 16m 2 + 9
fi SQ =
3
fi (3 – 2m)2 = 16m2 + 9
Hence, the length of SQ is 40/3.
fi 9 – 6m + 4m2 = 16m2 + 9
x2 y 2
12. As we know that the area of an ellipse 2 + 2 = 1 is fi 12m2 + 6m = 0
pab. a b fi 2m2 + m = 0
Y fi m(2m + 1) = 0
B
C 1
fi m = 0, —
2
X¢ A X
O Hence, the equation of tangents are y = 3 and
1
y = - x + 13.
Y¢ 2
17. The given line is
Thus, the area of OACBO 3x + 4y = 5
Ê3 1 ˆ fi 4y = –3x + 5
= Á ¥ p ◊ 4 ◊ 3 - ◊ 4 ◊ 3˜
Ë4 2 ¯
and the ellipse is
= (9p – 6) s.u. 9x2 + 16y2 = 144
x2 y 2
13. Let N = 1 + 1 -1 3 5 x2 y 2
4 3 fi y=- x + and + =1
4 4 16 9
4 9
Then, N = + -1=1+ 3 -1= 3 As we know that the line y = mx + c will be a tangent
4 3
Since, the value of N is positive at (2, 3), so the point x2 y 2
to the ellipse + = 1 , the points of contacts are
x2 y 2 a 2 b2
(2, 3) lies outside of the ellipse + = 1.
4 3 Ê a 2m b2 ˆ
ÁË ± ,± ˜
14. Since the point (l, –l) be an interior point of an ellipse c c¯
4x2 + 5y2 = 1, then
Ê Ê 3ˆ ˆ
4l2 + 5l2 – 1 < 0 Á ± 16 ÁË - 4 ˜¯ ± 9 ˜
fi 9l2 – 1 < 0 =Á , ˜
Á 5 5 ˜
fi (3l + 1)(3l – 1) < 0 Ë 4 4 ¯
1 1
fi - <l< Ê ± ( - 48) ± 36 ˆ
3 3 =Á , ˜
Ë 5 5 ¯
2
Therefore, m = 18. The equation of the given ellipse is
3 9x2 + 16y2 = 144
fi (3m – 2)2013 + 2013 = 0 + 2013 = 2013
x2 y 2
15. We have, fi + =1
16 9
4.4 + 3.3 – 12 = 16 + 9 – 12 = 13 > 0
Thus, the point (2, 3) lies outside of the ellipse. The line y = x + l will be a tangent to the ellipse, if
Thus, the number of tangents = 2 c2 = a2m2 + b2
16. The given ellipse is fi l2 = 16.1 + 9 = 25
9x2 + 16y2 = 144
fi l = ±5
x2 y 2
fi + =1 Hence, the values of l are ±5.
16 9
The equation of any tangent to the given ellipse is x2 y 2
19. The given ellipse is + =1
3 4
y = mx + a 2 m 2 + b 2
The equation of any ellipse is

fi y = mx + 16m 2 + 9 y = mx ± a 2 m 2 + b 2
which is passing through (2, 3), so fi y = 2x ± 3.4 + 2 = 2x ± 14

CG_05.indd 29 2/6/2017 4:05:06 PM


5.30 Coordinate Geometry Booster

x2 y 2 Ê Ê r 2 - b2 ˆ ˆ
20. The equation of any tangent to the ellipse 2 + 2 = 1 fi q = tan -1 Á Á 2 ˜
at P(q) is a b ÁË Ë a - r 2 ˜¯ ˜¯
x y
cos q + sin q = 1 …(i) Hence, the result.
a b 22. Let the end-points of a latus rectum are
The equation of perpendiculars from centre (0, 0) to
L(ae, 0) and L (–ae, 0).
tangent is
xe y
x y The tangent at L is + = 1 and the tangent at L¢ is
sin q - cos q = 0 …(ii) a a
b a xe y
- =1.
From Eq. (ii), we get a a
sin q cos q 1 On solving, we get,
= = …(iii) a
by ax a x + b2 y 2
2 2 x = and y = 0.
e
The locus of the feet of perpendiculars is the point of a
intersection of (i) and (ii). fi x= is the equation of directrix to the ellipse
e
It is obtained by eliminating q between Eqs (i) and (ii).
x2 y 2
Squaring Eqs (i) and (ii) and adding, we get + = 1.
x2 y2 x2 y2 a 2 b2
2 2 2
cos q + sin q + sin q + cos 2q = 1 23. The equation of any tangent to the ellipse at P(q) is
a2 b2 b2 a2
Ê x2 y 2 ˆ Ê y 2 x2 ˆ 2 x y
2 AB: cos q + sin q = 1.
fi ÁË 2 + 2 ˜¯ cos q + ÁË 2 + 2 ˜¯ sin q = 1 a b
a a b b Thus A = (a sec q, 0) and A = (0, b cosec q)
Ê cos 2q sin 2q ˆ Let (h, k) be the mid-point of the tangent AB.
fi ( x2 + y 2 ) Á 2 + 2 ˜ = 1
Ë a b ¯ a sec q b cosec q
Therefore, h = and k =
fi (x + y )(b cos q + a2 sin2 q) = a2b2
2 2 2 2
2 2
Ê b2a 2 x2 b2a 2 y 2 ˆ a b
fi ( x2 + y 2 ) Á 2 2 + 2 2 fi cos q = and sin q =
˜ =a b 2h 2k
Ë a x + b2 y 2 a 2 x2 + b2 y 2 ¯
Now squaring and adding, we get
fi (x2 + y2)2 = (a2x2 + b2y2)
Now, put, x = r cos q and y = r sin q, we get a2 b2
+ =1
r2 = a2cos2 q + b2 sin2 q. 4h 2 4k 2
Hence, the result. a 2 b2
fi + =4
21. The equation of circle is x2 + y2 = r2. h2 k 2
So the equation of any tangent to the ellipse is Hence the locus of (h, k) is
y = mx + a 2 m 2 + b 2 a 2 b2
+ =4
If it is a tangent to a circle also, then the length of the x2 y 2
perpendicular from the centre (0, 0) of a circle is equal Now, put x = r cos q c y = r sin q , we get,
to the radius of a circle. 4r2 = a2 sin2 + b2 cos2 q ,
Hence, the result.
a 2 m2 + b2
Thus, =r x2 y 2
1 + m2 24. Given ellipses are 2 + 2 = 1 …(i)
a b
fi a 2 m 2 + b 2 = r 2 (1 + m 2 ) = r 2 + r 2 m 2 x2 y 2
and + =a+b
fi (a2 – r2)m2 = (r2 – b2) a b
x2 y2
Ê r 2 - b2 ˆ fi + =1 …(ii)
fi m2 = Á 2 ˜ a ( a + b) b( a + b)
Ë a - r2 ¯
Let R(h, k) be the points of intersection of the tangents
Ê r 2 - b2 ˆ to the ellipse (ii) at P and Q. Then PQ will be the chord
fi m= Á 2 ˜ of contact.
Ë a - r2 ¯
Thus its equation will be
Ê r 2 - b2 ˆ hx ky
fi tan q = Á 2 ˜ + =1
Ë a - r2 ¯ a ( a + b) b( a + b)

CG_05.indd 30 2/6/2017 4:05:06 PM


Ellipse 5.31

bh b( a + b) Therefore, the equation of a director circle is


fi y=- x+ …(iii) x2 + y2 = a2 + b2
ak k
Since the line (iii) is a tangent to the ellipse (i), so we This means that the centre of the ellipse will always
have remain at a constant distance a 2 + b 2 from P.
b 2 ( a + b) 2 2 b2 h2 Hence, the locus of the centre is a circle.
2
= a ¥ 2
¥ 2 + b2
k a k x2 y 2
27. The equation of any tangent to the ellipse 2 + 2 = 1
fi b2(a + b)2 = b2h2 + b2k2 a b
at P(q) is
fi h2 + k2 = (a + b)2
Y
Thus the locus of (h, k) is
x2 + y2 = (a + b)2 Q
P
which is the director circle of the ellipse
p1 p2
x2 y 2 X¢ X
+ =a+b S¢ O S
a b
Hence, the tangents at P and Q are at right angles.
25. Let two foci of an ellipse are Y¢
S(ae, 0) and S(–ae, 0). x y
cos q + sin q = 1 …(i)
x2 y 2 a b
The equation of the tangent to the ellipse 2 + 2 = 1
a b
x y The equation of perpendiculars from foci (±ae, 0) to
at P(q) is cos q + sin q = 1 . the tangent is
a b
Let p1 and p2 be the lengths of perpendiculars from the x y ae sin q
sin q - cos q = ± …(ii)
given foci to the given ellipse. b a b
1 - e cos q Locus of the feet of perpendiculars is the point of inter-
Thus, p1 = section of (i) and (ii).
2 2
cos q sin q
+ 2 It is obtained by eliminating q between Eqs (i) and (ii).
a2 b
Squaring Eqs (i) and (ii) and adding, we get
1 + e cos q
and p2 = Ê cos 2q sin 2q ˆ Ê cos 2q sin 2q ˆ
cos 2q sin 2q fi x2 Á 2 + 2 ˜ + y 2 Á 2 + 2 ˜
+ 2 Ë a b ¯ Ë a b ¯
a2 b
a 2 e2 sin 2q
1 - e cos q 1 + e cos q =1+
fi p1 ¥ p2 = b2
cos 2q sin 2q cos 2q sin 2q
+ 2 + 2 Ê cos 2q sin 2q ˆ Ê cos 2q sin 2q ˆ
a2 b a2 b fi ( x 2 + y 2 )Á 2 + 2 ˜ = a 2 Á 2 + 2 ˜
Ë a b ¯ Ë a b ¯
Ê 1 - e2 cos 2q ˆ
=Á fi (x2 + y2) = a2
2 2 ˜
Á cos q + sin q ˜ Hence, the result.
Ë a2 b2 ¯ x y
28. The tangent at P(q) is cos q + sin q = 1 .
Ê a 2b 2 (1 - e 2cos 2q) ˆ a b
=Á 2 2 ˜
Ë b cos q + a 2sin 2q ¯ Y
Ê a 2b 2 (1 - e 2cos 2q) ˆ
=Á 2 2 2 2 2 ˜ M
Ë a (1 - e ) cos q + a sin q ¯ P(q)
p
= b2 X
O S(æ, 0)
26. Consider P is the point of intersection of two perpen-
dicular tangents. Thus the locus of P is the director
circle.

CG_05.indd 31 2/6/2017 4:05:07 PM


5.32 Coordinate Geometry Booster

Also SP = a – ex = a(1 – e cos q)


a 2b 2
Since p is the length of perpendicular from the focus fi = a 2sin 2q + b 2cos 2q
p2
S(ae, 0),
e cos q - 1 = a 2 (1 - cos 2q) + b 2 (1 - sin 2q)
then, p = = a2 + b2 – (a2 cos2 q + b2 sin2 q)
Ê cos 2q sin 2q ˆ
ÁË 2 + 2 ˜¯ = a2 + b2 – r2
a b Hence, the result.
1 (b 2cos 2q + a 2sin 2q ) 30. Any tangent to the ellipse
fi 2
= 2 2 x y
p a b (e cos q - 1) 2 cos q + sin q = 1
a b
b 2 (b 2cos 2q + a 2sin 2q ) Let it meet the x-axis at A and y-axis at B. Then the co-
fi =
p2 a 2 (e cos q - 1) 2 ordinates of A and B are

È a 2 (1 - e2 ) cos 2q + a 2sin 2q ˘ Ê a ˆ Ê b ˆ
A=Á , 0˜ and B = Á 0,
=Í ˙ Ë cos q ¯ Ë sin q ˜¯
Î a 2 (e cos q - 1) 2 ˚
Let M(h, k) be the mid-point of AB.
È (1 - e 2 ) cos 2q + sin 2q ˘ a b
=Í ˙ Then 2h = and 2k =
Î (e cos q - 1) 2 ˚ cos q sin q
a b
È (1 - e 2cos 2q) ˘ fi h= and k =
=Í 2˙ 2 cos q 2 sin q
Î (e cos q - 1) ˚ a b
fi cos q = and sin q =
È1 + e cos q ˘ 2h 2k
=Í ˙
Î1 - e cos q ˚ Squaring and adding, we get
Also, a2 b2
2
+ 2 =1
2a 2a 1 + e cos q 4h 4k
-1= -1= 2
SP a (1 - e cos q ) 1 - e cos q a b2
fi + =4
b 2 2a h2 k 2
Hence, = -1
p 2 SP Hence, the locus of M(h, k) is
29. Let the point P be (a cos q c b sin q). a 2 b2
+ =4
The equation of the tangent at P be x2 y 2
x y 31. The tangent at P(a cos q, b sin q) is
cos q + sin q = 1
a b x y
cos q + sin q = 1
a b
a
Let it meets the directrix x = at Q, where Q is
e
M
P Ê a b(e - cos q ) ˆ
r ÁË e , e sin q ˜¯ .
X¢ X
O
b(e - cos q )
Slope of SP = m1 =
e sin q
Y¢ b(e - cos q) Ê a ˆ
Slope of SQ = m2 = ÁË - ae˜¯
e sin q e
0 + 0 -1
Now, p = b(e - cos q )
cos 2q sin 2q =
+ 2 a (1 - e 2 )sin q
a2 b Thus, m1 ¥ m2
2 2
1 cos q sin q b sin q b(e - cos q )
fi = + 2 = ¥
p 2
a2 b a (cos q - e) a (1 - e 2 )sin q
b2 b2
a 2sin 2q + b 2cos 2q =- = - = -1
= a 2 (1 - e 2 ) b2
a 2b 2
Hence, the result.

CG_05.indd 32 2/6/2017 4:05:08 PM


Ellipse 5.33

32. Any tangent to the ellipse is


a2 b2
x y fi sin 2q = and cos 2
q =
cos q + sin q = 1 …(i) a 2 + b2 a 2 + b2
a b
Its point of contact is P(a cos q b sin q) and its slope is Hence the point P is
b Ê a2 b2 ˆ
- cot q ± , ±
a Á 2 2 2 2˜
Ë a +b a +b ¯
Also the focus is S(ae, 0).
Any line through the focus S and perpendicular to the 34. As we know that the locus of the point of intersection
tangent (i) is of two perpendicular tangents is the director circle.
a Hence, the equation of the director circle to the given
y - 0 = tan q ( x - ae) …(ii) ellipse is
b
x2 + y2 = a2 + b2 = 16 + 9 = 25
Also the equation of CP is
fi x2 + y2 = 25
a
y - 0 = tan q ( x - 0) …(iii) 35. As we know that the locus of the point of intersection
b of two perpendicular tangents to an ellipse is the direc-
Eliminating q between Eqs (ii) and (iii), we get tor circle.
Hence, the equation of the director circle is
Ê a 2 ˆ Ê x - ae ˆ
ÁË 2 ˜¯ ÁË ˜ =1 x2 + y2 = a2 + b2 = 4 + 1 = 5
b x ¯ fi x2 + y2 = 5
2
Ê x - ae ˆ Ê b ˆ which is the required locus of P.
fi ÁË ˜¯ = Á 2 ˜
x Ëa ¯ 36. The equations of the pair of tangents to the ellipse
2x2 + 3y2 = 1 from the point (1, 1) is
Ê ae ˆ Ê b 2 ˆ (2x2 + 3y2 – 1)(2 + 3 – 1) = (2x + 3y – 1)2
fi ÁË1 - ˜¯ = Á 2 ˜
x Ëa ¯ fi 4(2x2 + 3y2 – 1) = (2x + 3y – 1)2
fi 4(2x2 + 3y2 – 1) = 4x2 + 9y2 + 1 + 12xy – 4x – 6y
Ê b 2 ˆ Ê ae ˆ
fi ÁË 1 - ˜ =Á ˜ fi 4x2 – 3y2 – 12xy – 4x + 6y – 3 = 0
a2 ¯ Ë x ¯ 37. The equation of the given ellipse is
Ê a 2 (1 - e 2 ) ˆ Ê ae ˆ 3x2 + 2y2 = 5
fi ÁË 1 - =
¯˜ ËÁ x ¯˜
a2 x2 y2
fi + =1
Ê ae ˆ 5/3 5/2
fi e2 = Á ˜
Ë x¯ The equation of any tangents to the ellipse are

a y = mx ± b 2 m 2 + a 2
fi x=
e which is passing through (1, 2), so
Hence, the result.
5 2 5
33. Consider a point P on the ellipse whose coordinates are 2=m± m +
2 3
(a cos q b sin q)
5 5
The equation of the tangent at P is fi (2 — m) 2 = m 2 +
2 3
x y
cos q + sin q = 1 15m 2 + 6
a b fi (2 — m) 2 =
5
Since the tangent makes equal angles with the axes, so
fi 9m + 24m – 14 = 0
2
its slope is ±1.
Let its roots are m1 and m2.
b cos q
Thus, - = ±1 Then m1 + m2 = –24/9 and m1m2 = –14/9
a sin q
Let q be the angle between them.
b 2cos 2q Then
fi =1
a 2sin 2q m - m1
tan (q) = 2
sin 2q cos 2q 1 1 + m1m2
fi = = 2
a2 b2 a + b2 (m1 + m2 ) 2 - 4m1m2
=
1 + m1m2

CG_05.indd 33 2/6/2017 4:05:08 PM


5.34 Coordinate Geometry Booster

B
64 56 P
+
= 9 9 = 120 ¥ 9 = 12 ¥ 5
14 3 5 6
1- A¢
C G
A
9 F

Ê 12 6 ˆ
fi q = tan -1 Á ˜ B¢
Ë 5 ¯
38. Do yourself. N
39. Do yourself. The equation of the normal at P is
40. Do yourself. ax sec q – by cosec q = a2 – b2 …(i)
41. Do yourself.
Since the line (i) meets the major axis at G and minor
Ê b2 ˆ
42. Let one end of a latus rectum is L Á ae, ˜ and minor axis at G¢ respectively, then
Ë a¯
axis be B(0, –b). Ê a 2 - b2 ˆ
The equation of the normal to the ellipse at L is G=Á cos q, 0˜
Ë a ¯
a
x - ay = a 2 - b 2 Ê a 2 - b2 ˆ
e and E = Á 0, - sin q˜
Ë a ¯
which is passing through B¢(0, –b), so we have
PF = CQ = length of perpendicular from C(0, 0) on the
ab = a2 – b2
x y
fi ab = a2 – a2(1 – 32) = a2e2 tangent cos q + sin q = 1 at P.
a b
fi b = ae2 ab
fi b2 = a2c4 =
a sin q + b 2cos 2q
2 2
fi a2(1 – e2) = a2e4
fi e4 + e2 – 1 = 0 b 2 2
Also, PG = b cos q + a 2sin 2q
Hence, the result. a
a 2 2
x2 y 2 and PG ¢ = b cos q + a 2sin 2q
43. Let the ellipse be 2 + 2 = 1 and P(a cos q b sin q) be b
a b
Thus, PF ◊ PG = b2 and PF ◊ PG¢ = a2 .
any point on the ellipse.
45. The equation of the normal to the ellipse at P(q) is
Then, ax sec q – by cosec q = a2 – b2)
SP = a – ex = a – ae cos q = a(1 – e cos q) Y
and P
SP = a + ex = a + ae cos q = a(1 + e cos q)
The equation of the normal at P is O
X¢ X
M
ax sec q – by cosec q = a2 – b2 N
Now,
Ê a 2 - b2 ˆ
L=Á cos q , 0˜ = (ae 2cos q , 0) Y¢
Ë a ¯
Thus the co-ordinates of M and N are
fi OL = ae2 cos q
Ê Ê b2 - a 2 ˆ ˆ
Thus, S¢L = ae + ae2 cos q = a3 (1 + e cos q), ÁË 0, ÁË ˜¯ sin q˜¯
b
SL = ae – ae2 cos = ae(1 – e cos q)
S ¢P 1 + e cos q S ¢L Ê Ê a 2 - b2 ˆ ˆ
Now, = = and ÁË ËÁ ˜¯ cos q, 0¯˜
SP 1 - e cos q SL a
fi PL bisects the –S¢PS internally. 2
2Ê b2 - a 2
2 ˆ
Since PL ^ PT, therefore, PT will bisect –S¢PS exter- PM = a cos q + Á - b˜ sin 2q
Ë b ¯
nally.
x2 y 2 a4
44. Let the ellipse be 2 + 2 = 1 and P(a cos q, b sin q) = a 2cos 2q + sin 2q
a b b2
be any point on the ellipse.
cos 2q sin 2q
= a2 + 2
a2 b

CG_05.indd 34 2/6/2017 4:05:09 PM


Ellipse 5.35

2 Êa b ˆ
Ê a 2 - b2 ˆ 2 2 2 Áb
tan q - tan q
˜
Also, PN = Á a -
Ë ˜ cos q + b sin q \ tan j = Á a
a ¯ 1 + tan 2
q ˜¯
Ë
b 4cos 2q Ê a 2 - b 2 ˆ tan q
= + b 2sin 2q =Á
a2 Ë ab ˜¯ sec 2q
Ê a 2 - b2 ˆ
2 cos 2q sin 2q =Á
Ë 2ab ˜¯
¥ sin 2q
=b + 2
a2 b
Thus, it is maximum when
Thus, PM : PN = a2 : b2 2q = 90° fi q = 45°.
Hence, the result.
Therefore, the maximum value is
46. Since the ordinate P meets the circle at Q, the co-ordi-
Ê a 2 - b2 ˆ
nates of P and Q are (a cos q, b sin q) and (a cos q, a ÁË ˜
sin q), respectively. 2ab ¯
Y 48. The equation of any normal to the ellipse at P(q) is
ax secq – by cosec q = a2 – b2 …(i)
Q
Let C be the centre of the ellipse.
P
Then CM = Length of perpendicular from the centre C
X¢ X to the normal
O N
(a 2 - b 2 )
=
R a 2sec 2q + b 2cosec 2q
(a 2 - b 2 )
Y¢ =
The equation of the normal to the ellipse at P(q) is a 2 + b 2 + a 2 tan 2q + b 2cot 2q
ax sec q – by cosec q = a2 – b2 …(i) (a 2 - b 2 )
< =a-b
and the equation of normal to the auxiliary circle at Q(a a 2 + b 2 + 2ab
cos q, a sin q) is
49. The equation of tangent at P is
y = (tan q)x …(ii) x y
Solving, we get cos j + sin j = 1 …(i)
a b
x y and the equation of normal at P is
cos q = and sin q = ax sec j – by cosec j = a2 – b2 …(ii)
( a + b) ( a + b)
\ Q = (a sec f, 0)
Squaring and adding, we get
x2 + y2 = (a + b)2 Ê (a 2 - b 2 ) cos j ˆ
and R = Á , 0˜
47. The equation of any normal at P(q) is Ë a ¯
x sec q – by cosec q = a2 – b2.
a
\ Slope of normal = m1 = tan q
b P
b
and slope of CP = m2 = tan q . R a Q
a

P
f Therefore,
C
QR = a
(a 2 - b 2 ) cos j
N fi a sec j - =a
a
Let f be the angle between CP and the normal at P. fi a2 sin2 j + b2 cos2 j = a2 cos j

CG_05.indd 35 2/6/2017 4:05:10 PM


5.36 Coordinate Geometry Booster

fi a2 sin2 j + a2(1 – e2) cos2 j = a2 cos j P(h, k)


fi a2(sin2 + cos2) – a2e2cos2 j = a2cos j
fi a2 – a2e2 cos2 = a2 cos j
fi c2 cos2 j + cos j – 1 = 0
Q R
x2 y 2
50. The equation of the normal to the ellipse 2 + 2 = 1 90°
at P(x , y ), Q(x , y ) and R(x , y ) are a b C
1 1 2 2 2 3

a 2 x b2 y
- = a 2 - b2 …(i) It subtends a right angle at the centre (0, 0) of the
x1 y1
ellipse
a 2 x b2 y
- = a 2 - b2 …(ii) x2 y 2
x2 y2 + =1 …(ii)
a 2 b2
a 2 x b2 y
and - = a 2 - b2 …(iii) From Eqs (i) and (ii), we get
x3 y3
2
Eliminating x and y from Eqs (i), (ii) and (iii), we get x 2 y 2 Ê xh yk ˆ
+ =Á + ˜
a 2 b2 Ë a 2 b2 ¯
a2 b2
- (a 2 - b 2 ) Ê 1 h 2 ˆ 2 Ê 1 k 2 ˆ 2 2hxky
x1 y1 fi ÁË 2 - 4 ˜¯ x + ÁË 2 - 4 ˜¯ y - 2 2 = 0
a a b b a b
a2 b2
- (a 2 - b 2 ) = 0 Since these lines are at right angles, therefore sum of
x2 y2
the co-efficients of x2 and y2 is zero.
a2 b2
- (a 2 - b 2 ) Ê 1 h2 ˆ Ê 1 k 2 ˆ
x3 y3 fi ÁË 2 - 4 ˜¯ + ÁË 2 - 4 ˜¯ = 0
a a b b
1 1
1 Ê h2 k 2 ˆ 1 1
x1 y1 fi ÁË 4 + 4 ˜¯ = 2 + 2
1 1 a b a b
fi 1=0
x2 y2 Hence, the locus of (h, k) is
1 1 Ê x2 y 2 ˆ 1 1
1 ÁË 4 + 4 ˜¯ = 2 + 2
x3 y3 a b a b
x1 y1 x1 y1 52. Let the point from which the tangents are drawn be (h,
fi x2 y2 x2 y2 = 0 k).
So the equation of the chord of contact from the point
x3 y3 x3 y3
(h, k) to the given ellipse is
Also hx ky
+ =1 …(i)
a cos a b sin a ab cos a sin a a 2 b2
a cos b b sin b ab cos b sin b = 0
P(h, k)
a cos g b sin g ab cos g sin g
cosec a sec a 1
fi cosec b sec b 1 = 0 Q M R
c
cosec g sec g 1 C
sec a cosec a 1
fi sec b cosec b 1 = 0 It touches the circle x2 + y2 = c2.
sec g cosec g 1 Therefore, the length of the perpendicular from the
centre (0, 0) to the chord of contact (i) is equal to the
Hence the result. radius of the circle.
51. Let the point from which tangents are drawn be (h, k). 0 + 0 -1
The equation of the chord of contact from the point (h, Thus, =c
hx ky h2 k 2
k) to the given ellipse is 2 + 2 = 1 . +
a 4 b4
a b

CG_05.indd 36 2/6/2017 4:05:10 PM


Ellipse 5.37

From Eqs (i) and (ii), we get


h2 k 2 1
fi + = h k 1
a 4 b4 c2 = = 2
c cos q c sin q h k2
Hence, the locus of (h, k) is
2
+ 2
x2 y 2 1 a b
+ = hÊ h 2 2
k ˆ
a 4 b4 c2 fi cos q = Á 2 + 2 ˜
53. Let (x1, y1) be the point of intersection of the perpendic- cËa b ¯
ular tangents, so that (x1, y1) lies on the director circle k Ê h2 k 2 ˆ
and sin q = Á + ˜
x12 + y12 = a 2 + b 2 …(i) c Ë a 2 b2 ¯
The equation of the chord of contact from (x1, y1) is Squaring and adding, we get
xx1 yy1 2 2
+ 2 =1 …(ii) Ê h Ê h2 k 2 ˆ ˆ Ê k Ê h2 k 2 ˆ ˆ
a2 b Á Á 2 + 2 ˜˜ + Á Á 2 + 2 ˜˜ =1
ËcËa b ¯¯ ËcËa b ¯¯
Let its mid-point be (h, k).
2
\ The equation of the chord bisected at (h, k) is Ê h2 k 2 ˆ Ê c2 ˆ
hx ky h 2 k 2
fi ÁË 2 + 2 ˜¯ = Á 2 ˜
+ = + …(iii)
a b Ë h + k2 ¯
a 2 b2 a 2 b2 Hence, the locus of (h, k) is
From Eqs (ii) and (iii), we get 2
Ê x2 y 2 ˆ Ê c2 ˆ
x1 y1 1 ÁË 2 + 2 ˜¯ = Á 2 ˜
= = a b Ë x + y2 ¯
h k Ê h2 k 2 ˆ
ÁË 2 + 2 ˜¯ 55. Let the co-ordinates of P be (h, k).
a b
Y
Now from Eqs. (i), we get
2 2 P(h, k)
Ê h ˆ Ê k ˆ B
+ = (a 2 + b 2 )
Á Ê h2 k 2 ˆ ˜ Á Ê h2 k 2 ˆ ˜
ÁÁ 2 + 2˜˜ ÁÁ 2 + 2˜˜
ÁË Ë a b ¯ ˜¯ ÁË Ë a b ¯ ˜¯

O A
X

2
Ê h2 k 2 ˆ
fi (a 2 + b 2 ) Á 2 + 2 ˜ = (h 2 + k 2 )
Ëa b ¯ Y¢
Hence, the locus of (h, k) is The equation of its chord of contact with respect to the
2
Ê x2 y 2 ˆ ellipse
(a 2 + b 2 ) Á 2 + 2 ˜ = ( x 2 + y 2 )
Ëa b ¯ x2 y 2
+ = 1 is
54. Let the point be (c cosq, c sinq). a 2 b2
The equation of the tangent to the ellipse at (c cosq, c hx ky
sinq) is + =1
a 2 b2
x ◊ c cos q y ◊ c cos q
+ =1 …(i) It meets the axes in
a2 b2 Ê b2 ˆ
Ê a2 ˆ
Let the co-ordinates of the mid-point be (h, k). A Á , 0˜ and B ÁË 0, ˜¯
Ë h ¯ k
P
1
Now, area of the triangle OAB = ◊ OA ◊ OB
2
1 a 2 b2
O M(h, k) R = ◊ ◊
2 h k
a 2b 2
=
2◊ h ◊ k
l
= constant = (say)
2
\ The equation of the chord bisected at (h, k) is a 2b 2
Hence, the locus of (h, k) is x ◊ y =
hx ky h 2 k 2 l
+ = + …(ii) which represents a hyperbola.
a 2 b2 a 2 b2

CG_05.indd 37 2/6/2017 4:05:11 PM


5.38 Coordinate Geometry Booster

56. Let the point P be (a cos q, b sin q). 58. Let the point be (h, k).
\ The equation of normal at P to the ellipse is B
a ◊ x ◊ sec q – b ◊ y ◊ cosec q = a2 – b2 …(i)
C
90° M(h, k)

P
A

M(h, k) T The equation of the chord bisected at (h, k) to the el-


hx ky h 2 k 2
lipse is 2 + 2 = 2 + 2
N a b a b
Let its mid-point be (h, k). Ê Ê hx ky ˆ ˆ
\ The equation of the chord bisected at (h, k) is Á ÁË a 2 + b 2 ˜¯ ˜
fi Á 2 2
˜ =1
hx ky h 2 k 2 ÁÊh + k ˆ˜
+ = + …(ii) ÁË ÁË a 2 b 2 ˜¯ ˜¯
a 2 b2 a 2 b2
From Eqs (i) and (ii), we get Now we make it a homogenous equation of 2nd degree.
2 2 2
a sec q b cosec q a -b Ê Ê hx ky ˆ ˆ
= = 2 +
2
( h /a ) 2
( - k /b ) Ê h k2 ˆ Ê x 2 y 2 ˆ Á ÁË a 2 b 2 ˜¯ ˜
ÁË 2 + 2 ˜¯ Thus, Á 2 + 2 ˜ = Á 2 ˜
a b Ëa b ¯ ÁÊh k2 ˆ ˜
+
ÁË ÁË a 2 b 2 ˜¯ ˜¯
Ê h2 k 2 ˆ Ê h ˆ
fi cos q = Á 2 + 2 ˜ / Á 3 2 2 ˜
Ëa b ¯ Ë a (a - b ) ¯ 2 2
Ê x2 y 2 ˆ Ê h2 k 2 ˆ Ê hx ky ˆ
Ê h2 k 2 ˆ Ê
fi ÁË 2 + 2 ˜¯ ÁË 2 + 2 ˜¯ = ÁË 2 + 2 ˜¯
-k ˆ a b a b a b
and sin q = Á 2 + 2 ˜ / Á 3 2
Ëa b ¯ Ë b (a - b 2 ) ˜¯ h 2 x 2 k 2 y 2 2hkxy
= + 4 + 2 2
Squaring and adding, we get a4 b a b
2
Ê a 6 b6 ˆ Ê x 2 y 2 ˆ 2 2 2 Since the chord subtends right angle at the centre, so
Á 2 + 2 ˜ ÁË 2 + 2 ˜¯ = (a - b ) co-efficient of x2 + co-efficient of y2 = 0
Ëx y ¯ a b
2
57. Let the point be (h, k). a 2 x2 b 2 y 2 Ê x2 y 2 ˆ
+ 4 =Á 2 + 2˜
The equation of the chord bisected at (h, k) to the el- a4 b Ëa b ¯
x2 y 2 Ê h2 k 2 ˆ Ê 1
2
1 ˆ Ê h2 k 2 ˆ
lipse 2 + 2 = 1 is + +
a b fi ÁË 2 ˜ Á ˜ =Á + ˜
a b2 ¯ Ë a 2 b2 ¯ Ë a 4 b4 ¯
hx ky h 2 k 2
+ = + …(i) Hence, the locus of (h, k) is
a 2 b2 a 2 b2 2
Ê x2 y 2 ˆ Ê 1 1 ˆ Ê x2 y 2 ˆ
and the equation of the tangent to the auxiliary circle x2 Á 2 + 2 ˜ ÁË 2 + 2 ˜¯ = Á 4 + 4 ˜
+ y2 = a2 at (a cos qc a sin q) is Ëa b ¯ a b Ëa b ¯
x cos q + y sin q = a …(ii) 59. Let the point P be (a cos q, b sin q) and Q be
From Eqs (i) and (ii), we get
Ê Êp ˆ Êp ˆˆ
cos q sin q a ÁË a cos ÁË 2 + q˜¯ , b sin ÁË 2 + q˜¯ ˜¯ ,
2
= 2
= 2
h /a k /b Ê h k2 ˆ
ÁË 2 + 2 ˜¯ i.e. (– a sin qs b cos q).
a b
Squaring and adding, we get p
Q q+
2 2 P(q)
h2 k 2 ˆ Ê h2 k 2 ˆ
2Ê M(h, k)
a Á 4 + 4˜ =Á 2 + 2˜
Ëa b ¯ Ëa b ¯
Hence the locus of (h, k) is
2
x2 y 2 ˆ Ê x2 y 2 ˆ

a Á 4 + 4˜ =Á 2 + 2˜ Let M(h, k) be the mid-point of PQ.
Ëa b ¯ Ëa b ¯

CG_05.indd 38 2/6/2017 4:05:12 PM


Ellipse 5.39

a Hence, the locus of (a, b) is


Then, h = (cos q - sin q ) 2
2 Ê x2 + y 2 ˆ Ê x2 y 2 ˆ
Á 2 ˜ =Á + ˜
b
and k = (cos q + sin q ) Ë a + b2 ¯ Ë a 2 b2 ¯
2
61. Let the mid-point be (h, k).
2h 2k
fi = (cos q - sin q ) and = (cos q + sin q ) P
a b
Squaring and adding, we get
S(æ, 0)
Ê 4h 2 4k 2 ˆ
ÁË 2 + 2 ˜¯ = 2 M(h, k)
a b
Ê h2 k 2 ˆ 1
fi ÁË 2 + 2 ˜¯ = Q
a b 2
The equation of the chord of the ellipse whose mid-
Hence, the locus of (h, k) is
point (h, k) is
Ê x2 y 2 ˆ 1
ÁË 2 + 2 ˜¯ = hx ky h 2 k 2
a b 2 + = +
a 2 b2 a 2 b2
60. Equation of the chord of contact to the tangents at (h, k) which passes through the focus (ae, 0).
is hae h 2 k 2
hx ky Thus, 2 = 2 + 2
+ =1 …(i) a a b
a 2 b2
he b 2 h 2 + a 2 k 2
P(h, k) fi =
a a 2b 2
fi b h + a k = ab2he
2 2 2 2

Hence, the locus of (h, k) is


b2h2 + a2y2 = ab2xe
Q
R 62. Let the point be (h, k).
M(a ◊ b)
A M(h, k) B
P(q)
The equation of the chord of the ellipse whose mid-
point (a, b) is
ax b y a2 b2
+ = + …(ii)
a 2 b2 a 2 b2
The equation of the chord of the ellipse whose middle
Since Eqs (i) and (ii) are the same, therefore point (h, k) is
h k 1 hx ky h 2 k 2
= = 2 + = + …(i)
a b Êa b2 ˆ a 2 b2 a 2 b2
ÁË 2 + ˜
a b2 ¯ and the equation of the tangent to the ellipse
a x2 y 2
fi h= 2 + = 1 at (a cos q, b sin q) is
Êa b2 ˆ a2 b2
+
Á a 2 b2 ˜
Ë ¯ x y
cos q + sin q = 1
b a b
and k= 2 Since both the equations are identical, so
Êa b2 ˆ
ÁË 2 + 2 ˜¯ cos q /a sin q /b 1
a b = = 2
2 2
h /a k /b Êh k2 ˆ
Also, (h, k) lies on the director circle of the given ÁË 2 + 2 ˜¯
ellipse x2 + y2 = a2 + b2. a b
Squaring and adding, we get
Thus, h2 + k2 = a2 + b2 2
2 a 2h2 b 2k 2 Ê h2 k 2 ˆ
Ê a2 + b2 ˆ Ê a2 b2 ˆ + 4 =Á 2 + 2˜
fi a4 b Ëa b ¯
Á ˜ =Á + ˜
Ë a 2 + b2 ¯ Ë a 2 b2 ¯

CG_05.indd 39 2/6/2017 4:05:13 PM


5.40 Coordinate Geometry Booster

Hence the locus of (h, k) is 67. Let the pole be (h, k).
2 2 2 2
Êx 2 2ˆ2 The equation of polar w.r.t. the ellipse c2x2 + d2y2 = 1 is
a x b y y
+ 4 =Á 2 + 2˜ c2hx + d2ky = 1
a 4
b Ëa b ¯
fi d2ky = –c2 hx + 1
x2 y 2
63. Let the ellipse be + = 1 and its focus is (ae, 0). c2h 1
a 2 b2 fi y=- x+ 2
2
d k d k
The equation of the polar is x2 y 2
xx1 y ◊ y1 which is a tangent to the ellipse 2 + 2 = 1
+ 2 =1 a b
a2 b So, c2 = a2m2 + b2
x (ae) y (0) 2 2
fi + 2 =1 1 2Ê c hˆ
a2 fi = a Á - + b2
b
(d 2 k )2 Ë d 2 k ˜¯
a
fi x= fi a2c4h2 + b2d1k2 = 1
e
which is the directrix. Hence, the locus of (h, k) is
64. Let the pole be (x1, y1). a2c4x2 – b2d4y2 = 1
68. Let the pole be (h, k).
xx1 y ◊ y1
The equation of the polar is + 2 = 1 which is hx ky
a2 b The equation of the polar is + =1
identical with lx + my + n = 0 a 2 b2
0 + 0 -1
x1 /a 2 y1 /b 2 -1 It is given that =c
So, = = h2 k 2
l m n +
a 2l b2m a 4 b4
fi x1 = - , y1 = - h2 k 2 1
n n fi + =
a 4 b4 c2
Ê a 2l b 2 m ˆ
Hence, the pole is ÁË - ,- ˜ Hence, the locus of (h, k) is
n n ¯
x2 y 2 1
65. If (h, k) be the pole of a given line w.r.t. the ellipse, then + =
a 4 b4 c2
its equation is
69. Let the pole be (h, k).
hx ky
+ =1 …(i) hx ky
a 2 b2 The equation of the polar is 2 + 2 = 1 …(i)
a b
If tangents at its extremities meet at (a, b), then it is the
and the equation of the normal to the ellipse is
chord of contact of (a, b) and hence its equation is
ax by ax sec j – by cosec j = a2 – b2 …(ii)
+ =1 …(ii)
Comparing Eqs (i) and (ii), we get
a 2 b2
Comparing Eqs (i) and (ii), we get h /a 2 k /b 2 1
=- =
h /a 2 k /b 2 a sec j b cosec j a 2 - b 2
= =1
a /a 2 b /b 2 h cos j k sin j 1
fi 3
=- 3 = 2
fi h = a, k = b a b cosec j a - b 2
Thus, the pole (h, k) is the same as (a, b), i.e. the inter- a3 b3
section of tangents. fi cos j = , sin j = -
h( a 2 - b 2 ) k (a 2 - b 2 )
66. Let the pole be (h, k).
The equation of polar w.r.t. the ellipse x2 + 4y2 = 4 is Eliminating f, we get
hx + 4ky = 4 2 2
Ê a3 ˆ Ê b3 ˆ
which is identical with x + 4y = 4 Á 2 2 ˜
+ Á- 2 2 ˜
=1
Ë h( a - b ) ¯ Ë k (a - b ) ¯
h 4k 4
So, = =
1 4 4 a 6 b6
fi 2
+ 2 = (a 2 - b 2 )2
h k h k
fi = =1
1 1 a 6 b6
Hence, the pole is (1, 1) Hence, the locus of (h, k) is 2
+ 2 = (a 2 - b 2 )2
x y

CG_05.indd 40 2/6/2017 4:05:14 PM


Ellipse 5.41

70. Let the pole be (h, k) Q(–a cos j, b sin j),


The equation of the polar w.r.t. the parabola is S(–a sin j, b cos j)
yk = 2a(x + h) and R(a sin j, –b cos j)
fi yk = 2ax + 2ah 73.
2a 2ah D P
fi y= x+
k k
x2 y 2
which is a tangent to the ellipse 2 + 2 = 1 C
a b
2 2
Ê 2ah ˆ Ê 2 a ˆ
So, Á = a2 Á ˜ + b2
Ë k ˜¯ Ë k ¯
Let CP and CD be two conjugate semi-diameters of an
fi 4a2h2 = 4a2a2 + k2b 2
x2 y 2
Hence, the locus of pole (h, k) is ellipse 2 + 2 = 1 and let the eccentric angle of P is
a b p
4a2x2 = y2b 2 + 4a2a2 f. Thus the eccentric angle of D is + j .
2
71. Therefore the co-ordinates of P and D are (a cos j, b
B sin j) and
Ê Êp ˆ Êp ˆˆ
a cos Á + j ˜ , b sin Á + j ˜ ˜
k) ËÁ Ë2 ¯ Ë2 ¯¯
h,
A¢ M( A i.e. (–a sin j, b cos j)
Thus CP2 + CD2
y = mx + c = (a2 cos2 j + b2 sin2 j) + (a2 sin2 j + b2 cos2 j)
B¢ = a2 + b2
Let (h, k) be the mid-point of the chord y = mx + c of the 74.
x2 y 2 D P
ellipse 2 + 2 = 1 .
a b
Then T = S1
S¢(–a e, 0) C S(–a e, 0)
xh yk h 2 k 2
fi + = +
a 2 b2 a 2 b2
b2h
fi k=- Let CP and CD be the conjugate diameters of the
a2m ellipse.
b2 x
Hence, the locus of the mid-point is y = - Let P = (a cos j, b sin j). then the co-ordinates of D is
a2m (–a sin j, b cos j).
72. Two diameters are said to be conjugate when each bi-
Thus,
sects all chords parallel to the other.
SP ◊ S¢P = (a – ae cos j)(a + ae cos j)
If y = m1x and y = m2x be two conjugate diameters of an
= a2 – a2e2 cos2 j
b2
ellipse, then m1m2 = - 2 . = a2 – (a2 – b2) cos2 j
a
= a2 sin2 + b2 cos2 j
S P = CD2
75 Let the eccentric angle of P is (f) and the eccentric
Ê pˆ
C angle of M is ÁË j + ˜¯ .
2
R Then the co-ordinates of P and M are (a cos j, b sin j)
Q
and
Let PQ and RS be two conjugate diameters. Then the Ê pˆ p ˆˆ
Ê Ê
co-ordinates of the four extremities of two conjugate ÁË a cos ËÁ j + 2 ¯˜ , b sin ËÁ j + 2 ¯˜ ¯˜
diameters are
P(a cos j, b sin j), i.e. (a cos j, b sin j) and (–a sin j, b cos j)

CG_05.indd 41 2/6/2017 4:05:15 PM


5.42 Coordinate Geometry Booster

The equation of the tangent at P is


b3
x y fi m2 = -
cos j + sin j = 1 …(i) a3
a b b3
and the equation of the tangent at M is Hence, the required diameters is y = - x
a3
x y fi a3y + b3x = 0
sin j + cos j = 1 …(ii)
—a b 78. The given ellipses are
Squaring and adding Eqs (i) and (ii), we get
x2 y 2
x2
y 2 + =1 …(i)
+ =2 a 2 b2
a 2 b2
x2 y 2 x2 y 2
76. Let the equation of the ellipse be + = 1. and + =1 …(ii)
a 2 b2 c2 d 2
B The equation of lines passing through the point of in-
D P
tersection of the ellipses are
M(h, k)
Ê 1 1ˆ Ê 1 1ˆ
x2 Á 2 - 2 ˜ + y 2 Á 2 - 2 ˜ = 0 …(iii)
A A Ëa c ¯ Ë b d ¯
C
which represents a pair of lines through the origin.
Q R If y = mx be one of the lines, then y = mx must satisfy
B¢ (iii), then
Let the eccentric angle of P is (f) and the eccentric Ê 1 1ˆ 2Ê 1 1ˆ
pˆ ÁË 2 - 2 ˜¯ + m ÁË 2 - 2 ˜¯ = 0
Ê a c b d
angle of D is ÁË j + ˜¯ .
2 Ê 1 1ˆ 2 Ê 1 1ˆ
Then the co-ordinates of P and D are (a cos j, b sin j)
fi ÁË 2 - 2 ˜¯ m + ÁË 2 - 2 ˜¯ = 0
b d a c
and
which is a quadratic in m. Let it has roots m1 and m2.
Ê Ê pˆ Ê p ˆˆ
ÁË a cos ÁË j + 2 ˜¯ , b sin ÁË j + 2 ˜¯ ˜¯ Ê 1 1ˆ
ÁË 2 - 2 ˜¯
a c
i.e. (a cos j, b sin j) and (–a sin j, b cos j) Then, m1m2 = -
Ê 1 1ˆ
Let M(h, k) be the mid-point of PD. ÁË 2 - 2 ˜¯
b d
Then
Ê 1 1ˆ
a cos j - a sin j -
h= b 2 ÁË a 2 c 2 ˜¯
2 fi - 2 =
a Ê 1 1ˆ
a cos j + b sin j (2h) 2 (2k ) 2 ÁË 2 - 2 ˜¯
and k= fi 2 + 2 b d
2 a b Ê 1 1ˆ Ê 1 1ˆ
= (cos j – sin j)2 + (cos j + sin j)2 fi a 2 Á 2 - 2 ˜ = - b2 Á 2 - 2 ˜
Ëa c ¯ Ëb d ¯
h2 k 2 1
fi + = Ê a2 ˆ Ê b2 ˆ
a 2 b2 2 fi ÁË1 - 2 ˜¯ = ÁË -1 + 2 ˜¯
x2 y 2 1 c d
Hence, the locus of (h, k) is + =
a 2 b2 2 a 2 b2
fi + =2
77. The equation of the given diameter is c2 d 2
ax – by = 0 Hence, the result.
a 79. The co-ordinates of P and D are (a cos j, b sin j) and
fi y= x …(i)
b (–a sin j, b cos j)
a Let PM is the normal at P and DN is the normal at D.
Thus, m1 =
b B
D P
Let the diameter conjugate to (i) be y = m2x
As we know that, two diameters y = m1x and y = m2x H
M N
b2 A¢ A
are conjugate, if m1 ◊ m2 = - 2 C
a
a b2
fi ¥ m2 = - 2
b a

CG_05.indd 42 2/6/2017 4:05:16 PM


Ellipse 5.43

The equations of the normals at P and D are Area of the parallelogram = MNM¢N¢
ax sec j – by cosec j = a2 – b2 = 4 (the area of the parallelogram CPMR)
and –ax cosec j – by sec j = a2 – b2 ab
= 4 ¥ a 2cos 2j + b 2sin 2j ¥
respectively. a cos j + b 2sin 2j
2 2
Since, H(a, b) is the point of intersection of the nor- = 4ab
mals, so = constant
aa sec j – bb cosec j – (a2 – b2) = 0 …(i) Hence, the result.
Êp ˆ
and 81. Let the eccentric angles at P and Q be j and Á + j ˜
Ë2 ¯
aa cosec j + bb sec j + (a2 – b2) = 0 …(ii) respectively
Eliminating f from Eqs (i) and (ii), we get The equation of the tangents at P and Q
x y
sec j cosec j (a 2 - b 2 ) are cos j + sin j = 1
= = 2 2 a b
(aa - bb ) -(aa + bb ) (a a + b 2 b 2 )
x y
and - sin j + cos j = 1
( a 2a 2 + b 2 b 2 ) a b
fi cos j =
(a - b 2 )(aa - bb )
2
respectively
Squaring and adding, we get
(a 2a2 + b 2 b 2 )
and sin j = x2 y 2
- (a 2 - b 2 )(aa + b b) + =2
a 2 b2
Squaring and adding, we get
Hence, the result.
2(a2a2 + b2b2)3 = (a2 – b2)(a2a2 – b2b2)2 82. Thus, the equation of the ellipse is
Hence, the locus of H(a, b) is 2
y 2 + x2 = 1
3
2(a2x2 + b2y2)3 = (a2 – b2)(a2x2 – b2y2)2. 2
x y2
80. fi + =1
Y 3 2
Hence, the eccentricity is
R P
b2 2 1
e = 1- = 1- =
X¢ X a2 3 3
C
83. The given equation of an ellipse is
Q S
16x2 + 25y2 = 400

x2 y 2
Let PCQ and RCS be two conjugate diameters of the fi + =1
25 16
x2 y 2 From the reflection property of an ellipse, the reflection
ellipse 2 + 2 = 1 .
a b ray passes through the focus.
Then the co-ordinates of P, Q, R, and S are Thus, the co-ordinates of the other focus = (3, 0).
P(a cos j, b sin j), Q(–a cos j, –b sin j), When y = 4, then x = 0.
R(–a sin j, b cos j) and S(a sin j, –b cos j) respec-
So the point is (0, 4).
tively.
The equations of tangents at P, R, Q and S are So the equation of the reflection ray is
x y
x y + =1
cos j + sin j = 1, 3 4
a b
fi 4x + 3y = 12
x y
- sin j + cos j = 1, 84. The equation of the incident ray is
a b x–y+2=0
x y x y
- cos j - sin j = 1, fi + =1
a b -2 2
x y The equation of the ellipse is
and sin j - cos j = 1
a b 3x2 + 4y2 = 12
Thus, the tangents at P and Q are parallel. Also the tan- x2 y 2
gents at R and S are are parallel. Hence, the tangents at fi + =1
4 3
P, R, Q, S form a parallelogram.

CG_05.indd 43 2/6/2017 4:05:17 PM


5.44 Coordinate Geometry Booster

Clearly, the co-ordinates of the foci are (–2, 0) and


1 cos 2q sin 2q
(2, 0). fi = + 2
Since the incident ray x – y + 2 = 0 intersects the ellipse d2 a2 b
at (0, 2), so, the equation of the reflection ray = the Now,
equation of the line joining (0, 2) and (2, 0) Ê b2 ˆ
x y 4a 2 Á1 - 2 ˜
fi + =1 Ë d ¯
2 2
fi x+y=2 4a 2b 2
= 4a 2 -
d2
Ê cos 2q sin 2q ˆ
LEVEL III = 4a 2 - 4a 2b 2 Á 2 + 2 ˜
Ë a b ¯
x2 y 2 = 4a2 – 4b2 cos2q – 4a2sin2q
1. Given ellipse is + =1
a 2 b2 = 4a2(1 – sin2q) – 4b2cos2q
So, F1 = (ac, 0) and F2 = (–ac, 0) = 4a2cos2q – 4b2cos2q
Let P be (a cos q, b sin q). = 4 cos2q(a2 – b2)
Then = 4 cos2q(a2e2)
a cos q b sin q 1 = (2ae cos q)2
1
ar(DPF1F2) = ae 0 1 = [(a + ae cos q) – (a – ae cos q)]2
2
- ae 0 1 = (PF1 – PF2)2
1 3. Given ellipse is
=¥ 2ae ¥ b sin q
2 x2 y 2
= abe ¥ sin q + =1
16 9
Maximum value of A = abe
Ê 7 ˆ
b2 \ Foci = (± ae, 0) = Á ± 4 ◊ , 0˜ = (± 7, 0)
= ab 1 - 2 Ë 4 ¯
a
and the radius of the circle = 7 + 9 = 16 = 4
= b a 2 - b2 4. The equation of the tangent to the parabola y2 = 4x at
2. Given ellipse is (t2, 2t) is
yt = x + t2
x2 y 2
+ =1 fi x – yt + t2 = 0 …(i)
a 2 b2 The equation of the normal
Y 4x2 + 5y2 = 20
x2 y 2
i.e. + = 1 at ( 5 cos j , 2 sin j )
M 5 4
P
a 2 x b2 y
is - = a 2 - b2
X¢ X x1 y1
F2(a e, 0) F1(a e, 0)
5x 4y
fi - = 5 — 4 =1
5 cos j 2 sin j
Y¢ fi ( 5 sec j ) x - (2 cosec j ) y = 1 …(ii)
The equation of the tangent to the given ellipse at Since Eqs (i) and (ii) are the same line, so
P(a cos q, b sin q) is
x y 5 sec j 2 cosec j —1
cos q + sin q = 1 = = 2
a b 1 t t
1 1
Now, OM = d fi sec j = - 2
, cosec j = -
5t 2t
0 + 0 -1
fi =d
cos 2q sin 2q fi cos j = - t 2 5, sin j = - 2t
+ 2
a2 b

CG_05.indd 44 2/6/2017 4:05:18 PM


Ellipse 5.45

Thus, The equation of the chord of contact of the tangents


2 2
5t + 4t – 1 = 0 through A is
fi 5t4 + 5t2 – t2 – 1 = 0 hx ky
+ =1 …(iv)
fi 5t2 (t2 + 1) – 1(t2 + 1) = 0 6 3
fi (5t2 – 1) (t2 + 1) = 0 Since the eqs (ii) and (iv) are identical, so
fi (5t2 – 1) = 0 h k
1 6 = 3 =1
fi t=± cos q sin q
5
2
Also, tan j = ±2
fi h = 3 cos q, k = 3 sin q
fi j = tan–1 (±2)
5. We have B = (0, b), F = (ae, 0) and F¢ = (–ae, 0) Now, squaring and adding, we get
b b h2 = k2 = 9
Now, m( FB ) = - and m( BF ¢ ) =
ae ae Therefore, the locus of A is
It is given that, x2 + y2 = 9
m(FB) ¥ m(BF ¢) = –1 x2 y 2
b b which is the director circle of + =1
fi - ¥ = -1 6 3
ae ae
Thus, the angle between the tangents at P and Q of the
fi b2 = a2e2 ellipse x2 + 2y2 = 6 is right angle.
fi a2(1 – e2) = a2e2
7. Y
fi (1 – e2) = e2
fi 2e2 = 1 N
1 P
fi e=
2 X¢ X
6. O
L
Y
A(h, k)

Q
x2 y 2
P Given ellipse is + =1
X¢ X a 2 b2
O
Let P lies in the first quadrant, so
P = (a cos q, b sin q)
The equation of the tangent at P is

x y
cos q + sin q = 1
Given ellipse is a b
x2 + 4y2 = 4
-1
x2 y 2 Now, ON =
fi + =1 …(i) cos q sin 2q
2
4 1 + 2
a2 b
The equation of the tangent to the ellipse (i) is ab
x =
cos q + y sin q = 1 …(ii) a 2sin 2q + b 2cos 2q
2
The equation of ON is
and the equation of the 2nd ellipse can be written as
x y
x2 y 2 sin q - cos q = 0
+ =1 (iii) b a
6 3
and the equation of the normal at P is
Let the tangents at P and Q meet at A(h, k).
ax sec q – by cosec q = a2 – b2
So PQ is the chord of contact.

CG_05.indd 45 2/6/2017 4:05:19 PM


5.46 Coordinate Geometry Booster

Let q be the angle between the tangents


a 2 - b2
So, OL = m - m1
a 2sec 2q + b 2cosec 2q Then tan (q ) = 2
1 + m1m2
(a 2 - b 2 )sin q cos q (m2 + m1 ) 2 - 4m1m2
=
2 2
a sin q + b cos q 2 2 =
1 + m1m2
Now, NP = OL 64 56
+
2 2
(a - b )sin q cos q = 9 9 = 2 30 ¥ 9 = 6 6
fi NP = 14 3 5 5
2 2
a sin q + b cos q 2 2 1—
9
Therefore, Ê 6 6ˆ
ar DOPN Thus, q = tan -1 Á
Ë 5 ˜¯
1
= ¥ ON ¥ NP 9. Normal at P(a cos q, b sin q) is
2
ax sec q – by cosec q = a2 – b2
1 ab(a 2 - b 2 )sin q cos q fi ax sec q – by cosec q = 14 – 5 = 9
= ¥
2 a 2sin 2q + b 2cos 2q which meets the ellipse again at Q. So
1 ab(a 2 - b 2 ) a2 cos 2q sec q – b2 sin 2q cosec q = 9
= ¥ 2
2 a tan q + b 2cot q fi 14 cos 2q sec q – 5 sin 2q cosec q = 9
1 ab(a 2 - b 2 ) (a 2 - b 2 ) fi 28 cos2 q – 14 – 10 cos2 q = 9 cos q
£ ¥ = fi 18 cos2 q – 9 cos q – 14 = 0
2 2ab 4
b fi (6 cos q – 7)(3 cos q + 2) = 0
at tan q = . 2 7
a fi cos q = - ,
3 6
Thus, the point P is
2
Ê a2 b2 ˆ Thus, cos q = -
Á 2 , ˜ 3
2
Ë a +b a 2 + b2 ¯
x2 y 2
10. Given ellipse is + =1 …(i)
By symmetry, we have four such points. 25 16
Ê a2 b2 ˆ The equation of the chord bisected at (h, k) is
Thus, Á ± ,±
2 2 2 2˜ T = S1
Ë a +b a +b ¯
x2 y2 hx ky h 2 k 2
+ =1 fi + = +
8. Given ellipse is
(5/3) (5/2) a 2 b2 a 2 b2
hx ky h 2 k 2
The equation of any tangent to the given ellipse is fi + = +
25 16 25 16
y = mx + b 2 m 2 + a 2
(1/5) x (2/5) y (1/5) 2 (2/5) 2
5 2 5 fi + = +
fi y = mx + m + 25 16 25 16
2 3 Solving, we get
which is passing through (1, 2). 4x + 5y = 4 …(ii)
5 2 5 Solving Eqs (i) and (ii), we get
So, 2 = m + m +
2 3 12 16
x = 4, - 3 and y = - ,
Ê5 5ˆ 5 5
fi (2 - m) 2 = Á m 2 + ˜
Ë2 3¯ Let the chord be AB, where
fi 6m – 24m + 24 = 15m2 + 10
2
Ê 12 ˆ Ê 16 ˆ
fi 9m2 + 24m – 14 = 0 A = Á 4, - ˜ and B = Á - 3, ˜
Ë 5 ¯ Ë 5¯
Let its roots are m1 and m2
Hence, the length the AB is
8 14
So, m1 + m2 = - , m1m2 = - Ê 28 ˆ
2
16 7 41
3 9 = (7) 2 + Á ˜ = 7 1 + =
Ë 5¯ 25 5

CG_05.indd 46 2/6/2017 4:05:20 PM


Ellipse 5.47

11. Any tangent to the ellipse is Ê b2 ˆ Ê 5 ˆ


L = Á ae, ˜ = Á 2, ˜
x y Ë a ¯ Ë 3¯
cos q + sin q = 1 …(i)
a b
Ê b2 ˆ Ê 5ˆ
Its point of contact is P(a cos qc – b sin q) and its slope L ¢ = Á ae, - ˜ = Á 2, - ˜
Ë a¯ Ë 3¯
b
is - cot q . Also the focus is S(ae, 0).
a Ê b2 ˆ Ê 5ˆ
N = Á — ae, ˜ = Á — 2, ˜ and
Any line through the focus S and perpendicular to tan- Ë a¯ Ë 3¯
gent (i) is
a Ê b2 ˆ Ê 5ˆ
y-0= tan q ( x - ae) …(ii) N ¢ = Á — ae, - ˜ = Á — 2, - ˜
Ë a¯ Ë 3¯
b
Also the equation of CP is Now, the tangent at L is
a xx1 yy1
y-0= tan q ( x - 0) …(iii) + =1
b 9 5
Eliminating q between Eqs (ii) and (iii), we get 5
y
2x 3
Ê a 2 ˆ Ê x - ae ˆ fi + =1
ÁË 2 ˜¯ ÁË ˜ =1 9 5
b x ¯
x y
fi + =1
Ê x - ae ˆ Ê b
2ˆ 9/2 3
fi ÁË ˜ =Á ˜
x ¯ Ë a2 ¯ Ê9 ˆ
Thus, P = Á , 0˜ and S = (0, 3)
Ë2 ¯
Ê ae ˆ Ê b 2 ˆ
fi ÁË1 - ˜¯ = Á 2 ˜ Therefore,
x Ëa ¯ ar(quad PQRS) = 4 ¥ ar(DOPS)
1 9
Ê b 2 ˆ Ê ae ˆ = 4¥ ¥ ¥3
fi Á 1 - 2 ˜ = ÁË x ˜¯ 2 2
Ë a ¯
= 27 s.u.
Ê a 2 (1 - e 2 ) ˆ Ê ae ˆ 13. The equation of the tangent at (3 3 cos q , sin q ) to the
fi ÁË 1 - =
a2 ¯˜ ËÁ x ¯˜ given ellipse is
x ◊ 3 3 cos q
Ê ae ˆ + y ◊ sin q = 1
fi e2 = Á ˜ 27
Ë x¯
x y
a fi + =1
fi x= 3 3 sec q (cosec q )
e
Hence, the result. It is given that
12. S = 3 3 sec q + cosec q
Y
dS
fi = 3 3 sec q tan q - cosec q cot q
N
S
L dq
dS
For maximum or minimum, =0
X¢ X dq
R O P gives

N¢ L¢ fi 3 3 sec q tan q - cosec q cot q = 0


Q
fi 3 3 sec q tan q = cosec q cot q

cosec q cot q
x2 y 2 fi =3 3
Given ellipse is + =1 sec q tan q
9 5
fi cot 3q = 3 3
b2 5 2
Now, e = 1 - 2 = 1 - =
a 9 3 fi cot q = 3

CG_05.indd 47 2/6/2017 4:05:21 PM


5.48 Coordinate Geometry Booster

p 25m 2 + 4
fi q=
6 =4
p m2 + 1
Hence, the value of q is .
6 fi 25m2 + 4 = 16(m2 + 1)
14. Let the mid-point be (h, k). fi 9m2 = 12
The equation of the ellipse is x2 + 2y2 = 2.
4
x2 fi m2 =
+ y2 = 1 3
2 2
and the equation of the tangent to the ellipse at fi m=±
3
( 2 cos q , sin q ) is
Hence, the equation of the common tangent is
x ◊ 2 cos q
+ y ◊ sin q = 1. 2 100
2 y=- x+ +4
3 3
x y
fi + = 1. 2 112
2 sec q cosec q y=- x+
3 3
Let A = ( 2 sec q , 0) and B = (0, cosec q )
Ê 112 ˆ
Let A = (2 7, 0) and B = Á 0, ˜
2 sec q cosec q Ë 3 ¯
Thus, h = ,k =
2 2 Thus, the length of the tangent
1 1
fi cos q = and sin q = 112 196 14
2h 2 k = 28 + = =
3 3 3
Squaring and adding, we get
1 1 17. Let (h, k) be the point of intersection of tangents at q
2
+ 2 =1 and j. Then
2h 4k
Êq + jˆ Êq + jˆ
cos Á ˜ sin Á
Ë 2 ˜¯
Hence, the locus of (h, k) is
h Ë 2 ¯ k
1 1 = and =
+ 2 =1 a Êq - jˆ b Êq - jˆ
2 cos Á cos Á
2x 4y Ë 2 ˜¯ Ë 2 ˜¯
15. The equation of any tangent to the given ellipse at (a
cos q, b sin q) is Squaring and adding, we get
x y h2 k 2 1
cos q + sin q = 1 + = …(i)
a b a 2 b 2 cos 2 Ê q - j ˆ
ÁË ˜
x y 2 ¯
fi + =1
a sec q b cosec q It is given that,
b(sin q + sin j) = 3
Let A = (a sec q, 0) and B = (0, b cosec q)
1 fi (sin q + sin j) = 1 ( b = 3)
D = ar ( DOAB) =
2 ab sin q cosec q Êq + jˆ Êq - jˆ
fi 2 sin Á cos Á =1
1 Ë 2 ˜¯ Ë 2 ˜¯
=
ab sin 2q Ê q + jˆ
sin Á
1 k Ë 2 ˜¯ 1
Hence, the minimum area is sq. unit. Now, = = …(ii)
ab b Ê q - jˆ Ê q - jˆ
cos Á 2 cos 2 Á
Ë 2 ˜¯ Ë 2 ˜¯
16. The equation of any tangent to the ellipse is
From Eqs (i) and (ii), we get
y = mx + a 2 m 2 + b 2
h 2 k 2 2k
2 + =
fi y = mx + 25m + 4 a 2 b2 b
2 2
fi mx — y + 25m 2 + 4 = 0 h k 2k
fi + =
25 9 3
which is also a tangent to the given circle. So, the
length of the perpendicular from the centre to the tan- Hence, the locus of (h, k) is
gent is equal to the radius of a circle x 2 y 2 2y
+ =
25 9 3

CG_05.indd 48 2/6/2017 4:05:22 PM


Ellipse 5.49

20. Any point on the ellipse be (a cos q + a sin q).


x2 y 2
18. Given ellipse is + =1 Clearly, the centre is (0, 0)
25 16
Now, distance, r = a 2cos 2q + a 2sin 2q = a
Y
Also,
P a2 cos2 q + 2a2 sin q cos q + 2a2 sin2 q = 1
1
X¢ S O S¢
X fi a2 =
Q(h, k) 1 + sin 2q + sin 2q
1
fi a=
Y¢ 1 + sin 2q + sin 2q
1
Let the point Q be (h, k). Therefore, r = a =
Clearly, k is negative. 1 + sin 2q + sin 2q
It is given that, 1
fi r=
SP = NQ 3 1
+ sin 2q — cos 2q
fi –k = a + eh 2 2
3 r will be maximum when Dr will provide us minimum
fi -k = 5 + h
5 value
Hence, the locus of Q is Hence, the maximum value of r
3 2
-y=5+ x =
5 3- 5
fi 3x + 5y + 25 = 0
21. Let M(h, k) the mid-point of the chord PQ.
19. The Equation of the tangent to the ellipse is
Since the length of PQ is 2c, so P and Q can be consid-
y = mx + a 2 m 2 + b 2 ered as (h + c cos q, k + c sin q) and (h – c cos q, k – c
sin q) respectively.
= 2x + 4a 2 + b 2
(h + c cos q ) 2 (k + c sin q ) 2
which is a normal to the given circle Thus, + =1
a2 b2
Clearly, it will pass through the centre of a circle, so
0 = — 4 + 4a 2 + b 2 (h - c cos q ) 2 (k - c sin q ) 2
and + =1
a2 b2
fi 4a 2 + b 2 = 4
Adding, we get
fi 4a2 + b2 = 16
b2h2 + a2k2 – a2b2 + c2(a2sin2q + a2cos2q) = 0
Let P = ab = a 16 - 4a 2 …(i)
fi P2 = a2(16 – 4a2) and subtracting, we get
fi Q = 16a2 – 4a4 4ch 4ch
cos q - 2 sin q = 0
dQ a2 b
fi = 32a - 16a 3
da sin q cos q 1
= = …(ii)
d 2Q b2h -a 2k h b + k 2a4
2 4
fi = 32 - 48a 2
da 2 From Eqs (i) and (ii), we get
For maximum or minimum,
b2h2 + a2k2 – a2b2
dQ
=0 Ê b4h2a 2 a 4 k 2b 2 ˆ
da + c2 Á 4 2 + ˜ =0
fi 32a – 16a3 = 0 Ë b h + k 2a 4 b4h2 + k 2a 4 ¯
fi 16a(a2 – 2) = 0 Hence, the locus of M(h, k) is
fi a = 0, ± 2 Ê b2 x2 + a 2 y 2 ˆ 1 Ê x2 y 2 ˆ
Á 4 2 ˜ = Á 1 - - ˜
when a = ± 2, then b = ± 2 2 Ë a y + b4 x2 ¯ c2 Ë a 2 b2 ¯
Hence, the maximum value of ab is 4.

CG_05.indd 49 2/6/2017 4:05:23 PM


5.50 Coordinate Geometry Booster

Putting the values of sin a and cosa in Eq. (i), we get


x2 y 2
22. Let the ellipse be + = 1 and its pole be (h, k)
a 2 b2 x( x + y ) + y ( y - x) = a 2 ( y + x) 2 + b 2 ( y - x) 2
The equation of polar w.r.t. the given ellipse is
fi ( x 2 + y 2 ) = a 2 ( y + x) 2 + b 2 ( y - x) 2
hx ky
+ =1
a 2 b2
fi ( x 2 + y 2 ) = (a 2 + b 2 )( x 2 + y 2 ) + 2xy (a 2 - b 2 )
ky hx
fi = - 2 +1
b 2
a fi (x2 + y2)2 = (a2 + b2)(x2 + y2) + 2xy(a2 – b2)
b2h b2 fi (x2 + y2)(x2 + y2 – a2 – b2) = 2xy(a2 – b2)
fi y=- x + …(i)
a2k k
which is the required locus.
which is a tangent to the parabola
ay2 = –2b2x x2 y 2
24. Let the ellipse be + = 1 and the point P be (h, k).
Ê 2b 2 ˆ a 2 b2
fi y2 = Á - x
Ë a ˜¯ The equation of any tangent to the ellipse be

Ê b2 ˆ y = mx + a 2 m 2 + b 2
fi y2 = 4 Á - ˜ x …(ii)
Ë 2a ¯
which is passing through P.
Since (i) is tangent to (ii), so
Ê b2 ˆ So, k = mh + a 2 m 2 + b 2
b 2 ÁË - ˜¯
= 2a fi (k – mh)2 = (a2m2 + b2)
k Ê b2h ˆ fi k2 – 2hkm + m2h2 = (a2m2 + b2)
ÁË - 2 ˜¯
a k fi (h2 – a2)m2 – 2hkm + (k2 – b2) = 0
2
b ak
fi = It has two roots, say m1 and m2.
k 2h
2hk
2 2b 2 Thus, m1 + m2 =
fi k = h (h - a 2 )
2
a
Hence, the locus of (h, k) is k 2 - b2
and m1m2 =
Ê 2b 2 ˆ h2 - a 2
y2 = Á x
Ë a ˜¯
It is given that, q1 + q2 = 2a
which represents a parabola.
fi tan (q1 + q2) = tan (2a)
Ê x2 y 2 ˆ
23. Let the equation of the ellipse be Á 2 + 2 ˜ = 1
Ëa b ¯ tan q1 + tan q 2
fi = tan (2a )
The equation of a tangent to the given ellipse is 1 - tan q1 ◊ tan q 2

x cos a + y sin a = a 2cos 2a + b 2sin 2a …(i) m1 + m2


fi = tan (2a )
1 - m1m2
After rotation, the equation of the tangent is
x cos (a + 90°) + y sin (a + 90°) 2hk
fi = tan (2a )
h - a - k 2 + b2
2 2
= a 2sin 2a + b 2cos 2a
2hk
fi - x sin a + y cos a = a 2sin 2a + b 2cos 2a fi = tan (2a )
(h 2 - k 2 ) + (b 2 - a 2 )
…(ii)
On subtraction, we get Hence, the locus of P(h, k) is
(x + y) sin a + (x – y) cos a = 0 2xy
= tan (2a )
sin a cos a 1 ( x 2 - y 2 ) + (b 2 - a 2 )
fi = =
( y - x) ( x + y ) 2( x + y 2 )
2
fi {(x2 – y2) + (b2 – a2)}tan (2a) = 2xy

CG_05.indd 50 2/6/2017 4:05:24 PM


Ellipse 5.51

x2 y 2 h - 2 k -1 2( - 4)
25. Given ellipse is + =1 So, = =-
16 9 1 1 2
Y h - 2 k -1
fi = =4
B 1 1
Thus, Q = (6, 3)
X As we know that,
X¢ O
C A SP + S ¢P = 2a
fi SP + QP = 2a
D Thus, S, P and Q are collinear.
Y¢ So, S¢Q : 4x – 5y = 9
Clearly, the vertices of the square lie on the director Therefore P is a point of intersection of
x2 y 2 S¢Q : 4x – 5y = 9 and L : x + y = 5
circle of the given ellipse + = 1.
16 9 Ê 34 11ˆ
Hence, the point P is Á , ˜ .
Ë 9 9¯
The equation of the director circle is
28. Clearly, the centre of the ellipse is (2, 2).
x2 + y2 = 16 + 9 = 25
Y
Thus, the length of AC is 10 which is diagonal of the
square.
S(3, 3)
Thus, a 2 = 10
fi a=5 2 C(2, 2)
Hence, the length of the side of the square is 5 2 . S(1, 1)
26. Let PQ be the double ordinate, where X
O
P = (3 cos q, 2 sin q) and Q = (3 cos q, –2 sin q)
Let the point R(h, k) divides the double ordinate in the Since x-axis and y-axis are two perpendicular tangents
ratio 2 : 1 to the ellipse, so (0, 0) lies on the director circle and
2 (2, 2) is the centre of the director circle.
Thus h = 3 cos q and k = sin q
3 2 2
Thus, the radius = 2 + 2 = 8 = 2 2
Squaring and adding, we get
Hence, the area of the director circle = 8p
2 2
Ê hˆ Ê 3k ˆ x2 y 2
ÁË ˜¯ + ÁË ˜¯ = 1 29. The co-ordinates of any point on the ellipse 2 + 2 = 1
3 2 be (a cos q, b sin q). a b
h 2 9k 2 If it lies on the line bx = ay, we have
fi + =1
9 4 ba cos q = ab sin q
fi tan q = 1
Hence, the locus of (h, k) is p 5p
fi q = or
x 2 9y 2 4 4
+ =1 x2 y 2
9 4 30. Let the equation of the ellipse be 2 + 2 = 1.
a b
27. Let S = (2, –1) and S ¢ =(1, –1) and Q is the image of S
w.r.t. x + y = 5. 1 cos 2q sin 2q
Its polar form is = + 2
Q r2 a2 b
Let r1 and r2 be the lengths of the radius vectors CP and
P
p
CQ which are inclined at angles q and + q.
A L=0 2
S¢ 1 cos 2q sin 2q
So, = + 2
S r12 a2 b

1 cos 2q sin 2q
and = + 2
r22 a2 b

CG_05.indd 51 2/6/2017 4:05:25 PM


5.52 Coordinate Geometry Booster

Now, On solving the equations AB: x + 2y = 8 and AD: y = 2x


2 2 2 2 + 14, and AB and DC, we get
1 1 cos q sin q sin q cos q
+ = + 2 + 2 + 2 A = (–4, 6) and D = (–5, 4)
r12 r22 a2 b a b
When c = –6, we get,
1 1
= 2 + 2 A = (3, 2) and D = (3, 0).
a b
31. The given ellipse is
4(x – 2y + 1)2 + 9(2x + y + 2)2 = 25 …(i) LEVEL IV
Let X = x – 2y + 1, Y = 2x + y + 2
1. Given ellipse is
Equation (i) reduces to
Y
4X2 + 9Y2 = 25
X2 Y2 P
fi + =1
25/4 25/9
X¢ X
b2 O S(ae, 0) A
Thus, e = 1 -
a2 Q
25/9 4 5
= 1- = 1- =
25/4 9 3 Y¢
32. Let AD: y = 2x + c We have A = ar(DPQA)
So, BC: y = 2x + 4, AB: x + 2y = 8 1
= (2b sin q )(a - a cos q )
and DC: x + 2y – 3 = 0 2
y = 2x + c Ê 1 ˆ
A D = ab Á sin q - sin 2q ˜
Ë 2 ¯
dA
= ab(cos q - cos 2q )
dq
O d2A
fi = ab(— sin q + 2 sin 2q )
dq 2
For maximum or minimum
B(0, 4) C(–1, 2) dA
=0
Let BC = 2a and AB = 2b. dq
gives
5 ab(cos q – cos 2q) = 0
Clearly, 2a = 5 fi a =
2 fi (cos q – cos 2q) = 0
It is given that, fi cos 2q = cos q
5 fi cos 2q = cos (2p – q)
p ab = p
2 fi 2q = 2p – q
5 fi 3q = 2p
fi ab =
2 2p
fi q= = 120°
5 5 3
fi b=
2 2 d2A
Clearly, is - ve
fi b= 5 d 2q
Thus, A is maximum.
c-4
Also, b = Hence, the maximum value of A
2 5
Ê 1 ˆ
c-4 = ab Á sin 120° - sin (240°)˜
fi = 5 Ë 2 ¯
2 5
Ê 3 3ˆ
fi c – 4 = ±10 =Á + ˜ ab
Ë 2 4 ¯
fi c = 4 ± 10 = 14, –6
when c = 14 3 3
= ab s.u.
4

CG_05.indd 52 2/6/2017 4:05:26 PM


Ellipse 5.53

2. The equation of the tangent to the parabola y2 = 4x at But m1m2 = –1


(t2, 2t) is
1 cos q 2 sin q
yt = x + t2 fi - ¥ = -1
2 sin q 4 cos q - 1
fi x – yt + t2 = 0 …(i)
and the equation of the normal cos q
fi =1
4x2 + 5y2 = 20 4 cos q - 1
x2 y 2 fi 4 cos q – 1 = cos q
i.e. + =1
5 4 fi 3 cos q = 1
at ( 5 cos j, 2 sin j) is 1
fi cos q =
a 2 x b2 y 3
- = a 2 - b2
x1 y1 Thus, the radius of the circle,

5x 4y r = (a cos q - 1) 2 + b 2sin 2q
fi - = 5 — 4 =1
5 cos j 2 sin j 2
Ê4 ˆ Ê 1ˆ
fi ( 5 sec j ) x - (2 cosec j ) y = 1 …(ii) = Á - 1˜ + 4 Á1 - ˜
Ë3 ¯ Ë 9¯
Since (i) and (ii) are the same line, so 1 32 33
= + =
5 sec j 2 cosec j —1 9 9 3
= = 2 Hence, the equation of the circle is
1 t t
1 1 33 11
fi sec j = - 2
, cosec j = - ( x - 1) 2 + y 2 = =
5t 2t 9 3
fi cos j = - t 2 5, sin j = - 2t 4. Let the point P be (a cos q, b sin q) and the point Q be
4 2
Thus, 5t + 4t – 1 = 0 Ê pˆ p ˆˆ
Ê Ê
fi 5t4 + 5t2 – t2 – 1 = 0 a cos Á q + ˜ , b sin Á q + ˜ ˜
ËÁ Ë 4¯ Ë 4¯¯
fi 5t2(t2 + 1) – 1(t2 + 1) = 0
fi (5t2 – 1)(t2 + 1) = 0 The equation of the chord joining P and Q is
fi (5t2 – 1) = 0 x pˆ y pˆ
Ê Ê Êpˆ
1 cos Á q + ˜ + sin Á q + ˜ = cos Á ˜
t=± a Ë 8¯ b Ë 8¯ Ë 8¯

5
Also, tan j = ±2 which is identical with
fi j = tan–1(±2) px + qy = r
3. The equation of the given ellipse is Comparing the co-efficients, we get
x2 + 4y2 = 16
Ê pˆ Ê pˆ Ê pˆ
x2 cos Á q + ˜ sin ÁË q + ˜¯ cos ÁË ˜¯
Ë 8¯ = 8 = 8
fi + y2 = 1 Thus,
16 ap bq r
Given centre of the circle is C(1, 0).
Ê p ˆ ap Ê pˆ
Let the equation of the circle be cos Á q + ˜ = cos Á ˜
Ë 8¯ r Ë 8¯
(x – 1)2 + y2 = r2
Since the circle is the largest, so it will touch the ellipse Ê p ˆ bq
at some point P(a cos q, b sin q). and sin Á q + ˜ =
Ë 8¯ r
The equation of the tangent to the ellipse is
x y Squaring and adding, we get
cos q + sin q = 1
a b Ê ap ˆ
2
Ê bq ˆ
2

b cos q 1 ÁË ˜¯ + ÁË ˜¯ = 1
r r
whose slope is m1 = - = - cot q
a sin q 2
a2p2 + b2q2 = r2
b sin q 2 sin q
and m2 = m(CP) = = which is the required condition.
(a cos q - 1) 4 cos q - 1

CG_05.indd 53 2/6/2017 4:05:27 PM


5.54 Coordinate Geometry Booster

5. Let R(h, k) be any point on the locus.


x2 y 2
Let OP: y = x tan a and OQ : y = –x tan a 7. Given ellipse is + =1
a 2 b2
It is given that,
Y
(h sin a – k cos a)2 + (h sin a + k cos a)2 = 2l2
fi h2 sin2a + k2 cos2a = l2 P(x¢, y¢)
L
h2 k2
fi + =1 X
l 2cosec 2a l 2sec 2a X¢ A¢ S(–ae, 0) O N A
which represents an ellipse.
x2 y 2
6. Let the equation of the ellipse be + = 1.
a 2 b2

Let C be the centre and PQ be the chord whose equa-
tion is x cos a + y sin a = p. The equation of tangent at P is
Now, we make above two equations a homogeneous xx ¢ yy ¢
+ 2 =1
equation of 2nd degree. a2 b
2 b2 x¢
x 2
y 2
Ê x cos a + y sin aˆ The slope of the tangent is -
Thus,
2
+ 2 =Á ˜¯ a2 y¢
a b Ë p

Ê p2 2 ˆ 2
and the slope of SP is
fi ÁË 2 - cos a˜¯ x - 2xy sin a cos a x ¢ + ae
a
Ê p2 ˆ y¢ b2 x¢
+ 2
+ Á 2 - sin 2a˜ y 2 = 0 x ¢ + ae a y ¢
Ëb ¯ Now, tan q =
Ê y¢ b2 x¢ ˆ
Since the pair of diameters CP and CQ are at right an- 1- Á ◊ 2 ˜
Ë x ¢ + ae a y ¢ ¯
gles, so
a 2b 2 + b 2 x ¢ae
Ê p2 2 ˆ Ê p2 2 ˆ =
ÁË 2 - cos a ˜¯ ÁË 2 - sin a ˜¯ = 0
+ x ¢y ¢a 2e 2 + a 2ey ¢
a b
b 2 a (a + ex ¢ ) b2
p2 p2 = =
fi + =1 2
a ey ¢ (a + ex ¢ ) aey ¢
a 2 b2
Ê 1 1ˆ Ê b2 ˆ
fi p2 Á 2 + 2 ˜ = 1 fi q = tan -1 Á
Ëa b ¯ Ë aey ¢ ˜¯
1 a 2b 2 x2 y 2
fi p2 = = + =1
Ê 1 1ˆ a 2 + b2 8. Given ellipse is
a 2 b2
ÁË 2 + 2 ˜¯
a b
and the circle is x2 + y2 = ab.
ab
fi p= On solving, we get,
a 2 + b2
ab a 2b ab 2
Thus, PQ : x cos a + y sin a = x2 = and y 2 =
a 2 + b2 a+b a+b
Now, the length of the perpendicular draw a from the The equation of tangent to the ellipse at (x1, y1) is
centre to the line PQ xx1 yy1
+ 2 = 1 and the circle is xx1 + yy1 = ab
ab a2 b
0+0-
a + b2
2
ab The slope of the tangent to the ellipse is
= =
b 2 x1
cos 2a + sin 2a a 2 + b2 m1 = -
a 2 y1
= constant
and the slope of tangent to the circle is
Hence, the line PQ touches a fixed circle whose centre
ab a
is (0, 0) and the radius is . m2 = -
a + b2
2 b

CG_05.indd 54 2/6/2017 4:05:28 PM


Ellipse 5.55

m2 - m1 10. The equation of the tangent at P is


Now, tan q = x y
1 + m1m2 cos j + sin j = 1 …(i)
a b
aÊ b2 ˆ and the equation of the normal at P is
ÁË1 - 2 ˜¯
b a ax sec j – by cosec j = a2 – b2 …(ii)
=
ab 2 Then Q = (a sec j, 0)
1+ 2
ba Ê (a 2 - b 2 )cos j ˆ
a - b2 a
2 and R = ÁË , 0˜
a ¯
=
a ( a + b) b
a-b
=
ab
Ê a - bˆ P
Thus, q = tan -1 Á
Ë ab ˜¯
R a Q
9. The normal at P is
ax by
- = a 2 - b2
cos q sin q
and the normal at Q is
ax by Therefore, QR = a
fi - = a 2 - b2 (a 2 - b 2 ) cos j
Ê pˆ Ê pˆ fi a sec j - =a
cos Á q + ˜ sin Á q + ˜ a
Ë 2¯ Ë 2¯
ax by fi a2 sin2 j + b2 cos2 j = a2 cos j
- - = a 2 - b2 fi a2 sin2 j + a2(1 – e2) cos2 j = a2 cos j)
sin q cos q
fi a2(sin2 j + cos2 j) – a2e2 cos2 j = a2 cos j
ax by
fi + = b2 - a 2 fi a2 – a2e2 cos2 j = a2 cos j
sin q cos q
fi e2 cos2 j + cos j – 1 = 0
The slope of the normal at P is 11. Let the point P be (a cos j, b sin j)
a sin q a and S = (ae, 0), S¢ = (–ae, 0)
m1 = = tan q \ SP = a – ex = a – e cos j
b cos q b
and S¢P = a + ae cos j
and the slope of the normal at Q is
Also, SS¢ = 2ae
a Let (a, b) be the incentre of DPSS¢. So
m2 = - cot q
b 2ae ◊ a cos j + a(1 - e cos j )( - ae)
m - m2 + a (1 - e cos j )(ae)
Now, tan a = 1 a=
1 + m1m2 2ae + a (1 - e cos j ) + a(1 + e cos j )
a fi a = ae cos j
(tan q + cot q )
= b be sin j
a2 Similarly, b =
1- 2 1+ e
b Eliminating f, we get
ab 2 2 2
= 2 2 Êaˆ Ê b(1 + e) ˆ
(b - a ) sin 2q ÁË ˜¯ + Á =1
ae Ë be ˜¯
2ab 1
= 2 2 ¥ a2 b 2 (1 + e) 2
a e sin 2q fi + =1
2 2
a e b2e2
2a 2 1 - e 2 1 Hence, the locus of incentre is
= 2 2
¥
a e sin 2q
x2 y2
2 + =1
2 1- e a 2e2 b 2e2
=
e 2 (sin 2q ) (1 + e) 2

CG_05.indd 55 2/6/2017 4:05:29 PM


5.56 Coordinate Geometry Booster

Let e1 be its eccentricity, then \ Circumcentre of DPQR be


2 2
b e (a 2 - b 2 )
h=
(1 + e) 2 4a
e1 = 1 - 2 2
a e (cos a + cos b + cos g + cos (a+ b + g ))
a 2 (1 - e 2 ) (a 2 - b 2 )
= 1- and k=
a 2 (1 + e) 2 4a
(sin a + sin b + sin g - sin (a+ b + g ))
1- e 2e
= 1- = On simplification, we get
1+ e 1+ e
h(a 2 + 3b 2 )
12. Let the point P be (a cos j, b sin j). cos (a + b + g ) =
Y
a(a 2 - b 2 )
(a 2 + 3b 2 )k
B and sin (a + b + g ) =
P b( a 2 - b 2 )
Squaring and adding, we get
q
X¢ X 2 2
A¢ C S A Ê ( a 2 + 3b 2 ) h ˆ Ê ( a 2 + 3b 2 ) k ˆ
Á ˜ + Á ˜ =1
Ë a(a 2 - b 2 ) ¯ Ë b( a 2 - b 2 ) ¯

Hence, the locus of the centroid (h, k) is
Y¢ 2 2
Ê (a 2 + 3b 2 ) x ˆ Ê ( a 2 + 3b 2 ) y ˆ
b sin j (b /a ) sin j Á 2 2 ˜
+Á ˜ =1
tan q = = Ë a(a - b ) ¯ Ë b( a 2 - b 2 ) ¯
a cos j — ae cos j - e
14. The equation of the tangent to the ellipse at P is
1 - e 2 sin j
= x y
cos j - e cos j + sin j = 1 …(i)
a b
fi tan q cos j - e tan q = 1 - e 2 sin j Let the equation of the auxilliary circle be
x2 + y2 = a2 …(ii)
2 tan (q /2) Ê 1 - tan 2 (j /2) ˆ
fi 2
¥ Á 2
- e˜ Y
1 - tan (q /2) Ë 1 + tan (j /2) ¯
Ê 2 tan (q /2) ˆ Ê 1 - tan 2 (j /2) ˆ B
= 1 - e2 Á Á ˜
Ë 1 + tan 2 (q /2) ˜¯ Ë 1 + tan 2 (j /2) ¯ P
A
On simplification, we get X¢ X
O
Ê jˆ Ê qˆ
fi 1 + e tan Á ˜ - 1 - e tan Á ˜ = 0
Ë 2¯ Ë 2¯
Ê jˆ Ê qˆ
1 + e tan Á ˜ = 1 - e tan Á ˜
Ë 2¯ Ë 2¯ Y¢

The equation of the pair of lines OA and OB are ob-


Ê qˆ 1+ e Ê jˆ
fi tan Á ˜ = tan Á ˜ tained by making homogeneous of (i) and (ii). So
Ë 2¯ 1- e Ë 2¯
2
Êx y ˆ
13. Let the vertices of the equilateral triangle P, Q, R, x 2 + y 2 = a 2 Á cos j + sin j˜
whose eccentric angles are a, b, g respectively. Ëa b ¯
Let (h, k) be the centroid of DPQR. Ê x2 y2 xy ˆ
= a 2 Á 2 cos 2j + 2 sin 2j + 2 sin j cos j˜
a Ëa b ab ¯
Then h= (cos a + cos b + cos g )
3
Ê a2 ˆ
a fi (1 - cos 2j) x 2 + Á1 - 2 sin 2j˜ y 2 + (....) xy = 0
and k = (sin a + sin b + sin g ) Ë b ¯
3
It is given that, –AOB = 90°. So
Since DPQR is an equilateral triangle, so
co-efficient of x2 + co-efficient of y2 = 0
centroid = circumcentre.

CG_05.indd 56 2/6/2017 4:05:30 PM


Ellipse 5.57

Since SF1 + S¢F1 = 2a = F1S¢ + F2S¢


a2
fi 1 - cos 2j + 1 - sin 2j = 0 B
b2
a2
fi sin 2j + 1 - sin 2j = 0
2 2
a (1 - e ) F1

1 q
fi sin 2j + 1 - sin 2j = 0 S¢ A
(1 - e 2 ) S¢ C S

Ê 1 ˆ 2 2
fi ÁË1 — 1 — e 2 ˜¯ sin j = -1

Ê — e2 ˆ B¢
fi Á 2˜
sin 2j = -1
Ë1 — e ¯ Clearly, CS = ae and CF1 = ae¢
Let q be the angle between their axes.
fi e2 sin2 j = (1 – e2)
Then, SF12 = a 2e 2 + a 2e¢ 2 - 2a 2ee¢ cos q
fi e2(1 + sin2 j) = 1
and S ¢F12 = a 2e 2 + a 2e¢ 2 + 2a 2ee¢ cos q
2 1
fi e = Now, 2a = SF1 + S¢F1
(1 + sin 2j )
fi 4a2 = (SF1 + S¢F1)2
1
fi e= fi 4a 2 = SF12 + S ¢F12 + 2( SF1 )( S ¢F1 )
2
(1 + sin j )
fi 4a2 = 2a2(e2 + e¢2) +
15. Let two points on the ellipse be P and Q whose eccen-
tric angles are j1 and j2 where j1 – j2 = a. 2 (a 2e 2 + a 2e¢ 2 ) - 4a 4e 2e¢ 2cos 2q
The equation of the tangent at P and Q are fi (2 – e2 – e¢2)2 = (e2 + e¢2)2 – 4e2e¢2 cos2 q
x y fi 4 – 4(e2 + e¢2) = –4e2e¢2 cos2 q
cos j1 + sin j1 = 1
a b fi 1 – (e2 + e¢2) = –e2e¢2 cos2 q
x y Ê e 2 + e¢ 2 - 1ˆ
and cos j 2 + sin j 2 = 1
a b fi cos 2q = Á ˜
Ë e2e¢ 2 ¯
Let R(h, k) be the point of intersection of the tangents.
Ê e2 + e¢ 2 - 1 ˆ
Ê j + j2 ˆ Ê j + j2 ˆ fi cos q = Á ˜¯
a cos Á 1 b sin Á 1 Ë
Ë 2 ˜¯ Ë 2 ˜¯ ee¢
Thus, h = and k =
Ê j - j2 ˆ Ê j - j2 ˆ 17. Let the point of concurrency be (h, k).
cos Á 1 cos Á 1
Ë 2 ˜¯ Ë 2 ˜¯ The equation of the normal to the given ellipse at (x¢,
y¢) is
Ê j + j2 ˆ Ê j + j2 ˆ
cos Á 1 ˜ sin Á 1 a 2 x b2 y
h Ë 2 ¯ k Ë 2 ˜¯ - = a 2 - b2
= and = x¢ y¢
a Ê aˆ b Êaˆ
cos Á ˜ cos Á ˜
Ë 2¯ Ë 2¯ which is passing through (h, k). So
a 2h b2k
Squaring and adding, we get - = a 2 - b2 …(i)
x¢ y¢
2 2
Ê hˆ Ê kˆ 1 x2 y 2
ÁË ˜¯ + ÁË ˜¯ = Also, the point (x¢ y¢) lies on 2 + 2 = 1 . So
a b Êaˆ a b
cos 2 Á ˜
Ë 2¯
x¢ 2 y ¢ 2
+ 2 =1 …(ii)
Hence, the locus of R(h, k) is a2 b
x2 y 2 Êaˆ On simplification, we get
+ = sec 2 Á ˜
a 2 b2 Ë 2¯ –(a2 – b2) x¢4 + 2a2(a2 – b2)hx¢3 + (…)x¢2
16. Let S and S¢ be the foci of one ellipse and F1 and F2 of –2a4(a2 – b2)hx¢ + a6h2 = 0
the other, where C being the common centre. which is a bi-quadratic equation. So, it has four roots,
So, SF1S¢F2 will form a parallelogram. say x1, x2, x3 and x4.

CG_05.indd 57 2/6/2017 4:05:31 PM


5.58 Coordinate Geometry Booster

The equations of tangents at P and Q are


2ha 2
Then x1 + x2 + x3 + x4 = x y
(a 2 - b 2 ) cos a + sin a = 1
a b
and x1x2x3x4 = a2 x y
and cos b + sin b = 1
1 1 1 1 Â x1 x2 x3 a b
Now, + + + =
x1 x2 x3 x4 x1 x2 x3 x4 On solving, we get the point of intersection A, i.e.
2(a 2 - b 2 ) Ê Êa + bˆ Êa + bˆ ˆ
= Á a cos ÁË 2 ˜¯ a sin ÁË 2 ˜¯ ˜
a2h A=Á , ˜
Hence, the value of Á cos ÊÁ a - b ˆ˜ cos ÊÁ a - b ˆ˜ ˜
Ë Ë 2 ¯ Ë 2 ¯ ¯
Ê1 1 1 1ˆ
( x1 + x2 + x3 + x4 ) Á + + + ˜ x2 y 2
Ë x1 x2 x3 x4 ¯ Since the point A lies on + = 4 , so
a 2 b2
2ha 2 (a 2 - b 2 )
= ¥ =1 Ê a + b ˆ sin 2 Ê a + b ˆ
(a 2 - b 2 ) 2ha 2 cos 2 Á
Ë 2 ˜¯ ÁË
2 ¯
˜
+ =4
x2 y 2 Êa - bˆ Êa - bˆ
+ = 1. cos 2 Á ˜ cos 2 Á ˜
18. Let the ellipse be Ë 2 ¯ Ë 2 ¯
a 2 b2
Êa - bˆ 1
Let P and Q be two points lie on the ellipse whose ec- fi cos 2 Á =
centric angles are a and b such that q = a – b. Ë 2 ˜¯ 4
Given that the tangents at P and Q are at right angles. Ê a - bˆ 1
fi cos Á ˜ =±
Ê b ˆÊ b ˆ Ë 2 ¯ 2
ÁË - cot a ˜¯ ÁË - cot b ˜¯ = -1
a a \ The equations of the normals PR and QR are
ax sec a – by cosec a = a2 – b2
fi a sin a sin b + b cos a cos b = 0
2 2
and ax sec b – by cosec b = a2 – b2
But the diameter parallel to the tangent at P will be
On simplification, we get
conjugate to the diameter CP, then its extremities will
be (–a sin a, b cos a).
Thus, d12 = a 2sin 2a + b 2cos 2a
ax
2
a -b 2
Ê a + bˆ
= ± cos Á
Ë 2 {
˜¯ cos (a + b) -
1
2 }
Similarly, d 22 = a 2sin 2 b + b 2cos 2 b
Now,
by
2
a -b 2
Ê a + bˆ
= ± sin Á
Ë 2 {
˜¯ cos (a + b) +
1
2 }
Squaring and adding, we get
d12 d 22 = (a2 sin2 a + b2 cos2 a)(a2 sin2 b + b2 cos2 b) 2 2
Ê ax ˆ Ê by ˆ
= (a2 sin a sin b + b2 cos a cos b)2 ÁË a 2 - b 2 ˜¯ + ÁË a 2 - b 2 ˜¯
+ a2b2 (sin a cos b – cos a sin b)2
1
= 0 + a2b2 sin2(a – b) = cos 2 (a + b ) + - cos 2 (a + b )
4
= a2b2 sin2q
1 2
fi d1d2 = ab sin q fi a 2 x2 + b2 y 2 = (a - b 2 )2
4
Hence, the result. Hence, the result.
2 2 2 2
x y x y 20. Let the circle
19. Given conics are + = 4 and 2 + 2 = 1 .
a 2 b2 a b x2 + y2 + 2gx + 2fy + c = 0 …(i)
Y 2 2
x y
intersect the ellipse 2
+ 2 = 1,
a b
i.e. b2x2 + a2y2 = a2b2 …(ii)
P in four points (x1, y1), (x2, y2), (x3, y3) and (x4, y4), re-
X¢ X spectively.
A R(h, k) It is given that, one point (x1, y1) = (h, k) is fixed and
Q other two points (x2, y2) and (x3, y3) are extremities of a
diameter of the ellipse.
Since j and p + j are the eccentric angles of the ex-
Y¢ tremities of diameters of ellipse,

CG_05.indd 58 2/6/2017 4:05:32 PM


Ellipse 5.59

So, (x2, y2) = (a cos j, b sin j)


Integer Type Questions
and (x3, y3) = (–a cos j, –b sin j)
Thus, x1 + x2 + x3 = h and y1 + y2 + y3 = k 1. The equation of the tangent to the ellipse is
x y
Now, from Eq. (i), we get + =1
3 sec q 2 cosec q
x2 + y2 + 2gx + 2fy + c = 0
fi a2x2 + a2y2 + 2ga2x + 2fa2y + a2c = 0 Now,
fi a2x2 + a2y2 + 2ga2x + a2c = –2fa2y D = ar DOAB
1 6
fi (a2x2 + a2y2 + 2ga2x + a2c)2 = 4f 2a4y2 = ¥ 3 sec q ¥ 2 cosec q =
2 sin 2q
fi (a2x2 + a2b2 – b2x2 + 2ga2x + a2c)2
Minimum area of the triangle = 6 s.u.
= 4f 2a2(a2b2 – b2x2)
2. We have F1 = (2, 0) and F2 = (–2, 0)
fi (a2 – b2)2x4 + 4a2g(a2 – b2)x3 + ax2 + bx + g = 0
Now, A = ar(DPF1F2)
where a, b and g are constants which is a bi-quadratic
equation. 5 cos q sin q 1
Let x1, x2, x3 and x4 are its roots. 1
= 2 0 1
4ga 2 2
So, x1 + x2 + x3 + x4 = - -2 0 1
(a 2 - b 2 )
1
= (4 sin q ) = 2 sin q
4ga 2 2
fi h + x4 = - 2
(a - b 2 ) Hence, the maximum value of A is 2 s.u.
2 3. Given ellipse is 16x2 + 11y2 = 256.
4ga
fi x4 = - -h x2 y2
(a 2 - b 2 ) + =1
16 (16/ 11) 2
4fb 2
Similarly, y4 = - -k The equation of the tangent to the given ellipse at
(a - b 2 )
2
Ê 16 ˆ
Since, (x4, y4) lies on the ellipse, so ÁË 4 cos j , sin j ˜ is
¯
11
x42 y42 x cos j y sin j
+ =1 + =1 …(i)
a 2 b2 4 (16/ 11)
2 2
Ê 4ga 2 ˆ Ê 4fb 2 ˆ which is also a tangent to the circle
Á - a 2 - b 2 - h˜ Á - a 2 - b2 - k ˜ x2 + y2 – 2x – 15 = 0
Ë ¯ Ë ¯
fi + =1 fi (x – 1)2 + y2 = 16
a2 b2
So, the length of the perpendicular from the centre to
16a 2 g 2 8gh 16b 2 f 2 the circle is equal to the radius of a circle.
fi 2 2 2
+ 2 2
+ 2
(a - b ) a -b (a - b 2 ) 2 Ê cos j ˆ
ÁË ˜ -1
8fk h2 k 2 4 ¯
- + 2 + 2 =1 =4
2 2
(a - b ) a b cos 2j sin 2j
+
16 256/11
2 g 2a2 2 f 2b 2 2
fi 2 2
+ gh + 2 - fk = 0 Ê Ê cos j ˆ ˆ Ê cos 2j sin 2j ˆ
(a - b ) (a - b 2 ) fi ÁË ÁË 4 ˜¯ ˜¯ = 16 Á 16 + 256/11˜
- 1
Ë ¯
2( g 2 a 2 + f 2b 2 )
fi + gh - fk = 0 Ê cos 2j cos j ˆ 11
(a 2 - b 2 ) fi ÁË -2 + 1˜ = cos 2j + sin 2j
16 4 ¯ 16
fi 2(g2a2 + f2b2) = (gh – fk)(a2 – b2) fi 4 cos2j + 8 cos j – 5 = 0
Hence, the locus of the centre (–g, –f) is fi (2 cos j – 1)(2 cos j + 5) = 0
2(x2a2 + y2b2) = (hx – ky)(a2 – b2) fi (2 cos j – 1) = 0
fi 2(a2x2 + b2y2) = (a2 – b2)(hx – ky) 1
fi cos j =
Hence, the result. 2

CG_05.indd 59 2/6/2017 4:05:32 PM


5.60 Coordinate Geometry Booster

p 5p 5
fi j= , fi sin q = -
6 6 24
Hence, the number of values of f is 2. Ê 5ˆ Ê 5ˆ
4. The required area of a parallelogram is made by the fi q = 2p - sin -1 Á ˜ , p + sin -1 Á ˜
Ë 24 ¯ Ë 24 ¯
tangents at the extremities of a pair of conjugate diam-
eter Also, y-axis is one of the normals.
= 4ab Hence, it has three normals.
1
= 4¥3¥ Previous Years’ JEE-Advanced Examinations
4
= 3 s.u. 1. Given line is
x2 y2 9
5. Given curve is + =1 x=-
a - 10 4 - a 2
(4 – a)x2 + (a – 10)y2 – (a – 10)(4 – a) = 0 fi 2x + 9 = 0
which represents an ellipse, if h2 – ab < 0 Let the point be P(h, k).
fi –(4 – a)(a – 10) < 0 2 Ê 2x - 9 ˆ
( x + 2) 2 + y 2 = ¥
3 ÁË ˜
fi (a – 4)(a – 10) < 0 Given
4 ¯
fi 4 < a < 10 2
Hence, the number of integral values of a is 5. 2 4 Ê 2x - 9 ˆ
2
fi ( x + 2) + y = ¥ Á
6. Given ellipse is 9 Ë 4 ˜¯
x2 + 4y2 = 4 1
x2 y 2 fi ( x + 2) 2 + y 2 =
¥ (2x - 9) 2
fi + =4 9
4 1 fi 9((x + 2)2 + y2) = (2x – 9)2
Let A be the area of the rectangle. fi 9(x2 + y2 + 4x + 4) = 4x2 – 36x + 81
So, A = (4 cos q)(2 sin q) = 4 sin (2q) fi 5x2 + 9y2 + 72x – 55 = 0
Thus, the greatest area of the rectangle is 4.
which represents an ellipse.
7. Given ellipse is 9x2 + 16y2 = 144
x2 y 2 1 4
+ =1 2. Clearly, + -1> 0
16 9 9 4
Hence, the value of fi P lies outside of E.
PF1 + PF2 – 2 = 2a – 2 Also, 1 + 4 – 9 = –4 < 0
=8–2 fi P lies inside C.
=6 Thus, P lies inside C but outside E.
8. Clearly, (S1F1) ◊ (S2F2) = b2 = 3 x2 y 2
9. The minimum length of the intercept of the tangent to 3. Given ellipse is + =1
a 2 b2
x2 y 2
the ellipse + = 1 between the axes = a + b = 7. So, F1 = (ae, 0) and F2 = (–ae, 0)
16 9 Let P be (a cos q, b sin q)
10. The equation of the normal to the ellipse is Then
a 2 x b2 y ar(DPF1F2)
- = a 2 - b2
x1 y1 a cos q b sin q 1
1
169x 25y = ae 0 1
fi - = 169 - 25 2
13 cos q 5 sin q - ae 0 1
13x 5y 1
fi - = 144 =¥ 2ae ¥ b sin q
cos q sin q 2
which is passing through (0, 6). So = abe ¥ sin q
30 Maximum value of A = abe
0- = 144
sin q b2
= ab 1 -
5 a2
fi - = 24
sin q
= b a 2 - b2

CG_05.indd 60 2/6/2017 4:05:33 PM


Ellipse 5.61

Y
x2 y 2
4. Given ellipse is + =1 A(h, k)
16 9
Q
Ê 7 ˆ
Foci = (± ae, 0) = Á ± 4 ◊ , 0˜ = (± 7, 0) P
Ë 4 ¯
X¢ X
O
Radius of a circle = 7 + 9 = 16 = 4

x2 y 2
5. Given ellipse is + =1
a 2 b2 Y¢
The equation of the tangent to the ellipse (i) is
Y
x
cos q + y sin q = 1 …(ii)
2
M
P The equation of the 2nd ellipse can be written as
x2 y 2
X¢ X + =1 …(iii)
F2(–ae, 0) F1(ae, 0) 6 3
Let the tangents at P and Q meet at A(h, k).
So PQ is the chord of contact.
Y¢ The equation of the chord of contact of the tangents
The equation of the tangent to the given ellipse at through A is
P(a cos q, b sin q) is hx ky
+ =1 …(iv)
x y 6 3
cos q + sin q = 1 Since the equations (ii) and (iv) are identical, so
a b
h k
Now,
OM = d 6 = 3 =1
cos q sin q
0 + 0 -1 2
fi =d
cos 2q sin 2q fi h = 3 cos q, k = 3 sin q
+ 2
a2 b Now, squaring and adding, we get
h2 + k2 = 9
1 cos 2q sin 2q
fi = + 2 Therefore, the locus of A is
d2 a2 b x2 + y2 = 9
Now, x2 y 2
which is the director circle of + = 1.
Ê b2 ˆ 4a 2b 2 6 3
4a 2 Á1 - 2 ˜ = 4a 2 -
Ë d ¯ d2 Thus, the angle between the tangents at P and Q of the
ellipse x2 + 2y2 = 6 right angle.
Ê cos 2q sin 2q ˆ
= 4a 2 - 4a 2b 2 Á 2 + 2 ˜ x2 y 2
Ë a b ¯ 7. Let the ellipse be + = 1.
a 2 b2
= 4a2 – 4b2 cos2q – 4a2sin2q B = (0, b), F = (ae, 0) and F¢ = (–ae, 0)
= 4a2(1 – sin2q) – 4b2cos2q It is given that, FBF¢ is a right angle. So
= 4a2cos2q – 4b2cos2q FB2 + F¢B2 = (FF¢)2
= 4 cos2q(a2 – b2) fi a2e2 + b2 + a2e2 + b2 = 4a2e2
= 4 cos2q(a2e2) fi 2a2d2 = 2b2
= (2ae cos q)2 fi a2e2 = b2 = a2(1 – e2)
= [(a + ae cos q) – (a – ae cos q)]2 fi a2d2 = a2 – a2e2
= (PF1 – PF2)2 fi 2a2e2 = a2
fi 2e2 = 1
6. Given ellipse is
x2 + 4y2 = 4 1
fi e2 =
x2 y 2 2
fi + =1 …(i)
4 1 1
fi e=
2

CG_05.indd 61 2/6/2017 4:05:33 PM


5.62 Coordinate Geometry Booster

8. Ans. (c) 1
Given ellipse is Therefore, ar DOPN = ¥ ON ¥ NP
2
16x2 + 25y2 = 400 1 ab(a 2 - b 2 ) sin q cos q
= ¥
x2 y 2 2 a 2sin 2q + b 2cos 2q
fi + =1
25 16 1 ab(a 2 - b 2 )
= ¥ 2
Now, 2 a tan q + b 2cot q
PF1 + PF2 = 2a
1 ab(a 2 - b 2 ) (a 2 - b 2 )
= 10 £ ¥ =
2 2ab 4
x2 y 2 b
9. Given ellipse is + =1 at tan q =
a 2 b2 a
Y Thus, the point P is
Ê a2 b2 ˆ
Á 2 ,

N 2 2
P Ë a +b a +b ¯

X¢ X By symmetry, we have four such points.


O
L
Ê a2 b2 ˆ
Thus, Á ± ,± ˜
Ë a 2 + b2 a 2 + b2 ¯

Let P lies in the first quadrant. 10. Given ellipse is 4x2 + 9y2 = 1 …(i)
So P = (a cos q, b sin q) Differentiating w.r.t. x, we get,
The equation of the tangent at P is dy
8x + 18y ◊ =0
x y dx
cos q + sin q = 1
a b dy 8x 4x
fi =— =-
-1 dx 18y 9y
Now, ON =
cos q sin 2q
2 Since the tangent is parallel to 8x = 9y, so
+ 2
a2 b —
4x 8
=
ab 9y 9
=
a 2sin 2q + b 2cos 2q x 2
fi — =
The equation of ON is y 1
x y fi x = –2y.
sin q - cos q = 0
b a Put x = –2y in Eq. (i), we get
and the equation of the normal at P is fi 4(4y2) + 9y2 = 1
fi 25y2 = 1
ax sec q – by cosec q = a2 – b2
1
a 2 - b2 fi y2 =
So, OL = 25
a 2sec 2q + b 2cosec 2q
1
(a 2 - b 2 ) sin q cos q fi y=±
= 5
a 2sin 2q + b 2cos 2q
1 2
when y = ± , then x =
Now, NP = OL 5 5
(a 2 - b 2 ) sin q cos q Ê 2 1ˆ Ê 2 1ˆ
fi NP = Hence, the points are Á - , ˜ and Á , - ˜ .
2 2
a sin q + b cos q 2 2 Ë 5 5¯ Ë 5 5¯

CG_05.indd 62 2/6/2017 4:05:34 PM


Ellipse 5.63

11. Let the co-ordinates of P be (a cos q, b sin q) and of Q


Ê ae ˆ Ê b 2 ˆ
be (a cos q, a sin q) respectively. fi ÁË1 - ˜¯ = ÁË 2 ˜¯
x a
Y Ê b 2 ˆ Ê ae ˆ
Q fi ÁË1 - 2 ˜¯ = ÁË ˜¯
a x
R
P Ê a 2 (1 - e 2 ) ˆ Ê ae ˆ
fi ÁË 1 - =
a2 ¯˜ ËÁ x ¯˜
X¢ X
O
Ê ae ˆ
fi e2 = Á ˜
Ë x¯
a
fi x=
Y¢ e
Hence, the result.
Let R(h, k) divides PQ in the ratio r : s.
x2 y 2
s (a cos q ) + r (a cos q ) 13. Given ellipse is + =1
Then h = = a cos q 9 5
r+s Y
s (b sin q ) + r (a sin q ) (ar + bs ) sin q
and k= = S
r+s r+s N L

h k (r + s)
Thus, = cos q , = sin q X¢
R O X
a (ar + bs ) P

Squaring and adding, we get N¢ L¢


Q
h 2 k 2 (r + s)2 Y¢
+ =1
a 2 (ar + bs ) 2 b2 5 2
Now, e = 1 - 2
= 1- =
Hence, the locus of R(h, k) is a 9 3
x 2 (r + s)2 y 2 Ê b2 ˆ Ê 5 ˆ
+ =1 L = Á ae, ˜ = Á 2, ˜
a 2 (ar + bs ) 2 Ë a ¯ Ë 3¯
which represents an ellipse. Ê b2 ˆ Ê 5ˆ
L ¢ = Á ae, - ˜ = Á 2, - ˜
12. Any tangent to the ellipse is Ë a¯ Ë 3¯
x y Ê b2 ˆ Ê 5ˆ
cos q + sin q = 1 …(i) N = Á — ae, ˜ = Á —2, ˜
a b Ë a¯ Ë 3¯
Its point of contact is P(a cos qc b sin q) and its slope Ê b2 ˆ Ê 5ˆ
b and N ¢ = Á — ae, - ˜ = Á — 2, - ˜
is - cot q . Also the focus is S(ae, 0). Ë a¯ Ë 3¯
a
Any line through the focus S and the perpendicular to Now, the the tangent at L is
tangent (i) is xx1 yy1
+ =1
a 9 5
y-0= tan q ( x - ae) …(ii) 5
b y
2x 3
Also the equation of CP is fi + =1
9 5
a x y
y-0= tan q ( x - 0) …(iii) fi + =1
b 9/2 3
Eliminating q between Eqs (ii) and (iii), we get Ê9 ˆ
Thus, P = Á , 0˜ and S = (0, 3)
Ë2 ¯
Ê a 2 ˆ Ê x - ae ˆ Therefore,
ÁË 2 ˜¯ ÁË ˜ =1
b x ¯ ar(quad PQRS) = 4 ¥ ar(DOPS)
2
Ê x - ae ˆ Ê b ˆ 1 9
fi ÁË ˜ = = 4¥ ¥ ¥3
x ¯ ÁË a 2 ˜¯ 2 2
= 27 s.u.

CG_05.indd 63 2/6/2017 4:05:35 PM


5.64 Coordinate Geometry Booster

14. Any tangent to the ellipse Thus, the equation of the common tangent is
x y 2 7
cos q + sin q = 1 y=- x+4
a b 3 3
Let it meet the x-axis at A and y-axis at B. Then the co- This tangent meets the co-ordinate axes in P and Q re-
ordinates of A and B are spectively.
Ê a ˆ Ê b ˆ
Ê 4 7ˆ
A=Á , 0˜ and B = Á 0,
Ë cos q ¯ Ë sin q ˜¯ So, P = (2 7, 0) and Q = Á 0, ˜
Ë 3 ¯
Let M(h, k) be the mid-point of AB.
a b 112 84 + 112 196 14
Then 2h = and 2k = Length of PQ = 28 + = = =
cos q sin q 3 3 3 3
a b
fi h= and k = x2 y 2
2 cos q 2 sin q 16. Given ellipse is + =1
a b a 2 b2
fi cos q = and sin q =
2h 2k The equation of any tangent to the given ellipse at P(a
Squaring and adding, we get cos q, b sin q) is
a2 b2 x y
2
+ 2 =1 fi cos q + sin q = 1
4h 4k a b
2
a b2 x y
fi + =4 fi + =1
h2 k 2 a sec q b cosec q
Hence, the locus of M(h, k) is
Y
a 2 b2
+ =4 B
x2 y 2 P
15. The equation of any tangent to the given ellipse is
X¢ X
O A
2 …(i)
y = mx + 25m + 4
Y

Q
B(0, 2) M Here, A = (a sec q, 0) and B = (0, cosec q)
Thus,
P 1

A¢(–5, 0) O A(5, 0) X ar(DOAB) = ¥ a sec q ¥ b cosec q
2
B¢(1, –2) ab
=
|sin 2q |

Y¢ Ê 1 ˆ
≥ ab, |sin 2q| £ 1 fi ≥1
Hence OM = 4 ËÁ |sin 2q| ¯˜

m.0 - 0 + 25m 2 + 4 17. Given ellipse is


fi =4
2
m +1 x2 + 4y2 = 4

25m 2 + 4 x2 y 2
fi =4 fi + =1
4 1
m2 + 1
fi (25m2 + 4) = 16(m2 + 1) L¢ L
fi 9m2 = 12
S¢ S
fi 3m2 = 4 X¢
V
X
2
fi m=±
3 Q P
2 Y¢
=- ( m < 0)
3

CG_05.indd 64 2/6/2017 4:05:36 PM


Ellipse 5.65

Y
b2 1 3
Thus, e = 1 - 2 = 1 - =
a 4 2 M
B(0, 1)
Foci:
S = (ae, 0) = ( 3, 0) X¢
A N O
X
A(3, 0)
and S ¢ = (— ae, 0) = ( - 3, 0)
End-points of locus recta:
Ê b2 ˆ Ê 1ˆ Y¢
L = Á ae, ˜ = Á 3, ˜
Ë a¯ Ë 2¯
Put x = 3(1 – y) in Eq. (i), we get
Ê b2 ˆ Ê 1ˆ 9(1 – y)2 + y2 = 9
and L ¢ = Á — ae, ˜ = Á - 3, ˜
Ë a ¯ Ë 2¯ fi 9(y2 – 2y + 1) + y2 = 9
fi 10y2 – 18y = 0
Ê 1ˆ Ê 1ˆ
Thus, P = Á 3, - ˜ and Q = Á - 3, - ˜ fi 5y2 – 9y = 0
Ë 2 ¯ Ë 2¯
9
As we know that, the focus is the mid-point of P and Q. fi y = 0,
5
1
Thus, the focus of a parabola is ÊÁ 0, - ˆ˜ . Thus, the y co-ordinate of M is .
9
Ë 2¯ 5
Now,
and the length of PQ = 2 3
1
ar(DOAM) = ¥ OA ¥ MN
2
3
Now, 4a = 2 3 fi a = 1 9
2 = ¥3¥
2 5
Thus, the vertices of a desired parabola 27
=
Ê 1 ˆ Ê 1 3ˆ 10
= Á 0, - ± a˜ = Á 0, - ± ˜
Ë 2 ¯ Ë 2 2 ¯ 19. Given ellipse is x2 + 4y2 = 16
Therefore, two desired parabolas are
fi x2 y 2 …(i)
+ =1
Ê Ê 1 3ˆˆ 16 4
fi x 2 = ± 4a Á y - Á - ± ˜˜
Ë Ë 2 2 ¯¯ Y

Ê 1 3ˆ
fi x2 = 2 3 Á y + + ˜ P
Ë 2 2 ¯
M
or X¢ X
C Q
Ê 1 3ˆ
x2 = - 2 3 Á y + - ˜
Ë 2 2 ¯
fi x 2 = 2 3 y + (3 + 3) Y¢
or
Let the co-ordinates of P be (4 cos q, 2 sin q).
x 2 = - 2 3 y + (3 — 3)
The equation of the normal to the given ellipse at P(4
18. Given ellipse is cos q, 2 sin q) is
x2 + 9y2 = 9 4x sec q – 2y cosec q = 42 – 22
x2 y 2 fi 4x sec q – 2y cosec q = 12
fi + =1
9 1 fi 2x sec q – y cosec q = 6
The equation of the auxiliary circle is So, the point Q is (3 cos q, 0).
x2 + y2 = 9 …(i) Let M(h, k) be the mid-point of PQ. So
and the equation of the line AB is
7 cos q
x y h= and k = sin q
fi + =1 2
3 1
x = 3(1 – y)

CG_05.indd 65 2/6/2017 4:05:36 PM


5.66 Coordinate Geometry Booster

Solving Eqs (i) and (ii), we get


4h 2
\ + k2 = 1
49 11 8
x= ,y=
Hence, the locus of M(h, k) is 5 5
4x 2 Ê 11 8 ˆ
+ y2 = 1 …(ii) Hence, the orthocentre is Á , ˜ .
49 Ë 5 5¯
The equation of the latus rectum of (i) is 22. Let the point be M(x, y).
3 It is given that PM = Length of perpendicular from Q to
x = ± ae = ± 4 ¥ = ±2 3 AB
2
x
Put x = ± 2 3 in Eq. (ii), we get + y -1
fi 2
( x — 3) + ( y - 4) = 3 2
2
48 1 Ê 1ˆ
y2 = 1 - = ÁË ˜¯ + 1
49 49 3
1 fi 10((x – 3)2 + (y – 4)2) = (x + 3y – 3)2
fi y=±
7 fi 10(x2 + y2 – 6x – 8y + 25) = (x + 3y – 3)2
Ê 1ˆ
Hence, the required points are Á ± 2 3, ± ˜ . fi 10(x2 + y2 – 6x – 8y + 25)
Ë 7¯
= x2 + 9y2 + 9 + 6xy – 6x – 18y
Y fi 9x2 + y2 – 6xy – 54x – 62y + 241 = 0
P(3, 4)
23. Equation of ellipse is
A
(y + 2)(y – 2) + l(x + 3)(x – 3) = 0
X¢ X Y
O B
Y
B(0, 4)
S(–3, 2) P(3, 2)

X¢ X
x2 y 2 C
20. Given ellipse is + =1 …(i)
9 4 R(–3, –2)
Q(3, –2)
Hence AB is the chord of contact. So
xx1 yy1 Y¢
+ =1
9 4
3x 4y which is passing through B(0, 4). So
fi + =1 6 ¥ 2 + l(–9) = 0
9 4
fi 9l = 12
fi x
+ y =1 …(ii) 4
3 fi l=
3
Solving Eqs (i) and (ii), we get
Thus, the required ellipse is
Ê 9 8ˆ 4
A = Á - , ˜ , B = (3, 0) ( y 2 - 4) + ( x 2 - 9) = 0
Ë 5 5¯ 3
21. The equation of the altitude through A is fi 3(y – 4) + 4(x2 – 9) = 0
2

8 fi 4x2 + 3y2 = 48
y= …(i)
5
1 x2 y 2
and the slope of AB is - fi + =1
3 12 16
The equation of the altitude through P is 12 1
y – 4 = 3(x – 3) …(ii) Thus, e = 1 - =
16 2

CG_05.indd 66 2/6/2017 4:05:37 PM


Ellipse 5.67

Therefore,
x2 y 2
24. Given ellipse is + =1
4 3 8 8 45 5 9
D1 - 8D 2 = ¥ -8¥
5 5 8 2
Y
= 45 – 36
P =9
25. Clearly, the point P(h, 1) lies on the ellipse. So
h2 1
X¢ X + =1
O M(h, 0) R(x, 0) 6 3
h2 1 2
Q fi =1- =
6 3 3
h2
Y¢ fi =2
2
when x = h, then fi h2 = 4
fi h = ±2
y2 h2 4 - h2
=1- = Now, the tangent at (2, 1) is
3 4 4
2x y
3 fi + =1
fi y=± 4 - h2 6 3
2
x y
Thus, the points P and Q are + =1
3 3
Ê 3 ˆ Ê 3 ˆ fi x+y=3
ÁË h, 4 - h 2 ˜ and Á h, - 4 - h2 ˜
2 ¯ Ë 2 ¯ Hence, the value of h is 2.
26. The equation of P1 is y2 – 8x = 0 and P2 is y2 + 16x = 0
Let the tangents at P and Q meet at R(x1, 0).
Tangent to y2 = 8x passes through (–4, 0)
Therefore PQ is a chord of contact. So
2
0 = m1 (- 4) +
xx1 m1
=1
4 1
=2
4 m12
fi x=
x1
Also, tangent to y2 + 16x = 0 passes through (2, 0)
which is an equation of PQ at x = h. So
4
4 0 = m2 ¥ 2 -
h= m2
x1
m22 = 2
4
fi x1 = Ê 1 ˆ
h Hence, the value of Á 2 + m22 ˜
Ë m1 ¯
Now, D (h) = area of DPQR =2+2
1 = 4.
= ¥ PQ ¥ RT 27. For the given line, point of contact for
2
x2 y 2 Ê a 2 b2 ˆ
1 2 3 E1: 2 + 2 = 1 is Á , ˜
= ¥ ¥ 4 - h 2 ¥ ( x1 - h) a b Ë 3 3¯
2 2
x2 y 2 Ê B 2 A2 ˆ
3(4 + 2h 2 ) and for E2 : + 2 = 1 is Á ,
fi D ¢ (h) = ¥ 4 - h2 h B 2
A Ë 3 3 ˜¯
2
Point of contact of x + y = 3 and the circle is (1, 2)
which is always decreasing. Also, the general point on x + y = 3 can be taken as
45 5 1 Ê r r ˆ
D1 = Maximum of D (h) = at h = ÁË1 ,2 ˜
8 2 2 2¯
9 2 2
D 2 = Minimum of D (h) = at h = 1 where r =
2 3

CG_05.indd 67 2/6/2017 4:05:38 PM


5.68 Coordinate Geometry Booster

Ê 1 8ˆ Ê 5 4ˆ For orthocentre: One altitude is y = 0 (MN is per-


So, required points are Á , ˜ and Á , ˜ pendicular)
Ë 3 3¯ Ë 3 3¯
Other altitude is
Comparing with points of contact of ellipse
5 Ê 3ˆ
a2 = 5, B2 = 8 and b2 = 4, A2 = 1 ( y - 6) = Á x - ˜¯
2 6Ë 2
7 43
e1e2 = and e12 + e22 = Ê 9 ˆ
2 10 40 Hence, the orthocentre is Á - , 0˜
Ë 10 ¯
28. (i) (ii) Equation of tangent at M and N are
x y 6
± =1
6 8
Thus, R is (6, 0)
Equation of normal at M is
6Ê 3ˆ
( y - 6) = - Áx - ˜
2 Ë 2¯
Ê7 ˆ
Thus, Q is Á , 0˜
Ë2 ¯
1 5 5 6
ar ( DMQR) = ¥ 6¥ =
2 2 4
1 6 3 6 4 6
Here, e = , F1 = (-1, 0), F2 = (1, 0) ar ( MF1 NF2 ) = + =
3 2 2 2
The given parabola is y2 = 4x
5 6 4 6 5
Ê3 ˆ Ê3 ˆ Ratio = : =
Thus, M = Á , 6 ˜ and N = Á , - 6 ˜ 4 2 8
Ë2 ¯ Ë2 ¯

CG_05.indd 68 2/6/2017 4:05:38 PM


Hyperbola 6.1

C H A P T E R

6 Hyperbola

CONCEPT BOOSTER i.e. S¢P – SP = 2a

1. INTRODUCTION P

The word ‘hyperbola’ has ben derived from the Greek lan-
S¢ S
guage meaning ‘over-thrown’ or ‘excessive’, from which the
English term hyperbole is also derived. The term hyperbola is
believed to have been coined by Apollonius of Perga (c.262–
c.190 BC), who was a Greek geometer and astronomer noted
for his writings on conic sections. His innovative method- Definition 3
ology and terminology, especially in the field of the conic A conic section is said to be a hyperbola, if its eccentricity is
sections, the conics. According to him, hyperbola, the incli- more than 1, i.e. e > 1.
nation of the plane to the base of the cone exceeds that of the
side of the cone Definition 4
A conic
2. MATHEMATICAL DEFINITIONS ax2 + 2hxy + by2 + 2gx + 2fy + c = 0
represents a hyperbola if
Definition 1 (i) D π 0
It is the locus of a point which moves in a plane in such a way (ii) h2 – ab > 0,
that its distance from a fixed point (focus) to a fixed straight a h g
line (directrix) is constant (> 1), i.e.
where D = h b f
SP g f c
= e (> 1)
PM = abc + 2fgh – af2 – bg2 – ch2
M P Definition 5
The section of a double right circular cone and a plane is said
S (Focus) to be a hyperbola if the plane is parallel to the axis of a double
right circular cone.
Z (Directrix)

Definition 2
A hyperbola is a conic section defined as the locus of all
points P in the plane, the difference of whose distances from
two fixed points (the foci S and S¢) is a constant,

CG_06.indd 1 2/6/2017 4:06:15 PM


6.2 Coordinate Geometry Booster

Definition 6
x 2 (e 2 - 1) y2
Let z, z1 and z2 be three complex numbers such that fi - =1
a 2 (e 2 - 1) a 2 (e 2 - 1)
|z – z1| – |z – z2| = k,
where k < |z1 – z2| x2 y2
Then the locus of z is known as a hyperbola. fi - = 1,
a2 b2
where, b2 = a2(e2 – 1)
P(z)
which is the standard equation of a hyperbola.

S¢(z2) A¢ 2a A S(z1) 4. DEFINITION AND BASIC TERMINOLOGY OF THE


x2 y2
HYPERBOLA - =1
a2 b2

(i) Centre: C(0, 0). All chords passing through C and are
3. STANDARD EQUATION OF A HYPERBOLA bisected at C.
(ii) Vertices: A(a, 0) and A¢(–a, 0)
Y
(iii) Co-vertices: B(0, b) and B¢(0, –b)
B
(iv) Transverse axis: AA¢ = 2a
M P (v) Conjugate axis: BB¢ = 2b
b2
(vi) Eccentricity: e = 1 +
X¢ X a2
A¢ Z¢ C Z A N S (vii) Latus rectum: Any chord passing through the focus
and perpendicular to the axis is known as latus rectum.
B¢ 2b 2
The length of the latus rectum = .
a
Y¢ (viii) End-points of a latus recta: Let LL¢ and L1L¢1 be
Let ZM be the directrix, S be the focus and SZ be the per- two latus recta pass through the focus S(ae, 0) and
pendicular to the directrix. S¢(–ae, 0).
From the definition of hyperbola, we can write Ê b2 ˆ Ê b2 ˆ
Then, L Á ae, ˜ ; L ¢ Á ae, - ˜ ;
SA = e.AZ …(i) Ë a¯ Ë a¯
and SA¢ = eA¢Z …(ii) Ê b2 ˆ Ê b2 ˆ
Let the length of AA¢ = 2a and C be the mid-point of AA¢. L1 Á - ae, ˜ ; L1¢ Á - ae, - ˜
Ë a¯ Ë a¯
Adding Eqs (i) and (ii), we get
SA + SA¢ = e(AZ + A¢Z) Y
fi (CS – CA) + (CS + CA) = e(AZ + AA¢ – AZ) L1 L
fi 2CS = e(AA¢) = 2e
fi CS = ae …(iii)
X¢ X
Subtracting Eq. (i) from Eq. (ii), we get S¢ O S
(SA¢ – SA) = e(A¢Z – AZ)
fi AA¢ = e((CA¢ + CZ) – (CA – CZ))
L¢1
fi 2a = e ◊ 2CZ Y¢ L¢
a
fi CZ = …(iv) (ix) Equation of the latus recta: x = ±ae
e (x) Co-ordinates of Foci: S(ae, 0) and S¢(–ae, 0)
Let P(x, y) be any point on the curve and PM be the (xi) Distances between two foci (Focal length): 2ae
perpendicular to the directrix. a
Now from the definition of hyperbola, we get, (xii) Equation of directrices: x = ±
e
SP2 = e2 ◊ PM2
2
Ê ˆ 2a
2 2 2 ex - a 2 (xiii) Distance between the directrices:
fi ( x - ae) + y = e Á ˜ = (ex - a ) e
Ë e2 ¯
fi x2 – 2aex + a2e2 + y2 = (e2x2 – 2aex + a2) (xiv) Focal distances
fi x2(e2 – 1) – y2 = a2(e2 – 1) SP = ex – a and S¢P = ex + a
(xv) |S¢P – SP| = 2a

CG_06.indd 2 2/6/2017 4:06:17 PM


Hyperbola 6.3

Join NU.
P Let –UCN = j
Here P and U are the corresponding points of the hy-
S¢ S perbola and the auxiliary circle and f is the eccentric
angle of P, where 0 £ f < 2p.
Now, U = (a cos j, a sin j)
CN
Also, x = CN = ◊ CU = a.sec j
(xvi) Diameter: Any chord passing through the centre of the CU
hyperbola is known as the diameter. Thus the co-ordinates of P be (a sec f, y).
(xvii) Focal chord: Any chord passing through the focus is Since the point P lies on the hyperbola, so
called the focal chord.
a 2 sec 2j y2
Y P 2
- =1
a b2
y2
S
X fi sec 2j - =1

C b2

y2
Q fi 2
= sec 2j - 1 = tan 2j
b

fi y2 = b2 tan2j
(xviii) Auxiliary circle: Any circle is drawn with centre C
fi y = ±b tan j
and the transverse axis as a diameter is called the aux-
fi y = b tan j
iliary circle.
(since P lies in the first quadrant)
The equation of the auxiliary circle is given by
Hence, the parametric equations of the hyperbola are
x2 + y2 = a2
x = a sec j and y = b tan j.
Y (xx) Any point on the hyperbola can be considered as (a sec
j, b tan j).
(xxi) Eccentric angle: If two points P(a sec j, b tan j) and
U(a cos j, a sin j) are the corresponding points on the
X¢ X hyperbola and the auxiliary circle, then f is called the
A¢ O A
eccentric angle of the point P on the hyperbola, where
0 £ j < 2p.
Y

P
(xix) Parametrics equations: Let the auxiliary circle be
U
x2 + y2 = a2 and U (a cos j, b sin j) be any point on the
auxiliary circle. f
X¢ N X
O
Y

P
U

f

O N X (xxii) Conjugate hyperbola: Corresponding to every hyper-
bola, there exists a hyperbola in which the conjugate
and the transverse axes of one is equal to the transverse
and the conjugate axes of the other. Such types of hy-
Y¢ perbolas are known as the conjugate hyperbola.
The equation of the conjugate hyperbola to the
x2 y2 x2 y 2
Let P(x, y) be any point on the hyperbola - =1. hyperbola 2 - 2 = 1 is
a2 b2 a b
Draw PN perpendicular to x-axis
Let NU ba tangent to the auxiliary circle. x2 y2
- = -1
a2 b2

CG_06.indd 3 2/6/2017 4:06:17 PM


6.4 Coordinate Geometry Booster

(xxvii) Condition of a focal chord: The equation of


the chord joining the points P(a sec j1, b tan j1)
and Q(a sec j2, b tan j2) is
x Ê j - j2 ˆ y Ê j + j2 ˆ Ê j + j2 ˆ
cos Á 1 - sin Á 1 = cos Á 1
a Ë 2 ˜¯ b Ë 2 ˜¯ Ë 2 ˜¯
which is passing through the focus S(ae, 0). So
Ê j - j2 ˆ Ê j + j2 ˆ
e cos Á 1 = cos Á 1
Ë 2 ˜¯ Ë 2 ˜¯
It is also known as vertical hyperbola.
(xxiii) Rectangular or equilateral hyperbola: If the semi- Ê j - j2 ˆ
cos Á 1
transverse axis is equal to the semi-conjugate axis of Ë 2 ˜¯ 1
fi =
a hyperbola, i.e. a = b, then it is known as the rectan- Ê j1 + j 2 ˆ e
cos Á
gular hyperbola or the equilateral hyperbola. Ë 2 ˜¯
Y
Ê j - j2 ˆ Ê j + j2 ˆ
cos Á 1 - cos Á 1
B Ë 2 ˜¯ Ë 2 ˜¯ 1 - e
fi =
Ê j1 - j 2 ˆ Ê j1 + j 2 ˆ 1 + e
cos Á + cos Á
Ë 2 ˜¯ Ë 2 ˜¯
X¢ X
A¢ C A
Êj ˆ Ê j ˆ 1- e
fi tan Á 1 ˜ tan Á 2 ˜ =
B¢ Ë 2¯ Ë 2 ¯ 1+ e

Y¢ (xxviii) Condition of a focal chord with respect to


eccentricity (e): As we know that if the chord
The equation of a rectangular hyperbola is
passing through the focus, then
x2 – y2 = a2.
Clearly its eccentricity (e) Ê j - j2 ˆ Ê j + j2 ˆ
e cos Á 1 = cos Á 1
Ë 2 ˜¯ Ë 2 ˜¯
b2 a2
= 1+ = 1+ = 1+1 = 2 Ê j + j2 ˆ Ê j - j2 ˆ
a2 a2 fi e ¥ 2 sin Á 1 ˜ cos Á 1
Ë 2 ¯ Ë 2 ˜¯
(xxiv) Equation of a hyperbola whose axes are paral- Ê j + j2 ˆ Ê j + j2 ˆ
= 2 sin Á 1 cos Á 1
lel to the co-ordinate axes and the centre (h, k) is Ë 2 ˜¯ Ë 2 ˜¯
( x - h) 2 ( y - k ) 2
- =1, fi e ¥ (sin j1 + sin j2) = sin (j1 + j2)
a2 b2
where the foci are (h ± ae, k) and the directrix is sin(j1 + j 2 )
fi e=
a (sinj1 + sinj 2 )
x = h±
e which is the required condition.
Y (xxix) Rule to find out the centre of the hyperbola
P(x, y) If f(x, y) = 0 is the equation of a hyperbola, the
centre of the hyperbola is obtained by the rela-
C(h, k) df df
tion = 0 and = 0.
dx dy
X¢ X Let
O
f(x, y) = ax2 + 2hxy + by2 + 2gx + 2fy + c = 0
be the equation of a hyperbola.
df

Now, =0
dx
(xxvi) Equation of the chord joining the points P(j1) fi 2ax + 2hy + 2g = 0
and Q(j2): The equation of the chord joining the fi ax + y + g = 0 …(i)
points
df
P(a sec j1, b tan j1) and Q(a sec j2, b tan j2) is and =0
dy
x Ê j - j2 ˆ y Ê j + j2 ˆ Ê j + j2 ˆ fi
cos Á 1 - sin Á 1 = cos Á 1 2hx + 2by + 2f = 0
a Ë 2 ˜¯ b Ë 2 ˜¯ Ë 2 ˜¯ fi hx + by + f = 0 …(ii)

CG_06.indd 4 2/6/2017 4:06:18 PM


Hyperbola 6.5

Solving Eqs (i) and (ii), we get the required 6. INTERSECTION OF A LINE AND A HYPERBOLA
centre.
Y
(xxx) Polar form of a hyperbola
The equation of hyperbola is
x2 y2
=1 -
a 2 b2 X¢ X
C
Putting x = r cos q and y = r sin q, we get
+c
1 cos 2q sin 2q =m
x
= - y
r2 a2 b2 Y¢
2 2
2 a b a 2 (e 2 - 1) 2 2
fi r = = x y
b 2 cos 2q - a 2 sin 2q (e 2 cos 2q - 1) Let the hyperbola be 2
- =1 …(i)
a b2
(xxxi) Polar form of a hyperbola if centred at focus and the line be y = mx + c …(ii)
The equation of a hyperbola is From Eqs (i) and (ii), we get
x2 y2 x2 (mx + c) 2
2
- 2
=1 2
- =1
a b a b2
If centred C(0, 0) at the focus (ae, 0), then the fi b2x2 – a2(mx + c)2 = a2b2
polar form of the hyperbola becomes fi (a2m2 – b2)x2 + 2mca2x + a2(b2 + c2) = 0
a (e 2 - 1) Now,
r=
1 - e cosq D = 4m2c2a4 – 4a2(a2m2 – b2)(b2 + c2)
= 4(m2c2a4 – a4m2b2 – a4m2c2 + a2b4 + a2b2c2)
= 4(–a4m2b2 + a2b4 + a2b2c2)
= 4a2b2(b2 + c2 – a2m2)
r
q (i) The line y = mx + c will never intersect the hyper-
a (0, 0) bola, if
D<0
fi c2 < a2m2 – b2
(ii) The line y = mx + c will be a tangent to the hyper-
(xxxii) Polar form of a rectangular hyperbola bola if D = 0
The equation of rectangular hyperbola is fi c2 = a2m2 – b2
x2 – y2 = a2 This is known as the condition of tangency.
Putting x = r cos q and y = r sin q, we get, (iii) The line y = mx + c will intersect the hyperbola in
r2 cos 2q = a2 two real and distinct points, if
D>0
5. POSITION OF A POINT WITH RESPECT TO A HYPERBOLA fi c2 > a2m2 – b2
(iv) Any tangent to the hyperbola can be considered as
y = mx + a 2 m 2 - b 2 .
(v) Co-ordinates of the point of contact.
If the line y = mx + c be a tangent to the hyperbola
x2 y2
2
- = 1 , the co-ordinates of the point of contact is
a b2
Ê a2m b2 ˆ
Á± c , ± c ˜.
Ë ¯
which is also known as m-point on the hyperbola.
The point (x1, y1) lies outside, on or inside the hyperbola (vi) Number of tangents: If a point lies outside, on
and inside of a hyperbola, the number of tangents
x2 y2
- = 1 according as are 2, 1 and 0 respectively.
a2 b2 (vii) If the line lx + my + n = 0 be a tangent to the
x12 y12 x2 y 2
- - 1 < 0, = 0, > 0 hyperbola 2 - 2 = 1 , then
a2 b2 a b
a2l2 – b2m2 = n2

CG_06.indd 5 2/6/2017 4:06:19 PM


6.6 Coordinate Geometry Booster

7. DIFFERENT FORMS OF TANGENTS Y

(i) Point form: The equation of the tangent to the hy-


x2 y 2 P(h, k)
perbola 2 - 2 = 1 at the point (x1, y1) is
a b
xx1 yy1 X¢ X
- =1 O
a 2 b2
(ii) Parametric form: Equation of tangent to the hyper-
x2 y 2
bola 2 - 2 = 1 at (a sec js b tan j) is
a b Y¢
x y
sec j - cosecj = 1 Let it passes through the point (h, k).
a b Then,
(iii) The point of intersection of the tangents at P(q) and
k = mh + a 2 m 2 - b 2
x2 y 2
Q(j) on the hyperbola 2 - 2 = 1 is fi
a b (k - mh) 2 = ( a 2 m 2 - b 2 ) 2
Ê Ê q - j ˆ Êq +jˆ ˆ fi k2 + m2h2 – 2kmh = a2m2 – b2
Á a cos ÁË 2 ˜¯ b sin ÁË 2 ˜¯ ˜ fi (h2 – a2)m2 – 2khm + (k2 + b2) = 0
Á , ˜ It has two roots, say m1 and m2. Then
Á cos ÊÁ q + j ˆ˜ cos ÊÁ q + j ˆ˜ ˜
ÁË Ë 2 ¯ Ë 2 ¯ ¯ ˜ k 2 + b2
m1 ◊ m2 =
(iv) Slope form: The equation of the tangent to the hyper- h2 - a 2
x2 y 2 k 2 + b2
bola 2 - 2 = 1 in terms of the slope m is fi = -1
a b h2 - a 2
fi k2 + b2 = –h2 + a2
y = mx + a 2 m 2 - b 2
fi h2 + k2 = a2 – b2
The co-ordinates of the point of contact are Hence, the locus of (h, k) is
Ê a2m b2 ˆ x2 + y2 = a2 – b2
Á± 2 2 2

2 2 2 ˜.
Ë a m -b a m -b ¯
Notes
(v) The equation of the tangent to the conjugate hy- x2 y 2
1. The director circle of a hyperbola - = 1 exists
x2 y 2 a 2 b2
perbola 2 - 2 = -1 is
a b only when a > b.
2
y = mx + b - a m 2 2 2. If a < b, the equation of the director circle x2 + y2 =
a2 – b2 does not exist.
The co-ordinates of the point of contact are 3. The equation of the director circle to the conjugate
Ê a2m b2 ˆ x2 y 2
Á ± , ± ˜ hyperbola 2 - 2 = -1 is
Ë 2 2 2 2 2 2 a b
b -a m b -a m ¯
(vi) The line y = mx + c be a tangent to the hyperbola x2 + y2 = b2 – a2
It exists only when b > a.
x2 y 2
- = 1 , if 4. If b < a, the equation of the director circle does not
a 2 b2 exist.
c2 = a2m2 – b2
(vii) Director circle: The locus of the point of intersec- (viii) Pair of tangents
tion of two perpendicular tangents to a hyperbola Y
is known as the director circle of the hyperbola.
The equation of the director circle to the hyper-
x2 y 2
bola 2 - 2 = 1 is
a b X¢ X
x2 + y2 = a2 – b2 O
P(x1, y1)
The equation of any tangent to a hyperbola is
y = mx + a 2 m 2 - b 2 .

CG_06.indd 6 2/6/2017 4:06:20 PM


Hyperbola 6.7

The combined equation of the pair of tangents 9. CHORD OF CONTACT


drawn from a point P(x1, y1) lies outside of the hy-
If a tangent is drawn from a point P(x1, y1) to the hyperbola
x2 y 2 x2 y 2 x2 y 2
perbola 2 - 2 = 1 to the hyperbola 2 - 2 = 1 is - = 1 touching the hyperbola at Q and R, the equation
a b a b a 2 b2
of the chord of contact QR is
SS1 = T2
xx1 yy1
- =1
Ê x 2 y 2 ˆ Ê x12 y12 ˆ Ê xx1 yy1 ˆ 2 a 2 b2
fi Á 2 - 2 - 1˜ Á 2 - 2 - 1˜ = ÁË 2 - 2 - 1˜¯
Ëa b ¯Ëa b ¯ a b

8. DIFFERENT FORMS OF NORMALS

F1 F2
10. EQUATION OF THE CHORD BISECTED AT A POINT
x2 y 2
The equation of the chord of the hyperbola 2 - 2 = 1
(i) Point form: The equation of the normal to the hyper- a b
bisected at the point (x1, y1) is
x2 y 2 SS1 = T2
bola - = 1 at (x1, y1) is
a 2 b2 2
Ê x 2 y 2 ˆ Ê x 2 y 2 ˆ Ê xx yy ˆ
i.e. Á 2 - 2 - 1˜ Á 12 - 12 - 1˜ = Á 21 - 21 - 1˜
a 2 x b2 y Ëa b ¯Ë a b ¯ Ëa b ¯
+ = a 2 + b2
x1 y1
Y
(ii) Parametric form: The equation of the normal to the B
x2 y 2
hyperbola 2 - 2 = 1 at (a sec j, b tan j) is
a b
M(x1, y1)
ax cos j + by cot j = a2 + b2 X¢ X
C
(iii) Slope form: The equation of the normal to the hyper-
x2 y 2 A
bola - = 1 in terms of the slope m is
a 2 b2 Y¢
m( a 2 + b 2 )
y = mx
a 2 - m 2b 2 11. POLE AND POLAR
The co-ordinates of the point of contact are Y
Ê ˆ Q
a2 mb 2 B
Á ± , ˜
Ë a 2 - b2 m2 a 2 - b2m2 ¯
A
X¢ X
O
(iv) The line y = mx + c will be a normal to the hyperbola if P(x1, y1) Polar

2Ê m2 (a 2 + b 2 )2 ˆ
c =Á 2 ˜
Ë ( a - m 2b 2 ) ¯ Y¢ R

which is also known as the condition of the normalcy
to a hyperbola. Let P be any point inside or outside of the hyperbola. Sup-
pose any straight line through P intersects the hyperbola at

CG_06.indd 7 2/6/2017 4:06:22 PM


6.8 Coordinate Geometry Booster

A and B. Then the locus of the point of intersection of the


b2 x
tangents to the hyperbola at A and B is called the polar of the Hence, the locus of the mid-point is y = .
given point P with respect to the hyperbola and the point P is a2m
called the pole of the polar. 13. CONJUGATE DIAMETERS
The equation of the polar from a point (x1, y1) to the
Two diameters are said to be conjugate when each bisects all
x2 y 2 xx yy the chords parallel to the others.
hyperbola 2 - 2 = 1 is 21 - 21 = 1 Y
a b a b
Properties related to pole and polar
(i) The polar of the focus is the directrix.
(ii) Any tangent is the polar of the point of contact.
(iii) The pole of a line lx + my + n = 0 with respect to the

x2 y 2 X
ellipse - = 1 is
a 2 b2
Ê a 2l b 2 m ˆ
Á- n , - n ˜
Ë ¯ Y¢
(iv) The pole of a given line is same as point of intersection If y = m1x and y = m2x be two conjugate diameters,
then,
of tangents at its extremities.
b2
(v) If the polar of P(x1, y1) passes through Q(x2, y2), the m1 ◊ m2 = 2
polar of Q(x2, y2) goes through P(x1, y1) and such points a
are said to be conjugate points. Properties of diameters
(vi) If the pole of a line lx + my + n = 0 lies on the another Property-I
line l¢x + m¢y + n¢ = 0, the pole of the second line will If a pair of the diameters be conjugate with respect to a
lie on the first and such lines are said to be conjugate hyperbola, they are conjugate with respect to its conjugate
lines. hyperbola also.
Property-II
12. DIAMETER The parallelogram formed by the tangents at the extremities
The locus of the mid-points of a system of parallel chords of of the conjugate diameters of a hyperbola has its vertices ly-
a hyperbola is called a diameter of the hyperbola. ing on the asymptotes and is of conjugate area.
Y Property-III
P(x1, y1) If the normal at P meets the transverse axis in G, then
SG = e.SP
R(h, k)
Also, the tangent and the normal bisects the angle
C between the focal distances of P.
X¢ A
A
Q( Property-IV
x1 ,
y1 ) If a pair of conjugate diameters meet the hyperbola in P and
P¢ and its conjugate in D and D, then the asymptotes bisect
Y¢ PD, P¢D ◊ PD¢ and P¢D¢.
x2 y 2
The equation of a diameter to a hyperbola - = 1 is 14. ASYMPTOTES
a 2 b2
b2 Asymptote
y= x Asymptote
a2m b
y= a x Y b
y= a x
Let (h, k) be the mid-point of the chord y = mx + c of
x2 y 2 x2 – y2 –1
the hyperbola 2 - 2 = 1 . b
a b a2 b2
Then, T = S1 –a a a x
2 2
xh yk h k –b
fi 2
- 2 -1 = 2 - 2 -1
a b a b
b2h
Slope =
a2k

CG_06.indd 8 2/6/2017 4:06:22 PM


Hyperbola 6.9

An asymptotes of any hyperbola or a curve is a straight line be the equations of the hyperbola, its asymptote and its
which touches it in two points at infinity. conjugate, respectively, we can write from the above
The equation of the asymptotes of the hyperbola equations,
2
x y2 C + H = 2A
2
- 2 = 1 are
a b
15. RECTANGULAR HYPERBOLA
b
y=± x A hyperbola is said to be rectangular, if the angle between its
a
asymptote is 90°.
As we know that the difference between the 2nd de-
Ê bˆ
gree curve and pair of asymptotes is constant. Thus, 2 tan -1 Á ˜ = 90°
Ë a¯
x2 y 2
Given hyperbola is 2 - 2 = 1
a b Ê bˆ
fi tan -1 Á ˜ = 45°
The equation of the pair of asymptotes are Ë a¯
x2 y 2 b
- +l =0 …(i) fi = tan 45°
a 2 b2 a
Equation (i) represents a pair of straight lines, then fi b=a
D=0 Hence, the equation of the rectangular hyperbola is
1 Ê 1ˆ x2 – y2 = a2
fi Á - ˜ ◊l + 0 - 0 - 0 - l ◊0 = 0 The eccentricity (e) of the rectangular hyperbola is
a 2 Ë b2 ¯
fi l=0 a2
x2 y2 e = 1+ = 1+1 = 2
From Eq. (i) we get the equation of asymptotes as - = 0. a2
a2 b2
b
fi y=± x 16. RECTANGULAR HYPERBOLA XY = C2
a
The equation of a rectangular hyperbola is x2 – y2 = a2 and its
14.1 Important Points Related to Asymptotes asymptote are
(i) The asymptotes pass through the centre of the hyper- x – y = 0 and x + y = 0,
bola. where the asymptotes are inclined at 45° and 135°,
(ii) A hyperbola and its conjugate have the same asymp- respectively
totes. If we rotate the axes through an angle of 45° in clockwise
(iii) The equation of the hyperbola and its asymptotes differ direction without changing the origin, then we replace x by
by a constant only. [x cos (–45°) – y sin (–45°)] and y by [x sin (–45°) + y cos
(iv) The equation of the asymptotes of a rectangular hyper- (–45°)],
bola x2 – y2 = a2 are y = ±x Ê x + yˆ Ê -x + yˆ
i.e. x by Á and y by Á ˜
(v) The angle between the asymptotes of the hyperbola Ë 2 ˜¯ Ë 2 ¯
x2 y 2 Then the equation, x2 – y2 = a2 reduces to
- = 1 is
a 2 b2 Ê x + yˆ
2
Ê -x + yˆ
2
2
Ê bˆ ÁË ˜¯ - ÁË ˜ =a
2 tan -1 Á ˜ 2 2 ¯
Ë a¯
1
fi (2xy + 2xy ) = a 2
(vi) The bisectors of the angles between the asymptotes are 2
the co-ordinate axes.
a2
(vii) No tangent to the hyperbola can be drawn from its cen- fi xy = = c 2 (say)
tre. 2
(viii) Only one tangent to the hyperbola can be drawn from a fi xy = c2
point lying on its asymptotes other than its centre. y
y=x
(ix) Two tangents can be drawn to the hyperbola from any
of its external points which does not lie at its asymp-
totes.
x2 y 2 x2 y 2
(x) Let H: - - 1 = 0 , A: - = 0 and x
a 2 b2 a 2 b2
x2 y 2
C: - +1 = 0
a 2 b2 v = –x

CG_06.indd 9 2/6/2017 4:06:23 PM


6.10 Coordinate Geometry Booster

x2 = y1 = 1 xy = 1 Ê cˆ
(vi) The equation of the tangent at Á ct , ˜ to the hyperbola
Ë t¯
q x
F1 F2 O xy = c2 is + yt = 2 c .
t
(vii) The equation of the normal at (x1, y1) to the hyperbola
xy = c2 is
xx1 - yy1 = x12 - y12
(viii) The equation of the normal at t to the hyperbola xy = c2
Rectangular hyperbola
is
xt3 – yt – ct4 + c = 0
Properties of rectangular hyperbola
(i) The asymptotes of the rectangular hyperbola xy = c2 are 17. REFLECTION PROPERTY OF A HYPERBOLA
x = 0 and y = 0
(ii) The parametric equation of the rectangular hyperbola If an incoming light ray passing through one focus, after
c striking the convex side of the hyperbola, it will get reflected
xy = c2 are x = ct and y = . towards other focus.
t
(iii) Any point on the rectangular hyperbola xy = c2 can be Y
Ê cˆ
considered as Á ct , ˜ .
Ë t¯
(iv) The equation of the chord joining the points t1 and t2 is
x + yt1t2 – c(t1 + t2) = 0 X¢ X
(v) The equation of the tangent to the rectangular hyperbola S¢ C S
xy = c2 at (x1, y1) is
xy1 + x1y = c2

E XERCISES

LEVEL I 5. Find the equation of the hyperbola, whose one focus is


(2, 1), the directrix is x + 2y = 1 and the eccentricity is
(Problems Based on Fundamentals) 2.
6. Find the equation of the hyperbola, whose distance be-
ABC OF HYPERBOLA
tween foci is 16 and the eccentricity is 2 .
1. Find the centre, the vertices, the co-vertices, the length
7. Find the equation of the hyperbola,whose foci are (6, 4)
of transverse axis, the conjugate axis and the latus rec-
and (–6, 4) and the eccentricity is 2.
tum, the eccentricity, the foci and the equation of direc-
8. Find the eccentricity of the hyperbola whose latus rec-
trices of each of the following hyperbolas.
tum is half of its transverse axis.
x2 y 2 9. If e1 and e2 be the eccentricities of a hyperbola and its
(i) - =1
9 4 1 1
conjugate, prove that 2 + 2 = 1 .
x2 y 2 e1 e2
(ii) - = -1 10. An ellipse and a hyperbola are confocal (have the same
16 9
focus) and the conjugate axis of the hyperbola is equal
(iii) 9x2 – 16y2 – 36x + 96y – 252 = 0 to the minor axis of the ellipse. If e1 and e2 are the ec-
2. Find the equation of the hyperbola, whose centre is centricities of the ellipse and the hyperbola, prove that
(1, 0), one focus is (6, 0) and the length of transverse
e12 + e22 = 2 .
axis is 6.
3. Find the equation of the hyperbola, whose centre is 11. Find the centre of the hyperbola
(3, 2), one focus is (5, 2) and one vertex is (4, 2). 4(2y – x – 3)2 – 9(2x + y – 1)2 = 80
4. Find the equation of the hyperbola, whose centre is 12. Find the centre of the hyperbola
(–3, 2), one vertex is (–3, 4) and eccentricity is 5/2. 3x2 – 5y2 – 6x + 20y – 32 = 0

CG_06.indd 10 2/6/2017 4:06:24 PM


Hyperbola 6.11

13. Prove that the straight lines TANGENT AND TANGENCY


x y x y 1
- = 2013 and + = 30. Find the equation of the tangent to the curve x2 – y2 – 8x
a b a b 2013 + 2y + 11 = 0 at (2, 1).
always meet on a hyperbola. 31. Find the equation of the tangent to the curve 4x2 – 3y2 =
Ê1+ t2 ˆ 24 at y = 2.
14. Prove that the locus represented by x = 3 Á ˜ and
Ë1- t2 ¯ 32. Find the angle between the tangents to the curve
4t 9x2 – 16y2 = 144 drawn from the point (4, 3).
y= 2 is a hyperbola.
t -1 33. Find the equations of the common tangent to the curves
1
15. Prove that the locus represented by x = (et + e - t ) x2 y 2 x2 y 2
2 - = 1 and - = -1 .
1 t a 2 b2 b2 a 2
-t
and y = (e - e ) is a hyperbola. 34. Find the equations of the common tangents to the
2
x2 y 2 x2 y 2
x2 y2 curves - = 1 and + = 1.
16. If the equation + = 1 represents a 9 4 9 4
2014 - l 2013 - l
35. Find the equation of the common tangents to the curves
hyperbola, find l.
x2 y 2
x2 y 2 - = 1 and x2 + y2 = 9.
17. If the foci of the ellipse + = 1 and the hyperbola 16 9
16 b 2
36. Find the equation of the common tangents to the curves
x2 y 2 1
- = coincide, find the value of b2. x2 y 2
144 81 25 y2 = 8x and - = 1.
9 5
18. If the latus rectum subtends right angle at the centre of
37. Find the locus of the point of intersection of the perpen-
x2 y 2
the hyperbola 2 - 2 = 1 , find its eccentricity. x2 y 2
a b dicular tangents to the curve - = 1.
16 9
19. Find the location of point (1, 4) w.r.t the hyperbola
38. Find the product of the perpendiculars from foci upon
2x2 – 3y2 = 6.
x2 y 2
20. If (l, –1) is an exterior point of the curve 4x2 – 3y2 = any tangent to the hyperbola 2 - 2 = 1 .
1 such that the length of the interval where l lies is m, a b
2
find the value of m + 10. 39. If the tangent to the parabola y = 4ax intersects the
x2 y 2
INTERSECTIONS OF A LINE AND A HYPERBOLA hyperbola 2 - 2 = 1 at A and B respectively, find the
a b
21. Find the points common to the hyperbola 25x2 – 9y2 locus of the points of intersection of tangents at A and B.
= 225 and the straight line 25x + 12y – 45 = 0.
22. For what value of l, does the line y = 3x + l touch the NORMAL AND NORMALCY
hyperbola 9x2 – 5y2 = 45? x2 y 2
23. For all real values of m, the straight line 40. Find the equation of the normal to the curve - =1
16 9
y = mx + 9m 2 - 4 is a tangent to a hyperbola, find at (8, 3 3) .
the equation of the hyperbola. 41. A normal is drawn at one end of the latus rectum of the
24. Find the equations of tangents to the curve 4x2 – 9y2 = x2 y 2
hyperbola 2 - 2 = 1 which meets the axes at points
36, which is parallel to 5x – 4y + 7 = 0. a b
25. Find the equations of tangents to the curve 9x2 – 16y2 = A and B respectively. Find the area of the DOAB.
144, which is perpendicular to the straight line 3x + 4y 42. Prove that the locus of the foot of the perpendicular
+ 10 = 0. from the centre upon any normal to the hyperbola
26. If the line 5x + 12y – 9 = 0 touches the hyperbola x2 y 2
x2 – 9y2 = 9, then find its point of contact. - = 1 is
27. Find the equation of tangents to the curve 4x2 – 9y2 = 36 a 2 b2
from the point (3, 2). (x2 + y2)(a2y2 – b2x2) = (a2 + b2)2x2y2
28. Find the number of tangents from the point (1, –2) to
x2 y 2
the curve 2x2 – 3y2 = 12. 43. A normal to the hyperbola - = 1 meets the axes
29. Find the equation of the tangent to the curve 3x2 – 4y2 = a 2 b2
12 having slope 4. in M and N and the lines MP and NP are drawn perpen-
diculars to the axes meeting at P. Prove that the locus
of P is the hyperbola a2x2 – b2y2 = (a2 + b2)2.

CG_06.indd 11 2/6/2017 4:06:25 PM


6.12 Coordinate Geometry Booster

x2 y 2 x2 y 2
44. If the normal at j on the hyperbola - = 1 meets 52. A tangent to the hyperbola 2 - 2 = 1 cuts the ellipse
a 2 b2 a b
x2 y 2
the transverse axis at G such that + = 1 in points P and Q. Find the locus of the
AG ◊ A¢G = am(en secp q – 1), a 2 b2
where A, A¢ are the vertices of the hyperbola and m, n mid-point PQ.
and p are positive integers, find the value of (m + n + 53. Chords of the hyperbola x2 – y2 = a2 touch the parabola
p)2 + 36. y2 = 4ax. Prove that the locus of their mid-points is the
45. If the normals at (xi, yi), i = 1, 2, 3, 4 on the rectangular curve y2(x – a) = x3.
x2 y 2
hyperbola xy = c2 meet at the point (a, b), prove that 54. A variable chord of the hyperbola 2 - 2 = 1 is a tan-
(i) x1 + x2 + x3 + x4 = a a b
(ii) y1 + y2 + y3 + y4 = b gent to the circle x2 + y2 = c2. Prove that the locus of its
2 2 2 2 2 mid-points is
(iii) x1 + x2 + x3 + x4 = a 2
Ê x2 y 2 ˆ 2Ê x
2
y2 ˆ
y12 y22 y32 y42 Á 2 - 2˜ =c Á 4 + 4˜
2
(iv) + + + =b
Ëa b ¯ Ëa b ¯
(v) x1 ◊ x2 ◊ x3 ◊ x4 = –c4
(vi) y1 ◊ y2 ◊ y3 ◊ y4 = –c4. 55. A variable chord of the circle x2 + y2 = a2 touches the
46. If the normals at (xi, yi), i = 1, 2, 3, 4 on the hyperbola x2 y 2
hyperbola 2 - 2 = 1 . Prove that the locus of its mid-
x2 y 2 a b
- = 1 are concurrent, prove that
a 2 b2 points is (x2 + y2)2 = a2x2 – b2y2.
Ê1 1 1 1ˆ 56. A tangent to the parabola y2 = 4ax meets the hyperbola
(i) ( x1 + x2 + x3 + x4 ) Á + + + ˜ = 4 xy = c2 in two points P and Q. Prove that the locus of
Ë x1 x2 x3 x4 ¯ the mid-point of PQ lies on a parabola.
Ê1 1 1 1ˆ 57. From a point P, tangents are drawn to the circle x2 +
(ii) ( y1 + y2 + y3 + y4 ) Á y +
y
+ + ˜ =4
y y
y2 = a2. If the chord of contact of the circle is a normal
Ë 1 2 3 4¯
x2 y 2
chord of a hyperbola 2 - 2 = 1 , prove that the locus
a b
CHORD OF CONTACT/CHORD BISECTED AT A POINT 2
2
Êa b ˆ Ê a 2 + b2 ˆ
2
47. Find the equation of the chord of contact of tangents of the point P is Á 2 - 2 ˜ = Á ˜ .
Ëx y ¯ Ë a2 ¯
x2 y 2
from the point (2, 3) to the hyperbola - = 1. 58. Prove that the locus of the mid-points of the focal chord
9 4
48. Find the locus of the mid-points of the portions of the x2 y 2 x 2 y 2 ex
of the hyperbola 2 - 2 = 1 is 2 - 2 = .
x2 y 2 a b a b a
tangents to the hyperbola - = 1 included be- 59. If the chords of contact of tangents from two points
9 4
tween the axes. x2 y 2
49. From the points on the circle x2 + y2 = a2, tangents are (x1, y1) and (x2, y2) to the hyperbola 2 - 2 = 1 are
a b
drawn to the hyperbola x2 – y2 = a2. Prove that the locus
x1 x2 am
of the mid-points of the chord of contact is at right angles such that = - n , where m, n are
(x2 – y2)2 = a2(x2 + y2) y1 y2 b
50. Prove that the locus of the mid-points of the hyperbola 10
Ê m + nˆ
positive integers, find the value of Á
Ë 4 ˜¯
x2 y 2 .
- = 1 which subtend right angle at the centre is
a 2 b2
2 POLE AND POLAR
Ê x2 y 2 ˆ Ê 1 1 ˆ Ê x2 y 2 ˆ
Á 2 - 2 ˜ ÁË 2 - 2 ˜¯ = Á 4 + 4 ˜ 60. Find the polar of the focus (–ae, 0) with respect to the
Ëa b ¯ a b Ëa b ¯
x2 y 2
51. Tangents are drawn from a point P to the parabola hyperbola 2 - 2 = 1 .
a b
y2 = 4ax. If the chord of contact of the parabola be a
61. If the polars of (x1, y1) and (x2, y2) with respect to the
x2 y 2
tangent to the hyperbola 2 - 2 = 1 , find the locus
a b x2 y 2
hyperbola - = 1 are at right angles, prove that
of the point P. a 2 b2
x1 x2 a 4
+ = 0.
y1 y2 b 4

CG_06.indd 12 2/6/2017 4:06:26 PM


Hyperbola 6.13

62. Find the pole of the line x – y = 3 w.r.t. the hyperbola


x2 y 2
x2 – 3y2 = 3. 77. Let P be a variable point on the hyperbola - =1
63. Prove that the locus of the poles of the normal chords a 2 b2
such that its distance from the transverse axis is equal
x2 y 2
with respect to the hyperbola - = 1 is the curve to its distance from an asymptote to the given hyper-
a 2 b2 bola. Prove that the locus of P is (x2 – y2)2 = 4x2(x2 – a2).
y2a6 – x2b6 = (a2 + b2)2x2y2. 78. Show that the tangent at any point of a hyperbola cuts
64. Prove that the locus of the poles with respect to the off a triangle of constant area from the asymptotes and
parabola y2 = 4ax of the tangent to the hyperbola x2 – y2 that the portion of it intercepted between the asymp-
= a2 is the ellipse 4x2 + y2 = 4a2. totes is bisected at the point of contact.
65. Prove that the locus of the pole with respect to the 79. If p1 and p2 are the perpendiculars from any point on
x2 y 2 x2 y 2
hyperbola 2 - 2 = 1 of any tangent to the circle,
a b the hyperbola 2 - 2 = 1 on its asymptotes, prove
a b
whose diameter is the line joining the foci, is the ellipse
1 1 1
x2 y 2 1 that = + .
+ = . p1 p2 a 2 b 2
a 4 b4 a 2 + b2
80. If the normal at t1 to the hyperbola xy = c2 meets it again
DIAMETER
at t2, prove that t13t2 = - 1 .
81. A triangle has its vertices on a rectangular hyperbola.
66. Prove that the equation of the diameter which bisects
Prove that the orthocentre of the triangle also lies on
x2 y 2 the same hyperbola.
the chord 7x + y – 2 = 0 of the hyperbola - =1
is x + 3y = 0. 3 7 82. Find the locus of the poles of the normal chords of the
rectangular hyperbola xy = c2
67. Find the equation of the diameter of the hyperbola 83. If the angle between the asymptote is 2a, prove that the
eccentricity of the hyperbola is sec a.
x2 y 2
- = 1 , which corresponds the line 3x + 4y + 10 84. A circle cuts the rectangular hyperbola xy = 1 in points
9 4 (xr, yr), r = 1, 2, 3, 4, prove that x1x2x3x4 = 1 and y1y2y3y4
= 0. =1.
68. Find the equation of the diameter to the hyperbola 85. If the tangent and the normal to a rectangular hyperbola
x2 y 2 xy = c2 at a point cuts off intercepts a1 and a2 on one
- = 1 parallel to the chord 2x + 3y + 5 = 0. axis and b1, b2 on the other axis, prove that a1a2 + b1b2
9 4
69. In the hyperbola 16x2 – 9y2 = 144, find the equation of = 0.
the diameter which is conjugate to the diameter whose 86. If e1 and e2 be the eccentricities of the hyperbola
equation is x = 2y. xy = c2 and x2 – y2 = a2, find the value of (e1 + e2)2.
87. Find the product of the lengths of the perpendiculars
ASYMPTOTES x2
drawn from any point on the hyperbola - y 2 = 1 to
70. Find the asymptotes of the curve xy – 3y – 2x = 0. 2
71. Find the equations of the asymptotes of the curve its asymptote.
(a sec j, a tan j). 88. If A, B and C be three points on the rectangular hyper-
72. Find the eccentricity of the hyperbola whose asymp- bola xy = c2, find
totes are 3x + 4y = 10 and 4x – 3y = 5. (i) the area of the DABC
73. Find the equation of a hyperbola whose asymptotes are (ii) the area of the triangle formed by the tangents at
2x – y = 3 and 3x + y = 7 and which pass through the A, B and C.
point (1, 1). 89. Find the length of the transverse axis of the rectangular
74. The asymptotes of a hyperbola having centre at the hyperbola xy = 18.
point (1, 2) are parallel to the lines 2x + 3y = 0 and 90. Prove that the locus of a point whose chord of contact
3x + 2y = 0. If the hyperbola passes through the point with respect to the circle x2 + y2 = 4 is a tangent to the
(5, 3), prove that its equation is (2x + 3y – 8)(3x + 2y hyperbola xy = 1 is a hyperbola.
– 7) – 154 = 0. 91. Find the asymptotes of the hyperbola xy = hx + ky.
75. Find the product of the lengths of the perpendiculars x2 y 2
92. If e be the eccentricity of the hyperbola 2 - 2 = 1
from any point on the hyperbola x2 – 2y2 = 2 to its as- a b
ymptotes. and q is the angle between the asymptotes, find cos (q/2).
76. Find the area of the triangle formed by any tangent to 93. A ray is emanating from the point (5, 0) is incident on
x2 y 2 the hyperbola 9x2 – 16y2 = 144 at a point P with ab-
the hyperbola - = 1 and its asymptotes. scissa 8. Find the equation of the reflected ray after first
9 4
reflection and point P lies in first quadrant.

CG_06.indd 13 2/6/2017 4:06:26 PM


6.14 Coordinate Geometry Booster

94. A ray is coming along the line 2x – y + 3 = 0 (not 7. If the eccentricity of the hyperbola x2 – y2 sec2a = 5 is
x2 y 2 3 times the eccentricity of the ellipse x2 sec2a + y2 =
through the focus) to the hyperbola - = 1.
4 3 5, the value of a is
After striking the hyperbolic mirror, it is reflected (a) p/6 (b) p/4 (c) p/3 (d) p/2
(not through the other focus). 8. For all real values of m, the straight line
Find the equation of the line containing the reflected y = mx + 9m 2 - 4 is a tangent to the curve
ray.
(a) 9x2 + 4y2 = 36 (b) 4x2 + 9y2 = 36
(c) 9x2 – 4y2 = 36 (d) 4x2 – 9y2 = 36
LEVEL II 9. The foci of the ellipse
x2 y 2
+ = 1 and the hyperbola
(Mixed Problems) 16 b 2
1. The magnitude of the gradient of the tangent at extrem- x2 y 2 1
- = coincide. The value of b2 is
x2 y 2 144 81 25
ity latus rectum of the hyperbola 2 - 2 = 1 is equal to
a b (a) 5 (b) 7 (c) 9 (d) 4
(a) be (b) e (c) ab (d) ae 10. The locus of the mid-points of the parallel chords with
2. The eccentricity of the hyperbola conjugate to the hy- gradient m of the rectangular hyperbola xy = c2 is
x2 y 2 (a) mx + y = 0 (b) y – mx = 0
perbola - = 1 is (c) my – x = 0 (d) my + x = 0
4 12
11. The locus of the foot of the perpendicular from the cen-
2 4
(a) (b) 2 (c) 3 (d) tre of the hyperbola xy = c2 on a variable tangent is
3 3 (a) (x2 – y2)2 = 4c2xy (b) (x2 + y2)2 = 2c2xy
x2 y 2 (c) (x2 + y2) = 4c2xy (d) (x2 + y2)2 = 4c2xy
3. The asymptote of the hyperbola - = 1 form with
a 2 b2 x2 y 2
12. P is a point on the hyperbola 2 - 2 = 1 , N is the foot
any tangent to the hyperbola a triangle whose area is a b
a2 tan l in magnitude, its eccentricity is of the perpendicular from P on the transverse axis. The
(a) sec l (b) cosec l tangent to the hyperbola at P meets transverse axis at T.
(c) sec2 l (d) cosec2 l If O be the centre of the hyperbola, OT.ON is
(a) e2 (b) a2 (c) b2 (d) b2/a2
x2 y2
4. The equation + = 1 ( p π 4, 9) represents 13. If PN be the perpendicular from a point on the rectan-
29 - p 4 - p gular hyperbola (x2 – y2) = a2 on any on its asymptotes,
(a) an ellipse if p is any constant greater than 4 the locus of the mid-point of PN is a/an
(b) a hyperbola if p is any constant between 4 and 29 (a) circle (b) parabola
(c) a rectangular hyperbola if p is any constant greater (c) ellipse (d) hyperbola
than 29 14. The equation to the chord joining two points (x1, y1) and
(d) no real curve if p is less than 29. (x2, y2) on the rectangular hyperbola xy = c2 is
5. Locus of the feet of the perpendiculars drawn from
x y x y
either foci on a variable tangent to the hyperbola (a) + = 1 (b) + =1
16y2 – 9x2 = 1 x1 + x2 y1 + y2 x1 - x2 y1 - y2
1
(a) x2 + y2 = 9 (b) x 2 + y 2 = x y x y
9 (c) + = 1 (d) + =1
y1 + y2 x1 + x2 y1 - y2 x1 - x2
7 1
(c) x 2 + y 2 = (d) x 2 + y 2 = 15. If P(x1, y1), Q(x2, y2), R (x3, y3) and S(x4, y4) are 4 con-
144 16
cyclic points on the rectangular hyperbola xy = c2, the
6. The locus of the point of intersection of the lines co-ordinates of the orthocentre of DPQR is
(a) (x4 – y4) (b) (x4, y4)
3 x - y = 4 3t = 0
(c) (–x4, –y4) (d) (–x4, y4)
3 tx + ty - 4 3 = 0 16. The chord PQ of the rectangular hyperbola xy = a2
and
meets the axis of x at A, C is the mid-point of PQ and O
(where t is a parameter) is a hyperbola, whose eccen- the origin. The DACO is a/an
tricity is (a) equilateral (b) isosceles triangle
(a) 3 (b) 2 (c) 2/ 3 (d) 4/3 (c) right angled D (d) right isosceles triangle.
17. A conic passes through the point (2, 4) and is such that
the segment of any of its tangents at any point con-

CG_06.indd 14 2/6/2017 4:06:27 PM


Hyperbola 6.15

tained between the co-ordinates is bisected at the point the equation of the circle through the points of intersec-
of tangency. The foci of the conic are tion of two conics is
(a) x2 + y2 = 5
(a) (2 2, 0) and (— 2 2, 0)
(b) 5(x 2 + y 2 ) - 3x - 4y = 0
(b) (2 2, 2 2) and (—2 2, - 2 2)
(c) 5( x 2 + y 2 ) + 3x + 4y = 0
(c) (4, 4) and (– 4, – 4)
(d) x2 + y2 = 25
(d) (4 2, 4 2) and (—4 2, —4 2) 26. At the point of intersection of the rectangular hyper-
18. The latus rectum of the conic satisfying the differential bola xy = c2 and the parabola y2 = 4ax, tangents to the
equation, xdy + ydx = 0 and passing through the point rectangular hyperbola and the parabola make angles
(2, 8) is q and j, respectively with the axis of x, then
(a) q = tan–1(–2 tan j) (b) j = tan–1(–2 tan j)
(a) 4 2 (b) 8 (c) 8 2 (d) 16
1 1
19. If the normal to the rectangular hyperbola xy = c2 at the (c) q = tan -1 ( - tanj ) (d) j = tan -1 ( - tanq )
2 2
point t meets the curve again at t1, the value of t3t1 is 27. The area of the quadrilateral formed with the foci of the
(a) 1 (b) –1 (c) 0 (d) None
x2 y 2 x2 y 2
x 2
y 2 hyperbola - = 1 and - = -1 is
20. With one focus of the hyperbola - = 1 as the a 2 b2 a 2 b2
9 16
(a) 4a2 + b2) (b) 2(a2 + b3)
centre, a circle is drawn which is the tangent to the
1 2
(c) (a2 + b2) (d) (a + b 2 )
hyperbola with no part of the circle being outside the 2
hyperbola. The radius of the circle is
28 The eccentricity of the hyperbola whose latus rectum
(a) < 2 (b) 2 (c) 11/3 (d) None
is 8 and the conjugate axis is equal to half the distance
x2 y 2 between the foci is
21. AB is a double ordinate of the hyperbola 2 - 2 = 1
a b 4 4 2
(a) (b) (c) (d) None.
such that DAOB (where O is the origin) is an equilateral 3 3 3
triangle, the eccentricity of the hyperbola satisfies
29. If P( 2 secq , 2 tanq ) is a point on the hyperbola
2
(a) e > 3 (b) 1 < e < whose distance from the origin is 6 , where P is in
3 the first quadrant, then q is equal to
2 2 p p p
(c) e = (d) e > (a)
4
(b)
3
(c)
6
(d) None
3 3
30. An ellipse and a hyperbola have same centre origin, the
22. If the product of the perpendicular distances from any same foci and the minor axis of the one is the same as
point on the hyperbola of the eccentricity from its as- the conjugate axis of the other. If e1 and e2 be their ec-
ymptotes is equal to 6, the length of the transverse axis 1 1
of the hyperbola is centricities, respectively, then 2 + 2 is equal to
(a) 3 (b) 6 (c) 8 (d) 12 e1 e2
(a) 1 (b) 2 (c) 3 (d) None
23. If x + iy = j + iy where i = -1 and j and y are 31. The number of possible tangents can be drawn to the
non-zero real parameters, then j = constant and y =
curve 4x2 9y2 = 36, which are perpendicular to the
constant, represents two systems of rectangular hyper-
straight line 5x + 2y – 10 = 0 is
bola which intersect at an angle of
(a) p/6 (b) p/3 (c) p/4 (d) p/2 (a) 0 (b) 1 (c) 2 (d) 4
24. The tangent to the hyperbola xy = c2 at the point P in- 32. The equation of a tangent passing through (2, 8) to the
tersects the x-axis at T and the y-axis at T ¢. The normal hyperbola 5x2 – y2 = 5 is
to the hyperbola at P intersects the x-axis at N and the (a) 3x – y + 2 = 0 (b) 3x + y – 14 = 0
y-axis at N¢. The area of DPNT and PN¢T ¢ are D and D¢ (c) 23x – 3y – 2 2 = 0 (d) 3x – 23y + 178 = 0.
1 1 33. If P(x1, y1), Q(x2, y2), R(x3, y3 and (Sx4, y4) are four con-
respectively, then + is
D D¢ cyclic points on the rectangular hyperbola xy = c2, the
(a) equal to 1 (b) depends on t co-ordinates of the orthocentre of DPQR are
(c) depends on c (d) equal to 2 (a) (x4, y4) (b) (x4, – y4)
25. The ellipse 4x2 + y = 5 and the hyperbola 4x2 – y2 = 4 (c) (–x4, – y4) (d) (–x4, y4).
have the same foci and they intersect at right angles,

CG_06.indd 15 2/6/2017 4:06:28 PM


6.16 Coordinate Geometry Booster

34. If the hyperbolas x2 + 3xy + 2y2 + 2x + 3y + 2 = 0 and 45. If two distinct tangents can be drawn from the point
x2 + 3xy + 2y2 + 2x + 3y + c = 0 are conjugate of each (a, 2) on different branches of the hyperbola
other, the value of c is x2 y 2
- = 1 , then
(a) –2 (b) 0 (c) 4 (d) 1 9 16
35. A rectangular hyperbola circumscribes a DABC, it will 3 2
always pass through its (a) a < (b) a >
2 3
(a) orthocentre (b) circumcentre
(c) centroid (d) incentre (c) |a| > 3 (d) |a| > 5
36. The co-ordinates of a point on the hyperbola x2 y 2
46. From any point on the hyperbola 2 - 2 = 1 , tangents
x2 y 2 a b
- = 1 , which is nearest to the line
24 18 x2 y 2
are drawn to the hyperbola 2 - 2 = 2 . The area cut
3x + 2y + 1 = 0 are a b
(a) (6, 3) (b) (–6, –3) off by the chord of contact on the asymptotes is
(c) (6, –3) (d) (–6, 3) (a) a/2 (b) ab (c) 2ab (d) 4ab
37. The latus rectum of the hyperbola 9x2 – 16y2 – 18x – 47. A hyperbola passes through (2, 3) and has asymptotes
32y – 151 = 0 is 3x – 4y + 5 = 0 and 12x + 5y = 40, the equation of its
(a) 9/4 (b) 9 (c) 3/2 (d) 9/2 transverse axis is
38. If the eccentricity of the hyperbola x2 – y` sec2 a = 5 is (a) 77x – 21y – 265 = 0 (b) 21x – 77y + 265 = 0
3 times the eccentricity of the ellipse x2 sec2 a + y2 = (c) 21x + 77y – 326 = 0 (d) 21x + 77y – 273 = 0
25, the value of a can be 48. The centre of a rectangular hyperbola lies on the line
p p p p y = 2x. If one of the asymptotes is x + y + c = 0, the
(a) (b) (c) (d) other asymptote is
6 4 3 2
(a) x – y – 3c = 0 (b) 2x – y + c = 0
39. If values of m for which the line y = mx + 2 5 touches (c) x – y – c = 0 (d) None
49. The equation of a rectangular hyperbola, whose asymp-
the hyperbola 16x2 – 9y2 = 144 are the roots of x2 –
totes are x = 3 and y = 5 and passing through (7, 8) is
(a + b)x – 4 = 0, then the value of (a + b) is
(a) xy – 3y + 5x + 3 = 0 (b) xy + 3y + 4x + 3 = 0
(a) 2 (b) 4 (c) 0 (d) none
(c) xy – 3y + 5x – 3 = 0 (d) xy – 3y + 5x + 5 = 0
40. The locus of the feet of the perpendiculars drawn from 50. The equation of the conjugate axis of the hyperbola
either focus on a variable tangent to the hyperbola xy – 3y – 4x + 7 = 0 is
16x2 – 9y = 1 is (a) x + y = 3 (b) x + y = 7
(a) x2 + y2 = 9 (b) x2 + y2 = 1/9 (c) y – x = 3 (d) none
2 2
(c) x + y = 7/144 (d) x2 + y2 = 1/16 51. The curve xy = c (c > 0) and the circle x2 + y2 = 1 touch
41. The locus of the foot of the perpendicular from the cen- at two points, the distance between the points of con-
tre of the hyperbola xy = 1 on a variable tangent is tact is
(a) (x2 – y2)2 = 4xy (b) (x2 – y2)2 = 2xy
2 2 2 (a) 2 (b) 3 (c) 4 (d) 2 2
(c) (x + y ) = 2xy (d) (x2 + y2) = 4xy
52. Let the curves (x – 1)(y – 2) = 5 and (x – 1)2 + (y – 2)2
x2 y 2 = r2 intersect at four points P, Q, R, S. If the centroid of
42. The tangent at a point P on the hyperbola - =1
a 2 b2 DPQR lies on the line y = 3x – 4, the locus of S is
meets one of the directrix in F. If PF subtends an angle (a) y = 3x (b) x2 + y2 + 3x + 1 = 0
q at the corresponding focus, the value of q is (c) 3y = x + 1 (d) y = 3x + 1
p p 3p 53. The ellipse 4x2 + 9y2 = 36 and the hyperbola a2x2 – y2
(a) (b) (c) (d) None
4 2 4 = 4 intersect at right angles, the equation of the circle
through the point of intersection of the two conics is
x2 y 2
43. The number of points on the hyperbola - =3 (a) x2 + y2 = 5
a 2 b2
, from which mutually perpendicular tangents can be (b) 5( x 2 + y 2 ) = 3x + 4y

drawn to the circle x2 + y2 = a2, is (c) 5( x 2 + y 2 ) + 3x + 4y = 0


(a) 0 (b) 2 (c) 3 (d) 4
(d) x2 + y2 = 25
44. If the sum of the slopes of the normal from a point P to
the hyperbola xy = c2 is equal to l, where l ΠR+, the 54. The angle between the lines joining the origin to the
locus of the point P is points of intersection of the line 3 x + y = 2 and the
(a) x2 = lc2 (b) y2 = lc2
curve 3 x + y = 2 is
(c) xy = lc 2
(d) x/y = lc2.

CG_06.indd 16 2/6/2017 4:06:28 PM


Hyperbola 6.17

Ê 2 ˆ 7. The perpendicular from the centre upon the normal at


p
(a) tan -1 Á ˜ (b) x2 y 2
Ë 3¯ 6 any point of the hyperbola 2 - 2 = 1 meets at Q.
a b
Ê 3ˆ p Prove that the locus of Q is
(c) tan -1 Á ˜ (d) (x2 + y2)(a2y2 – b2x2) = (a2 + b2)x2y2
Ë 2 ¯ 2
8. From the points on the circle x2 + y2 = a2, tangents are
55. If S = 0 be the equation of the hyperbola x2 + 4xy + drawn to the hyperbola x2 – y2 = a2. Prove that the locus
3y2 – 4x + 2y + 1 = 0, the value of k for which S + k = 0 of the mid-points of the chord of contact is
represents its asymptotes is (x2 – y2)2 = a2(x2 + y2)
(a) 20 (b) –16 (c) –22 (d) 18 9. Prove that the locus of the mid-points of the hyperbola
x2 y 2
- = 1 which subtend right angle at the centre is
LEVEL III a 2 b2
2
(Problems for JEE Advanced) Ê x2 y 2 ˆ Ê 1 1 ˆ Ê x2 y 2 ˆ
Á 2 - 2 ˜ ÁË 2 - 2 ˜¯ = Á 4 + 4 ˜ .
1. For all real values of m, the straight line Ëa b ¯ a b Ëa b ¯

y = mx + 9m 2 - 4 is a tangent to a hyperbola, find 10. Tangents are drawn from a point P to the parabola
the equation of the hyperbola. y2 = 4ax. If the chord of contact of the parabola be a
2. Find the equations of the common tangent to the curves x2 y 2
tangent to the hyperbola 2 - 2 = 1 , find the locus
x2 y 2 x2 y 2 a b
- = 1 and - = -1 and also find its length. of the point P.
a 2 b2 b2 a 2 11. Chords of the hyperbola x2 – y2 = a2 touch the parabola
3. Find the equation of the common tangents to the curves y2 = 4ax. Prove that the locus of their mid-points is the
x2 y 2 curve y2(x – a) = x3.
- = 1 and x2 + y2 = 9.
16 9 12. Prove that the locus of the mid-points of the rectangu-
x2 y 2
4. If the normal at f on the hyperbola 2 - 2 = 1 meets lar hyperbola xy = c2 of constant length 2d is
a b (x2 + y2)(xy – c2) = d2xy
the transverse axis at G such that
AG ◊ A¢G = am(en secp q – 1), x2 y 2
13. A variable chord of the hyperbola - = 1 is a tan-
where A, A¢ are the vertices of the hyperbola and m, n a 2 b2
and p are positive integers, find the value of gent to the circle x2 + y2 = c2. Prove that the locus of its
(m + n + p)2 + 36 mid-points is
5. If the normals at (xi, yi), i = 1, 2, 3, 4 on the rectangular 2
Ê x2 y 2 ˆ 2Ê x
2
y2 ˆ
hyperbola xy = c2 meet at the point (a, b), prove that - = c +
Á 2 ˜ Á 4 ˜
(i) x1 + x2 + x3 + x4 = a Ëa b2 ¯ Ëa b4 ¯
(ii) y1 + y2 + y3 + y4 = a
14. A variable chord of the circle x2 + y2 = a2 touch the
(iii) x12 + x22 + x32 + x42 =b 2
x2 y 2
hyperbola 2 - 2 = 1 . Prove that the locus of its mid-
(iv) y12 + y22 + y32 + y42 = b2 a b
points is (x + y ) = a2x2 – b2y2.
2 2 2

(v) x1 ◊ x2 ◊ x3 ◊ x4 = –c4 15. A tangent to the parabola y2 = 4ax meets the hyperbola
(vi) y1 ◊ y2 ◊ y3 ◊ y4 = –c4 xy = c2 in two points P and Q. Prove that the locus of
6. If the normals at (xi, yi), i = 1, 2, 3, 4 on the hyperbola the mid-point of PQ lies on a parabola.
x2 y 2 16. From a point P, tangents are drawn to the circle x2 +
- = 1 are concurrent, prove that
a 2 b2 y2 = a2. If the chord of contact of the circle is a normal
x2 y 2
Ê1 1 1 1ˆ chord of a hyperbola 2 - 2 = 1 , prove that the locus
(i) ( x1 + x2 + x3 + x4 ) Á + + + ˜ = 4 . a b
Ë x1 x2 x3 x4 ¯ 2
Ê a 2 b2 ˆ Ê a 2 + b2 ˆ
Ê1 of the point P is Á 2 - 2 ˜ = Á ˜ .
1 1 1ˆ Ëx y ¯ Ë a2 ¯
(ii) ( y1 + y2 + y3 + y4 ) Á + + + ˜ = 4.
Ë y1 y2 y3 y4 ¯

CG_06.indd 17 2/6/2017 4:06:29 PM


6.18 Coordinate Geometry Booster

x2 y 2 x2 y 2
17. The normal to the hyperbola - = 1 meets the 4. A tangent to the hyperbola - = 1 cuts the ellipse
a 2 b2 a 2 b2
axes in M and N, and lines MP and NP are drawn at x2 y 2
right angles to the axes. Prove that the locus of P is the + = 1 in P and Q. Prove that the locus of the
a 2 b2
hyperbola a2x2 – b2y2 = (a2 + b2)2. 2
18. Prove that the locus of the point of intersection of tan- Ê x2 y 2 ˆ Ê x2 y 2 ˆ
mid-point of PQ is Á 2 + 2 ˜ = Á 2 - 2 ˜ .
gents to a hyperbola which meet at a constant angle b Ëa b ¯ Ëa b ¯
is the curve 5. If a triangle is inscribed in a rectangular hyperbola,
(x2 + y2 + b2 – a2)2 = 4 cot2b(a2b2 – b2x2 + a2b2) prove that its orthocentre lies on the curve.
19. Prove that the chords of a hyperbola, which touch the 6. Prove that the locus of the poles of the normal chords
conjugate hyperbola, are bisected at the point of contact. of the rectangular hyperbola xy = c2 is the curve
20. A straight line is drawn parallel to the conjugate axis of (x2 – y2)2 + 4c2xy = 0.
a hyperbola to meet it and the conjugate hyperbola in 7. If a circle cuts a rectangular hyperbola xy = c2 in A, B,
the points P and Q. Show that the tangents at P and Q C and D and the parameters of these four points be t1, t2,
y 4 Ê y 2 x 2 ˆ 4x 2 t3 and t4 respectively. Prove that the centre of the circle
meet the curve Á + ˜= and the normals through A, B and C is
b4 Ë b2 a 2 ¯ a 2
meet on the axis of x. ÊcÊ 1 ˆ cÊ1 1 1 ˆˆ
21. Prove that the locus of the mid-points of the chords of Á 2 ËÁ t1 + t2 + t3 + t t t ¯˜ , 2 ËÁ t + t + t + t1t2t3 ¯˜ ˜
Ë 12 3 1 2 3 ¯
the circle x2 + y2 = 16, which are tangent to the hyper-
bola 9x2 – 16y2 = 144 is (x2 + y2)2 = 16x2 – 9y2. 8. If a triangle is inscribed in a rectangular hyperbola,
22. Find the asymptotes of the curve 2x2 + 5xy + 2y2 + 4x + prove that the orthocentre of the triangle lies on the
5y = 0. Also, find the general equation of all hyperbolas curve.
having the same asymptotes. 9. If a circle cuts a rectangular hyperbola xy = c2 in A, B,
23. Find the equation of the hyperbola whose asymptotes C, D and the parameters of the four points be t1, t2, t3,
are the straight lines x + 2y + 3 = 0 and 3x + 4y + 5 = 0 and t4 respectively, prove that the centre of the mean
and pass through the point (1, –1). position of the four points bisects the distance between
the centres of the two curves.
x2 y 2 10. A circle of variable radius cuts the rectangular hyper-
24. Let C be the centre of the hyperbola - = 1 and
a 2 b2 bola x2 – y2 = 9a2 in points P, Q, R and S. Find the equa-
the tangent at any point P meets the asymptotes at the tion of the locus of the centroid of DPQR.
points Q and R. Prove that the equation to the locus
of the centre of the circle circumscribing, the DCQR is Integer Type Questions
4(a2x2 – b2y2) = (a2 + b2)2.
25. If P, Q, R be three points on the rectangular hyperbola 1. Find the eccentricity of the hyperbola conjugate to the
xy = c2, whose abscissae are x1, x2, x3, prove that the x2 y 2
hyperbola - = 1.
area of DPQR is 4 12
c 2 ( x1 - x2 )( x2 - x3 )( x3 - x1 ) x2 y 2
¥ 2. If the foci of the ellipse + = 1 and the hyperbola
2 x1 x2 x3 16 b 2
x2 y 2 1
- = coincide, find the value of (b2 + 1).
144 81 25
LEVEL IV 3. If e1 and e2 be the eccentricities of a hyperbola and its
(Tougher Problems for JEE Ê 1 1 ˆ
Advanced) conjugate, find the value of Á 2 + 2 + 3˜ .
Ë e1 e2 ¯
1. A tangent to the parabola x2 = 4ay meets the hyperbola 4. Find the number of tangents to the hyperbola
xy = k2 in two points P and Q. Prove that the locus of x2 y 2
- = 1 from the point (4, 3).
the mid-point of PQ lies on the parabola. 4 3
2. Find the equation of the chord of the hyperbola 25x2 – x2 y 2
5. Find the number of points on the hyperbola 2 - 2 = 3
16y2 = 400 which is bisected at the point (6, 2). a b
3. Find the locus of the mid-points of the chords of the from which mutually perpendicular tangents can be
circle x2 + y2 = 16 which are tangents to the hyperbola drawn to the circle x2 + y2 = a2.
9x2 – 16y2 = 144.

CG_06.indd 18 2/6/2017 4:06:29 PM


Hyperbola 6.19

6. If the latus rectum of the hyperbola 9x2 – 16y2 – 18x – 1. The equation of the asymptotes are
32y – 151 = 0 is m, find the value of (2m – 3). (a) 5xy + 2x2 + 3y2 = 0 (b) 3x2 + 4y2 + 6xy = 0
7. If the number of possible tangents can be drawn to the (c) 2x3 + 2y2 – 5xy = 0 (d) 2x2 + y2 – 5xy = 0
curve 4x2 – 9y2 = 36, which are perpendicular to the 2. The angle subtended by AB at the centre of the
straight line 5x + 2y = 10 is m, find the value of (m + 4). hyperbola is
8. If values of m for which the line y = mx + 2 5 touches Ê 4ˆ Ê 2ˆ
(a) sin -1 Á ˜ (b) sin -1 Á ˜
the hyperbola 16x2 – 9y2 = 144 are the roots of x2 – Ë 5¯ Ë 5¯
(a + b)x – 4 = 0, find the value of (a + b + 3). Ê 3ˆ Ê 1ˆ
9. The curve xy = c, (c > 0) and the circle x2 + y2 = 1 touch (c) sin -1 Á ˜ (d) sin -1 Á ˜
Ë 5¯ Ë 5¯
at two points, find the distance between the points of
contact. Ê 7ˆ
3. The equation of the tangent to the hyperbola at Á -1, ˜
10. If the product of the perpendicular distances from any Ë 2¯
is
x2 y 2
point on the hyperbola - = 1 of eccentricity (a) 5x + 2y = 2 (b) 3x + 2y = 4
a 2 b2 (c) 3x + 4y =11 (d) 3x – 4y = 10
e = 3 from its asymptote is equal to 6, find the length
of the transverse axis of the hyperbola. Passage III
11. The tangent to the hyperbola xy = c2 at the point P in- A point P moves such that the sum of the slopes of the nor-
tersects the x-axis at T and the y-axis at T ¢. The normal mals drawn from it to the hyperbola xy = 16 is equal to the
to the hyperbola at P intersects the x-axis at N and N ¢, sum of the ordinates of the feet of normals. The locus of the
respectively. The area of Ds PNT and PN ¢T ¢ are D and P is a curve C.
Ê c2 c2 ˆ 1. The equation of the curve C is
D¢ respectively, find the value of Á + + 4˜ . (a) x2 = 4y (b) x2 = 16y (c) x2 = 12y (d) y2 = 8x
Ë D D¢ ¯ 2. If the tangent to the curve C cuts the co-ordinate axes at
12. Find the area of the triangle formed by any tangent A and B, the locus of the mid-point of AB is
x2 y 2 (a) x2 = 4y (b) x2 = 2y
to the hyperbola - = 1 and its asymptotes.
9 4 2
(c) x + 2y = 0 (d) x2 + 4y = 0
3. The area of the equilateral triangle, inscribed in the
Comprehension Link Passage curve C, having one vertex as the vertex of the curve C
is
Passage I (a) 772 3 s. u. (b) 776 3 s. u.
The locus of the foot of the perpendicular from any focus of
a hyperbola upon any tangent to the hyperbola is an auxiliary (c) 760 3 s. u. (d) 768 3 s. u.
circle of the hyperbola. Consider the foci of a hyperbola as Passage IV
(–3, –2) and (5, 6) and the foot of the perpendicular from the
The vertices of DABC lie on a rectangular hyperbola such that
focus (5, 6) upon a tangent to the hyperbola as (2, 5).
the orthocentre of the triangle is (3, 2) and the asymptotes of
1. The conjugate axis of the hyperbola is
the rectangular hyperbola are parallel to the co-ordinate axes.
(a) 4 11 (b) 2 11 (c) 4 22 (d) 2 22 The two perpendicular tangents of the hyperbola intersect at
2. The directrix of the hyperbola corresponding to the point (1, 1).
the focus (5, 6) is 1. The equation of the asymptotes is
(a) 2x + 2y – 1 = 0 (b) 2x + 2y – 11 = 0
(a) xy – x + y – 1 = 0 (b) xy – x – y + 1 = 0
(c) 2x + 2y – 7 = 0 (d) 2x + 2y – 9 = 0
(c) 2xy + x + y (d) 2xy = x + y + 1
3. The point of contact of the tangent with the hyper-
bola is 2. The equation of the rectangular hyperbola is
Ê 7 23 ˆ (a) xy = 2x + y – 2 (b) xy = 2x + y + 5
2 31
(a) ÊÁ , ˆ˜ (b) Á , ˜ (c) xy = x + y + 1 (d) xy = x + y + 10
Ë9 3 ¯ Ë4 4 ¯
3. The number of real tangents that can be drawn from the
2 Ê7 ˆ
(c) ÊÁ ,9ˆ˜ (d) Á ,7˜ point (1, 1) to the rectangular hyperbola is
Ë3 ¯ Ë9 ¯ (a) 4 (b) 0 (c) 3 (d) 2
Passage II Passage V
The portion of the tangent intercepted between the asymp- A line is drawn through the point P(–1, 2) meets the hyper-
totes of the hyperbola is bisected at the point of contact. bola xy = c2 at the (points A and B lie on the same side of P)
Consider a hyperbola whose centre is at the origin. A line and Q is a point on the line segment AB.
x + y = 2 touches this hyperbola at P(1, 1) and intersects the 1. If the point Q be chosen such that PA, PQ, and PB are
asymptotes at A and B such that AB = 6 2 . in AP, the locus of the point Q is

CG_06.indd 19 2/6/2017 4:06:30 PM


6.20 Coordinate Geometry Booster

(a) x = y + 2xy (b) x = y + xy (C) (R) a circle


2
(c) 2x = y + 2xy (d) 2x = y + xy x=
2. If the point Q be chosen such that PA, PQ and PB are in (e + e - iq )
iq

GP, the locus of the point Q is and


(a) xy – y + 2x – c2 = 0 (b) xy + y – 2x + c2 = 0 Ê eiq - e - iq ˆ
(c) xy + y + 2x + c2 = 0 (d) xy – y – 2x – c2 = 0 y = i ¥ Á iq ˜ is
3. If the point Q be chosen such that PA, PQ, and PB are Ë e + e - iq ¯
in HP, the locus of the point Q is
(D) i - iq (S) a parabola
(a) 2x – y = 2c2 (b) x – 2y = 2c2 x= (e - eiq )
2
(c) 2x + y + 2c = 0 (d) x + 2y = 2c2 2
Passage VI and
The graph of the conic x2 – (y – 1)2 = 1 has one tangent line Ê eiq + e - iq ˆ
with positive slope that passes through the origin, the point of y=Á ˜ is
tangency being (a, b). Ë sin 2q ¯
Ê aˆ
1. The value of sin -1 Á ˜ is 3. Match the following columns
Ë b¯
The locus of the point of intersection of two perpen-
(a) 5p/12 (b) p/6 (c) p/3 (d) p/4
dicular tangents to a conic is a director circle to the
2. The length of the latus rectum of the conic is given conic.
(a) 1 (b) 2 (c) 2 (d) none
Column I Column II
3. The eccentricity of the conic is
(A) The director circle of (P) x2 + y2 = 45
(a) 4/3 (b) 3 (c) 2 (d) none
x2 y 2
- = 1 is
Matrix Match 16 9
(For JEE-Advanced Examination Only) (B) The director circle of (Q) x2 + y2 = 7
x2 y 2
1. Match the following columns + = 1 is
Let z, z1 and z2 be three complex numbers and b, c ΠR+. 20 25
Then the locus of z
(C) The director circle of (R) x2 + y2 = 0
Column I Column II x2 – y2 = 16 is
(A) is an ellipse, if (P) |z – c| = b
(D) The director circle of (S) x + 2 = 0
(B) is a hyperbola, (Q) |z – z1| + |z – z2| = 2b, x2 + y2 = 25 is
if where 2b > |z1 – z2|
(E) The director circle of (T) x2 + y2 = 50
(C) is a straight (R) |z – z1| – |z – z2| = 2b y2 = 8x is
line, if Where 2b < |z1 – z2|
(D) is a circle, if (S) |z – z1| + |z – z2| = |z1 – z2| (F) The director circle of (U) x + 4 = 0
xy = 1 is
2. Match the following columns:
The locus of a variable point P, whose co-ordinates are 4. Match the following columns:
given by The equation of the common tangent between the given
curves
Column I Column II
(A) (P) an ellipse Column I Column II
Ê1- t2 ˆ
x = 3Á ˜ (A) x + y2 = 9
2
(P) y = x + 7
Ë1+ t2 ¯ and
and x2 y 2
Ê 8t ˆ is - = 1 is
y=Á 16 9
Ë 1 - t 2 ˜¯
(B) (Q)
x2 y 2 x2 y 2
(B) 1 t (Q) a hyperbola - =1 - =1
x= (e + e - t ) 25 9 9 25
2 and
and
2 16
1 t y=3 x+ is
y= (e - e - t ) is 7 7
2

CG_06.indd 20 2/6/2017 4:06:30 PM


Hyperbola 6.21

(C) y2 = 8x (R) y = x + 2 (B) Tangents are drawn from a (Q) p


and point (4, 3) to the hyperbola
2
xy = –1 is x2 y 2
- = 1 , the angle be-
(D) x2 – y2 = 9 (S)
3 2 16 9
and y= x+6 tween their tangents is
x2 + y2 = 4 5 3
(C) The angle between the as- (R) Ê 24 ˆ
ymptotes to the hyperbola sin -1 Á ˜
5. Match the following columns: Ë 25 ¯
x2 y 2
Column I Column II - = 1 is
16 9
(A) The product of the perpendicu- (P) 16
(D) The angle between the as- (S) p
lars from the foci of any tangent
ymptotes to the rectangular
x2 y 2 3
to the hyperbola - = 1 is hyperbola x2 – y2 = 2013 is
16 9
(B) The product of the perpendicu- (Q) 9
lars from the foci of any tan- Questions asked in Previous Years’
JEE-Advanced Examinations
x2 y 2
gent to the ellipse + =1
25 16 x2 y2
is 1. The equation - = 1, r > 1 represents a/an
1- r 1+ r
(C) The product of the perpendicu- (R) 144/25 (a) ellipse (b) hyperbola
lars from any point on the on (c) circle (d) None [IIT-JEE, 1981]
x2 y 2 2. Each of the four inequalities given below defines a re-
the hyperbola - = 1 to gion in the xy-plane. One of these four regions does not
9 16
its asymptotes is have the following property. For any two points (x1, y1)
Ê x + x y + y2 ˆ
(D) The product of the perpen- (S) 2/3 and (x2, y2) in the region, the point Á 1 2 , 1 ˜
diculars from any point on the Ë 2 2 ¯
x2 is also in the region. The inequality defining the region
hyperbola - y 2 = 1 to its is
2
asymptotes is (a) x2 + 2y2 £ 1 (b) max.{|x|, |y|} £ 1
(c) x2 – y2 £ 1 (d) y2 – x £ 0
6. Match the following columns: [IIT-JEE, 1981]
Column I Column II No questions asked from 1982 to 1993.
(A) Two intersecting (P) have a common 3. The equation 2x2 + 3y2 – 8x – 18y + 35 = k represents
circles tangent (a) no locus if k > 0
(b) an ellipse if k < 0
(B) Two mutually (Q) have a common
(c) a point if k = 0
external circles normal
(d) a hyperbola if k > 0 [IIT-JEE, 1994]
(C) Two circles, one (R) do not have a com-
No questions asked in 1995.
strictly inside the mon tangent
other 4. An ellipse has eccentricity 1/2 and one focus at S(1/2, 1).
Its one directrix is the common tangent (nearer to S) to
(D) Two branches of a (S) do not have a com- the circle x2 + y2 = 1 and x2 – y2 = 1. The equation of the
hyperbola mon normal. ellipse in the standard form is … [IIT-JEE, 1996]
7. Match the following columns: 5. A variable straight line of slope 4 intersects the hyper-
bola xy = 1 at two points. Find the locus of the point
Column I Column II which divides the line segment between these two
(A) Tangents are drawn from a (P) Ê 3ˆ points in the ratio 1 : 2. [IIT-JEE, 1997]
point on the circle x2 + y2 = 11 sin -1 Á ˜
Ë 5¯ 6. If the circle x2 + y2 = a2 intersects the hyperbola xy = c2
x2 y 2 in four points P(x1, y1), Q(x2, y2), R(x3, y3) and S(x4, y4),
to the hyperbola - = 1,
25 14 then
the angle between the tangent (a) x1 + x2 + x3 + x4 = 0 (b) y1 + y2 + y3 + y4 = 0
is (c) x1 ◊ x2 ◊ x3 ◊ x4 = c4 (d) y1 ◊ y2 ◊ y3 ◊ y4 = c4
[IIT-JEE, 1998]

CG_06.indd 21 2/6/2017 4:06:30 PM


6.22 Coordinate Geometry Booster

7. The angle between a pair of tangents drawn from a (c) focus of the hyperbola is (5, 0)
point P to the parabola y2 = 4ax is 45°. Show that the (d) vertex of the hyperbola is (5 3, 0)
locus of the point P is a hyperbola. [IIT-JEE, 2006]
8. If x = 9 is the chord of contact of the hyperbola x2 – y2 14. A hyperbola, having the transverse axis of length 2 sin q
= 9, the equation of the corresponding pair of tangents is confocal with the ellipse 3x2 + 4y2 = 12, then its equa-
is tion is
(a) 9x2 – 8y2 + 18x – 9 = 0 (a) x2 cosec2 q – y2 sec2 q = 1
(b) 9x2 – 8y2 – 18x – 9 = 0 (b) x2 sec2 q – y2 cosec2 q = 1
(c) 9x2 – 8y2 – 18x + 9 = 0 (c) x2 sin2 q – y2 cos2 q = 1
(d) 9x2 – 8y2 + 18x + 9 = 0 [IIT-JEE, 1999] (d) x2 cos2 q – y2 sin2 q = 1 [IIT-JEE, 2007]
9. Let P(a sec q, b tan q) and Q(a sec j, b tan j), where 15. Consider a branch of the hyperbola
p x2 y 2 x 2 - 2y 2 - 2 2 x - 4 2 y - 6 = 0 with the vertex at the
q +j = , be two points on the hyperbola 2 - 2 = 1 .
2 a b point A. Let B be one of the end points of its latus rec-
If (h, k) is the point of the intersection of the normals at tum. If C be the focus of the hyperbola nearest to the
P and Q, then k is point A, the area of DABC is

Ê a 2 + b2 ˆ Ê a 2 + b2 ˆ Ê 2ˆ Ê 3 ˆ
(b) - Á (a) Á1 - (b) Á - 1˜
(a) Á
Ë a ¯
˜ Ë a ¯
˜ Ë 3 ˜¯ Ë 2 ¯

Ê a 2 + b2 ˆ Ê a 2 + b2 ˆ Ê 2ˆ Ê 3 ˆ
(d) - Á (c) Á1 + (d) Á + 1˜
3 ˜¯
(c) Á ˜ ˜
Ë b ¯ Ë b ¯ Ë Ë 2 ¯

[IIT-JEE, 1999] [IIT-JEE, 2008]


16. An ellipse intersects the hyperbola 2x2 – 2y2 = 1 or-
No questions asked in 2000 to 2002.
thogonally. The eccentricity of the ellipse is reciprocal
x2 y2 of that of the hyperbola. If the axes of the ellipse are
10. For a hyperbola - = 1 , which of the fol-
cos 2a sin 2a along the co-ordinate axes, then the
lowing remains constant with the change of a? (a) ellipse is x2 + 2y2 = 2
(a) abscissae of vertices (b) abscissae of foci (b) ellipse is x2 + 2y2 = 4
(c) eccentricity (d) directrix (c) foci of the ellipse are (±1, 0)
[IIT-JEE, 2003] (d) foci of the ellipse are (± 2, 0) .
11. The point of contact of the line 2x + 6 y = 2 and the [IIT-JEE, 2009]
hyperbola x2 – 2y2 = 4 is 17. Match the following columns:
(a) (4, - 6) (b) ( 6, 1) Column I Column II
(c) (1/2, 1/ 6) (d) (1/6, 3/2) (A) Circle (P) The locus of the point (h, k)
[IIT-JEE, 2004] for which the line
12. Tangents are drawn to the circle x2 + y2 = 9 from a hx + ky = 1
touches the circle
x2 y 2
points on the hyperbola - = 1 . Find the locus of x2 + y2 = 4.
9 4
the mid-point of the chord of contact. [IIT-JEE, 2005] (B) Parabola (Q) A point z in the complex
13. Let a hyperbola passes through the focus of an ellipse plane satisfying
|z + 2| – |z – 2| = ±3
x2 y 2
+ = 1 . The transverse and conjugate axis of this (C) Ellipse (R) The points of the conic have
25 16
hyperbola coincide with the major and minor axes of parametric representations
the given ellipse, also the product of eccentricities of Ê1- t2 ˆ
the given ellipse and hyperbola is 1, then the x = 3Á ˜ and
Ë1+ t2 ¯
x2 y 2
(a) hyperbola is - =1 Ê 2t ˆ
9 16 y=Á
Ë 1 + t 2 ˜¯
x2 y 2
(b) hyperbola is - =1
9 25

CG_06.indd 22 2/6/2017 4:06:31 PM


Hyperbola 6.23

If the hyperbola passes through a focus of the


(D) Hyperbola (S) The eccentricity of the
ellipse, then
conic lies in the interval
1£x< . x2 y 2
(a) the hyperbola is - =1
3 2
(T) The points z in the complex
plane satisfying (b) a focus of hyperbola is (2, 0)
Re(z + 1)2 = |z|2 + 1 2
(c) the eccentricity of hyperbola is
3
[IIT-JEE, 2009]
(d) the hyperbola is x2 – 3y2 = 3 [IIT-JEE, 2011]
Comprehension
x2 y 2
2 2
23. Tangents are drawn to the hyperbola - = 1,
x y 9 4
The circle x2 + y2 – 8x = 0 and the hyperbola - =1
9 4 parallel to the straight line 2x – y = 1. The points
intersect at the points A and B.
of contact of the tangents on the hyperbola are
18. The equation of the common tangent with positive
slope to the circle as well as to the hyperbola is Ê 9 1 ˆ Ê 9 1 ˆ
(a) Á , (b) Á - ,-
(a) 2x - 5 y - 20 = 0 (b) 2x - 5 y + 4 = 0 Ë 2 2 2 ˜¯ Ë 2 2 ˜

(c) 3x – 4y + 8 = 0 (d) 4x – 3y + 4 = 0
(c) (3 3, - 2 2) (d) ( -3 3, 2 2)
19. The equation of a circle with AB as its diameter is
(a) x2 + y2 – 12x + 24 = 0 [IIT-JEE, 2012]
(b) x2 + y2 + 12x + 24 = 0 No questions asked in between 2013-2014.
(c) x2 + y2 + 24x – 12 = 0 24. Consider the hyperbola H : x2 – y2 = 1 and a circle S
with center N(x2, 0). Suppose that H and S touch each
(d) x2 + y2 – 24x – 12 = 0 [IIT-JEE, 2010]
other at a point P(x1, y1) with x1 > 1 and y1 > 0. The
20. The line 2x + y = 1 is the tangent to the hyperbola
common tangent to H and S at P intersects the x-axis at
x2 y 2 point M. If (l, m) is the centroid of the triangle DPMN,
- = 1 . If this line passes through the point of
a 2 b2 then the correct expression(s) is (are)
intersection of the nearest directrix and the x-axis,
dI 1
the eccentricity of the hyperbola is … (a) = 1 - 2 for x1 > 1
[IIT-JEE, 2010] dx1 3x1

x2 y 2 dm x1
21. Let P(6, 3) be a point on the hyperbola - = 1. If (b) = for x1 > 1
a 2 b2 dx1 3( x 2 - 1)
1
the normal at the point P intersects the x-axis at (9, 0),
dI 1
the eccentricity of the hyperbola is (c) = 1 + 2 for x1 > 1
dx1 3x1
5 3
(a) (b) (c) 2 (d) 3 dm 1
2 2 (d) = for y1 > 0 [IIT-JEE-2015]
dy1 3
x2 y 2
22. Let the eccentricity of the hyperbola 2 - 2 = 1
a b No questions asked in 2016.
be reciprocal to that of the ellipse x2 + 4y2 = 4.

A NSWERS

LEVEL II 26.
31.
(a)
(a)
27.
32.
(b)
()
28.
33.
(c)
(c)
29.
34.
(a)
(b)
30.
35.
(b)
(a)
36. (c) 37. (d) 38. (b) 39. (c) 40. (d)
1. (b) 2. (a) 3. (a) 4. (b) 5. (d)
41. (d) 42. (b) 43. (a) 44. (a) 45. (a)
6. (b) 7. (b) 8. (d) 9. (b) 10. (a)
46. (a) 47. (d) 48. (c) 49. (d) 50. (b)
11. (d) 12. (b) 13. (d) 14. (a) 15. (c)
16. (b) 17. (c) 18. (c) 19. (b) 20. (b) 51. (a) 52. (a) 53. (a) 54. (c) 55. (c)
21. (d) 22. (b) 23. (d) 24. (c) 25. (a)

CG_06.indd 23 2/6/2017 4:06:32 PM


6.24 Coordinate Geometry Booster

LEVEL III INTEGER TYPE QUESTIONS


1. 2 2. 8 3. 4 4. 1 5. 0
2 2 6. 6 7. 4 8. 3 9. 2 10. 6
x y
1. - =1 11. 6 12. 6
9 4
2 2 COMPREHENSIVE LINK PASSAGES
2. y = ± x ± a - b
Passage I: 1. (d) 2. (b) 3. (c)
7 11 Passage II: 1. (a) 2. (c) 3. (b)
3. y = ± x+
10 5 Passage III: 1. (b) 2. (c) 3. (d)
Passage IV: 1. (b) 2. (c) 3. (d)
4. 72
Passage V: 1. (c) 2. (b) 3. (a)
10. 4a2x2 + b2y2 = 4a4
Passage VI: 1. (d) 2. (c) 3. (d)
22. 2x2 + 5xy + 2y2 + 4x + 5y + 2 = 0
23. 3x2 + 10xy + 8y2 + 4x + 6y + 1 = 0
MATRIX MATCH
1. (A) Æ Q; (B) Æ R; (C) Æ S; (D) Æ P
LEVEL IV 2. (A) Æ P; (B) Æ Q; (C) Æ Q; (D) Æ Q
3. (A) Æ Q; (B) Æ P; (C) Æ R; (D) Æ T; (E) Æ S;
2. 75x – 16y = 418 (F) Æ R
3. (x2 + y2)2 = 16x2 – 9y2 4. (A) Æ Q; (B) Æ P; (C) Æ R; (D) Æ S
2h ˆ
2
2k ˆ
2 5. (A) Æ Q; (B) Æ P; (C) Æ R; (D) Æ S
Ê Ê 2
10. Á x - ˜ + Á y - ˜ = a 6. (A) Æ P, Q; (B) Æ P, Q; (C) Æ Q, R; (D) Æ Q, R
Ë 3 ¯ Ë 3¯
7. (A) Æ Q; (B) Æ P; (C) Æ R; (D) Æ S

H INTS AND S OLUTIONS

LEVEL I (c) Co-vertices: B(a, 0) = B(4, 0)


and B¢(–a, 0) = B¢(–4, 0)
(d) The length of the transverse axis = 2a = 6
1.
x2 y 2 (e) The length of the conjugate axis = 2b = 8
(i) The equation of the given hyperbola is - =1
9 4 2a 2 16
(a) Centre: (0, 0) (f) The length of the latus rectum = =
(b) Vertices: A(a, 0) = A(3, 0) b 3
and A (–a, 0) = A(–3, 0) a2 9 5
(c) Co-vertices: B(0, b) = B(0, 2) (g) Eccentricity = e = 1+
= 1+ =
b2 16 4
and B¢(0, –b) = B¢(0, –2) (h) Equation of the directrices :
(d) The length of the transverse axis = 2a = 6 b 3 12
(e) The length of the conjugate axis = 2b = 4 y=± =± =± .
e 5/4 5
2b 2 8
(f) The length of the latus rectum = = (iii) The equation of the hyperbola is
a 3 9x2 – 16y2 – 36x + 96y – 252 = 0
b2 4 13 fi 9(x2 – 4x) – 16(y2 – 6y) = 252
(g) Eccentricity = e = 1+ 2
= 1+ = fi 9(x – 2)2 – 16(y – 3)2 = 252 + 36 – 144 = 144
a 9 3
(h) Equation of the directrices: 9( x - 2) 2 16(y - 3) 2
fi - =1
a 2 6 144 144
x=± =± =±
e 13/3 13 ( x - 2) 2 ( y - 3) 2
(ii) The equation of the given hyperbola is fi - =1
16 9
x2 y 2 (a) Centre : (0, 0)
- = -1
16 9 fi X = 0, Y = 0
(a) Centre: (0, 0) fi x – 2 = 0, y – 3 = 0
(b) Vertices: A(0, b) = A(0, 3) fi x = 2 and y = 3
and A(0, –b) = A(0, –3) Hence, the centre is (2, 3).

CG_06.indd 24 2/6/2017 4:06:32 PM


Hyperbola 6.25

(b) Vertices : (±a, 0) 4


fi X = ±a, Y = 0 fi b=
5
fi x – 2 = 4, y – 3 = 0 16 Ê 25 ˆ 84
2 2 2
fi x = 2 ± 4, y = 3 Also, a = b (e - 1) = Á - 1˜¯ =
25 Ë 4 25
Hence, the vertices are (6, 3) and (–2, 3).
(c) Co-vertices: (0, ±b) Hence, the equation of the hyperbola is
2
fi X = 0, Y = ±b ( x + 3) 2 ( y - 2)
- =1
fi x –2 = 0, y – 3 = ±3 a2 b2
fi x = 2, y = 3 ± 3
( x + 3) 2 ( y - 2) 2 1
Hence, the co-vertices are (2, 6) and (2, 0) fi - =
(d) The length of the transverse axis = 2a = 8 84 16 25
(e) The length of the conjugate axis = 2b = 6 5. From the definition of the hyperbola, we can write
SP
9 5 =e,
(f) Eccentricity = e = 1+ = PM
16 4
where S = focus, P = (x, y)
(g) Co-ordinates of Foci : (±ae, 0)
fi SP2 = e2 PM2
X = ±5, Y = 0
ÏÔÊ x + 2y - 1ˆ 2 ¸Ô
fi X – 2 = ±5, y – 3 = 0 fi ( x - 2) 2 + ( y - 1) 2 = 4 ÌÁ ˝
fi X = 2 ± 5, y = 3 Ë 1 + 4 ˜¯ Ô
ÓÔ ˛
Hence, the co-ordinates of the foci are (7, 3)
fi 5{(x – 2)2 + (y – 1)2} = 4(x + 2y – 1)2
and (–3, 3)
6. Given relation is
2. The equation of the hyperbola with centre (1, 0) is
2 ae = 16
( x - 1) 2 y 2 fi ae = 8
- 2 =1
a2 b fi a = 8/e fi a = 2
Here, 2a = 6 fi a = 3 Now, b2 – a2(e2 – 1) – a2e2 – a2 = 64 – 4 = 60
Also, one focus = (6, 0) Hence, the equation of the hyperbola is
fi 1 + ae = 6 x2 y 2
fi ae = 5 - =1
a 2 b2
fi 3e = 5
fi e = 5/3 x2 y 2
fi - =1
Therefore, 4 60
7. Since foci are (6, 4) and (–6, 4), so the axis of the
Ê 25 ˆ
b 2 = a 2 (e 2 - 1) = 9 Á - 1˜ = 16 hyperbola is parallel to x-axis.
Ë 9 ¯
It is given that the distance between two foci is 12, so
Hence, the equation of the hyperbola is 2 ae = 12
( x - 1) 2 y 2 fi ae = 6
- =1 fi 2a=6
9 16
fi a=3
3. Since the focus is (5, 2) and the vertex is (4, 2), so the
Also,
axis of the hyperbola is parallel to x-axis and a = 1,
b2 – a2(e2 – 1) – a2e2 –a2 – 36 – 9 = 27
ae = 2
Hence, the equation of the hyperbola is
Let the equation of the hyperbola be
x2 y 2
( x - 3) 2 ( y - 2) 2 - =1
- =1 …(i) a 2 b2
a2 b2
As we know that, the relation in a, b and e with respect x2 y 2
fi - =1
to a hyperbola is 9 27
b2 – a2(e2 – 1) – a2e2 – a2 – 4 – 1 – 3 8. Given relation is
Now from Eq. (i), we get 2b 2 1
= ¥ (2a ) = a
a 2
( x - 3) 2 ( y - 2) 2
- =1 2b 2
1 3 fi =a
4. Since the focus is (–3, 2) and the vertex is (–3,4), so the a
axis of the hyperbola is parallel to y-axis and be = 2 fi 2b2 = a2
5 Also, relation in a, b and e is
fi b¥ = 2 b2 = a2(c2 – 1)
2

CG_06.indd 25 2/6/2017 4:06:32 PM


6.26 Coordinate Geometry Booster

fi 3(x2 –2x) – 5(y2 – 4y) = 32


a2
fi = a 2 (e 2 - 1) fi 3(x – 1)2 – 5(y – 2)2 = 32 + 3 – 20 = 15
2
1 3( x - 1) 2 5( y - 2) 2
fi (e 2 - 1) = fi - =1
2 15 15
3 ( x - 1) 2 ( y - 2) 2
e= fi - =1
fi 5 3
2
x2 y 2 13. The given straight lines are
9. Let the equation of the hyperbola is - = 1 and x y
a 2 b2 - = 2013 …(i)
x2 y 2 a b
its conjugate is 2 - 2 = -1 . and
a b x y 1
Therefore, + = …(ii)
a b 2013
b2 a2 Multiplying Eqs (i) and (ii), we get
e1 = 1 + and e2 = 1 +
a2 b2 Ê x yˆ Ê x yˆ 1
ÁË - ˜¯ ÁË + ˜¯ = 2013 ¥ =1
Ê a 2 + b2 ˆ Ê a 2 + b2 ˆ a b a b 2013
fi e12 = Á 2
2
˜ and e2 = Á ˜
Ë a ¯ Ë b2 ¯ x2 y 2
fi - =1
a 2 b2
1 1 Ê a 2 ˆ Ê b2 ˆ which represents a hyperbola.
fi + = Á ˜ +Á ˜
e12 e22 Ë a 2 + b 2 ¯ Ë a 2 + b 2 ¯ 14. We have,
Ê a 2 + b2 ˆ Ê1+ t2 ˆ 4t
=Á 2 ˜ =1 x = 3Á 2˜
and y = 2
Ë a + b2 ¯ Ë1- t ¯ t -1
2 2
x2 y 2 x2 y 2 Ê 1 + t 2 ˆ Ê 2t ˆ
10. Let the equation of the ellipse be 2 + 2 = 1 and the fi - =Á - ÁË ˜
a b 9 4 Ë 1 - t 2 ˜¯ 1- t2 ¯
x2 y 2 2
equation of the hyperbola is is - =1 x 2 y 2 (1 + t 2 ) 2 - 4t 2 Ê 1 - t 2 ˆ
a 2 b2 fi - = =Á ˜ =1
Also, it is given that,
9 4 (1 - t 2 ) 2 Ë1- t2 ¯
2b = 2a x2 y 2
fi b=a fi - =1
9 4
Let e1 and e2 be the eccentricities of the ellipse and the which represents a hyperbola.
hyperbola. 15. We have,
b2 a2 1 1
Then e1 = 1 - = 1 - =0 x = (et + e - t ) and y = (et - e - t )
a2 a2 2 2
fi 2x = (et + e–t) and 2y = (et – e–t)
b2 a2 fi 4x2 – 4y2 = (et + e–t)2 – (et – e–t)2
and e2 = 1 + = 1 + = 2 fi 4x2 – 4y2 = 2 + 2
a2 a2
fi x2 – y2 = 1
Thus, e12 + e22 = 2 Which represents a rectangular hyperbola.
16. The given equation of the hyperbola is
11. The equation of the hyperbola is
4(2y – x – 3)2 – 9(2x + y – 1)2 = 80 x2 y2
+ =1
The centre of the hyperbola is obtained from the equa- 2014 - l 2013 - l
tions fi (2013 – l)x2 + (2014 – l)y2
2y – x – 3 = 0 and 2x + y – 1 = 0 –(2013 – l)(2014 – l) = 0
Solving, we get, fi (2013 – l)x2 + (l – 2014)y2
x = –2/5 –(2013 – l)(2014 – l) = 0
and y = 13/10 The given equation represents a hyperbola, if
Ê 2 13 ˆ h2 – ab > 0
Hence, the centre is Á - , ˜ . fi 0 – (2013 – l)(l – 2014) > 0
Ë 5 10 ¯
fi (2013 – l)(l – 2014) < 0
12. The equation of the given hyperbola is fi 2013 < l < 2014
3x2 Р5y2 Р6x + 20y Р32 = 0 fi l Π(2013, 2014)

CG_06.indd 26 2/6/2017 4:06:33 PM


Hyperbola 6.27

17. The given equation of the hyperbola is 19. We have,


x2 y 2 1 2 x12 - 3 y12 - 1 = 2 – 48 – 1 = – 47 < 0
- =
144 81 25 Thus, the point (1, 4) lies outside of the hyperbola.
We have, 20. Since the point (l, –1) is an exterior point of the curve
b2 81 225 15 5 4x2 – 3y2 = 1, so
e = 1+ 2
= 1+ = = = 4l2 – 3 – 1 < 0
a 144 144 12 4
fi 4l2 – 4 < 0
144 fi l2 – 1 < 0
Also, a 2 =
25 fi (l + 1)(l – 1) < 0
Thus, the foci are fi –1 < l < 1
Ê 12 5 ˆ fi l Œ (–1, 1)
(± ae, 0) = Á ± ¥ , 0˜ = (± 3, 0). Thus, the length of the interval, where l lies is 2.
Ë 5 4 ¯
Therefore, m = 2
Now, for the ellipse, Hence, the value of m + 10 = 2 + 10 = 12.
ae = 3
fi a2e2 = 9 21. As we know that if the line y = mx + c be a tangent
Thus, b2 = a2(1 – e2) x2 y 2
to the hyperbola 2 - 2 = 1 , the co-ordinates of the
= a2 – a2e2 a b
= 16 – 9 = 7
Ê a 2m b2 ˆ
Hence, the value of b2 is 7. point of contact is Á ± ,± ˜ .
18. Let LL¢ be the latus rectum of the given hyperbola. Ë c c¯
Ê b2 ˆ Ê b2 ˆ The equation of the given hyperbola is
Therefore, L Á ae, ˜ and L ¢ Á ae, - ˜ and the cen- 25x2 – 9y2 = 225
Ë a¯ Ë a¯
x2 y 2
tre of the hyperbola is C(0, 0) fi - =1 …(i)
9 25
(b 2 /a ) b 2 Also, the given line is
Now, slope of CL = m1 = = 2
ae a e 25x + 12y = 45
( - b 2 /a ) b2 fi 12y = –25x + 45
and the slope of m2 = =- 2 25 45
ae a e fi y = - x+ …(ii)
Since, the latus rectum subtends right angle at the cen- 12 12
tre, so Here, a = 3, b = 5 and m = – 25/12.
m1 ¥ m2 = –1 Thus, the common point is
Ê a2m b2 ˆ Ê 20 ˆ
Ê b2 ˆ Ê b2 ˆ Á ± c , ± c ˜ = ËÁ 5, ± 3 ¯˜ .
fi Á 2 ˜ ¥ Á - 2 ˜ = -1 Ë ¯
Ë a e¯ Ë a e¯
22. As we know that, the line y = mx + c will be a tangent
Ê b4 ˆ
fi x2 y 2
Á 4 2˜ =1 to the hyperbola 2 - 2 = 1 , if
Ëa e ¯ a b
fi b4 = a4e2 c2 = a2m2 – b2 …(i)
fi a4(e2 – 1)2 = a4e2 The equation of the hyperbola is
fi (e2 – 1)2 = e2 9x2 – 5y2 = 45
fi e4 – 3e2 + 1 = 0 x2 y 2
fi - =1
3± 5 5 9
fi e2 = Here, a2 = 5, b2 = 9, m = 3, c = l
2
Therefore, from Eq. (i), we get
3+ 5 l2 = a2m2 – b2 = 15 – 9 = 6
fi e2 =
2
fi l=± 6
2
3 + 5 6 + 2 5 Ê 5 + 1ˆ 23. The equation of any tangent to the hyperbola
fi e2 = = =Á ˜
2 4 Ë 2 ¯ x2 y 2
- = 1 is
a 2 b2
Ê 5 + 1ˆ …(i)
fi e=Á ˜ y = mx + a 2 m 2 - b 2
Ë 2 ¯

CG_06.indd 27 2/6/2017 4:06:34 PM


6.28 Coordinate Geometry Booster

The equation of the given tangent is If the line (ii) be a tangent to the hyperbola (i), the co-
2 …(ii) ordinates of the point of contact can be
y = mx + 9m - 4
Since Eqs (i) and (ii) are identical, so Ê a 2m b2 ˆ Ê 4ˆ
a2 = 9, b2 = 4 Á ± c , ± c ˜ = ÁË ± ( -5), ± 3 ˜¯
Ë ¯
Hence, the equation of the hyperbola is
27. The equation of the given hyperbola is
x2 y 2
- =1 4x2 – 9y2 = 36
9 4
fi 4x2 – 9y2 = 36 x2 y 2
fi - =1
24. The equation of any tangent to the parallel to 5 x – 4y + 9 4
7 = 0 is Here, a2 = 9, b2 = 4.
5x – 4y + l = 0 The equation of any tangent to the hyperbola
fi 4y = 5x + l
x2 y 2
5 l - = 1 is
fi y = x+ …(i) a 2 b2
4 4
The equation of the given hyperbola is y = mx + a 2 m 2 - b 2
4x2 – 9y2 = 36
x2 y 2 fi y = mx + 9 m 2 - 4
fi - =1 …(ii)
9 4 which is passing through (3, 2). So
Since, the line (i) is a tangent to the hyperbola (ii), so (2 – 3m)2 = 9m2 – 4
l2 Ê 25 ˆ fi 4 – 12m + 9m2 = 9m2 – 4
= 9Á ˜ - 4
16 Ë 16 ¯ fi 4 – 12m = –4
l 2 225 - 64 2
fi = fi m ,m
16 16 3
Hence, the equations of the tangents are
fi l = 161
2

2
fi l = ± 161 x + 3 = 0 and y = x .
Hence, the equations of the tangents are 3
28. We have,
5 x - 4 y ± 161 = 0
25. The equation of any line perpendicular to 2 x12 - 3y12 - 12 = 2.1 – 3. 4 – 12
3x + 4y + 10 = 0 is = 2 – 12 – 12 = 2 – 24 = –22 < 0
4x – 3y + l = 0 So, the point (1, –2) lies outside of the hyperbola.
Ê 4ˆ l Thus, the number of tangents is 2.
fi y = Á ˜ x+ …(i) 29. The equation of the given hyperbola is
Ë 3¯ 3
The equation of the given hyperbola is 3x2 – 4y2 = 12
9x2 – 16y2 = 144
x2 y 2
x2 y 2 fi - =1 …(i)
fi - =1 …(ii) 4 3
16 9
Here, a2 = 4, b2 = 3 and m = 4
The line (i) will be a tangent to the hyperbola (ii), if
2 2 The equation of any tangent to the hyperbola (i) is
Ê lˆ Ê 4ˆ
ÁË ˜¯ = 16 ÁË ˜¯ - 9 y = mx + a 2 m 2 - b 2
3 3
fi l2 = 256 – 81 = 175 fi y = 4x ± 64 - 3
fi l = ± 175 = ± 5 7
fi y = 4x ± 61
Hence, the equation of the tangents are
4x - 37 ± 5 7 = 0. Hence, the equations of tangents are
26. The given hyperbola is y = 4x + 61 and y = 4x - 61.
x2 – 9y2 = 9 30. The equation of tangent to the curve
2 2 x2 – y2 – 8x + 2y + 11 = 0 is
x y
fi - =1 …(i) xx1 – yy1 – 4(x – x1) + (y – y1) + 1 = 0
9 1
The given line is 5x + 12y – 9 = 0 fi 2x – y – 4(x + 2) + (y + 1) + 11 = 0
Ê 5ˆ 3 fi –2x + 4 = 0
fi y = Á- ˜ x+ …(ii) fi x–2=0
Ë 12 ¯ 4

CG_06.indd 28 2/6/2017 4:06:34 PM


Hyperbola 6.29

31. When y = 2, then 4x2 = 36


x2 y 2
fi x = ±3 34. The equation of the given curves are - = 1 and
Hence, the points are (3, 2) and (–3, 2). 9 4
x2 y 2
The equation of the tangent at (3, 2) is + = 1.
12x – 6y = 24 9 4
fi 2x – y = 4 Y
Also, the equation of the tangent at (–3, 2) is
–12x – 6y = 24
y=2
fi 2x + y + 4 = 0
32. The equation of the given hyperbola is X¢ X
9x2 – 16y2 = 144 c

x2 y 2 y=2
fi - =1 …(i)
16 9
The equation of any tangent to the hyperbola (i) can be x = –3
Y¢ x=3
2
considered as y = mx + 16m - 9 Here, the length of the major axis of the ellipse is equal
which is passing through (4, 3). So to the length of the transverse axis and also the length of
(3 – 4m)2 = 16m2 – 9. the minor axis is equal to the length of the conjugate axis.
fi 9 – 24 m + 16 m2 = 16 m2 – 9 Thus, the equations of the common tangents are x = ±3
fi 24m = 18 and y = ±2.
3 35. The equation of any tangent to the hyperbola
fi m and m x2 y 2
4 - = 1 is
Let q be the angle between them. Then 16 9
3 3 y = mx + 16m 2 - 9
1 4
tan(q ) = 4 = 4 = fi m x - y + 16m 2 - 9 = 0 …(i)
3 1 3 3
1 . Y
4 4
Ê 4ˆ
fi q = tan -1 Á ˜
Ë 3¯
M
Hence, the angle between the tangents is
X¢ X
C

-1 Ê
tan Á ˜ .
Ë 3¯
33. The equation of any tangent to the hyperbola
x2 y 2 Y¢
- = 1 is
a 2 b2 If the tangent (i) is also the tangent to the circle x2 + y2
= 9, the length of the perpendicular from the centre of
…(i)
y = m1 x + a 2 m12 - b 2 the circle is equal to the radius of the circle. So
The equation of any tangent to the hyperbola
0 - 16m 2 - 9
x2 y2 =3
- = 1 is m2 + 1
( -b ) ( - a 2 )
2

fi 16m2 – 9 = 9(m2 + 1)
y = m2 x + ( -b 2 )m22 - ( - a 2 ) …(ii) fi 16m2 – 9m2 = 9 + 1 = 10
Since the equations (i) and (ii) are identical, so fi 7m2 = 10
a 2 m12 - b 2 = ( -b 2 )m22 - ( - a 2 ) 7
fi m=±
10
fi m12 = 1 and m22 = 1 7 11
Thus, m1 = ±1 = m2 Hence, the equation of tangents are y = ± x+ .
10 5
Hence, the equations of the common tan gents are
36. The equation of any tangent to the parabola y2 = 8x is
y = ± x + a 2 - b2 2
y = mx + …(i)
m

CG_06.indd 29 2/6/2017 4:06:35 PM


6.30 Coordinate Geometry Booster

Y Then F1 = (ae, 0) and F2 = (–ae, 0).


The equation of any tangent to the hyperbola is
x y
sec q - tan q = 1 …(i)
a b
Let p1 and p2 be two perpendiculars from foci upon the
X¢ X
C tangent (i).

a 2 (e 2 - 1)(e 2 sec 2q - 1)
fi p1 p2 =
((e 2 - 1) sec 2q + tan 2q )
Y¢ a 2 (e 2 - 1)(e 2 sec 2q - 1)
Since the tangent to the parabola is also a tangent to the fi p1 p2 =
((e 2 - 1) sec 2q + (sec 2q - 1))
x2 y 2
hyperbola - = 1 , so
9 5 a 2 (e 2 - 1)(e 2sec 2q - 1)
2 fi p1 p2 =
Ê 2ˆ (e 2sec 2q - 1)
5 x 2 - 9 Á mx + ˜ = 45
Ë m¯ fi p1p2 = a2(e2 – 1)
Ê 4 ˆ fi p1p2 = b2
fi 5 x 2 - 9 Á m 2 x 2 + 2 + 4m˜ = 45
Ë m ¯ Hence, the product of the lengths of the perpendiculars
Ê 4 ˆ is b2.
fi (5 - 9m 2 ) x 2 - 36 x - 9 Á 2 + 5˜ = 0 39. Let P(a, b) be the point of intersection of tangents at A
Ëm ¯
Now, and B.
D=0 Y
B
Ê 4 ˆ
fi (36) 2 + 36(5 - 9m 2 ) Á 2 + 5˜ = 0
Ëm ¯
20
fi 36 + 2
+ 25 - 36 - 45m 2 = 0
m P
4 X¢ X
fi + 5 - 9 m2 = 0
m2
fi 9m4 – 5m2 – 4 = 0
fi (m2 – 1)(9m2 + 4) = 0 A
fi (m2 – 1) = 0
fi m = ±1 Y¢
Hence, the equations of the common tangents are Clearly, the point of intersection of the tangents at A
y = ± x ± 2. and B is the chord of contact.
37. As we know that the locus of the perpendicular tan- Therefore, the chord of contact AB is
gents is the director circle. ax by
Hence, the equation of the director circle is - =1
x2 + y2 = a2 – b2 = 16 – 9 = 7 a 2 b2
fi x2 + y2 = 7 b2 x a b2
fi y= 2 -
x2 y 2 a yb b
38. Let F1 and F2 be two foci of the hyperbola 2 - 2 = 1.
a b which is a tangent to the parabola y2 = 4ax. So
Y
b2 a
- =
b Ê b 2a ˆ
N Á a2b ˜
Ë ¯
Ê b4 ˆ

F1
X fi b2 = Á - 3 a˜
F2 C Ë a ¯
Hence, the locus of P(a, b) is
M Ê b4 ˆ
y 2 = Á - 3 x˜
Ë a ¯

CG_06.indd 30 2/6/2017 4:06:36 PM


Hyperbola 6.31

ax
fi + ay = a 2 + b 2
e sec q - 1 e
Then, p1 =
sec 2q tan 2q x y
+ 2 fi + =1
a2 b Ê e( a 2 + b 2 ) ˆ Ê a 2 + b 2 ˆ
Á a ˜ Á a ˜
Ë ¯ Ë ¯

e sec q + 1 Hence, the area of DOAB


and p2 =
sec 2q tan 2q 1 e( a 2 + b 2 ) Ê a 2 + b 2 ˆ
+ 2 = ¥ ¥Á ˜
a2 b 2 a Ë a ¯


(e sec q - 1)(e sec q + 1) 1 e( a 2 + b 2 ) 2
p1 p2 = = ¥
sec 2q tan 2q 2 a2
+ 2
a2 b 1
2 2 2 2
= ¥ a 2 e5
a b (e sec q - 1) 2
= 2 2
(b sec q + a 2 tan 2q ) 42. The equation of any normal at (a sec j, b tan j) to the
x2 y 2
a 4 (e 2 - 1)(e 2 sec 2q - 1) hyperbola 2 - 2 = 1 is
= a b
(a 2 (e 2 - 1)sec 2q + a 2 tan 2q ) ax cos + by cot j = a2 + b2 …(i)
x2 y 2 The equation of any line perpendicular to (i) and pass-
40. The equation of the normal to the curve - = 1 at
16 9 ing through the origin is
(8, 3 3) is (b cot j)x – (a cos j)y = 0
fi bx – a sin jy = 0 …(ii)
a 2 x b2 y
+ = a 2 + b2 If we eliminate j between Eqs (i) and (ii), we get the
x1 y1
required locus of the foot of the perpendicular.
16x 9y From (ii), we get,
fi + = 16 + 9
8 3 3
bx
sin (j ) =
fi 2 x + 3 y = 25 ay
Hence, the required equation of the normal is
2 x + 3 y = 25. (a 2 y 2 - b 2 x 2 )
fi cos (j ) =
41. Let, one end of the latus rectum of the given hyperbola ay
Ê b2 ˆ (a 2 y 2 - b 2 x 2 )
is L Á ae, ˜ . and cot (j ) =
Ë a¯ bx
Y From Eq. (i), we get
B
(a 2 y 2 - b 2 x 2 ) (a 2 y 2 - b 2 x 2 )
ax ¥ + by ¥
L ay bx
= a2 + b2


O S A
C
fi ( x2 + y 2 ) ( (a y - b x ) ) = (a + b ) xy
2 2 2 2 2 2

fi (x2 + y2)2(a2y2 – b2x2) = (a2 + b2)2 x2y2


which is the required locus of the foot of the perpen-
L¢ dicular.

43. The equation of any normal to the hyperbola
The equation of the normal to the given hyperbola at L
is x2 y 2
- = 1 is
a 2 x b2 y a 2 b2
+ = a 2 + b2
x1 y1 ax cosj + by cotj = (a2 + b2) …(i)

CG_06.indd 31 2/6/2017 4:06:37 PM


6.32 Coordinate Geometry Booster

Y
P Ê a 2 + b2 ˆ
N Thus, the co-ordinates of G = Á , 0˜
Ë a cos j ¯
Clearly, the vertices, A = (a, 0) and A¢ = (–a, 0)
Q
Ê a 2 + b2 ˆ
Now, AG = Á - a˜
X¢ X Ë a cos j ¯
O M
Ê a 2 + b2 ˆ
and A¢ G = Á + a˜
Ë a cos j ¯
Y¢ Therefore,
Since the normal (i) meets the x-axis at M and y-axis at Ê a 2 + b2 ˆ Ê a 2 + b2 ˆ
N respectively. Then, AG. A¢G = Á - a˜ Á + a˜
Ë a cos j ¯ Ë a cos j ¯
Ê a 2 + b2 ˆ Ê Ê a 2 + b2 ˆ ˆ
M= Á , 0˜ and N = Á 0, Á ˜ tan j ˜ Ê Ê a 2 + b2 ˆ 2 ˆ
Ë a cos j ¯ Ë Ë b ¯ ¯ = ÁÁ sec 2
j - a 2
˜
Let the co-ordinates of the point P be (a, b). ÁË Ë a ˜¯ ˜¯
Since PM and PN are perpendiculars to the axes, so the
= (a2e2 sec2j – a2)
co-ordinates of P are
= a2(e2 sec2j – 1)
Ê Ê a 2 + b2 ˆ Ê a 2 + b2 ˆ ˆ fi m = 2, n = 2, p = 2
ÁÁ ˜ sec j , Á ˜ tan j ˜
ËË a ¯ Ë b ¯ ¯ Hence,
(m + n + p)2 + 36 = 36 + 36 = 72.
Therefore,
45. The equation of the normal to the hyperbola xy = c2 at
Ê a 2 + b2 ˆ Ê a 2 + b2 ˆ
a=Á ˜ sec j and b = Á ˜ tan j Ê cˆ
Ë a ¯ Ë b ¯ ÁË ct , ˜¯ is
t
Ê a ˆ Ê b ˆ xt3 – yt – ct4 + c = 0
fi aÁ 2 = sec j and b Á 2 = tan j
Ë a + b 2 ˜¯ Ë a + b 2 ˜¯ fi ct4 – xt3 + yt – c = 0
As we know that, which is passing through (a, b), so
sec2 j – tan2 j = 1 ct4 – at3 + bt – c = 0.
2 2
Ê a ˆ Ê b ˆ Let its four roots are t1, t2, t3, t4.
a2 Á 2 -b2 Á 2 =1
Ë a + b 2 ˜¯ Ë a + b 2 ˜¯
a
Therefore, t1 + t2 + t3 + t4 = ,
fi a2a2 – b2b2 = (a2 + b2)2 c
Hence, the locus of (a, b) is b
a2x2 – b2y2 = (a2 + b2)2 S(t1t2) = 0, S(t1t2t3) = -
c
44. The equation of any normal to the hyperbola and S(t1t2t3t4) = 1.
x2 y 2
- = 1 at (f) is (i) x1 + x2 + x3 + x4 = c(t1 + t2 + t3 + t4 )
a 2 b2
ax cosj + by cotj = a2 + b2 Êaˆ
= cÁ ˜ = a
Ë c¯
Y
Ê1 1 1 1ˆ
(ii) y1 + y2 + y3 + y4 = c Á + + + ˜
Ë t1 t2 t3 t4 ¯
Q Ê Â (t1t2 t3 ) ˆ
= cÁ ˜
Ë Â (t1t2 t3t4 ) ¯
X¢ X ÊÊ bˆˆ
A¢ O A G
Á ÁË - c ˜¯ ˜
= cÁ ˜ =b
Á -1 ˜
ÁË ˜¯

CG_06.indd 32 2/6/2017 4:06:37 PM


Hyperbola 6.33

(iii) x12 + x22 + x32 + x42 = c 2 (t12 + t22 + t32 + t42 ) Ê 1 ˆ Â ( x1 x2 x3 ) 2e 2


Therefore,  ÁË x ˜¯ = x x x x = b
= c2 {(Â t ) - 2Â (t t )}
1
2
12
1 1 2 3 4

Ê1 1 1 1ˆ
ÏÔÊ a ˆ 2 ¸Ô (i) ( x1 + x2 + x3 + x4 ) Á + + + ˜
= c Ì Á ˜ - 0˝ = a 2
2
Ë x1 x2 x3 x4 ¯
Ë ¯
ÓÔ c ˛Ô Ê ˆ 2
= ( x1 )ÁË Â ÊÁË x1 ˆ˜¯ ˜¯ = 2eb2 ¥ 2be =4
(Â y1 )
2
(iv) y12 + y22 + y32 + y42 = - 2Â ( y1 y2 ) 1

Ê 1 ˆ (ii) Similarly we can have


= (b )2 - 2 c 2 Â Á
Ë t1t2 ˜¯ Ê1 1 1 1ˆ
( y1 + y2 + y3 + y4 ) Á + + +
Ê Â t1 t2 ˆ Ë y1 y2 y3 y4 ˜¯
= (b )2 - 2 c 2 Á ˜ Ê ˆ
Ë t1t2t3t4 ¯ = ( y1 )ÁË Â ÊÁË y1 ˆ˜¯ ˜¯
= (b )2 1

= 4.
(v) x1 ◊ x2 ◊ x3 ◊ x4 = c4(t1t2t3t4) = –c4
47. The equation of the chord of contact of tangents drawn
Ê 1 ˆ from the point (2, 3) is
(vi) y1 ◊ y2 ◊ y3 ◊ y4 = c 4
ÁË t t t t ˜¯ 2x y
12 3 4
- =1
Ê 1ˆ 9 2
= c 4 Á ˜ = -c 4
Ë -1¯ fi 4x – 9y = 18
46. the equation of any normal to the given hyperbola at x2 y 2
48. Any tangent to the hyperbola - = 1 is
(x, y) is 9 4
x y x y
( k - y ) = 2 ( x - b) sec q - tan q = 1 …(i)
a2 b 3 2
Y
Ê 1 1ˆ y kx
fi ÁË 2 + 2 ˜¯ xy - - 2 = 0
a b b a
b 2 kx
fi y= …(i)
a 2 (e 2 x - b ) X¢
C
X
M(h, k)
The equation of the hyperbola is B
x2 y 2
- =1 …(ii)
a 2 b2
Solving Eqs (i) and (ii), we get, Y¢
2 4 2 2
x b k x Let the tangent intersects the x-axis at A and y-axis at B,
- =1
a 2 b 2 a 4 (e 2 x - b ) 2 respectively.
Then A = (3 cos q, 0) and B = (0, –2 cot q)
fi a2e4x4 – 2ba2e2x3 – (a2b2 + b2k2 + a4e4)x2 + 2ba4e2x
Let (h, k) be the mid-point of AB.
+ a4b2 – 0 …(iii)
Therefore,
Let x1, x2, x3, x4 are the roots of Eq. (iii).
2h = 3 cos q and 2k = –2 cot q
2b
Then, x1 + x2 + x3 + x4 = , 3 1
e2 fi sec q = and tan q = -
2h k
a 2b2 + b2 k 2 + a 4 e2
 (x1x 2 ) = - a 2 e4
As we know that,
sec2 q – tan2 q = 1
2ba 2 9 1
 ( x1 x2 x3 ) = - e2
fi 2
- 2 =1
4h k
a 2b2 9 1
and  ( x1 x2 x3 x4 ) = e4
Hence, the locus of (h, k) is -
4 x2 y 2
= 1.

CG_06.indd 33 2/6/2017 4:06:38 PM


Hyperbola

6.34 Coordinate Geometry Booster

49. Any point on the circle x2 + y2 = a2 be (a cos q, a sin q). 2


Ê hx ky ˆ
Y x2 y 2 ÁË 2 - 2 ˜¯
a b
- =
Q a 2 b2 Ê h2 k 2 ˆ 2
Á a 2 - b2 ˜
Ë ¯
M(h, k) 2 2
P 1 Ê h2 k 2 ˆ 2 1 Ê h2 k 2 ˆ 2
X¢ X fi Á - ˜ x - 2 Á 2 - 2˜ y
O a 2 Ë a 2 b2 ¯ b Ëa b ¯
R
h 2 2 k 2 2 2hk
= x + 4 y - 2 2 xy …(iii)
a4 b a b
Y¢ Equation (iii) will be a right angle, if co-efficient of
The equation of the chord of contact from the point x2 + co-efficient of y2 = 0
2 2
(a cos q, a sin q) to the hyperbola x2 – y2 = a2 is 1 Ê h2 k 2 ˆ h2 1 Ê h2 k 2 ˆ k2
(a cos q)x – (a sin q)y = a2 fi Á - ˜ - - Á - ˜ - =0
a 2 Ë a 2 b2 ¯ a 4 b2 Ë a 2 b2 ¯ b4
fi x cos q – y sin q = a …(i)
2
Let the mid-point be (h, k). Ê h2 k 2 ˆ Ê 1 1 ˆ Ê h2 k 2 ˆ
The equation of the chord bisected at (h, k) to the hy- fi Á 2 - 2 ˜ ÁË 2 - 2 ˜¯ = Á 4 + 4 ˜
Ëa b ¯ a b Ëa b ¯
perbola x2 – y2 = a2 is
hx – ky = h2 – k2 …(ii) Hence, the locus of (h, k) is
Since the Eqs (i) and (ii) are identical, so 2
Ê x2 y 2 ˆ Ê 1 1 ˆ Ê x2 y 2 ˆ
h -k h2 - k 2 Á 2 - ˜ Á - ˜ = Á + ˜.
= = Ëa b2 ¯ Ë a 2 b2 ¯ Ë a 4 b4 ¯
cos q sin q a
51. Let the point on the parabola be (h, k).
ah - ak
fi cos q = 2 and sin q = 2 Y
h - k2 h - k2
Squaring and adding, we get
2 2
Ê ah ˆ Ê - ak ˆ Q
ÁË 2 ˜ +Á ˜ =1
h - k 2 ¯ Ë h2 - k 2 ¯ P(h, k)
Q
fi a (h + k ) = (h – k )
2 2 2 2 2 2
X¢ X
Hence, the locus of (h, k) is a2(x2 + y2) = (x2 – y2)2. O
50. Let the mid-point be (h, k). R
Y
A

M(h, k)
X¢ X The equation of the chord of contact of the parabola is
O
yk = 2a(x + h) …(i)
B
2a 2 ah
Y¢ fi y= x+ …(ii)
The equation of the chord bisected at (h, k) to the given k k
hyperbola is Since, the line (ii) is a tangent to the hyperbola
T = S1 x2 y 2
- = 1, so,
hx ky h 2 k 2 a 2 b2
fi - = - …(i) c2 = a2m2 – b2
a 2 b2 a 2 b2
The equation of the hyperbola is 2 2
Ê 2ah ˆ 2 Ê 2a ˆ 2
fi ÁË ˜ = a ÁË ˜¯ - b
x2 y 2 …(ii) k ¯ k
- =1
a 2 b2 fi 4a2h2 = 4a4 – k2b2
Since the chord (i) subtends right angle at the centre, so Hence, the locus of (h, k) is
we can write 4a2x2 = 4a4 – y2b2

CG_06a.indd 34 2/6/2017 4:08:05 PM


Hyperbola 6.35

fi hx – ky = h2 – k2
x2 y 2
52. Any tangent to the hyperbola 2 - 2 = 1 at R(q) is fi ky = hx + (k2 – h2)
a b
x y Ê hˆ Ê k 2 - h2 ˆ
sec q - tan q = 1 …(i) fi y = Á ˜ x+Á ˜ …(i)
a b Ë k¯ Ë k ¯
Y Y

P
M(h, k)
X¢ X
C M(h, k) X¢ X
O
B
Q

Y¢ Y¢
Let the mid-point of PQ be (h, k). If the line (i) touches the parabola y2 = 4 ax, so
Then the equation of the chord bisected at (h, k) to the
x2 y 2 a
ellipse + = 1 is c=
m
a 2 b2
hx ky h 2 k 2 Ê h2 - k 2 ˆ a ak
+ = + …(ii) fi Á k ˜ = ( h /k ) = h
a 2 b2 a 2 b2 Ë ¯
Therefore, the Eqs (i) and (ii) are identical. So fi h(h2 – k2) = ak2
secq /a - tan q /b 1 Hence, the locus of (h, k) is
= = 2
h k Êh k2 ˆ x(x2 – y2) = ay2
2 2 +
Á a 2 b2 ˜
a b Ë ¯ 54. If (h, k) be the mid-point of the chord of the hyperbola
x2 y 2
secq - tanq 1 - = 1 , then
fi = = 2 a 2 b2
h k Êh k2 ˆ
a b +
Á a 2 b2 ˜ T = S1
Ë ¯
hx ky h 2 k 2
h k fi - = -
a 2 b2 a 2 b2
fi sec q = a and tan q = b
Ê h2 k 2 ˆ Ê h2 k 2 ˆ ky hx Ê h 2 k 2 ˆ
Á a 2 + b2 ˜ Á a 2 + b2 ˜ fi = -Á - ˜
Ë ¯ Ë ¯ b2 a 2 Ë a 2 b2 ¯
We know that,
Ê b2h ˆ b2 Ê h2 k 2 ˆ
sec2 q – tan2 q = 1 fi y=Á 2 ˜ x - Á 2 - 2 ˜ …(i)
2 2 Ë a k¯ k Ëa b ¯
Ê hˆ Ê kˆ
ÁË ˜¯ ÁË ˜¯
a b
fi 2
- 2
=1
Ê h2 k 2 ˆ Ê h2 k 2 ˆ M(h, k) P
Á a 2 + b2 ˜ Á a 2 + b2 ˜
Ë ¯ Ë ¯
2 Q
Ê h2 k 2 ˆ Ê h2 k 2 ˆ
fi Á 2 - 2˜ =Á 2 + 2˜ X¢ O X
Ëa b ¯ Ëa b ¯
2
Ê x2 y 2 ˆ Ê x2 y 2 ˆ
Hence, the locus of (h, k) is Á 2 - 2 ˜ = Á 2 + 2 ˜ .
Ëa b ¯ Ëa b ¯

53. If (h, k) be the mid-point of the chord of the hyperbola
x2 – y2 = a2, then If the line (i) be a tangent to the circle x2 + y2 = c2, then
T = S1 C2 = A2(1 + m2)

CG_06a.indd 35 2/6/2017 4:08:06 PM


6.36 Coordinate Geometry Booster

2
b4 Ê h2 k 2 ˆ 2Ê b4h2 ˆ k - h -c 2
fi - = c 1 + Thus, = = 2
Á ˜ Á ˜ 1 t at
k 2 Ë a 2 b2 ¯ Ë a4k 2 ¯
fi (b2h2 – a2k2)2 = c2(a4k2 + b2h2) - h 2 -c 2
fi t= ,t =
Hence, the locus of (h, k) is k ak
(b2x2 – a2y2)2 = c2(a4y2 + b4x2) Eliminating t, we get,
2 h2a = –kc2
Ê x2 y 2 ˆ 2Ê x
2
y2 ˆ
fi Á 2 - 2˜ =c Á 4 + 4˜ Hence the locus of (h, k) is
Ëa b ¯ Ëa b ¯
yc2 + x2a – 0
55. If (h, k) be the mid-point of the chord of the circle Ê aˆ
x2 + y2 = a2, then fi y = Á - 2 ˜ x2.
Ë c ¯
T = S1
fi hx + ky = h3 + k2 57. Let the point P be (h, k).
Then the equation of the chord of contact of the circle
Ê hˆ Ê h2 + k 2 ˆ x2 + y2 = a2 is
fi y = Á- ˜ x+Á ˜ (i)
Ë k¯ Ë k ¯ hx + ky = a2 …(i)
Y Y
P
R
R
O Q
X¢ X X¢ X
O O
M(h, k) Q
P



2 2
x y The equation of the normal chord of the hyperbola
If the line (i) be a tangent to the hyperbola 2
- 2 = 1,
a b x2 y 2
so, c2 = a2m2 – b2 - = 1 at (f) is
2
a 2 b2
Ê h2 + k 2 ˆ 2
2Êh ˆ 2 ax cos j – by cot j = a2 + b2 …(ii)
fi Á k ˜ = a Á 2˜ -b
Ë ¯ Ëk ¯ Equations (i) and (ii) are identical. Therefore,
fi (h2 + k2)2 = (a2h2 – b2k2) a cos j b cot j (a 2 + b 2 )
=- =
Hence, the locus of (h, k) is h k a2
(x2 + y2)2 = (a2x2 – b2y2) a3 a 2b
56. Any tangent to the parabola y2 = 4ax at (at2, 2at) is sec j = and tan j = -
h( a 2 + b 2 ) k (a 2 + b 2 )
yt = x + at2 …(i)
Y
We have,
P sec2 j – tan2 j = 1
2 2
Ê a3 ˆ Ê a 2b ˆ
fi Á -
˜ Á - ˜ =1
M(h, k) Ë h( a 2 + b 2 ) ¯ Ë k ( a 2 + b 2 ) ¯

Ê a 6 a 4b 2 ˆ 2 2 2
fi Á 2 - 2 ˜ = (a + b )
X¢ X
O
Q Ëh k ¯
2
Ê a 2 b2 ˆ Ê a 2 + b2 ˆ
fi Á 2 - 2˜ = Á ˜
Y¢ Ëh k ¯ Ë a2 ¯

If (h, k) be the mid-point of the chord of the hyperbola Hence, the locus of (h, k) is
xy = c2, then 2
Ê a 2 b2 ˆ Ê a 2 + b2 ˆ
xk + yh = c2 …(ii) Á 2 - 2˜ = Á ˜
Therefore, the Eqs (i) and (ii) are identical. Ëx y ¯ Ë a2 ¯

CG_06a.indd 36 2/6/2017 4:08:07 PM


Hyperbola 6.37

58. If (h, k) be the mid-point of the chord of the hyperbola


x1 x2 a4
x2 y 2 fi =- 4
- = 1 , then y1 y2 b
a 2 b2 Thus, m = 4 and n = 4
T = S1 10 10
Ê m + nˆ Ê 4 + 4ˆ
hx ky h 2 k 2 Hence, the value of Á =Á = 210 = 1024.
fi - = - …(i) Ë 4 ˜¯ Ë 4 ˜¯
x2 b2 a 2 b2 60. The equation of the polar with respect to the hyperbola
Y
x2 y 2
P - = 1 is
a 2 b2
xx1 yy1
M(h, k) - =1
X¢ a 2 b2
O S
x( - ae) y ◊ 0
fi - 2 =1
Q a2 b
a
Y¢ fi x=-
e
which is passing through the focus (ae, 0) of the given
61. The equation of the polars from points (x1, y1) and (x2,
hyperbola.
y2) to the hyperbola
Therefore,
aeh h 2 k 2 x2 y2
= 2- 2 - = 1 are
a2 a b a 2 b2
xx1 yy1
h 2 k 2 eh - =1 …(i)
fi - = a 2 b2
a 2 b2 a
Hence, the locus of (h, k) is xx2 yy2
and - 2 =1 …(ii)
x2 y 2 Ê e ˆ a2 b
- =Á ˜x
a 2 b2 Ë a ¯ Now, slopes of (i) and (ii) are
59. The equations of the chord of contact of the tangents to b 2 x1 b2 x
the given hyperbola at (x1, y1) and (x2, y2) are m1 = 2
and m2 = 2 2
a y1 a y2
xx1 yy1
- =1 …(i) Since the polars of the given points are perpendicular,
a 2 b2
xx2 yy2 so
and - 2 =1 …(ii) m1 ¥ m2 = –1
a2 b
Y Ê b 2 x1 ˆ Ê b 2 x2 ˆ
fi Á 2 ˜ ¥ Á 2 ˜ = -1
Ë a y1 ¯ Ë a y2 ¯
x1 x2 a4
fi =- 4
Q(

y1 y2 b
x2

)
y1
,y

1,
2

(x
)

x1 x2 a 4
Q

X¢ X fi + =0
O
y1 y2 b 4
62. Let (x1, y1) be the pole of the hyperbola. Then the equa-
tion of the polar from a point (x1, y1) w.r.t. the hyperbola
x2 –3y2 = 3 is

xx1 – 3yy1 = 3 …(i)
The equation of the given polar is
The slopes of the lines (i) and (ii) are
x–y=3 …(ii)
Ê b2 x ˆ Ê b2 x ˆ Therefore, the Eqs (i) and (ii) are identical. So
m1 = Á 2 1 ˜ and m2 = Á 2 1 ˜
Ë a y1 ¯ Ë a y1 ¯ x1 -3y1 3
= =
Since, (i) and (ii) meet at right angles, so 1 -1 3
m1m2 = –1 1
fi x1 = 1, y1 =
Ê b 2 x1 ˆ Ê b 2 x1 ˆ 3
fi Á 2 ˜ ¥ Á 2 ˜ = -1 Ê 1ˆ
Ë a y1 ¯ Ë a y1 ¯ Hence, the pole is Á1, ˜ .
Ë 3¯

CG_06a.indd 37 2/6/2017 4:08:07 PM


6.38 Coordinate Geometry Booster

63. Let the pole be (h, k). Hence, the locus of (h, k) is
The equation of the polar from the point (h, k) w.r.t. the
y2
x2 y 2 x2 + = a2
hyperbola 2 - 2 = 1 is 4
a b fi 4x2 + y2 = 4a2
hx ky
- =1 …(i) 65. Let (h, k) be the pole.
a 2 b2 Then the equation of the polar w.r.t. the hyperbola
The equation of the normal chord of the given hyper-
x2 y 2
bola at (f) is - = 1 is
ax cos j – by cot j = (a2 + b2) …(ii) a 2 b2
Equations (i) and (ii) are identical. Therefore, hx ky
- =1
a cos j b cot j (a 2 + b 2 ) a 2 b2
= =
( h /a 2 ) ( k /b 2 ) 1 Ê b2h ˆ Ê b2 ˆ
fi y = Á 2 ˜ x+Á- ˜ …(i)
h k Ë a k¯ Ë k¯
fi cos j = (a 2 + b 2 )
3
, cot j = (a 2 + b 2 ) 3
a b The foci of the given hyperbola are (ae, 0) and (–ae, 0)
a3 b3 The equation of the circle is
fi sec j = 2 2 , tan j = 2 2 (x – ae)(x + ae) + y2 = 0
(a + b )h (a + b )k
fi x2 + y2 = (ae)2 …(ii)
We know that, If the line (i) be a tangent to the circle (ii), then
sec2 j – tan2 j = 1 c2 = a2(1 + m2)
2 2
Ê a3 ˆ Ê b3 ˆ b4 2Ê b4k 2 ˆ
fi Á 2 2 ˜ - Á 2 2 ˜ =1 fi = ( ae ) Á 1 + ˜
Ë (a + b )h ¯ Ë (a + b )k ¯ k2 Ë a 4h2 ¯
2 2
Ê a3 ˆ Ê b3 ˆ b4 4 2 4 2
fi - 2 2 2 2Êa h +b k ˆ
Á h˜ Á k ˜ = (a + b ) fi = e Á ˜
Ë ¯ Ë ¯ k2 Ë a2k 2 ¯
Hence, the locus of (h, k) is
a 2b 4
(a6y2 – b6x2) = (a2 + b2)2(x2y2) fi 2
= (a 4 h 2 + b 4 k 2 )
64. Let (h, k) be the pole. e
The equation of the polar from the point (h, k) w.r.t. the a 4b 4
parabola y2 = 4ax is fi (a 4 h 2 + b 4 k 2 ) =
(a 2 + b 2 )
yk = 2a(x + h) = 2ax + 2ah
Ê 2a ˆ Ê 2 ah ˆ Ê h2 k 2 ˆ 1
fi y =Á ˜ x+Á fi Á 4 + 4˜ = 2
Ë k ˜¯
…(i)
Ë k ¯ Ëa b ¯ (a + b 2 )
Y Ê x2 y 2 ˆ 1
Hence, the locus of (h, k) is Á 4 + 4 ˜ = 2
Q Ëa b ¯ (a + b 2 )
66. The equation of the chord is
P 7x + y = 2y = –7x + 2 …(i)
X¢ X Hence, the equation of the diameter is
b2 x 7x x
y= 2 = =-
R a m 3 ¥ ( -7) 3
x + 3y = 0
Y¢ 67. The given line is 3x + 4y + 10 = 0
If the line (i) be a tangent to the hyperbola x2 – y2 = a2,
Ê 3ˆ Ê 5ˆ
then y = Á- ˜ x+Á- ˜ …(i)
c2a2m2 – a2 Ë 4¯ Ë 2¯
2 2 Hence, the equation of the diameter corresponds to the
Ê 2ah ˆ 2 Ê 2a ˆ 2
fi ÁË ˜¯ = a ÁË ˜¯ - a line (i) is
k k
b2 x 4x 16
fi 2
4h = 4a – k 2 2 y= 2
= =- x
a m 9Ê - 3ˆ 27
k2 ÁË ˜¯
4
fi h2 + = a2
4 fi 16x + 27y = 0

CG_06a.indd 38 2/6/2017 4:08:08 PM


Hyperbola 6.39

2 73. The combined equation of the asymptotes is


68. The slope of the given chord is m = ÊÁ - ˆ˜ . (2x – y – 3)(3x + y – 7) = 0
Ë 3¯
fi 6x2 – xy – y2 – 23x + 4y + 21 = 0
Hence, the equation of the diameter parallel to the Let the equation of the hyperbola be
given chord is 6x2 – xy – y2 – 23x + 4y + l = 0,
b2 x 4x 2 where l is any constant such that it represents two
y=- 2 =- =- x
a m Ê 2ˆ 3 straight lines which passes through (1, 1), so l = 15.
9Á - ˜
Ë 3¯ Hence, the equation of the hyperbola becomes
fi 2x + 3y = 0 6x2 – xy – y2 –23x + 4y + 15 = 0.
69. Let the equation of the diameter, which is conjugate to 74. The combined equation of the asymptotes parallel to
x = 2y is the lines 2x + 3y = 0 and 3x + 2y = 0 is
y = m1 x. (2x + 3y + l)(3x + 2y + m) = 0
Ê 1ˆ
As we know that two diameters y = Á ˜ x and y = m1x which is passing through (1, 2).
Ë 2¯ Therefore,
are conjugates, if (2 ◊ 1 + 3 ◊ 2 + l)(3 ◊ 1 + 2 ◊ 2 + m) = 0
b2 fi (8 + l)(7 + m) = 0
m1 m2 =
a2 fi l = –8, m = –7
1 16 Thus, the combined equation of the assymptotes is
fi m1 ¥ = (2x + 3y – 8)(3x + 2y – 7) = 0
2 9
32 Let the equation of the hyperbola be
fi m1 = 2x + 3y = 0 and 2x – 3y = 0
9
which is passing through (5, 3). So
Hence, the equation of the conjugate diameters is
(2 ◊ 5 + 3 ◊ 3 – 8)(3 ◊ 5 + 2 ◊ 3 – 7) + l = 0
32
y= x fi 11 ¥ 14 + l = 0
9 fi l = –154
fi 32x = 9y
Hence, the equation of the hyperbola is
70. Equations of the asymptotes to the hyperbola
(2x + 3y – 8)(3x + 2y – 7) – 154 = 0.
xy – 2x – 3y = 0 is
75. The equation of the given hyperbola is
xy – 2x – 3y + l = 0,
x2 – 2y2 = 2.
where l is any constant such that it represents two
So, the equations of its asymptotes are
straight lines.
Therefore, x - 2 y = 0 and x + 2 y = 0.
abc + 2 fgh – af 2 – bg 2 – ch 2 = 0 Let any point on the hyperbola be P ( 2 sec j , tan j ) .
2 Let PM and PN are two perpendiculars from the point
Ê -3 ˆ Ê 1ˆ Ê 1ˆ
fi 0 + 2 ¥ Á ˜ ¥ ( -1) ¥ Á ˜ - 0 - 0 - l Á ˜ = 0 P to the asymptotes.
Ë 2¯ Ë 2¯ Ë 2¯
Then, PM ◊ PN
fi l=6
Hence, the required asymptotes are 2 secj - 2 tanj 2 secj + 2 tanj
= ¥
xy – 2x – 3y + 6 = 0 1+ 2 1+ 2
fi (x – 2)(y – 3) = 0
2
fi x = 2 and y = 3 = ¥ (sec j - tan j ) ¥ (sec j + tan j )
71. The equations of the asymptotes of the given curve is 3
3x2 + 10xy + 8y2 + 14x + 22y + l = 0, 2
= ¥ (sec 2 j - tan 2 j )
where l is any constant such that it represents two 3
straight lines. 2
= .
Therefore, 3
abc + 2fgh – af2 – bg2 – ch 2 = 0 x2 y 2
fi 3 ◊ 8 ◊ l – +2 ◊ 7 ◊ 11 ◊ 5 – 3 ◊ 121 – 8 ◊ 49 – l ◊ 25 = 0 76. The equation of the given hyperbola is - = 1.
9 4
fi 24l – + 770 – 363 – 392 – 25l = 0 Thus, the equations of the asymptotes are
fi l = 15
Hence, the combined equation of the given asymptotes Êx yˆ Ê x yˆ
ÁË + ˜ Á - ˜ =0
is 3 2¯ Ë 3 2¯
3x2 + 10xy + 8y2 + 14x + 22y + 15 = 0
fi Ê x yˆ Ê x yˆ
72. Since the asymptotes are perpendicular to each other, ÁË + ˜¯ = 0 and ÁË - ˜¯ = 0
so the hyperbola is rectangular. Hence, its eccentricity 3 2 3 2
is 2 . fi 2x + 3y = 0 and 2x – 3y = 0

CG_06a.indd 39 2/6/2017 4:08:09 PM


6.40 Coordinate Geometry Booster

The equation of any tangent to the hyperbola The equation of the tangent to the hyperbola (i) at P is
x y
x2 y 2 sec j - tan j = 1 …(ii)
- = 1 is a b
9 4 The equations of the asymptotes of the hyperbola (i)
x y are
sec j - tan j = 1
3 2 bx – ay = 0 and bx + ay = 0
Let the points of intersection of 2x + 3y = 0, 2x – 3y = 0 Let the points of the intersection of the asymptotes and
the tangent are O, Q, R respectively.
x y
and sec j - tan j = 1 are O, P and Q respectively. Then, O = (0, 0),
3 2 Q = [a(sec j + tan j), b(sec j + tan j)] and
Therefore, O = (0, 0), R = [a(sec j – tan j), b(sec j – tan j)]
Ê 3 2 ˆ Clearly, mid-point of QR is (a sec j, b tan j), which is
P=Á ,-
Ë secj + tanj secj + tanj ¯˜ co-ordinates of P.
Thus, the area of DOQR
Ê 3 2 ˆ
and Q=Á , 0 0
Ë secj - tanj secj - tanj ¯˜
1 a (secj + tan j ) b (secj + tanj )
Hence, the area of DOPQ =
2 a (secj - tanj ) -b(secj - tanj )
0 0 0 0
3 2 1
- = - ab - ab
1 sec j + tan j sec j + tan j 2
=
2 3 2 = ab.
sec j - tan j sec j - tan j 79. The equations of the asymptotes of the given hyperbola
0 0 x2 y 2
- = 1 are
a 2 b2
1
= (6 + 6) = 6 s. u. bx – ay = 0 and bx + ay = 0
2 Let P be any point on the given hyperbola be
77. Let the point P be (a sec j, b tan j). (a sec j, b tan j).
The equations of the asymptotes of the given hyperbola It is given that, p1 and p2 be the lengths of perpendicu-
x2 y 2 lars from the point P to the asymptotes
- = 1 are bx – ay = 0 and bx + ay = 0
a 2 b2
bx – ay = 0 and bx + ay = 0 ab sec j - ab tan j
Let PM and PN be two perpendiculars from the point P Thus, p1 =
b2 + a 2
to the transverse axis and the asymptote
bx – ay = 0 ab sec j + ab tan j
and p2 =
Thus, PM = b tan j b2 + a 2
ab sec j - ab tan j 1
and PN = Therefore,
b2 + a 2 p1 p2
It is given that, PM = PN Ê a 2 + b2 ˆ
=Á ˜
ab sec j - ab tan j Ë ( ab sec j - ab tan j )( ab sec j + ab tan j ) ¯
fi b tan j =
b2 + a 2 Ê a 2 + b2 ˆ 1 1
=Á 2 2 ˜ = 2 + 2
Ë a b ¯ a b
sec j - tan j
fi tan j = a Hence, the result.
b2 + a 2 Ê cˆ
80. The equation of the normal at Á ct1 , ˜ is
Ë t ¯ 1
78. Let the equation of the hyperbola be
t13 x - t1 y - ct14 +c= 0 …(i)
x2 y 2
- =1 …(i) Also it meets the hyperbola again at
a 2 b2
and any point on the hyperbola (i) be Ê cˆ
ÁË ct2 , t ˜¯
P(a sec j, b tan j) 2

CG_06a.indd 40 2/6/2017 4:08:10 PM


Hyperbola 6.41

Therefore, Eliminating t, we get


t1 (h2 – k2)2 + 4c2hk = 0
ct2t13 - c - ct14 + c = 0
t2 Hence, the locus of (h, k) is
t1 4 (x2 – y2)2 + 4c2xy = 0
fi t2t13 - - t1 + 1 = 0
t2 83. The equations of the asymptotes of the rectangular
hyperbola xy = c2 are x = 0 and y = 0.
fi t22t13 - t1 - t14t2 + t2 = 0
Clearly, the angle between the asymptotes
fi t2t13 (t2 - t1 ) + (t2 - t1 ) = 0
p
fi (t2t13 + 1)(t2 - t1 ) = 0 =
2
fi (t2t13 + 1) = 0 ( t1 π t2) p
fi 2a =
2
fi t2t13 = -1
fi p
Hence, the result. a=
81. Let P, Q and R are the vertices of a triangle such that 4
Ê cˆ Ê cˆ Ê cˆ Thus, the eccentricity,
P = Á ct1 , ˜ , Q = Á ct2 , ˜ , R = Á ct3 , ˜
Ë t1¯ Ë t 2¯ Ë t3¯ Êpˆ
e = 2 = sec Á ˜ = sec a
c c Ë 4¯
-
t t 1
Now, slope of QR = 3 2 = - 84. Let the equation of the circle be
ct3 - ct2 t2t3 x2 + y2 + 2gx + 2fy + c + 0 …(i)
Therefore, slope of PM is t2t3. and the equation of the given hyperbola be
The equation of the perpendicular PM on QR is xy = 1 …(ii)
c Solving, we get
y - = t2t3 ( x - ct1 ) …(i)
t1 1 2f
x2 + 2
+ 2 gx + +c = 0
Similarly, the equation of the perpendicular BN on PR is x x
c fi x4 + 2gx3 + cx2 + 2fx + 1 = 0
y- = t1t3 ( x - ct2 ) …(ii)
t2 Let its roots are x1, x2, x3 and x4.
Solving Eq. (i) and (ii), we get, Then, x1x2x3x4 = 1
Similarly, we can easily prove that
c y1y2y3y4 = 1
x=- and y = - ct1t2t3 .
t1t2t3 85. Any point on the rectangular hyperbola
Ê c ˆ Ê cˆ
Thus, the point Á - , - ct1t2t3 ˜ lies on the rectan- xy = c2 is P Á ct , ˜
Ë t1t2t3 ¯ Ë t¯
gular hyperbola xy = c . 2 The equation of any tangent to the rectangular hyperbola
Hence, the result. xy = c2 at t is
82. The equation of the normal to the rectangular hyperbola x
+ yt = 2c …(i)
xy = c2 at t is t
c The equation of any normal to the rectangular hyperbola
xt 2 - y = ct 3 - …(i)
t xy = c2 at t is
Let the pole be (h, k). xt3 – yt – xt4 + c = 0 …(ii)
Then the equation of the polar from the point (h, k) to 2c
the rectangular hyperbola Therefore, a1 = 2ct , a2 =
t
xy = c2 is
Ê 1ˆ Ê1 ˆ
xk + yh = 2c2 (ii) and b1 = c Á t - 3 ˜ , b2 = c Á - t 3 ˜
Therefore, the Eqs (i) and (ii) are identical. So Ë t ¯ Ët ¯

k h 2c 2 Now,
= =
t 2
-1 ct 3 - c Ê 1 ˆ 2c 2 Ê1 3ˆ
a1a2 + b1b2 = 2c 2 t Á t - 3 ˜ + ÁË - t ˜¯
t Ë t ¯ t t
k 2ct Ê 1 1 ˆ
fi t2 = - and h = - 4 = 2c 2 Á t 2 - 2 + 2 - t 2 ˜ = 0
h t -1 Ë t t ¯

CG_06a.indd 41 2/6/2017 4:08:10 PM


6.42 Coordinate Geometry Booster

86. We have, e1 = 2 and e2 = 2 Ê 2ct t 2c ˆ


Q=Á 13,
Now, (e1 + e2 ) 2 = ( 2 + 2) 2 = (2 2) 2 = 8 Ë t1 + t3 t1 + t3 ˜¯
87. Any point on the given hyperbola Ê 2ct t 2c ˆ
and R = Á 2 3 , respectively
x2 Ë t2 + t3 t2 + t3 ˜¯
- y 2 = 1 be P ( 2 secj , tanj ) .
2 Hence, the area of the DPQR
The equations of the asymptotes of the given hyperbola
are 2ct1t2 2c
1
x - 2 y = 0 and x + 2 y = 0. t1 + t2 t1 + t2
Let PM and PN be the lengths of perpendiculars from 1 2ct1t3 2c
= 1
the point P on the asymptotes. 2 t1 + t3 t1 + t3
Thus,
2ct2t3 2c
PM.PN 1
t2 + t3 t2 + t3
2secj - 2 tanj 2secj + 2 tanj
= ¥ 2c 2 (t1 - t2 )(t2 - t3 )(t3 - t1 )
1+ 2 1+ 2 =
(t1 + t2 )(t1 + t2 )(t1 + t2 )
2
= 89. The given rectangular hyperbola is
3
xy = 18 …(i)
= (1 ± 2 2, 2)
Replacing x by x cos (45°) + y sin (45°) and y by –x sin
88. Let the points A, B, C be (45°) + y cos (45°) in (i), we get
Ê cˆ Ê cˆ Ê cˆ Ê x y ˆÊ x y ˆ
ÁË ct1 , t ˜¯ , ÁË ct2 , t ˜¯ , ÁË ct3 , t ˜¯ respectively. ÁË
2
+
2
˜¯ ÁË -
2
+ ˜ = 18

1 2 3
(i) Then the area of the DABC fi x2 – y2 = –18
Hence, the length of the transverse axis
c
ct1 1 = 2a = 2.6 = 12 units.
t1 90. Let (h, k) be any point.
1 c The equation of the chord of contact of the tangents
= ct2 1
2 t2 from (h, k) to the circle x2 + y2 = 4 is
hx + ky = 4.
c
ct3 1 Also, the given hyperbola is
t3 xy = 1
t12 1 t1 Ê 4 - hx ˆ
fi xÁ =1
Ë k ˜¯
2
1 c
= t22 1 t2
2 t1t2t3 fi 4x – hx2 = k
t32 1 t3
fi hx2 – 4x + k = 0
c 2 (t1 - t2 )(t2 - t3 )(t3 - t1 ) Thus, its roots are equal. So
= ¥ D=0
2 t1t2t3
fi 16 – 4hk = 0
(ii) The equations of the tangents at A, B and C are fi hk = 4
x Hence, the locus of (h, k) is xy = 4.
+ yt1 = 2c …(i)
t1 91. The combined equation of the asymptotes of the given
x hyperbola is
+ yt2 = 2c …(ii) xy – hx – ky + l = 0
t2
where l is any constant such that it represents two
x straight lines.
and + yt3 = 2c …(iii)
t3 Therefore,
Thus, the points of intersections of (i) and (ii), (i) abc + 2fgh – af 2 – bg 2 – ch2 = 0.
and (iii), (ii) and (iii) meet at P, Q, R respectively.
fi Ê k ˆ Ê hˆ Ê 1ˆ l
0 + 2Á - ˜ Á - ˜ Á ˜ - = 0
Ê 2ct t 2c ˆ Ë 2¯ Ë 2¯ Ë 2¯ 4
Thus, P = Á 1 2 , ,
Ë 1 2 1 + t2 ˜¯
t + t t fi l = hk

CG_06a.indd 42 2/6/2017 4:08:11 PM


Hyperbola 6.43

Hence, the asymptotes are If (i) and (ii) are the same, then
xy – hx – ky + hk = 0 a2m2 – b2 = –b2m2 + a2
fi (x – h)(y – k) = 0 fi (a2 + b2) m2 = (a2 + b2)
fi (x – h) = 0 and (y – k) = 0
fi m2 = 1
Ê bˆ
92. We have, q = 2 tan -1 Á ˜ fi m = ±1
Ë a¯
Hence, the equation of the common tangent be
Êqˆ b
fi tan Á ˜ = y = ± x ± a 2 - b2 .
Ë 2¯ a
Also, 3. The equation of any tangent to the hyperbola
b2 Êqˆ Êqˆ x2 y 2
e = 1+ = 1 + tan 2 Á ˜ = sec Á ˜ - = 1 is
a 2 Ë 2 ¯ Ë 2¯ 16 9
Êqˆ 1 y = mx + 16m 2 - 9
fi cos Á ˜ =
Ë 2¯ e
93. The equation of the given hyperbola is fi mx - y + 16m 2 - 9 = 0
9x2 – 16y2 = 144 which is also a tangent of x2 + y2 = 9. So
2 2
x y 16m 2 - 9
fi - =1 …(i)
16 9 =3
Let any point on the given hyperbola be P(8, k). m2 + 1
Since the point P lies on (i), so
fi 16m 2 - 9 = 3 m 2 + 1
2
64 k
- =1 fi 16m2 – 9 = 9m2 + 9
16 9
fi 16m2 – 9m2 = 9 + 9
fi k2 = 27
fi 7m2 = 18
fi k =3 3
3 2
Hence, the co-ordinates of P be (8, 3 3) . fi m=±
7
Thus, the equation of the reflected ray is Hence, the equation of the common tangent be
0-3 3 Ê3 2ˆ 288
y -3 3 = ( x - 8) y = ±Á ˜ x+ 7 -9
-5 - 8 Ë 7¯
fi 3 3 x - 13y + 15 3 = 0
fi 7 y = ± (3 2) x + 15
4. The equation of any normal to the hyperbola
LEVEL III x2 y 2
- = 1 at (f) is
1. As we know that y = mx + c will be the tangent to the a 2 b2
ax cos j + by cot j = a2 + b2
x2 y 2
hyperbola 2 - 2 = 1 if Y
a b
c = a m – b2 = 9m2 – 4
2 2 2

Hence, the equation of the hyperbola is


x2 y 2 Q
- =1
9 4
2. The equation of any tangent to the hyperbola X¢ X
A¢ O A G
2 2
x y 2 2 2
2
- 2 = 1 is y = mx + a m - b …(i)
a b
and the equation of any tangent to the hyperbola

2 2 2 2
x y x y Ê a 2 + b2 ˆ
- = -1 , i.e. - = 1 is Thus, the co-ordinates of G = Á , 0˜
b2 a 2 ( -b 2 ) ( - a 2 ) Ë a cos j ¯
y = mx + ( -b 2 )m 2 + a 2 …(ii) Clearly, the vertices, A = (a, 0) and A¢ = (–a, 0)

CG_06a.indd 43 2/6/2017 4:08:11 PM


6.44 Coordinate Geometry Booster

(v) x1x2x3x4 = c4(t1t2t3t4) = –c4


Ê a 2 + b2 ˆ
Now, AG = Á - a˜ Ê 1 ˆ
Ë a cos j ¯ (vi) y1y2y3y4 = c 4 4Ê 1 ˆ 4
ÁË t t t t ˜¯ = c ÁË -1˜¯ = -c
12 3 4
Ê a 2 + b2 ˆ
and A¢ G = Á + a˜ 6. The equation of any normal to the given hyperbola at
Ë a cos j ¯
(x, y) is
Therefore, x y
Ê a 2 + b2 ˆ Ê a 2 + b2 ˆ 2
( k - y ) = 2 ( x - b)
AG ◊ A¢G = Á - a˜ Á + a˜ a b
Ë a cos j ¯ Ë a cos j ¯ Ê 1 1ˆ y kx
fi ÁË 2 + 2 ˜¯ xy - - 2 = 0
Ê Ê a 2 + b2 ˆ 2 ˆ a b b a
2 2
= ÁÁ sec j - a ˜
ÁË Ë a ˜¯ ˜¯ b 2 kx
fi y= …(i)
= (a c sec j – a )
2 2 2 2 a 2 (e 2 x - b )
= a2 (e2 sec2 j – 1) The equation of the hyperbola is
fi m = 2, n = 2, p = 2 x2 y 2
Hence, (m+ n + p)2 + 36 = 36 + 36 = 72. - =1 …(ii)
a 2 b2
5. The equation of the normal to the hyperbola xy = c2 at
Solving Eqs (i) and (ii), we get
Ê cˆ
ÁË ct , ˜¯ is
t x2 b4k 2 x2
- =1
xt3 – yt – ct4 + c = 0 a 2 b 2 a 4 (e 2 x - b ) 2
fi ct4 – xt3 + yt – c = 0
fi a2e4x4 – 2ba2e2x3 – (a2b2 + b2k2 + a4e4) x2
which is passing through (a, b), so ct4 – at3 + bt – c = 0
+ 2ba4e2x + a4b2 = 0 …(iii)
Let its four roots are t1t2, t3, t3.
Let x1, x2, x3, x4 are the roots of Eq. (iii).
Therefore,
a 2b
t1 + t2 + t3 + t4 = , S(t1t2) = 0, Then, x1 + x2 + x3 + x4 = ,
c e2
b a 2b2 + b2 k 2 + a 4 e2
S(t1t2t3) = - and S(t1t2t3t4) = 0.
c  ( x1 x2 ) = - a 2 e4
(i) x1 + x2 + x3 + x4 = c(t1 + t2 + t3 + t4 )
2ba 2
Êaˆ
= cÁ ˜ = a
 ( x1 x2 x3 ) = - e2
Ë c¯
a 2b2
Ê1 1 1 1ˆ and  ( x1 x2 x3 x4 ) = e4
(ii) y1 + y2 + y3 + y4 = c
ÁË t + t + t + t ˜¯
1 2 3 4
Ê 1ˆ Â ( x1x2 x3 ) = 2e2
ÊÊ bˆˆ Therefore,  ÁË x ˜¯ = x1 x2 x3 x4 b
Ê Â (t1t2t3 ) ˆ Á ËÁ - c ¯˜ ˜ 1
= cÁ ˜ = cÁ ˜ =b Ê1 1 1 1ˆ
Ë Â (t1t2t3t4 ) ¯ Á -1 ˜ (i) ( x1 + x2 + x3 + x4 ) Á + + + ˜
ÁË ˜¯ Ë x1 x2 x3 x4 ¯
Ê ˆ 2
(iii) x12 + x22 + x32 + x42 = c 2 (t12 + t22 + t32 + t42 ) = ( x1 )ÁË Â ÊÁË x1 ˆ˜¯ ˜¯ = 2eb2 ¥ 2be =4
=c 2
{(Â t ) - 2Â (t t )}
1
2
12
(ii) Similarly, we can have
1

ÏÔÊ a ˆ 2 ¸Ô
= c 2 Ì Á ˜ - 0˝ = a 2 Ê1 1 1 1ˆ
Ë ¯ ( y1 + y2 + y3 + y4 ) Á + + + ˜
ÓÔ c ˛Ô Ë y1 y2 y3 y4 ¯
(Â y1 )
2
2 2 2 2
(iv) y1 + y2 + y3 + y4 = - 2Â ( y1 y2 ) Ê ˆ
Ê 1 ˆ
= ( y1 )ÁË Â ÊÁË y1 ˆ˜¯ ˜¯
1
= (b )2 - 2 c 2  ÁË t t ˜¯ = 4.
12

Ê Â t1 t2 ˆ 7. The equation of the normal to the hyperbola


= (b )2 - 2 c 2 Á ˜
Ë t1t2t3t4 ¯ x2 y 2
- = 1 at (a sec q, b tan q) is
a 2 b2
= (b )2

CG_06a.indd 44 2/6/2017 4:08:12 PM


Hyperbola 6.45

ax cos q + by cot q = a2 + b2 …(i) 9. Let the mid-point be M(h, k)


Let the point Q be (a, b). The equation of the chord of the hyperbola
The equation of any line perpendicular to (i) is x2 y 2
(b cot q)x – (a cos q)y + k = 0 - = 1 is
a 2 b2
which is passing through the centre. So
T = S1
k=0
Thus, (b cot q)x – (a cos q) y = 0 hx ky h 2 k 2
fi - = -
which meets at Q. a 2 b2 a 2 b2
Thus, First we make it a homogeneous equation of 2nd
(b cot q)a – (a cos q)b = 0 degree.
2
ba Ê hx ky ˆ
fi sinq = Á a 2 - b2 ˜
ab Ê x2 y 2 ˆ
Á 2 2˜
=Á 2 - 2˜
a 2 b 2 - b 2a 2 Áh -k ˜ Ëa b ¯
ÁË 2 2˜
fi cosq = a b ¯
ab
Ê h2 x2 k 2 y 2 ˆ Ê x2 y 2 ˆ Ê h2 k 2 ˆ
fi Á 4 + 4 - 2(…) xy˜ = Á 2 - 2 ˜ Á 2 - 2 ˜
a 2 b 2 - b 2a 2 Ë a b ¯ Ëa b ¯Ëa b ¯
and cotq =
ba which subtends right angle at the centre, i.e.
Putting the values of cos q and cot q, in Eq. (i) we get co-efficient of x2 + co-efficient of y2 = 0
Ê a 2 b 2 - b 2a 2 ˆ Ê a 2 b 2 - b 2a 2 ˆ Ê h2 1 Ê h2 k 2 ˆ k 2 1 Ê h2 k 2 ˆ ˆ
2 2
ax Á ˜ + by Á ˜ = a +b Á 4 - 2 Á 2 - 2 ˜ + 4 + 2 Á 2 - 2 ˜˜ = 0
ÁË ab ˜¯ ÁË ba ˜¯ Ëa a Ëa b ¯ b b Ëa b ¯¯

Hence, the locus of Q is Ê h2 k 2 ˆ Ê 1 1 ˆ Ê h2 k 2 ˆ


fi Á 4 + =
˜ Á + ˜Á - ˜
Ëa b4 ¯ Ë a 2 b2 ¯ Ë a 2 b2 ¯
x
y (ay 2 2
- b2 x2 + ) xy ( a y - b x ) = a + b
2 2 2 2 2 2
Hence, the locus of M(h, k) is


Ê x yˆ
ÁË y + x ˜¯ ( a2 y2 -b x )= a +b
2 2 2 2 Ê x2 y 2 ˆ Ê 1
Á 4
Ëa
+ =
˜ Á +
1 ˆ Ê x2 y 2 ˆ
˜Á - ˜
b4 ¯ Ë a 2 b2 ¯ Ë a 2 b2 ¯
2
Ê x yˆ 2 2 2 2 2 2 2 10. Let the point P be (h, k).
fi ÁË y + x ˜¯ (a y - b x ) = (a + b ) The equation of the tangent to the parabola
y2 = 4ax at P is
fi (x2 + y2)2(a2y2 – b2x2) = (a2 + b2)2 yk = 2a(a + h)
8. Any point on the circle x2 + y2 = a2 is (a cos q, a sin q). 2ax 2ah
The chord of contact of this point with respect to the fi y= +
k k
hyperbola x2 – y2 = a2 is which is a tangent to the hyperbola. So
x cos q – y sin q = a …(i) c2 = a2m2 – b2
If its mid-point be (h, k), then it is same as 2 2
Ê 2ah ˆ 2 Ê 2a ˆ 2
T = S1 fi ÁË ˜ = a ÁË ˜¯ - b
i.e. hx – ky = h2 – k2 k ¯ k
Comparing Eqs (i) and (ii), we get 4a 2 h 2 4a 4
fi 2
= 2 - b2
cosq sinq a k k
= = 2
h k h - k2 fi 4a2h2 + k2b2 = 4a4
Hence, the locus of (h, k) is
We know that
sin2 q + cos2 q = 1 4a2x2 + b2y2 = 4a2
2 2
11. Let M(h, k) be the mid-point of the hyperbola
Ê ah ˆ Ê ak ˆ x2 – y2 = a2
fi ÁË 2 2˜
+Á 2 =1
h -k ¯ Ë h - k 2 ˜¯ The equation of the chord at M is
fi a2(h2 + k2) = (h2 – k2)2 T = S1
Hence, the locus of (h, k) is fi hx – ky = h2 – k2
a2(x2 + y2) = (x2 – y2)2 fi ky = hx – (h2 – k2)

CG_06a.indd 45 2/6/2017 4:08:12 PM


6.46 Coordinate Geometry Booster

Hence, the locus of the mid-point M(h, k) is


Ê hˆ Ê h2 - k 2 ˆ 2
fi y = Á ˜ x-Á Ê x2 y 2 ˆ 2
y2 ˆ
Ë k¯ Ë k ¯
˜ 2Ê x
Á 2 - ˜ = c Á 4 + ˜
which is a tangent to the parabola y2 = 4ax. So Ëa b2 ¯ Ëa b4 ¯
a 14. Let the mid-point be M(h, k).
c= The equation of the chord of the circle
m
x2 + y2 = a2 is
Ê k 2 - h 2 ˆ ak T = S1
fi Á k ˜= h
Ë ¯ fi hx + ky = h2 + k2
fi ky = –hx + (h2 + k2)
fi ak2 = k2h – h3
fi k2(h – a) = h3 Ê hˆ Ê h2 + k 2 ˆ
fi y = -Á ˜ x+ Á ˜
Hence, the locus of M(h, k) is Ë k¯ Ë k ¯
y2(x – a) = x3 which is a tangent to the hyperbola. So
12. Let M(h, k) be the mid-point of the chord of length 2d c2 = a2m2 – b2
inclined at an angle q with the x-axis. Then its extremi- 2 2
Ê h2 + k 2 ˆ 2 Ê hˆ 2
ties are fi Á k ˜ = a ÁË ˜¯ - b
(h + d cos q ck + d sin q) Ë ¯ k
and (h – d cos q ck – d sin q) fi (h2 + k2)2 = (a2h2 – k2b2)
These extremities lie on the hyperbola xy = c2 Hence, the locus of M(h, k) is
So, (h + d cos q) (k + d sin q) = c2 …(i)
(x2 + y2)2 = (a2x2 – b2y2)
and (h – d cos q)(k – d sin q) = c2 …(ii)
15. Let the point P be (at2, 2at).
Adding and subtracting Eqs (i) and (ii), we get
hk + d2 sin q cos q = c2 …(iii) Y
and h sin q + k cos q = 0
P
k
i.e. tanq = - …(iv) Q
h M(h, k)
Eliminating q between Eqs (iii) and (iv), we get X¢
O
X
R
2
Êh +k 2ˆ 2
d k
(hk - c 2 ) Á 2 ˜= h
Ë h ¯
fi (hk – c2)(h2 + k2) = d2hk Y¢
Hence, the locus of M(h, k) is The equation of the tangent to the parabola at P is
(xy – c2)(x2 + y2) = d2xy yt = x + at2 …(i)
13. Let the mid-point be M(h, k). Given hyperbola is
x2 y 2 xy = c2 …(ii)
The equation of the chord of the hyperbola 2 - 2 = 1
is a b The equation of the chord of the hyperbola xy = c2 at
M(h, k) is
T = S1 xk + yh = 2hk …(ii)
hx ky h 2 k 2 So, Eqs (i) and (ii) are the same line. So
fi - = -
a 2 b2 a 2 b2 1 t at 2
2 2ˆ
=- =-
ky hx Ê h k k h 2 hk
fi = 2 -Á 2 - 2˜
b 2
a Ëa b ¯ h 2h
fi t = - and t 2 = -
k a
Ê b2h ˆ b2 Ê h2 k 2 ˆ 2
fi y=Á 2 ˜ x- Á 2 - 2˜ h 2h
Ëa k¯ k Ëa b ¯ fi 2
=-
k a
which is a tangent to the circle x2 + y2 = c2. So h 2
c2 = a2(1 + m2) fi =-
k2 a
2
Ê b2 Ê h2 k 2 ˆ ˆ b4h2 ˆ ah
2Ê fi k2 = -
fi Á Á 2 - ˜˜ = c Á 1 + ˜ 2
Ë k Ëa b2 ¯ ¯ Ë a4k 2 ¯
Hence, the locus of M(h, k) is
2 ax
b4 Ê h2 k 2 ˆ 2Ê b4h2 ˆ y2 = -
fi Á - ˜ = c Á 1 + ˜ 2
k 2 Ë a 2 b2 ¯ Ë a4k 2 ¯ which is a parabola.

CG_06a.indd 46 2/6/2017 4:08:13 PM


Hyperbola 6.47

16. The equation of the chord of contact of the circle Let the co-ordinates of the point P be (a, b).
x2 + y2 = a2 is hx + ky = a2 …(i) Since PM and PN are perpendicular to the axes, so the
and the equation of the normal chord of the hyperbola co-ordinates of P are
is Ê Ê a 2 + b2 ˆ ˆ
Ê a 2 + b2 ˆ
ax cos j + by cot j = a2 + b2 …(ii) ÁÁ ˜ sec j , Á ˜ tan j ˜
Solving, we get ËË a ¯ Ë b ¯ ¯
a cosj b cotj a 2 + b 2 Therefore,
= =
h k a2 Ê a 2 + b2 ˆ Ê a 2 + b2 ˆ
a =Á ˜ sec j and b = Á b ˜ tan j
(a 2 + b 2 )h (a 2 + b 2 )k Ë a ¯ Ë ¯
fi cosj = 3
, cotj =
a a 2b Ê a ˆ Ê b ˆ
fi aÁ 2 = sec j and b Á 2 = tan j
(a 2 + b 2 )h bh Ë a + b 2 ˜¯ Ë a + b 2 ˜¯
fi cos j = 3
, sin j =
a ak
As we know that, sec2 j – tan2 j = 1
We know that sin j + cos j = 1
2 2
2 2
Ê a ˆ Ê b ˆ
2 a2 Á 2 -b2 Á 2 =1
Ë a + b 2 ˜¯ Ë a + b 2 ˜¯
2
Ê bh ˆ Ê (a 2 + b 2 )h ˆ
fi ÁË ˜¯ + Á ˜ =1
ak Ë a3 ¯ fi a2a2 – b2b2 = (a2 + b2)2
2ˆ Hence, the locus of (a, b) is
h2 Ê b2 Ê a 2 + b2 ˆ
fi Á +Á ˜ ˜ =1 a2x2 – b2y2 = (a2 + b2)2
a 2 ËÁ k 2 Ë a 2 ¯ ¯˜ 18. Let the equation of the hyperbola be
Ê b2 Ê a 2 + b2 ˆ 2 ˆ a 2 x2 y 2
- = 1 and the point of intersection be P(h, k).
fi Á 2 +Á ˜ ˜= a 2 b2
ËÁ k Ë a 2 ¯ ¯˜ h 2 The equation of any tangent to the hyperbola is

Ê a 2 b2 ˆ Ê a 2 + b2 ˆ
2 y = mx + a 2 m 2 - b 2
fi Á 2 - 2˜ = Á ˜
Ëh k ¯ Ë a2 ¯ which is passing through P(h, k).
Hence, the locus of M(h, k) is k = mh + a 2 m 2 - b 2
Ê a 2 b2 ˆ Ê a 2 + b2 ˆ
2 fi (k – mh)2 = (a2m2 – b2)
Á 2 - 2˜ = Á ˜ fi (h2 – a2)m2 – 2(kh)m + (k2 + b2) = 0
Ëx y ¯ Ë a2 ¯ It has two roots, say m1 and m2
17. The equation of any normal to the hyperbola 2hk
Thus, m1 + m2 = 2
x2 y 2 (h - a 2 )
- = 1 is
a 2 b2 k 2 + b2
ax cos j + by cot j = (a2 + b2) …(i) and m1m2 =
h2 - a 2
Y Clearly, b = q1 – q2
N P
fi tan b = tan (q1 – q2)
tan q1 - tan q 2
Q =
1 + tanq1tanq 2
m - m2
= 1

O M
X 1 + m1m2

(m1 + m2 ) 2 - 4 m1m2
=
1 + m1m2

4h 2 k 2 Ê k 2 + b2 ˆ
Since the normal (i) meets the x-axis at M and y-axis at 2 2 2
- 4Á 2 ˜
N respectively. Then, (h - a ) Ë h - a2 ¯
=
Ê a 2 + b2 ˆ Ê Ê a 2 + b2 ˆ ˆ Ê k 2 + b2 ˆ
M= Á , 0˜ and N = Á 0, Á tan j ˜ 1+ Á 2 ˜
˜ Ë h - a2 ¯
Ë a cos j ¯ Ë Ë b ¯ ¯

CG_06a.indd 47 2/6/2017 4:08:13 PM


6.48 Coordinate Geometry Booster

Ê h2 + k 2 + b2 - a 2 ˆ
2
Ê b 2 ˆ
2 Let P be Á k , k - a2 ˜
fi tan b Á ˜ Ë a ¯
Ë h2 - a 2 ¯
Ê b 2 ˆ
4h 2 k 2 - 4(k 2 + b 2 )(h 2 - a 2 ) and Q be Á k , k + a2 ˜
= Ë a ¯
(h 2 - a 2 )2
The equation of the tangent at P to the hyperbola is
fi (h2 + k2 + b2 – a2) tan2 b xk y
- k 2 - a2 = 1 …(i)
= 4(h2k2 – (k2 + b2)(h2 – a2)) a 2 ab
fi (h + k2 + b2 – a2)2 tan2 b = 4(a2k2 + h2b2 + a2b2)
2
and the equation of the tangent at Q to the conjugate
fi (h2 + k2 + b2 – a2)2 = 4 cot b(a2k2 – h2b2 + a2b2) hyperbola is
Hence, the locus of P(h, k) is xk y
- k 2 + a 2 = -1 …(ii)
(x2 + y2 + b2 – a2)2 = 4 cot2 b(a2y2 – b2x2 + a2b2) a 2 ab
Squaring and adding Eqs (i) and (ii), we get
x2 y 2
19. Let the hyperbola be - =1 …(i) 1
a 2 b2 k2 = 2
x y2
x2 y 2 4
+ 2 2
and its conjugate be - = -1 …(ii) a a b
a 2 b2
1
The equation of any tangent, say AB at (p, q) is fi k=
2
px qy x y2
- = -1 +
a 2 b2
…(iii) a 4 a 2b 2
Solving, we get
p2 q2
where - +1 = 0 1 Ê ay ˆ
2
a 2 b2 x= Á ˜
i.e. b2p2 – a2q2 + a2b2 = 0 2k Ë b ¯
4
Eliminating y between Eqs (i) and (iii), we get 1 Ê ay ˆ
fi x2 = Á ˜
2
x 2 1 Ê xp ˆ Ê b 2 ˆ
2
4k2 Ë b ¯
- Á 1 + ˜ Á ˜ =1 4
a 2 b2 Ë a 2 ¯ Ë q ¯ 1 Ê ay ˆ Ê x 2 y2 ˆ
fi x2 = ÁË ˜¯ ¥ Á 4 + 2 2 ˜
Ê 1 p 2b 2 ˆ 2 Ê 2pb 2 ˆ Ê b2 ˆ 4 b Ëa a b ¯
fi Á 2 - ˜ x - Á 2 2˜ x - Á 2 + 1˜ = 0
Ëa a4q2 ¯ Ëa q ¯ Ëp ¯ 4x 2 y 4 Ê x 2 y 2 ˆ
fi = 4 ¥Á 2 + 2˜
Ê a 2b 2 ˆ 2 Ê 2pb 2 ˆ Ê b2 ˆ a2 b Ëa b ¯
fi Á 4 2˜ x - Á 2 2˜ x - Á 2 + 1˜ = 0
Ëa q ¯ Ëa q ¯ Ëp ¯ 21. Let the mid-point be M(h, k).
The equation of the chord is
Let its roots are x1 and x2. Then
T = S1
2p b 2 a 2b 2 fi hx + ky = h2 + k2
x1 + x2 = ◊ ∏ = 2p fi ky = –hx + (h2 + k2)
q2 a2 a4q2
Ê hˆ Ê h2 + k 2 ˆ
x1 + x2 fi y = -Á ˜ x + Á …(i)
fi =p Ë k¯ ˜
2 Ë k ¯
y + y2 which is a tangent to the hyperbola
Similarly 1 =q 9x2 – 16y2 = 144
2
Hence, the point of contact is the mid-point of the x2 y 2
fi - =1 …(ii)
chord AB. 16 9
20. Let the equation of the hyperbola be So, c2 = a2m2 – b2
x2 y 2 2 2
- =1 Ê h2 + k 2 ˆ Ê hˆ
a 2 b2 fi Á k ˜ = 16 ÁË - ˜ -9
2 2 Ë ¯ k¯
x y
and its conjugate be 2
- 2 = -1 . fi (h2 + k2)2 = 16h2 – 9k2
a b
Hence, the locus of M(h, k) is
The equation of any line parallel to the conjugate axis be
x = k. (x2 + y2)2 = 16x2 – 9y2

CG_06a.indd 48 2/6/2017 4:08:14 PM


Hyperbola 6.49

22. Given hyperbola is Also, OC = OR


2 2
2x2 + 5xy + 2y2 + 4x + 5y = 0 …(i) Ê acosj ˆ Ê bcosj ˆ
fi h2 + k 2 = Á h - ˜ + Ák +
The equation of the asymptote of the above hyperbola Ë 1 + sinj ¯ Ë 1 + sinj ¯˜
is
Ê cos j ˆ
2x2 + 5xy + 2y2 + 4x + 5y + k = 0 …(ii) fi 2(ah - bk ) = (a 2 + b 2 ) Á …(v)
If (ii) is an asymptote of (i), then Ë 1 + sin j ˜¯
abc + 2fgh – af2 – bg2 – ch2 = 0 Multiplying Eqs (iv) and (v), we get
5 4 5 25 25 4(a2h2 – b2k2) = (a2 + b2)2
fi 2◊ 2◊ k + 2◊ ◊ ◊ - 2◊ - 2◊ 4 - k ◊ = 0 Hence, the locus of (h, k) is
2 2 2 4 4
4(a2x2 – b2y2) = (a2 + b2)2
fi 16 ◊ k + 100 – 50 – 32 – 25k = 0
25. The area of the DPQR
fi 9k = 18
fi k=2 c2
x1 1
Putting k = 2 in Eq. (ii), we get x1
2x2 + 5xy + 2y2 + 4x + 5y + 2 = 0
1 c2
fi (2x + y + 2)(x + 2y + 1) = 0 = x2 1
fi (2x + y + 2) = 0, (x + 2y + 1) = 0 2 x2
Hence, the equation of the hyperbola is c2
(2x + y + 2)(x + 2y + 1) = c x3 1
x3
23. Let the equation of the hyperbola is
(x + 2y + 3)(3x + 4y + 5) = c x12 1 x1
2
which is passing through (1, –1). So c 1
= ¥ x22 1 x2
(1 – 2 + 3)(3 – 4 + 5) = c 2 x1 x2 x3
fi c = 2.4 = 8 x32 1 x3
Hence, the equation of the hyperbola is
c 2 ( x1 - x2 )( x2 - x3 )( x3 - x1 )
(x + 2y + 3)(3x + 4y + 5) = 8 = ¥
fi 3x2 + 10xy + 8y2 + 14x + 22y + 7 = 0 2 x1 x2 x3
24. Let the point P be (a sec j, b tan j).
The equation of the tangent to the hyperbola at P is LEVEL IV
x y
secj - tanj = 1 …(i) 1. The equation of any tangent to the parabola
a b a
and the equation of the asymptotes to the hyperbola x2 = 4ay is x = my +
m
x2 y 2 fi mx – m2y – a = 0 …(i)
- = 1 is Let the mid-point be M(h, k).
a 2 b2
The equation of the chord of the hyperbola
x y xy = c2 is
= …(ii)
a b xk + yh – 2c2 = 0 …(ii)
x y Since the lines (i) and (ii) are the same line, so
and =- …(iii)
a b m m2 a
Solving Eqs (i) and (ii), we get =- = 2
k h 2c
Ê a cosj b cosj ˆ ak ah
Q=Á , fi m = 2 and m 2 = - 2
Ë 1 - sinj 1 - sinj ˜¯ 2c 2c
2
Ê a cosj -b cosj ˆ Ê ak ˆ ah
and R = Á , fi ÁË 2 ˜¯ = - 2
Ë 1 + sinj 1 + sinj ˜¯ 2c 2c
Let O be the centre of the circle passing through C, Q a2k 2 2ah
and R having its co-ordinate as (h, k). fi 4
=- 2
c c
Thus, OC = OQ = OR
Ê 2c 2 ˆ
Now, OC = OQ fi k2 = -Á ˜h
2 2 Ë a ¯
2 2Ê acosj ˆ Ê bcosj ˆ Hence, the locus of M is
fi h + k = Áh - ˜ + Ák -
Ë 1 - sinj ¯ Ë 1 - sinj ˜¯ Ê 2c 2 ˆ
Ê cos j ˆ y2 = - Á ˜x
fi 2(ah + bk ) = (a 2 + b 2 ) Á …(iv) Ë a ¯
Ë 1 - sin j ˜¯ which is a parabola

CG_06a.indd 49 2/6/2017 4:08:15 PM


6.50 Coordinate Geometry Booster

2. Given hyperbola is 25x2 – 16y2 = 400 Solving, we get


2 2 2
x y Ê h2 k 2 ˆ
- =1 Á a 2 + b2 ˜
16 25 Ë ¯
(a 2 m2 - b 2 ) =
The equation of the chord of the hyperbola bisected at ( k /b 2 ) 2
(6, 2) is
2Ê 2 2 ˆ Ê h2 k 2 ˆ 2
T = S1 Ê kˆ 2 Ê hb ˆ 2
fi ÁË 2 ˜¯ Á a Á 2 ˜ - b ˜ = Á 2 + 2 ˜
6x 2y 36 4 b ÁË Ë ka ¯ ˜¯ Ë a b ¯
fi - = -
16 25 16 25 Ê h2 k 2 ˆ Ê h2 k 2 ˆ
2

fi 150x – 32y = 900 – 64 fi Á 2 - 2˜ =Á 2 + 2˜


Ëa b ¯ Ëa b ¯
fi 150x – 32y = 836
Hence, the locus of M is
fi 75x – 16y = 418
2
3. Let the mid-point be M(h, k). Ê x2 y 2 ˆ Ê x2 y 2 ˆ
Á 2 - 2˜ =Á 2 + 2˜
The equation of the chord bisected at M to the given Ëa b ¯ Ëa b ¯
circle is 5. Let the parameters of the vertices A, B and C of the
hx + ky = h2 + k2 points on the hyperbola xy = c2 be t1, t2 and t3 respec-
fi ky = –hx + (h2 + k2) tively.
Now the equation of the side BC is
Ê hˆ Ê h2 + k 2 ˆ x + yt2t3 – c(t2 + t3) = 0
fi y = -Á ˜ x + Á ˜
Ë k¯ Ë k ¯ Any line through A perpendicular to BC is
c
which is a tangent to the given hyperbola. So y - = t2t3 ( x - ct1 )
t1
c2 = a2m2 – b2
c
2 2 fi y - x t2t3 = - ct1t2t3 …(i)
Ê h2 + k 2 ˆ Ê hˆ t1
fi Á k ˜ = 16 ÁË k ˜¯ - 9
Ë ¯ Similarly, any line through B perpendicular to AC is
c
fi (h2 + k2)2 = 16h2 – 9k2 y - xt1t3 = - ct1t2t3 …(ii)
t2
Hence, the locus of (h, k) is
Solving Eqs (i) and (ii), we get, the orthocentre as
(x2 + y2)2 = 16x2 – 9y2 Ê c ˆ
4. The equation of any tangent to the given hyperbola is ÁË - t t t , - ct1t2t3 ˜¯ .
12 3
y = mx + a 2 m 2 - b 2 Clearly, it satisfies the hyperbola
2 2 2 xy = c2
fi mx - y + a m - b = 0 …(i)
Hence, the result.
Let the mid-point be M(h, k). 6. The equation of the normal to the rectangular hyper-
The equation of the chord of the ellipse is bola is
T = S1 xt3 – yt = c(t4 – 1) …(i)
hx ky h 2 k 2 Let the pole be M(h, k).
fi + = + …(ii)
a 2 b2 a 2 b2 The equation of the polar at M is
Since the lines (i) and (ii) are the same line. So xk + yh = 2c2 …(ii)
Since the lines (i) and (ii) are the same line, so
m -1 a 2m2 - b2
= = t 3 -t c(t 4 - 1)
( h / a 2 ) ( k /b 2 ) Ê h 2 k 2 ˆ = =
k h 2c 2
Á a 2 + b2 ˜
Ë ¯
t 3 -t (t 4 - 1)
2 fi = =
( h /a ) k h 2c
fi m=-
( k /b 2 ) Solving, we get

Ê h2 k 2 ˆ k t 2 (t 4 - 1) 2
t2 = - , 2 =
Á a 2 + b2 ˜ h h 4c 2
Ë ¯
and a 2 m2 - b2 = - k (t 4 - 1) 2
( k /b 2 ) fi - 3=
h 4c 2

CG_06a.indd 50 2/6/2017 4:08:15 PM


Hyperbola 6.51

2 Y
Ê k2 ˆ
Á h 2 - 1˜
k Ë ¯ A
fi - =
h3 4c 2 E
C
fi –4c2kh = (k2 – h2)2 X¢
O X
fi (h2 – k2)2 + 4c2hk = 0 D
B
Hence, the locus of (h, k) is
(x2 – y2)2 + 4c2xy = 0
7. Let the equation of the circle be Y¢
x2 + y2 + 2gx + 2fy + k = 0 …(i) Slope of AD = t2t3
Now the equation of the altitude AD is
and the equation of the rectangular hyperbola is c
xy = c2 …(ii) y - = t2t3 ( x - ct1 ) …(i)
t1
Putting x = ct and yc/t, then Similarly, equation of the altitude BE is
c2 c
Ê cˆ y - = t1t3 ( x - ct2 ) …(ii)
c 2t 2 + + 2g (ct ) + 2f Á ˜ + k = 0 t2
t2 Ët¯
Solving, Eqs (i) and (ii), we get the co-ordinates of the
fi c2t4 + c2 + 2g(ct3) + 2f(ct) + kt2 = 0 orthocentre as
fi c2t4 + 2gct3 + kt2 + 2fct + c2 = 0 Ê c ˆ
which is a bi-quadratic equation of t. So, it has four ÁË - t t t , - ct1t2t3 ˜¯
12 3
roots t1, t2, t3 and t4. Then which lies on xy = c2.
2g 9. Let the equation of the circle be
S t1 = - x2 + y2 + 2gx + 2fy + k = 0 …(i)
c and the equation of the rectangular hyperbola is
k xy = c2 …(ii)
S t1t2 =
c2 Putting x = ct and yc/t, then
2f c2 Ê cˆ
S t1t2 t3 = - c 2t 2 + + 2g (ct ) + 2f Á ˜ + k = 0
c t 2 Ët¯
and S t1t2 t3t4 = 1 fi c2t4 + c2 + 2g(ct3) + 2f(ct) + kt2 = 0
fi c2t4 + 2gct3 + kt2 + 2fct + c2 = 0
1 Â t1t2t3 = - 2f which is a bi-quadratic equation of t. So, it has four
Also, Ât =
1 Â t1t2t3t4 c roots, say t1, t2, t3 and t4.
2g
Now, (–g, –f) Then  t1 = -
c
Ê c Ê 2g ˆ c Ê 2f ˆ ˆ k
= Á Á- ˜ , Á- ˜˜
Ë 2 Ë c ¯ 2 Ë c ¯¯
 t1t2 = c 2
2f
Êc c Ê 1 1 1 1 ˆˆ  t1t2t3 = - c
= Á (t1 + t2 + t3 + t4 ) , Á + + + ˜ ˜
Ë2 2 Ë t1 t2 t3 t4 ¯ ¯ and  t1t2t3t4 = 1
ÊcÊ 1 ˆ cÊ1 1 1 ˆˆ 1  t1t2t3 = - 2f
= Á Á t1 + t2 + t3 + ˜ , Á + + + t1t2t3 ˜ ˜ Also, Ât =
Ë 2Ë t1t2t3 ¯ 2 Ë t1 t2 t3 ¯¯ 1  t1t2t3t4 c
8. Let t1, t2 and t3 are the vertices of DABC described on The centre of the mean position of the four points is
the rectangular hyperbola xy = c2. Êc c Ê 1 1 1 1 ˆˆ
So the co-ordinates of A, B and C are Á 4 (t1 + t2 + t3 + t4 ), 4 ÁË t + t + t + t ˜¯ ˜
Ë 1 2 3 4 ¯
Ê cˆ Ê cˆ Ê cˆ Êc c 1ˆ
ÁË ct1 , t ˜¯ , ÁË ct2 , t ˜¯ and ÁË ct3 , t ˜¯ respectively = Á  t1 ,  ˜
1 2 3 Ë4 4 t1 ¯
1 Ê g fˆ
Now, slope of BC = - = Á- ,- ˜
t2t3 Ë 2 2¯

CG_06a.indd 51 2/6/2017 4:08:16 PM


6.52 Coordinate Geometry Booster

Thus, the centres of the circle and the rectangular 1 1 1 1


hyperbola are (–g, –f) and (0, 0). 3. Clearly, 2
+ 2 = +
e1 e2 Ê b ˆ Ê a2 ˆ
2

Á1 + 2 ˜ Á1 + 2 ˜
and the mid-points of the centres of the circle and the
Ê g fˆ Ë a ¯ Ë b ¯
hyperbola is Á - , - ˜
Ë 2 2¯ a2 b2
= +
Hence, the result. a 2 + b2 a 2 + b2
10. Let the equation of the circle be
a 2 + b2
x2 + y2 – 6ax – 6by + k = 0 …(i) = =1
and the equation of the rectangular hyperbola is a 2 + b2
x2 – y2 = 9a2 …(ii)
Ê 1 1 ˆ
Eliminating y between Eqs (i) and (ii), we get Hence, the value of Á 2 + 2 + 3˜ is 4.
(x2 + x2 – 9a2 – 6ax + k)2 = 36b2(x2 – 9a)2 Ë e1 e2 ¯
fi 4x4 – 24ax3 + (…)x2 + (…) x +.. = 0 4. Clearly, the point (4, 3) lies on the hyperbola. So, the
which is a bi-quadratic equation of x. number of tangents is 1.
Let the abscissae of four points P, Q, R, S be x1, x2, x3 5. The director circle of the given circle x2 + y2 = a2 is
and x4, respectively. x2 + y2 = 2a2
Thus, x1 + x2 + x3 + x4 = 6a So the radius of the circle is a 2 , whereas the length
Similarly, y1 + y2 + y3 + y4 = 6b of the transverse axis is a 3 .
Let (h, k) be the centroid of DPQR. So So, the director circle and the hyperbola will never in-
x +x +x y + y2 + y3 tersect.
h = 1 2 3 ,k = 1
3 3 So, the number of points is zero.
6a - x4 6b - y4 6. Given hyperbola is
fi h= ,k = 9x2 – 16y2 – 18x – 32y – 151 = 0
3 3
fi x4 = 6a – 3h, y4 = 6b – 3k fi 9(x2 – 2x) – (y2 + 2y) = 151
Since (x4, y4) lies on the curve, so fi 9(x – 1)2 – 16(y + 1)2 = 144
x42 - y42 = 9a 2 ( x - 1) 2 ( y + 1) 2
fi - =1
16 9
fi (6a – 3h)2 – (6b – 3k)2 = 9a2
fi (2a – h)2 – (2b – k)2 = a2 2b 2 2.9 9
Latus rectum = m = = = .
Hence, the locus of (h, k) is a 4 2
(2a – x)2 – (2b – y)2 = a2 Hence, the value of 2m – 3 = 9 – 3 = 6.
(x –2a)2 – (y – 2b)2 = a2 7. No real tangent can be drawn.
So, the value of m is zero.
Integer Type Questions Hence, the value of (m + 4) is 4.
8. Given hyperbola is
b2 12 16x2 – 9y2 = 144
1. e = 1 + = 1+ = 4=2
a2 4 x2 y 2
fi - =1
2. We have, 9 16
The equation of any tangent to the given hyperbola is
b2 81 225 15 5
e = 1+ = 1+ = = =
a2 144 144 12 4 y = mx + a 2 m 2 - b 2
144 fi y = mx + 9 m 2 - 16
Also, a 2 =
25 It is given that,
Thus, the foci are (9 m 2 - 16) = 2 5
Ê 12 5 ˆ
(±ae, 0) = Á ± ¥ , 0 = (± 3, 0) fi (9m2 – 16) = 20
Ë 5 4 ˜¯ fi 9m2 = 36
Now, for the ellipse, fi m2 = 4
ae = 3 fi m = ±2
fi a2c2 = 9 So, a + b = 2 – 2 = 0
Thus, b2 = a2(1 – e2) = a2 – a2e2 Hence, the value of (a + b + 3) is 3.
= 16 – 9 = 7 9. Given curves are xy = c, (c > 0)
Hence, the value of (b2 + 1) is 8. and x2 + y2 = 1

CG_06a.indd 52 2/6/2017 4:08:16 PM


Hyperbola 6.53

Y Ê c ˆ Ê c ˆ
Thus, N = Á ct - 3 , 0˜ and N ¢ = Á 0, - ct 3 ˜ .
Ë t ¯ Ë t ¯
Now, D = ar(DPNT)

X¢ c
O X ct 1
t
1 c
= ct - 3 0 1
2 t
2ct 0 1

Hence, the distance between the points of contact = c2 Ê 1ˆ


= Át + ˜
diameter of a circle = 2 2t Ë t 3 ¯
x2 y 2
10. The equation of the asymptotes of 2 - 2 = 1 are c2 Ê 1ˆ
a b = ÁË1 + 4 ˜¯
2 t
x y x y
+ = 0, - = 0 and D = ar(DPN¢T¢)
a b a b
i.e. bx + ay = 0, bx – ay = 0 c
ct 1
Let any point on the hyperbola be t
P(a sec j, b tan j). 1 c
= 0 - ct 3 1
It is given that, p1p2 = 6 2 t
2c
0 1
ab secj + ab tanj ab secj - ab tanj t
fi =6
a 2 + b2 a 2 + b2 ct Ê c 3 2c ˆ
= Á - ct - ˜¯
2Ët t
a 2b 2 (sec 2j - tan 2j )
fi =6
(a 2 + b 2 ) ct Ê 3 c ˆ
= Á ct + ˜¯
2Ë t
a 2b 2
fi =6 c2 4
(a 2 + b 2 ) = (t + 1)
2
a 2 ◊ 2a 2
fi =6 1 1 2 Ê t4 ˆ 2 Ê 1 ˆ
(a 2 + 2 a 2 ) Now, + = 2Á 4 ˜+ 2Á 4 ˜
D D¢ c Ë t + 1¯ c Ë t + 1¯
fi 2a2 = 18
2 Ê t 4 + 1ˆ
fi a2 = 9 = Á ˜
c 2 Ë t 4 + 1¯
fi 2a = 6
2
\ Length of the transverse axis = 6 =
11. The equation of the tangent to the hyperbola xy = c2 at c2
Ê cˆ Ê c2 c2 ˆ
ÁË ct , ˜¯ is Hence, the value of Á + + 4˜ is 6.
t Ë D D¢ ¯
x + yt2 = 2ct …(i)
x2 y 2
12. The equation of the given hyperbola is - = 1.
Ê 2c ˆ 9 4
Thus, T = (2ct , 0) and T ¢ = Á 0, ˜ .
Ë t ¯ Thus, the equations of the asymptotes are
2
The equation of the normal to the hyperbola xy = c at Ê x yˆ Ê x yˆ
Ê cˆ ÁË + ˜¯ ÁË - ˜¯ = 0
3 2 3 2
ÁË ct , ˜¯ is
t Ê x yˆ Ê x yˆ
fi ÁË + ˜¯ = 0 and ÁË - ˜¯ = 0
xt3 – yt – ct4 + c = 0 …(ii) 3 2 3 2

CG_06a.indd 53 2/6/2017 4:08:17 PM


6.54 Coordinate Geometry Booster

fi 2x + 3y = 0 and 2x – 3y = 0 Y
The equation of any tangent to the hyperbola
P(1/2. 1)
x2 y 2
- = 1 is
9 4
X¢ X
x y O
sec j - tan j = 1
3 2
x = –1
Let the points of intersection of 2x + 3y = 0, 2x – 3y = x=1
0 and
x y Y¢
sec j - tan j = 1 are Thus, x = 1 is nearer to P(1/2, 1).
3 2
O, P and Q respectively. Therefore, the directrix of the ellipse is x = 1.
Let Q(x, y) be any point on the ellipse.
Therefore, O = (0, 0),
Now, the length of the perpendicular from Q to the di-
Ê 3 2 ˆ rectrix x – 1 = 0 is
P=Á ,-
Ë secj + tanj secj + tanj ¯˜ |x – 1|.
Ê 3 2 ˆ By the definition of the ellipse, we have
and Q=Á ,
Ë secj - tanj secj - tanj ¯˜
QP = e|x – 1|
2
Hence, the area of DOPQ Ê 1ˆ 2
fi ÁË x - ˜¯ + ( y - 1) = e | x - 1|
0 0 2
3 2 2
- Ê 1ˆ 2 2 2
1 sec j + tan j sec j + tan j fi ÁË x - ˜¯ + ( y - 1) = e x - 1
= 2
2 3 2
2
sec j - tan j sec j - tan j Ê 1ˆ 2 1 2
fi ÁË x - ˜¯ + ( y - 1) = ( x - 2x + 1)
0 0 2 4
1 1 x2 1 1
= (6 + 6) = 6 sq. u. fi x2 - x + + y 2 - 2y + 1 = - x+
2 4 4 2 4
x2 x
Previous Years’ JEE-Advanced Examinations fi x 2 - x + y 2 - 2y + 1 = -
4 2
1. Given curve is
fi 4x2 – 4x + 4y2 – 8y + 4 = x2 – 2x
x2 y2 fi 3x2 + 4y2 – 2x – 8y + 4 = 0
- = 1, r > 1
1- r 1+ r fi (3x2 – 2x) + (4y2 – 8y) + 4 = 0
fi (1 + r) x2 – (1 – r)y2 = (1 – r)(1 + r)
Ê 2 ˆ
fi (1 + r) x2 – (1 – r)y2 + (r2 – 1) = 0 fi 3 Á x 2 - x˜ + 4( y 2 - 2y ) + 4 = 0
Ë 3 ¯
Now, h2 – ab = 0 – (1 + r)(r – 1)
= (1 – r2) < 0, as r > 1 ÏÔÊ 2
1ˆ 1 ¸Ô
So, it represents an ellipse. fi
ÔÓË 3¯ 9 Ô˛
{
3 ÌÁ x - ˜ - ˝ + 4 ( y - 1) 2 - 1 + 4 = 0 }
2. Hints
3. Given curve is 2
Ê 1ˆ 1
2x2 + 3y2 – 8x – 18y + 35 = k fi 3 Á x - ˜ + 4( y - 1) 2 = -4 + + 4
Ë 3¯ 3
fi 2(x2 – 4x) + 3(y2 – 6y) = k = 35
fi 2{(x –2)2 – 4} + 3 {(y – 3)2 – 9} = k – 35 Ê 1ˆ
2
1
fi 2(x – 2)2 + 3(y – 3)2 = k – 35 + 8 + 27 fi 3 Á x - ˜ + 4( y - 1) 2 =
Ë 3¯ 3
fi 2(x – 2)2 + 3(y – 3)2 = k
2
It represents a point if k = 0. Ê 1ˆ
ÁË x - ˜¯ ( y - 1) 2
4. Clearly common tangents of the given curves are 3
fi + =1
x = 1 and x = –1, respectively. 1/9 1/12

CG_06a.indd 54 2/6/2017 4:08:17 PM


Hyperbola 6.55

5. Let M(h, k) be the point. fi x4 + c4 = a2x2


The equation of any line through M(h, k) having slope fi x4 – a2x2 + c4 = 0
4 is Let its roots be x1, x2, x3 and x4.
y – k = 4(x – h) Thus, x1 + x2 + x3 + x4 = 0
Suppose the line meets the curve xy = 1 at P(x1, y1) and and x1x2x3x4 = c4
Q(x2, y2). Similarly, y1 + y2 + y3 + y4 = 0
Y and y1y2y3y4 = c4
7. Let the point P be (h, k)
The equation of any tangent to the parabola is
a
y = mx + which is passing through P(h, k).
m
P(x1, y1)
a
X¢ \ k = mh +
X m
O fi m2h – km + a = 0
M(h, k) Since it has two roots say m1 and m2. Thus,
k a
Q(x2, y2) m1 + m2 = and m1m2 =
m h
m2 - m1
Y¢ Now, tan(45°) =
1 + m1m2
Now,
2
y – k = 4(x – h) Ê m2 - m1 ˆ
1
fi ÁË 1 + m m ˜¯ = 1
1 2
fi - k = 4( x - h)
x fi (m2 – m1)2 = (1 + m1m2)2
fi 4x2 – (4h – k) x – 1 = 0 fi (m2 + m1)2 – 4m1m2 = (1 + m1m2)2
Let its roots be x1, x2. 2 2
fi Ê kˆ Ê aˆ Ê aˆ
4h - k ÁË ˜¯ - 4 ÁË ˜¯ = ÁË1 + ˜¯
\ x1 + x2 = …(i) h h h
4
k 2 - 4ah (a + h) 2
1 fi =
and x1 x2 = - …(ii) h2 h2
4
fi (a + h) = k – 4ah
2 2
2x1 + x2
Also, h = fi h2 + 6ah + a2 = k2
3
fi (h + 3a)2 = k2 – 8a2
2x1 + x2 = 3h …(iii) Hence, the locus of P(h, k) is
From Eqs (i) and (iii), we get (x + 3a)2 = y2 – 8a2
4h - k 8h + k 8. Let P(h, k) be the point whose chord of contact w.r.t.
x1 = 3h - = the hyperbola x2 – y2 = 9 is
4 4
8h + k (2h + k ) x=9 …(i)
fi x2 = 3h - =- Also, the equation of the chord contact of the tangents
2 2
from P(h, k) is
Putting the values of x1 and x2 in Eq. (ii), we get hx – ky = 9 …(ii)
(8h + k ) (2h + k ) 1 Since the Eqs (i) and (ii) are identical, so
¥- =-
4 2 4 h = 1 and k = 0
fi (8h + k)(2h + k) = 2 Thus, the point P is (1, 0).
The equations of the pair of tangents is
fi 16h2 + 10hk + k2 – 2 = 0
SS1 = T2
Hence, the locus of M(h, k) is fi (x2 – y2 – 9) (1 – 0 – 9) = (x – 9)2
16x2 + 10xy + y2 – 2 = 0. fi –8(x2 – y2 – 9) = x2 – 18x + 81
6. Given x2 + y2 = a2 and xy = c2 fi 9x2 – 8y2 – 18x + 9 = 0
We have, 9. The equation of the normal at P(a sec q, b tan q) is
2
Ê c2 ˆ ax cos q + by cot q = a2 + b2.
x + Á ˜ = a2
2
fi ax + by cosec q = (a2 + b2) sec q
Ë x¯
and the equation of the normal at Q(a sec j, b tan j) is

CG_06a.indd 55 2/6/2017 4:08:18 PM


6.56 Coordinate Geometry Booster

ax cos j + by cot j = a2 + b2
x2 y 2
fi ax + by cosec j = (a2 + b2) sec j 12. Given hyperbola is - = 1.
9 4
Solving, we get
The equation of any point on the above hyperbola is
b(cosec q – cosec j) y = (a2 + b2)(sec q – sec j)
P(3 sec q, 2 tan q).
Ê a 2 + b 2 ˆ Ê secq - secj ˆ and the equation of the chord of contact of the circle
fi y=Á ˜Á ˜ x2 + y2 = 9 relative to the point P is
Ë b ¯ Ë cosecq - cosecj ¯
3x sec q + 2y sin q = 9 …(i)
Ê a 2 + b 2 ˆ Ê cosecj - cosecq ˆ Let M(h, k) be the mid-point of (i).
fi y=Á ˜Á ˜ The equation of the chord of the circle bisected at M is
Ë b ¯ Ë cosecq - cosecj ¯
T = S1
Ê a 2 + b2 ˆ fi hx + ky = h2 + k2 …(ii)
fi y = -Á ˜
Ë b ¯ Y
A
Ê a 2 + b2 ˆ P
Thus, k = - Á ˜
Ë b ¯ M(h, k)
2 2 X¢ X
x y O
10. Given curve is 2
- 2 =1
cos a sin a B
We have
b2 = a2 (e2 – 1)
fi a2e2 = a2 + b2 Y¢
fi a2e2 = cos2 a + sin2 a = 1 Clearly, Eqs (i) and (ii) are identical. So
fi (ae)2 = 1 3secq 2 tanq 9
fi (ae) = ±1 = = 2
h k h + k2
Abscissae of foci are ±1 irrespective of the value of a.
3h 9k
11. Given hyperbola is x2 – 2y2 = 4 fi secq = 2 2
, tanq =
h +k 2(h + k 2 )
2
x2 y 2
fi - =1 …(i) As we know that,
4 2
sec2 q – tan2 q = 1
Given line is
2 2
Ê 3h ˆ Ê 9k ˆ
2x + 6 y = 2 fi ÁË 2 -Á =1
2˜ ˜
h +k ¯ 2 2
Ë 2(h + k ) ¯
fi 6 y = -2x + 2
fi 36h2 – 81k2 = (h2 +k2)2
2 2 Hence, the locus of M(h, k) is
fi y=- x+
6 6 36x2 – 81y2 = (x2 + y2)2
x2 y 2
2 2 13. Let e be the eccentricity of the given ellipse + =1
fi y=- x+ 25 16
3 3
b2 16 3
As we know that if the line y = mx + c be a tangent Thus, e = 1 - 2
= 1- =
a 25 5
x2 y 2 5
to the hyperbola 2 - 2 = 1 , the co-ordinates of the Therefore, the eccentricity of the hyperbola is .
a b 3
3
Also, ae = 5◊ = 3
Ê a2m b2 ˆ 5
point of contact is Á ± ,± ˜.
Ë c c¯ Thus, the hyperbola passes through the focus, i.e. (3, 0)
of the given ellipse.
Ê Ê 2ˆ ˆ So, the semi-transverse axis is 3, i.e. a = 3
Á 4Á - ˜ ˜
Ë 3¯ 2 ˜ So, the semi conjugate axis is 4, i.e. b = 4
= Á± ,±
Á 2 2˜ Hence, the equation of the hyperbola is
Á ˜ x2 y 2
Ë 3 3¯ - =1
a 2 b2
= (± ( -4), ± 6)
x2 y 2
= (4, - 6) satisfies the given curve. fi - =1
9 16

CG_06a.indd 56 2/6/2017 4:08:19 PM


Hyperbola 6.57

14. Given, the transverse axis = 2 sin q


fi x - 2 = 6, y + 2 = 1
fi 2a = 2 sin q
fi a = sin q fi x = 2 + 6, y = 1 - 2
Given ellipse is
3x2 + 4y2 = 12 So, B = ( 2 + 6,1 - 2) .
1
x2 y 2 Now, ar(DABC) = ¥ ( 6 - 2) ¥ 1
fi + =1 2
4 3
Ê 3 ˆ
The eccentricity of the ellipse, =Á - 1˜
Ë 2 ¯
b2 3 1
e = 1- 2
= 1- =
a 4 2 16. Given hyperbola is
and the foci of the ellipse = (±ae, 0) 2x2 – 2y2 = 1

Ê 1 ˆ x2 y 2
= Á ± 2 ◊ , 0˜ = (± 1,0) fi - =1
Ë 2 ¯ 1/2 1
Let e1 be the eccentricity of the hyperbola. b2 2
Thus, e = 1 + = 1+ = 2
Now, b 2 = a 2 (e12 - 1) a 2
2
= a 2e12 - a 2 1
The eccentricity of the ellipse is .
= 1 – sin2 q = cos2 q 2
Hence, the equation of the hyperbola is 2
Ê 1ˆ a
2 2 Also, b 2 = a 2 (1 - e 2 ) = a 2 Á1 - ˜ =
x y Ë 2¯ 2
- =1
a 2 b2 Thus, the equation of the ellipse is
x2 y2
fi - =1 x2 y 2
sin 2q cos 2q + =1
a 2 b2
fi x2 cosec2 q – y2 sec2 q = 1
15. Given curve is x2 y2
fi + =1
x 2 - 2y 2 - 2 2 x - 4 2 y - 6 = 0 a 2 a 2 /2

fi ( x 2 - 2 2 x) - 2( y 2 + 2 2 y ) = 6 fi x2 + 2y2 = a2
Let P(h, k) be the point of intersection of the ellipse and
fi {( x - } {
2) 2 - 2 - 2 ( y + 2) 2 - 2 = 6} the hyperbola.
Thus, h2 + 2k2 = a2 and 2h2 – 2k2 = 1 …(i)
fi ( x - 2) 2 - 2( y + 2) 2 = 6 + 2 - 4 The Equations of tangents at P(h, k) to the ellipse and
fi ( x - 2) 2 - 2(y + 2) 2 = 4 hyperbola are
hx + 2ky = a2 and 2hx – 2ky = 1
( x - 2) 2 ( y + 2) 2 h h
fi - =1 Now, m( ET ) = - and m( H T ) =
4 2 2k k
Vertex: (a, 0) Since both the curves cut at right angles, so
fi x - 2 = 2, y + 2 = 0 m(ET) ¥ m(HT) = –1
h h
fi x = 2 + 2, y = - 2 fi - ¥ = -1
2k k
Thus, A ( 2 + 2, - 2) fi h2 = 2k2 …(ii)
Focus: x - 2 = ae, y + 2 = 0 From Eqs (i) and (ii), we get,
2h2 – h2 = 1
3
fi x = 2 + 2◊ ,y= - 2 fi h2 = 1
2 and h2 + 2k2 = a2
fi x = 2 + 6, y = - 2 fi h2 + h2 = a2
Therefore, the focus is C: ( 2 + 6, - 2) fi a2 = 1 + 1 = 2
Thus, the equation of the ellipse is
Ê b2 ˆ fi x2 + 2y2 = 2
End-point of L.R. = Á ae, ˜
Ë a¯ and its foci are (±ae, 0) = (±1, 0).

CG_06a.indd 57 2/6/2017 4:08:20 PM


6.58 Coordinate Geometry Booster

17. (P) As hx + ky –1 = 0 touches the circle x2 + y2 = 4, so, Now, CM = 4


0 + 0 -1
=2 4m - 0 + 9m 2 - 4
fi =4
h2 + k 2 m2 + 1
1
fi (h 2 + k 2 ) = fi (4m + 9m 2 - 4) 2 = 16(m 2 + 1)
4
1 fi 16m 2 + 9m 2 - 4 + 8m 9m 2 - 4
Thus, (h, k) lies on ( x 2 + y 2 ) = = 16m2 + 16
4
(Q) z lies on the hyperbola, since fi (9m 2 - 20) 2 = ( -8m 9m 2 - 4) 2
|SP – S¢P| = 2a. fi 81m4 + 400 – 360m = 64m(9m2 – 4)
2 2 fi 495m4 + 104m2 – 400 = 0
Ê x ˆ Ê 1 - t 2 ˆ Ê 2t ˆ 2
(R) Á ˜ + y 2 = Á ˜ +Á ˜ fi (99m2 + 100)(5m2 – 4) = 0
Ë 3¯ Ë1+ t2 ¯ Ë1+ t2 ¯
fi (5m2 – 4) = 0
(1 - t 2 ) 2 + 4t 2 2
= fi m= , since m > 0
(1 + t 2 ) 2 5
Thus, the equation of the tangent is
(1 + t 2 ) 2 2 4
= y= x+
(1 + t 2 ) 2 5 5
=1 fi 2x - 5 y + 4 = 0

which represents an ellipse. 19. Y


(S) For x > 1, the given conic is a hyperbola.
For x = 1, the conic is a parabola. M A
(T) Let z = x + iy
Given Re(z + 1)2 = |z|2 + 1 X¢ X
O C
fi (x+ 1)2 = x2 + y2 + 1
fi x2 + 2x + 1 = x2 + y2 + 1
fi y2 = 2x B
which represents a parabola.

21. Given circle is
Let the co-ordinates of A be
x2 + y2 – 8x = 0
(3 sec q, 2 tan q)
fi (x – 4)2 + y2 = 16
As A lies on the circle, so
Y 9 sec2 q = 4 tan2 q – 24 sec q = 0
fi 9 sec2 q + 4(sec2 q – 1) – 24 sec q = 0
A
M fi 13 sec2 q – 24 sec q – 4 = 0
fi (13 sec q + 2) (sec q – 2) = 0
X¢ X fi sec q = 2, since |sec q| ≥ 1
O C
Thus, the points A and B are
(6, 2 3) and (6, - 2 3)
B
The equation of the circle with AB as diameter is

(x – 6)2 + y2 = 12
fi x2 + y2 – 12x + 24 = 0.
The centre is (4, 0) and the radius = 4.
20. Given line is 2x + y = 1 …(i)
x2 y 2 Êa ˆ
Given hyperbola is - =1 which is passing through Á , 0˜
9 4 Ëe ¯
The equation of any tangent to the given hyperbola is a
fi 2◊ = 1
y = mx + a 2 m 2 - b 2 e
e
fi a= .
fi y = mx + 9 m 2 - 4 2

CG_06a.indd 58 2/6/2017 4:08:20 PM


Hyperbola 6.59

Since the equation (i) is a tangent to the given hy- The hyperbola passing through the focus of the
perbola, so ellipse, so
c2 = a2m2 – b2 3
fi 1 = a2, 4 – b2 -0 =1
a2
fi 4a2 – b2 = 1
fi a2 = 3
fi 4a2 – a2(e2 – 1) = 1
fi 5a2 – a2e2 = 1 Now, b2 = a2 (e2 –1)
2
fi a2(5 – e2) = 1 Ê4 ˆ a
fi b 2 = a 2 Á - 1˜ = =1
e2 Ë3 ¯ 3
fi (5 - e 2 ) = 1
4 Hence, the equation of the hyperbola is
fi 5e2 – e4 = 4 x2 y 2
fi e4 – 5e2 + 4 = 0 - =1
3 1
fi (e2 – 1)(e2 – 4) = 0
fi (e2 – 4) = 0, as e π 1 fi x2 – 3y2 = 3
fi e=2 \ Focus of a hyperbola = (2, 0)
21. The equation of the normal at P is 23. Given line is 2x – y = 1. So,
a 2 x b2 y m=2
+ = a 2 + b2 The equation of any tangent to the given hyper-
x1 y1
bola is
a 2 x b2 y
fi + = a 2 + b2 y = mx ± a 2 m 2 - b 2
6 3
which is passing through (9, 0). So fi y = 2 x ± 9.4 - 4
2 2
a ◊9 b ◊ 0 fi y = 2 x ± 32
+ = a 2 + b2
6 3
fi y = 2x± 4 2
3a 2
fi = a 2 + b2
2 fi 2 x - y + 4 2 = 0, 2 x - y - 4 2 = 0
3a 2 x y x y
fi = a 2 + a 2 (e 2 - 1) fi - = 1 and - + =1
2 2 2 4 2 2 2 4 2
3
fi = 1 + (e 2 - 1) xx yy
2 Comparing it with 1 - 1 = 1 . we get the points
9 4
3 1 of contact as
fi (e 2 - 1) = - 1 =
2 2
Ê 9 1 ˆ Ê 9 1 ˆ
1 3 ÁË , ˜¯ and ÁË - ,- ˜.
fi e2 = 1 + = 2 2 2 2 2 2¯
2 2
24. Tangent at P is xx1 – yy1 = 1 intersects the x-axis at
3
fi e= . Ê1 ˆ
2 M Á , 0˜
Ë x1 ¯
3
Hence, the eccentricity is . y1 y -0
2 Slope of normal = - = 1
22. Given ellipse is x1 x1 - x2
x2 + 4y2 = 4 fi x2 = 2x1
2 2 Thus, N = (2x1, 0)
x y
+ =1 1
fi 4 1 3x1 +
x1 y1
The eccentricity, For centroid, l = ,m=
3 3
b2 1 3
e = 1- = 1- = dl 1
a2 4 2 fi =1- 2
dx1 3x1
2
Thus, the eccentricity of the hyperbola is . dm 1 dm 1 dy1 x1
3 and = , = =
dy1 3 dx1 3 dx1 3 x 2 - 1
Foci of the ellipse = (±ae, 0) = (± 3 , 0) . 1

CG_06a.indd 59 2/6/2017 4:08:21 PM


CG_06a.indd 60 2/6/2017 4:08:21 PM

You might also like